Sie sind auf Seite 1von 746

National Territory

G.R No. 187167               August 16, 2011

PROF. MERLIN M. MAGALLONA, AKBAYAN PARTY-LIST REP. RISA HONTIVEROS, PROF.


HARRY C. ROQUE, JR., AND UNIVERSITY OF THE PHILIPPINES COLLEGE OF LAW
STUDENTS, ALITHEA BARBARA ACAS, VOLTAIRE ALFERES, CZARINA MAY ALTEZ,
FRANCIS ALVIN ASILO, SHERYL BALOT, RUBY AMOR BARRACA, JOSE JAVIER BAUTISTA,
ROMINA BERNARDO, VALERIE PAGASA BUENAVENTURA, EDAN MARRI CAÑETE, VANN
ALLEN DELA CRUZ, RENE DELORINO, PAULYN MAY DUMAN, SHARON ESCOTO, RODRIGO
FAJARDO III, GIRLIE FERRER, RAOULLE OSEN FERRER, CARLA REGINA GREPO, ANNA
MARIE CECILIA GO, IRISH KAY KALAW, MARY ANN JOY LEE, MARIA LUISA MANALAYSAY,
MIGUEL RAFAEL MUSNGI, MICHAEL OCAMPO, JAKLYN HANNA PINEDA, WILLIAM
RAGAMAT, MARICAR RAMOS, ENRIK FORT REVILLAS, JAMES MARK TERRY RIDON,
JOHANN FRANTZ RIVERA IV, CHRISTIAN RIVERO, DIANNE MARIE ROA, NICHOLAS
SANTIZO, MELISSA CHRISTINA SANTOS, CRISTINE MAE TABING, VANESSA ANNE TORNO,
MARIA ESTER VANGUARDIA, and MARCELINO VELOSO III, Petitioners, 
vs.
HON. EDUARDO ERMITA, IN HIS CAPACITY AS EXECUTIVE SECRETARY, HON. ALBERTO
ROMULO, IN HIS CAPACITY AS SECRETARY OF THE DEPARTMENT OF FOREIGN AFFAIRS,
HON. ROLANDO ANDAYA, IN HIS CAPACITY AS SECRETARY OF THE DEPARTMENT OF
BUDGET AND MANAGEMENT, HON. DIONY VENTURA, IN HIS CAPACITY AS
ADMINISTRATOR OF THE NATIONAL MAPPING & RESOURCE INFORMATION AUTHORITY,
and HON. HILARIO DAVIDE, JR., IN HIS CAPACITY AS REPRESENTATIVE OF THE
PERMANENT MISSION OF THE REPUBLIC OF THE PHILIPPINES TO THE UNITED NATIONS,
Respondents.

DECISION

CARPIO, J.:

The Case

This original action for the writs of certiorari and prohibition assails the constitutionality of Republic
Act No. 95221(RA 9522) adjusting the country’s archipelagic baselines and classifying the baseline
regime of nearby territories.

The Antecedents

In 1961, Congress passed Republic Act No. 3046 (RA 3046)2 demarcating the maritime baselines of
the Philippines as an archipelagic State.3 This law followed the framing of the Convention on the
Territorial Sea and the Contiguous Zone in 1958 (UNCLOS I), 4 codifying, among others, the
sovereign right of States parties over their "territorial sea," the breadth of which, however, was left
undetermined. Attempts to fill this void during the second round of negotiations in Geneva in 1960
(UNCLOS II) proved futile. Thus, domestically, RA 3046 remained unchanged for nearly five
decades, save for legislation passed in 1968 (Republic Act No. 5446 [RA 5446]) correcting
typographical errors and reserving the drawing of baselines around Sabah in North Borneo.
In March 2009, Congress amended RA 3046 by enacting RA 9522, the statute now under scrutiny.
The change was prompted by the need to make RA 3046 compliant with the terms of the United
Nations Convention on the Law of the Sea (UNCLOS III), 5 which the Philippines ratified on 27
February 1984.6 Among others, UNCLOS III prescribes the water-land ratio, length, and contour of
baselines of archipelagic States like the Philippines7 and sets the deadline for the filing of application
for the extended continental shelf.8 Complying with these requirements, RA 9522 shortened one
baseline, optimized the location of some basepoints around the Philippine archipelago and classified
adjacent territories, namely, the Kalayaan Island Group (KIG) and the Scarborough Shoal, as
"regimes of islands" whose islands generate their own applicable maritime zones.

Petitioners, professors of law, law students and a legislator, in their respective capacities as
"citizens, taxpayers or x x x legislators," 9 as the case may be, assail the constitutionality of RA 9522
on two principal grounds, namely: (1) RA 9522 reduces Philippine maritime territory, and logically,
the reach of the Philippine state’s sovereign power, in violation of Article 1 of the 1987
Constitution,10 embodying the terms of the Treaty of Paris11 and ancillary treaties,12 and (2) RA 9522
opens the country’s waters landward of the baselines to maritime passage by all vessels and
aircrafts, undermining Philippine sovereignty and national security, contravening the country’s
nuclear-free policy, and damaging marine resources, in violation of relevant constitutional
provisions.13

In addition, petitioners contend that RA 9522’s treatment of the KIG as "regime of islands" not only
results in the loss of a large maritime area but also prejudices the livelihood of subsistence
fishermen.14 To buttress their argument of territorial diminution, petitioners facially attack RA 9522 for
what it excluded and included – its failure to reference either the Treaty of Paris or Sabah and its use
of UNCLOS III’s framework of regime of islands to determine the maritime zones of the KIG and the
Scarborough Shoal.

Commenting on the petition, respondent officials raised threshold issues questioning (1) the
petition’s compliance with the case or controversy requirement for judicial review grounded on
petitioners’ alleged lack of locus standi and (2) the propriety of the writs of certiorari and prohibition
to assail the constitutionality of RA 9522. On the merits, respondents defended RA 9522 as the
country’s compliance with the terms of UNCLOS III, preserving Philippine territory over the KIG or
Scarborough Shoal. Respondents add that RA 9522 does not undermine the country’s security,
environment and economic interests or relinquish the Philippines’ claim over Sabah.

Respondents also question the normative force, under international law, of petitioners’ assertion that
what Spain ceded to the United States under the Treaty of Paris were the islands and all the
waters found within the boundaries of the rectangular area drawn under the Treaty of Paris.

We left unacted petitioners’ prayer for an injunctive writ.

The Issues

The petition raises the following issues:

1. Preliminarily –

1. Whether petitioners possess locus standi to bring this suit; and

2. Whether the writs of certiorari and prohibition are the proper remedies to assail the
constitutionality of RA 9522.
2. On the merits, whether RA 9522 is unconstitutional.

The Ruling of the Court

On the threshold issues, we hold that (1) petitioners possess locus standi to bring this suit as
citizens and (2) the writs of certiorari and prohibition are proper remedies to test the constitutionality
of RA 9522. On the merits, we find no basis to declare RA 9522 unconstitutional.

On the Threshold Issues


Petitioners Possess Locus
Standi as Citizens

Petitioners themselves undermine their assertion of locus standi as legislators and taxpayers
because the petition alleges neither infringement of legislative prerogative15 nor misuse of public
funds,16 occasioned by the passage and implementation of RA 9522. Nonetheless, we recognize
petitioners’ locus standi as citizens with constitutionally sufficient interest in the resolution of the
merits of the case which undoubtedly raises issues of national significance necessitating urgent
resolution. Indeed, owing to the peculiar nature of RA 9522, it is understandably difficult to find other
litigants possessing "a more direct and specific interest" to bring the suit, thus satisfying one of the
requirements for granting citizenship standing.17

The Writs of Certiorari and Prohibition


Are Proper Remedies to Test
the Constitutionality of Statutes

In praying for the dismissal of the petition on preliminary grounds, respondents seek a strict
observance of the offices of the writs of certiorari and prohibition, noting that the writs cannot issue
absent any showing of grave abuse of discretion in the exercise of judicial, quasi-judicial or
ministerial powers on the part of respondents and resulting prejudice on the part of petitioners.18

Respondents’ submission holds true in ordinary civil proceedings. When this Court exercises its
constitutional power of judicial review, however, we have, by tradition, viewed the writs of certiorari
and prohibition as proper remedial vehicles to test the constitutionality of statutes,19 and indeed, of
acts of other branches of government.20 Issues of constitutional import are sometimes crafted out of
statutes which, while having no bearing on the personal interests of the petitioners, carry such
relevance in the life of this nation that the Court inevitably finds itself constrained to take cognizance
of the case and pass upon the issues raised, non-compliance with the letter of procedural rules
notwithstanding. The statute sought to be reviewed here is one such law.

RA 9522 is Not Unconstitutional


RA 9522 is a Statutory Tool
to Demarcate the Country’s
Maritime Zones and Continental
Shelf Under UNCLOS III, not to
Delineate Philippine Territory

Petitioners submit that RA 9522 "dismembers a large portion of the national territory" 21 because it
discards the pre-UNCLOS III demarcation of Philippine territory under the Treaty of Paris and related
treaties, successively encoded in the definition of national territory under the 1935, 1973 and 1987
Constitutions. Petitioners theorize that this constitutional definition trumps any treaty or statutory
provision denying the Philippines sovereign control over waters, beyond the territorial sea
recognized at the time of the Treaty of Paris, that Spain supposedly ceded to the United States.
Petitioners argue that from the Treaty of Paris’ technical description, Philippine sovereignty over
territorial waters extends hundreds of nautical miles around the Philippine archipelago, embracing
the rectangular area delineated in the Treaty of Paris.22

Petitioners’ theory fails to persuade us.

UNCLOS III has nothing to do with the acquisition (or loss) of territory. It is a multilateral treaty
regulating, among others, sea-use rights over maritime zones (i.e., the territorial waters [12 nautical
miles from the baselines], contiguous zone [24 nautical miles from the baselines], exclusive
economic zone [200 nautical miles from the baselines]), and continental shelves that UNCLOS III
delimits.23 UNCLOS III was the culmination of decades-long negotiations among United Nations
members to codify norms regulating the conduct of States in the world’s oceans and submarine
areas, recognizing coastal and archipelagic States’ graduated authority over a limited span of waters
and submarine lands along their coasts.

On the other hand, baselines laws such as RA 9522 are enacted by UNCLOS III States parties to
mark-out specific basepoints along their coasts from which baselines are drawn, either straight or
contoured, to serve as geographic starting points to measure the breadth of the maritime zones and
continental shelf. Article 48 of UNCLOS III on archipelagic States like ours could not be any clearer:

Article 48. Measurement of the breadth of the territorial sea, the contiguous zone, the exclusive
economic zone and the continental shelf. – The breadth of the territorial sea, the contiguous zone,
the exclusive economic zone and the continental shelf shall be measured from archipelagic
baselines drawn in accordance with article 47. (Emphasis supplied)

Thus, baselines laws are nothing but statutory mechanisms for UNCLOS III States parties to delimit
with precision the extent of their maritime zones and continental shelves. In turn, this gives notice to
the rest of the international community of the scope of the maritime space and submarine areas
within which States parties exercise treaty-based rights, namely, the exercise of sovereignty over
territorial waters (Article 2), the jurisdiction to enforce customs, fiscal, immigration, and sanitation
laws in the contiguous zone (Article 33), and the right to exploit the living and non-living resources in
the exclusive economic zone (Article 56) and continental shelf (Article 77).

Even under petitioners’ theory that the Philippine territory embraces the islands and all the
waters within the rectangular area delimited in the Treaty of Paris, the baselines of the Philippines
would still have to be drawn in accordance with RA 9522 because this is the only way to draw the
baselines in conformity with UNCLOS III. The baselines cannot be drawn from the boundaries or
other portions of the rectangular area delineated in the Treaty of Paris, but from the "outermost
islands and drying reefs of the archipelago."24

UNCLOS III and its ancillary baselines laws play no role in the acquisition, enlargement or, as
petitioners claim, diminution of territory. Under traditional international law typology, States acquire
(or conversely, lose) territory through occupation, accretion, cession and prescription, 25 not by
executing multilateral treaties on the regulations of sea-use rights or enacting statutes to comply with
the treaty’s terms to delimit maritime zones and continental shelves. Territorial claims to land
features are outside UNCLOS III, and are instead governed by the rules on general international
law.26

RA 9522’s Use of the Framework


of Regime of Islands to Determine the
Maritime Zones of the KIG and the
Scarborough Shoal, not Inconsistent
with the Philippines’ Claim of Sovereignty
Over these Areas

Petitioners next submit that RA 9522’s use of UNCLOS III’s regime of islands framework to draw the
baselines, and to measure the breadth of the applicable maritime zones of the KIG, "weakens our
territorial claim" over that area.27Petitioners add that the KIG’s (and Scarborough Shoal’s) exclusion
from the Philippine archipelagic baselines results in the loss of "about 15,000 square nautical miles
of territorial waters," prejudicing the livelihood of subsistence fishermen.28 A comparison of the
configuration of the baselines drawn under RA 3046 and RA 9522 and the extent of maritime space
encompassed by each law, coupled with a reading of the text of RA 9522 and its congressional
deliberations, vis-à-vis the Philippines’ obligations under UNCLOS III, belie this view.
1avvphi1

The configuration of the baselines drawn under RA 3046 and RA 9522 shows that RA 9522 merely
followed the basepoints mapped by RA 3046, save for at least nine basepoints that RA 9522
skipped to optimize the location of basepoints and adjust the length of one baseline (and thus
comply with UNCLOS III’s limitation on the maximum length of baselines). Under RA 3046, as under
RA 9522, the KIG and the Scarborough Shoal lie outside of the baselines drawn around the
Philippine archipelago. This undeniable cartographic fact takes the wind out of petitioners’ argument
branding RA 9522 as a statutory renunciation of the Philippines’ claim over the KIG, assuming that
baselines are relevant for this purpose.

Petitioners’ assertion of loss of "about 15,000 square nautical miles of territorial waters" under RA
9522 is similarly unfounded both in fact and law. On the contrary, RA 9522, by optimizing the
location of basepoints, increased the Philippines’ total maritime space (covering its internal waters,
territorial sea and exclusive economic zone) by 145,216 square nautical miles, as shown in the table
below:29

Extent of maritime
area using RA 3046, Extent of maritime
as amended, taking area using RA 9522,
into account the taking into account
 
Treaty of Paris’ UNCLOS III (in
delimitation (in square nautical
square nautical miles)
miles)
Internal or
archipelagic
waters 166,858 171,435
Territorial Sea 274,136 32,106
Exclusive
Economic Zone   382,669
TOTAL 440,994 586,210

Thus, as the map below shows, the reach of the exclusive economic zone drawn under RA 9522
even extends way beyond the waters covered by the rectangular demarcation under the Treaty of
Paris. Of course, where there are overlapping exclusive economic zones of opposite or adjacent
States, there will have to be a delineation of maritime boundaries in accordance with UNCLOS III.30
Further, petitioners’ argument that the KIG now lies outside Philippine territory because the
baselines that RA 9522 draws do not enclose the KIG is negated by RA 9522 itself. Section 2 of the
law commits to text the Philippines’ continued claim of sovereignty and jurisdiction over the KIG and
the Scarborough Shoal:

SEC. 2. The baselines in the following areas over which the Philippines likewise exercises
sovereignty and jurisdiction shall be determined as "Regime of Islands" under the Republic of the
Philippines consistent with Article 121 of the United Nations Convention on the Law of the Sea
(UNCLOS):

a) The Kalayaan Island Group as constituted under Presidential Decree No. 1596 and

b) Bajo de Masinloc, also known as Scarborough Shoal. (Emphasis supplied)


Had Congress in RA 9522 enclosed the KIG and the Scarborough Shoal as part of the Philippine
archipelago, adverse legal effects would have ensued. The Philippines would have committed a
breach of two provisions of UNCLOS III. First, Article 47 (3) of UNCLOS III requires that "[t]he
drawing of such baselines shall not depart to any appreciable extent from the general configuration
of the archipelago." Second, Article 47 (2) of UNCLOS III requires that "the length of the baselines
shall not exceed 100 nautical miles," save for three per cent (3%) of the total number of baselines
which can reach up to 125 nautical miles.31

Although the Philippines has consistently claimed sovereignty over the KIG32 and the Scarborough
Shoal for several decades, these outlying areas are located at an appreciable distance from the
nearest shoreline of the Philippine archipelago,33 such that any straight baseline loped around them
from the nearest basepoint will inevitably "depart to an appreciable extent from the general
configuration of the archipelago."

The principal sponsor of RA 9522 in the Senate, Senator Miriam Defensor-Santiago, took pains to
emphasize the foregoing during the Senate deliberations:

What we call the Kalayaan Island Group or what the rest of the world call[] the Spratlys and the
Scarborough Shoal are outside our archipelagic baseline because if we put them inside our
baselines we might be accused of violating the provision of international law which states: "The
drawing of such baseline shall not depart to any appreciable extent from the general configuration of
the archipelago." So sa loob ng ating baseline, dapat magkalapit ang mga islands. Dahil malayo ang
Scarborough Shoal, hindi natin masasabing malapit sila sa atin although we are still allowed by
international law to claim them as our own.

This is called contested islands outside our configuration. We see that our archipelago is defined by
the orange line which [we] call[] archipelagic baseline. Ngayon, tingnan ninyo ang maliit na circle
doon sa itaas, that is Scarborough Shoal, itong malaking circle sa ibaba, that is Kalayaan Group or
the Spratlys. Malayo na sila sa ating archipelago kaya kung ilihis pa natin ang dating archipelagic
baselines para lamang masama itong dalawang circles, hindi na sila magkalapit at baka hindi na
tatanggapin ng United Nations because of the rule that it should follow the natural configuration of
the archipelago.34 (Emphasis supplied)

Similarly, the length of one baseline that RA 3046 drew exceeded UNCLOS III’s limits.  The need to
1avvphi1

shorten this baseline, and in addition, to optimize the location of basepoints using current maps,
became imperative as discussed by respondents:

[T]he amendment of the baselines law was necessary to enable the Philippines to draw the outer
limits of its maritime zones including the extended continental shelf in the manner provided by Article
47 of [UNCLOS III]. As defined by R.A. 3046, as amended by R.A. 5446, the baselines suffer from
some technical deficiencies, to wit:

1. The length of the baseline across Moro Gulf (from Middle of 3 Rock Awash to Tongquil
Point) is 140.06 nautical miles x x x. This exceeds the maximum length allowed under Article
47(2) of the [UNCLOS III], which states that "The length of such baselines shall not exceed
100 nautical miles, except that up to 3 per cent of the total number of baselines enclosing
any archipelago may exceed that length, up to a maximum length of 125 nautical miles."

2. The selection of basepoints is not optimal. At least 9 basepoints can be skipped or deleted
from the baselines system. This will enclose an additional 2,195 nautical miles of water.
3. Finally, the basepoints were drawn from maps existing in 1968, and not established by
geodetic survey methods. Accordingly, some of the points, particularly along the west coasts
of Luzon down to Palawan were later found to be located either inland or on water, not on
low-water line and drying reefs as prescribed by Article 47.35

Hence, far from surrendering the Philippines’ claim over the KIG and the Scarborough Shoal,
Congress’ decision to classify the KIG and the Scarborough Shoal as "‘Regime[s] of Islands’ under
the Republic of the Philippines consistent with Article 121" 36 of UNCLOS III manifests the Philippine
State’s responsible observance of its pacta sunt servanda obligation under UNCLOS III. Under
Article 121 of UNCLOS III, any "naturally formed area of land, surrounded by water, which is above
water at high tide," such as portions of the KIG, qualifies under the category of "regime of islands,"
whose islands generate their own applicable maritime zones.37

Statutory Claim Over Sabah under


RA 5446 Retained

Petitioners’ argument for the invalidity of RA 9522 for its failure to textualize the Philippines’ claim
over Sabah in North Borneo is also untenable. Section 2 of RA 5446, which RA 9522 did not repeal,
keeps open the door for drawing the baselines of Sabah:

Section 2. The definition of the baselines of the territorial sea of the Philippine Archipelago as
provided in this Act is without prejudice to the delineation of the baselines of the territorial sea
around the territory of Sabah, situated in North Borneo, over which the Republic of the
Philippines has acquired dominion and sovereignty. (Emphasis supplied)

UNCLOS III and RA 9522 not


Incompatible with the Constitution’s
Delineation of Internal Waters

As their final argument against the validity of RA 9522, petitioners contend that the law
unconstitutionally "converts" internal waters into archipelagic waters, hence subjecting these waters
to the right of innocent and sea lanes passage under UNCLOS III, including overflight. Petitioners
extrapolate that these passage rights indubitably expose Philippine internal waters to nuclear and
maritime pollution hazards, in violation of the Constitution.38

Whether referred to as Philippine "internal waters" under Article I of the Constitution 39 or as
"archipelagic waters" under UNCLOS III (Article 49 [1]), the Philippines exercises sovereignty over
the body of water lying landward of the baselines, including the air space over it and the submarine
areas underneath. UNCLOS III affirms this:

Article 49. Legal status of archipelagic waters, of the air space over archipelagic waters and of their
bed and subsoil. –

1. The sovereignty of an archipelagic State extends to the waters enclosed by the


archipelagic baselines drawn in accordance with article 47, described as archipelagic
waters, regardless of their depth or distance from the coast.

2. This sovereignty extends to the air space over the archipelagic waters, as well as to
their bed and subsoil, and the resources contained therein.

xxxx
4. The regime of archipelagic sea lanes passage established in this Part shall not in other
respects affect the status of the archipelagic waters, including the sea lanes, or the
exercise by the archipelagic State of its sovereignty over such waters and their air
space, bed and subsoil, and the resources contained therein. (Emphasis supplied)

The fact of sovereignty, however, does not preclude the operation of municipal and international law
norms subjecting the territorial sea or archipelagic waters to necessary, if not marginal, burdens in
the interest of maintaining unimpeded, expeditious international navigation, consistent with the
international law principle of freedom of navigation. Thus, domestically, the political branches of the
Philippine government, in the competent discharge of their constitutional powers, may pass
legislation designating routes within the archipelagic waters to regulate innocent and sea lanes
passage.40 Indeed, bills drawing nautical highways for sea lanes passage are now pending in
Congress.41

In the absence of municipal legislation, international law norms, now codified in UNCLOS III, operate
to grant innocent passage rights over the territorial sea or archipelagic waters, subject to the treaty’s
limitations and conditions for their exercise.42 Significantly, the right of innocent passage is a
customary international law,43 thus automatically incorporated in the corpus of Philippine law. 44 No
modern State can validly invoke its sovereignty to absolutely forbid innocent passage that is
exercised in accordance with customary international law without risking retaliatory measures from
the international community.

The fact that for archipelagic States, their archipelagic waters are subject to both the right of
innocent passage and sea lanes passage45 does not place them in lesser footing vis-à-
vis continental coastal States which are subject, in their territorial sea, to the right of innocent
passage and the right of transit passage through international straits. The imposition of these
passage rights through archipelagic waters under UNCLOS III was a concession by archipelagic
States, in exchange for their right to claim all the waters landward of their baselines, regardless of
their depth or distance from the coast, as archipelagic waters subject to their territorial sovereignty.
More importantly, the recognition of archipelagic States’ archipelago and the waters enclosed by
their baselines as one cohesive entity prevents the treatment of their islands as separate islands
under UNCLOS III.46 Separate islands generate their own maritime zones, placing the waters
between islands separated by more than 24 nautical miles beyond the States’ territorial sovereignty,
subjecting these waters to the rights of other States under UNCLOS III.47

Petitioners’ invocation of non-executory constitutional provisions in Article II (Declaration of


Principles and State Policies)48 must also fail. Our present state of jurisprudence considers the
provisions in Article II as mere legislative guides, which, absent enabling legislation, "do not embody
judicially enforceable constitutional rights x x x."49 Article II provisions serve as guides in formulating
and interpreting implementing legislation, as well as in interpreting executory provisions of the
Constitution. Although Oposa v. Factoran50 treated the right to a healthful and balanced ecology
under Section 16 of Article II as an exception, the present petition lacks factual basis to substantiate
the claimed constitutional violation. The other provisions petitioners cite, relating to the protection of
marine wealth (Article XII, Section 2, paragraph 251 ) and subsistence fishermen (Article XIII, Section
752 ), are not violated by RA 9522.

In fact, the demarcation of the baselines enables the Philippines to delimit its exclusive economic
zone, reserving solely to the Philippines the exploitation of all living and non-living resources within
such zone. Such a maritime delineation binds the international community since the delineation is in
strict observance of UNCLOS III. If the maritime delineation is contrary to UNCLOS III, the
international community will of course reject it and will refuse to be bound by it.
UNCLOS III favors States with a long coastline like the Philippines. UNCLOS III creates a sui
generis maritime space – the exclusive economic zone – in waters previously part of the high seas.
UNCLOS III grants new rights to coastal States to exclusively exploit the resources found within this
zone up to 200 nautical miles.53 UNCLOS III, however, preserves the traditional freedom of
navigation of other States that attached to this zone beyond the territorial sea before UNCLOS III.

RA 9522 and the Philippines’ Maritime Zones

Petitioners hold the view that, based on the permissive text of UNCLOS III, Congress was not bound
to pass RA 9522.54 We have looked at the relevant provision of UNCLOS III 55 and we find petitioners’
reading plausible. Nevertheless, the prerogative of choosing this option belongs to Congress, not to
this Court. Moreover, the luxury of choosing this option comes at a very steep price. Absent an
UNCLOS III compliant baselines law, an archipelagic State like the Philippines will find itself devoid
of internationally acceptable baselines from where the breadth of its maritime zones and continental
shelf is measured. This is recipe for a two-fronted disaster: first, it sends an open invitation to the
seafaring powers to freely enter and exploit the resources in the waters and submarine areas around
our archipelago; and second, it weakens the country’s case in any international dispute over
Philippine maritime space. These are consequences Congress wisely avoided.

The enactment of UNCLOS III compliant baselines law for the Philippine archipelago and adjacent
areas, as embodied in RA 9522, allows an internationally-recognized delimitation of the breadth of
the Philippines’ maritime zones and continental shelf. RA 9522 is therefore a most vital step on the
part of the Philippines in safeguarding its maritime zones, consistent with the Constitution and our
national interest.

WHEREFORE, we DISMISS the petition.

SO ORDERED.

G.R. No. 183591             October 14, 2008

THE PROVINCE OF NORTH COTABATO, duly represented by GOVERNOR JESUS SACDALAN


and/or VICE-GOVERNOR EMMANUEL PIÑOL, for and in his own behalf, petitioners, 
vs.
THE GOVERNMENT OF THE REPUBLIC OF THE PHILIPPINES PEACE PANEL ON
ANCESTRAL DOMAIN (GRP), represented by SEC. RODOLFO GARCIA, ATTY. LEAH
ARMAMENTO, ATTY. SEDFREY CANDELARIA, MARK RYAN SULLIVAN and/or GEN.
HERMOGENES ESPERON, JR., the latter in his capacity as the present and duly-appointed
Presidential Adviser on the Peace Process (OPAPP) or the so-called Office of the Presidential
Adviser on the Peace Process, respondents.

x--------------------------------------------x

G.R. No. 183752             October 14, 2008

CITY GOVERNMENT OF ZAMBOANGA, as represented by HON. CELSO L. LOBREGAT, City


Mayor of Zamboanga, and in his personal capacity as resident of the City of Zamboanga,
Rep. MA. ISABELLE G. CLIMACO, District 1, and Rep. ERICO BASILIO A. FABIAN, District 2,
City of Zamboanga, petitioners, 
vs.
THE GOVERNMENT OF THE REPUBLIC OF THE PHILIPPINES PEACE NEGOTIATING PANEL
(GRP), as represented by RODOLFO C. GARCIA, LEAH ARMAMENTO, SEDFREY
CANDELARIA, MARK RYAN SULLIVAN and HERMOGENES ESPERON, in his capacity as the
Presidential Adviser on Peace Process,respondents.

x--------------------------------------------x

G.R. No. 183893             October 14, 2008

THE CITY OF ILIGAN, duly represented by CITY MAYOR LAWRENCE LLUCH CRUZ, petitioner, 
vs.
THE GOVERNMENT OF THE REPUBLIC OF THE PHILIPPINES PEACE PANEL ON
ANCESTRAL DOMAIN (GRP), represented by SEC. RODOLFO GARCIA, ATTY. LEAH
ARMAMENTO, ATTY. SEDFREY CANDELARIA, MARK RYAN SULLIVAN; GEN. HERMOGENES
ESPERON, JR., in his capacity as the present and duly appointed Presidential Adviser on the
Peace Process; and/or SEC. EDUARDO ERMITA, in his capacity as Executive
Secretary. respondents.

x--------------------------------------------x

G.R. No. 183951             October 14, 2008

THE PROVINCIAL GOVERNMENT OF ZAMBOANGA DEL NORTE, as represented by HON.


ROLANDO E. YEBES, in his capacity as Provincial Governor, HON. FRANCIS H. OLVIS, in his
capacity as Vice-Governor and Presiding Officer of the Sangguniang Panlalawigan, HON.
CECILIA JALOSJOS CARREON, Congresswoman, 1st Congressional District, HON. CESAR G.
JALOSJOS, Congressman, 3rd Congressional District, and Members of the Sangguniang
Panlalawigan of the Province of Zamboanga del Norte, namely, HON. SETH FREDERICK P.
JALOSJOS, HON. FERNANDO R. CABIGON, JR., HON. ULDARICO M. MEJORADA II, HON.
EDIONAR M. ZAMORAS, HON. EDGAR J. BAGUIO, HON. CEDRIC L. ADRIATICO, HON.
FELIXBERTO C. BOLANDO, HON. JOSEPH BRENDO C. AJERO, HON. NORBIDEIRI B.
EDDING, HON. ANECITO S. DARUNDAY, HON. ANGELICA J. CARREON and HON.
LUZVIMINDA E. TORRINO, petitioners, 
vs.
THE GOVERNMENT OF THE REPUBLIC OF THE PHILIPPINES PEACE NEGOTIATING PANEL
[GRP], as represented by HON. RODOLFO C. GARCIA and HON. HERMOGENES ESPERON, in
his capacity as the Presidential Adviser of Peace Process, respondents.

x--------------------------------------------x

G.R. No. 183962             October 14, 2008

ERNESTO M. MACEDA, JEJOMAR C. BINAY, and AQUILINO L. PIMENTEL III, petitioners, 


vs.
THE GOVERNMENT OF THE REPUBLIC OF THE PHILIPPINES PEACE NEGOTIATING PANEL,
represented by its Chairman RODOLFO C. GARCIA, and the MORO ISLAMIC LIBERATION
FRONT PEACE NEGOTIATING PANEL, represented by its Chairman MOHAGHER
IQBAL, respondents.

x--------------------------------------------x
FRANKLIN M. DRILON and ADEL ABBAS TAMANO, petitioners-in-intervention.

x--------------------------------------------x

SEN. MANUEL A. ROXAS, petitioners-in-intervention.

x--------------------------------------------x

MUNICIPALITY OF LINAMON duly represented by its Municipal Mayor NOEL N.


DEANO, petitioners-in-intervention,

x--------------------------------------------x

THE CITY OF ISABELA, BASILAN PROVINCE, represented by MAYOR CHERRYLYN P.


SANTOS-AKBAR,petitioners-in-intervention.

x--------------------------------------------x

THE PROVINCE OF SULTAN KUDARAT, rep. by HON. SUHARTO T. MANGUDADATU, in his


capacity as Provincial Governor and a resident of the Province of Sultan Kudarat, petitioner-in-
intervention.

x-------------------------------------------x

RUY ELIAS LOPEZ, for and in his own behalf and on behalf of Indigenous Peoples in
Mindanao Not Belonging to the MILF, petitioner-in-intervention.

x--------------------------------------------x

CARLO B. GOMEZ, GERARDO S. DILIG, NESARIO G. AWAT, JOSELITO C. ALISUAG and


RICHALEX G. JAGMIS, as citizens and residents of Palawan, petitioners-in-intervention.

x--------------------------------------------x

MARINO RIDAO and KISIN BUXANI, petitioners-in-intervention.

x--------------------------------------------x

MUSLIM LEGAL ASSISTANCE FOUNDATION, INC (MUSLAF), respondent-in-intervention.

x--------------------------------------------x

MUSLIM MULTI-SECTORAL MOVEMENT FOR PEACE & DEVELOPMENT


(MMMPD), respondent-in-intervention.

x--------------------------------------------x

DECISION

CARPIO MORALES, J.:
Subject of these consolidated cases is the extent of the powers of the President in pursuing the
peace process.While the facts surrounding this controversy center on the armed conflict in Mindanao
between the government and the Moro Islamic Liberation Front (MILF), the legal issue involved has
a bearing on all areas in the country where there has been a long-standing armed conflict. Yet again,
the Court is tasked to perform a delicate balancing act. It must uncompromisingly delineate the
bounds within which the President may lawfully exercise her discretion, but it must do so in strict
adherence to the Constitution, lest its ruling unduly restricts the freedom of action vested by that
same Constitution in the Chief Executive precisely to enable her to pursue the peace process
effectively.

I. FACTUAL ANTECEDENTS OF THE PETITIONS

On August 5, 2008, the Government of the Republic of the Philippines (GRP) and the MILF, through
the Chairpersons of their respective peace negotiating panels, were scheduled to sign a
Memorandum of Agreement on the Ancestral Domain (MOA-AD) Aspect of the GRP-MILF Tripoli
Agreement on Peace of 2001 in Kuala Lumpur, Malaysia.

The MILF is a rebel group which was established in March 1984 when, under the leadership of the
late Salamat Hashim, it splintered from the Moro National Liberation Front (MNLF) then headed by
Nur Misuari, on the ground, among others, of what Salamat perceived to be the manipulation of the
MNLF away from an Islamic basis towards Marxist-Maoist orientations.1

The signing of the MOA-AD between the GRP and the MILF was not to materialize, however, for
upon motion of petitioners, specifically those who filed their cases before the scheduled signing of
the MOA-AD, this Court issued a Temporary Restraining Order enjoining the GRP from signing the
same.

The MOA-AD was preceded by a long process of negotiation and the concluding of several prior
agreements between the two parties beginning in 1996, when the GRP-MILF peace negotiations
began. On July 18, 1997, the GRP and MILF Peace Panels signed the Agreement on General
Cessation of Hostilities. The following year, they signed the General Framework of Agreement of
Intent on August 27, 1998.

The Solicitor General, who represents respondents, summarizes the MOA-AD by stating that the
same contained, among others, the commitment of the parties to pursue peace negotiations, protect
and respect human rights, negotiate with sincerity in the resolution and pacific settlement of the
conflict, and refrain from the use of threat or force to attain undue advantage while the peace
negotiations on the substantive agenda are on-going.2

Early on, however, it was evident that there was not going to be any smooth sailing in the GRP-MILF
peace process. Towards the end of 1999 up to early 2000, the MILF attacked a number of
municipalities in Central Mindanao and, in March 2000, it took control of the town hall of Kauswagan,
Lanao del Norte.3 In response, then President Joseph Estrada declared and carried out an "all-out-
war" against the MILF.

When President Gloria Macapagal-Arroyo assumed office, the military offensive against the MILF
was suspended and the government sought a resumption of the peace talks. The MILF, according to
a leading MILF member, initially responded with deep reservation, but when President Arroyo asked
the Government of Malaysia through Prime Minister Mahathir Mohammad to help convince the MILF
to return to the negotiating table, the MILF convened its Central Committee to seriously discuss the
matter and, eventually, decided to meet with the GRP.4
The parties met in Kuala Lumpur on March 24, 2001, with the talks being facilitated by the Malaysian
government, the parties signing on the same date the Agreement on the General Framework for the
Resumption of Peace Talks Between the GRP and the MILF. The MILF thereafter suspended all its
military actions.5

Formal peace talks between the parties were held in Tripoli, Libya from June 20-22, 2001, the
outcome of which was the GRP-MILF Tripoli Agreement on Peace (Tripoli Agreement 2001)
containing the basic principles and agenda on the following aspects of the
negotiation: Security Aspect, Rehabilitation Aspect, and Ancestral Domain Aspect. With regard to
the Ancestral Domain Aspect, the parties in Tripoli Agreement 2001 simply agreed "that the same be
discussed further by the Parties in their next meeting."

A second round of peace talks was held in Cyberjaya, Malaysia on August 5-7, 2001 which ended
with the signing of the Implementing Guidelines on the Security Aspect of the Tripoli Agreement
2001 leading to a ceasefire status between the parties. This was followed by the Implementing
Guidelines on the Humanitarian Rehabilitation and Development Aspects of the Tripoli Agreement
2001, which was signed on May 7, 2002 at Putrajaya, Malaysia. Nonetheless, there were many
incidence of violence between government forces and the MILF from 2002 to 2003.

Meanwhile, then MILF Chairman Salamat Hashim passed away on July 13, 2003 and he was
replaced by Al Haj Murad, who was then the chief peace negotiator of the MILF. Murad's position as
chief peace negotiator was taken over by Mohagher Iqbal.6

In 2005, several exploratory talks were held between the parties in Kuala Lumpur, eventually leading
to the crafting of the draft MOA-AD in its final form, which, as mentioned, was set to be signed last
August 5, 2008.

II. STATEMENT OF THE PROCEEDINGS

Before the Court is what is perhaps the most contentious "consensus" ever embodied in an
instrument - the MOA-AD which is assailed principally by the present petitions bearing docket
numbers 183591, 183752, 183893, 183951 and 183962.

Commonly impleaded as respondents are the GRP Peace Panel on Ancestral Domain 7 and the
Presidential Adviser on the Peace Process (PAPP) Hermogenes Esperon, Jr.

On July 23, 2008, the Province of North Cotabato 8 and Vice-Governor Emmanuel Piñol filed a
petition, docketed as G.R. No. 183591, for Mandamus and Prohibition with Prayer for the Issuance
of Writ of Preliminary Injunction and Temporary Restraining Order.9 Invoking the right to information
on matters of public concern, petitioners seek to compel respondents to disclose and furnish them
the complete and official copies of the MOA-AD including its attachments, and to prohibit the slated
signing of the MOA-AD, pending the disclosure of the contents of the MOA-AD and the holding of a
public consultation thereon. Supplementarily, petitioners pray that the MOA-AD be declared
unconstitutional.10

This initial petition was followed by another one, docketed as G.R. No. 183752, also for Mandamus
and Prohibition11 filed by the City of Zamboanga, 12 Mayor Celso Lobregat, Rep. Ma. Isabelle Climaco
and Rep. Erico Basilio Fabian who likewise pray for similar injunctive reliefs. Petitioners herein
moreover pray that the City of Zamboanga be excluded from the Bangsamoro Homeland and/or
Bangsamoro Juridical Entity and, in the alternative, that the MOA-AD be declared null and void.
By Resolution of August 4, 2008, the Court issued a Temporary Restraining Order commanding and
directing public respondents and their agents to cease and desist from formally signing the MOA-
AD.13 The Court also required the Solicitor General to submit to the Court and petitioners the official
copy of the final draft of the MOA-AD,14 to which she complied.15

Meanwhile, the City of Iligan16 filed a petition for Injunction and/or Declaratory Relief, docketed
as G.R. No. 183893, praying that respondents be enjoined from signing the MOA-AD or, if the same
had already been signed, from implementing the same, and that the MOA-AD be declared
unconstitutional. Petitioners herein additionally implead Executive Secretary Eduardo Ermita as
respondent.

The Province of Zamboanga del Norte,17 Governor Rolando Yebes, Vice-Governor Francis Olvis,
Rep. Cecilia Jalosjos-Carreon, Rep. Cesar Jalosjos, and the members 18 of the Sangguniang
Panlalawigan of Zamboanga del Norte filed on August 15, 2008 a petition for Certiorari, Mandamus
and Prohibition,19 docketed as G.R. No. 183951. They pray, inter alia, that the MOA-AD be declared
null and void and without operative effect, and that respondents be enjoined from executing the
MOA-AD.

On August 19, 2008, Ernesto Maceda, Jejomar Binay, and Aquilino Pimentel III filed a petition for
Prohibition,20docketed as G.R. No. 183962, praying for a judgment prohibiting and permanently
enjoining respondents from formally signing and executing the MOA-AD and or any other agreement
derived therefrom or similar thereto, and nullifying the MOA-AD for being unconstitutional and illegal.
Petitioners herein additionally implead as respondent the MILF Peace Negotiating Panel
represented by its Chairman Mohagher Iqbal.

Various parties moved to intervene and were granted leave of court to file their petitions-/comments-
in-intervention. Petitioners-in-Intervention include Senator Manuel A. Roxas, former Senate
President Franklin Drilon and Atty. Adel Tamano, the City of Isabela 21 and Mayor Cherrylyn Santos-
Akbar, the Province of Sultan Kudarat 22 and Gov. Suharto Mangudadatu, the Municipality of Linamon
in Lanao del Norte,23 Ruy Elias Lopez of Davao City and of the Bagobo tribe, Sangguniang
Panlungsod member Marino Ridao and businessman Kisin Buxani, both of Cotabato City; and
lawyers Carlo Gomez, Gerardo Dilig, Nesario Awat, Joselito Alisuag, Richalex Jagmis, all of
Palawan City. The Muslim Legal Assistance Foundation, Inc. (Muslaf) and the Muslim Multi-Sectoral
Movement for Peace and Development (MMMPD) filed their respective Comments-in-Intervention.

By subsequent Resolutions, the Court ordered the consolidation of the petitions. Respondents filed
Comments on the petitions, while some of petitioners submitted their respective Replies.

Respondents, by Manifestation and Motion of August 19, 2008, stated that the Executive
Department shall thoroughly review the MOA-AD and pursue further negotiations to address the
issues hurled against it, and thus moved to dismiss the cases. In the succeeding exchange of
pleadings, respondents' motion was met with vigorous opposition from petitioners.

The cases were heard on oral argument on August 15, 22 and 29, 2008 that tackled the following
principal issues:

1. Whether the petitions have become moot and academic

(i) insofar as the mandamus aspect is concerned, in view of the disclosure of official


copies of the final draft of the Memorandum of Agreement (MOA); and
(ii) insofar as the prohibition aspect involving the Local Government Units is
concerned, if it is considered that consultation has become fait accompli with the
finalization of the draft;

2. Whether the constitutionality and the legality of the MOA is ripe for adjudication;

3. Whether respondent Government of the Republic of the Philippines Peace Panel


committed grave abuse of discretion amounting to lack or excess of jurisdiction when it
negotiated and initiated the MOA vis-à-vis ISSUES Nos. 4 and 5;

4. Whether there is a violation of the people's right to information on matters of public


concern (1987 Constitution, Article III, Sec. 7) under a state policy of full disclosure of all its
transactions involving public interest (1987 Constitution, Article II, Sec. 28) including public
consultation under Republic Act No. 7160 (LOCAL GOVERNMENT CODE OF 1991)[;]

If it is in the affirmative, whether prohibition under Rule 65 of the 1997 Rules of Civil


Procedure is an appropriate remedy;

5. Whether by signing the MOA, the Government of the Republic of the Philippines would be
BINDING itself

a) to create and recognize the Bangsamoro Juridical Entity (BJE) as a separate


state, or a juridical, territorial or political subdivision not recognized by law;

b) to revise or amend the Constitution and existing laws to conform to the MOA;

c) to concede to or recognize the claim of the Moro Islamic Liberation Front for
ancestral domain in violation of Republic Act No. 8371 (THE INDIGENOUS
PEOPLES RIGHTS ACT OF 1997), particularly Section 3(g) & Chapter VII
(DELINEATION, RECOGNITION OF ANCESTRAL DOMAINS)[;]

If in the affirmative, whether the Executive Branch has the authority to so bind the
Government of the Republic of the Philippines;

6. Whether the inclusion/exclusion of the Province of North Cotabato, Cities of Zamboanga,


Iligan and Isabela, and the Municipality of Linamon, Lanao del Norte in/from the areas
covered by the projected Bangsamoro Homeland is a justiciable question; and

7. Whether desistance from signing the MOA derogates any prior valid commitments of the
Government of the Republic of the Philippines.24

The Court, thereafter, ordered the parties to submit their respective Memoranda. Most of the parties
submitted their memoranda on time.

III. OVERVIEW OF THE MOA-AD

As a necessary backdrop to the consideration of the objections raised in the subject five petitions
and six petitions-in-intervention against the MOA-AD, as well as the two comments-in-intervention in
favor of the MOA-AD, the Court takes an overview of the MOA.

The MOA-AD identifies the Parties to it as the GRP and the MILF.


Under the heading "Terms of Reference" (TOR), the MOA-AD includes not only four earlier
agreements between the GRP and MILF, but also two agreements between the GRP and the MNLF:
the 1976 Tripoli Agreement, and the Final Peace Agreement on the Implementation of the 1976
Tripoli Agreement, signed on September 2, 1996 during the administration of President Fidel
Ramos.

The MOA-AD also identifies as TOR two local statutes - the organic act for the Autonomous Region
in Muslim Mindanao (ARMM)25 and the Indigenous Peoples Rights Act (IPRA),26 and several
international law instruments - the ILO Convention No. 169 Concerning Indigenous and Tribal
Peoples in Independent Countries in relation to the UN Declaration on the Rights of the Indigenous
Peoples, and the UN Charter, among others.

The MOA-AD includes as a final TOR the generic category of "compact rights entrenchment
emanating from the regime of dar-ul-mua'hada (or territory under compact) and dar-ul-sulh (or
territory under peace agreement) that partakes the nature of a treaty device."

During the height of the Muslim Empire, early Muslim jurists tended to see the world through a
simple dichotomy: there was the dar-ul-Islam (the Abode of Islam) and dar-ul-harb (the Abode
of War). The first referred to those lands where Islamic laws held sway, while the second denoted
those lands where Muslims were persecuted or where Muslim laws were outlawed or
ineffective.27 This way of viewing the world, however, became more complex through the centuries
as the Islamic world became part of the international community of nations.

As Muslim States entered into treaties with their neighbors, even with distant States and inter-
governmental organizations, the classical division of the world into dar-ul-Islam and dar-ul-
harb eventually lost its meaning. New terms were drawn up to describe novel ways of perceiving
non-Muslim territories. For instance, areas like dar-ul-mua'hada (land of compact) and dar-ul-
sulh (land of treaty) referred to countries which, though under a secular regime, maintained peaceful
and cooperative relations with Muslim States, having been bound to each other by treaty or
agreement. Dar-ul-aman (land of order), on the other hand, referred to countries which, though not
bound by treaty with Muslim States, maintained freedom of religion for Muslims.28

It thus appears that the "compact rights entrenchment" emanating from the regime of dar-ul-
mua'hada and dar-ul-sulh simply refers to all other agreements between the MILF and the Philippine
government - the Philippines being the land of compact and peace agreement - that partake of the
nature of a treaty device, "treaty" being broadly defined as "any solemn agreement in writing that
sets out understandings, obligations, and benefits for both parties which provides for a framework
that elaborates the principles declared in the [MOA-AD]."29

The MOA-AD states that the Parties "HAVE AGREED AND ACKNOWLEDGED AS FOLLOWS," and
starts with its main body.

The main body of the MOA-AD is divided into four strands, namely, Concepts and Principles,
Territory, Resources, and Governance.

A. CONCEPTS AND PRINCIPLES

This strand begins with the statement that it is "the birthright of all Moros and all Indigenous peoples
of Mindanao to identify themselves and be accepted as ‘Bangsamoros.'" It defines "Bangsamoro
people" as the natives or original inhabitants of Mindanao and its adjacent islands including
Palawan and the Sulu archipelago at the time of conquest or colonization, and their
descendants whether mixed or of full blood, including their spouses.30
Thus, the concept of "Bangsamoro," as defined in this strand of the MOA-AD, includes not only
"Moros" as traditionally understood even by Muslims,31 but all indigenous peoples of Mindanao and
its adjacent islands. The MOA-AD adds that the freedom of choice of indigenous peoples shall be
respected. What this freedom of choice consists in has not been specifically defined.

The MOA-AD proceeds to refer to the "Bangsamoro homeland," the ownership of which is vested
exclusively in the Bangsamoro people by virtue of their prior rights of occupation.32 Both parties to
the MOA-AD acknowledge that ancestral domain does not form part of the public domain.33

The Bangsamoro people are acknowledged as having the right to self-governance, which right is
said to be rooted on ancestral territoriality exercised originally under the suzerain authority of their
sultanates and the Pat a Pangampong ku Ranaw. The sultanates were described as states or
"karajaan/kadatuan" resembling a body politic endowed with all the elements of a nation-state in the
modern sense.34

The MOA-AD thus grounds the right to self-governance of the Bangsamoro people on the past
suzerain authority of the sultanates. As gathered, the territory defined as the Bangsamoro homeland
was ruled by several sultanates and, specifically in the case of the Maranao, by the Pat a
Pangampong ku Ranaw, a confederation of independent principalities (pangampong) each ruled by
datus and sultans, none of whom was supreme over the others.35

The MOA-AD goes on to describe the Bangsamoro people as "the ‘First Nation' with defined territory
and with a system of government having entered into treaties of amity and commerce with foreign
nations."

The term "First Nation" is of Canadian origin referring to the indigenous peoples of that territory,
particularly those known as Indians. In Canada, each of these indigenous peoples is equally entitled
to be called "First Nation," hence, all of them are usually described collectively by the plural "First
Nations."36 To that extent, the MOA-AD, by identifying the Bangsamoro people as "the First Nation" -
suggesting its exclusive entitlement to that designation - departs from the Canadian usage of the
term.

The MOA-AD then mentions for the first time the "Bangsamoro Juridical Entity" (BJE) to which it
grants the authority and jurisdiction over the Ancestral Domain and Ancestral Lands of the
Bangsamoro.37

B. TERRITORY

The territory of the Bangsamoro homeland is described as the land mass as well as the maritime,
terrestrial, fluvial and alluvial domains, including the aerial domain and the atmospheric space above
it, embracing the Mindanao-Sulu-Palawan geographic region.38

More specifically, the core of the BJE is defined as the present geographic area of the ARMM - thus
constituting the following areas: Lanao del Sur, Maguindanao, Sulu, Tawi-Tawi, Basilan, and Marawi
City. Significantly, this core also includes certain municipalities of Lanao del Norte that voted for
inclusion in the ARMM in the 2001 plebiscite.39

Outside of this core, the BJE is to cover other provinces, cities, municipalities and barangays, which
are grouped into two categories, Category A and Category B. Each of these areas is to be subjected
to a plebiscite to be held on different dates, years apart from each other. Thus, Category A areas are
to be subjected to a plebiscite not later than twelve (12) months following the signing of the MOA-
AD.40 Category B areas, also called "Special Intervention Areas," on the other hand, are to be
subjected to a plebiscite twenty-five (25) years from the signing of a separate agreement - the
Comprehensive Compact.41

The Parties to the MOA-AD stipulate that the BJE shall have jurisdiction over all natural resources
within its "internalwaters," defined as extending fifteen (15) kilometers from the coastline of the BJE
area;42 that the BJE shall also have "territorial waters," which shall stretch beyond the BJE internal
waters up to the baselines of the Republic of the Philippines (RP) south east and south west of
mainland Mindanao; and that within these territorial waters, the BJE and the "Central
Government" (used interchangeably with RP) shall exercise joint jurisdiction, authority and
management over all natural resources.43 Notably, the jurisdiction over the internal waters is not
similarly described as "joint."

The MOA-AD further provides for the sharing of minerals on the territorial waters between the


Central Government and the BJE, in favor of the latter, through production sharing and economic
cooperation agreement.44 The activities which the Parties are allowed to conduct on
the territorial waters are enumerated, among which are the exploration and utilization of natural
resources, regulation of shipping and fishing activities, and the enforcement of police and safety
measures.45 There is no similar provision on the sharing of minerals and allowed activities with
respect to the internal waters of the BJE.

C. RESOURCES

The MOA-AD states that the BJE is free to enter into any economic cooperation and trade relations
with foreign countries and shall have the option to establish trade missions in those countries. Such
relationships and understandings, however, are not to include aggression against the GRP. The BJE
may also enter into environmental cooperation agreements.46

The external defense of the BJE is to remain the duty and obligation of the Central Government. The
Central Government is also bound to "take necessary steps to ensure the BJE's participation in
international meetings and events" like those of the ASEAN and the specialized agencies of the UN.
The BJE is to be entitled to participate in Philippine official missions and delegations for the
negotiation of border agreements or protocols for environmental protection and equitable sharing of
incomes and revenues involving the bodies of water adjacent to or between the islands forming part
of the ancestral domain.47

With regard to the right of exploring for, producing, and obtaining all potential sources of energy,
petroleum, fossil fuel, mineral oil and natural gas, the jurisdiction and control thereon is to be vested
in the BJE "as the party having control within its territorial jurisdiction." This right carries
the proviso that, "in times of national emergency, when public interest so requires," the Central
Government may, for a fixed period and under reasonable terms as may be agreed upon by both
Parties, assume or direct the operation of such resources.48

The sharing between the Central Government and the BJE of total production pertaining to natural
resources is to be 75:25 in favor of the BJE.49

The MOA-AD provides that legitimate grievances of the Bangsamoro people arising from any unjust
dispossession of their territorial and proprietary rights, customary land tenures, or their
marginalization shall be acknowledged. Whenever restoration is no longer possible, reparation is to
be in such form as mutually determined by the Parties.50
The BJE may modify or cancel the forest concessions, timber licenses, contracts or agreements,
mining concessions, Mineral Production and Sharing Agreements (MPSA), Industrial Forest
Management Agreements (IFMA), and other land tenure instruments granted by the Philippine
Government, including those issued by the present ARMM.51

D. GOVERNANCE

The MOA-AD binds the Parties to invite a multinational third-party to observe and monitor the
implementation of the Comprehensive Compact. This compact is to embody the "details for the
effective enforcement" and "the mechanisms and modalities for the actual implementation" of the
MOA-AD. The MOA-AD explicitly provides that the participation of the third party shall not in any way
affect the status of the relationship between the Central Government and the BJE.52

The "associative" relationship 


between the Central Government 
and the BJE

The MOA-AD describes the relationship of the Central Government and the BJE as "associative,"
characterized by shared authority and responsibility. And it states that the structure of governance is
to be based on executive, legislative, judicial, and administrative institutions with defined powers and
functions in the Comprehensive Compact.

The MOA-AD provides that its provisions requiring "amendments to the existing legal framework"
shall take effect upon signing of the Comprehensive Compact and upon effecting the aforesaid
amendments, with due regard to the non-derogation of prior agreements and within the stipulated
timeframe to be contained in the Comprehensive Compact. As will be discussed later, much of
the present controversy hangs on the legality of this provision.

The BJE is granted the power to build, develop and maintain its own institutions inclusive of civil
service, electoral, financial and banking, education, legislation, legal, economic, police and internal
security force, judicial system and correctional institutions, the details of which shall be discussed in
the negotiation of the comprehensive compact.

As stated early on, the MOA-AD was set to be signed on August 5, 2008 by Rodolfo Garcia and
Mohagher Iqbal, Chairpersons of the Peace Negotiating Panels of the GRP and the MILF,
respectively. Notably, the penultimate paragraph of the MOA-AD identifies the signatories as "the
representatives of the Parties," meaning the GRP and MILF themselves, and not merely of the
negotiating panels.53 In addition, the signature page of the MOA-AD states that it is "WITNESSED
BY" Datuk Othman Bin Abd Razak, Special Adviser to the Prime Minister of Malaysia, "ENDORSED
BY" Ambassador Sayed Elmasry, Adviser to Organization of the Islamic Conference (OIC) Secretary
General and Special Envoy for Peace Process in Southern Philippines, and SIGNED "IN THE
PRESENCE OF" Dr. Albert G. Romulo, Secretary of Foreign Affairs of RP and Dato' Seri Utama Dr.
Rais Bin Yatim, Minister of Foreign Affairs, Malaysia, all of whom were scheduled to sign the
Agreement last August 5, 2008.

Annexed to the MOA-AD are two documents containing the respective lists cum maps of the
provinces, municipalities, and barangays under Categories A and B earlier mentioned in the
discussion on the strand on TERRITORY.

IV. PROCEDURAL ISSUES


A. RIPENESS

The power of judicial review is limited to actual cases or controversies. 54 Courts decline to issue
advisory opinions or to resolve hypothetical or feigned problems, or mere academic questions. 55 The
limitation of the power of judicial review to actual cases and controversies defines the role assigned
to the judiciary in a tripartite allocation of power, to assure that the courts will not intrude into areas
committed to the other branches of government.56

An actual case or controversy involves a conflict of legal rights, an assertion of opposite legal claims,
susceptible of judicial resolution as distinguished from a hypothetical or abstract difference or
dispute. There must be a contrariety of legal rights that can be interpreted and enforced on the basis
of existing law and jurisprudence.57 The Court can decide the constitutionality of an act or treaty only
when a proper case between opposing parties is submitted for judicial determination.58

Related to the requirement of an actual case or controversy is the requirement of ripeness. A


question is ripe for adjudication when the act being challenged has had a direct adverse effect on
the individual challenging it.59 For a case to be considered ripe for adjudication, it is a prerequisite
that something had then been accomplished or performed by either branch before a court may come
into the picture,60 and the petitioner must allege the existence of an immediate or threatened injury to
itself as a result of the challenged action. 61 He must show that he has sustained or is immediately in
danger of sustaining some direct injury as a result of the act complained of.62

The Solicitor General argues that there is no justiciable controversy that is ripe for judicial review in
the present petitions, reasoning that

The unsigned MOA-AD is simply a list of consensus points subject to further negotiations
and legislative enactments as well as constitutional processes aimed at attaining a final
peaceful agreement. Simply put, the MOA-AD remains to be a proposal that does not
automatically create legally demandable rights and obligations until the list of operative acts
required have been duly complied with. x x x

xxxx

In the cases at bar, it is respectfully submitted that this Honorable Court has no authority to
pass upon issues based on hypothetical or feigned constitutional problems or interests
with no concrete bases. Considering the preliminary character of the MOA-AD, there are no
concrete acts that could possibly violate petitioners' and intervenors' rights since the acts
complained of are mere contemplated steps toward the formulation of a final peace
agreement. Plainly, petitioners and intervenors' perceived injury, if at all, is merely imaginary
and illusory apart from being unfounded and based on mere conjectures. (Underscoring
supplied)

The Solicitor General cites63 the following provisions of the MOA-AD:

TERRITORY

xxxx

2. Toward this end, the Parties enter into the following stipulations:

xxxx
d. Without derogating from the requirements of prior agreements, the Government stipulates
to conduct and deliver, using all possible legal measures, within twelve (12) months following
the signing of the MOA-AD, a plebiscite covering the areas as enumerated in the list and
depicted in the map as Category A attached herein (the "Annex"). The Annex constitutes an
integral part of this framework agreement. Toward this end, the Parties shall endeavor to
complete the negotiations and resolve all outstanding issues on the Comprehensive
Compact within fifteen (15) months from the signing of the MOA-AD.

xxxx

GOVERNANCE

xxxx

7. The Parties agree that mechanisms and modalities for the actual implementation of this
MOA-AD shall be spelt out in the Comprehensive Compact to mutually take such steps to
enable it to occur effectively.

Any provisions of the MOA-AD requiring amendments to the existing legal framework shall
come into force upon the signing of a Comprehensive Compact and upon effecting the
necessary changes to the legal framework with due regard to non-derogation of prior
agreements and within the stipulated timeframe to be contained in the Comprehensive
Compact.64 (Underscoring supplied)

The Solicitor General's arguments fail to persuade.

Concrete acts under the MOA-AD are not necessary to render the present controversy ripe.
In Pimentel, Jr. v. Aguirre,65 this Court held:

x x x [B]y the mere enactment of the questioned law or the approval of the challenged action,
the dispute is said to have ripened into a judicial controversy even without any other overt
act. Indeed, even a singular violation of the Constitution and/or the law is enough to awaken
judicial duty.

xxxx

By the same token, when an act of the President, who in our constitutional scheme is a
coequal of Congress, is seriously alleged to have infringed the Constitution and the laws x x
x settling the dispute becomes the duty and the responsibility of the courts.66

In Santa Fe Independent School District v. Doe,67 the United States Supreme Court held that the
challenge to the constitutionality of the school's policy allowing student-led prayers and speeches
before games was ripe for adjudication, even if no public prayer had yet been led under the policy,
because the policy was being challenged as unconstitutional on its face.68

That the law or act in question is not yet effective does not negate ripeness. For example, in New
York v. United States,69 decided in 1992, the United States Supreme Court held that the action by
the State of New York challenging the provisions of the Low-Level Radioactive Waste Policy Act was
ripe for adjudication even if the questioned provision was not to take effect until January 1, 1996,
because the parties agreed that New York had to take immediate action to avoid the provision's
consequences.70
The present petitions pray for Certiorari, 71 Prohibition, and Mandamus. Certiorari and Prohibition are
remedies granted by law when any tribunal, board or officer has acted, in the case of certiorari, or is
proceeding, in the case of prohibition, without or in excess of its jurisdiction or with grave abuse of
discretion amounting to lack or excess of jurisdiction.72 Mandamus is a remedy granted by law when
any tribunal, corporation, board, officer or person unlawfully neglects the performance of an act
which the law specifically enjoins as a duty resulting from an office, trust, or station, or unlawfully
excludes another from the use or enjoyment of a right or office to which such other is
entitled.73 Certiorari, Mandamus and Prohibition are appropriate remedies to raise constitutional
issues and to review and/or prohibit/nullify, when proper, acts of legislative and executive officials.74

The authority of the GRP Negotiating Panel is defined by Executive Order No. 3 (E.O. No. 3), issued
on February 28, 2001.75 The said executive order requires that "[t]he government's policy framework
for peace, including the systematic approach and the administrative structure for carrying out the
comprehensive peace process x x x be governed by this Executive Order."76

The present petitions allege that respondents GRP Panel and PAPP Esperon drafted the terms of
the MOA-AD without consulting the local government units or communities affected, nor informing
them of the proceedings. As will be discussed in greater detail later, such omission, by itself,
constitutes a departure by respondents from their mandate under E.O. No. 3.

Furthermore, the petitions allege that the provisions of the MOA-AD violate the Constitution. The
MOA-AD provides that "any provisions of the MOA-AD requiring amendments to the existing legal
framework shall come into force upon the signing of a Comprehensive Compact and upon effecting
the necessary changes to the legal framework," implying an amendment of the Constitution to
accommodate the MOA-AD. This stipulation, in effect, guaranteed to the MILF the amendment of
the Constitution. Such act constitutes another violation of its authority . Again, these points will be
discussed in more detail later.

As the petitions allege acts or omissions on the part of respondent that exceed their authority, by
violating their duties under E.O. No. 3 and the provisions of the Constitution and statutes, the
petitions make a prima facie case for Certiorari, Prohibition, and Mandamus, and an actual case or
controversy ripe for adjudication exists. When an act of a branch of government is seriously
alleged to have infringed the Constitution, it becomes not only the right but in fact the duty of
the judiciary to settle the dispute.77

B. LOCUS STANDI

For a party to have locus standi, one must allege "such a personal stake in the outcome of the
controversy as to assure that concrete adverseness which sharpens the presentation of issues upon
which the court so largely depends for illumination of difficult constitutional questions."78

Because constitutional cases are often public actions in which the relief sought is likely to affect
other persons, a preliminary question frequently arises as to this interest in the constitutional
question raised.79

When suing as a citizen, the person complaining must allege that he has been or is about to be
denied some right or privilege to which he is lawfully entitled or that he is about to be subjected to
some burdens or penalties by reason of the statute or act complained of.80 When the issue concerns
a public right, it is sufficient that the petitioner is a citizen and has an interest in the execution of the
laws.81
For a taxpayer, one is allowed to sue where there is an assertion that public funds are illegally
disbursed or deflected to an illegal purpose, or that there is a wastage of public funds through the
enforcement of an invalid or unconstitutional law.82 The Court retains discretion whether or not to
allow a taxpayer's suit.83

In the case of a legislator or member of Congress, an act of the Executive that injures the institution
of Congress causes a derivative but nonetheless substantial injury that can be questioned by
legislators. A member of the House of Representatives has standing to maintain inviolate the
prerogatives, powers and privileges vested by the Constitution in his office.84

An organization may be granted standing to assert the rights of its members, 85 but the mere
invocation by the Integrated Bar of the Philippines or any member of the legal profession of the duty
to preserve the rule of law does not suffice to clothe it with standing.86

As regards a local government unit (LGU), it can seek relief in order to protect or vindicate an
interest of its own, and of the other LGUs.87

Intervenors, meanwhile, may be given legal standing upon showing of facts that satisfy the
requirements of the law authorizing intervention,88 such as a legal interest in the matter in litigation,
or in the success of either of the parties.

In any case, the Court has discretion to relax the procedural technicality on locus standi, given the
liberal attitude it has exercised, highlighted in the case of David v. Macapagal-Arroyo,89 where
technicalities of procedure were brushed aside, the constitutional issues raised being of paramount
public interest or of transcendental importance deserving the attention of the Court in view of their
seriousness, novelty and weight as precedents.90 The Court's forbearing stance on locus standi on
issues involving constitutional issues has for its purpose the protection of fundamental rights.

In not a few cases, the Court, in keeping with its duty under the Constitution to determine whether
the other branches of government have kept themselves within the limits of the Constitution and the
laws and have not abused the discretion given them, has brushed aside technical rules of
procedure.91

In the petitions at bar, petitioners Province of North Cotabato (G.R. No. 183591) Province of


Zamboanga del Norte (G.R. No. 183951), City of Iligan (G.R. No. 183893) and City of
Zamboanga (G.R. No. 183752) and petitioners-in-intervention Province of Sultan Kudarat, City of
Isabela and Municipality of Linamon have locus standi in view of the direct and substantial injury
that they, as LGUs, would suffer as their territories, whether in whole or in part, are to be included in
the intended domain of the BJE. These petitioners allege that they did not vote for their inclusion in
the ARMM which would be expanded to form the BJE territory. Petitioners' legal standing is thus
beyond doubt.

In G.R. No. 183962, petitioners Ernesto Maceda, Jejomar Binay and Aquilino Pimentel III would


have no standing as citizens and taxpayers for their failure to specify that they would be denied
some right or privilege or there would be wastage of public funds. The fact that they are a former
Senator, an incumbent mayor of Makati City, and a resident of Cagayan de Oro, respectively, is of
no consequence. Considering their invocation of the transcendental importance of the issues at
hand, however, the Court grants them standing.

Intervenors Franklin Drilon and Adel Tamano, in alleging their standing as taxpayers, assert that


government funds would be expended for the conduct of an illegal and unconstitutional plebiscite to
delineate the BJE territory. On that score alone, they can be given legal standing. Their allegation
that the issues involved in these petitions are of "undeniable transcendental importance" clothes
them with added basis for their personality to intervene in these petitions.

With regard to Senator Manuel Roxas, his standing is premised on his being a member of the
Senate and a citizen to enforce compliance by respondents of the public's constitutional right to be
informed of the MOA-AD, as well as on a genuine legal interest in the matter in litigation, or in the
success or failure of either of the parties. He thus possesses the requisite standing as an intervenor.

With respect to Intervenors Ruy Elias Lopez, as a former congressman of the 3rd district of Davao
City, a taxpayer and a member of the Bagobo tribe; Carlo B. Gomez, et al., as members of the IBP
Palawan chapter, citizens and taxpayers; Marino Ridao, as taxpayer, resident and member of
the Sangguniang Panlungsod of Cotabato City; and Kisin Buxani, as taxpayer, they failed to allege
any proper legal interest in the present petitions. Just the same, the Court exercises its discretion to
relax the procedural technicality on locus standi given the paramount public interest in the issues at
hand.

Intervening respondents Muslim Multi-Sectoral Movement for Peace and Development, an


advocacy group for justice and the attainment of peace and prosperity in Muslim Mindanao;
and Muslim Legal Assistance Foundation Inc., a non-government organization of Muslim lawyers,
allege that they stand to be benefited or prejudiced, as the case may be, in the resolution of the
petitions concerning the MOA-AD, and prays for the denial of the petitions on the grounds therein
stated. Such legal interest suffices to clothe them with standing.

B. MOOTNESS

Respondents insist that the present petitions have been rendered moot with the satisfaction of all the
reliefs prayed for by petitioners and the subsequent pronouncement of the Executive Secretary that
"[n]o matter what the Supreme Court ultimately decides[,] the government will not sign the MOA."92

In lending credence to this policy decision, the Solicitor General points out that the President had
already disbanded the GRP Peace Panel.93

In David v. Macapagal-Arroyo,94 this Court held that the "moot and academic" principle not being a
magical formula that automatically dissuades courts in resolving a case, it will decide cases,
otherwise moot and academic, if it finds that (a) there is a grave violation of the Constitution;95 (b) the
situation is of exceptional character and paramount public interest is involved; 96 (c) the constitutional
issue raised requires formulation of controlling principles to guide the bench, the bar, and the
public;97 and (d) the case is capable of repetition yet evading review.98

Another exclusionary circumstance that may be considered is where there is a voluntary cessation of


the activity complained of by the defendant or doer. Thus, once a suit is filed and the doer voluntarily
ceases the challenged conduct, it does not automatically deprive the tribunal of power to hear and
determine the case and does not render the case moot especially when the plaintiff seeks damages
or prays for injunctive relief against the possible recurrence of the violation.99

The present petitions fall squarely into these exceptions to thus thrust them into the domain of
judicial review. The grounds cited above in David are just as applicable in the present cases as they
were, not only in David, but also in Province of Batangas v. Romulo100 and Manalo v.
Calderon101 where the Court similarly decided them on the merits, supervening events that would
ordinarily have rendered the same moot notwithstanding.
Petitions not mooted

Contrary then to the asseverations of respondents, the non-signing of the MOA-AD and the eventual
dissolution of the GRP Peace Panel did not moot the present petitions. It bears emphasis that the
signing of the MOA-AD did not push through due to the Court's issuance of a Temporary Restraining
Order.

Contrary too to respondents' position, the MOA-AD cannot be considered a mere "list of consensus
points," especially given its nomenclature, the need to have it signed or initialed by all the parties
concerned on August 5, 2008, and the far-reaching Constitutional implications of these
"consensus points," foremost of which is the creation of the BJE.

In fact, as what will, in the main, be discussed, there is a commitment on the part of respondents
to amend and effect necessary changes to the existing legal framework for certain provisions
of the MOA-AD to take effect. Consequently, the present petitions are not confined to the terms
and provisions of the MOA-AD, but to other on-going and future negotiations and agreements
necessary for its realization. The petitions have not, therefore, been rendered moot and academic
simply by the public disclosure of the MOA-AD,102 the manifestation that it will not be signed as well
as the disbanding of the GRP Panel not withstanding.

Petitions are imbued with paramount public interest

There is no gainsaying that the petitions are imbued with paramount public interest, involving a
significant part of the country's territory and the wide-ranging political modifications of affected LGUs.
The assertion that the MOA-AD is subject to further legal enactments including possible
Constitutional amendments more than ever provides impetus for the Court to formulate
controlling principles to guide the bench, the bar, the public and, in this case, the
government and its negotiating entity.

Respondents cite Suplico v. NEDA, et al.103 where the Court did not "pontificat[e] on issues which no
longer legitimately constitute an actual case or controversy [as this] will do more harm than good to
the nation as a whole."

The present petitions must be differentiated from Suplico. Primarily, in Suplico, what was assailed
and eventually cancelled was a stand-alone government procurement contract for a national
broadband network involving a one-time contractual relation between two parties-the government
and a private foreign corporation. As the issues therein involved specific government procurement
policies and standard principles on contracts, the majority opinion in Suplico found nothing
exceptional therein, the factual circumstances being peculiar only to the transactions and parties
involved in the controversy.

The MOA-AD is part of a series of agreements

In the present controversy, the MOA-AD is a significant part of a series of agreements  necessary


to carry out the Tripoli Agreement 2001. The MOA-AD which dwells on the Ancestral Domain
Aspect of said Tripoli Agreement is the third such component to be undertaken following the
implementation of the Security Aspect in August 2001 and the Humanitarian, Rehabilitation and
Development Aspect in May 2002.

Accordingly, even if the Executive Secretary, in his Memorandum of August 28, 2008 to the Solicitor
General, has stated that "no matter what the Supreme Court ultimately decides[,] the government
will not sign the MOA[-AD]," mootness will not set in in light of the terms of the Tripoli Agreement
2001.

Need to formulate principles-guidelines

Surely, the present MOA-AD can be renegotiated or another one will be drawn up to carry out the
Ancestral Domain Aspect of the Tripoli Agreement 2001, in another or in any form, which could
contain similar or significantly drastic provisions. While the Court notes the word of the Executive
Secretary that the government "is committed to securing an agreement that is both constitutional and
equitable because that is the only way that long-lasting peace can be assured," it is minded to
render a decision on the merits in the present petitions to formulate controlling principles to
guide the bench, the bar, the public and, most especially, the government in negotiating with
the MILF regarding Ancestral Domain.

Respondents invite the Court's attention to the separate opinion of then Chief Justice Artemio
Panganiban in Sanlakas v. Reyes104 in which he stated that the doctrine of "capable of repetition yet
evading review" can override mootness, "provided the party raising it in a proper case has been
and/or continue to be prejudiced or damaged as a direct result of their issuance." They contend that
the Court must have jurisdiction over the subject matter for the doctrine to be invoked.

The present petitions all contain prayers for Prohibition over which this Court exercises original
jurisdiction. While G.R. No. 183893 (City of Iligan v. GRP) is a petition for Injunction and Declaratory
Relief, the Court will treat it as one for Prohibition as it has far reaching implications and raises
questions that need to be resolved.105 At all events, the Court has jurisdiction over most if not the rest
of the petitions.

Indeed, the present petitions afford a proper venue for the Court to again apply the doctrine
immediately referred to as what it had done in a number of landmark cases. 106 There is
a reasonable expectation that petitioners, particularly the Provinces of North Cotabato, Zamboanga
del Norte and Sultan Kudarat, the Cities of Zamboanga, Iligan and Isabela, and the Municipality of
Linamon, will again be subjected to the same problem in the future as respondents' actions are
capable of repetition, in another or any form.

It is with respect to the prayers for Mandamus that the petitions have become moot, respondents
having, by Compliance of August 7, 2008, provided this Court and petitioners with official copies of
the final draft of the MOA-AD and its annexes. Too, intervenors have been furnished, or have
procured for themselves, copies of the MOA-AD.

V. SUBSTANTIVE ISSUES

As culled from the Petitions and Petitions-in-Intervention, there are basically two SUBSTANTIVE
issues to be resolved, one relating to the manner in which the MOA-AD was negotiated and
finalized, the other relating to its provisions, viz:

1. Did respondents violate constitutional and statutory provisions on public consultation and the right
to information when they negotiated and later initialed the MOA-AD?

2. Do the contents of the MOA-AD violate the Constitution and the laws?

ON THE FIRST SUBSTANTIVE ISSUE


Petitioners invoke their constitutional right to information on matters of public concern, as
provided in Section 7, Article III on the Bill of Rights:

Sec. 7. The right of the people to information on matters of public concern shall be
recognized. Access to official records, and to documents, and papers pertaining to official
acts, transactions, or decisions, as well as to government research data used as basis for
policy development, shall be afforded the citizen, subject to such limitations as may be
provided by law.107

As early as 1948, in Subido v. Ozaeta,108 the Court has recognized the statutory right to examine and
inspect public records, a right which was eventually accorded constitutional status.

The right of access to public documents, as enshrined in both the 1973 Constitution and the 1987
Constitution, has been recognized as a self-executory constitutional right.109

In the 1976 case of Baldoza v. Hon. Judge Dimaano,110 the Court ruled that access to public records
is predicated on the right of the people to acquire information on matters of public concern since,
undoubtedly, in a democracy, the pubic has a legitimate interest in matters of social and political
significance.

x x x The incorporation of this right in the Constitution is a recognition of the fundamental role of free
exchange of information in a democracy. There can be no realistic perception by the public of the
nation's problems, nor a meaningful democratic decision-making if they are denied access to
information of general interest. Information is needed to enable the members of society to cope with
the exigencies of the times. As has been aptly observed: "Maintaining the flow of such information
depends on protection for both its acquisition and its dissemination since, if either process is
interrupted, the flow inevitably ceases." x x x111

In the same way that free discussion enables members of society to cope with the exigencies of
their time, access to information of general interest aids the people in democratic decision-making
by giving them a better perspective of the vital issues confronting the nation 112 so that they may be
able to criticize and participate in the affairs of the government in a responsible, reasonable and
effective manner. It is by ensuring an unfettered and uninhibited exchange of ideas among a well-
informed public that a government remains responsive to the changes desired by the people. 113

The MOA-AD is a matter of public concern

That the subject of the information sought in the present cases is a matter of public concern 114 faces
no serious challenge. In fact, respondents admit that the MOA-AD is indeed of public concern.115 In
previous cases, the Court found that the regularity of real estate transactions entered in the Register
of Deeds,116 the need for adequate notice to the public of the various laws, 117 the civil service
eligibility of a public employee,118 the proper management of GSIS funds allegedly used to grant
loans to public officials,119 the recovery of the Marcoses' alleged ill-gotten wealth,120 and the identity of
party-list nominees,121 among others, are matters of public concern. Undoubtedly, the MOA-AD
subject of the present cases is of public concern, involving as it does the sovereignty and
territorial integrity of the State, which directly affects the lives of the public at large.

Matters of public concern covered by the right to information include steps and negotiations leading
to the consummation of the contract. In not distinguishing as to the executory nature or commercial
character of agreements, the Court has categorically ruled:
x x x [T]he right to information "contemplates inclusion of negotiations leading to the
consummation of the transaction." Certainly, a consummated contract is not a
requirement for the exercise of the right to information. Otherwise, the people can never
exercise the right if no contract is consummated, and if one is consummated, it may be too
late for the public to expose its defects.

Requiring a consummated contract will keep the public in the dark until the contract, which
may be grossly disadvantageous to the government or even illegal, becomes fait accompli.
This negates the State policy of full transparency on matters of public concern, a situation
which the framers of the Constitution could not have intended. Such a requirement will
prevent the citizenry from participating in the public discussion of any proposed contract,
effectively truncating a basic right enshrined in the Bill of Rights. We can allow neither an
emasculation of a constitutional right, nor a retreat by the State of its avowed "policy of full
disclosure of all its transactions involving public interest." 122 (Emphasis and italics in the
original)

Intended as a "splendid symmetry"123 to the right to information under the Bill of Rights is the policy of
public disclosure under Section 28, Article II of the Constitution reading:

Sec. 28. Subject to reasonable conditions prescribed by law, the State adopts and
implements a policy of full public disclosure of all its transactions involving public interest.124

The policy of full public disclosure enunciated in above-quoted Section 28 complements the right of


access to information on matters of public concern found in the Bill of Rights. The right to information
guarantees the right of the people to demand information, while Section 28 recognizes the duty of
officialdom to give information even if nobody demands.125

The policy of public disclosure establishes a concrete ethical principle for the conduct of public
affairs in a genuinely open democracy, with the people's right to know as the centerpiece. It is a
mandate of the State to be accountable by following such policy.126 These provisions are vital to the
exercise of the freedom of expression and essential to hold public officials at all times accountable to
the people.127

Whether Section 28 is self-executory, the records of the deliberations of the Constitutional


Commission so disclose:

MR. SUAREZ. And since this is not self-executory, this policy will not be enunciated or will
not be in force and effect until after Congress shall have provided it.

MR. OPLE. I expect it to influence the climate of public ethics immediately but, of course, the
implementing law will have to be enacted by Congress, Mr. Presiding Officer.128

The following discourse, after Commissioner Hilario Davide, Jr., sought clarification on the issue, is
enlightening.

MR. DAVIDE. I would like to get some clarifications on this. Mr. Presiding Officer, did I get
the Gentleman correctly as having said that this is not a self-executing provision? It would
require a legislation by Congress to implement?
MR. OPLE. Yes. Originally, it was going to be self-executing, but I accepted an amendment
from Commissioner Regalado, so that the safeguards on national interest are modified by
the clause "as may be provided by law"

MR. DAVIDE. But as worded, does it not mean that this will immediately take effect and
Congress may provide for reasonable safeguards on the sole ground national interest?

MR. OPLE. Yes. I think so, Mr. Presiding Officer, I said earlier that it should
immediately influence the climate of the conduct of public affairs but, of course,
Congress here may no longer pass a law revoking it, or if this is approved, revoking this
principle, which is inconsistent with this policy.129 (Emphasis supplied)

Indubitably, the effectivity of the policy of public disclosure need not await the passing of a
statute. As Congress cannot revoke this principle, it is merely directed to provide for "reasonable
safeguards." The complete and effective exercise of the right to information necessitates that its
complementary provision on public disclosure derive the same self-executory nature. Since both
provisions go hand-in-hand, it is absurd to say that the broader 130 right to information on matters of
public concern is already enforceable while the correlative duty of the State to disclose its
transactions involving public interest is not enforceable until there is an enabling law. Respondents
cannot thus point to the absence of an implementing legislation as an excuse in not effecting such
policy.

An essential element of these freedoms is to keep open a continuing dialogue or process of


communication between the government and the people. It is in the interest of the State that the
channels for free political discussion be maintained to the end that the government may perceive
and be responsive to the people's will.131Envisioned to be corollary to the twin rights to information
and disclosure is the design for feedback mechanisms.

MS. ROSARIO BRAID. Yes. And lastly, Mr. Presiding Officer, will the people be able to
participate? Will the government provide feedback mechanisms so that the people
can participate and can react where the existing media facilities are not able to
provide full feedback mechanisms to the government? I suppose this will be part of
the government implementing operational mechanisms.

MR. OPLE. Yes. I think through their elected representatives and that is how these courses
take place. There is a message and a feedback, both ways.

xxxx

MS. ROSARIO BRAID. Mr. Presiding Officer, may I just make one last sentence?

I think when we talk about the feedback network, we are not talking about public
officials but also network of private business o[r] community-based organizations that
will be reacting. As a matter of fact, we will put more credence or credibility on the private
network of volunteers and voluntary community-based organizations. So I do not think we
are afraid that there will be another OMA in the making.132(Emphasis supplied)

The imperative of a public consultation, as a species of the right to information, is evident in the
"marching orders" to respondents. The mechanics for the duty to disclose information and to conduct
public consultation regarding the peace agenda and process is manifestly provided by E.O. No.
3.133 The preambulatory clause of E.O. No. 3 declares that there is a need to further enhance the
contribution of civil society to the comprehensive peace process by institutionalizing the people's
participation.

One of the three underlying principles of the comprehensive peace process is that it "should be
community-based, reflecting the sentiments, values and principles important to all Filipinos" and
"shall be defined not by the government alone, nor by the different contending groups only, but by all
Filipinos as one community."134 Included as a component of the comprehensive peace process is
consensus-building and empowerment for peace, which includes "continuing consultations on both
national and local levels to build consensus for a peace agenda and process, and the mobilization
and facilitation of people's participation in the peace process."135

Clearly, E.O. No. 3 contemplates not just the conduct of a plebiscite to effectuate
"continuing" consultations, contrary to respondents' position that plebiscite is "more than
sufficient consultation."136

Further, E.O. No. 3 enumerates the functions and responsibilities of the PAPP, one of which is to
"[c]onduct regular dialogues with the National Peace Forum (NPF) and other peace partners to seek
relevant information, comments, recommendations as well as to render appropriate and timely
reports on the progress of the comprehensive peace process."137 E.O. No. 3 mandates the
establishment of the NPF to be "the principal forum for the PAPP to consult with and seek advi[c]e
from the peace advocates, peace partners and concerned sectors of society on both national and
local levels, on the implementation of the comprehensive peace process, as well as for
government[-]civil society dialogue and consensus-building on peace agenda and initiatives."138

In fine, E.O. No. 3 establishes petitioners' right to be consulted on the peace agenda, as a
corollary to the constitutional right to information and disclosure.

PAPP Esperon committed grave abuse of discretion

The PAPP committed grave abuse of discretion  when he failed to carry out the pertinent
consultation. The furtive process by which the MOA-AD was designed and crafted runs contrary to
and in excess of the legal authority , and amounts to a whimsical, capricious, oppressive, arbitrary
and despotic exercise thereof.

The Court may not, of course, require the PAPP to conduct the consultation in a particular way or
manner. It may, however, require him to comply with the law and discharge the functions within the
authority granted by the President.139

Petitioners are not claiming a seat at the negotiating table, contrary to respondents' retort in justifying
the denial of petitioners' right to be consulted. Respondents' stance manifests the manner by which
they treat the salient provisions of E.O. No. 3 on people's participation. Such disregard of the
express mandate of the President is not much different from superficial conduct toward token
provisos that border on classic lip service.140 It illustrates a gross evasion of positive duty and a
virtual refusal to perform the duty enjoined.

As for respondents' invocation of the doctrine of executive privilege, it is not tenable under the
premises. The argument defies sound reason when contrasted with E.O. No. 3's explicit provisions
on continuing consultation and dialogue on both national and local levels. The executive order
even recognizes the exercise of the public's right even before the GRP makes its official
recommendations or before the government proffers its definite propositions.141 It bear emphasis that
E.O. No. 3 seeks to elicit relevant advice, information, comments and recommendations from the
people through dialogue.
AT ALL EVENTS, respondents effectively waived the defense of executive privilege in view of their
unqualified disclosure of the official copies of the final draft of the MOA-AD. By unconditionally
complying with the Court's August 4, 2008 Resolution, without a prayer for the document's
disclosure in camera, or without a manifestation that it was complying therewith ex abundante ad
cautelam.

Petitioners' assertion that the Local Government Code (LGC) of 1991 declares it a State policy to
"require all national agencies and offices to conduct periodic consultations with appropriate local
government units, non-governmental and people's organizations, and other concerned sectors of the
community before any project or program is implemented in their respective jurisdictions"142 is well-
taken. The LGC chapter on intergovernmental relations puts flesh into this avowed policy:

Prior Consultations Required. - No project or program shall be implemented by government


authorities unlessthe consultations mentioned in Sections 2 (c) and 26 hereof are complied
with, and prior approval of the sanggunian concerned is obtained: Provided, That occupants
in areas where such projects are to be implemented shall not be evicted unless appropriate
relocation sites have been provided, in accordance with the provisions of the
Constitution.143 (Italics and underscoring supplied)

In Lina, Jr. v. Hon. Paño,144 the Court held that the above-stated policy and above-quoted provision
of the LGU apply only to national programs or projects which are to be implemented in a particular
local community. Among the programs and projects covered are those that are critical to the
environment and human ecology including those that may call for the eviction of a particular group of
people residing in the locality where these will be implemented.145 The MOA-AD is one peculiar
program that unequivocally and unilaterally vests ownership of a vast territory to the
Bangsamoro people,146 which could pervasively and drastically result to the diaspora or
displacement of a great number of inhabitants from their total environment.

With respect to the indigenous cultural communities/indigenous peoples (ICCs/IPs), whose interests
are represented herein by petitioner Lopez and are adversely affected by the MOA-AD, the ICCs/IPs
have, under the IPRA, the right to participate fully at all levels of decision-making in matters which
may affect their rights, lives and destinies. 147 The MOA-AD, an instrument recognizing ancestral
domain, failed to justify its non-compliance with the clear-cut mechanisms ordained in said
Act,148 which entails, among other things, the observance of the free and prior informed consent of
the ICCs/IPs.

Notably, the IPRA does not grant the Executive Department or any government agency the power to
delineate and recognize an ancestral domain claim by mere agreement or compromise. The
recognition of the ancestral domain is the raison d'etre of the MOA-AD, without which all other
stipulations or "consensus points" necessarily must fail. In proceeding to make a sweeping
declaration on ancestral domain, without complying with the IPRA, which is cited as one of the TOR
of the MOA-AD, respondents clearly transcended the boundaries of their authority. As it
seems, even the heart of the MOA-AD is still subject to necessary changes to the legal framework.
While paragraph 7 on Governance suspends the effectivity of all provisions requiring changes to the
legal framework, such clause is itself invalid, as will be discussed in the following section.

Indeed, ours is an open society, with all the acts of the government subject to public scrutiny and
available always to public cognizance. This has to be so if the country is to remain democratic, with
sovereignty residing in the people and all government authority emanating from them.149

ON THE SECOND SUBSTANTIVE ISSUE


With regard to the provisions of the MOA-AD, there can be no question that they cannot all be
accommodated under the present Constitution and laws. Respondents have admitted as much in the
oral arguments before this Court, and the MOA-AD itself recognizes the need to amend the existing
legal framework to render effective at least some of its provisions. Respondents, nonetheless,
counter that the MOA-AD is free of any legal infirmity because any provisions therein which are
inconsistent with the present legal framework will not be effective until the necessary changes to that
framework are made. The validity of this argument will be considered later. For now, the Court shall
pass upon how

The MOA-AD is inconsistent with the Constitution and laws as presently worded.

In general, the objections against the MOA-AD center on the extent of the powers conceded therein
to the BJE. Petitioners assert that the powers granted to the BJE exceed those granted to any local
government under present laws, and even go beyond those of the present ARMM. Before assessing
some of the specific powers that would have been vested in the BJE, however, it would be useful to
turn first to a general idea that serves as a unifying link to the different provisions of the MOA-AD,
namely, the international law concept of association. Significantly, the MOA-AD explicitly alludes to
this concept, indicating that the Parties actually framed its provisions with it in mind.

Association is referred to in paragraph 3 on TERRITORY, paragraph 11 on RESOURCES, and


paragraph 4 on GOVERNANCE. It is in the last mentioned provision, however, that the MOA-AD
most clearly uses it to describe the envisioned relationship between the BJE and the Central
Government.

4. The relationship between the Central Government and the Bangsamoro juridical
entity shall be associative characterized by shared authority and responsibility  with a
structure of governance based on executive, legislative, judicial and administrative
institutions with defined powers and functions in the comprehensive compact. A period of
transition shall be established in a comprehensive peace compact specifying the relationship
between the Central Government and the BJE. (Emphasis and underscoring supplied)

The nature of the "associative" relationship may have been intended to be defined more precisely in
the still to be forged Comprehensive Compact. Nonetheless, given that there is a concept of
"association" in international law, and the MOA-AD - by its inclusion of international law instruments
in its TOR- placed itself in an international legal context, that concept of association may be brought
to bear in understanding the use of the term "associative" in the MOA-AD.

Keitner and Reisman state that

[a]n association is formed when two states of unequal power voluntarily establish durable


links. In the basic model, one state, the associate, delegates certain responsibilities to
the other, the principal, while maintaining its international status as a state. Free
associations represent a middle ground between integration and independence. x x
x150 (Emphasis and underscoring supplied)

For purposes of illustration, the Republic of the Marshall Islands and the Federated States of
Micronesia (FSM), formerly part of the U.S.-administered Trust Territory of the Pacific Islands, 151 are
associated states of the U.S. pursuant to a Compact of Free Association. The currency in these
countries is the U.S. dollar, indicating their very close ties with the U.S., yet they issue their own
travel documents, which is a mark of their statehood. Their international legal status as states was
confirmed by the UN Security Council and by their admission to UN membership.
According to their compacts of free association, the Marshall Islands and the FSM generally have
the capacity to conduct foreign affairs in their own name and right, such capacity extending to
matters such as the law of the sea, marine resources, trade, banking, postal, civil aviation, and
cultural relations. The U.S. government, when conducting its foreign affairs, is obligated to consult
with the governments of the Marshall Islands or the FSM on matters which it (U.S. government)
regards as relating to or affecting either government.

In the event of attacks or threats against the Marshall Islands or the FSM, the U.S. government has
the authority and obligation to defend them as if they were part of U.S. territory. The U.S.
government, moreover, has the option of establishing and using military areas and facilities within
these associated states and has the right to bar the military personnel of any third country from
having access to these territories for military purposes.

It bears noting that in U.S. constitutional and international practice, free association is understood as
an international association between sovereigns. The Compact of Free Association is a treaty which
is subordinate to the associated nation's national constitution, and each party may terminate the
association consistent with the right of independence. It has been said that, with the admission of the
U.S.-associated states to the UN in 1990, the UN recognized that the American model of free
association is actually based on an underlying status of independence.152

In international practice, the "associated state" arrangement has usually been used as a transitional
device of former colonies on their way to full independence . Examples of states that have passed
through the status of associated states as a transitional phase are Antigua, St. Kitts-Nevis-Anguilla,
Dominica, St. Lucia, St. Vincent and Grenada. All have since become independent states.153

Back to the MOA-AD, it contains many provisions which are consistent with the international legal
concept of association, specifically the following: the BJE's capacity to enter into economic and trade
relations with foreign countries, the commitment of the Central Government to ensure the BJE's
participation in meetings and events in the ASEAN and the specialized UN agencies, and the
continuing responsibility of the Central Government over external defense. Moreover, the BJE's right
to participate in Philippine official missions bearing on negotiation of border agreements,
environmental protection, and sharing of revenues pertaining to the bodies of water adjacent to or
between the islands forming part of the ancestral domain, resembles the right of the governments of
FSM and the Marshall Islands to be consulted by the U.S. government on any foreign affairs matter
affecting them.

These provisions of the MOA indicate, among other things, that the Parties aimed to vest in the
BJE the status of an associated state or, at any rate, a status closely approximating it.

The concept of association is not recognized under the present Constitution

No province, city, or municipality, not even the ARMM, is recognized under our laws as having an
"associative" relationship with the national government. Indeed, the concept implies powers that go
beyond anything ever granted by the Constitution to any local or regional government. It also implies
the recognition of the associated entity as a state. The Constitution, however, does not contemplate
any state in this jurisdiction other than the Philippine State, much less does it provide for a transitory
status that aims to prepare any part of Philippine territory for independence.

Even the mere concept animating many of the MOA-AD's provisions, therefore, already requires for
its validity the amendment of constitutional provisions, specifically the following provisions of Article
X:
SECTION 1. The territorial and political subdivisions of the Republic of the Philippines are
the provinces, cities, municipalities, and barangays. There shall be autonomous
regions in Muslim Mindanao and the Cordilleras as hereinafter provided.

SECTION 15. There shall be created autonomous regions in Muslim Mindanao and in the
Cordilleras consisting of provinces, cities, municipalities, and geographical areas sharing
common and distinctive historical and cultural heritage, economic and social structures, and
other relevant characteristics within the framework of this Constitution and the national
sovereignty as well as territorial integrity of the Republic of the Philippines.

The BJE is a far more powerful 


entity than the autonomous region 
recognized in the Constitution

It is not merely an expanded version of the ARMM, the status of its relationship with the national
government being fundamentally different from that of the ARMM. Indeed, BJE is a state in all but
name as it meets the criteria of a state laid down in the Montevideo Convention,154 namely,
a permanent population, a defined territory, a government, and a capacity to enter into relations with
other states.

Even assuming arguendo that the MOA-AD would not necessarily sever any portion of Philippine
territory, the spirit animating it - which has betrayed itself by its use of the concept of association
- runs counter to the national sovereignty and territorial integrity of the Republic.

The defining concept underlying the relationship between the national government and the
BJE being itself contrary to the present Constitution, it is not surprising that many of the
specific provisions of the MOA-AD on the formation and powers of the BJE are in conflict
with the Constitution and the laws.

Article X, Section 18 of the Constitution provides that "[t]he creation of the autonomous region shall
be effective when approved by a majority of the votes cast by the constituent units in a plebiscite
called for the purpose, provided that only provinces, cities, and geographic areas voting
favorably in such plebiscite shall be included in the autonomous region." (Emphasis supplied)

As reflected above, the BJE is more of a state than an autonomous region. But even assuming that it
is covered by the term "autonomous region" in the constitutional provision just quoted, the MOA-AD
would still be in conflict with it. Under paragraph 2(c) on TERRITORY in relation to 2(d) and 2(e), the
present geographic area of the ARMM and, in addition, the municipalities of Lanao del Norte which
voted for inclusion in the ARMM during the 2001 plebiscite - Baloi, Munai, Nunungan, Pantar,
Tagoloan and Tangkal - are automatically part of the BJE without need of another plebiscite, in
contrast to the areas under Categories A and B mentioned earlier in the overview. That the present
components of the ARMM and the above-mentioned municipalities voted for inclusion therein in
2001, however, does not render another plebiscite unnecessary under the Constitution, precisely
because what these areas voted for then was their inclusion in the ARMM, not the BJE.

The MOA-AD, moreover, would not


comply with Article X, Section 20 of 
the Constitution

since that provision defines the powers of autonomous regions as follows:


SECTION 20. Within its territorial jurisdiction and subject to the provisions of this Constitution
and national laws, the organic act of autonomous regions shall provide for legislative powers
over:

(1) Administrative organization;

(2) Creation of sources of revenues;

(3) Ancestral domain and natural resources;

(4) Personal, family, and property relations;

(5) Regional urban and rural planning development;

(6) Economic, social, and tourism development;

(7) Educational policies;

(8) Preservation and development of the cultural heritage; and

(9) Such other matters as may be authorized by law for the promotion of the general welfare
of the people of the region. (Underscoring supplied)

Again on the premise that the BJE may be regarded as an autonomous region, the MOA-AD would
require an amendment that would expand the above-quoted provision. The mere passage of new
legislation pursuant to sub-paragraph No. 9 of said constitutional provision would not suffice, since
any new law that might vest in the BJE the powers found in the MOA-AD must, itself, comply with
other provisions of the Constitution. It would not do, for instance, to merely pass legislation vesting
the BJE with treaty-making power in order to accommodate paragraph 4 of the strand on
RESOURCES which states: "The BJE is free to enter into any economic cooperation and trade
relations with foreign countries: provided, however, that such relationships and understandings do
not include aggression against the Government of the Republic of the Philippines x x x." Under our
constitutional system, it is only the President who has that power. Pimentel v. Executive
Secretary155 instructs:

In our system of government, the President, being the head of state, is regarded as the sole
organ and authority in external relations and is the country's sole representative with
foreign nations. As the chief architect of foreign policy, the President acts as the country's
mouthpiece with respect to international affairs. Hence, the President is vested with the
authority to deal with foreign states and governments, extend or withhold
recognition, maintain diplomatic relations, enter into treaties, and otherwise transact
the business of foreign relations. In the realm of treaty-making, the President has the
sole authority to negotiate with other states. (Emphasis and underscoring supplied)

Article II, Section 22 of the Constitution must also be amended if the scheme envisioned in
the MOA-AD is to be effected. That constitutional provision states: "The State recognizes and
promotes the rights of indigenous cultural communities within the framework of national unity and
development." (Underscoring supplied) An associative arrangement does not uphold national unity.
While there may be a semblance of unity because of the associative ties between the BJE and the
national government, the act of placing a portion of Philippine territory in a status which, in
international practice, has generally been a preparation for independence, is certainly not conducive
to national unity.

Besides being irreconcilable with the Constitution, the MOA-AD is also inconsistent with prevailing
statutory law, among which are R.A. No. 9054156 or the Organic Act of the ARMM, and the IPRA.157

Article X, Section 3 of the Organic Act of the ARMM is a bar to the adoption of the definition
of "Bangsamoro people" used in the MOA-AD. Paragraph 1 on Concepts and Principles states:

1. It is the birthright of all Moros and all Indigenous peoples of Mindanao to identify


themselves and be accepted as "Bangsamoros". The Bangsamoro people refers to those
who are natives or original inhabitants of Mindanao and its adjacent islands including
Palawan and the Sulu archipelago at the time of conquest or colonization of its descendants
whether mixed or of full blood. Spouses and their descendants are classified as
Bangsamoro. The freedom of choice of the Indigenous people shall be respected. (Emphasis
and underscoring supplied)

This use of the term Bangsamoro sharply contrasts with that found in the Article X, Section 3 of the
Organic Act, which, rather than lumping together the identities of the Bangsamoro and other
indigenous peoples living in Mindanao, clearly distinguishes between Bangsamoro people and
Tribal peoples, as follows:

"As used in this Organic Act, the phrase "indigenous cultural community" refers to Filipino
citizens residing in the autonomous region who are:

(a) Tribal peoples. These are citizens whose social, cultural and economic conditions
distinguish them from other sectors of the national community; and

(b) Bangsa Moro people. These are citizens who are believers in Islam and who have
retained some or all of their own social, economic, cultural, and political institutions."

Respecting the IPRA, it lays down the prevailing procedure for the delineation and recognition of
ancestral domains. The MOA-AD's manner of delineating the ancestral domain of the Bangsamoro
people is a clear departure from that procedure. By paragraph 1 of Territory, the Parties simply
agree that, subject to the delimitations in the agreed Schedules, "[t]he Bangsamoro homeland and
historic territory refer to the land mass as well as the maritime, terrestrial, fluvial and alluvial
domains, and the aerial domain, the atmospheric space above it, embracing the Mindanao-Sulu-
Palawan geographic region."

Chapter VIII of the IPRA, on the other hand, lays down a detailed procedure, as illustrated in the
following provisions thereof:

SECTION 52. Delineation Process. - The identification and delineation of ancestral domains
shall be done in accordance with the following procedures:

xxxx

b) Petition for Delineation. - The process of delineating a specific perimeter may be initiated
by the NCIP with the consent of the ICC/IP concerned, or through a Petition for Delineation
filed with the NCIP, by a majority of the members of the ICCs/IPs;
c) Delineation Proper. - The official delineation of ancestral domain boundaries including
census of all community members therein, shall be immediately undertaken by the Ancestral
Domains Office upon filing of the application by the ICCs/IPs concerned. Delineation will be
done in coordination with the community concerned and shall at all times include genuine
involvement and participation by the members of the communities concerned;

d) Proof Required. - Proof of Ancestral Domain Claims shall include the testimony of elders
or community under oath, and other documents directly or indirectly attesting to the
possession or occupation of the area since time immemorial by such ICCs/IPs in the concept
of owners which shall be any one (1) of the following authentic documents:

1) Written accounts of the ICCs/IPs customs and traditions;

2) Written accounts of the ICCs/IPs political structure and institution;

3) Pictures showing long term occupation such as those of old improvements, burial
grounds, sacred places and old villages;

4) Historical accounts, including pacts and agreements concerning boundaries


entered into by the ICCs/IPs concerned with other ICCs/IPs;

5) Survey plans and sketch maps;

6) Anthropological data;

7) Genealogical surveys;

8) Pictures and descriptive histories of traditional communal forests and hunting


grounds;

9) Pictures and descriptive histories of traditional landmarks such as mountains,


rivers, creeks, ridges, hills, terraces and the like; and

10) Write-ups of names and places derived from the native dialect of the community.

e) Preparation of Maps. - On the basis of such investigation and the findings of fact based
thereon, the Ancestral Domains Office of the NCIP shall prepare a perimeter map, complete
with technical descriptions, and a description of the natural features and landmarks
embraced therein;

f) Report of Investigation and Other Documents. - A complete copy of the preliminary census
and a report of investigation, shall be prepared by the Ancestral Domains Office of the NCIP;

g) Notice and Publication. - A copy of each document, including a translation in the native
language of the ICCs/IPs concerned shall be posted in a prominent place therein for at least
fifteen (15) days. A copy of the document shall also be posted at the local, provincial and
regional offices of the NCIP, and shall be published in a newspaper of general circulation
once a week for two (2) consecutive weeks to allow other claimants to file opposition thereto
within fifteen (15) days from date of such publication: Provided, That in areas where no such
newspaper exists, broadcasting in a radio station will be a valid substitute: Provided, further,
That mere posting shall be deemed sufficient if both newspaper and radio station are not
available;

h) Endorsement to NCIP. - Within fifteen (15) days from publication, and of the inspection
process, the Ancestral Domains Office shall prepare a report to the NCIP endorsing a
favorable action upon a claim that is deemed to have sufficient proof. However, if the proof is
deemed insufficient, the Ancestral Domains Office shall require the submission of additional
evidence: Provided, That the Ancestral Domains Office shall reject any claim that is deemed
patently false or fraudulent after inspection and verification: Provided, further, That in case of
rejection, the Ancestral Domains Office shall give the applicant due notice, copy furnished all
concerned, containing the grounds for denial. The denial shall be appealable to the NCIP:
Provided, furthermore, That in cases where there are conflicting claims among ICCs/IPs on
the boundaries of ancestral domain claims, the Ancestral Domains Office shall cause the
contending parties to meet and assist them in coming up with a preliminary resolution of the
conflict, without prejudice to its full adjudication according to the section below.

xxxx

To remove all doubts about the irreconcilability of the MOA-AD with the present legal system, a
discussion of not only the Constitution and domestic statutes, but also of international law is in order,
for

Article II, Section 2 of the Constitution states that the Philippines "adopts the generally
accepted principles of international law as part of the law of the land."

Applying this provision of the Constitution, the Court, in Mejoff v. Director of Prisons,158 held that the
Universal Declaration of Human Rights is part of the law of the land on account of which it ordered
the release on bail of a detained alien of Russian descent whose deportation order had not been
executed even after two years. Similarly, the Court in Agustin v. Edu159 applied the aforesaid
constitutional provision to the 1968 Vienna Convention on Road Signs and Signals.

International law has long recognized the right to self-determination of "peoples," understood not
merely as the entire population of a State but also a portion thereof. In considering the question of
whether the people of Quebec had a right to unilaterally secede from Canada, the Canadian
Supreme Court in REFERENCE RE SECESSION OF QUEBEC160 had occasion to acknowledge that
"the right of a people to self-determination is now so widely recognized in international conventions
that the principle has acquired a status beyond ‘convention' and is considered a general principle of
international law."

Among the conventions referred to are the International Covenant on Civil and Political Rights161 and
the International Covenant on Economic, Social and Cultural Rights 162 which state, in Article 1 of both
covenants, that all peoples, by virtue of the right of self-determination, "freely determine their political
status and freely pursue their economic, social, and cultural development."

The people's right to self-determination should not, however, be understood as extending to a


unilateral right of secession. A distinction should be made between the right of internal and external
self-determination. REFERENCE RE SECESSION OF QUEBEC is again instructive:

"(ii) Scope of the Right to Self-determination


126. The recognized sources of international law establish that the right to self-
determination of a people is normally fulfilled through internal self-determination - a
people's pursuit of its political, economic, social and cultural development within the
framework of an existing state. A right to external self-determination (which in this
case potentially takes the form of the assertion of a right to unilateral secession)
arises in only the most extreme of cases and, even then, under carefully defined
circumstances. x x x

External self-determination can be defined as in the following statement from


the Declaration on Friendly Relations, supra, as

The establishment of a sovereign and independent State, the free association or


integration with an independent State or the emergence into any other political status
freely determined by a peopleconstitute modes of implementing the right of self-
determination by that people. (Emphasis added)

127. The international law principle of self-determination has evolved within a


framework of respect for the territorial integrity of existing states. The various
international documents that support the existence of a people's right to self-determination
also contain parallel statements supportive of the conclusion that the exercise of such a right
must be sufficiently limited to prevent threats to an existing state's territorial integrity or the
stability of relations between sovereign states.

x x x x (Emphasis, italics and underscoring supplied)

The Canadian Court went on to discuss the exceptional cases in which the right to external self-
determination can arise, namely, where a people is under colonial rule, is subject to foreign
domination or exploitation outside a colonial context, and - less definitely but asserted by a number
of commentators - is blocked from the meaningful exercise of its right to internal self-determination.
The Court ultimately held that the population of Quebec had no right to secession, as the same is
not under colonial rule or foreign domination, nor is it being deprived of the freedom to make political
choices and pursue economic, social and cultural development, citing that Quebec is equitably
represented in legislative, executive and judicial institutions within Canada, even occupying
prominent positions therein.

The exceptional nature of the right of secession is further exemplified in the REPORT OF THE
INTERNATIONAL COMMITTEE OF JURISTS ON THE LEGAL ASPECTS OF THE AALAND
ISLANDS QUESTION.163 There, Sweden presented to the Council of the League of Nations the
question of whether the inhabitants of the Aaland Islands should be authorized to determine by
plebiscite if the archipelago should remain under Finnish sovereignty or be incorporated in the
kingdom of Sweden. The Council, before resolving the question, appointed an International
Committee composed of three jurists to submit an opinion on the preliminary issue of whether the
dispute should, based on international law, be entirely left to the domestic jurisdiction of Finland. The
Committee stated the rule as follows:

x x x [I]n the absence of express provisions in international treaties, the right of disposing


of national territory is essentially an attribute of the sovereignty of every State.
Positive International Law does not recognize the right of national groups, as such, to
separate themselves from the State of which they form part by the simple expression
of a wish, any more than it recognizes the right of other States to claim such a
separation. Generally speaking, the grant or refusal of the right to a portion of its
population of determining its own political fate by plebiscite or by some other method,
is, exclusively, an attribute of the sovereignty of every State which is definitively
constituted. A dispute between two States concerning such a question, under normal
conditions therefore, bears upon a question which International Law leaves entirely to the
domestic jurisdiction of one of the States concerned. Any other solution would amount to an
infringement of sovereign rights of a State and would involve the risk of creating difficulties
and a lack of stability which would not only be contrary to the very idea embodied in term
"State," but would also endanger the interests of the international community. If this right is
not possessed by a large or small section of a nation, neither can it be held by the State to
which the national group wishes to be attached, nor by any other State. (Emphasis and
underscoring supplied)

The Committee held that the dispute concerning the Aaland Islands did not refer to a question which
is left by international law to the domestic jurisdiction of Finland, thereby applying the exception
rather than the rule elucidated above. Its ground for departing from the general rule, however, was a
very narrow one, namely, the Aaland Islands agitation originated at a time when Finland was
undergoing drastic political transformation. The internal situation of Finland was, according to the
Committee, so abnormal that, for a considerable time, the conditions required for the formation of a
sovereign State did not exist. In the midst of revolution, anarchy, and civil war, the legitimacy of the
Finnish national government was disputed by a large section of the people, and it had, in fact, been
chased from the capital and forcibly prevented from carrying out its duties. The armed camps and
the police were divided into two opposing forces. In light of these circumstances, Finland was not,
during the relevant time period, a "definitively constituted" sovereign state. The Committee,
therefore, found that Finland did not possess the right to withhold from a portion of its population the
option to separate itself - a right which sovereign nations generally have with respect to their own
populations.

Turning now to the more specific category of indigenous peoples, this term has been used, in
scholarship as well as international, regional, and state practices, to refer to groups with distinct
cultures, histories, and connections to land (spiritual and otherwise) that have been forcibly
incorporated into a larger governing society. These groups are regarded as "indigenous" since they
are the living descendants of pre-invasion inhabitants of lands now dominated by others. Otherwise
stated, indigenous peoples, nations, or communities are culturally distinctive groups that find
themselves engulfed by settler societies born of the forces of empire and conquest. 164 Examples of
groups who have been regarded as indigenous peoples are the Maori of New Zealand and the
aboriginal peoples of Canada.

As with the broader category of "peoples," indigenous peoples situated within states do not have a
general right to independence or secession from those states under international law,165 but they do
have rights amounting to what was discussed above as the right to internal self-determination.

In a historic development last September 13, 2007, the UN General Assembly adopted the United
Nations Declaration on the Rights of Indigenous Peoples (UN DRIP) through General Assembly
Resolution 61/295. The vote was 143 to 4, the Philippines being included among those in favor,
and the four voting against being Australia, Canada, New Zealand, and the U.S. The Declaration
clearly recognized the right of indigenous peoples to self-determination, encompassing the
right to autonomy or self-government, to wit:

Article 3

Indigenous peoples have the right to self-determination. By virtue of that right they freely
determine their political status and freely pursue their economic, social and cultural
development.
Article 4

Indigenous peoples, in exercising their right to self-determination, have the right


to autonomy or self-government in matters relating to their internal and local affairs,
as well as ways and means for financing their autonomous functions.

Article 5

Indigenous peoples have the right to maintain and strengthen their distinct political, legal,
economic, social and cultural institutions, while retaining their right to participate fully, if they
so choose, in the political, economic, social and cultural life of the State.

Self-government, as used in international legal discourse pertaining to indigenous peoples, has been
understood as equivalent to "internal self-determination."166 The extent of self-determination provided
for in the UN DRIP is more particularly defined in its subsequent articles, some of which are quoted
hereunder:

Article 8

1. Indigenous peoples and individuals have the right not to be subjected to forced
assimilation or destruction of their culture.

2. States shall provide effective mechanisms for prevention of, and redress for:

(a) Any action which has the aim or effect of depriving them of their integrity as
distinct peoples, or of their cultural values or ethnic identities;

(b) Any action which has the aim or effect of dispossessing them of their lands,
territories or resources;

(c) Any form of forced population transfer which has the aim or effect of violating or
undermining any of their rights;

(d) Any form of forced assimilation or integration;

(e) Any form of propaganda designed to promote or incite racial or ethnic


discrimination directed against them.

Article 21

1. Indigenous peoples have the right, without discrimination, to the improvement of their
economic and social conditions, including, inter alia, in the areas of education, employment,
vocational training and retraining, housing, sanitation, health and social security.

2. States shall take effective measures and, where appropriate, special measures to ensure
continuing improvement of their economic and social conditions. Particular attention shall be
paid to the rights and special needs of indigenous elders, women, youth, children and
persons with disabilities.

Article 26
1. Indigenous peoples have the right to the lands, territories and resources which they
have traditionally owned, occupied or otherwise used or acquired.

2. Indigenous peoples have the right to own, use, develop and control the lands, territories
and resources that they possess by reason of traditional ownership or other traditional
occupation or use, as well as those which they have otherwise acquired.

3. States shall give legal recognition and protection to these lands, territories and resources.
Such recognition shall be conducted with due respect to the customs, traditions and land
tenure systems of the indigenous peoples concerned.

Article 30

1. Military activities shall not take place in the lands or territories of indigenous peoples,
unless justified by a relevant public interest or otherwise freely agreed with or requested by
the indigenous peoples concerned.

2. States shall undertake effective consultations with the indigenous peoples concerned,
through appropriate procedures and in particular through their representative institutions,
prior to using their lands or territories for military activities.

Article 32

1. Indigenous peoples have the right to determine and develop priorities and strategies for
the development or use of their lands or territories and other resources.

2. States shall consult and cooperate in good faith with the indigenous peoples concerned
through their own representative institutions in order to obtain their free and informed
consent prior to the approval of any project affecting their lands or territories and other
resources, particularly in connection with the development, utilization or exploitation of
mineral, water or other resources.

3. States shall provide effective mechanisms for just and fair redress for any such activities,
and appropriate measures shall be taken to mitigate adverse environmental, economic,
social, cultural or spiritual impact.

Article 37

1. Indigenous peoples have the right to the recognition, observance and enforcement of
treaties, agreements and other constructive arrangements concluded with States or their
successors and to have States honour and respect such treaties, agreements and other
constructive arrangements.

2. Nothing in this Declaration may be interpreted as diminishing or eliminating the rights of


indigenous peoples contained in treaties, agreements and other constructive arrangements.

Article 38

States in consultation and cooperation with indigenous peoples, shall take the appropriate
measures, including legislative measures, to achieve the ends of this Declaration.
Assuming that the UN DRIP, like the Universal Declaration on Human Rights, must now be regarded
as embodying customary international law - a question which the Court need not definitively resolve
here - the obligations enumerated therein do not strictly require the Republic to grant the
Bangsamoro people, through the instrumentality of the BJE, the particular rights and powers
provided for in the MOA-AD. Even the more specific provisions of the UN DRIP are general in scope,
allowing for flexibility in its application by the different States.

There is, for instance, no requirement in the UN DRIP that States now guarantee indigenous
peoples their own police and internal security force. Indeed, Article 8 presupposes that it is the State
which will provide protection for indigenous peoples against acts like the forced dispossession of
their lands - a function that is normally performed by police officers. If the protection of a right so
essential to indigenous people's identity is acknowledged to be the responsibility of the State, then
surely the protection of rights less significant to them as such peoples would also be the duty of
States. Nor is there in the UN DRIP an acknowledgement of the right of indigenous peoples to the
aerial domain and atmospheric space. What it upholds, in Article 26 thereof, is the right of
indigenous peoples to the lands, territories and resources which they have traditionally owned,
occupied or otherwise used or acquired.

Moreover, the UN DRIP, while upholding the right of indigenous peoples to autonomy, does not
obligate States to grant indigenous peoples the near-independent status of an associated state. All
the rights recognized in that document are qualified in Article 46 as follows:

1. Nothing in this Declaration may be interpreted as implying for any State, people, group
or person any right to engage in any activity or to perform any act contrary to the Charter of
the United Nations or construed as authorizing or encouraging any action which would
dismember or impair, totally or in part, the territorial integrity or political unity of
sovereign and independent States.

Even if the UN DRIP were considered as part of the law of the land pursuant to Article II, Section 2
of the Constitution, it would not suffice to uphold the validity of the MOA-AD so as to render its
compliance with other laws unnecessary.

It is, therefore, clear that the MOA-AD contains numerous provisions that cannot be
reconciled with the Constitution and the laws as presently worded. Respondents proffer,
however, that the signing of the MOA-AD alone would not have entailed any violation of law or grave
abuse of discretion on their part, precisely because it stipulates that the provisions thereof
inconsistent with the laws shall not take effect until these laws are amended. They cite paragraph 7
of the MOA-AD strand on GOVERNANCE quoted earlier, but which is reproduced below for
convenience:

7. The Parties agree that the mechanisms and modalities for the actual implementation of
this MOA-AD shall be spelt out in the Comprehensive Compact to mutually take such steps
to enable it to occur effectively.

Any provisions of the MOA-AD requiring amendments to the existing legal framework shall
come into force upon signing of a Comprehensive Compact and upon effecting the
necessary changes to the legal framework with due regard to non derogation of prior
agreements and within the stipulated timeframe to be contained in the Comprehensive
Compact.

Indeed, the foregoing stipulation keeps many controversial provisions of the MOA-AD from coming
into force until the necessary changes to the legal framework are effected. While the word
"Constitution" is not mentioned in the provision now under consideration or anywhere else
in the MOA-AD, the term "legal framework" is certainly broad enough to include the
Constitution.

Notwithstanding the suspensive clause, however, respondents, by their mere act of incorporating in
the MOA-AD the provisions thereof regarding the associative relationship between the BJE and the
Central Government, have already violated the Memorandum of Instructions From The President
dated March 1, 2001, which states that the "negotiations shall be conducted in accordance with x x x
the principles of the sovereignty and territorial integrityof the Republic of the Philippines."
(Emphasis supplied) Establishing an associative relationship between the BJE and the Central
Government is, for the reasons already discussed, a preparation for independence, or worse, an
implicit acknowledgment of an independent status already prevailing.

Even apart from the above-mentioned Memorandum, however, the MOA-AD is defective because
the suspensive clause is invalid, as discussed below.

The authority of the GRP Peace Negotiating Panel to negotiate with the MILF is founded on E.O. No.
3, Section 5(c), which states that there shall be established Government Peace Negotiating Panels
for negotiations with different rebel groups to be "appointed by the President as her official
emissaries to conduct negotiations, dialogues, and face-to-face discussions with rebel groups."
These negotiating panels are to report to the President, through the PAPP on the conduct and
progress of the negotiations.

It bears noting that the GRP Peace Panel, in exploring lasting solutions to the Moro Problem through
its negotiations with the MILF, was not restricted by E.O. No. 3 only to those options available under
the laws as they presently stand. One of the components of a comprehensive peace process, which
E.O. No. 3 collectively refers to as the "Paths to Peace," is the pursuit of social, economic, and
political reforms which may require new legislation or even constitutional amendments. Sec. 4(a) of
E.O. No. 3, which reiterates Section 3(a), of E.O. No. 125,167 states:

SECTION 4. The Six Paths to Peace. - The components of the comprehensive peace
process comprise the processes known as the "Paths to Peace". These component
processes are interrelated and not mutually exclusive, and must therefore be pursued
simultaneously in a coordinated and integrated fashion. They shall include, but may not be
limited to, the following:

a. PURSUIT OF SOCIAL, ECONOMIC AND POLITICAL REFORMS. This component


involves the vigorous implementation of various policies, reforms, programs and
projects aimed at addressing the root causes of internal armed conflicts and social
unrest. This may require administrative action, new legislation or even constitutional
amendments.

x x x x (Emphasis supplied)

The MOA-AD, therefore, may reasonably be perceived as an attempt of respondents to address,


pursuant to this provision of E.O. No. 3, the root causes of the armed conflict in Mindanao. The E.O.
authorized them to "think outside the box," so to speak. Hence, they negotiated and were set on
signing the MOA-AD that included various social, economic, and political reforms which cannot,
however, all be accommodated within the present legal framework, and which thus would require
new legislation and constitutional amendments.
The inquiry on the legality of the "suspensive clause," however, cannot stop here, because it must
be asked whether the President herself may exercise the power delegated to the GRP Peace
Panel under E.O. No. 3, Sec. 4(a).

The President cannot delegate a power that she herself does not possess. May the President, in the
course of peace negotiations, agree to pursue reforms that would require new legislation and
constitutional amendments, or should the reforms be restricted only to those solutions which the
present laws allow? The answer to this question requires a discussion of the extent of the
President's power to conduct peace negotiations.

That the authority of the President to conduct peace negotiations with rebel groups is not explicitly
mentioned in the Constitution does not mean that she has no such authority. In Sanlakas v.
Executive Secretary,168 in issue was the authority of the President to declare a state of rebellion - an
authority which is not expressly provided for in the Constitution. The Court held thus:

"In her ponencia in Marcos v. Manglapus, Justice Cortes put her thesis into jurisprudence.
There, the Court, by a slim 8-7 margin, upheld the President's power to forbid the return of
her exiled predecessor. The rationale for the majority's ruling rested on the President's

. . . unstated residual powers which are implied from the grant of executive


power and which are necessary for her to comply with her duties under the
Constitution. The powers of the President are not limited to what are expressly
enumerated in the article on the Executive Department and in scattered
provisions of the Constitution. This is so, notwithstanding the avowed intent of the
members of the Constitutional Commission of 1986 to limit the powers of the
President as a reaction to the abuses under the regime of Mr. Marcos, for the result
was a limitation of specific powers of the President, particularly those relating to the
commander-in-chief clause, but not a diminution of the general grant of executive
power.

Thus, the President's authority to declare a state of rebellion springs in the main from
her powers as chief executive and, at the same time, draws strength from her
Commander-in-Chief powers. x x x (Emphasis and underscoring supplied)

Similarly, the President's power to conduct peace negotiations is implicitly included in her powers as
Chief Executive and Commander-in-Chief. As Chief Executive, the President has the general
responsibility to promote public peace, and as Commander-in-Chief, she has the more specific duty
to prevent and suppress rebellion and lawless violence.169

As the experience of nations which have similarly gone through internal armed conflict will show,
however, peace is rarely attained by simply pursuing a military solution. Oftentimes, changes as far-
reaching as a fundamental reconfiguration of the nation's constitutional structure is required. The
observations of Dr. Kirsti Samuels are enlightening, to wit:

x x x [T]he fact remains that a successful political and governance transition must form the
core of any post-conflict peace-building mission. As we have observed in Liberia and Haiti
over the last ten years, conflict cessation without modification of the political environment,
even where state-building is undertaken through technical electoral assistance and
institution- or capacity-building, is unlikely to succeed. On average, more than 50 percent of
states emerging from conflict return to conflict. Moreover, a substantial proportion of
transitions have resulted in weak or limited democracies.
The design of a constitution and its constitution-making process can play an important role in
the political and governance transition. Constitution-making after conflict is an opportunity to
create a common vision of the future of a state and a road map on how to get there. The
constitution can be partly a peace agreement and partly a framework setting up the rules by
which the new democracy will operate.170

In the same vein, Professor Christine Bell, in her article on the nature and legal status of peace
agreements, observed that the typical way that peace agreements establish or confirm mechanisms
for demilitarization and demobilization is by linking them to new constitutional
structures addressing governance, elections, and legal and human rights institutions.171

In the Philippine experience, the link between peace agreements and constitution-making has been
recognized by no less than the framers of the Constitution. Behind the provisions of the Constitution
on autonomous regions172 is the framers' intention to implement a particular peace agreement,
namely, the Tripoli Agreement of 1976 between the GRP and the MNLF, signed by then
Undersecretary of National Defense Carmelo Z. Barbero and then MNLF Chairman Nur Misuari.

MR. ROMULO. There are other speakers; so, although I have some more questions, I will
reserve my right to ask them if they are not covered by the other speakers. I have only two
questions.

I heard one of the Commissioners say that local autonomy already exists in the
Muslim region; it is working very well; it has, in fact, diminished a great deal of the
problems. So, my question is: since that already exists, why do we have to go into
something new?

MR. OPLE. May I answer that on behalf of Chairman Nolledo. Commissioner Yusup
Abubakar is right that certain definite steps have been taken to implement the
provisions of the Tripoli Agreement with respect to an autonomous region in
Mindanao. This is a good first step, but there is no question that this is merely a
partial response to the Tripoli Agreement itself and to the fuller standard of regional
autonomy contemplated in that agreement, and now by state policy.173(Emphasis
supplied)

The constitutional provisions on autonomy and the statutes enacted pursuant to them have, to the
credit of their drafters, been partly successful. Nonetheless, the Filipino people are still faced with
the reality of an on-going conflict between the Government and the MILF. If the President is to be
expected to find means for bringing this conflict to an end and to achieve lasting peace in Mindanao,
then she must be given the leeway to explore, in the course of peace negotiations, solutions that
may require changes to the Constitution for their implementation. Being uniquely vested with the
power to conduct peace negotiations with rebel groups, the President is in a singular position to
know the precise nature of their grievances which, if resolved, may bring an end to hostilities.

The President may not, of course, unilaterally implement the solutions that she considers viable, but
she may not be prevented from submitting them as recommendations to Congress, which could
then, if it is minded, act upon them pursuant to the legal procedures for constitutional amendment
and revision. In particular, Congress would have the option, pursuant to Article XVII, Sections 1 and
3 of the Constitution, to propose the recommended amendments or revision to the people, call a
constitutional convention, or submit to the electorate the question of calling such a convention.

While the President does not possess constituent powers - as those powers may be exercised only
by Congress, a Constitutional Convention, or the people through initiative and referendum - she may
submit proposals for constitutional change to Congress in a manner that does not involve the
arrogation of constituent powers.

In Sanidad v. COMELEC,174 in issue was the legality of then President Marcos' act of directly
submitting proposals for constitutional amendments to a referendum, bypassing the interim National
Assembly which was the body vested by the 1973 Constitution with the power to propose such
amendments. President Marcos, it will be recalled, never convened the interim National Assembly.
The majority upheld the President's act, holding that "the urges of absolute necessity" compelled the
President as the agent of the people to act as he did, there being no interim National Assembly to
propose constitutional amendments. Against this ruling, Justices Teehankee and Muñoz Palma
vigorously dissented. The Court's concern at present, however, is not with regard to the point on
which it was then divided in that controversial case, but on that which was not disputed by either
side.

Justice Teehankee's dissent,175 in particular, bears noting. While he disagreed that the President may
directly submit proposed constitutional amendments to a referendum, implicit in his opinion is a
recognition that he would have upheld the President's action along with the majority had the
President convened the interim National Assembly and coursed his proposals through it. Thus
Justice Teehankee opined:

"Since the Constitution provides for the organization of the essential departments of
government, defines and delimits the powers of each and prescribes the manner of the
exercise of such powers, and the constituent power has not been granted to but has been
withheld from the President or Prime Minister, it follows that the President's questioned
decrees proposing and submitting constitutional amendments directly to the people (without
the intervention of the interim National Assembly in whom the power is expressly
vested) are devoid of constitutional and legal basis."176 (Emphasis supplied)

From the foregoing discussion, the principle may be inferred that the President - in the course of
conducting peace negotiations - may validly consider implementing even those policies that require
changes to the Constitution, but she may not unilaterally implement them without the intervention
of Congress, or act in any way as if the assent of that body were assumed as a certainty.

Since, under the present Constitution, the people also have the power to directly propose
amendments through initiative and referendum, the President may also submit her
recommendations to the people, not as a formal proposal to be voted on in a plebiscite similar to
what President Marcos did in Sanidad, but for their independent consideration of whether these
recommendations merit being formally proposed through initiative.

These recommendations, however, may amount to nothing more than the President's suggestions to
the people, for any further involvement in the process of initiative by the Chief Executive may vitiate
its character as a genuine "people's initiative." The only initiative recognized by the Constitution is
that which truly proceeds from the people. As the Court stated in Lambino v. COMELEC:177

"The Lambino Group claims that their initiative is the ‘people's voice.' However, the Lambino
Group unabashedly states in ULAP Resolution No. 2006-02, in the verification of their
petition with the COMELEC, that ‘ULAP maintains its unqualified support to the agenda of
Her Excellency President Gloria Macapagal-Arroyo for constitutional reforms.' The Lambino
Group thus admits that their ‘people's' initiative is an ‘unqualified support to the
agenda' of the incumbent President to change the Constitution. This forewarns the Court to
be wary of incantations of ‘people's voice' or ‘sovereign will' in the present initiative."
It will be observed that the President has authority, as stated in her oath of office,178 only to preserve
and defend the Constitution. Such presidential power does not, however, extend to allowing her to
change the Constitution, but simply to recommend proposed amendments or revision. As long as
she limits herself to recommending these changes and submits to the proper procedure for
constitutional amendments and revision, her mere recommendation need not be construed as an
unconstitutional act.

The foregoing discussion focused on the President's authority to


propose constitutional amendments, since her authority to propose new legislation is not in
controversy. It has been an accepted practice for Presidents in this jurisdiction to propose new
legislation. One of the more prominent instances the practice is usually done is in the yearly State of
the Nation Address of the President to Congress. Moreover, the annual general appropriations bill
has always been based on the budget prepared by the President, which - for all intents and
purposes - is a proposal for new legislation coming from the President.179

The "suspensive clause" in the MOA-AD viewed in light of the above-discussed standards

Given the limited nature of the President's authority to propose constitutional amendments,
she cannot guaranteeto any third party that the required amendments will eventually be put in
place, nor even be submitted to a plebiscite. The most she could do is submit these proposals as
recommend ations either to Congress or the people, in whom constituent powers are vested.

Paragraph 7 on Governance of the MOA-AD states, however, that all provisions thereof which
cannot be reconciled with the present Constitution and laws "shall come into force upon signing of a
Comprehensive Compact and upon effecting the necessary changes to the legal framework." This
stipulation does not bear the marks of a suspensive condition - defined in civil law as a future
and uncertain event - but of a term. It is not a question of whether the necessary changes to the
legal framework will be effected, but when. That there is no uncertainty being contemplated is plain
from what follows, for the paragraph goes on to state that the contemplated changes shall be "with
due regard to non derogation of prior agreements and within the stipulated timeframe to be
contained in the Comprehensive Compact."

Pursuant to this stipulation, therefore, it is mandatory for the GRP to effect the changes to the legal
framework contemplated in the MOA-AD - which changes would include constitutional amendments,
as discussed earlier. It bears noting that,

By the time these changes are put in place, the MOA-AD itself would be counted among the
"prior agreements" from which there could be no derogation.

What remains for discussion in the Comprehensive Compact would merely be the implementing
details for these "consensus points" and, notably, the deadline for effecting the contemplated
changes to the legal framework.

Plainly, stipulation-paragraph 7 on GOVERNANCE is inconsistent with the limits of the


President's authority to propose constitutional amendments, it being a virtual guarantee that
the Constitution and the laws of the Republic of the Philippines will certainly be adjusted to conform
to all the "consensus points" found in the MOA-AD. Hence, it must be struck down
as unconstitutional.

A comparison between the "suspensive clause" of the MOA-AD with a similar provision appearing in
the 1996 final peace agreement between the MNLF and the GRP is most instructive.
As a backdrop, the parties to the 1996 Agreement stipulated that it would be implemented in two
phases. Phase Icovered a three-year transitional period involving the putting up of new
administrative structures through Executive Order, such as the Special Zone of Peace and
Development (SZOPAD) and the Southern Philippines Council for Peace and Development
(SPCPD), while Phase II covered the establishment of the new regional autonomous
government through amendment or repeal of R.A. No. 6734, which was then the Organic Act of the
ARMM.

The stipulations on Phase II consisted of specific agreements on the structure of the expanded
autonomous region envisioned by the parties. To that extent, they are similar to the provisions of the
MOA-AD. There is, however, a crucial difference between the two agreements. While the MOA-
AD virtually guarantees that the "necessary changes to the legal framework" will be put in
place, the GRP-MNLF final peace agreement states thus: "Accordingly, these provisions [on Phase
II] shall be recommended by the GRP to Congress for incorporation in the amendatory or repealing
law."

Concerns have been raised that the MOA-AD would have given rise to a binding international law
obligation on the part of the Philippines to change its Constitution in conformity thereto, on the
ground that it may be considered either as a binding agreement under international law, or a
unilateral declaration of the Philippine government to the international community that it would grant
to the Bangsamoro people all the concessions therein stated. Neither ground finds sufficient support
in international law, however.

The MOA-AD, as earlier mentioned in the overview thereof, would have included foreign dignitaries
as signatories. In addition, representatives of other nations were invited to witness its signing in
Kuala Lumpur. These circumstances readily lead one to surmise that the MOA-AD would have had
the status of a binding international agreement had it been signed. An examination of the prevailing
principles in international law, however, leads to the contrary conclusion.

The Decision on Challenge to Jurisdiction: Lomé Accord Amnesty180 (the Lomé Accord case) of the
Special Court of Sierra Leone is enlightening. The Lomé Accord was a peace agreement signed on
July 7, 1999 between the Government of Sierra Leone and the Revolutionary United Front (RUF), a
rebel group with which the Sierra Leone Government had been in armed conflict for around eight
years at the time of signing. There were non-contracting signatories to the agreement, among which
were the Government of the Togolese Republic, the Economic Community of West African States,
and the UN.

On January 16, 2002, after a successful negotiation between the UN Secretary-General and the
Sierra Leone Government, another agreement was entered into by the UN and that Government
whereby the Special Court of Sierra Leone was established. The sole purpose of the Special Court,
an international court, was to try persons who bore the greatest responsibility for serious violations of
international humanitarian law and Sierra Leonean law committed in the territory of Sierra Leone
since November 30, 1996.

Among the stipulations of the Lomé Accord was a provision for the full pardon of the members of the
RUF with respect to anything done by them in pursuit of their objectives as members of that
organization since the conflict began.

In the Lomé Accord case, the Defence argued that the Accord created an internationally
binding obligation not to prosecute the beneficiaries of the amnesty provided therein, citing, among
other things, the participation of foreign dignitaries and international organizations in the finalization
of that agreement. The Special Court, however, rejected this argument, ruling that the Lome Accord
is not a treaty and that it can only create binding obligations and rights between the parties in
municipal law, not in international law. Hence, the Special Court held, it is ineffective in depriving an
international court like it of jurisdiction.

"37. In regard to the nature of a negotiated settlement of an internal armed conflict it is easy


to assume and to argue with some degree of plausibility, as Defence counsel for the
defendants seem to have done, that the mere fact that in addition to the parties to the
conflict, the document formalizing the settlement is signed by foreign heads of state
or their representatives and representatives of international organizations, means the
agreement of the parties is internationalized so as to create obligations in
international law.

xxxx

40. Almost every conflict resolution will involve the parties to the conflict and the mediator or
facilitator of the settlement, or persons or bodies under whose auspices the settlement took
place but who are not at all parties to the conflict, are not contracting parties and who do not
claim any obligation from the contracting parties or incur any obligation from the settlement.

41. In this case, the parties to the conflict are the lawful authority of the State and the
RUF which has no status of statehood and is to all intents and purposes a faction
within the state. The non-contracting signatories of the Lomé Agreement were moral
guarantors of the principle that, in the terms of Article XXXIV of the Agreement, "this
peace agreement is implemented with integrity and in good faith by both parties". The
moral guarantors assumed no legal obligation. It is recalled that the UN by its
representative appended, presumably for avoidance of doubt, an understanding of the extent
of the agreement to be implemented as not including certain international crimes.

42. An international agreement in the nature of a treaty must create rights and obligations
regulated by international law so that a breach of its terms will be a breach determined under
international law which will also provide principle means of enforcement. The Lomé
Agreement created neither rights nor obligations capable of being regulated by
international law. An agreement such as the Lomé Agreement which brings to an end
an internal armed conflict no doubt creates a factual situation of restoration of peace
that the international community acting through the Security Council may take note
of. That, however, will not convert it to an international agreement which creates an
obligation enforceable in international, as distinguished from municipal, law. A breach
of the terms of such a peace agreement resulting in resumption of internal armed conflict or
creating a threat to peace in the determination of the Security Council may indicate a
reversal of the factual situation of peace to be visited with possible legal consequences
arising from the new situation of conflict created. Such consequences such as action by the
Security Council pursuant to Chapter VII arise from the situation and not from the agreement,
nor from the obligation imposed by it. Such action cannot be regarded as a remedy for the
breach. A peace agreement which settles an internal armed conflict cannot be
ascribed the same status as one which settles an international armed conflict which,
essentially, must be between two or more warring States. The Lomé Agreement
cannot be characterised as an international instrument. x x x" (Emphasis, italics and
underscoring supplied)

Similarly, that the MOA-AD would have been signed by representatives of States and international
organizations not parties to the Agreement would not have sufficed to vest in it a binding character
under international law.
In another vein, concern has been raised that the MOA-AD would amount to a unilateral declaration
of the Philippine State, binding under international law, that it would comply with all the stipulations
stated therein, with the result that it would have to amend its Constitution accordingly regardless of
the true will of the people. Cited as authority for this view is Australia v. France,181 also known as
the Nuclear Tests Case, decided by the International Court of Justice (ICJ).

In the Nuclear Tests Case, Australia challenged before the ICJ the legality of France's nuclear tests
in the South Pacific. France refused to appear in the case, but public statements from its President,
and similar statements from other French officials including its Minister of Defence, that its 1974
series of atmospheric tests would be its last, persuaded the ICJ to dismiss the case.182 Those
statements, the ICJ held, amounted to a legal undertaking addressed to the international community,
which required no acceptance from other States for it to become effective.

Essential to the ICJ ruling is its finding that the French government intended to be bound to the
international community in issuing its public statements, viz:

43. It is well recognized that declarations made by way of unilateral acts, concerning legal or
factual situations, may have the effect of creating legal obligations. Declarations of this kind
may be, and often are, very specific. When it is the intention of the State making the
declaration that it should become bound according to its terms, that intention confers
on the declaration the character of a legal undertaking, the State being thenceforth
legally required to follow a course of conduct consistent with the declaration. An
undertaking of this kind, if given publicly, and with an intent to be bound, even though not
made within the context of international negotiations, is binding. In these circumstances,
nothing in the nature of a quid pro quo nor any subsequent acceptance of the declaration,
nor even any reply or reaction from other States, is required for the declaration to take effect,
since such a requirement would be inconsistent with the strictly unilateral nature of the
juridical act by which the pronouncement by the State was made.

44. Of course, not all unilateral acts imply obligation; but a State may choose to take
up a certain position in relation to a particular matter with the intention of being
bound-the intention is to be ascertained by interpretation of the act. When States make
statements by which their freedom of action is to be limited, a restrictive interpretation is
called for.

xxxx

51. In announcing that the 1974 series of atmospheric tests would be the last, the
French Government conveyed to the world at large, including the Applicant, its
intention effectively to terminate these tests. It was bound to assume that other States
might take note of these statements and rely on their being effective. The validity of
these statements and their legal consequences must be considered within the general
framework of the security of international intercourse, and the confidence and trust
which are so essential in the relations among States. It is from the actual substance of
these statements, and from the circumstances attending their making, that the legal
implications of the unilateral act must be deduced. The objects of these statements
are clear and they were addressed to the international community as a whole, and the
Court holds that they constitute an undertaking possessing legal effect. The Court
considers *270 that the President of the Republic, in deciding upon the effective cessation of
atmospheric tests, gave an undertaking to the international community to which his words
were addressed. x x x (Emphasis and underscoring supplied)
As gathered from the above-quoted ruling of the ICJ, public statements of a state representative may
be construed as a unilateral declaration only when the following conditions are present: the
statements were clearly addressed to the international community, the state intended to be bound to
that community by its statements, and that not to give legal effect to those statements would be
detrimental to the security of international intercourse. Plainly, unilateral declarations arise only in
peculiar circumstances.

The limited applicability of the Nuclear Tests Case ruling was recognized in a later case decided by
the ICJ entitled Burkina Faso v. Mali,183 also known as the Case Concerning the Frontier Dispute.
The public declaration subject of that case was a statement made by the President of Mali, in an
interview by a foreign press agency, that Mali would abide by the decision to be issued by a
commission of the Organization of African Unity on a frontier dispute then pending between Mali and
Burkina Faso.

Unlike in the Nuclear Tests Case, the ICJ held that the statement of Mali's President was not a
unilateral act with legal implications. It clarified that its ruling in the Nuclear Tests case rested on the
peculiar circumstances surrounding the French declaration subject thereof, to wit:

40. In order to assess the intentions of the author of a unilateral act, account must be taken
of all the factual circumstances in which the act occurred. For example, in the Nuclear
Tests cases, the Court took the view that since the applicant States were not the only
ones concerned at the possible continuance of atmospheric testing by the French
Government, that Government's unilateral declarations had ‘conveyed to the world at
large, including the Applicant, its intention effectively to terminate these tests‘ (I.C.J.
Reports 1974, p. 269, para. 51; p. 474, para. 53). In the particular circumstances of those
cases, the French Government could not express an intention to be bound otherwise
than by unilateral declarations. It is difficult to see how it could have accepted the
terms of a negotiated solution with each of the applicants without thereby
jeopardizing its contention that its conduct was lawful. The circumstances of the
present case are radically different. Here, there was nothing to hinder the Parties from
manifesting an intention to accept the binding character of the conclusions of the
Organization of African Unity Mediation Commission by the normal method: a formal
agreement on the basis of reciprocity. Since no agreement of this kind was concluded
between the Parties, the Chamber finds that there are no grounds to interpret the declaration
made by Mali's head of State on 11 April 1975 as a unilateral act with legal implications in
regard to the present case. (Emphasis and underscoring supplied)

Assessing the MOA-AD in light of the above criteria, it would not have amounted to a unilateral
declaration on the part of the Philippine State to the international community. The Philippine panel
did not draft the same with the clear intention of being bound thereby to the international community
as a whole or to any State, but only to the MILF. While there were States and international
organizations involved, one way or another, in the negotiation and projected signing of the MOA-AD,
they participated merely as witnesses or, in the case of Malaysia, as facilitator. As held in the Lomé
Accord case, the mere fact that in addition to the parties to the conflict, the peace settlement is
signed by representatives of states and international organizations does not mean that the
agreement is internationalized so as to create obligations in international law.

Since the commitments in the MOA-AD were not addressed to States, not to give legal effect to such
commitments would not be detrimental to the security of international intercourse - to the trust and
confidence essential in the relations among States.
In one important respect, the circumstances surrounding the MOA-AD are closer to that of Burkina
Faso wherein, as already discussed, the Mali President's statement was not held to be a binding
unilateral declaration by the ICJ. As in that case, there was also nothing to hinder the Philippine
panel, had it really been its intention to be bound to other States, to manifest that intention by formal
agreement. Here, that formal agreement would have come about by the inclusion in the MOA-AD of
a clear commitment to be legally bound to the international community, not just the MILF, and by an
equally clear indication that the signatures of the participating states-representatives would
constitute an acceptance of that commitment. Entering into such a formal agreement would not have
resulted in a loss of face for the Philippine government before the international community, which
was one of the difficulties that prevented the French Government from entering into a formal
agreement with other countries. That the Philippine panel did not enter into such a formal agreement
suggests that it had no intention to be bound to the international community. On that ground, the
MOA-AD may not be considered a unilateral declaration under international law.

The MOA-AD not being a document that can bind the Philippines under international law
notwithstanding, respondents' almost consummated act of guaranteeing amendments to the legal
framework is, by itself, sufficient to constitute grave abuse of discretion. The grave abuse lies
not in the fact that they considered, as a solution to the Moro Problem, the creation of a state within
a state, but in their brazen willingness to guarantee that Congress and the sovereign Filipino
people would give their imprimatur to their solution. Upholding such an act would amount to
authorizing a usurpation of the constituent powers vested only in Congress, a Constitutional
Convention, or the people themselves through the process of initiative, for the only way that the
Executive can ensure the outcome of the amendment process is through an undue influence or
interference with that process.

The sovereign people may, if it so desired, go to the extent of giving up a portion of its own territory
to the Moros for the sake of peace, for it can change the Constitution in any it wants, so long as the
change is not inconsistent with what, in international law, is known as Jus Cogens.184 Respondents,
however, may not preempt it in that decision.

SUMMARY

The petitions are ripe for adjudication. The failure of respondents to consult the local government
units or communities affected constitutes a departure by respondents from their mandate under E.O.
No. 3. Moreover, respondents exceeded their authority by the mere act of guaranteeing
amendments to the Constitution. Any alleged violation of the Constitution by any branch of
government is a proper matter for judicial review.

As the petitions involve constitutional issues which are of paramount public interest or of
transcendental importance, the Court grants the petitioners, petitioners-in-intervention and
intervening respondents the requisite locus standi in keeping with the liberal stance adopted in David
v. Macapagal-Arroyo.

Contrary to the assertion of respondents that the non-signing of the MOA-AD and the eventual
dissolution of the GRP Peace Panel mooted the present petitions, the Court finds that the present
petitions provide an exception to the "moot and academic" principle in view of (a) the grave violation
of the Constitution involved; (b) the exceptional character of the situation and paramount public
interest; (c) the need to formulate controlling principles to guide the bench, the bar, and the public;
and (d) the fact that the case is capable of repetition yet evading review.

The MOA-AD is a significant part of a series of agreements necessary to carry out the GRP-MILF
Tripoli Agreement on Peace signed by the government and the MILF back in June 2001. Hence, the
present MOA-AD can be renegotiated or another one drawn up that could contain similar or
significantly dissimilar provisions compared to the original.

The Court, however, finds that the prayers for mandamus have been rendered moot in view of the
respondents' action in providing the Court and the petitioners with the official copy of the final draft of
the MOA-AD and its annexes.

The people's right to information on matters of public concern under Sec. 7, Article III of the
Constitution is in splendid symmetry with the state policy of full public disclosure of all its
transactions involving public interest under Sec. 28, Article II of the Constitution. The right to
information guarantees the right of the people to demand information, while Section 28 recognizes
the duty of officialdom to give information even if nobody demands. The complete and effective
exercise of the right to information necessitates that its complementary provision on public
disclosure derive the same self-executory nature, subject only to reasonable safeguards or
limitations as may be provided by law.

The contents of the MOA-AD is a matter of paramount public concern involving public interest in the
highest order. In declaring that the right to information contemplates steps and negotiations leading
to the consummation of the contract, jurisprudence finds no distinction as to the executory nature or
commercial character of the agreement.

An essential element of these twin freedoms is to keep a continuing dialogue or process of


communication between the government and the people. Corollary to these twin rights is the design
for feedback mechanisms. The right to public consultation was envisioned to be a species of these
public rights.

At least three pertinent laws animate these constitutional imperatives and justify the exercise of the
people's right to be consulted on relevant matters relating to the peace agenda.

One, E.O. No. 3 itself is replete with mechanics for continuing consultations on both national and
local levels and for a principal forum for consensus-building. In fact, it is the duty of the Presidential
Adviser on the Peace Process to conduct regular dialogues to seek relevant information, comments,
advice, and recommendations from peace partners and concerned sectors of society.

Two, Republic Act No. 7160 or the Local Government Code of 1991 requires all national offices to
conduct consultations before any project or program critical to the environment and human ecology
including those that may call for the eviction of a particular group of people residing in such locality,
is implemented therein. The MOA-AD is one peculiar program that unequivocally and unilaterally
vests ownership of a vast territory to the Bangsamoro people, which could pervasively and
drastically result to the diaspora or displacement of a great number of inhabitants from their total
environment.

Three, Republic Act No. 8371 or the Indigenous Peoples Rights Act of 1997 provides for clear-cut
procedure for the recognition and delineation of ancestral domain, which entails, among other things,
the observance of the free and prior informed consent of the Indigenous Cultural
Communities/Indigenous Peoples. Notably, the statute does not grant the Executive Department or
any government agency the power to delineate and recognize an ancestral domain claim by mere
agreement or compromise.

The invocation of the doctrine of executive privilege as a defense to the general right to information
or the specific right to consultation is untenable. The various explicit legal provisions fly in the face of
executive secrecy. In any event, respondents effectively waived such defense after it unconditionally
disclosed the official copies of the final draft of the MOA-AD, for judicial compliance and public
scrutiny.

In sum, the Presidential Adviser on the Peace Process committed grave abuse of discretion when he
failed to carry out the pertinent consultation process, as mandated by E.O. No. 3, Republic Act No.
7160, and Republic Act No. 8371. The furtive process by which the MOA-AD was designed and
crafted runs contrary to and in excess of the legal authority, and amounts to a whimsical, capricious,
oppressive, arbitrary and despotic exercise thereof. It illustrates a gross evasion of positive duty and
a virtual refusal to perform the duty enjoined.

The MOA-AD cannot be reconciled with the present Constitution and laws. Not only its specific
provisions but the very concept underlying them, namely, the associative relationship envisioned
between the GRP and the BJE, are unconstitutional, for the concept presupposes that the
associated entity is a state and implies that the same is on its way to independence.

While there is a clause in the MOA-AD stating that the provisions thereof inconsistent with the
present legal framework will not be effective until that framework is amended, the same does not
cure its defect. The inclusion of provisions in the MOA-AD establishing an associative relationship
between the BJE and the Central Government is, itself, a violation of the Memorandum of
Instructions From The President dated March 1, 2001, addressed to the government peace panel.
Moreover, as the clause is worded, it virtually guarantees that the necessary amendments to the
Constitution and the laws will eventually be put in place. Neither the GRP Peace Panel nor the
President herself is authorized to make such a guarantee. Upholding such an act would amount to
authorizing a usurpation of the constituent powers vested only in Congress, a Constitutional
Convention, or the people themselves through the process of initiative, for the only way that the
Executive can ensure the outcome of the amendment process is through an undue influence or
interference with that process.

While the MOA-AD would not amount to an international agreement or unilateral declaration binding
on the Philippines under international law, respondents' act of guaranteeing amendments is, by
itself, already a constitutional violation that renders the MOA-AD fatally defective.

WHEREFORE, respondents' motion to dismiss is DENIED. The main and intervening petitions are
GIVEN DUE COURSE and hereby GRANTED.

The Memorandum of Agreement on the Ancestral Domain Aspect of the GRP-MILF Tripoli
Agreement on Peace of 2001 is declared contrary to law and the Constitution.

SO ORDERED.

ART. II DECLARATION OF PRINCIPLES AND STATE POLICIES

G.R. No. 178160               February 26, 2009

BASES CONVERSION AND DEVELOPMENT AUTHORITY, Petitioner, 


vs.
COMMISSION ON AUDIT, Respondent.
DECISION

CARPIO, J.:

The Case

This is a petition for certiorari1 with prayer for the issuance of a temporary restraining order and a writ
of preliminary injunction. The petition seeks to nullify Decision No. 2007-020 2 dated 12 April 2007 of
the Commission on Audit (COA).

The Facts

On 13 March 1992, Congress approved Republic Act (RA) No. 72273 creating the Bases Conversion
and Development Authority (BCDA). Section 9 of RA No. 7227 states that the BCDA Board of
Directors (Board) shall exercise the powers and functions of the BCDA. Under Section 10, the
functions of the Board include the determination of the organizational structure and the adoption of a
compensation and benefit scheme at least equivalent to that of the Bangko Sentral ng Pilipinas
(BSP). Accordingly, the Board determined the organizational structure of the BCDA and adopted a
compensation and benefit scheme for its officials and employees.

On 20 December 1996, the Board adopted a new compensation and benefit scheme which included
a ₱10,000 year-end benefit granted to each contractual employee, regular permanent employee,
and Board member. In a memorandum4 dated 25 August 1997, Board Chairman Victoriano A. Basco
(Chairman Basco) recommended to President Fidel V. Ramos (President Ramos) the approval of
the new compensation and benefit scheme. In a memorandum 5 dated 9 October 1997, President
Ramos approved the new compensation and benefit scheme.

In 1999, the BSP gave a ₱30,000 year-end benefit to its officials and employees. In 2000, the BSP
increased the year-end benefit from ₱30,000 to ₱35,000. Pursuant to Section 10 of RA No. 7227
which states that the compensation and benefit scheme of the BCDA shall be at least equivalent to
that of the BSP, the Board increased the year-end benefit of BCDA officials and employees from
₱10,000 to ₱30,000. Thus in 2000 and 2001, BCDA officials and employees received a ₱30,000
year-end benefit, and, on 1 October 2002, the Board passed Resolution No. 2002-10-193 6 approving
the release of a ₱30,000 year-end benefit for 2002.

Aside from the contractual employees, regular permanent employees, and Board members, the full-
time consultants of the BCDA also received the year-end benefit.

On 20 February 2003, State Auditor IV Corazon V. Españo of the COA issued Audit Observation
Memorandum (AOM) No. 2003-0047 stating that the grant of year-end benefit to Board members
was contrary to Department of Budget and Management (DBM) Circular Letter No. 2002-2 dated 2
January 2002. In Notice of Disallowance (ND) No. 03-001-BCDA-(02)8 dated 8 January 2004,
Director IV Rogelio D. Tablang (Director Tablang), COA, Legal and Adjudication Office-Corporate,
disallowed the grant of year-end benefit to the Board members and full-time consultants. In Decision
No. 2004-0139 dated 13 January 2004, Director Tablang "concurred" with AOM No. 2003-004 and
ND No. 03-001-BCDA-(02).

In a letter10 dated 20 February 2004, BCDA President and Chief Executive Officer Rufo Colayco
requested the reconsideration of Decision No. 2004-013. In a Resolution11 dated 22 June 2004,
Director Tablang denied the request. The BCDA filed a notice of appeal12 dated 8 September 2004
and an appeal memorandum13 dated 23 December 2004 with the COA.
The COA’s Ruling

In Decision No. 2007-020,14 the COA affirmed the disallowance of the year-end benefit granted to the
Board members and full-time consultants and held that the presumption of good faith did not apply to
them. The COA stated that:

The granting of YEB x x x is not without x x x limitation. DBM Circular Letter No. 2002-02 dated
January 2, 2002 stating, viz:

"2.0 To clarify and address issues/requests concerning the same, the following
compensation policies are hereby reiterated:

2.1 PERA, ADCOM, YEB and retirement benefits, are personnel benefits granted in addition
to salaries. As fringe benefits, these shall be paid only when the basic salary is also paid.

2.2 Members of the Board of Directors of agencies are not salaried officials of the
government. As non-salaried officials they are not entitled to PERA, ADCOM, YEB and
retirement benefits unless expressly provided by law.

2.3 Department Secretaries, Undersecretaries and Assistant Secretaries who serve as Ex-


officio Members of the Board of Directors are not entitled to any remuneration in line with the
Supreme Court ruling that their services in the Board are already paid for and covered by the
remuneration attached to their office." (underscoring ours)

Clearly, as stated above, the members and ex-officio members of the Board of Directors are
not entitled to YEB, they being not salaried officials of the government. The same goes
with full time consultants wherein no employer-employee relationships exist between them and the
BCDA. Thus, the whole amount paid to them totaling ₱342,000 is properly disallowed in audit.

Moreover, the presumption of good faith may not apply to the members and ex-officio members of
the Board of Directors because despite the earlier clarification on the matter by the DBM thru the
issuance on January 2, 2002 of DBM Circular Letter No. 2002-02, still, the BCDA Board of Directors
enacted Resolution No. 2002-10-93 on October 1, 2002 granting YEB to the BCDA personnel
including themselves. Full time consultants, being non-salaried personnel, are also not entitled to
such presumption since they knew from the very beginning that they are only entitled to the amount
stipulated in their contracts as compensation for their services. Hence, they should be made to
refund the disallowed YEB.15 (Boldfacing in the original)

Hence, this petition.

The Court’s Ruling

The Board members and full-time consultants of the BCDA are not entitled to the year-end benefit.

First, the BCDA claims that the Board can grant the year-end benefit to its members and full-time
consultants because, under Section 10 of RA No. 7227, the functions of the Board include the
adoption of a compensation and benefit scheme.

The Court is not impressed. The Board’s power to adopt a compensation and benefit scheme is not
unlimited. Section 9 of RA No. 7227 states that Board members are entitled to a per diem:
Members of the Board shall receive a per diem of not more than Five thousand pesos
(₱5,000) for every board meeting: Provided, however, That the per diem collected per month
does not exceed the equivalent of four (4) meetings: Provided, further, That the amount of per
diem for every board meeting may be increased by the President but such amount shall not be
increased within two (2) years after its last increase. (Emphasis supplied)
1awphi1

Section 9 specifies that Board members shall receive a per diem for every board meeting; limits the
amount of per diem to not more than ₱5,000; and limits the total amount of per diem for one month
to not more than four meetings. In Magno v. Commission on Audit,16 Cabili v. Civil Service
Commission,17 De Jesus v. Civil Service Commission,18Molen, Jr. v. Commission on
Audit,19 and Baybay Water District v. Commission on Audit,20 the Court held that the specification of
compensation and limitation of the amount of compensation in a statute indicate that Board
members are entitled only to the per diem authorized by law and no other. In Baybay Water
District, the Court held that:

By specifying the compensation which a director is entitled to receive and by limiting the amount
he/she is allowed to receive in a month, x x x the law quite clearly indicates that directors x x x are
authorized to receive only the per diem authorized by law and no other compensation or allowance
in whatever form.21

Also, DBM Circular Letter No. 2002-2 states that, "Members of the Board of Directors of agencies
are not salaried officials of the government. As non-salaried officials they are not entitled
to PERA, ADCOM, YEBand retirement benefits unless expressly provided by law." RA No. 7227
does not state that the Board members are entitled to a year-end benefit.

With regard to the full-time consultants, DBM Circular Letter No. 2002-2 states that, "YEB and
retirement benefits, are personnel benefits granted in addition to salaries. As fringe benefits,
these shall be paid only when the basic salary is also paid." The full-time consultants are not
part of the BCDA personnel and are not paid the basic salary. The full-time consultants’ consultancy
contracts expressly state that there is no employer-employee relationship between the BCDA and
the consultants, and that the BCDA shall pay the consultants a contract price. For example, the
consultancy contract22 of a certain Dr. Faith M. Reyes states:

SECTION 2. Contract Price. For and in consideration of the services to be performed by the
CONSULTANT (16 hours/week), BCDA shall pay her the amount of TWENTY THOUSAND PESOS
and 00/100 (₱20,000.00), Philippine currency, per month.

xxxx

SECTION 4. Employee-Employer Relationship. It is understood that no employee-employer


relationship shall exist between BCDA and the CONSULTANT.

SECTION 5. Period of Effectivity. This CONTRACT shall have an effectivity period of one (1) year,
from January 01, 2002 to December 31, 2002, unless sooner terminated by BCDA in accordance
with Section 6 below.

SECTION 6. Termination of Services. BCDA, in its sole discretion may opt to terminate this
CONTRACT when it sees that there is no more need for the services contracted for. (Boldfacing in
the original)
Since full-time consultants are not salaried employees of BCDA, they are not entitled to the year-end
benefit which is a "personnel benefit granted in addition to salaries" and which is "paid only when
the basic salary is also paid."

Second, the BCDA claims that the Board members and full-time consultants should be granted the
year-end benefit because the granting of year-end benefit is consistent with Sections 5 and 18,
Article II of the Constitution. Sections 5 and 18 state:

Section 5. The maintenance of peace and order, the protection of life, liberty, and property, and the
promotion of the general welfare are essential for the enjoyment by all people of the blessings of
democracy.

Section 18. The State affirms labor as a primary social economic force. It shall protect the rights of
workers and promote their welfare.

The Court is not impressed. Article II of the Constitution is entitled Declaration of Principles and
State Policies. By its very title, Article II is a statement of general ideological principles and policies.
It is not a source of enforceable rights.23 In Tondo Medical Center Employees Association v. Court of
Appeals,24 the Court held that Sections 5 and 18, Article II of the Constitution are not self-
executing provisions. In that case, the Court held that "Some of the constitutional provisions
invoked in the present case were taken from Article II of the Constitution — specifically, Sections 5 x
x x and 18 — the provisions of which the Court categorically ruled to be non self-executing."

Third, the BCDA claims that the denial of year-end benefit to the Board members and full-time
consultants violates Section 1, Article III of the Constitution. 25 More specifically, the BCDA claims that
there is no substantial distinction between regular officials and employees on one hand, and Board
members and full-time consultants on the other. The BCDA states that "there is here only a
distinction, but no difference" because both "have undeniably one common goal as humans, that is x
x x ‘to keep body and soul together’" or, "[d]ifferently put, both have mouths to feed and stomachs to
fill."

The Court is not impressed. Every presumption should be indulged in favor of the


constitutionality of RA No. 7227 and the burden of proof is on the BCDA to show that there is
a clear and unequivocal breach of the Constitution.26 In Abakada Guro Party List v.
Purisima,27 the Court held that:

A law enacted by Congress enjoys the strong presumption of constitutionality. To justify its
nullification, there must be a clear and unequivocal breach of the Constitution, not a doubtful and
unequivocal one. To invalidate [a law] based on x x x baseless supposition is an affront to the
wisdom not only of the legislature that passed it but also of the executive which approved it.

The BCDA failed to show that RA No. 7227 unreasonably singled out Board members and full-time
consultants in the grant of the year-end benefit. It did not show any clear and unequivocal breach of
the Constitution. The claim that there is no difference between regular officials and employees, and
Board members and full-time consultants because both groups "have mouths to feed and stomachs
to fill" is fatuous. Surely, persons are not automatically similarly situated — thus, automatically
deserving of equal protection of the laws — just because they both "have mouths to feed and
stomachs to fill." Otherwise, the existence of a substantial distinction would become forever highly
improbable.

Fourth, the BCDA claims that the Board can grant the year-end benefit to its members and the full-
time consultants because RA No. 7227 does not expressly prohibit it from doing so.
The Court is not impressed. A careful reading of Section 9 of RA No. 7227 reveals that the Board is
prohibited from granting its members other benefits. Section 9 states:

Members of the Board shall receive a per diem of not more than Five thousand pesos
(₱5,000) for every board meeting: Provided, however, That the per diem collected per month
does not exceed the equivalent of four (4) meetings: Provided, further, That the amount of per
diem for every board meeting may be increased by the President but such amount shall not be
increased within two (2) years after its last increase. (Emphasis supplied)

Section 9 specifies that Board members shall receive a per diem for every board meeting; limits the
amount of per diem to not more than ₱5,000; limits the total amount of per diem for one month to not
more than four meetings; and does not state that Board members may receive other benefits.
In Magno,28 Cabili,29 De Jesus,30 Molen, Jr.,31 and Baybay Water District,32 the Court held that the
specification of compensation and limitation of the amount of compensation in a statute
indicate that Board members are entitled only to the per diem authorized by law and no other.

The specification that Board members shall receive a per diem of not more than ₱5,000 for every
meeting and the omission of a provision allowing Board members to receive other benefits lead the
Court to the inference that Congress intended to limit the compensation of Board members to
the per diem authorized by law and no other. Expressio unius est exclusio alterius. Had Congress
intended to allow the Board members to receive other benefits, it would have expressly stated
so.33 For example, Congress’ intention to allow Board members to receive other benefits besides
the per diem authorized by law is expressly stated in Section 1 of RA No. 9286:34

SECTION 1. Section 13 of Presidential Decree No. 198, as amended, is hereby amended to read as
follows:

"SEC. 13. Compensation. — Each director shall receive per diem to be determined by the Board, for
each meeting of the Board actually attended by him, but no director shall receive per diems in any
given month in excess of the equivalent of the total per diem of four meetings in any given month.

Any per diem in excess of One hundred fifty pesos (₱150.00) shall be subject to the approval of the
Administration. In addition thereto, each director shall receive allowances and benefits as the
Board may prescribe subject to the approval of the Administration." (Emphasis supplied)

The Court cannot, in the guise of interpretation, enlarge the scope of a statute or insert into a statute
what Congress omitted, whether intentionally or unintentionally.35

When a statute is susceptible of two interpretations, the Court must "adopt the one in consonance
with the presumed intention of the legislature to give its enactments the most reasonable and
beneficial construction, the one that will render them operative and effective." 36 The Court always
presumes that Congress intended to enact sensible statutes.37 If the Court were to rule that the
Board could grant the year-end benefit to its members, Section 9 of RA No. 7227 would become
inoperative and ineffective — the specification that Board members shall receive a per diem of not
more than ₱5,000 for every meeting; the specification that the per diem received per month shall not
exceed the equivalent of four meetings; the vesting of the power to increase the amount of per
diem in the President; and the limitation that the amount of per diem shall not be increased within
two years from its last increase would all become useless because the Board could always grant its
members other benefits.

With regard to the full-time consultants, DBM Circular Letter No. 2002-2 states that, "YEB and
retirement benefits, are personnel benefits granted in addition to salaries. As fringe benefits,
these shall be paid only when the basic salary is also paid." The full-time consultants are not
part of the BCDA personnel and are not paid the basic salary. The full-time consultants’ consultancy
contracts expressly state that there is no employer-employee relationship between BCDA and the
consultants and that BCDA shall pay the consultants a contract price. Since full-time consultants are
not salaried employees of the BCDA, they are not entitled to the year-end benefit which is a
"personnel benefit granted in addition to salaries" and which is "paid only when the basic salary
is also paid."

Fifth, the BCDA claims that the Board members and full-time consultants are entitled to the year-end
benefit because (1) President Ramos approved the granting of the benefit to the Board members,
and (2) they have been receiving it since 1997.

The Court is not impressed. The State is not estopped from correcting a public officer’s erroneous
application of a statute, and an unlawful practice, no matter how long, cannot give rise to any vested
right.38

The Court, however, notes that the Board members and full-time consultants received the year-end
benefit in good faith. The Board members relied on (1) Section 10 of RA No. 7227 which authorized
the Board to adopt a compensation and benefit scheme; (2) the fact that RA No. 7227 does not
expressly prohibit Board members from receiving benefits other than the per diem authorized by law;
and (3) President Ramos’ approval of the new compensation and benefit scheme which included the
granting of a year-end benefit to each contractual employee, regular permanent employee, and
Board member. The full-time consultants relied on Section 10 of RA No. 7227 which authorized the
Board to adopt a compensation and benefit scheme. There is no proof that the Board members and
full-time consultants knew that their receipt of the year-end benefit was unlawful. In keeping
with Magno,39 De Jesus,40 Molen, Jr.,41 and Kapisanan ng mga Manggagawa sa Government Service
Insurance System (KMG) v. Commission on Audit,42 the Board members and full-time consultants
are not required to refund the year-end benefits they have already received.

WHEREFORE, the petition is PARTIALLY GRANTED. Commission on Audit Decision No. 2007-020
dated 12 April 2007 is AFFIRMED with the MODIFICATION that the Board members and full-time
consultants of the Bases Conversion and Development Authority are not required to refund the year-
end benefits they have already received.

SO ORDERED.

G.R. No. 205728               January 21, 2015

THE DIOCESE OF BACOLOD, REPRESENTED BY THE MOST REV. BISHOP VICENTE M.


NAVARRA and THE BISHOP HIMSELF IN HIS PERSONAL CAPACITY, Petitioners, 
vs.
COMMISSION ON ELECTIONS AND THE ELECTION OFFICER OF BACOLOD CITY, ATTY.
MAVIL V. MAJARUCON, Respondents.

DECISION

LEONEN, J.:
"The Philippines is a democratic and republican State. Sovereignty resides in the people and all
government authority emanates from them." – Article II, Section 1, Constitution

All governmental authority emanates from our people. No unreasonable restrictions of the
fundamental and preferred right to expression of the electorate during political contests no matter
how seemingly benign will be tolerated.

This case defines the extent that our people may shape the debates during elections. It is significant
and of first impression. We are asked to decide whether the Commission on Elections (COMELEC)
has the competence to limit expressions made by the citizens — who are not candidates — during
elections.

Before us is a special civil action for certiorari and prohibition with application for preliminary
injunction and temporary restraining order  under Rule 65 of the Rules of Court seeking to nullify
1

COMELEC’s Notice to Remove Campaign Materials  dated February 22, 2013 and letter  issued on
2 3

February 27, 2013.

The facts are not disputed.

On February 21, 2013, petitioners posted two (2) tarpaulins within a private compound housing the
San Sebastian Cathedral of Bacolod. Each tarpaulin was approximately six feet (6') by ten feet (10')
in size. They were posted on the front walls of the cathedral within public view. The first tarpaulin
contains the message "IBASURA RH Law" referring to the Reproductive Health Law of 2012 or
Republic Act No. 10354. The second tarpaulin is the subject of the present case.  This tarpaulin
4

contains the heading "Conscience Vote" and lists candidates as either "(Anti-RH) Team Buhay" with
a check mark, or "(Pro-RH) Team Patay" with an "X" mark.  The electoral candidates were classified
5

according to their vote on the adoption of Republic Act No. 10354, otherwise known as the RH
Law. Those who voted for the passing of the law were classified by petitioners as comprising "Team
6

Patay," while those who voted against it form "Team Buhay": 7

TEAM BUHAY TEAM PATAY


Estrada, JV Angara, Juan Edgardo
Honasan, Gregorio Casiño, Teddy
Magsaysay, Mitos Cayetano, Alan Peter
Pimentel, Koko Enrile, Jackie
Trillanes, Antonio Escudero, Francis
Villar, Cynthia Hontiveros, Risa
Party List Buhay Legarda, Loren
Party List Ang Pamilya Party List Gabriela
  Party List Akbayan
  Party List Bayan Muna
  Party List Anak Pawis

During oral arguments, respondents conceded that the tarpaulin was neither sponsored nor paid for
by any candidate. Petitioners also conceded that the tarpaulin contains names ofcandidates for the
2013 elections, but not of politicians who helped in the passage of the RH Law but were not
candidates for that election.

On February 22, 2013, respondent Atty. Mavil V. Majarucon, in her capacity as Election Officer of
Bacolod City, issued a Notice to Remove Campaign Materials  addressed to petitioner Most Rev.
8

Bishop Vicente M. Navarra. The election officer ordered the tarpaulin’s removal within three (3) days
from receipt for being oversized. COMELEC Resolution No. 9615 provides for the size requirement
of two feet (2’) by three feet (3’).
9

On February 25, 2013, petitioners replied  requesting, among others, that (1) petitioner Bishop be
10

given a definite ruling by COMELEC Law Department regarding the tarpaulin; and (2) pending this
opinion and the availment of legal remedies, the tarpaulin be allowed to remain. 11

On February 27, 2013, COMELEC Law Department issued a letter  ordering the immediate removal
12

of the tarpaulin; otherwise, it will be constrained to file an election offense against petitioners. The
letter of COMELEC Law Department was silenton the remedies available to petitioners. The letter
provides as follows:

Dear Bishop Navarra:

It has reached this Office that our Election Officer for this City, Atty. Mavil Majarucon, had already
given you notice on February 22, 2013 as regards the election propaganda material posted on the
church vicinity promoting for or against the candidates and party-list groups with the following names
and messages, particularly described as follows:

Material size : six feet (6’) by ten feet (10’)

Description : FULL COLOR TARPAULIN

Image of : SEE ATTACHED PICTURES

Message : CONSCIENCE VOTE (ANTI RH) TEAM

BUHAY; (PRO RH) TEAM PATAY

Location : POSTED ON THE CHURCH VICINITY


OF THE DIOCESE OF BACOLOD CITY

The three (3) – day notice expired on February 25, 2013.

Considering that the above-mentioned material is found to be in violation of Comelec Resolution No.
9615 promulgated on January 15, 2013 particularly on the size (even with the subsequent division of
the said tarpaulin into two), as the lawful size for election propaganda material is only two feet (2’) by
three feet (3’), please order/cause the immediate removal of said election propaganda material,
otherwise, we shall be constrained to file an election offense case against you.

We pray that the Catholic Church will be the first institution to help the Commission on Elections
inensuring the conduct of peaceful, orderly, honest and credible elections.

Thank you and God Bless!


[signed]
ATTY. ESMERALDA AMORA-LADRA
Director IV 13

Concerned about the imminent threatof prosecution for their exercise of free speech, petitioners
initiated this case through this petition for certiorari and prohibition with application for preliminary
injunction and temporary restraining order.  They question respondents’ notice dated February 22,
14

2013 and letter issued on February 27, 2013. They pray that: (1) the petition be given due course;
(2) a temporary restraining order (TRO) and/or a writ of preliminary injunction be issued restraining
respondents from further proceeding in enforcing their orders for the removal of the Team Patay
tarpaulin; and (3) after notice and hearing, a decision be rendered declaring the questioned orders of
respondents as unconstitutional and void, and permanently restraining respondents from enforcing
them or any other similar order. 15

After due deliberation, this court, on March 5, 2013, issued a temporary restraining order enjoining
respondents from enforcing the assailed notice and letter, and set oral arguments on March 19,
2013. 16

On March 13, 2013, respondents filed their comment  arguing that (1) a petition for certiorari and
17

prohibition under Rule 65 of the Rules of Court filed before this court is not the proper remedy to
question the notice and letter of respondents; and (2) the tarpaulin is an election propaganda subject
to regulation by COMELEC pursuant to its mandate under Article IX-C, Section 4 of the Constitution.
Hence, respondents claim that the issuances ordering its removal for being oversized are valid and
constitutional. 18

During the hearing held on March 19, 2013, the parties were directed to file their respective
memoranda within 10 days or by April 1, 2013, taking into consideration the intervening holidays. 19

The issues, which also served as guide for the oral arguments, are: 20

I.

WHETHER THE 22 FEBRUARY 2013 NOTICE/ORDER BY ELECTION OFFICER MAJARUCON


AND THE 27 FEBRUARY 2013 ORDER BY THE COMELEC LAW DEPARTMENT ARE
CONSIDERED JUDGMENTS/FINAL ORDERS/RESOLUTIONS OF THE COMELEC WHICH
WOULD WARRANT A REVIEW OF THIS COURT VIA RULE 65 PETITION[;]

A. WHETHER PETITIONERS VIOLATED THE HIERARCHY OF COURTS


DOCTRINE AND JURISPRUDENTIAL RULES GOVERNING APPEALS FROM
COMELEC DECISIONS;

B. ASSUMING ARGUENDO THAT THE AFOREMENTIONED ORDERS ARE NOT


CONSIDERED JUDGMENTS/FINAL ORDERS/RESOLUTIONS OF THE
COMELEC, WHETHER THERE ARE EXCEPTIONAL CIRCUMSTANCES WHICH
WOULD ALLOW THIS COURT TO TAKE COGNIZANCE OF THE CASE[;]

II.

WHETHER IT IS RELEVANT TODETERMINE WHETHER THE TARPAULINS ARE "POLITICAL


ADVERTISEMENT" OR "ELECTION PROPAGANDA" CONSIDERING THAT PETITIONER IS NOT
A POLITICAL CANDIDATE[;]
III.

WHETHER THE TARPAULINS ARE A FORM OR EXPRESSION (PROTECTED SPEECH), OR


ELECTION PROPAGANDA/POLITICAL ADVERTISEMENT[;]

A. ASSUMING ARGUENDO THAT THE TARPAULINS ARE A FORM OF


EXPRESSION, WHETHER THE COMELEC POSSESSES THE AUTHORITY TO
REGULATE THE SAME[;]

B. WHETHER THIS FORM OF EXPRESSION MAY BE REGULATED[;]

IV.

WHETHER THE 22 FEBRUARY 2013 NOTICE/ ORDER BY ELECTION OFFICER MAJARUCON


AND THE 27 FEBRUARY 2013 ORDER BY THE COMELEC LAW DEPARTMENT VIOLATES THE
PRINCIPLE OF SEPARATION OF CHURCH AND STATE[;] [AND]

V.

WHETHER THE ACTION OF THE PETITIONERS IN POSTING ITS TARPAULIN VIOLATES THE
CONSTITUTIONAL PRINCIPLE OF SEPARATION OF CHURCH AND STATE.

I
PROCEDURAL ISSUES

I.A

This court’s jurisdiction over COMELEC cases

Respondents ask that this petition be dismissed on the ground that the notice and letter are not final
orders, decisions, rulings, or judgments of the COMELEC En Banc issued in the exercise of its
adjudicatory powers, reviewable via Rule 64 of the Rules of Court. 21

Rule 64 is not the exclusive remedy for all acts of the COMELEC. Rule 65 is applicable especially to
raise objections relating to a grave abuse of discretion resulting in the ouster of jurisdiction.  As a
22

special civil action, there must also be a showing that there be no plain, speedy, and adequate
remedy in the ordinary course of the law.

Respondents contend that the assailed notice and letter are not subject to review by this court,
whose power to review is "limited only to final decisions, rulings and orders of the COMELEC En
Banc rendered in the exercise of its adjudicatory or quasi-judicial power."  Instead, respondents
23

claim that the assailed notice and letter are reviewable only by COMELEC itself pursuant to Article
IX-C, Section 2(3) of the Constitution  on COMELEC’s power to decide all questions affecting
24

elections.  Respondents invoke the cases of Ambil, Jr. v. COMELEC,  Repol v.


25 26

COMELEC,  Soriano, Jr. v. COMELEC,  Blanco v. COMELEC,  and Cayetano v. COMELEC,  to
27 28 29 30

illustrate how judicialintervention is limited to final decisions, orders, rulings and judgments of the
COMELEC En Banc. 31

These cases are not applicable.


In Ambil, Jr. v. COMELEC, the losing party in the gubernatorial race of Eastern Samar filed the
election protest.  At issue was the validity of the promulgation of a COMELEC Division
32

resolution.  No motion for reconsideration was filed to raise this issue before the COMELEC En
33

Banc. This court declared that it did not have jurisdiction and clarified:

We have interpreted [Section 7, Article IX-A of the Constitution]  to mean final orders, rulings and
34

decisionsof the COMELEC rendered in the exercise of its adjudicatory or quasi-judicial powers." This
decision must be a final decision or resolution of the Comelec en banc, not of a division, certainly not
an interlocutory order of a division.The Supreme Court has no power to review viacertiorari, an
interlocutory order or even a final resolution of a Division of the Commission on
Elections.  (Emphasis in the original, citations omitted)
35

However, in the next case cited by respondents, Repol v. COMELEC, this court provided exceptions
to this general rule. Repolwas another election protest case, involving the mayoralty elections in
Pagsanghan, Samar.  This time, the case was brought to this court because the COMELEC First
36

Division issued a status quo ante order against the Regional Trial Court executing its decision
pending appeal.  This court’s ponencia discussed the general rule enunciated in Ambil, Jr. that it
37

cannot take jurisdiction to review interlocutory orders of a COMELEC Division. However, consistent
38

with ABS-CBN Broadcasting Corporation v. COMELEC,  it clarified the exception:


39

This Court, however, has ruled in the past that this procedural requirement [of filing a motion for
reconsideration] may be glossed over to prevent miscarriage of justice, when the issue involves the
principle of social justice or the protection of labor, when the decision or resolution sought to be set
aside is a nullity, or when the need for relief is extremely urgent and certiorari is the only adequate
and speedy remedy available. 40

Based on ABS-CBN, this court could review orders and decisions of COMELEC — in electoral
contests — despite not being reviewed by the COMELEC En Banc, if:

1) It will prevent the miscarriage of justice;

2) The issue involves a principle of social justice;

3) The issue involves the protection of labor;

4) The decision or resolution sought tobe set aside is a nullity; or

5) The need for relief is extremely urgent and certiorari is the only adequate and speedy
remedy available.

Ultimately, this court took jurisdiction in Repoland decided that the status quo anteorder issued by
the COMELEC Division was unconstitutional.

Respondents also cite Soriano, Jr. v. COMELEC.This case was also an election protest case
involving candidates for the city council of Muntinlupa City.  Petitioners in Soriano, Jr.filed before this
41

court a petition for certiorari against an interlocutory order of the COMELEC First

Division.  While the petition was pending in this court, the COMELEC First Division dismissed the
42

main election protest case.  Sorianoapplied the general rule that only final orders should be
43

questioned with this court. The ponencia for this court, however, acknowledged the exceptions to the
general rule in ABS-CBN. 44
Blanco v. COMELEC, another case cited by respondents, was a disqualification case of one of the
mayoralty candidates of Meycauayan, Bulacan.  The COMELEC Second Division ruled that
45

petitioner could not qualify for the 2007 elections due to the findings in an administrative case that he
engaged in vote buying in the 1995 elections. No motion for reconsideration was filed before the
46

COMELEC En Banc. This court, however, took cognizance of this case applying one of the
exceptions in ABS-CBN: The assailed resolution was a nullity. 47

Finally, respondents cited Cayetano v. COMELEC, a recent election protest case involving the
mayoralty candidates of Taguig City.  Petitioner assailed a resolution of the COMELEC denying her
48

motion for reconsideration to dismiss the election protest petition for lack of form and
substance.  This court clarified the general rule and refused to take cognizance of the review of the
49

COMELEC order. While recognizing the exceptions in ABS-CBN, this court ruled that these
exceptions did not apply. 50

Ambil, Jr., Repol, Soriano, Jr., Blanco, and Cayetano cited by respondents do not operate as
precedents to oust this court from taking jurisdiction over this case. All these cases cited involve
election protests or disqualification cases filed by the losing candidate against the winning candidate.

In the present case, petitioners are not candidates seeking for public office. Their petition is filed to
assert their fundamental right to expression.

Furthermore, all these cases cited by respondents pertained to COMELEC’s exercise of its
adjudicatory or quasi-judicial power. This case pertains to acts of COMELEC in the implementation
of its regulatory powers. When it issued the notice and letter, the COMELEC was allegedly
enforcingelection laws.

I.B

Rule 65, grave abuse of discretion,

and limitations on political speech

The main subject of thiscase is an alleged constitutional violation: the infringement on speech and
the "chilling effect" caused by respondent COMELEC’s notice and letter.

Petitioners allege that respondents committed grave abuse of discretion amounting to lack or excess
of jurisdiction in issuing the notice  dated February 22,2013 and letter  dated February 27, 2013
51 52

ordering the removal of the tarpaulin.  It is their position that these infringe on their fundamental right
53

to freedom of expression and violate the principle of separation of church and state and, thus, are
unconstitutional.54

The jurisdiction of this court over the subject matter is determined from the allegations in the petition.
Subject matter jurisdiction is defined as the authority "to hear and determine cases of the general
class to which the proceedings in question belong and is conferred by the sovereign authority which
organizes the court and defines its powers." Definitely, the subject matter in this case is different
55

from the cases cited by respondents.

Nothing less than the electorate’s political speech will be affected by the restrictions imposed by
COMELEC. Political speech is motivated by the desire to be heard and understood, to move people
to action. It is concerned with the sovereign right to change the contours of power whether through
the election of representatives in a republican government or the revision of the basic text of the
Constitution. The zeal with which we protect this kind of speech does not depend on our evaluation
of the cogency of the message. Neither do we assess whether we should protect speech based on
the motives of COMELEC. We evaluate restrictions on freedom of expression from their effects. We
protect both speech and medium because the quality of this freedom in practice will define the
quality of deliberation in our democratic society.

COMELEC’s notice and letter affect preferred speech. Respondents’ acts are capable of repetition.
Under the conditions in which it was issued and in view of the novelty of this case,it could result in a
"chilling effect" that would affect other citizens who want their voices heard on issues during the
elections. Other citizens who wish to express their views regarding the election and other related
issues may choose not to, for fear of reprisal or sanction by the COMELEC. Direct resort to this court
is allowed to avoid such proscribed conditions. Rule 65 is also the procedural platform for raising
grave abuse of discretion.

Both parties point to constitutional provisions on jurisdiction. For petitioners, it referred to this court’s
expanded exercise of certiorari as provided by the Constitution as follows:

Judicial power includes the duty of the courts of justice to settle actual controversies involving rights
which are legally demandable and enforceable, and to determine whether ornot there has been a
grave abuse of discretion amounting to lack or excess of jurisdiction on the part of any branch or
instrumentality of the Government. (Emphasis supplied)
56

On the other hand, respondents relied on its constitutional mandate to decide all questions
affectingelections. Article IX-C, Section 2(3) of the Constitution, provides:

Sec. 2. The Commission on Elections shall exercise the following powers and functions:

....

(3) Decide, except those involving the right to vote, all questions affecting elections, including
determination of the number and location of polling places, appointment of election officials and
inspectors, and registration of voters.

Respondents’ reliance on this provision is misplaced.

We are not confronted here with the question of whether the COMELEC, in its exercise of
jurisdiction, gravely abused it. We are confronted with the question as to whether the COMELEC had
any jurisdiction at all with its acts threatening imminent criminal action effectively abridging
meaningful political speech.

It is clear that the subject matter of the controversy is the effect of COMELEC’s notice and letter on
free speech. This does not fall under Article IX-C, Section 2(3) of the Constitution. The use of the
word "affecting" in this provision cannot be interpreted to mean that COMELEC has the exclusive
power to decide any and allquestions that arise during elections. COMELEC’s constitutional
competencies during elections should not operate to divest this court of its own jurisdiction.

The more relevant provision for jurisdiction in this case is Article VIII, Section 5(1) of the
Constitution.This provision provides for this court’s original jurisdiction over petitions for certiorari
and prohibition. This should be read alongside the expanded jurisdiction of the court in Article VIII,
Section 1 of the Constitution.
Certainly, a breach of the fundamental right of expression by COMELEC is grave abuse of
discretion. Thus, the constitutionality of the notice and letter coming from COMELEC is within this
court’s power to review.

During elections, we have the power and the duty to correct any grave abuse of discretion or any act
tainted with unconstitutionality on the part of any government branch or instrumentality. This includes
actions by the COMELEC. Furthermore, it is this court’s constitutional mandate to protect the people
against government’s infringement of their fundamental rights. This constitutional mandate out
weighs the jurisdiction vested with the COMELEC.

It will, thus, be manifest injustice if the court does not take jurisdiction over this case.

I.C

Hierarchy of courts

This brings us to the issue of whether petitioners violated the doctrine of hierarchy of courts in
directly filing their petition before this court.

Respondents contend that petitioners’ failure to file the proper suit with a lower court of concurrent
jurisdiction is sufficient ground for the dismissal of their petition.  They add that observation of the
57

hierarchy of courts is compulsory, citing Heirs of Bertuldo Hinog v. Melicor.  While respondents claim
58

that while there are exceptions to the general rule on hierarchy of courts, none of these are present
in this case.59

On the other hand, petitioners cite Fortich v. Corona  on this court’s discretionary power to take
60

cognizance of a petition filed directly to it if warranted by "compelling reasons, or [by] the nature and
importance of the issues raised. . . ."  Petitioners submit that there are "exceptional and compelling
61

reasons to justify a direct resort [with] this Court."


62

In Bañez, Jr. v. Concepcion,  we explained the necessity of the application of the hierarchy of
63

courts:

The Court must enjoin the observance of the policy on the hierarchy of courts, and now affirms that
the policy is not to be ignored without serious consequences. The strictness of the policy is designed
to shield the Court from having to deal with causes that are also well within the competence of the
lower courts, and thus leave time to the Court to deal with the more fundamental and more essential
tasks that the Constitution has assigned to it. The Court may act on petitions for the extraordinary
writs of certiorari, prohibition and mandamus only when absolutely necessary or when serious and
important reasons exist to justify an exception to the policy. 64

In Bañez, we also elaborated on the reasons why lower courts are allowed to issue writs of certiorari,
prohibition, and mandamus, citing Vergara v. Suelto: 65

The Supreme Court is a court of lastresort, and must so remain if it is to satisfactorily perform the
functions assigned to it by the fundamental charter and immemorial tradition. It cannot and should
not be burdened with the task of dealing with causes in the first instance. Its original jurisdiction to
issue the so-called extraordinary writs should be exercised only where absolutely necessary or
where serious and important reasons exist therefore. Hence, that jurisdiction should generally be
exercised relative to actions or proceedings before the Court of Appeals, or before constitutional or
other tribunals, bodies or agencies whose acts for some reason or another are not controllable by
the Court of Appeals. Where the issuance of an extraordinary writ is also within the competence of
the Court of Appeals or a Regional Trial Court, it is in either of these courts that the specific action
for the writ’s procurement must be presented. This is and should continue to be the policy in this
regard, a policy that courts and lawyers must strictly observe.  (Emphasis omitted)
66

The doctrine that requires respect for the hierarchy of courts was created by this court to ensure that
every level of the judiciary performs its designated roles in an effective and efficient manner. Trial
courts do not only determine the facts from the evaluation of the evidence presented before them.
They are likewise competent to determine issues of law which may include the validity of an
ordinance, statute, or even an executive issuance in relation to the Constitution.  To effectively
67

perform these functions, they are territorially organized into regions and then into branches. Their
writs generally reach within those territorial boundaries. Necessarily, they mostly perform the all-
important task of inferring the facts from the evidence as these are physically presented before
them. In many instances, the facts occur within their territorial jurisdiction, which properly present the
‘actual case’ that makes ripe a determination of the constitutionality of such action. The
consequences, of course, would be national in scope. There are, however, some cases where resort
to courts at their level would not be practical considering their decisions could still be appealed
before the higher courts, such as the Court of Appeals.

The Court of Appeals is primarily designed as an appellate court that reviews the determination of
facts and law made by the trial courts. It is collegiate in nature. This nature ensures more
standpoints in the review of the actions of the trial court. But the Court of Appeals also has original
jurisdiction over most special civil actions. Unlike the trial courts, its writs can have a nationwide
scope. It is competent to determine facts and, ideally, should act on constitutional issues thatmay not
necessarily be novel unless there are factual questions to determine.

This court, on the other hand, leads the judiciary by breaking new ground or further reiterating — in
the light of new circumstances or in the light of some confusions of bench or bar — existing
precedents. Rather than a court of first instance or as a repetition of the actions of the Court of
Appeals, this court promulgates these doctrinal devices in order that it truly performs that role.

In other words, the Supreme Court’s role to interpret the Constitution and act in order to protect
constitutional rights when these become exigent should not be emasculated by the doctrine in
respect of the hierarchy of courts. That has never been the purpose of such doctrine.

Thus, the doctrine of hierarchy of courts is not an iron-clad rule.  This court has "full discretionary
68

power to take cognizance and assume jurisdiction [over] special civil actions for certiorari . . .filed
directly with it for exceptionally compelling reasons  or if warranted by the nature of the issues
69

clearly and specifically raised in the petition."  As correctly pointed out by petitioners,  we have
70 71

provided exceptions to this doctrine:

First, a direct resort to this court is allowed when there are genuine issues of constitutionality that
must be addressed at the most immediate time. A direct resort to this court includes availing of the
remedies of certiorari and prohibition toassail the constitutionality of actions of both legislative and
executive branches of the government. 72

In this case, the assailed issuances of respondents prejudice not only petitioners’ right to freedom of
expression in the present case, but also of others in future similar cases. The case before this court
involves an active effort on the part of the electorate to reform the political landscape. This has
become a rare occasion when private citizens actively engage the public in political discourse. To
quote an eminent political theorist:
[T]he theory of freedom of expression involves more than a technique for arriving at better social
judgments through democratic procedures. It comprehends a vision of society, a faith and a whole
way of life. The theory grew out of an age that was awakened and invigorated by the idea of new
society in which man's mind was free, his fate determined by his own powers of reason, and his
prospects of creating a rational and enlightened civilization virtually unlimited. It is put forward as a
prescription for attaining a creative, progressive, exciting and intellectually robust community. It
contemplates a mode of life that, through encouraging toleration, skepticism, reason and initiative,
will allow man to realize his full potentialities.It spurns the alternative of a society that is tyrannical,
conformist, irrational and stagnant. 73

In a democracy, the citizen’s right tofreely participate in the exchange of ideas in furtherance of
political decision-making is recognized. It deserves the highest protection the courts may provide, as
public participation in nation-building isa fundamental principle in our Constitution. As such, their
right to engage in free expression of ideas must be given immediate protection by this court.

A second exception is when the issuesinvolved are of transcendental importance.  In these cases,
74

the imminence and clarity of the threat to fundamental constitutional rights outweigh the necessity for
prudence. The doctrine relating to constitutional issues of transcendental importance prevents courts
from the paralysis of procedural niceties when clearly faced with the need for substantial protection.

In the case before this court, there is a clear threat to the paramount right of freedom of speech and
freedom of expression which warrants invocation of relief from this court. The principles laid down in
this decision will likely influence the discourse of freedom of speech in the future, especially in the
context of elections. The right to suffrage not only includes the right to vote for one’s chosen
candidate, but also the right to vocalize that choice to the public in general, in the hope of influencing
their votes. It may be said that in an election year, the right to vote necessarily includes the right to
free speech and expression. The protection of these fundamental constitutional rights, therefore,
allows for the immediate resort to this court.

Third, cases of first impression  warrant a direct resort to this court. In cases of first impression, no
75

jurisprudence yet exists that will guide the lower courts on this matter. In Government of the United
States v. Purganan,  this court took cognizance of the case as a matter of first impression that may
76

guide the lower courts:

In the interest of justice and to settle once and for all the important issue of bail in extradition
proceedings, we deem it best to take cognizance of the present case. Such proceedings constitute a
matter of first impression over which there is, as yet, no local jurisprudence to guide lower courts. 77

This court finds that this is indeed a case of first impression involving as it does the issue of whether
the right of suffrage includes the right of freedom of expression. This is a question which this court
has yet to provide substantial answers to, through jurisprudence. Thus, direct resort to this court is
allowed.

Fourth, the constitutional issues raisedare better decided by this court. In Drilon v. Lim,  this court
78

held that:

. . . it will be prudent for such courts, if only out of a becoming modesty, to defer to the higher
judgmentof this Court in the consideration of its validity, which is better determined after a thorough
deliberation by a collegiate body and with the concurrence of the majority of those who participated
in its discussion.  (Citation omitted)
79
In this case, it is this court, with its constitutionally enshrined judicial power, that can rule with finality
on whether COMELEC committed grave abuse of discretion or performed acts contrary to the
Constitution through the assailed issuances.

Fifth, the time element presented in this case cannot be ignored. This case was filed during the 2013
election period. Although the elections have already been concluded, future cases may be filed that
necessitate urgency in its resolution. Exigency in certain situations would qualify as an exception for
direct resort to this court.

Sixth, the filed petition reviews the act of a constitutional organ. COMELEC is a constitutional body.
In Albano v. Arranz,  cited by petitioners, this court held that "[i]t is easy to realize the chaos that
80

would ensue if the Court of First Instance ofeach and every province were [to] arrogate itself the
power to disregard, suspend, or contradict any order of the Commission on Elections: that
constitutional body would be speedily reduced to impotence." 81

In this case, if petitioners sought to annul the actions of COMELEC through pursuing remedies with
the lower courts, any ruling on their part would not have been binding for other citizens whom
respondents may place in the same situation. Besides, thiscourt affords great respect to the
Constitution and the powers and duties imposed upon COMELEC. Hence, a ruling by this court
would be in the best interest of respondents, in order that their actions may be guided accordingly in
the future.

Seventh, petitioners rightly claim that they had no other plain, speedy, and adequate remedy in the
ordinary course of law that could free them from the injurious effects of respondents’ acts in violation
of their right to freedom of expression.

In this case, the repercussions of the assailed issuances on this basic right constitute an
exceptionally compelling reason to justify the direct resort to this court. The lack of other sufficient
remedies in the course of law alone is sufficient ground to allow direct resort to this court.

Eighth, the petition includes questionsthat are "dictated by public welfare and the advancement of
public policy, or demanded by the broader interest of justice, or the orders complained of were found
to be patent nullities, or the appeal was consideredas clearly an inappropriate remedy."  In the past,
82

questions similar to these which this court ruled on immediately despite the doctrine of hierarchy of
courts included citizens’ right to bear arms,  government contracts involving modernization of voters’
83

registration lists,  and the status and existence of a public office.


84 85

This case also poses a question of similar, if not greater import. Hence, a direct action to this court is
permitted.

It is not, however, necessary that all of these exceptions must occur at the same time to justify a
direct resort to this court. While generally, the hierarchy of courts is respected, the present case falls
under the recognized exceptions and, as such, may be resolved by this court directly.

I.D

The concept of a political question

Respondents argue further that the size limitation and its reasonableness is a political question,
hence not within the ambit of this court’s power of review. They cite Justice Vitug’s separate opinion
in Osmeña v. COMELEC  to support their position:
86
It might be worth mentioning that Section 26, Article II, of the Constitution also states that the "State
shall guarantee equal access to opportunities for public service, and prohibit political dynasties as
may be defined by law." I see neither Article IX (C)(4) nor Section 26, Article II, of the Constitution to
be all that adversarial or irreconcilably inconsistent with the right of free expression. In any event, the
latter, being one of general application, must yield to the specific demands of the Constitution. The
freedom of expression concededly holds, it is true, a vantage point in hierarchy of constitutionally-
enshrined rights but, like all fundamental rights, it is not without limitations.

The case is not about a fight between the "rich" and the "poor" or between the "powerful" and the
"weak" in our society but it is to me a genuine attempt on the part of Congress and the Commission
on Elections to ensure that all candidates are given an equal chance to media coverage and thereby
be equally perceived as giving real life to the candidates’ right of free expression rather than being
viewed as an undue restriction of that freedom. The wisdom in the enactment of the law, i.e., that
which the legislature deems to be best in giving life to the Constitutional mandate, is not for the
Court to question; it is a matter that lies beyond the normal prerogatives of the Court to pass upon. 87

This separate opinion is cogent for the purpose it was said. But it is not in point in this case.

The present petition does not involve a dispute between the rich and poor, or the powerful and weak,
on their equal opportunities for media coverage of candidates and their right to freedom of
expression. This case concerns the right of petitioners, who are non-candidates, to post the tarpaulin
in their private property, asan exercise of their right of free expression. Despite the invocation of the
political question doctrine by respondents, this court is not proscribed from deciding on the merits of
this case.

In Tañada v. Cuenco,  this court previously elaborated on the concept of what constitutes a political
88

question:

What is generally meant, when it is said that a question is political, and not judicial, is that it is a
matter which is to be exercised by the people in their primary political capacity, or that it has been
specifically delegated to some other department or particular officer of the government,
withdiscretionary power to act.  (Emphasis omitted)
89

It is not for this court to rehearse and re-enact political debates on what the text of the law should be.
In political forums, particularly the legislature, the creation of the textof the law is based on a general
discussion of factual circumstances, broadly construed in order to allow for general application by
the executive branch. Thus, the creation of the law is not limited by particular and specific facts that
affect the rights of certain individuals, per se.

Courts, on the other hand, rule on adversarial positions based on existing facts established on a
specific case-to-case basis, where parties affected by the legal provision seek the courts’
understanding of the law.

The complementary nature of the political and judicial branches of government is essential in order
to ensure that the rights of the general public are upheld at all times. In order to preserve this
balance, branches of government must afford due respectand deference for the duties and functions
constitutionally delegated to the other. Courts cannot rush to invalidate a law or rule. Prudence
dictates that we are careful not to veto political acts unless we can craft doctrine narrowly tailored to
the circumstances of the case.
The case before this court does not call for the exercise of prudence or modesty. There is no political
question. It can be acted upon by this court through the expanded jurisdiction granted to this court
through Article VIII, Section 1 of the Constitution.

A political question arises in constitutional issues relating to the powers or competence of different
agencies and departments of the executive or those of the legislature. The political question doctrine
is used as a defense when the petition asks this court to nullify certain acts that are exclusively
within the domain of their respective competencies, as provided by the Constitution or the law. In
such situation, presumptively, this court should act with deference. It will decline to void an act
unless the exercise of that power was so capricious and arbitrary so as to amount to grave abuse of
discretion.

The concept of a political question, however, never precludes judicial review when the act of a
constitutional organ infringes upon a fundamental individual or collective right. Even assuming
arguendo that the COMELEC did have the discretion to choose the manner of regulation of the
tarpaulin in question, it cannot do so by abridging the fundamental right to expression.

Marcos v. Manglapus  limited the use of the political question doctrine:


90

When political questions are involved, the Constitution limits the determination to whether or not
there has been a grave abuse of discretion amounting to lack or excess of jurisdiction on the part of
the official whose action is being questioned. If grave abuse is not established, the Court will not
substitute its judgment for that of the official concerned and decide a matter which by its nature or by
law is for the latter alone to decide.
91

How this court has chosen to address the political question doctrine has undergone an evolution
since the timethat it had been first invoked in Marcos v. Manglapus. Increasingly, this court has
taken the historical and social context of the case and the relevance of pronouncements of carefully
and narrowly tailored constitutional doctrines. This trend was followed in cases such as Daza v.
Singson  and Coseteng v. Mitra Jr.
92 93

Daza and Coseteng involved a question as to the application of Article VI, Section 18 of the 1987
Constitution involving the removal of petitioners from the Commission on Appointments. In times
past, this would have involved a quint essentially political question as it related to the dominance of
political parties in Congress. However, in these cases, this court exercised its power of judicial
review noting that the requirement of interpreting the constitutional provision involved the legality and
not the wisdom of a manner by which a constitutional duty or power was exercised. This approach
was again reiterated in Defensor Santiago v. Guingona, Jr. 94

In Integrated Bar of the Philippines v. Zamora,  this court declared again that the possible existence
95

ofa political question did not bar an examination of whether the exercise of discretion was done with
grave abuse of discretion. In that case, this court ruled on the question of whether there was grave
abuse of discretion in the President’s use of his power to call out the armed forces to prevent and
suppress lawless violence.

In Estrada v. Desierto,  this court ruled that the legal question as to whether a former President
96

resigned was not a political question even if the consequences would be to ascertain the political
legitimacy of a successor President.

Many constitutional cases arise from political crises. The actors in such crises may use the
resolution of constitutional issues as leverage. But the expanded jurisdiction of this court now
mandates a duty for it to exercise its power of judicial review expanding on principles that may avert
catastrophe or resolve social conflict.

This court’s understanding of the political question has not been static or unbending. In Llamas v.
Executive Secretary Oscar Orbos,  this court held:
97

While it is true that courts cannot inquire into the manner in which the President's discretionary
powers are exercised or into the wisdom for its exercise, it is also a settled rule that when the issue
involved concerns the validity of such discretionary powers or whether said powers are within the
limits prescribed by the Constitution, We will not decline to exercise our power of judicial review. And
such review does not constitute a modification or correction of the act of the President, nor does it
constitute interference with the functions of the President.
98

The concept of judicial power in relation to the concept of the political question was discussed most
extensively in Francisco v. HRET.  In this case, the House of Representatives arguedthat the
99

question of the validity of the second impeachment complaint that was filed against former Chief
Justice Hilario Davide was a political question beyond the ambit of this court. Former Chief Justice
Reynato Puno elaborated on this concept in his concurring and dissenting opinion:

To be sure, the force to impugn the jurisdiction of this Court becomes more feeble in light of the new
Constitution which expanded the definition of judicial power as including "the duty of the courts of
justice to settle actual controversies involving rights which are legally demandable and enforceable,
and to determine whether or not there has been a grave abuse of discretion amounting to lack or
excess of jurisdiction on the part of any branch or instrumentality of the Government." As well
observed by retired Justice Isagani Cruz, this expanded definition of judicial power considerably
constricted the scope of political question. He opined that the language luminously suggests that this
duty (and power) is available even against the executive and legislative departments including the
President and the Congress, in the exercise of their discretionary powers.  (Emphasis in the
100

original, citations omitted)

Francisco also provides the cases which show the evolution of the political question, as applied in
the following cases:

In Marcos v. Manglapus, this Court, speaking through Madame Justice Irene Cortes, held: The
present Constitution limits resort to the political question doctrine and broadens the scope of judicial
inquiry into areas which the Court,under previous constitutions, would have normally left to the
political departments to decide. x x x

In Bengzon v. Senate Blue Ribbon Committee, through Justice Teodoro Padilla, this Court declared:

The "allocation of constitutional boundaries" is a task that this Court must perform under the
Constitution. Moreover, as held in a recent case, "(t)he political question doctrine neither interposes
an obstacle to judicial determination of the rival claims. The jurisdiction to delimit constitutional
boundaries has been given to this Court. It cannot abdicate that obligation mandated by the 1987
Constitution, although said provision by no means does away with the applicability of the principle in
appropriate cases." (Emphasis and italics supplied)

And in Daza v. Singson, speaking through Justice Isagani Cruz, this Court ruled:

In the case now before us, the jurisdictional objection becomes even less tenable and decisive. The
reason is that, even if we were to assume that the issue presented before us was political in nature,
we would still not be precluded from resolving it under the expanded jurisdiction conferred upon us
that now covers, in proper cases, even the political question.x x x (Emphasis and italics supplied.)

....

In our jurisdiction, the determination of whether an issue involves a truly political and non-justiciable
question lies in the answer to the question of whether there are constitutionally imposed limits on
powers or functions conferred upon political bodies. If there are, then our courts are duty-bound to
examine whether the branch or instrumentality of the government properly acted within such
limits.  (Citations omitted)
101

As stated in Francisco, a political question will not be considered justiciable if there are no
constitutionally imposed limits on powers or functions conferred upon political bodies. Hence, the
existence of constitutionally imposed limits justifies subjecting the official actions of the body to the
scrutiny and review of this court.

In this case, the Bill of Rights gives the utmost deference to the right to free speech. Any instance
that this right may be abridged demands judicial scrutiny. It does not fall squarely into any doubt that
a political question brings.

I.E

Exhaustion of administrative remedies

Respondents allege that petitioners violated the principle of exhaustion of administrative remedies.
Respondents insist that petitioners should have first brought the matter to the COMELEC En Banc or
any of its divisions.
102

Respondents point out that petitioners failed to comply with the requirement in Rule 65 that "there is
no appeal, or any plain, speedy, and adequate remedy in the ordinary course of law."  They add
103

that the proper venue to assail the validity of the assailed issuances was in the course of an
administrative hearing to be conducted by COMELEC.  In the event that an election offense is filed
104

against petitioners for posting the tarpaulin, they claim that petitioners should resort to the remedies
prescribed in Rule 34 of the COMELEC Rules of Procedure. 105

The argument on exhaustion of administrative remedies is not proper in this case.

Despite the alleged non-exhaustion of administrative remedies, it is clear that the controversy is
already ripe for adjudication. Ripeness is the "prerequisite that something had by then been
accomplished or performed by either branch [or in this case, organ of government] before a court
may come into the picture." 106

Petitioners’ exercise of their rightto speech, given the message and their medium, had
understandable relevance especially during the elections. COMELEC’s letter threatening the filing of
the election offense against petitioners is already an actionable infringement of this right. The
impending threat of criminal litigation is enough to curtail petitioners’ speech.

In the context of this case, exhaustion of their administrative remedies as COMELEC suggested in
their pleadings prolongs the violation of their freedom of speech.
Political speech enjoys preferred protection within our constitutional order. In Chavez v.
Gonzales,  Justice Carpio in a separate opinion emphasized: "[i]f everthere is a hierarchy of
107

protected expressions, political expression would occupy the highest rank, and among different
kinds of political expression, the subject of fair and honest elections would be at the
top."  Sovereignty resides in the people.  Political speech is a direct exercise of the sovereignty.
108 109

The principle of exhaustion of administrative remedies yields in order to protect this fundamental
right.

Even assuming that the principle of exhaustion of administrative remedies is applicable, the current
controversy is within the exceptions to the principle. In Chua v. Ang,  this court held:
110

On the other hand, prior exhaustion of administrative remedies may be dispensed with and judicial
action may be validly resorted to immediately: (a) when there is a violation of due process; (b) when
the issue involved is purely a legal question; (c) when the administrative action is patently illegal
amounting to lack or excess of jurisdiction; (d) when there is estoppel on the part ofthe
administrative agency concerned; (e) when there is irreparable injury; (f) when the respondent is a
department secretary whose acts as analter ego of the President bear the implied and assumed
approval of the latter; (g) when to require exhaustion of administrative remedies would be
unreasonable; (h) when it would amount to a nullification of a claim; (i) when the subject matter is a
private land in land case proceedings; (j) whenthe rule does not provide a plain, speedy and
adequate remedy; or (k) when there are circumstances indicating the urgency of judicial
intervention."  (Emphasis supplied, citation omitted)
111

The circumstances emphasized are squarely applicable with the present case. First, petitioners
allegethat the assailed issuances violated their right to freedom of expression and the principle of
separation of church and state. This is a purely legal question. Second, the circumstances of the
present case indicate the urgency of judicial intervention considering the issue then on the RH Law
as well as the upcoming elections. Thus, to require the exhaustion of administrative remedies in this
case would be unreasonable.

Time and again, we have held that this court "has the power to relax or suspend the rules or to
except a case from their operation when compelling reasons so warrant, or whenthe purpose of
justice requires it, [and when] [w]hat constitutes [as] good and sufficient cause that will merit
suspension of the rules is discretionary upon the court". Certainly, this case of first impression
112

where COMELEC has threatenedto prosecute private parties who seek to participate in the elections
by calling attention to issues they want debated by the publicin the manner they feel would be
effective is one of those cases.

II
SUBSTANTIVE ISSUES

II.A

COMELEC had no legal basis to regulate expressions made by private citizens

Respondents cite the Constitution, laws, and jurisprudence to support their position that they had the
power to regulate the tarpaulin.  However, all of these provisions pertain to candidates and political
113

parties. Petitioners are not candidates. Neither do theybelong to any political party. COMELEC does
not have the authority to regulate the enjoyment of the preferred right to freedom of expression
exercised by a non-candidate in this case.

II.A.1
First, respondents cite Article IX-C, Section 4 of the Constitution, which provides:

Section 4. The Commission may,during the election period, supervise or regulate the enjoyment or
utilization of all franchises or permits for the operation of transportation and other public utilities,
media of communication or information, all grants, special privileges, or concessions granted by the
Government or any subdivision, agency, or instrumentality thereof, including any government-owned
or controlled corporation or its subsidiary. Such supervision or regulation shall aim to ensure equal
opportunity, time, and space, and the right to reply, including reasonable, equal rates therefor, for
public information campaigns and forums among candidates in connection with the objective of
holding free, orderly, honest, peaceful, and credible elections.  (Emphasis supplied)
114

Sanidad v. COMELEC  involved the rules promulgated by COMELEC during the plebiscite for the
115

creation of the Cordillera Autonomous Region.  Columnist Pablito V. Sanidad questioned the
116

provision prohibiting journalists from covering plebiscite issues on the day before and on plebiscite
day.  Sanidad argued that the prohibition was a violation of the "constitutional guarantees of the
117

freedom of expression and of the press. . . ."  We held that the "evil sought to be prevented by this
118

provision is the possibility that a franchise holder may favor or give any undue advantage to a
candidate in terms of advertising space or radio or television time."  This court found that "[m]edia
119

practitioners exercising their freedom of expression during plebiscite periods are neither the
franchise holders nor the candidates[,]"  thus, their right to expression during this period may not be
120

regulated by COMELEC. 121

Similar to the media, petitioners in the case at bar are neither franchise holders nor candidates.
II.A.2

Respondents likewise cite Article IX-C, Section 2(7) of the Constitution as follows: 122

Sec. 2. The Commission on Elections shall exercise the following powers and functions:

....

(7) Recommend to the Congress effective measures to minimize election spending, including
limitation of places where propaganda materials shall be posted, and to prevent and penalize all
forms of election frauds, offenses, malpractices, and nuisance candidates. (Emphasis supplied)
Based on the enumeration made on actsthat may be penalized, it will be inferred that this provision
only affects candidates.

Petitioners assail the "Notice to Remove Campaign Materials" issued by COMELEC. This was
followed bythe assailed letter regarding the "election propaganda material posted on the church
vicinity promoting for or against the candidates and party-list groups. . . ."
123

Section 9 of the Fair Election Act  on the posting of campaign materials only mentions "parties" and
124

"candidates":

Sec. 9. Posting of Campaign Materials. - The COMELEC may authorize political parties and party-
list groups to erect common poster areas for their candidates in not more than ten (10) public places
such as plazas, markets, barangay centers and the like, wherein candidates can post, display or
exhibit election propaganda: Provided, That the size ofthe poster areas shall not exceed twelve (12)
by sixteen (16) feet or its equivalent. Independent candidates with no political parties may likewise
be authorized to erect common poster areas in not more than ten (10) public places, the size of
which shall not exceed four (4) by six (6) feet or its equivalent. Candidates may post any lawful
propaganda material in private places with the consent of the owner thereof, and in public places or
property which shall be allocated equitably and impartially among the candidates. (Emphasis
supplied)

Similarly, Section 17 of COMELEC Resolution No. 9615, the rules and regulations implementing the
Fair Election Act, provides as follows:

SECTION 17. Posting of Campaign Materials. - Parties and candidates may post any lawful
campaign material in:

a. Authorized common poster areasin public places subject to the requirements and/or
limitations set forth in the next following section; and

b. Private places provided it has the consent of the owner thereof.

The posting of campaign materials in public places outside of the designated common poster areas
and those enumerated under Section 7 (g) of these Rules and the like is prohibited. Persons posting
the same shall be liable together with the candidates and other persons who caused the posting. It
will be presumed that the candidates and parties caused the posting of campaign materials outside
the common poster areas if they do not remove the same within three (3) days from notice which
shall be issued by the Election Officer of the city or municipality where the unlawful election
propaganda are posted or displayed.

Members of the PNP and other law enforcement agencies called upon by the Election Officeror
other officials of the COMELEC shall apprehend the violators caught in the act, and file the
appropriate charges against them. (Emphasis supplied)

Respondents considered the tarpaulin as a campaign material in their issuances. The above
provisions regulating the posting of campaign materials only apply to candidates and political parties,
and petitioners are neither of the two.

Section 3 of Republic Act No. 9006on "Lawful Election Propaganda" also states that these are
"allowed for all registered political parties, national, regional, sectoral parties or organizations
participating under the party-list elections and for all bona fide candidates seeking national and local
elective positions subject to the limitation on authorized expenses of candidates and political parties.
. . ." Section 6 of COMELEC Resolution No. 9615 provides for a similar wording. These provisions
show that election propaganda refers to matter done by or on behalf of and in coordination with
candidates and political parties. Some level of coordination with the candidates and political parties
for whom the election propaganda are released would ensure that these candidates and political
parties maintain within the authorized expenses limitation.

The tarpaulin was not paid for byany candidate or political party.  There was no allegation that
125

petitioners coordinated with any of the persons named in the tarpaulin regarding its posting. On the
other hand, petitioners posted the tarpaulin as part of their advocacy against the RH Law.
Respondents also cite National Press Club v. COMELEC  in arguing that its regulatory power under
126

the Constitution, to some extent, set a limit on the right to free speech during election period.
127

National Press Club involved the prohibition on the sale and donation of space and time for political
advertisements, limiting political advertisements to COMELEC-designated space and time. This
case was brought by representatives of mass media and two candidates for office in the 1992
elections. They argued that the prohibition on the sale and donation of space and time for political
advertisements is tantamount to censorship, which necessarily infringes on the freedom of speech of
the candidates. 128

This court upheld the constitutionality of the COMELEC prohibition in National Press Club. However,
this case does not apply as most of the petitioners were electoral candidates, unlike petitioners in
the instant case. Moreover, the subject matter of National Press Club, Section 11(b) of Republic Act
No. 6646,  only refers to a particular kind of media such as newspapers, radio broadcasting, or
129

television.  Justice Feliciano emphasized that the provision did not infringe upon the right of
130

reporters or broadcasters to air their commentaries and opinions regarding the candidates, their
qualifications, and program for government. Compared to Sanidadwherein the columnists lost their
ability to give their commentary on the issues involving the plebiscite, National Press Clubdoes not
involve the same infringement.

In the case at bar, petitioners lost their ability to give a commentary on the candidates for the 2013
national elections because of the COMELEC notice and letter. It was not merelya regulation on the
campaigns of candidates vying for public office. Thus, National Press Clubdoes not apply to this
case.

Finally, Section 79 of Batas Pambansa Blg. 881, otherwise known as the Omnibus Election Code,
defines an"election campaign" as follows:

....

(b) The term "election campaign" or "partisan political activity" refers to an act designed to promote
the election or defeat of a particular candidate or candidates to a public office which shall include:

(1) Forming organizations, associations, clubs, committees or other groups of persons for the
purpose of soliciting votes and/or undertaking any campaign for or against a candidate;

(2) Holding political caucuses, conferences, meetings, rallies, parades, or other similar
assemblies, for the purpose of soliciting votes and/or undertaking any campaign or
propaganda for or against a candidate;

(3) Making speeches, announcements or commentaries, or holding interviews for or against


the election of any candidate for public office;

(4) Publishing or distributing campaign literature or materials designed to support or oppose


the election of any candidate; or

(5) Directly or indirectly soliciting votes, pledges or support for or against a candidate.

The foregoing enumerated acts ifperformed for the purpose of enhancing the chances of aspirants
for nomination for candidacy to a public office by a political party, aggroupment, or coalition of
parties shall not be considered as election campaign or partisan election activity. Public expressions
or opinions or discussions of probable issues in a forthcoming electionor on attributes of or criticisms
against probable candidates proposed to be nominated in a forthcoming political party convention
shall not be construed as part of any election campaign or partisan political activity contemplated
under this Article. (Emphasis supplied)
True, there is no mention whether election campaign is limited only to the candidates and political
parties themselves. The focus of the definition is that the act must be "designed to promote the
election or defeat of a particular candidate or candidates to a public office."

In this case, the tarpaulin contains speech on a matter of public concern, that is, a statement of
either appreciation or criticism on votes made in the passing of the RH law. Thus, petitioners invoke
their right to freedom of expression.

II.B

The violation of the constitutional right

to freedom of speech and expression

Petitioners contend that the assailed notice and letter for the removal of the tarpaulin violate their
fundamental right to freedom of expression.

On the other hand, respondents contend that the tarpaulin is an election propaganda subject to their
regulation pursuant to their mandate under Article IX-C, Section 4 of the Constitution. Thus, the
assailed notice and letter ordering itsremoval for being oversized are valid and constitutional.131

II.B.1

Fundamental to the consideration of this issue is Article III, Section 4 of the Constitution:

Section 4. No law shall be passed abridging the freedom of speech, of expression, or of the press, or
the right of the people peaceably to assemble and petition the government for redress of
grievances. 132

No law. . .

While it is true that the present petition assails not a law but an opinion by the COMELEC Law
Department, this court has applied Article III, Section 4 of the Constitution even to governmental
acts.

In Primicias v. Fugoso,  respondent Mayor applied by analogy Section 1119 of the Revised
133

Ordinances of 1927 of Manila for the public meeting and assembly organized by petitioner
Primicias.  Section 1119 requires a Mayor’s permit for the use of streets and public places for
134

purposes such as athletic games, sports, or celebration of national holidays.  What was questioned
135

was not a law but the Mayor’s refusal to issue a permit for the holding of petitioner’s public
meeting.  Nevertheless, this court recognized the constitutional right to freedom of speech, to
136

peaceful assembly and to petition for redress of grievances, albeit not absolute,  and the petition for
137

mandamus to compel respondent Mayor to issue the permit was granted. 138

In ABS-CBN v. COMELEC, what was assailed was not a law but COMELEC En Banc Resolution
No. 98-1419 where the COMELEC resolved to approve the issuance of a restraining order to stop
ABS-CBN from conducting exit surveys.  The right to freedom of expression was similarly upheld in
139

this case and, consequently, the assailed resolution was nullified and set aside. 140

. . . shall be passed abridging. . .


All regulations will have an impact directly or indirectly on expression. The prohibition against the
abridgment of speech should not mean an absolute prohibition against regulation. The primary and
incidental burden on speech must be weighed against a compelling state interest clearly allowed in
the Constitution. The test depends on the relevant theory of speech implicit in the kind of society
framed by our Constitution.

. . . of expression. . .

Our Constitution has also explicitly included the freedom of expression, separate and in addition to
the freedom of speech and of the press provided in the US Constitution. The word "expression" was
added in the 1987 Constitution by Commissioner Brocka for having a wider scope:

MR. BROCKA: This is a very minor amendment, Mr. Presiding Officer. On Section 9, page 2, line 29,
it says: "No law shall be passed abridging the freedom of speech." I would like to recommend to the
Committee the change of the word "speech" to EXPRESSION; or if not, add the words AND
EXPRESSION after the word "speech," because it is more expansive, it has a wider scope, and it
would refer to means of expression other than speech.

THE PRESIDING OFFICER (Mr.Bengzon): What does the Committee say?

FR. BERNAS: "Expression" is more broad than speech. We accept it.

MR. BROCKA: Thank you.

THE PRESIDING OFFICER (Mr.Bengzon): Is it accepted?

FR. BERNAS: Yes.

THE PRESIDING OFFICER (Mr.Bengzon): Is there any objection? (Silence) The Chair hears none;
the amendment is approved.

FR. BERNAS: So, that provision will now read: "No law shall be passed abridging the freedom of
speech, expression or of the press . . . ."  Speech may be said to be inextricably linked to freedom
141

itself as "[t]he right to think is the beginning of freedom, and speech must be protected from the
government because speech is the beginning of thought." 142

II.B.2

Communication is an essential outcome of protected speech.  Communication exists when "(1) a


143

speaker, seeking to signal others, uses conventional actions because he orshe reasonably believes
that such actions will be taken by the audience in the manner intended; and (2) the audience so
takes the actions."  "[I]n communicative action[,] the hearer may respond to the claims by . . . either
144

accepting the speech act’s claims or opposing them with criticism or requests for justification."
145

Speech is not limited to vocal communication. "[C]onduct is treated as a form of speech sometimes
referred to as ‘symbolic speech[,]’"  such that "‘when ‘speech’ and ‘nonspeech’ elements are
146

combined in the same course of conduct,’ the ‘communicative element’ of the conduct may be
‘sufficient to bring into play the [right to freedom of expression].’"
147

The right to freedom of expression, thus, applies to the entire continuum of speech from utterances
made to conduct enacted, and even to inaction itself as a symbolic manner of communication.
In Ebralinag v. The Division Superintendent of Schools of Cebu,  students who were members of
148

the religious sect Jehovah’s Witnesses were to be expelled from school for refusing to salute the
flag, sing the national anthem, and recite the patriotic pledge.  In his concurring opinion, Justice
149

Cruz discussed how the salute is a symbolic manner of communication and a valid form of
expression.  He adds that freedom of speech includes even the right to be silent:
150

Freedom of speech includes the right to be silent. Aptly has it been said that the Bill of Rights that
guarantees to the individual the liberty to utter what is in his mind also guarantees to him the liberty
not to utter what is not in his mind. The salute is a symbolic manner of communication that conveys
its messageas clearly as the written or spoken word. As a valid form of expression, it cannot be
compelled any more than it can be prohibited in the face of valid religious objections like those
raised in this petition. To impose it on the petitioners is to deny them the right not to speak when
their religion bids them to be silent. This coercion of conscience has no place in the free society.

The democratic system provides for the accommodation of diverse ideas, including the
unconventional and even the bizarre or eccentric. The will of the majority prevails, but it cannot
regiment thought by prescribing the recitation by rote of its opinions or proscribing the assertion of
unorthodox or unpopular views as inthis case. The conscientious objections of the petitioners, no
less than the impatience of those who disagree with them, are protected by the Constitution. The
State cannot make the individual speak when the soul within rebels. 151

Even before freedom "of expression" was included in Article III, Section 4 of the present
Constitution,this court has applied its precedent version to expressions other than verbal utterances.

In the 1985 case of Gonzalez v. Chairman Katigbak,  petitioners objected to the classification of the
152

motion picture "Kapit sa Patalim" as "For Adults Only." They contend that the classification "is
without legal and factual basis and is exercised as impermissible restraint of artistic
expression."  This court recognized that "[m]otion pictures are important both as a medium for the
153

communication of ideas and the expression of the artistic impulse."  It adds that "every writer,actor,
154

or producer, no matter what medium of expression he may use, should be freed from the
censor."  This court found that "[the Board’s] perception of what constitutes obscenity appears to be
155

unduly restrictive."  However, the petition was dismissed solely on the ground that there were not
156

enough votes for a ruling of grave abuse of discretion in the classification made by the Board. 157

II.B.3

Size does matter

The form of expression is just as important as the information conveyed that it forms part of the
expression. The present case is in point.

It is easy to discern why size matters.

First, it enhances efficiency in communication. A larger tarpaulin allows larger fonts which make it
easier to view its messages from greater distances. Furthermore, a larger tarpaulin makes it easier
for passengers inside moving vehicles to read its content. Compared with the pedestrians, the
passengers inside moving vehicles have lesser time to view the content of a tarpaulin. The larger the
fonts and images, the greater the probability that it will catch their attention and, thus, the greater the
possibility that they will understand its message.
Second, the size of the tarpaulin may underscore the importance of the message to the reader.
From an ordinary person’s perspective, those who post their messages in larger fonts care more
about their message than those who carry their messages in smaller media. The perceived
importance given by the speakers, in this case petitioners, to their cause is also part of the message.
The effectivity of communication sometimes relies on the emphasis put by the speakers and onthe
credibility of the speakers themselves. Certainly, larger segments of the public may tend to be more
convinced of the point made by authoritative figures when they make the effort to emphasize their
messages.

Third, larger spaces allow for more messages. Larger spaces, therefore, may translate to more
opportunities to amplify, explain, and argue points which the speakers might want to communicate.
Rather than simply placing the names and images of political candidates and an expression of
support, larger spaces can allow for brief but memorable presentations of the candidates’ platforms
for governance. Larger spaces allow for more precise inceptions of ideas, catalyze reactions to
advocacies, and contribute more to a more educated and reasoned electorate. A more educated
electorate will increase the possibilities of both good governance and accountability in our
government.

These points become more salient when it is the electorate, not the candidates or the political
parties, that speaks. Too often, the terms of public discussion during elections are framed and kept
hostage by brief and catchy but meaningless sound bites extolling the character of the candidate.
Worse, elections sideline political arguments and privilege the endorsement by celebrities. Rather
than provide obstacles to their speech, government should in fact encourage it. Between the
candidates and the electorate, the latter have the better incentive to demand discussion of the more
important issues. Between the candidates and the electorate, the former have better incentives to
avoid difficult political standpoints and instead focus on appearances and empty promises.

Large tarpaulins, therefore, are not analogous to time and place.  They are fundamentally part of
158

expression protected under Article III, Section 4 of the Constitution.

II.B.4

There are several theories and schools of thought that strengthen the need to protect the basic right
to freedom of expression.

First, this relates to the right ofthe people to participate in public affairs, including the right to criticize
government actions.

Proponents of the political theory on "deliberative democracy" submit that "substantial, open, [and]
ethical dialogue isa critical, and indeed defining, feature of a good polity."  This theory may be
159

considered broad, but it definitely "includes [a] collective decision making with the participation of all
who will beaffected by the decision."  It anchors on the principle that the cornerstone of every
160

democracy is that sovereignty resides in the people.  To ensure order in running the state’s affairs,
161

sovereign powers were delegated and individuals would be elected or nominated in key government
positions to represent the people. On this note, the theory on deliberative democracy may evolve to
the right of the people to make government accountable. Necessarily, this includes the right of the
people to criticize acts made pursuant to governmental functions.

Speech that promotes dialogue on publicaffairs, or airs out grievances and political discontent,
should thus be protected and encouraged.
Borrowing the words of Justice Brandeis, "it is hazardous to discourage thought, hope and
imagination; that fear breeds repression; that repression breeds hate; that hate menaces stable
government; that the path of safety lies in the opportunity to discuss freely supposed grievances and
proposed remedies." 162

In this jurisdiction, this court held that "[t]he interest of society and the maintenance of good
government demand a full discussion of public affairs."  This court has, thus, adopted the principle
163

that "debate on public issues should be uninhibited, robust,and wide open . . . [including even]
unpleasantly sharp attacks on government and public officials." 164

Second, free speech should be encouraged under the concept of a market place of ideas. This
theory was articulated by Justice Holmes in that "the ultimate good desired is better reached by [the]
free trade in ideas:"165

When men have realized that time has upset many fighting faiths, they may come to believe even
more than they believe the very foundations of their own conduct that the ultimate good desired is
better reached by free trade in ideas - that the best test of truth is the power of the thought to get
itself accepted in the competition of the market, and that truth is the only ground upon which their
wishes safely can be carried out. 166

The way it works, the exposure to the ideas of others allows one to "consider, test, and develop their
own conclusions."  A free, open, and dynamic market place of ideas is constantly shaping new
167

ones. This promotes both stability and change where recurring points may crystallize and weak ones
may develop. Of course, free speech is more than the right to approve existing political beliefs and
economic arrangements as it includes, "[t]o paraphrase Justice Holmes, [the] freedom for the
thought that we hate, no less than for the thought that agrees with us."  In fact, free speech may
168

"best serve its high purpose when it induces a condition of unrest, creates dissatisfaction with
conditions as they are, or even stirs people to anger."  It is in this context that we should guard
169

against any curtailment of the people’s right to participate in the free trade of ideas.

Third, free speech involves self-expression that enhances human dignity. This right is "a means of
assuring individual self-fulfillment,"  among others. In Philippine Blooming Mills Employees
170

Organization v. Philippine Blooming Mills Co., Inc,  this court discussed as follows:
171

The rights of free expression, free assembly and petition, are not only civil rights but also political
rights essential to man's enjoyment of his life, to his happiness and to his full and complete
fulfillment.Thru these freedoms the citizens can participate not merely in the periodic establishment
of the government through their suffrage but also in the administration of public affairs as well as in
the discipline of abusive public officers. The citizen is accorded these rights so that he can appeal to
the appropriate governmental officers or agencies for redress and protection as well as for the
imposition of the lawful sanctions on erring public officers and employees.  (Emphasis supplied)
172

Fourth, expression is a marker for group identity. For one, "[v]oluntary associations perform [an]
important democratic role [in providing] forums for the development of civil skills, for deliberation,
and for the formation of identity and community spirit[,] [and] are largely immune from [any]
governmental interference."  They also "provide a buffer between individuals and the state - a free
173

space for the development of individual personality, distinct group identity, and dissident ideas - and
a potential source of opposition to the state."  Free speech must be protected as the vehicle to find
174

those who have similar and shared values and ideals, to join together and forward common goals.

Fifth, the Bill of Rights, free speech included, is supposed to "protect individuals and minorities
against majoritarian abuses perpetrated through [the] framework [of democratic
governance]."  Federalist framers led by James Madison were concerned about two potentially
175

vulnerable groups: "the citizenry at large - majorities - who might be tyrannized or plundered by
despotic federal officials"  and the minorities who may be oppressed by "dominant factions of the
176

electorate [that] capture [the] government for their own selfish ends[.]"  According to Madison, "[i]t is
177

of great importance in a republic not only to guard the society against the oppression of its rulers, but
to guard one part of the society against the injustice of the other part."  We should strive to ensure
178

that free speech is protected especially in light of any potential oppression against those who find
themselves in the fringes on public issues.

Lastly, free speech must be protected under the safety valve theory.  This provides that "nonviolent
179

manifestations of dissent reduce the likelihood of violence[.]"  "[A] dam about to burst . . . resulting
180

in the ‘banking up of a menacing flood of sullen anger behind the walls of restriction’"  has been 181

used to describe the effect of repressing nonviolent outlets.  In order to avoid this situation and
182

prevent people from resorting to violence, there is a need for peaceful methods in making
passionate dissent. This includes "free expression and political participation"  in that they can "vote
183

for candidates who share their views, petition their legislatures to [make or] change laws, . . .
distribute literature alerting other citizens of their concerns[,]"  and conduct peaceful rallies and
184

other similar acts.  Free speech must, thus, be protected as a peaceful means of achieving one’s
185

goal, considering the possibility that repression of nonviolent dissent may spill over to violent means
just to drive a point.

II.B.5

Every citizen’s expression with political consequences enjoys a high degree of protection.
Respondents argue that the tarpaulinis election propaganda, being petitioners’ way of endorsing
candidates who voted against the RH Law and rejecting those who voted for it.  As such, it is 186

subject to regulation by COMELEC under its constitutional mandate.  Election propaganda is 187

defined under Section 1(4) of COMELEC Resolution No. 9615 as follows: SECTION 1. Definitions . .
.

....

4. The term "political advertisement" or "election propaganda" refers to any matter broadcasted,
published, printed, displayed or exhibited, in any medium, which contain the name, image, logo,
brand, insignia, color motif, initials, and other symbol or graphic representation that is capable of
being associated with a candidate or party, and is intended to draw the attention of the public or a
segment thereof to promote or oppose, directly or indirectly, the election of the said candidate or
candidates to a public office. In broadcast media, political advertisements may take the form of
spots, appearances on TV shows and radio programs, live or taped announcements, teasers, and
other forms of advertising messages or announcements used by commercial advertisers. Political
advertising includes matters, not falling within the scope of personal opinion, that appear on any
Internet website, including, but not limited to, social networks, blogging sites, and micro-blogging
sites, in return for consideration, or otherwise capable of pecuniary estimation.

On the other hand, petitioners invoke their "constitutional right to communicate their opinions, views
and beliefs about issues and candidates."  They argue that the tarpaulin was their statement of
188

approval and appreciation of the named public officials’ act of voting against the RH Law, and their
criticism toward those who voted in its favor. It was "part of their advocacy campaign against the RH
189

Law,"  which was not paid for by any candidate or political party.  Thus, "the questioned orders
190 191

which . . . effectively restrain[ed] and curtail[ed] [their] freedom of expression should be declared
unconstitutional and void." 192
This court has held free speech and other intellectual freedoms as "highly ranked in our scheme of
constitutional values."  These rights enjoy precedence and primacy.  In Philippine Blooming Mills,
193 194

this court discussed the preferred position occupied by freedom of expression:

Property and property rights can belost thru prescription; but human rights are imprescriptible. If
human rights are extinguished by the passage of time, then the Bill of Rights is a useless attempt to
limit the power of government and ceases to be an efficacious shield against the tyranny of officials,
of majorities, ofthe influential and powerful, and of oligarchs - political, economic or otherwise.

In the hierarchy of civil liberties, the rights of free expression and of assembly occupy a preferred
position as they are essential to the preservation and vitality of our civil and political institutions; and
such priority "gives these liberties the sanctity and the sanction not permitting dubious
intrusions."  (Citations omitted)
195

This primordial right calls for utmost respect, more so "when what may be curtailed is the
dissemination of information to make more meaningful the equally vital right of suffrage."  A similar
196

idea appeared in our jurisprudence as early as 1969, which was Justice Barredo’s concurring and
dissenting opinion in Gonzales v. COMELEC: 197

I like to reiterate over and over, for it seems this is the fundamental point others miss, that genuine
democracy thrives only where the power and right of the people toelect the men to whom they would
entrust the privilege to run the affairs of the state exist. In the language of the declaration of
principles of our Constitution, "The Philippines is a republican state. Sovereignty resides in the
people and all government authority emanates from them" (Section 1, Article II). Translating this
declaration into actuality, the Philippines is a republic because and solely because the people in it
can be governed only by officials whom they themselves have placed in office by their votes. And in
it is on this cornerstone that I hold it tobe self-evident that when the freedoms of speech, press and
peaceful assembly and redress of grievances are being exercised in relation to suffrage or asa
means to enjoy the inalienable right of the qualified citizen to vote, they are absolute and timeless. If
our democracy and republicanism are to be worthwhile, the conduct of public affairs by our officials
must be allowed to suffer incessant and unabating scrutiny, favorable or unfavorable, everyday and
at all times. Every holder of power in our government must be ready to undergo exposure any
moment of the day or night, from January to December every year, as it is only in this way that he
can rightfully gain the confidence of the people. I have no patience for those who would regard
public dissection of the establishment as an attribute to be indulged by the people only at certain
periods of time. I consider the freedoms of speech, press and peaceful assembly and redress of
grievances, when exercised in the name of suffrage, as the very means by which the right itself to
vote can only be properly enjoyed.It stands to reason therefore, that suffrage itself would be next to
useless if these liberties cannot be untrammelled [sic] whether as to degree or time.  (Emphasis
198

supplied)

Not all speech are treated the same. In Chavez v. Gonzales, this court discussed that some types of
speech may be subject to regulation:

Some types of speech may be subjected to some regulation by the State under its pervasive police
power, in order that it may not be injurious to the equal right of others or those of the community or
society. The difference in treatment is expected because the relevant interests of one type of
speech, e.g., political speech, may vary from those of another, e.g., obscene speech.
Distinctionshave therefore been made in the treatment, analysis, and evaluation ofthe permissible
scope of restrictions on various categories of speech. We have ruled, for example, that in our
jurisdiction slander or libel, lewd and obscene speech, as well as "fighting words" are not entitled to
constitutional protection and may be penalized.  (Citations omitted)
199
We distinguish between politicaland commercial speech. Political speech refers to speech "both
intended and received as a contribution to public deliberation about some issue,"  "foster[ing]
200

informed and civicminded deliberation."  On the other hand, commercial speech has been defined
201

as speech that does "no more than propose a commercial transaction."  The expression resulting
202

from the content of the tarpaulin is, however, definitely political speech. In Justice Brion’s dissenting
opinion, he discussed that "[t]he content of the tarpaulin, as well as the timing of its posting, makes it
subject of the regulations in RA 9006 and Comelec Resolution No. 9615."  He adds that "[w]hile
203

indeed the RH issue, by itself,is not an electoralmatter, the slant that the petitioners gave the issue
converted the non-election issue into a live election one hence, Team Buhay and Team Patay and
the plea to support one and oppose the other." 204

While the tarpaulin may influence the success or failure of the named candidates and political
parties, this does not necessarily mean it is election propaganda. The tarpaulin was not paid for or
posted "in return for consideration" by any candidate, political party, or party-list group.

The second paragraph of Section 1(4) of COMELEC Resolution No. 9615, or the rules and
regulations implementing Republic Act No. 9006 as an aid to interpret the law insofar as the facts of
this case requires, states:

4. The term "political advertisement" or "election propaganda" refers to any matter broadcasted,
published, printed, displayed or exhibited, in any medium, which contain the name, image, logo,
brand, insignia, color motif, initials, and other symbol or graphic representation that is capable of
being associated with a candidate or party, and is intended to draw the attention of the public or a
segment thereof to promote or oppose, directly or indirectly, the election of the said candidate or
candidates to a public office. In broadcast media, political advertisements may take the form of
spots, appearances on TV shows and radio programs, live or taped announcements, teasers, and
other forms of advertising messages or announcements used by commercial advertisers. Political
advertising includes matters, not falling within the scope of personal opinion, that appear on any
Internet website, including, but not limited to, social networks, blogging sites, and micro-blogging
sites, in return for consideration, or otherwise capable of pecuniary estimation. (Emphasis supplied)

It is clear that this paragraph suggests that personal opinions are not included, while sponsored
messages are covered.

Thus, the last paragraph of Section 1(1) of COMELEC Resolution No. 9615 states:

SECTION 1. Definitions - As used in this Resolution:

1. The term "election campaign" or "partisan political activity" refers to an act designed to promote
the election or defeat of a particular candidate or candidates to a public office, and shall include any
of the following:

....

Personal opinions, views, and preferences for candidates, contained in blogs shall not be considered
acts of election campaigning or partisan politicalactivity unless expressed by government officials in
the Executive Department, the Legislative Department, the Judiciary, the Constitutional
Commissions, and members of the Civil Service.

In any event, this case does not refer to speech in cyberspace, and its effects and parameters
should be deemed narrowly tailored only in relation to the facts and issues in this case. It also
appears that such wording in COMELEC Resolution No. 9615 does not similarly appear in Republic
Act No. 9006, the law it implements.

We should interpret in this manner because of the value of political speech.

As early as 1918, in United States v. Bustos,  this court recognized the need for full discussion of
205

public affairs. We acknowledged that free speech includes the right to criticize the conduct of public
men:

The interest of society and the maintenance of good government demand a full discussion of public
affairs. Complete liberty to comment on the conduct of public men is a scalpel in the case of free
speech. The sharp incision of its probe relieves the abscesses of official dom. Men in public life may
suffer under a hostile and an unjust accusation; the wound can be assuaged with the balm of a clear
conscience. A public officer must not be too thin-skinned with reference to comment upon his official
acts. Only thus can the intelligence and dignity of the individual be exalted.
206

Subsequent jurisprudence developed the right to petition the government for redress of grievances,
allowing for criticism, save for some exceptions.  In the 1951 case of Espuelas v. People,  this
207 208

court noted every citizen’s privilege to criticize his or her government, provided it is "specific and
therefore constructive, reasoned or tempered, and not a contemptuous condemnation of the entire
government set-up." 209

The 1927 case of People v. Titular  involved an alleged violation of the Election Law provision
210

"penaliz[ing] the anonymous criticism of a candidate by means of posters or circulars."  This court
211

explained that it is the poster’s anonymous character that is being penalized.  The ponente adds
212

that he would "dislike very muchto see this decision made the vehicle for the suppression of public
opinion."
213

In 1983, Reyes v. Bagatsing  discussed the importance of allowing individuals to vent their views.
214

According to this court, "[i]ts value may lie in the fact that there may be something worth hearing
from the dissenter [and] [t]hat is to ensurea true ferment of ideas."
215

Allowing citizens to air grievances and speak constructive criticisms against their government
contributes to every society’s goal for development. It puts forward matters that may be changed for
the better and ideas that may be deliberated on to attain that purpose. Necessarily, it also makes the
government accountable for acts that violate constitutionally protected rights.

In 1998, Osmeña v. COMELEC found Section 11(b) of Republic Act No. 6646, which prohibits mass
media from selling print space and air time for campaign except to the COMELEC, to be a
democracy-enhancing measure. This court mentioned how "discussion of public issues and debate
216

on the qualifications of candidates in an election are essential to the proper functioning of the
government established by our Constitution." 217

As pointed out by petitioners, "speech serves one of its greatest public purposes in the context of
elections when the free exercise thereof informs the people what the issues are, and who are
supporting what issues."  At the heart of democracy is every advocate’s right to make known what
218

the people need to know,  while the meaningful exercise of one’s right of suffrage includes the right
219

of every voter to know what they need to know in order to make their choice.
Thus, in Adiong v. COMELEC,  this court discussed the importance of debate on public issues, and
220

the freedom of expression especially in relation to information that ensures the meaningful exercise
of the right of suffrage:

We have adopted the principle that debate on public issues should be uninhibited, robust, and wide
open and that it may well include vehement, caustic and sometimes unpleasantly sharp attacks on
government and public officials. Too many restrictions will deny to people the robust, uninhibited,
and wide open debate, the generating of interest essential if our elections will truly be free, clean and
honest.

We have also ruled that the preferred freedom of expression calls all the more for the utmost respect
when what may be curtailed is the dissemination of information to make more meaningful the equally
vital right of suffrage. (Emphasis supplied, citations omitted)
221

Speech with political consequences isat the core of the freedom of expression and must be
protected by this court.

Justice Brion pointed out that freedomof expression "is not the god of rights to which all other rights
and even government protection of state interest must bow." 222

The right to freedom of expression isindeed not absolute. Even some forms of protected speech are
still subjectto some restrictions. The degree of restriction may depend on whether the regulation is
content-based or content-neutral.  Content-based regulations can either be based on the viewpoint
223

of the speaker or the subject of the expression.

II.B.6

Content-based regulation

COMELEC contends that the order for removal of the tarpaulin is a content-neutral regulation. The
order was made simply because petitioners failed to comply with the maximum size limitation for
lawful election propaganda. 224

On the other hand, petitioners argue that the present size regulation is content-based as it applies
only to political speech and not to other forms of speech such as commercial speech.  "[A]ssuming
225

arguendo that the size restriction sought to be applied . . . is a mere time, place, and manner
regulation, it’s still unconstitutional for lack of a clear and reasonable nexus with a constitutionally
sanctioned objective."226

The regulation may reasonably be considered as either content-neutral or content-


based.  Regardless, the disposition of this case will be the same. Generally, compared with other
227

forms of speech, the proposed speech is content-based.

As pointed out by petitioners, the interpretation of COMELEC contained in the questioned order
applies only to posters and tarpaulins that may affect the elections because they deliver opinions
that shape both their choices. It does not cover, for instance, commercial speech.

Worse, COMELEC does not point to a definite view of what kind of expression of non-candidates will
be adjudged as "election paraphernalia." There are no existing bright lines to categorize speech as
election-related and those that are not. This is especially true when citizens will want to use their
resources to be able to raise public issues that should be tackled by the candidates as what has
happened in this case. COMELEC’s discretion to limit speech in this case is fundamentally
unbridled.

Size limitations during elections hit ata core part of expression. The content of the tarpaulin is not
easily divorced from the size of its medium.

Content-based regulation bears a heavy presumption of invalidity, and this court has used the clear
and present danger rule as measure.  Thus, in Chavez v. Gonzales:
228

A content-based regulation, however, bears a heavy presumption of invalidity and is measured


against the clear and present danger rule. The latter will pass constitutional muster only if justified by
a compelling reason, and the restrictions imposedare neither overbroad nor vague.  (Citations 229

omitted)

Under this rule, "the evil consequences sought to be prevented must be substantive, ‘extremely
serious and the degree of imminence extremely high.’"  "Only when the challenged act has
230

overcome the clear and present danger rule will it pass constitutional muster, with the government
having the burden of overcoming the presumed unconstitutionality." 231

Even with the clear and present danger test, respondents failed to justify the regulation. There is no
compelling and substantial state interest endangered by the posting of the tarpaulinas to justify
curtailment of the right of freedom of expression. There is no reason for the state to minimize the
right of non-candidate petitioners to post the tarpaulin in their private property. The size of the
tarpaulin does not affect anyone else’s constitutional rights.

Content-based restraint or censorship refers to restrictions "based on the subject matter of the
utterance or speech."  In contrast, content-neutral regulation includes controls merely on the
232

incidents of the speech such as time, place, or manner of the speech. 233

This court has attempted to define "content-neutral" restraints starting with the 1948 case of
Primicias v. Fugoso. The ordinance in this case was construed to grant the Mayor discretion only to
234

determine the public places that may be used for the procession ormeeting, but not the power to
refuse the issuance of a permit for such procession or meeting.  This court explained that free
235

speech and peaceful assembly are "not absolute for it may be so regulated that it shall not
beinjurious to the equal enjoyment of others having equal rights, nor injurious to the rights of the
community or society." 236

The earlier case of Calalang v. Williams  involved the National Traffic Commission resolution that
237

prohibited the passing of animal-drawn vehicles along certain roads at specific hours.  This court
238

similarly discussed police power in that the assailed rules carry outthe legislative policy that "aims to
promote safe transit upon and avoid obstructions on national roads, in the interest and convenience
of the public."
239

As early as 1907, United States v. Apurado  recognized that "more or less disorder will mark the
240

public assembly of the people to protest against grievances whether real or imaginary, because on
such occasions feeling is always wrought to a high pitch of excitement. . . ."  It is with this backdrop
241

that the state is justified in imposing restrictions on incidental matters as time, place, and manner of
the speech.

In the landmark case of Reyes v. Bagatsing, this court summarized the steps that permit applicants
must follow which include informing the licensing authority ahead of time as regards the date, public
place, and time of the assembly.  This would afford the public official time to inform applicants if
242

there would be valid objections, provided that the clear and present danger test is the standard used
for his decision and the applicants are given the opportunity to be heard.  This ruling was practically
243

codified in Batas Pambansa No. 880, otherwise known as the Public Assembly Act of 1985.

Subsequent jurisprudence have upheld Batas Pambansa No. 880 as a valid content-neutral
regulation. In the 2006 case of Bayan v. Ermita,  this court discussed how Batas Pambansa No. 880
244

does not prohibit assemblies but simply regulates their time, place, and manner.  In 2010, this court
245

found in Integrated Bar of the Philippines v. Atienza  that respondent Mayor Atienza committed
246

grave abuse of discretion when he modified the rally permit by changing the venue from Mendiola
Bridge to Plaza Miranda without first affording petitioners the opportunity to be heard. 247

We reiterate that the regulation involved at bar is content-based. The tarpaulin content is not easily
divorced from the size of its medium.

II.B.7

Justice Carpio and Justice Perlas-Bernabe suggest that the provisions imposing a size limit for
tarpaulins are content-neutral regulations as these "restrict the mannerby which speech is relayed
but not the content of what is conveyed." 248

If we apply the test for content-neutral regulation, the questioned acts of COMELEC will not pass the
three requirements for evaluating such restraints on freedom of speech.  "When the speech
249

restraints take the form of a content-neutral regulation, only a substantial governmental interest is
required for its validity,"  and it is subject only to the intermediate approach.
250 251

This intermediate approach is based on the test that we have prescribed in several cases.  A 252

content-neutral government regulation is sufficiently justified:

[1] if it is within the constitutional power of the Government; [2] if it furthers an important or
substantial governmental interest; [3] if the governmental interest is unrelated to the suppression of
free expression; and [4] if the incident restriction on alleged [freedom of speech & expression] is no
greater than is essential to the furtherance of that interest.253

On the first requisite, it is not within the constitutional powers of the COMELEC to regulate the
tarpaulin. As discussed earlier, this is protected speech by petitioners who are non-candidates. On
the second requirement, not only must the governmental interest be important or substantial, it must
also be compelling as to justify the restrictions made.

Compelling governmental interest would include constitutionally declared principles. We have held,
for example, that "the welfare of children and the State’s mandate to protect and care for them, as
parens patriae,  constitute a substantial and compelling government interest in regulating . . .
254

utterances in TV broadcast." 255

Respondent invokes its constitutional mandate to ensure equal opportunity for public information
campaigns among candidates in connection with the holding of a free, orderly, honest, peaceful, and
credible election.256

Justice Brion in his dissenting opinion discussed that "[s]ize limits to posters are necessary to ensure
equality of public information campaigns among candidates, as allowing posters with different sizes
gives candidates and their supporters the incentive to post larger posters[,] [and] [t]his places
candidates with more money and/or with deep-pocket supporters at an undue advantage against
candidates with more humble financial capabilities." 257

First, Adiong v. COMELEC has held that this interest is "not as important as the right of [a private
citizen] to freely express his choice and exercise his right of free speech."  In any case, faced with
258

both rights to freedom of speech and equality, a prudent course would be to "try to resolve the
tension in a way that protects the right of participation."259

Second, the pertinent election lawsrelated to private property only require that the private property
owner’s consent be obtained when posting election propaganda in the property.  This is consistent
260

with the fundamental right against deprivation of property without due process of law.  The present
261

facts do not involve such posting of election propaganda absent consent from the property owner.
Thus, this regulation does not apply in this case.

Respondents likewise cite the Constitution  on their authority to recommend effective measures to
262

minimize election spending. Specifically, Article IX-C, Section 2(7) provides:

Sec. 2. The Commission on Elections shall exercise the following powers and functions:

....

(7) Recommend to the Congress effective measures to minimize election spending, including
limitation of places where propaganda materials shall be posted, and to prevent and penalize all
forms of election frauds, offenses, malpractices, and nuisance candidates. (Emphasis supplied) This
does not qualify as a compelling and substantial government interest to justify regulation of the
preferred right to freedom of expression.

The assailed issuances for the removal of the tarpaulin are based on the two feet (2’) by three feet
(3’) size limitation under Section 6(c) of COMELEC Resolution No. 9615. This resolution implements
the Fair Election Act that provides for the same size limitation.263

This court held in Adiong v. COMELEC that "[c]ompared to the paramount interest of the State in
guaranteeing freedom of expression, any financial considerations behind the regulation are of
marginal significance."  In fact, speech with political consequences, as in this case, should be
264

encouraged and not curtailed. As petitioners pointed out, the size limitation will not serve the
objective of minimizing election spending considering there is no limit on the number of tarpaulins
that may be posted. 265

The third requisite is likewise lacking. We look not only at the legislative intent or motive in imposing
the restriction, but more so at the effects of such restriction, if implemented. The restriction must not
be narrowly tailored to achieve the purpose. It must be demonstrable. It must allow alternative
avenues for the actor to make speech.

In this case, the size regulation is not unrelated to the suppression of speech. Limiting the maximum
sizeof the tarpaulin would render ineffective petitioners’ message and violate their right to exercise
freedom of expression.

The COMELEC’s act of requiring the removal of the tarpaulin has the effect of dissuading
expressions with political consequences. These should be encouraged, more so when exercised to
make more meaningful the equally important right to suffrage.
The restriction in the present case does not pass even the lower test of intermediate scrutiny for
content-neutral regulations.

The action of the COMELEC in thiscase is a strong deterrent to further speech by the electorate.
Given the stature of petitioners and their message, there are indicators that this will cause a "chilling
effect" on robust discussion during elections.

The form of expression is just as important as the message itself. In the words of Marshall McLuhan,
"the medium is the message."  McLuhan’s colleague and mentor Harold Innis has earlier asserted
266

that "the materials on which words were written down have often counted for more than the words
themselves." 267

III
Freedom of expression and equality

III.A

The possibility of abuse

Of course, candidates and political parties do solicit the help of private individuals for the
endorsement of their electoral campaigns.

On the one extreme, this can take illicit forms such as when endorsement materials in the form of
tarpaulins, posters, or media advertisements are made ostensibly by "friends" but in reality are really
paid for by the candidate or political party. This skirts the constitutional value that provides for equal
opportunities for all candidates.

However, as agreed by the parties during the oral arguments in this case, this is not the situation
that confronts us. In such cases, it will simply be a matter for investigation and proof of fraud on the
part of the COMELEC.

The guarantee of freedom of expression to individuals without any relationship to any political
candidate should not be held hostage by the possibility of abuse by those seeking to be elected. It is
true that there can be underhanded, covert, or illicit dealings so as to hide the candidate’s real levels
of expenditures. However, labelling all expressions of private parties that tend to have an effect on
the debate in the elections as election paraphernalia would be too broad a remedy that can stifle
genuine speech like in this case. Instead, to address this evil, better and more effective enforcement
will be the least restrictive means to the fundamental freedom.

On the other extreme, moved by the credentials and the message of a candidate, others will spend
their own resources in order to lend support for the campaigns. This may be without agreement
between the speaker and the candidate or his or her political party. In lieu of donating funds to the
campaign, they will instead use their resources directly in a way that the candidate or political party
would have doneso. This may effectively skirt the constitutional and statutory limits of campaign
spending.

Again, this is not the situation in this case.

The message of petitioners in thiscase will certainly not be what candidates and political parties will
carry in their election posters or media ads. The message of petitioner, taken as a whole, is an
advocacy of a social issue that it deeply believes. Through rhetorical devices, it communicates the
desire of Diocese that the positions of those who run for a political position on this social issue be
determinative of how the public will vote. It primarily advocates a stand on a social issue; only
secondarily — even almost incidentally — will cause the election or non-election of a candidate.

The twin tarpaulins consist of satire of political parties. Satire is a "literary form that employs such
devices as sarcasm, irony and ridicule to deride prevailing vices or follies,"  and this may target any
268

individual or group in society, private and government alike. It seeks to effectively communicate a
greater purpose, often used for "political and social criticism"  "because it tears down facades,
269

deflates stuffed shirts, and unmasks hypocrisy. . . . Nothing is more thoroughly democratic than to
have the high-and-mighty lampooned and spoofed."  Northrop Frye, wellknown in this literary field,
270

claimed that satire had two defining features: "one is wit or humor founded on fantasy or a sense of
the grotesque and absurd, the other is an object of attack."  Thus, satire frequently uses
271

exaggeration, analogy, and other rhetorical devices.

The tarpaulins exaggerate. Surely, "Team Patay" does not refer to a list of dead individuals nor could
the Archbishop of the Diocese of Bacolod have intended it to mean that the entire plan of the
candidates in his list was to cause death intentionally. The tarpaulin caricatures political parties and
parodies the intention of those in the list. Furthermore, the list of "Team Patay" is juxtaposed with the
list of "Team Buhay" that further emphasizes the theme of its author: Reproductive health is an
important marker for the church of petitioners to endorse.

The messages in the tarpaulins are different from the usual messages of candidates. Election
paraphernalia from candidates and political parties are more declarative and descriptive and contain
no sophisticated literary allusion to any social objective. Thus, they usually simply exhort the public
to vote for a person with a brief description of the attributes of the candidate. For example "Vote for
[x], Sipag at Tiyaga," "Vote for [y], Mr. Palengke," or "Vote for [z], Iba kami sa Makati."

This court’s construction of the guarantee of freedom of expression has always been wary of
censorship or subsequent punishment that entails evaluation of the speaker’s viewpoint or the
content of one’s speech. This is especially true when the expression involved has political
consequences. In this case, it hopes to affect the type of deliberation that happens during elections.
A becoming humility on the part of any human institution no matter how endowed with the secular
ability to decide legal controversies with finality entails that we are not the keepers of all wisdom.

Humanity’s lack of omniscience, even acting collectively, provides space for the weakest dissent.
Tolerance has always been a libertarian virtue whose version is embedded in our Billof Rights.
There are occasional heretics of yesterday that have become our visionaries. Heterodoxies have
always given us pause. The unforgiving but insistent nuance that the majority surely and comfortably
disregards provides us with the checks upon reality that may soon evolve into creative solutions to
grave social problems. This is the utilitarian version. It could also be that it is just part of human
necessity to evolve through being able to express or communicate.

However, the Constitution we interpret is not a theoretical document. It contains other provisions
which, taken together with the guarantee of free expression, enhances each other’s value. Among
these are the provisions that acknowledge the idea of equality. In shaping doctrine construing these
constitutional values, this court needs to exercise extraordinary prudence and produce narrowly
tailored guidance fit to the facts as given so as not to unwittingly cause the undesired effect of
diluting freedoms as exercised in reality and, thus, render them meaningless.

III.B.

Speech and equality:


Some considerations We first establish that there are two paradigms of free speech that separate at
the point of giving priority to equality vis-à-vis liberty.
272

In an equality-based approach, "politically disadvantaged speech prevails over regulation[,] but


regulation promoting political equality prevails over speech."  This view allows the government
273

leeway to redistribute or equalize ‘speaking power,’ such as protecting, even implicitly subsidizing,
unpopular or dissenting voices often systematically subdued within society’s ideological
ladder.  This view acknowledges that there are dominant political actors who, through authority,
274

power, resources, identity, or status, have capabilities that may drown out the messages of others.
This is especially true in a developing or emerging economy that is part of the majoritarian world like
ours.

The question of libertarian tolerance

This balance between equality and the ability to express so as to find one’s authentic self or to
participate in the self determination of one’s communities is not new only to law. It has always been
a philosophical problematique.

In his seminal work, Repressive Tolerance, philosopher and social theorist Herbert Marcuse
recognized how institutionalized inequality exists as a background limitation, rendering freedoms
exercised within such limitation as merely "protect[ing] the already established machinery of
discrimination."  In his view, any improvement "in the normal course of events" within an unequal
275

society, without subversion, only strengthens existing interests of those in power and control.276

In other words, abstract guarantees of fundamental rights like freedom of expression may become
meaningless if not taken in a real context. This tendency to tackle rights in the abstract compromises
liberties. In his words:

Liberty is self-determination, autonomy—this is almost a tautology, but a tautology which results


from a whole series of synthetic judgments. It stipulates the ability to determine one’s own life: to be
able to determine what to do and what not to do, what to suffer and what not. But the subject of this
autonomy is never the contingent, private individual as that which he actually is or happens to be; it
is rather the individual as a human being who is capable of being free with the others. And the
problem of making possible such a harmony between every individual liberty and the other is not that
of finding a compromise between competitors, or between freedom and law, between general and
individual interest, common and private welfare in an established society, but of creating the society
in which man is no longer enslaved by institutions which vitiate self-determination from the
beginning. In other words, freedom is still to be created even for the freest of the existing
societies.  (Emphasis in the original)
277

Marcuse suggests that the democratic argument — with all opinions presented to and deliberated by
the people — "implies a necessary condition, namely, that the people must be capable of
deliberating and choosing on the basis of knowledge, that they must have access to authentic
information, and that, on this basis, their evaluation must be the result of autonomous thought."  He278

submits that "[d]ifferent opinions and ‘philosophies’ can no longer compete peacefully for adherence
and persuasion on rational grounds: the ‘marketplace of ideas’ is organized and delimited by those
who determine the national and the individual interest."  A slant toward left manifests from his belief
279

that "there is a ‘natural right’ of resistance for oppressed and overpowered minorities to use
extralegal means if the legal ones have proved to be inadequate."  Marcuse, thus, stands for an
280

equality that breaks away and transcends from established hierarchies, power structures, and
indoctrinations. The tolerance of libertarian society he refers to as "repressive tolerance."
Legal scholars

The 20th century also bears witness to strong support from legal scholars for "stringent protections
of expressive liberty,"  especially by political egalitarians. Considerations such as "expressive,
281

deliberative, and informational interests,"  costs or the price of expression, and background facts,
282

when taken together, produce bases for a system of stringent protections for expressive liberties. 283

Many legal scholars discuss the interest and value of expressive liberties. Justice Brandeis proposed
that "public discussion is a political duty."  Cass Sustein placed political speech on the upper tier of
284

his twotier model for freedom of expression, thus, warranting stringent protection.  He defined
285

political speech as "both intended and received as a contribution to public deliberation about some
issue."286

But this is usually related also tofair access to opportunities for such liberties.  Fair access to
287

opportunity is suggested to mean substantive equality and not mere formal equalitysince "favorable
conditions for realizing the expressive interest will include some assurance of the resources required
for expression and some guarantee that efforts to express views on matters of common concern will
not be drowned out by the speech of betterendowed citizens."  Justice Brandeis’ solution is to
288

"remedy the harms of speech with more speech."  This view moves away from playing down the
289

danger as merely exaggerated, toward "tak[ing] the costs seriously and embrac[ing] expression as
the preferred strategy for addressing them."  However, in some cases, the idea of more speech
290

may not be enough. Professor Laurence Tribe observed the need for context and "the specification
of substantive values before [equality] has full meaning."  Professor Catherine A. MacKinnon adds
291

that "equality continues to be viewed in a formal rather than a substantive sense."  Thus, more
292

speech can only mean more speech from the few who are dominant rather than those who are not.

Our jurisprudence

This court has tackled these issues.

Osmeña v. COMELEC affirmed National Press Club v. COMELEC on the validity of Section 11(b)
ofthe Electoral Reforms Law of 1987.  This section "prohibits mass media from selling or giving free
293

of charge print space or air time for campaign or other political purposes, except to the Commission
on Elections."  This court explained that this provision only regulates the time and manner of
294

advertising in order to ensure media equality among candidates.  This court grounded this measure
295

on constitutional provisions mandating political equality:  Article IX-C, Section 4


296

Section 4. The Commission may, during the election period, supervise or regulate the enjoyment or
utilization of all franchises or permits for the operation of transportation and other public utilities,
media of communication or information, all grants, special privileges, or concessions granted by the
Government or any subdivision, agency, or instrumentality thereof, including any government-owned
or controlled corporation or its subsidiary. Such supervision or regulation shall aim to ensure equal
opportunity, time, and space, and the right to reply, including reasonable, equal rates therefor, for
public information campaigns and forums among candidates in connection with the objective of
holding free, orderly, honest, peaceful, and credible elections. (Emphasis supplied)

Article XIII, Section 1

Section 1. The Congress shall give highest priorityto the enactment of measures that protect and
enhance the right of all the people to human dignity, reducesocial, economic, and political
inequalities, and remove cultural inequities by equitably diffusing wealth and political power for the
common good.
To this end, the State shall regulate the acquisition, ownership, use, and disposition of property and
its increments. (Emphasis supplied)

Article II, Section 26

Section 26. The State shall guarantee equal access to opportunities for public service, and prohibit
political dynasties as may be defined by law. (Emphasis supplied)

Thus, in these cases, we have acknowledged the Constitution’s guarantee for more substantive
expressive freedoms that take equality of opportunities into consideration during elections.

The other view

However, there is also the other view. This is that considerations of equality of opportunity or
equality inthe ability of citizens as speakers should not have a bearing in free speech doctrine.
Under this view, "members of the public are trusted to make their own individual evaluations of
speech, and government is forbidden to intervene for paternalistic or redistributive reasons . . .
[thus,] ideas are best left to a freely competitive ideological market."  This is consistent with the
297

libertarian suspicion on the use of viewpoint as well as content to evaluate the constitutional validity
or invalidity of speech.

The textual basis of this view is that the constitutional provision uses negative rather than affirmative
language. It uses ‘speech’ as its subject and not ‘speakers’.  Consequently, the Constitution
298

protects free speech per se, indifferent to the types, status, or associations of its
speakers.  Pursuant to this, "government must leave speakers and listeners in the private order to
299

their own devices in sorting out the relative influence of speech."300

Justice Romero’s dissenting opinion in Osmeña v. COMELEC formulates this view that freedom of
speech includes "not only the right to express one’s views, but also other cognate rights relevant to
the free communication [of] ideas, not excluding the right to be informed on matters of public
concern."  She adds:
301

And since so many imponderables may affect the outcome of elections — qualifications of voters
and candidates, education, means of transportation, health, public discussion, private animosities,
the weather, the threshold of a voter’s resistance to pressure — the utmost ventilation of opinion of
men and issues, through assembly, association and organizations, both by the candidate and the
voter, becomes a sine qua non for elections to truly reflect the will of the electorate.  (Emphasis
302

supplied)

Justice Romero’s dissenting opinion cited an American case, if only to emphasize free speech
primacy such that"courts, as a rule are wary to impose greater restrictions as to any attempt to
curtail speeches with political content,"  thus:
303

the concept that the government may restrict the speech of some elements in our society in order to
enhance the relative voice of the others is wholly foreign to the First Amendment which was
designed to "secure the widest possible dissemination of information from diverse and antagonistic
sources" and "to assure unfettered interchange of ideas for the bringing about of political and social
changes desired by the people." 304

This echoes Justice Oliver Wendell Holmes’ submission "that the market place of ideas is still the
best alternative to censorship."305
Parenthetically and just to provide the whole detail of the argument, the majority of the US Supreme
Court in the campaign expenditures case of Buckley v. Valeo "condemned restrictions (even if
content-neutral) on expressive liberty imposed in the name of ‘enhanc[ing] the relative voice of
others’ and thereby ‘equaliz[ing] access to the political arena."  The majority did not use the
306

equality-based paradigm.

One flaw of campaign expenditurelimits is that "any limit placed on the amount which a person can
speak, which takes out of his exclusive judgment the decision of when enough is enough, deprives
him of his free speech."307

Another flaw is how "[a]ny quantitative limitation on political campaigning inherently constricts the
sum of public information and runs counter to our ‘profound national commitment that debate on
public issues should be uninhibited, robust, and wide-open.’" 308

In fact, "[c]onstraining those who have funds or have been able to raise funds does not ease the
plight of those without funds in the first place . . . [and] even if one’s main concern isslowing the
increase in political costs, it may be more effective torely on market forces toachieve that result than
on active legal intervention."  According to Herbert Alexander, "[t]o oppose limitations is not
309

necessarily to argue that the sky’s the limit [because in] any campaign there are saturation levels
and a point where spending no longer pays off in votes per dollar." 310

III. C.

When private speech amounts

to election paraphernalia

The scope of the guarantee of free expression takes into consideration the constitutional respect for
human potentiality and the effect of speech. It valorizes the ability of human beings to express and
their necessity to relate. On the other hand, a complete guarantee must also take into consideration
the effects it will have in a deliberative democracy. Skewed distribution of resources as well as the
cultural hegemony of the majority may have the effect of drowning out the speech and the messages
of those in the minority. In a sense, social inequality does have its effect on the exercise and effect
of the guarantee of free speech. Those who have more will have better access to media that
reaches a wider audience than those who have less. Those who espouse the more popular ideas
will have better reception than the subversive and the dissenters of society.To be really heard and
understood, the marginalized view normally undergoes its own degree of struggle.

The traditional view has been to tolerate the viewpoint of the speaker and the content of his or her
expression. This view, thus, restricts laws or regulation that allows public officials to make judgments
of the value of such viewpoint or message content. This should still be the principal approach.

However, the requirements of the Constitution regarding equality in opportunity must provide limits to
some expression during electoral campaigns.

Thus clearly, regulation of speech in the context of electoral campaigns made by candidates or the
members of their political parties or their political parties may be regulated as to time, place, and
manner. This is the effect of our rulings in Osmeña v. COMELEC and National Press Club v.
COMELEC.
Regulation of speech in the context of electoral campaigns made by persons who are not candidates
or who do not speak as members of a political party which are, taken as a whole, principally
advocacies of a social issue that the public must consider during elections is unconstitutional. Such
regulation is inconsistent with the guarantee of according the fullest possible range of opinions
coming from the electorate including those that can catalyze candid, uninhibited, and robust debate
in the criteria for the choice of a candidate.

This does not mean that there cannot be a specie of speech by a private citizen which will not
amount toan election paraphernalia to be validly regulated by law.

Regulation of election paraphernalia will still be constitutionally valid if it reaches into speech of
persons who are not candidates or who do not speak as members of a political party if they are not
candidates, only if what is regulated is declarative speech that, taken as a whole, has for its principal
object the endorsement of a candidate only. The regulation (a) should be provided by law, (b)
reasonable, (c) narrowly tailored to meet the objective of enhancing the opportunity of all candidates
to be heard and considering the primacy of the guarantee of free expression, and (d) demonstrably
the least restrictive means to achieve that object. The regulation must only be with respect to the
time, place, and manner of the rendition of the message. In no situation may the speech be
prohibited or censored onthe basis of its content. For this purpose, it will notmatter whether the
speech is made with or on private property.

This is not the situation, however, in this case for two reasons. First, as discussed, the principal
message in the twin tarpaulins of petitioners consists of a social advocacy.

Second, as pointed out in the concurring opinion of Justice Antonio Carpio, the present law —
Section 3.3 of Republic Act No. 9006 and Section 6(c) of COMELEC Resolution No. 9615 — if
applied to this case, will not pass the test of reasonability. A fixed size for election posters or
tarpaulins without any relation to the distance from the intended average audience will be arbitrary.
At certain distances, posters measuring 2 by 3 feet could no longer be read by the general public
and, hence, would render speech meaningless. It will amount to the abridgement of speech with
political consequences.

IV
Right to property

Other than the right to freedom of expression  and the meaningful exercise of the right to
311

suffrage,  the present case also involves one’s right to property.


312 313

Respondents argue that it is the right of the state to prevent the circumvention of regulations relating
to election propaganda by applying such regulations to private individuals.  Certainly, any provision
314

or regulation can be circumvented. But we are not confronted with this possibility. Respondents
agree that the tarpaulin in question belongs to petitioners. Respondents have also agreed, during
the oral arguments, that petitioners were neither commissioned nor paid by any candidate or political
party to post the material on their walls.

Even though the tarpaulin is readily seen by the public, the tarpaulin remains the private property of
petitioners. Their right to use their property is likewise protected by the Constitution.

In Philippine Communications Satellite Corporation v. Alcuaz: 315


Any regulation, therefore, which operates as an effective confiscation of private property or
constitutes an arbitrary or unreasonable infringement of property rights is void, because it is
repugnant to the constitutional guaranties of due process and equal protection of the laws.  (Citation
316

omitted)

This court in Adiong held that a restriction that regulates where decals and stickers should be posted
is "so broad that it encompasses even the citizen’s private property."  Consequently, it violates
317

Article III, Section 1 of the Constitution which provides thatno person shall be deprived of his
property without due process of law. This court explained:

Property is more than the mere thing which a person owns, it includes the right to acquire, use, and
dispose of it; and the Constitution, in the 14th Amendment, protects these essential attributes.

Property is more than the mere thing which a person owns. It is elementary that it includes the right
to acquire, use, and dispose of it. The Constitution protects these essential attributes of property.
Holden v. Hardy, 169 U.S. 366, 391, 41 L. ed. 780, 790, 18 Sup. Ct. Rep. 383. Property consists of
the free use, enjoyment, and disposal of a person’s acquisitions without control or diminution save
by the law of the land. 1 Cooley’s Bl. Com. 127. (Buchanan v. Warley 245 US 60 [1917]) 318

This court ruled that the regulation in Adiong violates private property rights:

The right to property may be subject to a greater degree of regulation but when this right is joined by
a "liberty" interest, the burden of justification on the part of the Government must be exceptionally
convincing and irrefutable. The burden is not met in this case.

Section 11 of Rep. Act 6646 is so encompassing and invasive that it prohibits the posting or display
of election propaganda in any place, whether public or private, except inthe common poster areas
sanctioned by COMELEC. This means that a private person cannot post his own crudely prepared
personal poster on his own front dooror on a post in his yard. While the COMELEC will certainly
never require the absurd, there are no limits to what overzealous and partisan police officers, armed
with a copy of the statute or regulation, may do.  Respondents ordered petitioners, who are private
319

citizens, to remove the tarpaulin from their own property. The absurdity of the situation is in itself an
indication of the unconstitutionality of COMELEC’s interpretation of its powers.

Freedom of expression can be intimately related with the right to property. There may be no
expression when there is no place where the expression may be made. COMELEC’s infringement
upon petitioners’ property rights as in the present case also reaches out to infringement on their
fundamental right to speech.

Respondents have not demonstrated thatthe present state interest they seek to promote justifies the
intrusion into petitioners’ property rights. Election laws and regulations must be reasonable. It must
also acknowledge a private individual’s right to exercise property rights. Otherwise, the due process
clause will be violated.

COMELEC Resolution No. 9615 and the Fair Election Act intend to prevent the posting of election
propaganda in private property without the consent of the owners of such private property.
COMELEC has incorrectly implemented these regulations. Consistent with our ruling in Adiong, we
find that the act of respondents in seeking to restrain petitioners from posting the tarpaulin in their
own private property is an impermissible encroachments on the right to property.
V
Tarpaulin and its message are not religious speech

We proceed to the last issues pertaining to whether the COMELEC in issuing the questioned notice
and letter violated the right of petitioners to the free exercise of their religion.

At the outset, the Constitution mandates the separation of church and state.  This takes many
320

forms. Article III, Section 5 of the Constitution, for instance provides:

Section 5. No law shall be made respecting an establishment of religion, or prohibiting the free
exercise thereof. The free exercise and enjoyment of religious profession and worship, without
discrimination or preference, shall forever be allowed. Noreligious test shall be required for the
exercise of civil or political rights.

There are two aspects of this provision.  The first is the none stablishment clause.  Second is the
321 322

free exercise and enjoyment of religious profession and worship. 323

The second aspect is atissue in this case.

Clearly, not all acts done by those who are priests, bishops, ustadz, imams, or any other religious
make such act immune from any secular regulation.  The religious also have a secular existence.
324

They exist within a society that is regulated by law.

The Bishop of Bacolod caused the posting of the tarpaulin. But not all acts of a bishop amounts to
religious expression. This notwithstanding petitioners’ claim that "the views and position of the
petitioners, the Bishop and the Diocese of Bacolod, on the RH Bill is inextricably connected to its
Catholic dogma, faith, and moral teachings. . . ."325

The difficulty that often presents itself in these cases stems from the reality that every act can be
motivated by moral, ethical, and religious considerations. In terms of their effect on the corporeal
world, these acts range from belief, to expressions of these faiths, to religious ceremonies, and then
to acts of a secular character that may, from the point of view of others who do not share the same
faith or may not subscribe to any religion, may not have any religious bearing.

Definitely, the characterizations ofthe religious of their acts are not conclusive on this court.
Certainly, our powers of adjudication cannot be blinded by bare claims that acts are religious in
nature.

Petitioners erroneously relied on the case of Ebralinag v. The Division Superintendent of Schools of
Cebu  in claiming that the court "emphatically" held that the adherents ofa particular religion shall be
326

the ones to determine whether a particular matter shall be considered ecclesiastical in nature.  This
327

court in Ebralinagexempted Jehovah’s Witnesses from participating in the flag ceremony "out of
respect for their religious beliefs, [no matter how] "bizarre" those beliefsmay seem to others."  This
328

court found a balance between the assertion of a religious practice and the compelling necessities of
a secular command. It was an early attempt at accommodation of religious beliefs.

In Estrada v. Escritor,  this court adopted a policy of benevolent neutrality:


329

With religion looked upon with benevolence and not hostility, benevolent neutrality allows
accommodation of religion under certain circumstances. Accommodations are government policies
that take religion specifically intoaccount not to promote the government’s favored form of religion,
but to allow individuals and groups to exercise their religion without hindrance. Their purpose or
effect therefore is to remove a burden on, or facilitate the exercise of, a person’s or institution’s
religion. As Justice Brennan explained, the "government [may] take religion into account . . . to
exempt, when possible, from generally applicable governmental regulation individuals whose
religious beliefs and practices would otherwise thereby be infringed, or to create without state
involvement an atmosphere in which voluntary religious exercise may flourish." 330

This court also discussed the Lemon test in that case, such that a regulation is constitutional when:
(1) it has a secular legislative purpose; (2) it neither advances nor inhibits religion; and (3) it does not
foster an excessive entanglement with religion. 331

As aptly argued by COMELEC, however, the tarpaulin, on its face, "does not convey any religious
doctrine of the Catholic church."  That the position of the Catholic church appears to coincide with
332

the message of the tarpaulin regarding the RH Law does not, by itself, bring the expression within
the ambit of religious speech. On the contrary, the tarpaulin clearly refers to candidates classified
under "Team Patay" and "Team Buhay" according to their respective votes on the RH Law.

The same may be said of petitioners’ reliance on papal encyclicals to support their claim that the
expression onthe tarpaulin is an ecclesiastical matter. With all due respect to the Catholic faithful,
the church doctrines relied upon by petitioners are not binding upon this court. The position of the
Catholic religion in the Philippines as regards the RH Law does not suffice to qualify the posting by
one of its members of a tarpaulin as religious speech solely on such basis. The enumeration of
candidates on the face of the tarpaulin precludes any doubtas to its nature as speech with political
consequences and not religious speech.

Furthermore, the definition of an "ecclesiastical affair" in Austria v. National Labor Relations


Commission  cited by petitioners finds no application in the present case. The posting of the
333

tarpaulin does not fall within the category of matters that are beyond the jurisdiction of civil courts as
enumerated in the Austriacase such as "proceedings for excommunication, ordinations of religious
ministers, administration of sacraments and other activities withattached religious significance." 334

A FINAL NOTE

We maintain sympathies for the COMELEC in attempting to do what it thought was its duty in this
case. However, it was misdirected.

COMELEC’s general role includes a mandate to ensure equal opportunities and reduce spending
among candidates and their registered political parties. It is not to regulate or limit the speech of the
electorate as it strives to participate inthe electoral exercise.

The tarpaulin in question may be viewed as producing a caricature of those who are running for
public office.Their message may be construed generalizations of very complex individuals and party-
list organizations.

They are classified into black and white: as belonging to "Team Patay" or "Team Buhay."

But this caricature, though not agreeable to some, is still protected speech.

That petitioners chose to categorize them as purveyors of death or of life on the basis of a single
issue — and a complex piece of legislation at that — can easily be interpreted as anattempt to
stereo type the candidates and party-list organizations. Not all may agree to the way their thoughts
were expressed, as in fact there are other Catholic dioceses that chose not to follow the example of
petitioners.

Some may have thought that there should be more room to consider being more broad-minded and
non-judgmental. Some may have expected that the authors would give more space to practice
forgiveness and humility.

But, the Bill of Rights enumerated in our Constitution is an enumeration of our fundamental liberties.
It is not a detailed code that prescribes good conduct. It provides space for all to be guided by their
conscience, not only in the act that they do to others but also in judgment of the acts of others.

Freedom for the thought we can disagree with can be wielded not only by those in the minority. This
can often be expressed by dominant institutions, even religious ones. That they made their point
dramatically and in a large way does not necessarily mean that their statements are true, or that they
have basis, or that they have been expressed in good taste.

Embedded in the tarpaulin, however, are opinions expressed by petitioners. It is a specie of


expression protected by our fundamental law. It is an expression designed to invite attention, cause
debate, and hopefully, persuade. It may be motivated by the interpretation of petitioners of their
ecclesiastical duty, but their parishioner’s actions will have very real secular consequences.
Certainly, provocative messages do matter for the elections.

What is involved in this case is the most sacred of speech forms: expression by the electorate that
tends to rouse the public to debate contemporary issues. This is not speechby candidates or political
parties to entice votes. It is a portion of the electorate telling candidates the conditions for their
election. It is the substantive content of the right to suffrage.

This. is a form of speech hopeful of a quality of democracy that we should all deserve. It is protected
as a fundamental and primordial right by our Constitution. The expression in the medium chosen by
petitioners deserves our protection.

WHEREFORE, the instant petition is GRANTED. The temporary restraining order previously issued
is hereby made permanent. The act of the COMELEC in issuing the assailed notice dated February
22, 2013 and letter dated February 27, 2013 is declared unconstitutional.

SO ORDERED.

G.R. No. 207851               July 8, 2014

ANGEL G. NAVAL, Petitioner, 
vs.
COMMISSION ON ELECTIONS and NELSON B. JULIA, Respondents.

DECISION

REYES, J.:

A politician thinks of the next election –


a statesman of the next generation.

- James Freeman Clarke, American preacher and author

The Case

A provincial board member cannot be elected and serve for more than three consecutive terms. But
then, the Court is now called upon to resolve the following questions. First.What are the
consequences to the provincial board member’s eligibility to run for the same elective position if the
legislative district, which brought him orher to office to serve the first two consecutive terms, be
reapportioned in such a way that 8 out of its 10 town constituencies are carved out and renamed as
another district? Second. Is the provincial board member’s election to the same position for the third
and fourth time, but now in representation ofthe renamed district, a violation of the three-term limit
rule?

Before the Court is a Petition for Certiorariwith an Urgent Prayer for the Issuance of a Temporary
Restraining Order and a Writ of Preliminary Injunction  filed under Rule 64 of the Rules of Court to
1

assail the following resolutions of the public respondent Commission on Elections (COMELEC):

(a) Resolution  (first assailed resolution) issued by the Second Division on March 5, 2013, in SPA
2

No. 13-166 (DC), granting the petition filed by Nelson B. Julia (Julia), seeking to cancel the
Certificate of Candidacy  (COC) as Member of the Sangguniang Panlalawiganof Camarines Sur
3

(Sanggunian) of Angel G. Naval (Naval), who is allegedly violating the three-term limit imposed upon
elective local officials as provided for in Article X, Section 8  of the 1987 Constitution, and Section
4

43(b)  of the Local Government Code (LGC); and


5

(b) En BancResolution  (second assailed resolution) issued on June 5, 2013, denying Naval’s Motion
6

for Reconsideration  to the Resolution dated March 5, 2013.


7

Antecedents

From 2004 to 2007 and 2007 to 2010, Naval had been elected and had served as a member of the
Sanggunian, Second District, Province of Camarines Sur.

On October 12, 2009, the President approved Republic Act (R.A.) No. 9716,  which reapportioned
8

the legislative districts in Camarines Sur in the following manner:

[[reference - http://sc.judiciary.gov.ph/pdf/web/viewer.html?
file=/jurisprudence/2014/july2014/207851.pdf]]

District Before the Enactment of After the Enactment of

R.A. No. 9716 R.A. No. 9716

1st Libmanan, Minalabac, Del Gallego, Ragay, Lupi,

Pamplona, Pasacao, San Sipocot, Cabusao

Fernando, Del Gallego,


Ragay, Lupi, Sipocot,

Cabusao

2nd Naga City, Pili, Ocampo, Libmanan, Minalabac,

Camaligan, Canaman, Pamplona, Pasacao, San

Magarao, Bombon, Fernando, Gainza, Milaor

Calabanga,  Gainza,
9

Milaor

3rd Caramoan, Garchitorena, Naga City, Pili, Ocampo,

Goa, Lagonoy, Presentacion, Camaligan, Canaman,

Sangay, San Jose, Tigaon, Magarao, Bombon,

Tinambac, Siruma Calabanga

4th Iriga City, Baao, Balatan, Caramoan, Garchitorena,

Bato, Buhi, Bula, Nabua Goa, Lagonoy,

Presentacion, Sangay, San

Jose, Tigaon, Tinambac,

Siruma

5th   Iriga City, Baao, Balatan, Bato,


Buhi, Bula, Nabua

Notably, 8 out of 10 towns were taken from the old Second District to form the present Third District.
The present Second District is composed of the two remaining towns, Gainza and Milaor, merged
with five towns from the old First District.

In the 2010 elections, Naval once again won as among the members of the Sanggunian, Third
District. He served until 2013.

In the 2013 elections, Naval ran anewand was re-elected as Member of the Sanggunian, Third
District.

Julia was likewise a SanggunianMember candidate from the Third District in the 2013 elections. On
October 29, 2012, he invoked Section 78  of the Omnibus Election Code (OEC) and filed beforethe
10

COMELEC a Verified Petition to Deny Due Course or to Cancel the Certificate of Candidacy  of 11

Naval. Julia posited that Naval had fully served the entire Province of Camarines Sur for three
consecutive terms as a member of the Sanggunian, irrespective of the district he had been elected
from. The three-term limit rule’s application is more with reference to the same local elective post,
and not necessarily in connection with an identical territorial jurisdiction. Allowing Naval to run as a
Sanggunianmember for the fourth time is violative of the inflexible three-term limit rule enshrined in
the Constitution and the LGC, which must be strictly construed. 12

The Resolution of the COMELEC Second Division

In the first assailed resolution issued on March 5, 2013, the COMELEC Second Division cancelled
Naval’s COC on grounds stated below:

[W]hen a candidate for public office swears in his COC that he is eligible for the elective posts he
seeks, while, in reality, he knowingly lacks the necessary requirements for eligibility, he commits a
false material misrepresentation cognizable under Section 78 of the [OEC].

xxxx

The Supreme Court[,] in the case of Lonzanida v. [COMELEC][,] detailed the important components
of[Article X, Section 8 of the Constitution]:

This Court held that the two conditions for the application of the disqualification must concur: 1) that
the official concerned has been elected for three consecutive terms in the same local government
post and 2) that he has fully served three consecutive terms.It stated:

To recapitulate, the term limit for elective local officials must be taken to refer to the right to be
elected as well as the right to serve in the same elective position.

Consequently, it is not enough that an individual has servedthree consecutive terms in an elective
local office[;] he must also have been electedto the same position for the same number of times
before the disqualification can apply. x x x

x x x The first requisite does not only describe a candidate who has been elected for public office for
three consecutive terms. The candidate must have been elected in the samelocal government post.
This connotes that the candidate must have been inthe same elective position serving the same
constituency who elected him to office for three consecutive terms.

xxxx

The three-term limit rule was designed by the framers of the Constitution to prevent the monopoly of
power centered only on a chosen few. The said disqualification was primarily intended to forestall
the accumulation of massive political power by an elective local government official in a given locality
in order to perpetuate his tenure in office. The framers also considered the necessityof the
enhancement of the freedom of choice of the electorate by broadening the selection of would-be
elective public officers. By rendering ineligible for public office those who have been elected and
served for three consecutive terms in the same public elective post, the prohibition seeks to infuse
new blood in the political arena.

xxxx

x x x [T]he new Third District where [Naval] was elected and has served is composed of the same
municipalities comprising the previous Second District, absent the towns Gainza and [Milaor]. The
territorial jurisdiction [Naval] seeks to serve for the term 2013-2016 is the same as the territorial
jurisdiction he previously served. The electorate who voted for him in 2004, 2007 and 2010 isthe
same electorate who shall vote for him come May 13, 2013 Elections. They are the same group of
voters who elected him into office for three consecutive terms.

The resolution of this Commission in the case of Bandillo, et al[.] v. Hernandez (SPA No. 10-
078)  cannot be applied inthe case at bar. Hernandez who then hailed from Libmanan belonged to
13

the First District of Camarines Sur. With RepublicAct 9716, Libmanan, Minalabac, Pamplona,
Pasacao and San Fernando, all originally belonging to the First District, were merged with Gainza
and Milaor to form the Second District. With the addition of the municipalities of Gainza and Milaor, it
cannot be said that the previous First District became the Second District only by name. The voters
of Gainza and Milaoradded to the electorate of the new Second District formed a different electorate,
different from the one which voted for Hernandez in the 2001, 2004 and 2007 elections. In the case
at bar, the municipalities comprising the new Third District are the same municipalities that consisted
of the previous Second [District], absent Milaor and Gainza.

The Supreme Court, in Latasav. [COMELEC], ruled that the conversion of the municipality into a city
did not convert the office of the municipal mayor into a local government post different from the office
of the city mayor[.]

x x x x  (Citations omitted)
14

The Resolution of the COMELEC En Banc

In the second assailed resolution issued on June 5, 2013, the COMELEC en bancdenied Naval’s
Motion for Reconsideration to the above. The COMELEC pointed out thatabsent the verification
required under Section 3, Rule 19 of the COMELEC Rules of Procedure, Naval’s motion was
instantly dismissible. Nonetheless, the COMELEC proceeded to discuss the demerits of Naval’s
motion, viz:

The conditions for the application of the three-term limit rule are present in the instant case as the
records clearly establish that [Naval] is running for the 4th time for the same government post. To
put things in a proper perspective, it is imperative to review and discuss the salient points in the case
of Latasa v. [COMELEC]. The case involves the question of whether or not a municipal mayor,
having been elected and had already served for three (3) consecutive terms, canrun as city mayor in
light of the conversion of the municipality to a city. In applying the three-term limit rule, the Court
pointed out that the conversion of the municipality into a city did not convert the office of the
municipal mayor into a local government post different from the office of the city mayor. The Court
took into account the following circumstances: (1) That the territorial jurisdiction of [the] city was the
same as that of the municipality; (2) That the inhabitants were the same group of voters who elected
the municipal mayor for three (3) consecutive terms; and (3) That the inhabitants were the same
group of voters [over] whom he held power and authority as their chief executive for nine years.

Anchoring from the said case, it is therefore clear that the position to which [Naval] has filed his
candidacy for the 13 May 2013 x x x Elections is the same position for which he had been elected
and had served for the past nine (9) years.

xxxx

x x x The following circumstances establish that the subject posts are one and the same: First, the
territorial jurisdictions of the two (2) districts are the same except for the municipalities of Gainza and
Milaor which were excluded by R.A. No. 9716; Second, the inhabitants of the 3rd District of
Camarines Sur, where [Naval] is presently running as member of the [Sanggunian], are the same
voters who elected him for the past three (3) consecutive terms; and Lastly, the inhabitants of the
[3rd ] District are the same group of voters whom [Naval] had served as member of the [Sanggunian]
representing the 2nd District.

x x x The enactment of R.A. No. 9716 did not convert [Naval’s] post [into one] different from [w]hat
he [previously had]. As correctly ruled by the Commission (Second Division), [Naval] ha[d] already
been elected and ha[d] already served inthe same government post for three consecutive terms, x x
x[.]

x x x x.  (Citations omitted)
15

Unperturbed, Naval is now before the Court raising the issues of whether or not the COMELEC
gravely erred and ruled contrary to law and jurisprudence:

I. IN FINDING THAT NAVAL HAD ALREADY SERVED FOR THREE CONSECUTIVE TERMS IN
THE SAME GOVERNMENT POST; 16

II. IN IGNORING THE FACT THAT SANGGUNIAN MEMBERS ARE ELECTED BY LEGISLATIVE
DISTRICTS; and 17

III. WHEN IT RULED THAT THE PROHIBITION CONTEMPLATED BY SECTION 8, ARTICLE X OF


THE 1987 CONSTITUTION AND SECTION 43(B) OF THE LGC APPLIES TO NAVAL. 18

The Arguments of the Contending Parties

In support of the instant petition, Naval alleges that the First, Second and Third Legislative Districts
of Camarines Sur are not merely renamed but are composed of new sets of municipalities. With the
separation of Gainza and Milaor from the other eight towns which used to comprise the Second
District, the voters from the Third Legislative District are no longer the same ones as those who had
elected him to office in the 2004 and 2007 elections.

Naval further invokes Article 94  of Administrative Order No. 270 prescribing the Implementing Rules
19

and Regulations of the LGC to argue that Sanggunianmembers are elected by districts. Thus, the
right to choose representatives in the Sanggunianpertains to each of the districts. Naval was elected
as Sanggunian member in 2004 and 2007 by the Second District. In 2010 and 2013, it was the Third
District, which brought him to office. Essentially then, Naval’s election in 2013 is merely his second
term as Sanggunianmember for the Third District.

Naval likewise cites Borja, Jr. v. COMELEC  to point out that for the disqualification on the ground of
20

the three-term limit to apply, it is not enough that an individual has served three consecutive terms in
an elective local office, but it is also required that he or she had been elected to the same position
for the same number of times. 21

Naval also assails as erroneous the COMELEC’s interpretations of the rulings in Latasa v.
COMELEC  and Bandillo, et al. v. Hernandez.  In Latasa, the Court applied the three-term
22 23

prohibition only because notwithstanding the conversion of the Municipality of Digos into a city, the
mayor was to serve the same territorialjurisdiction and constituents. Naval asserts that the same
does not hold true in his case. Naval further avers that in Bandillo, which finds more application in
the instant petition, the COMELEC ruled that the three-term limit cannot be invoked in a situation
where the legislative districts have been altered. An extraction or an addition both yields a change
inthe composition of the voters.

Naval further emphasizes that he garnered the majority of the votes from his constituents, whose will
and mandate should be upheld. Besides, Julia’s counsel already withdrew his appearance,
indicating no less than his client’s lack of interest in still pursuing Naval’s ouster from office.
24

In its Comment,  the Office of the Solicitor General (OSG) seeks the denial of the instant petition.
25

The OSG contends that Naval had been elected and had fully served the same local elective post for
three consecutive terms. Naval thus violatedSection 78 of the OEC when he filed his COC despite
knowledge of his ineligibility. Naval’s reliance on Bandillo is also misplaced since in the said case,
two towns were instead added to form a new district. Apparently then, in Bandillo, there was a new
set of voters. The OSG also alleges that Naval is not entitled to the issuance of injunctive reliefs by
this Court. No clear and unmistakable right pertains to Naval and it is his eligibility to be elected as
Sanggunianmember for the Third District which is the issue at hand.

Ruling of the Court

The Court denies the petition.

As the issues are interrelated, they shall be discussed jointly.

The case before this Court is one of first impression. While the contending parties cite Latasa,
Lonzanida v. COMELEC,  Borja,Aldovino, Jr. v. COMELEC,  and Bandillo, which all involve the
26 27

application of the three-term limit rule, the factual and legal circumstances in those cases are
different and the doctrinal values therein do not directly address the issues now at hand.

In Latasa, the issue arose as a result of the conversion of a municipality into a city. The then
municipal mayor attempted to evade the application upon him of the three-term limit rule by arguing
that the position of a city mayor was not the same as the one he previously held. The Court was not
convinced and, thus, declared that there was no interruption of the incumbent mayor’s continuity of
service.

In Lonzanida, a candidate ran for the mayoralty post and won in three consecutive elections. While
serving his third term, his opponent filed an election protest. Months before the expiration of the
mayor’s third term, he was ousted from office. He ran again for the same post in the immediately
succeeding election. A petition was thereafter filed assailing his eligibility to run as mayor on the
ground of violation of the three-term limit rule. The Court ruled that the mayor could not
beconsidered as having served a full third term. An interruption for any length of time, if due to an
involuntary cause, is enough to break the elected official’s continuity of service.

In Borja, the mayor of Pateros died and was succeeded in office by the vice mayor. In the two
immediately succeeding elections, the latter vied for and won the mayoralty post. When he ran for
the same position for the third time, his disqualification was sought for alleged violation of the three-
term limit rule. The Court ruled that whenhe assumed the position of mayor by virtue of succession,
his service should not be treated as one full term. For the disqualification to apply, the candidate
should have been thrice elected for and had served the same post consecutively. In Aldovino,
preventive suspension was imposed upon an elected municipal councilor. The Court ruled that the
said suspension did not interrupt the elective official’s term. Although hewas barred from exercising
the functions of the position during the period of suspension, his continued stay and entitlement
tothe office remain unaffected.
In Bandillo, a case decided by the COMELEC, Gainza and Milaor were added to five of the ten
towns, which used to comprise Camarines Sur’s old First District, to form the new Second District.
The COMELEC declined to apply the three-term limit rule against the elected Provincial Board
member on the ground that the addition of Gainza and Milaor distinctively created a new district, with
an altered territory and constituency.

In the case before this Court, the task is to determine the application of the three-term limit rule upon
local elective officials in renamed and/or reapportioned districts. In the process of doing so, it is
inevitable to discuss the role of elections and the nature of public office in a democratic and
republican state like ours.

The Role of Elections in our

Democratic and Republican State,

and the Restraints Imposed Upon

Those Who Hold Public Office

The Court begins with general and undeniable principles.

The Philippines is a democratic and republican State. Sovereignty resides in the people and all
government authority emanates from them. 28

Then Associate Justice Reynato S. Puno explained the character of a republican state and a public
office, viz: A republic is a representative government, a government run by and for the people. It is
not a pure democracy where the people govern themselves directly. The essence of republicanism
is representation and renovation, the selection by the citizenry of a corps of public functionaries who
derive their mandate from the people and act on their behalf, serving for a limited period only, after
which they are replaced or retained, at the option of their principal. Obviously, a republican
government is a responsiblegovernment whose officials hold and discharge their position as a public
trust and shall, according to the Constitution, ‘at all times be accountable to the people’ they are
sworn to serve. The purpose of a republican government it is almost needless to state, is the
promotion of the common welfare according to the will of the people themselves.  (Emphasis ours
29

and italics in the original)

In Tolentino v. COMELEC,  Justice Puno likewise characterized the role of the electoral process in
30

the following wise:

The electoral process is one of the linchpins of a democratic and republican framework because it
isthrough the act of voting that government by consent is secured. Through the ballot, people
express their will on the defining issues of the day and they are able to choose their leaders in
accordance with the fundamental principle of representative democracy that the people should elect
whom they please to govern them. Voting has an important instrumental value in preserving the
viability of constitutional democracy. It has traditionally been taken as a prime indicator of democratic
participation.  (Citations omitted and italics ours)
31

The importance of elections cannottherefore be over emphasized. Thus,

True, election is the expression ofthe sovereign power of the people. In the exercise of suffrage, a
free people expects to achieve the continuity of government and the perpetuation of its benefits.
However, inspite of its importance, the privileges and rights arising from having been elected may be
enlarged or restricted by law. x x x. (Italics ours)
32

Hence, while it is settled that in elections, the first consideration of every democratic polity is to give
effect to the expressed will of the majority, there are limitations tobeing elected to a public
office.  Our Constitution and statutes are explicit anent the existence of term limits, the nature of
33

public office, and the guarantee from the State that citizens shall have equal access to public
service.  Section 8, Article X of our Constitution, on term limits, is significantly reiterated by Section
34

43(b) of the LGC. Moreover, the Court has time and again declared that a public office is a public
trust and not a vested property right. 35

The Deliberations of the Members

of the Constitutional Commission

on the Three-Term Limit’s

Application to Local Elective

Officials

Following are entries in the Journal of the Constitutional Commission regarding the exchanges of the
members on the subject of the three-term limit rule imposed on local elective officials: VOTING ON
THE TERMS OF LOCAL OFFICIALS

With respect to local officials, Mr. Nolledo, informed that the Committee on Local Governments had
not decided on the term of office for local officials and suggested that the Body decide on the matter.

xxxx

On Mr. Bacani’s inquiry regarding localofficials, Mr. Davide explained that local officials would
includethe governor, vice-governor and the members of the provincial board; the city mayor, city
vice-mayor and members of the city board; and the municipal mayor, municipal vice mayor and
members of the municipal council. He stated that barangay officials would be governed by
speciallaw, to which Mr. Nolledo agreed.

xxxx

MOTION TO VOTE ON THE PROPOSALS RELATIVE TO ALTERNATIVE NO. 3

In reply to Mr. Guingona’s query onwhether the Committee had decided on the interpretation of "two
reelections", Mr. Davide suggested that the matter be submitted to a vote.

Thereupon, Mr. Romulo moved for a vote on whether Alternative No. 3 as proposed by Mr. Garcia,
would allow a local official three terms, after which he would not be allowed to seek any reelection;
or whether, as interpreted by Mr. Davide, it would mean that after two successive reelections or a
consecutive periodof nine years, he could run for reelection after the lapse of three years.

xxxx

RESTATEMENT OF THE PROPOSALS


Mr. Garcia reiterated that the local officials could be reelected twice, after which, they would be
barred from ever runningfor reelection.

On the other proposal, Mr. Davide, on behalf ofthe Committee, stated that local officials after two
reelections would be allowed to run for reelection after the lapse of three years.

xxxx

MANIFESTATION OF MR. ROMULO

Upon resumption of session, Mr. Romulomanifested that the Body would proceed to the
consideration of two issues on the term of Representatives and local officials, namely: 1) Alternative
No. 1 (no further reelection after a total of three terms), and 2) Alternative No. 2 (no immediate
reelection after three successive terms).

SPONSORSHIP REMARKS OF MR. GARCIA ON ALTERNATIVE NO. 1

Mr. Garcia stated that he was advocating Alternative No. 1 on four grounds: 1) to prevent monopoly
of political power because the country’s history showed that prolonged stay in public office could
lead to the creation of entrenched preservesof political dynasties; 2) to broaden the choiceso that
more people could be enlisted to the cause of public service; 3) no one is indispensable in running
the affairs of the countryand that reliance on personalities would be avoided; and 4) the
disqualification from running for reelection after three terms would create a reserve of statesmen
both in the local and national levels.

He added that the turnover in public office after nine years would ensure the introduction of new
ideas and approaches. He stressed that public office would no longer be a preserve of conservatism
and tradition, and that public service would no longer be limited to those directly holding public office,
but would also include consultative bodiesorganized by the people. INQUIRY OF MR. REGALADO

In reply to Mr. Regalado’s query whether the three terms need not be served consecutively, Mr.
Garcia answered in the affirmative.

SPONSORSHIP REMARKS OF MR. MONSOD ON ALTERNATIVE NO. 2

Mr. Monsod stated that while the new Constitution would recognize people power because of a new
awareness, a new kind of voter and a new kind of Filipino, at the same time, it pre-screens the
candidates among whom the people would choose by barring those who would have served for nine
years from being reelected. He opined that this would actually require an additional qualification for
office to a certain number of people.

He stressed that, while the stand of the Commission is to create a reserve of statesmen, their future
participation is actually limited to some areas and only for a certain periodof time. He added thatit is
not for the Commission to decide on the future of our countrymen who may have more years ahead
of them to serve the country.

xxxx

INQUIRY OF MR. OPLE

xxxx
Thereupon, speaking in support of Mr. Monsod’s manifestation, Mr. Ople expressed apprehension
over the Body’s exercise of some sort of omnipotent power in disqualifying those who will have
served their tasks. He opined that the Commission had already taken steps to prevent the
accumulation of powers and prequisites that would permit officials to stay on indefinitely and to
transfer them to members of their families. He opined, however, that perpetual disqualification would
deprive the people of their freedom of choice.He stated that the Body had already succeeded in
striking a balance onpolicies which could ensure a redistribution of opportunities to the people both
in terms of political and economic power. He stated that Philippine politics had been unshackled
from the two-party system, which he said was the most critical support for the perpetuation of
political dynasties. Considering that such achievement is already a victory, Mr. Ople stated that the
role of political parties should not be despised because the strength of democracy depends on how
strong political parties are, that a splintering thereof will mean a great loss to the vitality and
resiliency of democracy.

Mr. Ople reiterated that he was against perpetual disqualification from office.

x x x x.

MR. GARCIA’S RESPONSE TOMR. OPLE’S STATEMENTS

Mr. Garcia stated that there are two principles involved in Alternative No. 1: 1) the recognition of the
ambivalent nature of political power, and 2) the recognition of alternative forms of public service. He
stated that it is important to remember the lessons learned from the recent past; that public service is
service to the people and not an opportunity to accumulate political power, and that a prolonged stay
in public office brings about political dynasties or vested interests. Regarding political parties, he
stated that it will encourage the constant renewal of blood in party leadership, approach, style and
ideas. He opined that this is very healthy for a pluralist and multi-party democracy.

On the recognition of alternative forms of public service, Mr. Garcia stressed that public service
could be limited to public office since many good leaders who were in the streets and in jail fought
against the dictatorship. He stressed that public service would also mean belonging to consultative
bodies or people’s councils which brought about new forms of service and leadership.

REMARKS OF MR. ABUBAKAR

Mr. Abubakar stated that in any democracy the voice of the people is the voice of God.He stated that
if the people want to elect a representative to serve them continuously, the Commission should not
arrogate unto itself the right to decide what the people want. He stated that in the United States, a
Senator had served for 30 years.

xxxx

REMARKS OF MS. AQUINO

Ms. Aquino stated that she differs from the views advanced by Mr. Garcia and Ms. Tan, although
she stated that they spoke of the same premises. She stated that she agrees with them that leaders
need not be projected and developed publicly in an election as leaders are better tempered and
tested in the various forms of mass struggles and organized work. She stated that if the people are
to be encouraged to have their own sense of responsibility in national leadership, what ultimately
matters is the political determination of the citizenry to chart their own national destiny. She opined
that the Body should allow the people to exercise their own sense of proportion and imbibe the
salutary effects of their own strength to curtail power when it overreaches itself. She stressed that in
the final analysis,the Commission cannot legislate into the Constitution the essence of new politics
as it is a chastening experience of learning and unlearning. Adverting to Mr. Garcia’s statement that
politics is an imperfect art, she stated that the Commission could correct politics with all its
imperfections and flaws by a constitutional provision. She opined that perpetual disqualification
cannot provide the cure. She maintained that perpetual disqualification is, at best, a palliative which
could also be counter-productive, in the sense that it could effectively foil the possibilities of
realpublic service.

REMARKS OF MR. BACANI

Mr. Bacani stated that when the Body granted the illiterates the right to vote and that proposals were
made to empower the people to engage in the legislative process,the Body presupposed the political
maturity of the people. He observed that in this instance, political maturity is denied with the
constitutional bar for reelection.He opined that the Body should stick to the premise that the people
are politically mature.

REJOINDER OF MR. GARCIA

By way of rejoinder to Mr. Bacani’s statements,Mr. Garcia stated that the proposal was basically
premisedon the undue advantage of the incumbent in accumulating power, money, party machine
and patronage and not on lack of trust in the people.

Mr. Garcia stated that politics isnot won by ideals alone but by solid organized work by
organizations. He stated that with three terms, an official would have served the people long enough.

xxxx

VOTING ON THE TWO ALTERNATIVES

Thereafter, the Body proceeded to vote by ballot on the two alternatives.

xxxx

RESULT OF THE VOTING

The result of the voting was as follows:

Alternative No. 1 (no further election after three successive terms) — 17 votes

Alternative No. 2 (no immediate reelection after three successive terms) — 26 votes

With 17 votes in favor of Alternative No. 1 and 26 in favor of Alternative No. 2, the Chair declared
Alternative No. 2 approved by the Body.  (Emphasis and italics ours)
36

The Constitution mandates the

strict implementation of the

three-term limit rule.


The Court notes that in the process of drafting the Constitution, the framers thereof had not
discussed with specifity the subject of the three-term limit rule’s application on reapportioned
districts.

From the above-cited deliberations, however, the divergent stances of the members of the
Constitutional Commission on the general application of the three-term limit rule show. On one side
were those who espoused the stern view that perpetual disqualification to hold public office after
three consecutive terms would ensure that new blood would be infused into our political system.
More choices for the voters would give fuller meaning to our democratic institutions. On the other
side of the fence were those who believed that the imposition of termlimits would be tantamount to
squandering the experience of seasoned public servants and a curtailment of the power of the
citizens to elect whoever they want to remain in office.

In the end, 26 members of the Commission cast their votes in favor of the proposal that no
immediate re-election after three successive terms shall be allowed. On the other hand, 17 members
stood pat on their view that there should be no further reelection after three successive terms.

Clearly, the drafters of our Constitution are in agreement about the possible attendant evils if there
would be no limit to re-election. Notwithstanding their conflicting preferences on whether the term
limit would disqualify the elected official perpetually or temporarily, they decided that only three
consecutive elections tothe same position would be allowed. Thereafter, the public official can once
again vie for the same post provided there be a gap of at least one term from his or her last election.
The rule answers the need to prevent the consolidation of political power in the hands of the few,
while at the same time giving to the people the freedom to call back to public service those who are
worthy to be called statesmen.

The compromise agreed upon by the drafters of our Constitution was a result of exhaustive
deliberations. The required gap after three consecutive elections is significant. Thus, the rulecannot
be taken with a grain of salt. Nothing less than its strict application is called for.

Ratio legis est anima. 37

"A foolproof yardstick in constitutional construction is the intention underlying the provision under
consideration.Thus, it has been held that the Court in construing a Constitution should bear in mind
the object sought to be accomplished by its adoption, and the evils, if any, sought to be prevented or
remedied. A doubtful provision will be examined in the light of the history of the times, and the
condition and circumstances under which the Constitution was framed. The object is to ascertain the
reason which induced the framers of the Constitution to enact the particular provision and the
purpose sought to be accomplished thereby, in order to construe the whole as to make the words
consonant to that reason and calculated to effect that purpose."  In Aldovino, the Court describes
38

the three-term limit rule as inflexible.

In Aldovino, a local elective official pleaded exemption from the application of the three-term limit on
the ground that there was an interruption in his service after the penalty of suspension was imposed
upon him. Although not in all four withNaval’s case, there are principles enunciated therein which
undeniably hold true, viz:

As worded, the constitutional provision fixes the term of a local elective office and limits an elective
official’s stay in office to no more than three consecutive terms. This is the first branch of the rule
embodied in Section 8, Article X.
Significantly, this provision refers to a "term" as a period of time – three years– during which an
official has title to office and can serve. x x x[.]

xxxx

The "limitation" under this first branch of the provision is expressed in the negative—"no such official
shall serve for more than three consecutive terms." This formulation—no more than three
consecutive terms—is a clear command suggesting the existence of an inflexible rule. x x x.

xxxx

This examination of the wording of the constitutional provision and of the circumstances surrounding
its formulation impresses upon us the clear intent to make term limitation a high priority constitutional
objective whose terms must be strictly construed and which cannot be defeated by, nor sacrificed
for, values of less than equal constitutional worth. x x x.

xxxx

x x x [T]he Court signalled how zealously it guards the three-term limit rule. Effectively, these cases
teach usto strictly interpret the term limitation rule in favor of limitation rather than its exception.

xxxx

[In] Latasa v. Commission on Electionsx x x[,] [t]he Court said:

This Court reiterates that the framers of the Constitution specifically included an exception to the
people’s freedom to choose those who will govern them in order to avoid the evil of a single person
accumulating excessive power over a particular territorial jurisdiction as a result of a prolonged stay
in the same office. x x x.

xxxx

To put it differently although at the risk of repetition, Section 8, Article X—both by structure and
substance—fixes an elective official’s term of office and limits his stay in office to three consecutive
terms as an inflexible rule that is stressed, no less, by citing voluntary renunciation as an example of
a circumvention. x x x.  (Citations omitted, italics and emphasis in the original and underscoring
39

ours)

Reapportionment and its Basis

Reapportionment is "the realignment orchange in legislative districts brought about by changes in


population and mandated by the constitutional requirement of equality of representation."  The aim
40

of legislative apportionment is to equalize population and voting power among districts.  The basis
41

for districting shall be the number of the inhabitants of a city or a province and not the number of
registered voters therein. 42

R.A. No. 9716 and the Reappor-

tioned Districts of Camarines Sur

Sections 1 to 3 of R.A. No. 9716 provide:


Section 1. The composition of the current First (1st) and Second (2nd) Legislative Districts in the
Province of Camarines Sur is hereby reapportioned in order to create an additional legislative
districtto commence in the next national elections after the effectivity of this Act.

Section 2. In furtherance of the reapportionment mandated by this Act, the municipalities of


Libmanan, Minalabac, Pamplona, Pasacao and San Fernando of the current First (1st) Legislative
District are hereby consolidated with the municipalities of Gainza and Milaor of the current Second
(2nd) Legislative District, to comprise the new legislative district authorized under this Act.

Section 3. The result of the reapportionment described in this Act are summarized as follows:

a) First District – The remaining municipalities in the current First (1st) Legislative District shall
continue to be designated as the First (1st) Legislative District, composed of the following
municipalities: Del Gallego, Ragay, Lupi, Sipicot and Cabusao;

b) Second District – This new legislative districtshall be composed of the municipalities enumerated
in Section 2 hereof;

c) Third District – The current Second (2nd) Legislative District shall be renamedas the Third (3rd)
Legislative District, composed of the following: Naga City and the municipalities of Pili, Ocampo,
Camaligan, Canaman, Magarao, Bombon and Calabanga;

d) Fourth District – The current Third (3rd) Legislative District, without any change in its composition,
shall be renamedas the Fourth (4th) Legislative District, composed of the following municipalities:
Caramoan, Garchitorena, Goa, Lagonoy, Presentacion, Sangay, San Jose, Tigaon, Tinambac and
Siruma; and

e) Fifth District – The current Fourth (4th) Legislative District, without any change inits composition,
shall be renamedas the Fifth (5th) Legislative District, composed of the following: Iriga City and the
municipalities of Baao, Balatan, Bato, Buhi, Bula and Nabua. (Italics and emphasis ours)

As a result of the reapportionment made by R.A. No. 9716, the old Second District of Camarines
Sur, minus only the two towns of Gainza and Milaor, is renamed as the Third District and now
configured as follows:43

[[reference - http://sc.judiciary.gov.ph/pdf/web/viewer.html?
file=/jurisprudence/2014/july2014/207851.pdf]]

Before the Enactment of After the Enactment of

RA 9716 RA 9716

2 3rd District

nd Population: 439,043

District Naga

Population: 474,899 Pili


Gainza Ocampo

Milaor Canaman

Naga Camaligan

Pili Magarao

Ocampo Bombon

Canaman Calabanga

Camaligan

Magarao

Bombon

Calabanga

R.A. No. 9716 created a new Second

District, but it merely renamed the

other four.

The Court notes that after the reapportionment of the districts in Camarines Sur, the current Third
District, which brought Naval to office in 2010 and 2013, has a population of 35,856 less than that of
the old Second District, which elected him in 2004 and 2007. However, the wordings of R.A. No.
9716 indicate the intent of the lawmakers to create a single new Second District from the merger of
the towns from the old First District with Gainza and Milaor. As to the current Third District, Section
3(c) of R.A. No. 9716 used the word "rename." Although the qualifier "without a change in its
composition" was not found in Section 3(c), unlike in Sections 3(d) and (e), still, what is pervasive
isthe clear intent to create a sole new district in that of the Second, while merely renaming the rest.

The following statutory construction rules surface:

First, the general rule in construing words and phrases used in a statute is that, in the absence of
legislative intent to the contrary, they should be given their plain, ordinary and common usage
meaning; the words should be read and considered intheir natural, ordinary, commonly accepted
usage, and without resorting to forced or subtle construction. Words are presumed to have been
employed by the lawmaker in their ordinary and common use and acceptation. Second, a word of
general significance ina statute is to be taken in its ordinary and comprehensive sense, unless it is
shown that the word is intended to be given a different or restricted meaning; what is generally
spoken shall be generally understood and general words shall be understood in a general
sense.  (Citations omitted)
44

The Court looks to the language of the document itself in our search for its meaning. 45
In Naval’s case, the words of R.A.No. 9716 plainly state that the new Second Districtis to be created,
but the Third Districtis to be renamed. Verba legis non est recedendum. The terms used in a legal
provision to be construed compels acceptanceand negates the power of the courts to alter it, based
on the postulate that the framers mean what they say. 46

The verb createmeans to "make or produce something new."  On the other hand, the verb
47

renamemeans to "give a new name to someone or something."  A complete reading of R.A. No.
48

9716 yields no logical conclusion other than that the lawmakers intended the old Second District to
be merely renamed as the current Third District.

It likewise bears noting that the actual difference in the population of the old Second District from
that of the current Third District amounts to less than 10% of the population of the latter. This
numericalfact renders the new Third District as essentially, although not literally, the same as the old
Second District. Hence, while Naval is correct in his argument that Sanggunianmembers are elected
by district, it does not alter the fact that the district which elected him for the third and fourth time is
the same one which brought him to office in 2004 and 2007.

The application upon Naval of the

three-term limit rule does not

undermine the constitutional

requirement to achieve equality of

representation among districts.

The rationale behind reapportionment is the constitutional requirement to achieve equality


ofrepresentation among the districts.  It is with this mindset that the Court should consider Naval’s
49

argument anent having a new set of constituents electing him into office in 2010 and 2013.

Naval’s ineligibility to run, by reason of violation of the three-term limit rule, does not undermine the
right toequal representation of any of the districts in Camarines Sur. With or without him, the
renamed Third District, which he labels as a new set of constituents, would still be represented,
albeit by another eligible person.

The presumed competence of the

COMELEC to resolve matters

falling within its jurisdiction is

upheld.

"Time and again, the Court has held that a petition for certiorariagainst actions of the COMELEC is
confined only to instances of grave abuse of discretion amounting to patent and substantial denial of
due process, because the COMELEC is presumed to be most competent in matters falling within its
domain." 50

"In a special civil action for certiorari, the burden rests on the petitioner to prove not merelyreversible
error, but grave abuse of discretion amounting to lack or excess of jurisdiction on the part of the
public respondent issuing the impugned order, decision or resolution."  "Grave abuse of discretion
51

arises when a court or tribunal violates the Constitution, the law or existing jurisprudence."
52

In the case at bar, the Court finds the COMELEC’s disquisitions to be amply supported by the
Constitution,law and jurisprudence.

Conclusion

In sum, the Court finds no compelling reason to grant the reliefs prayed for by Naval. For the Court
to declare otherwise would be to create a dangerous precedent unintended by the drafters of our
Constitution and of R.A. No. 9716. Considering that the one-term gap or rest after three consecutive
elections is a result of a compromise among the members of the Constitutional Commission, no
cavalier exemptions or exceptions to its application is to be allowed. Aldovinoaffirms this
interpretation. Further, sustaining Naval’s arguments would practically allow him to hold the same
office for 15 years. These are the circumstances the Constitution explicitly intends to avert.

Certainly, the Court accords primacy to upholding the will of the voting public, the real sovereign,
soto speak. However, let all the candidates for public office be reminded that as citizens, we have a
commitment to be bound by our Constitution and laws. Side by side our privileges as citizens are
restrictions too.

Einer Elhauge, a faculty member from Harvard Law School, wrote an article entitled "What Term
Limits Do That Ordinary Voting Cannot."  In the article, Greek mythology was tapped to make a
53

tempting analogy. The gist of the story follows.

In Odyssey Book XII, the goddess Circe warned Odysseus of the Sirens who seduce all men
approaching them with their voices. Those who fell into the Sirens’ trap never returnedhome to their
wives and children. A clever strategy was thus hatched to secure safe passage for Odysseus and
his men. The men were to plug their ears with wax to muffle the songs of the Sirens. Odysseus, on
the other hand, was to be tied to the mast of the ship so he could still listen to the songs, which may
contain clues on how they can get home. When the wind died down,Odysseus heard beautiful
voices calling out to them. The voices were incomparable to anything he had ever heard before.
Even whenOdysseus knew that the irresistible voices were coming from the Sirens, he struggled
with all his strength to free himself from the ropes, but was unable to do so. The voices became
fainter as the men continued to row. When the voices can no longer be heard, Odysseus realized
how he had nearly been beguiled. They had made it through safely and Odysseus was untied. It was
their clever plan which kept them all alive.
54

The same lesson holds true in the case before this Court. The drafters of the Constitution recognized
the propensity of public officers to perpetuate themselves in power, hence, the adoption of term
limits and a guarantee of every citizen's equal access to public service. These are the restrictions
statesmen should observe for they are intended to help ensure the continued vitality of our
republican institutions.

WHEREFORE, IN VIEW OF THE FOREGOING, the petition is DENIED. The Resolutions dated
March 5, 2013 and June 5, 2013 of the Commission on Elections in SPA No. 13-166 (DC) are
AFFIRMED.

SO ORDERED.
G.R. No. 101083 July 30, 1993

JUAN ANTONIO, ANNA ROSARIO and JOSE ALFONSO, all surnamed OPOSA, minors, and
represented by their parents ANTONIO and RIZALINA OPOSA, ROBERTA NICOLE SADIUA,
minor, represented by her parents CALVIN and ROBERTA SADIUA, CARLO, AMANDA SALUD
and PATRISHA, all surnamed FLORES, minors and represented by their parents ENRICO and
NIDA FLORES, GIANINA DITA R. FORTUN, minor, represented by her parents SIGRID and
DOLORES FORTUN, GEORGE II and MA. CONCEPCION, all surnamed MISA, minors and
represented by their parents GEORGE and MYRA MISA, BENJAMIN ALAN V. PESIGAN,
minor, represented by his parents ANTONIO and ALICE PESIGAN, JOVIE MARIE ALFARO,
minor, represented by her parents JOSE and MARIA VIOLETA ALFARO, MARIA
CONCEPCION T. CASTRO, minor, represented by her parents FREDENIL and JANE CASTRO,
JOHANNA DESAMPARADO, 
minor, represented by her parents JOSE and ANGELA DESAMPRADO, CARLO JOAQUIN T.
NARVASA, minor, represented by his parents GREGORIO II and CRISTINE CHARITY
NARVASA, MA. MARGARITA, JESUS IGNACIO, MA. ANGELA and MARIE GABRIELLE, all
surnamed SAENZ, minors, represented by their parents ROBERTO and AURORA SAENZ,
KRISTINE, MARY ELLEN, MAY, GOLDA MARTHE and DAVID IAN, all surnamed KING, minors,
represented by their parents MARIO and HAYDEE KING, DAVID, FRANCISCO and THERESE
VICTORIA, all surnamed ENDRIGA, minors, represented by their parents BALTAZAR and
TERESITA ENDRIGA, JOSE MA. and REGINA MA., all surnamed ABAYA, minors, represented
by their parents ANTONIO and MARICA ABAYA, MARILIN, MARIO, JR. and MARIETTE, all
surnamed CARDAMA, minors, represented by their parents MARIO and LINA CARDAMA,
CLARISSA, ANN MARIE, NAGEL, and IMEE LYN, all surnamed OPOSA, minors and
represented by their parents RICARDO and MARISSA OPOSA, PHILIP JOSEPH, STEPHEN
JOHN and ISAIAH JAMES, all surnamed QUIPIT, minors, represented by their parents JOSE
MAX and VILMI QUIPIT, BUGHAW CIELO, CRISANTO, ANNA, DANIEL and FRANCISCO, all
surnamed BIBAL, minors, represented by their parents FRANCISCO, JR. and MILAGROS
BIBAL, and THE PHILIPPINE ECOLOGICAL NETWORK, INC., petitioners, 
vs.
THE HONORABLE FULGENCIO S. FACTORAN, JR., in his capacity as the Secretary of the
Department of Environment and Natural Resources, and THE HONORABLE ERIBERTO U.
ROSARIO, Presiding Judge of the RTC, Makati, Branch 66, respondents.

Oposa Law Office for petitioners.

The Solicitor General for respondents.

DAVIDE, JR., J.:

In a broader sense, this petition bears upon the right of Filipinos to a balanced and healthful ecology
which the petitioners dramatically associate with the twin concepts of "inter-generational
responsibility" and "inter-generational justice." Specifically, it touches on the issue of whether the
said petitioners have a cause of action to "prevent the misappropriation or impairment" of Philippine
rainforests and "arrest the unabated hemorrhage of the country's vital life support systems and
continued rape of Mother Earth."

The controversy has its genesis in Civil Case No. 90-77 which was filed before Branch 66 (Makati,
Metro Manila) of the Regional Trial Court (RTC), National Capital Judicial Region. The principal
plaintiffs therein, now the principal petitioners, are all minors duly represented and joined by their
respective parents. Impleaded as an additional plaintiff is the Philippine Ecological Network, Inc.
(PENI), a domestic, non-stock and non-profit corporation organized for the purpose of, inter alia,
engaging in concerted action geared for the protection of our environment and natural resources.
The original defendant was the Honorable Fulgencio S. Factoran, Jr., then Secretary of the
Department of Environment and Natural Resources (DENR). His substitution in this petition by the
new Secretary, the Honorable Angel C. Alcala, was subsequently ordered upon proper motion by the
petitioners.  The complaint  was instituted as a taxpayers' class suit  and alleges that the plaintiffs
1 2 3

"are all citizens of the Republic of the Philippines, taxpayers, and entitled to the full benefit, use and
enjoyment of the natural resource treasure that is the country's virgin tropical forests." The same
was filed for themselves and others who are equally concerned about the preservation of said
resource but are "so numerous that it is impracticable to bring them all before the Court." The minors
further asseverate that they "represent their generation as well as generations yet
unborn."  Consequently, it is prayed for that judgment be rendered:
4

. . . ordering defendant, his agents, representatives and other persons acting in his
behalf to —

(1) Cancel all existing timber license agreements in the country;

(2) Cease and desist from receiving, accepting, processing, renewing or approving
new timber license agreements.

and granting the plaintiffs ". . . such other reliefs just and equitable under the premises." 5

The complaint starts off with the general averments that the Philippine archipelago of 7,100 islands
has a land area of thirty million (30,000,000) hectares and is endowed with rich, lush and verdant
rainforests in which varied, rare and unique species of flora and fauna may be found; these
rainforests contain a genetic, biological and chemical pool which is irreplaceable; they are also the
habitat of indigenous Philippine cultures which have existed, endured and flourished since time
immemorial; scientific evidence reveals that in order to maintain a balanced and healthful ecology,
the country's land area should be utilized on the basis of a ratio of fifty-four per cent (54%) for forest
cover and forty-six per cent (46%) for agricultural, residential, industrial, commercial and other uses;
the distortion and disturbance of this balance as a consequence of deforestation have resulted in a
host of environmental tragedies, such as (a) water shortages resulting from drying up of the water
table, otherwise known as the "aquifer," as well as of rivers, brooks and streams, (b) salinization of
the water table as a result of the intrusion therein of salt water, incontrovertible examples of which
may be found in the island of Cebu and the Municipality of Bacoor, Cavite, (c) massive erosion and
the consequential loss of soil fertility and agricultural productivity, with the volume of soil eroded
estimated at one billion (1,000,000,000) cubic meters per annum — approximately the size of the
entire island of Catanduanes, (d) the endangering and extinction of the country's unique, rare and
varied flora and fauna, (e) the disturbance and dislocation of cultural communities, including the
disappearance of the Filipino's indigenous cultures, (f) the siltation of rivers and seabeds and
consequential destruction of corals and other aquatic life leading to a critical reduction in marine
resource productivity, (g) recurrent spells of drought as is presently experienced by the entire
country, (h) increasing velocity of typhoon winds which result from the absence of windbreakers, (i)
the floodings of lowlands and agricultural plains arising from the absence of the absorbent
mechanism of forests, (j) the siltation and shortening of the lifespan of multi-billion peso dams
constructed and operated for the purpose of supplying water for domestic uses, irrigation and the
generation of electric power, and (k) the reduction of the earth's capacity to process carbon dioxide
gases which has led to perplexing and catastrophic climatic changes such as the phenomenon of
global warming, otherwise known as the "greenhouse effect."
Plaintiffs further assert that the adverse and detrimental consequences of continued and
deforestation are so capable of unquestionable demonstration that the same may be submitted as a
matter of judicial notice. This notwithstanding, they expressed their intention to present expert
witnesses as well as documentary, photographic and film evidence in the course of the trial.

As their cause of action, they specifically allege that:

CAUSE OF ACTION

7. Plaintiffs replead by reference the foregoing allegations.

8. Twenty-five (25) years ago, the Philippines had some sixteen (16) million hectares
of rainforests constituting roughly 53% of the country's land mass.

9. Satellite images taken in 1987 reveal that there remained no more than 1.2 million
hectares of said rainforests or four per cent (4.0%) of the country's land area.

10. More recent surveys reveal that a mere 850,000 hectares of virgin old-growth
rainforests are left, barely 2.8% of the entire land mass of the Philippine archipelago
and about 3.0 million hectares of immature and uneconomical secondary growth
forests.

11. Public records reveal that the defendant's, predecessors have granted timber
license agreements ('TLA's') to various corporations to cut the aggregate area of
3.89 million hectares for commercial logging purposes.

A copy of the TLA holders and the corresponding areas covered is hereto attached
as Annex "A".

12. At the present rate of deforestation, i.e. about 200,000 hectares per annum or 25


hectares per hour — nighttime, Saturdays, Sundays and holidays included — the
Philippines will be bereft of forest resources after the end of this ensuing decade, if
not earlier.

13. The adverse effects, disastrous consequences, serious injury and irreparable
damage of this continued trend of deforestation to the plaintiff minor's generation and
to generations yet unborn are evident and incontrovertible. As a matter of fact, the
environmental damages enumerated in paragraph 6 hereof are already being felt,
experienced and suffered by the generation of plaintiff adults.

14. The continued allowance by defendant of TLA holders to cut and deforest the
remaining forest stands will work great damage and irreparable injury to plaintiffs —
especially plaintiff minors and their successors — who may never see, use, benefit
from and enjoy this rare and unique natural resource treasure.

This act of defendant constitutes a misappropriation and/or impairment of the natural


resource property he holds in trust for the benefit of plaintiff minors and succeeding
generations.

15. Plaintiffs have a clear and constitutional right to a balanced and healthful ecology
and are entitled to protection by the State in its capacity as the parens patriae.
16. Plaintiff have exhausted all administrative remedies with the defendant's office.
On March 2, 1990, plaintiffs served upon defendant a final demand to cancel all
logging permits in the country.

A copy of the plaintiffs' letter dated March 1, 1990 is hereto attached as Annex "B".

17. Defendant, however, fails and refuses to cancel the existing TLA's to the
continuing serious damage and extreme prejudice of plaintiffs.

18. The continued failure and refusal by defendant to cancel the TLA's is an act
violative of the rights of plaintiffs, especially plaintiff minors who may be left with a
country that is desertified (sic), bare, barren and devoid of the wonderful flora, fauna
and indigenous cultures which the Philippines had been abundantly blessed with.

19. Defendant's refusal to cancel the aforementioned TLA's is manifestly contrary to


the public policy enunciated in the Philippine Environmental Policy which, in pertinent
part, states that it is the policy of the State —

(a) to create, develop, maintain and improve conditions under which man and nature
can thrive in productive and enjoyable harmony with each other;

(b) to fulfill the social, economic and other requirements of present and future
generations of Filipinos and;

(c) to ensure the attainment of an environmental quality that is conductive to a life of


dignity and well-being. (P.D. 1151, 6 June 1977)

20. Furthermore, defendant's continued refusal to cancel the aforementioned TLA's


is contradictory to the Constitutional policy of the State to —

a. effect "a more equitable distribution of opportunities, income and wealth" and
"make full and efficient use of natural resources (sic)." (Section 1, Article XII of the
Constitution);

b. "protect the nation's marine wealth." (Section 2, ibid);

c. "conserve and promote the nation's cultural heritage and resources (sic)" (Section
14, Article XIV, id.);

d. "protect and advance the right of the people to a balanced and healthful ecology in
accord with the rhythm and harmony of nature." (Section 16, Article II, id.)

21. Finally, defendant's act is contrary to the highest law of humankind — the natural
law — and violative of plaintiffs' right to self-preservation and perpetuation.

22. There is no other plain, speedy and adequate remedy in law other than the
instant action to arrest the unabated hemorrhage of the country's vital life support
systems and continued rape of Mother Earth.  6

On 22 June 1990, the original defendant, Secretary Factoran, Jr., filed a Motion to Dismiss the
complaint based on two (2) grounds, namely: (1) the plaintiffs have no cause of action against him
and (2) the issue raised by the plaintiffs is a political question which properly pertains to the
legislative or executive branches of Government. In their 12 July 1990 Opposition to the Motion, the
petitioners maintain that (1) the complaint shows a clear and unmistakable cause of action, (2) the
motion is dilatory and (3) the action presents a justiciable question as it involves the defendant's
abuse of discretion.

On 18 July 1991, respondent Judge issued an order granting the aforementioned motion to
dismiss.  In the said order, not only was the defendant's claim — that the complaint states no cause
7

of action against him and that it raises a political question — sustained, the respondent Judge further
ruled that the granting of the relief prayed for would result in the impairment of contracts which is
prohibited by the fundamental law of the land.

Plaintiffs thus filed the instant special civil action for certiorari under Rule 65 of the Revised Rules of
Court and ask this Court to rescind and set aside the dismissal order on the ground that the
respondent Judge gravely abused his discretion in dismissing the action. Again, the parents of the
plaintiffs-minors not only represent their children, but have also joined the latter in this case.
8

On 14 May 1992, We resolved to give due course to the petition and required the parties to submit
their respective Memoranda after the Office of the Solicitor General (OSG) filed a Comment in behalf
of the respondents and the petitioners filed a reply thereto.

Petitioners contend that the complaint clearly and unmistakably states a cause of action as it
contains sufficient allegations concerning their right to a sound environment based on Articles 19, 20
and 21 of the Civil Code (Human Relations), Section 4 of Executive Order (E.O.) No. 192 creating
the DENR, Section 3 of Presidential Decree (P.D.) No. 1151 (Philippine Environmental Policy),
Section 16, Article II of the 1987 Constitution recognizing the right of the people to a balanced and
healthful ecology, the concept of generational genocide in Criminal Law and the concept of man's
inalienable right to self-preservation and self-perpetuation embodied in natural law. Petitioners
likewise rely on the respondent's correlative obligation per Section 4 of E.O. No. 192, to safeguard
the people's right to a healthful environment.

It is further claimed that the issue of the respondent Secretary's alleged grave abuse of discretion in
granting Timber License Agreements (TLAs) to cover more areas for logging than what is available
involves a judicial question.

Anent the invocation by the respondent Judge of the Constitution's non-impairment clause,
petitioners maintain that the same does not apply in this case because TLAs are not contracts. They
likewise submit that even if TLAs may be considered protected by the said clause, it is well settled
that they may still be revoked by the State when the public interest so requires.

On the other hand, the respondents aver that the petitioners failed to allege in their complaint a
specific legal right violated by the respondent Secretary for which any relief is provided by law. They
see nothing in the complaint but vague and nebulous allegations concerning an "environmental right"
which supposedly entitles the petitioners to the "protection by the state in its capacity as parens
patriae." Such allegations, according to them, do not reveal a valid cause of action. They then
reiterate the theory that the question of whether logging should be permitted in the country is a
political question which should be properly addressed to the executive or legislative branches of
Government. They therefore assert that the petitioners' resources is not to file an action to court, but
to lobby before Congress for the passage of a bill that would ban logging totally.

As to the matter of the cancellation of the TLAs, respondents submit that the same cannot be done
by the State without due process of law. Once issued, a TLA remains effective for a certain period of
time — usually for twenty-five (25) years. During its effectivity, the same can neither be revised nor
cancelled unless the holder has been found, after due notice and hearing, to have violated the terms
of the agreement or other forestry laws and regulations. Petitioners' proposition to have all the TLAs
indiscriminately cancelled without the requisite hearing would be violative of the requirements of due
process.

Before going any further, We must first focus on some procedural matters. Petitioners instituted Civil
Case No. 90-777 as a class suit. The original defendant and the present respondents did not take
issue with this matter. Nevertheless, We hereby rule that the said civil case is indeed a class suit.
The subject matter of the complaint is of common and general interest not just to several, but to all
citizens of the Philippines. Consequently, since the parties are so numerous, it, becomes
impracticable, if not totally impossible, to bring all of them before the court. We likewise declare that
the plaintiffs therein are numerous and representative enough to ensure the full protection of all
concerned interests. Hence, all the requisites for the filing of a valid class suit under Section 12, Rule
3 of the Revised Rules of Court are present both in the said civil case and in the instant petition, the
latter being but an incident to the former.

This case, however, has a special and novel element. Petitioners minors assert that they represent
their generation as well as generations yet unborn. We find no difficulty in ruling that they can, for
themselves, for others of their generation and for the succeeding generations, file a class suit. Their
personality to sue in behalf of the succeeding generations can only be based on the concept of
intergenerational responsibility insofar as the right to a balanced and healthful ecology is concerned.
Such a right, as hereinafter expounded, considers 
the "rhythm and harmony of nature." Nature means the created world in its entirety.  Such rhythm
9

and harmony indispensably include, inter alia, the judicious disposition, utilization, management,
renewal and conservation of the country's forest, mineral, land, waters, fisheries, wildlife, off-shore
areas and other natural resources to the end that their exploration, development and utilization be
equitably accessible to the present as well as future generations.  Needless to say, every generation
10

has a responsibility to the next to preserve that rhythm and harmony for the full enjoyment of a
balanced and healthful ecology. Put a little differently, the minors' assertion of their right to a sound
environment constitutes, at the same time, the performance of their obligation to ensure the
protection of that right for the generations to come.

The locus standi of the petitioners having thus been addressed, We shall now proceed to the merits
of the petition.

After a careful perusal of the complaint in question and a meticulous consideration and evaluation of
the issues raised and arguments adduced by the parties, We do not hesitate to find for the
petitioners and rule against the respondent Judge's challenged order for having been issued with
grave abuse of discretion amounting to lack of jurisdiction. The pertinent portions of the said order
reads as follows:

xxx xxx xxx

After a careful and circumspect evaluation of the Complaint, the Court cannot help
but agree with the defendant. For although we believe that plaintiffs have but the
noblest of all intentions, it (sic) fell short of alleging, with sufficient definiteness, a
specific legal right they are seeking to enforce and protect, or a specific legal wrong
they are seeking to prevent and redress (Sec. 1, Rule 2, RRC). Furthermore, the
Court notes that the Complaint is replete with vague assumptions and vague
conclusions based on unverified data. In fine, plaintiffs fail to state a cause of action
in its Complaint against the herein defendant.
Furthermore, the Court firmly believes that the matter before it, being impressed with
political color and involving a matter of public policy, may not be taken cognizance of
by this Court without doing violence to the sacred principle of "Separation of Powers"
of the three (3) co-equal branches of the Government.

The Court is likewise of the impression that it cannot, no matter how we stretch our
jurisdiction, grant the reliefs prayed for by the plaintiffs, i.e., to cancel all existing
timber license agreements in the country and to cease and desist from receiving,
accepting, processing, renewing or approving new timber license agreements. For to
do otherwise would amount to "impairment of contracts" abhored (sic) by the
fundamental law.  11

We do not agree with the trial court's conclusions that the plaintiffs failed to allege with sufficient
definiteness a specific legal right involved or a specific legal wrong committed, and that the
complaint is replete with vague assumptions and conclusions based on unverified data. A reading of
the complaint itself belies these conclusions.

The complaint focuses on one specific fundamental legal right — the right to a balanced and
healthful ecology which, for the first time in our nation's constitutional history, is solemnly
incorporated in the fundamental law. Section 16, Article II of the 1987 Constitution explicitly provides:

Sec. 16. The State shall protect and advance the right of the people to a balanced
and healthful ecology in accord with the rhythm and harmony of nature.

This right unites with the right to health which is provided for in the preceding section
of the same article:

Sec. 15. The State shall protect and promote the right to health of the people and
instill health consciousness among them.

While the right to a balanced and healthful ecology is to be found under the Declaration of Principles
and State Policies and not under the Bill of Rights, it does not follow that it is less important than any
of the civil and political rights enumerated in the latter. Such a right belongs to a different category of
rights altogether for it concerns nothing less than self-preservation and self-perpetuation — aptly and
fittingly stressed by the petitioners — the advancement of which may even be said to predate all
governments and constitutions. As a matter of fact, these basic rights need not even be written in the
Constitution for they are assumed to exist from the inception of humankind. If they are now explicitly
mentioned in the fundamental charter, it is because of the well-founded fear of its framers that
unless the rights to a balanced and healthful ecology and to health are mandated as state policies by
the Constitution itself, thereby highlighting their continuing importance and imposing upon the state a
solemn obligation to preserve the first and protect and advance the second, the day would not be too
far when all else would be lost not only for the present generation, but also for those to come —
generations which stand to inherit nothing but parched earth incapable of sustaining life.

The right to a balanced and healthful ecology carries with it the correlative duty to refrain from
impairing the environment. During the debates on this right in one of the plenary sessions of the
1986 Constitutional Commission, the following exchange transpired between Commissioner Wilfrido
Villacorta and Commissioner Adolfo Azcuna who sponsored the section in question:

MR. VILLACORTA:
Does this section mandate the State to provide sanctions against all
forms of pollution — air, water and noise pollution?

MR. AZCUNA:

Yes, Madam President. The right to healthful (sic) environment


necessarily carries with it the correlative duty of not impairing the
same and, therefore, sanctions may be provided for impairment of
environmental balance.  12

The said right implies, among many other things, the judicious management and conservation of the
country's forests.

Without such forests, the ecological or environmental balance would be irreversiby disrupted.

Conformably with the enunciated right to a balanced and healthful ecology and the right to health, as
well as the other related provisions of the Constitution concerning the conservation, development
and utilization of the country's natural resources,   then President Corazon C. Aquino promulgated
13

on 10 June 1987 E.O. No. 192,   Section 4 of which expressly mandates that the Department of
14

Environment and Natural Resources "shall be the primary government agency responsible for the
conservation, management, development and proper use of the country's environment and natural
resources, specifically forest and grazing lands, mineral, resources, including those in reservation
and watershed areas, and lands of the public domain, as well as the licensing and regulation of all
natural resources as may be provided for by law in order to ensure equitable sharing of the benefits
derived therefrom for the welfare of the present and future generations of Filipinos." Section 3
thereof makes the following statement of policy:

Sec. 3. Declaration of Policy. — It is hereby declared the policy of the State to ensure
the sustainable use, development, management, renewal, and conservation of the
country's forest, mineral, land, off-shore areas and other natural resources, including
the protection and enhancement of the quality of the environment, and equitable
access of the different segments of the population to the development and the use of
the country's natural resources, not only for the present generation but for future
generations as well. It is also the policy of the state to recognize and apply a true
value system including social and environmental cost implications relative to their
utilization, development and conservation of our natural resources.

This policy declaration is substantially re-stated it Title XIV, Book IV of the Administrative Code of
1987,  specifically in Section 1 thereof which reads:
15

Sec. 1. Declaration of Policy. — (1) The State shall ensure, for the benefit of the
Filipino people, the full exploration and development as well as the judicious
disposition, utilization, management, renewal and conservation of the country's
forest, mineral, land, waters, fisheries, wildlife, off-shore areas and other natural
resources, consistent with the necessity of maintaining a sound ecological balance
and protecting and enhancing the quality of the environment and the objective of
making the exploration, development and utilization of such natural resources
equitably accessible to the different segments of the present as well as future
generations.
(2) The State shall likewise recognize and apply a true value system that takes into
account social and environmental cost implications relative to the utilization,
development and conservation of our natural resources.

The above provision stresses "the necessity of maintaining a sound ecological balance and
protecting and enhancing the quality of the environment." Section 2 of the same Title, on the other
hand, specifically speaks of the mandate of the DENR; however, it makes particular reference to the
fact of the agency's being subject to law and higher authority. Said section provides:

Sec. 2. Mandate. — (1) The Department of Environment and Natural Resources shall
be primarily responsible for the implementation of the foregoing policy.

(2) It shall, subject to law and higher authority, be in charge of carrying out the
State's constitutional mandate to control and supervise the exploration, development,
utilization, and conservation of the country's natural resources.

Both E.O. NO. 192 and the Administrative Code of 1987 have set the objectives which will serve as
the bases for policy formulation, and have defined the powers and functions of the DENR.

It may, however, be recalled that even before the ratification of the 1987 Constitution, specific
statutes already paid special attention to the "environmental right" of the present and future
generations. On 6 June 1977, P.D. No. 1151 (Philippine Environmental Policy) and P.D. No. 1152
(Philippine Environment Code) were issued. The former "declared a continuing policy of the State (a)
to create, develop, maintain and improve conditions under which man and nature can thrive in
productive and enjoyable harmony with each other, (b) to fulfill the social, economic and other
requirements of present and future generations of Filipinos, and (c) to insure the attainment of an
environmental quality that is conducive to a life of dignity and well-being."   As its goal, it speaks of
16

the "responsibilities of each generation as trustee and guardian of the environment for succeeding
generations."   The latter statute, on the other hand, gave flesh to the said policy.
17

Thus, the right of the petitioners (and all those they represent) to a balanced and healthful ecology is
as clear as the DENR's duty — under its mandate and by virtue of its powers and functions under
E.O. No. 192 and the Administrative Code of 1987 — to protect and advance the said right.

A denial or violation of that right by the other who has the corelative duty or obligation to respect or
protect the same gives rise to a cause of action. Petitioners maintain that the granting of the TLAs,
which they claim was done with grave abuse of discretion, violated their right to a balanced and
healthful ecology; hence, the full protection thereof requires that no further TLAs should be renewed
or granted.

A cause of action is defined as:

. . . an act or omission of one party in violation of the legal right or rights of the other;
and its essential elements are legal right of the plaintiff, correlative obligation of the
defendant, and act or omission of the defendant in violation of said legal right.  18

It is settled in this jurisdiction that in a motion to dismiss based on the ground that the complaint fails
to state a cause of action,   the question submitted to the court for resolution involves the sufficiency
19

of the facts alleged in the complaint itself. No other matter should be considered; furthermore, the
truth of falsity of the said allegations is beside the point for the truth thereof is deemed hypothetically
admitted. The only issue to be resolved in such a case is: admitting such alleged facts to be true,
may the court render a valid judgment in accordance with the prayer in the complaint?   In Militante
20

vs. Edrosolano,   this Court laid down the rule that the judiciary should "exercise the utmost care and
21

circumspection in passing upon a motion to dismiss on the ground of the absence thereof [cause of
action] lest, by its failure to manifest a correct appreciation of the facts alleged and deemed
hypothetically admitted, what the law grants or recognizes is effectively nullified. If that happens,
there is a blot on the legal order. The law itself stands in disrepute."

After careful examination of the petitioners' complaint, We find the statements under the introductory
affirmative allegations, as well as the specific averments under the sub-heading CAUSE OF
ACTION, to be adequate enough to show, prima facie, the claimed violation of their rights. On the
basis thereof, they may thus be granted, wholly or partly, the reliefs prayed for. It bears stressing,
however, that insofar as the cancellation of the TLAs is concerned, there is the need to implead, as
party defendants, the grantees thereof for they are indispensable parties.

The foregoing considered, Civil Case No. 90-777 be said to raise a political question. Policy
formulation or determination by the executive or legislative branches of Government is not squarely
put in issue. What is principally involved is the enforcement of a right vis-a-vis policies already
formulated and expressed in legislation. It must, nonetheless, be emphasized that the political
question doctrine is no longer, the insurmountable obstacle to the exercise of judicial power or the
impenetrable shield that protects executive and legislative actions from judicial inquiry or review. The
second paragraph of section 1, Article VIII of the Constitution states that:

Judicial power includes the duty of the courts of justice to settle actual controversies
involving rights which are legally demandable and enforceable, and to determine
whether or not there has been a grave abuse of discretion amounting to lack or
excess of jurisdiction on the part of any branch or instrumentality of the Government.

Commenting on this provision in his book, Philippine Political Law,   Mr. Justice Isagani A. Cruz, a
22

distinguished member of this Court, says:

The first part of the authority represents the traditional concept of judicial power,
involving the settlement of conflicting rights as conferred as law. The second part of
the authority represents a broadening of judicial power to enable the courts of justice
to review what was before forbidden territory, to wit, the discretion of the political
departments of the government.

As worded, the new provision vests in the judiciary, and particularly the Supreme
Court, the power to rule upon even the wisdom of the decisions of the executive and
the legislature and to declare their acts invalid for lack or excess of jurisdiction
because tainted with grave abuse of discretion. The catch, of course, is the meaning
of "grave abuse of discretion," which is a very elastic phrase that can expand or
contract according to the disposition of the judiciary.

In Daza vs. Singson,   Mr. Justice Cruz, now speaking for this Court, noted:
23

In the case now before us, the jurisdictional objection becomes even less tenable
and decisive. The reason is that, even if we were to assume that the issue presented
before us was political in nature, we would still not be precluded from revolving it
under the expanded jurisdiction conferred upon us that now covers, in proper cases,
even the political question. Article VII, Section 1, of the Constitution clearly
provides: . . .
The last ground invoked by the trial court in dismissing the complaint is the non-impairment of
contracts clause found in the Constitution. The court a quo declared that:

The Court is likewise of the impression that it cannot, no matter how we stretch our
jurisdiction, grant the reliefs prayed for by the plaintiffs, i.e., to cancel all existing
timber license agreements in the country and to cease and desist from receiving,
accepting, processing, renewing or approving new timber license agreements. For to
do otherwise would amount to "impairment of contracts" abhored (sic) by the
fundamental law.  24

We are not persuaded at all; on the contrary, We are amazed, if not shocked, by such a sweeping
pronouncement. In the first place, the respondent Secretary did not, for obvious reasons, even
invoke in his motion to dismiss the non-impairment clause. If he had done so, he would have acted
with utmost infidelity to the Government by providing undue and unwarranted benefits and
advantages to the timber license holders because he would have forever bound the Government to
strictly respect the said licenses according to their terms and conditions regardless of changes in
policy and the demands of public interest and welfare. He was aware that as correctly pointed out by
the petitioners, into every timber license must be read Section 20 of the Forestry Reform Code (P.D.
No. 705) which provides:

. . . Provided, That when the national interest so requires, the President may amend,
modify, replace or rescind any contract, concession, permit, licenses or any other
form of privilege granted herein . . .

Needless to say, all licenses may thus be revoked or rescinded by executive action. It is not
a contract, property or a property right protested by the due process clause of the
Constitution. In Tan vs. Director of Forestry,   this Court held:
25

. . . A timber license is an instrument by which the State regulates the utilization and
disposition of forest resources to the end that public welfare is promoted. A timber
license is not a contract within the purview of the due process clause; it is only a
license or privilege, which can be validly withdrawn whenever dictated by public
interest or public welfare as in this case.

A license is merely a permit or privilege to do what otherwise would be unlawful, and


is not a contract between the authority, federal, state, or municipal, granting it and
the person to whom it is granted; neither is it property or a property right, nor does it
create a vested right; nor is it taxation (37 C.J. 168). Thus, this Court held that the
granting of license does not create irrevocable rights, neither is it property or property
rights (People vs. Ong Tin, 54 O.G. 7576).

We reiterated this pronouncement in Felipe Ysmael, Jr. & Co., Inc. vs. Deputy Executive Secretary:  26

. . . Timber licenses, permits and license agreements are the principal instruments by
which the State regulates the utilization and disposition of forest resources to the end
that public welfare is promoted. And it can hardly be gainsaid that they merely
evidence a privilege granted by the State to qualified entities, and do not vest in the
latter a permanent or irrevocable right to the particular concession area and the
forest products therein. They may be validly amended, modified, replaced or
rescinded by the Chief Executive when national interests so require. Thus, they are
not deemed contracts within the purview of the due process of law clause
[See Sections 3(ee) and 20 of Pres. Decree No. 705, as amended. Also, Tan v.
Director of Forestry, G.R. No. L-24548, October 27, 1983, 125 SCRA 302].

Since timber licenses are not contracts, the non-impairment clause, which reads:

Sec. 10. No law impairing, the obligation of contracts shall be passed.  27

cannot be invoked.

In the second place, even if it is to be assumed that the same are contracts, the instant case does
not involve a law or even an executive issuance declaring the cancellation or modification of existing
timber licenses. Hence, the non-impairment clause cannot as yet be invoked. Nevertheless, granting
further that a law has actually been passed mandating cancellations or modifications, the same
cannot still be stigmatized as a violation of the non-impairment clause. This is because by its very
nature and purpose, such as law could have only been passed in the exercise of the police power of
the state for the purpose of advancing the right of the people to a balanced and healthful ecology,
promoting their health and enhancing the general welfare. In Abe vs. Foster Wheeler 
Corp.   this Court stated:
28

The freedom of contract, under our system of government, is not meant to be


absolute. The same is understood to be subject to reasonable legislative regulation
aimed at the promotion of public health, moral, safety and welfare. In other words,
the constitutional guaranty of non-impairment of obligations of contract is limited by
the exercise of the police power of the State, in the interest of public health, safety,
moral and general welfare.

The reason for this is emphatically set forth in Nebia vs. New York,   quoted in Philippine American
29

Life Insurance Co. vs. Auditor General,  to wit:


30

Under our form of government the use of property and the making of contracts are
normally matters of private and not of public concern. The general rule is that both
shall be free of governmental interference. But neither property rights nor contract
rights are absolute; for government cannot exist if the citizen may at will use his
property to the detriment of his fellows, or exercise his freedom of contract to work
them harm. Equally fundamental with the private right is that of the public to regulate
it in the common interest.

In short, the non-impairment clause must yield to the police power of the state.  31

Finally, it is difficult to imagine, as the trial court did, how the non-impairment clause could apply with
respect to the prayer to enjoin the respondent Secretary from receiving, accepting, processing,
renewing or approving new timber licenses for, save in cases of renewal, no contract would have as
of yet existed in the other instances. Moreover, with respect to renewal, the holder is not entitled to it
as a matter of right.

WHEREFORE, being impressed with merit, the instant Petition is hereby GRANTED, and the
challenged Order of respondent Judge of 18 July 1991 dismissing Civil Case No. 90-777 is hereby
set aside. The petitioners may therefore amend their complaint to implead as defendants the holders
or grantees of the questioned timber license agreements.

No pronouncement as to costs.
SO ORDERED.

G.R. No. 127882           January 27, 2004

LA BUGAL-B'LAAN TRIBAL ASSOCIATION, INC., represented by its Chairman F'LONG


MIGUEL M. LUMAYONG, WIGBERTO E. TAÑADA, PONCIANO BENNAGEN, JAIME TADEO,
RENATO R. CONSTANTINO, JR., F'LONG AGUSTIN M. DABIE, ROBERTO P. AMLOY, RAQIM
L. DABIE, SIMEON H. DOLOJO, IMELDA M. GANDON, LENY B. GUSANAN, MARCELO L.
GUSANAN, QUINTOL A. LABUAYAN, LOMINGGES D. LAWAY, BENITA P. TACUAYAN, minors
JOLY L. BUGOY, represented by his father UNDERO D. BUGOY, ROGER M. DADING,
represented by his father ANTONIO L. DADING, ROMY M. LAGARO, represented by his father
TOTING A. LAGARO, MIKENY JONG B. LUMAYONG, represented by his father MIGUEL M.
LUMAYONG, RENE T. MIGUEL, represented by his mother EDITHA T. MIGUEL, ALDEMAR L.
SAL, represented by his father DANNY M. SAL, DAISY RECARSE, represented by her mother
LYDIA S. SANTOS, EDWARD M. EMUY, ALAN P. MAMPARAIR, MARIO L. MANGCAL, ALDEN
S. TUSAN, AMPARO S. YAP, VIRGILIO CULAR, MARVIC M.V.F. LEONEN, JULIA REGINA
CULAR, GIAN CARLO CULAR, VIRGILIO CULAR, JR., represented by their father VIRGILIO
CULAR, PAUL ANTONIO P. VILLAMOR, represented by his parents JOSE VILLAMOR and
ELIZABETH PUA-VILLAMOR, ANA GININA R. TALJA, represented by her father MARIO JOSE
B. TALJA, SHARMAINE R. CUNANAN, represented by her father ALFREDO M. CUNANAN,
ANTONIO JOSE A. VITUG III, represented by his mother ANNALIZA A. VITUG, LEAN D.
NARVADEZ, represented by his father MANUEL E. NARVADEZ, JR., ROSERIO MARALAG
LINGATING, represented by her father RIO OLIMPIO A. LINGATING, MARIO JOSE B. TALJA,
DAVID E. DE VERA, MARIA MILAGROS L. SAN JOSE, SR., SUSAN O. BOLANIO, OND, LOLITA
G. DEMONTEVERDE, BENJIE L. NEQUINTO, 1 ROSE LILIA S. ROMANO, ROBERTO S.
VERZOLA, EDUARDO AURELIO C. REYES, LEAN LOUEL A. PERIA, represented by his father
ELPIDIO V. PERIA,2 GREEN FORUM PHILIPPINES, GREEN FORUM WESTERN VISAYAS, (GF-
WV), ENVIRONMETAL LEGAL ASSISTANCE CENTER (ELAC), PHILIPPINE KAISAHAN TUNGO
SA KAUNLARAN NG KANAYUNAN AT REPORMANG PANSAKAHAN
(KAISAHAN),3 KAISAHAN TUNGO SA KAUNLARAN NG KANAYUNAN AT REPORMANG
PANSAKAHAN (KAISAHAN), PARTNERSHIP FOR AGRARIAN REFORM and RURAL
DEVELOPMENT SERVICES, INC. (PARRDS), PHILIPPINE PART`NERSHIP FOR THE
DEVELOPMENT OF HUMAN RESOURCES IN THE RURAL AREAS, INC. (PHILDHRRA),
WOMEN'S LEGAL BUREAU (WLB), CENTER FOR ALTERNATIVE DEVELOPMENT
INITIATIVES, INC. (CADI), UPLAND DEVELOPMENT INSTITUTE (UDI), KINAIYAHAN
FOUNDATION, INC., SENTRO NG ALTERNATIBONG LINGAP PANLIGAL (SALIGAN), LEGAL
RIGHTS AND NATURAL RESOURCES CENTER, INC. (LRC), petitioners, 
vs.
VICTOR O. RAMOS, SECRETARY, DEPARTMENT OF ENVIRONMENT AND NATURAL
RESOURCES (DENR), HORACIO RAMOS, DIRECTOR, MINES AND GEOSCIENCES BUREAU
(MGB-DENR), RUBEN TORRES, EXECUTIVE SECRETARY, and WMC (PHILIPPINES),
INC.4 respondents.

DECISION

CARPIO-MORALES, J.:

The present petition for mandamus and prohibition assails the constitutionality of Republic Act No.
7942,5 otherwise known as the PHILIPPINE MINING ACT OF 1995, along with the Implementing
Rules and Regulations issued pursuant thereto, Department of Environment and Natural Resources
(DENR) Administrative Order 96-40, and of the Financial and Technical Assistance Agreement
(FTAA) entered into on March 30, 1995 by the Republic of the Philippines and WMC (Philippines),
Inc. (WMCP), a corporation organized under Philippine laws.

On July 25, 1987, then President Corazon C. Aquino issued Executive Order (E.O.) No.
2796 authorizing the DENR Secretary to accept, consider and evaluate proposals from foreign-
owned corporations or foreign investors for contracts or agreements involving either technical or
financial assistance for large-scale exploration, development, and utilization of minerals, which, upon
appropriate recommendation of the Secretary, the President may execute with the foreign
proponent. In entering into such proposals, the President shall consider the real contributions to the
economic growth and general welfare of the country that will be realized, as well as the development
and use of local scientific and technical resources that will be promoted by the proposed contract or
agreement. Until Congress shall determine otherwise, large-scale mining, for purpose of this
Section, shall mean those proposals for contracts or agreements for mineral resources exploration,
development, and utilization involving a committed capital investment in a single mining unit project
of at least Fifty Million Dollars in United States Currency (US $50,000,000.00).7

On March 3, 1995, then President Fidel V. Ramos approved R.A. No. 7942 to "govern the
exploration, development, utilization and processing of all mineral resources."8 R.A. No. 7942 defines
the modes of mineral agreements for mining operations,9 outlines the procedure for their filing and
approval,10 assignment/transfer11 and withdrawal,12and fixes their terms.13 Similar provisions govern
financial or technical assistance agreements.14

The law prescribes the qualifications of contractors 15 and grants them certain rights, including
timber,16 water17 and easement18 rights, and the right to possess explosives.19 Surface owners,
occupants, or concessionaires are forbidden from preventing holders of mining rights from entering
private lands and concession areas.20 A procedure for the settlement of conflicts is likewise provided
for.21

The Act restricts the conditions for exploration,22 quarry23 and other24 permits. It regulates the
transport, sale and processing of minerals,25 and promotes the development of mining communities,
science and mining technology,26and safety and environmental protection.27

The government's share in the agreements is spelled out and allocated, 28 taxes and fees are
imposed,29 incentives granted.30 Aside from penalizing certain acts,31 the law likewise specifies
grounds for the cancellation, revocation and termination of agreements and permits.32

On April 9, 1995, 30 days following its publication on March 10, 1995 in Malaya and Manila Times,
two newspapers of general circulation, R.A. No. 7942 took effect.33 Shortly before the effectivity of
R.A. No. 7942, however, or on March 30, 1995, the President entered into an FTAA with WMCP
covering 99,387 hectares of land in South Cotabato, Sultan Kudarat, Davao del Sur and North
Cotabato.34

On August 15, 1995, then DENR Secretary Victor O. Ramos issued DENR Administrative Order
(DAO) No. 95-23, s. 1995, otherwise known as the Implementing Rules and Regulations of R.A. No.
7942. This was later repealed by DAO No. 96-40, s. 1996 which was adopted on December 20,
1996.

On January 10, 1997, counsels for petitioners sent a letter to the DENR Secretary demanding that
the DENR stop the implementation of R.A. No. 7942 and DAO No. 96-40,35 giving the DENR fifteen
days from receipt36 to act thereon. The DENR, however, has yet to respond or act on petitioners'
letter.37
Petitioners thus filed the present petition for prohibition and mandamus, with a prayer for a
temporary restraining order. They allege that at the time of the filing of the petition, 100 FTAA
applications had already been filed, covering an area of 8.4 million hectares, 38 64 of which
applications are by fully foreign-owned corporations covering a total of 5.8 million hectares, and at
least one by a fully foreign-owned mining company over offshore areas.39

Petitioners claim that the DENR Secretary acted without or in excess of jurisdiction:

x x x in signing and promulgating DENR Administrative Order No. 96-40 implementing Republic Act
No. 7942, the latter being unconstitutional in that it allows fully foreign owned corporations to
explore, develop, utilize and exploit mineral resources in a manner contrary to Section 2, paragraph
4, Article XII of the Constitution;

II

x x x in signing and promulgating DENR Administrative Order No. 96-40 implementing Republic Act
No. 7942, the latter being unconstitutional in that it allows the taking of private property without the
determination of public use and for just compensation;

III

x x x in signing and promulgating DENR Administrative Order No. 96-40 implementing Republic Act
No. 7942, the latter being unconstitutional in that it violates Sec. 1, Art. III of the Constitution;

IV

x x x in signing and promulgating DENR Administrative Order No. 96-40 implementing Republic Act
No. 7942, the latter being unconstitutional in that it allows enjoyment by foreign citizens as well as
fully foreign owned corporations of the nation's marine wealth contrary to Section 2, paragraph 2 of
Article XII of the Constitution;

x x x in signing and promulgating DENR Administrative Order No. 96-40 implementing Republic Act
No. 7942, the latter being unconstitutional in that it allows priority to foreign and fully foreign owned
corporations in the exploration, development and utilization of mineral resources contrary to Article
XII of the Constitution;

VI

x x x in signing and promulgating DENR Administrative Order No. 96-40 implementing Republic Act
No. 7942, the latter being unconstitutional in that it allows the inequitable sharing of wealth contrary
to Sections [sic] 1, paragraph 1, and Section 2, paragraph 4[,] [Article XII] of the Constitution;

VII

x x x in recommending approval of and implementing the Financial and Technical Assistance


Agreement between the President of the Republic of the Philippines and Western Mining
Corporation Philippines Inc. because the same is illegal and unconstitutional.40
They pray that the Court issue an order:

(a) Permanently enjoining respondents from acting on any application for Financial or
Technical Assistance Agreements;

(b) Declaring the Philippine Mining Act of 1995 or Republic Act No. 7942 as unconstitutional
and null and void;

(c) Declaring the Implementing Rules and Regulations of the Philippine Mining Act contained
in DENR Administrative Order No. 96-40 and all other similar administrative issuances as
unconstitutional and null and void; and

(d) Cancelling the Financial and Technical Assistance Agreement issued to Western Mining
Philippines, Inc. as unconstitutional, illegal and null and void.41

Impleaded as public respondents are Ruben Torres, the then Executive Secretary, Victor O. Ramos,
the then DENR Secretary, and Horacio Ramos, Director of the Mines and Geosciences Bureau of
the DENR. Also impleaded is private respondent WMCP, which entered into the assailed FTAA with
the Philippine Government. WMCP is owned by WMC Resources International Pty., Ltd. (WMC), "a
wholly owned subsidiary of Western Mining Corporation Holdings Limited, a publicly listed major
Australian mining and exploration company."42 By WMCP's information, "it is a 100% owned
subsidiary of WMC LIMITED."43

Respondents, aside from meeting petitioners' contentions, argue that the requisites for judicial
inquiry have not been met and that the petition does not comply with the criteria for prohibition and
mandamus. Additionally, respondent WMCP argues that there has been a violation of the rule on
hierarchy of courts.

After petitioners filed their reply, this Court granted due course to the petition. The parties have since
filed their respective memoranda.

WMCP subsequently filed a Manifestation dated September 25, 2002 alleging that on January 23,
2001, WMC sold all its shares in WMCP to Sagittarius Mines, Inc. (Sagittarius), a corporation
organized under Philippine laws.44WMCP was subsequently renamed "Tampakan Mineral Resources
Corporation."45 WMCP claims that at least 60% of the equity of Sagittarius is owned by Filipinos
and/or Filipino-owned corporations while about 40% is owned by Indophil Resources NL, an
Australian company.46 It further claims that by such sale and transfer of shares, "WMCP has ceased
to be connected in any way with WMC."47

By virtue of such sale and transfer, the DENR Secretary, by Order of December 18,
2001,48 approved the transfer and registration of the subject FTAA from WMCP to Sagittarius. Said
Order, however, was appealed by Lepanto Consolidated Mining Co. (Lepanto) to the Office of the
President which upheld it by Decision of July 23, 2002.49 Its motion for reconsideration having been
denied by the Office of the President by Resolution of November 12, 2002, 50 Lepanto filed a petition
for review51 before the Court of Appeals. Incidentally, two other petitions for review related to the
approval of the transfer and registration of the FTAA to Sagittarius were recently resolved by this
Court.52

It bears stressing that this case has not been rendered moot either by the transfer and registration of
the FTAA to a Filipino-owned corporation or by the non-issuance of a temporary restraining order or
a preliminary injunction to stay the above-said July 23, 2002 decision of the Office of the
President.53 The validity of the transfer remains in dispute and awaits final judicial determination. This
assumes, of course, that such transfer cures the FTAA's alleged unconstitutionality, on which
question judgment is reserved.

WMCP also points out that the original claimowners of the major mineralized areas included in the
WMCP FTAA, namely, Sagittarius, Tampakan Mining Corporation, and Southcot Mining Corporation,
are all Filipino-owned corporations, 54 each of which was a holder of an approved Mineral Production
Sharing Agreement awarded in 1994, albeit their respective mineral claims were subsumed in the
WMCP FTAA;55 and that these three companies are the same companies that consolidated their
interests in Sagittarius to whom WMC sold its 100% equity in WMCP.56 WMCP concludes that in the
event that the FTAA is invalidated, the MPSAs of the three corporations would be revived and the
mineral claims would revert to their original claimants.57

These circumstances, while informative, are hardly significant in the resolution of this case, it
involving the validity of the FTAA, not the possible consequences of its invalidation.

Of the above-enumerated seven grounds cited by petitioners, as will be shown later, only the first
and the last need be delved into; in the latter, the discussion shall dwell only insofar as it questions
the effectivity of E. O. No. 279 by virtue of which order the questioned FTAA was forged.

Before going into the substantive issues, the procedural questions posed by respondents shall first
be tackled.

REQUISITES FOR JUDICIAL REVIEW

When an issue of constitutionality is raised, this Court can exercise its power of judicial review only if
the following requisites are present:

(1) The existence of an actual and appropriate case;

(2) A personal and substantial interest of the party raising the constitutional question;

(3) The exercise of judicial review is pleaded at the earliest opportunity; and

(4) The constitutional question is the lis mota of the case. 58

Respondents claim that the first three requisites are not present.

Section 1, Article VIII of the Constitution states that "(j)udicial power includes the duty of the courts of
justice to settle actual controversies involving rights which are legally demandable and enforceable."
The power of judicial review, therefore, is limited to the determination of actual cases and
controversies.59

An actual case or controversy means an existing case or controversy that is appropriate or ripe for
determination, not conjectural or anticipatory,60 lest the decision of the court would amount to an
advisory opinion.61 The power does not extend to hypothetical questions62 since any attempt at
abstraction could only lead to dialectics and barren legal questions and to sterile conclusions
unrelated to actualities.63
"Legal standing" or locus standi has been defined as a personal and substantial interest in the case
such that the party has sustained or will sustain direct injury as a result of the governmental act that
is being challenged,64alleging more than a generalized grievance.65 The gist of the question of
standing is whether a party alleges "such personal stake in the outcome of the controversy as to
assure that concrete adverseness which sharpens the presentation of issues upon which the court
depends for illumination of difficult constitutional questions."66 Unless a person is injuriously affected
in any of his constitutional rights by the operation of statute or ordinance, he has no standing.67

Petitioners traverse a wide range of sectors. Among them are La Bugal B'laan Tribal Association,
Inc., a farmers and indigenous people's cooperative organized under Philippine laws representing a
community actually affected by the mining activities of WMCP, members of said cooperative,68 as
well as other residents of areas also affected by the mining activities of WMCP. 69 These petitioners
have standing to raise the constitutionality of the questioned FTAA as they allege a personal and
substantial injury. They claim that they would suffer "irremediable displacement"70 as a result of the
implementation of the FTAA allowing WMCP to conduct mining activities in their area of residence.
They thus meet the appropriate case requirement as they assert an interest adverse to that of
respondents who, on the other hand, insist on the FTAA's validity.

In view of the alleged impending injury, petitioners also have standing to assail the validity of E.O.
No. 279, by authority of which the FTAA was executed.

Public respondents maintain that petitioners, being strangers to the FTAA, cannot sue either or both
contracting parties to annul it.71 In other words, they contend that petitioners are not real parties in
interest in an action for the annulment of contract.

Public respondents' contention fails. The present action is not merely one for annulment of contract
but for prohibition and mandamus. Petitioners allege that public respondents acted without or in
excess of jurisdiction in implementing the FTAA, which they submit is unconstitutional. As the case
involves constitutional questions, this Court is not concerned with whether petitioners are real parties
in interest, but with whether they have legal standing. As held in Kilosbayan v. Morato:72

x x x. "It is important to note . . . that standing because of its constitutional and public policy
underpinnings, is very different from questions relating to whether a particular plaintiff is the real
party in interest or has capacity to sue. Although all three requirements are directed towards
ensuring that only certain parties can maintain an action, standing restrictions require a partial
consideration of the merits, as well as broader policy concerns relating to the proper role of the
judiciary in certain areas.["] (FRIEDENTHAL, KANE AND MILLER, CIVIL PROCEDURE 328 [1985])

Standing is a special concern in constitutional law because in some cases suits are brought not by
parties who have been personally injured by the operation of a law or by official action taken, but by
concerned citizens, taxpayers or voters who actually sue in the public interest. Hence, the question
in standing is whether such parties have "alleged such a personal stake in the outcome of the
controversy as to assure that concrete adverseness which sharpens the presentation of issues upon
which the court so largely depends for illumination of difficult constitutional questions." (Baker v.
Carr, 369 U.S. 186, 7 L.Ed.2d 633 [1962].)

As earlier stated, petitioners meet this requirement.

The challenge against the constitutionality of R.A. No. 7942 and DAO No. 96-40 likewise fulfills the
requisites of justiciability. Although these laws were not in force when the subject FTAA was entered
into, the question as to their validity is ripe for adjudication.
The WMCP FTAA provides:

14.3 Future Legislation

Any term and condition more favourable to Financial &Technical Assistance Agreement contractors
resulting from repeal or amendment of any existing law or regulation or from the enactment of a law,
regulation or administrative order shall be considered a part of this Agreement.

It is undisputed that R.A. No. 7942 and DAO No. 96-40 contain provisions that are more favorable to
WMCP, hence, these laws, to the extent that they are favorable to WMCP, govern the FTAA.

In addition, R.A. No. 7942 explicitly makes certain provisions apply to pre-existing agreements.

SEC. 112. Non-impairment of Existing Mining/Quarrying Rights. – x x x That the provisions of


Chapter XIV on government share in mineral production-sharing agreement and of Chapter XVI on
incentives of this Act shall immediately govern and apply to a mining lessee or contractor unless the
mining lessee or contractor indicates his intention to the secretary, in writing, not to avail of said
provisions x x x Provided, finally, That such leases, production-sharing agreements, financial or
technical assistance agreements shall comply with the applicable provisions of this Act and its
implementing rules and regulations.

As there is no suggestion that WMCP has indicated its intention not to avail of the provisions of
Chapter XVI of R.A. No. 7942, it can safely be presumed that they apply to the WMCP FTAA.

Misconstruing the application of the third requisite for judicial review – that the exercise of the review
is pleaded at the earliest opportunity – WMCP points out that the petition was filed only almost two
years after the execution of the FTAA, hence, not raised at the earliest opportunity.

The third requisite should not be taken to mean that the question of constitutionality must be raised
immediately after the execution of the state action complained of. That the question of
constitutionality has not been raised before is not a valid reason for refusing to allow it to be raised
later.73 A contrary rule would mean that a law, otherwise unconstitutional, would lapse into
constitutionality by the mere failure of the proper party to promptly file a case to challenge the same.

PROPRIETY OF PROHIBITION AND MANDAMUS

Before the effectivity in July 1997 of the Revised Rules of Civil Procedure, Section 2 of Rule 65 read:

SEC. 2. Petition for prohibition. – When the proceedings of any tribunal, corporation, board, or
person, whether exercising functions judicial or ministerial, are without or in excess of its or his
jurisdiction, or with grave abuse of discretion, and there is no appeal or any other plain, speedy, and
adequate remedy in the ordinary course of law, a person aggrieved thereby may file a verified
petition in the proper court alleging the facts with certainty and praying that judgment be rendered
commanding the defendant to desist from further proceeding in the action or matter specified
therein.

Prohibition is a preventive remedy.74 It seeks a judgment ordering the defendant to desist from
continuing with the commission of an act perceived to be illegal.75

The petition for prohibition at bar is thus an appropriate remedy. While the execution of the contract
itself may be fait accompli, its implementation is not. Public respondents, in behalf of the
Government, have obligations to fulfill under said contract. Petitioners seek to prevent them from
fulfilling such obligations on the theory that the contract is unconstitutional and, therefore, void.

The propriety of a petition for prohibition being upheld, discussion of the propriety of the mandamus
aspect of the petition is rendered unnecessary.

HIERARCHY OF COURTS

The contention that the filing of this petition violated the rule on hierarchy of courts does not likewise
lie. The rule has been explained thus:

Between two courts of concurrent original jurisdiction, it is the lower court that should initially pass
upon the issues of a case. That way, as a particular case goes through the hierarchy of courts, it is
shorn of all but the important legal issues or those of first impression, which are the proper subject of
attention of the appellate court. This is a procedural rule borne of experience and adopted to
improve the administration of justice.

This Court has consistently enjoined litigants to respect the hierarchy of courts. Although this Court
has concurrent jurisdiction with the Regional Trial Courts and the Court of Appeals to issue writs of
certiorari, prohibition, mandamus, quo warranto, habeas corpus and injunction, such concurrence
does not give a party unrestricted freedom of choice of court forum. The resort to this Court's primary
jurisdiction to issue said writs shall be allowed only where the redress desired cannot be obtained in
the appropriate courts or where exceptional and compelling circumstances justify such invocation.
We held in People v. Cuaresma that:

A becoming regard for judicial hierarchy most certainly indicates that petitions for the issuance of
extraordinary writs against first level ("inferior") courts should be filed with the Regional Trial Court,
and those against the latter, with the Court of Appeals. A direct invocation of the Supreme Court's
original jurisdiction to issue these writs should be allowed only where there are special and important
reasons therefor, clearly and specifically set out in the petition. This is established policy. It is a
policy necessary to prevent inordinate demands upon the Court's time and attention which are better
devoted to those matters within its exclusive jurisdiction, and to prevent further over-crowding of the
Court's docket x x x.76 [Emphasis supplied.]

The repercussions of the issues in this case on the Philippine mining industry, if not the national
economy, as well as the novelty thereof, constitute exceptional and compelling circumstances to
justify resort to this Court in the first instance.

In all events, this Court has the discretion to take cognizance of a suit which does not satisfy the
requirements of an actual case or legal standing when paramount public interest is involved.77 When
the issues raised are of paramount importance to the public, this Court may brush aside
technicalities of procedure.78

II

Petitioners contend that E.O. No. 279 did not take effect because its supposed date of effectivity
came after President Aquino had already lost her legislative powers under the Provisional
Constitution.

And they likewise claim that the WMC FTAA, which was entered into pursuant to E.O. No. 279,
violates Section 2, Article XII of the Constitution because, among other reasons:
(1) It allows foreign-owned companies to extend more than mere financial or technical
assistance to the State in the exploitation, development, and utilization of minerals,
petroleum, and other mineral oils, and even permits foreign owned companies to "operate
and manage mining activities."

(2) It allows foreign-owned companies to extend both technical and financial assistance,
instead of "either technical or financial assistance."

To appreciate the import of these issues, a visit to the history of the pertinent constitutional provision,
the concepts contained therein, and the laws enacted pursuant thereto, is in order.

Section 2, Article XII reads in full:

Sec. 2. All lands of the public domain, waters, minerals, coal, petroleum, and other mineral oils, all
forces of potential energy, fisheries, forests or timber, wildlife, flora and fauna, and other natural
resources are owned by the State. With the exception of agricultural lands, all other natural
resources shall not be alienated. The exploration, development, and utilization of natural resources
shall be under the full control and supervision of the State. The State may directly undertake such
activities or it may enter into co-production, joint venture, or production-sharing agreements with
Filipino citizens, or corporations or associations at least sixty per centum of whose capital is owned
by such citizens. Such agreements may be for a period not exceeding twenty-five years, renewable
for not more than twenty-five years, and under such terms and conditions as may be provided by
law. In cases of water rights for irrigation, water supply, fisheries, or industrial uses other than the
development of water power, beneficial use may be the measure and limit of the grant.

The State shall protect the nation's marine wealth in its archipelagic waters, territorial sea, and
exclusive economic zone, and reserve its use and enjoyment exclusively to Filipino citizens.

The Congress may, by law, allow small-scale utilization of natural resources by Filipino citizens, as
well as cooperative fish farming, with priority to subsistence fishermen and fish-workers in rivers,
lakes, bays, and lagoons.

The President may enter into agreements with foreign-owned corporations involving either technical
or financial assistance for large-scale exploration, development, and utilization of minerals,
petroleum, and other mineral oils according to the general terms and conditions provided by law,
based on real contributions to the economic growth and general welfare of the country. In such
agreements, the State shall promote the development and use of local scientific and technical
resources.

The President shall notify the Congress of every contract entered into in accordance with this
provision, within thirty days from its execution.

THE SPANISH REGIME AND THE REGALIAN DOCTRINE

The first sentence of Section 2 embodies the Regalian doctrine or jura regalia. Introduced by Spain
into these Islands, this feudal concept is based on the State's power of dominium, which is the
capacity of the State to own or acquire property.79

In its broad sense, the term "jura regalia" refers to royal rights, or those rights which the King has by
virtue of his prerogatives. In Spanish law, it refers to a right which the sovereign has over anything in
which a subject has a right of property or propriedad. These were rights enjoyed during feudal times
by the king as the sovereign.

The theory of the feudal system was that title to all lands was originally held by the King, and while
the use of lands was granted out to others who were permitted to hold them under certain
conditions, the King theoretically retained the title. By fiction of law, the King was regarded as the
original proprietor of all lands, and the true and only source of title, and from him all lands were held.
The theory of jura regalia was therefore nothing more than a natural fruit of conquest.80

The Philippines having passed to Spain by virtue of discovery and conquest,81 earlier Spanish
decrees declared that "all lands were held from the Crown."82

The Regalian doctrine extends not only to land but also to "all natural wealth that may be found in
the bowels of the earth."83 Spain, in particular, recognized the unique value of natural resources,
viewing them, especially minerals, as an abundant source of revenue to finance its wars against
other nations.84 Mining laws during the Spanish regime reflected this perspective.85

THE AMERICAN OCCUPATION AND THE CONCESSION REGIME

By the Treaty of Paris of December 10, 1898, Spain ceded "the archipelago known as the Philippine
Islands" to the United States. The Philippines was hence governed by means of organic acts that
were in the nature of charters serving as a Constitution of the occupied territory from 1900 to
1935.86 Among the principal organic acts of the Philippines was the Act of Congress of July 1, 1902,
more commonly known as the Philippine Bill of 1902, through which the United States Congress
assumed the administration of the Philippine Islands.87 Section 20 of said Bill reserved the
disposition of mineral lands of the public domain from sale. Section 21 thereof allowed the free and
open exploration, occupation and purchase of mineral deposits not only to citizens of the Philippine
Islands but to those of the United States as well:

Sec. 21. That all valuable mineral deposits in public lands in the Philippine Islands, both surveyed
and unsurveyed, are hereby declared to be free and open to exploration, occupation and purchase,
and the land in which they are found, to occupation and purchase, by citizens of the United States or
of said Islands: Provided, That when on any lands in said Islands entered and occupied as
agricultural lands under the provisions of this Act, but not patented, mineral deposits have been
found, the working of such mineral deposits is forbidden until the person, association, or corporation
who or which has entered and is occupying such lands shall have paid to the Government of said
Islands such additional sum or sums as will make the total amount paid for the mineral claim or
claims in which said deposits are located equal to the amount charged by the Government for the
same as mineral claims.

Unlike Spain, the United States considered natural resources as a source of wealth for its nationals
and saw fit to allow both Filipino and American citizens to explore and exploit minerals in public
lands, and to grant patents to private mineral lands.88 A person who acquired ownership over a
parcel of private mineral land pursuant to the laws then prevailing could exclude other persons, even
the State, from exploiting minerals within his property.89 Thus, earlier jurisprudence90 held that:

A valid and subsisting location of mineral land, made and kept up in accordance with the provisions
of the statutes of the United States, has the effect of a grant by the United States of the present and
exclusive possession of the lands located, and this exclusive right of possession and enjoyment
continues during the entire life of the location. x x x.

x x x.
The discovery of minerals in the ground by one who has a valid mineral location perfects his claim
and his location not only against third persons, but also against the Government. x x x. [Italics in the
original.]

The Regalian doctrine and the American system, therefore, differ in one essential respect. Under the
Regalian theory, mineral rights are not included in a grant of land by the state; under the American
doctrine, mineral rights are included in a grant of land by the government.91

Section 21 also made possible the concession (frequently styled "permit", license" or
"lease")92 system.93 This was the traditional regime imposed by the colonial administrators for the
exploitation of natural resources in the extractive sector (petroleum, hard minerals, timber, etc.).94

Under the concession system, the concessionaire makes a direct equity investment for the purpose
of exploiting a particular natural resource within a given area.95 Thus, the concession amounts to
complete control by the concessionaire over the country's natural resource, for it is given exclusive
and plenary rights to exploit a particular resource at the point of extraction. 96 In consideration for the
right to exploit a natural resource, the concessionaire either pays rent or royalty, which is a fixed
percentage of the gross proceeds.97

Later statutory enactments by the legislative bodies set up in the Philippines adopted the contractual
framework of the concession.98 For instance, Act No. 2932,99 approved on August 31, 1920, which
provided for the exploration, location, and lease of lands containing petroleum and other mineral oils
and gas in the Philippines, and Act No. 2719,100 approved on May 14, 1917, which provided for the
leasing and development of coal lands in the Philippines, both utilized the concession system.101

THE 1935 CONSTITUTION AND THE NATIONALIZATION OF NATURAL RESOURCES

By the Act of United States Congress of March 24, 1934, popularly known as the Tydings-McDuffie
Law, the People of the Philippine Islands were authorized to adopt a constitution. 102 On July 30,
1934, the Constitutional Convention met for the purpose of drafting a constitution, and the
Constitution subsequently drafted was approved by the Convention on February 8, 1935. 103 The
Constitution was submitted to the President of the United States on March 18, 1935.104 On March 23,
1935, the President of the United States certified that the Constitution conformed substantially with
the provisions of the Act of Congress approved on March 24, 1934.105 On May 14, 1935, the
Constitution was ratified by the Filipino people.106

The 1935 Constitution adopted the Regalian doctrine, declaring all natural resources of the
Philippines, including mineral lands and minerals, to be property belonging to the State.107 As
adopted in a republican system, the medieval concept of jura regalia is stripped of royal overtones
and ownership of the land is vested in the State.108

Section 1, Article XIII, on Conservation and Utilization of Natural Resources, of the 1935 Constitution
provided:

SECTION 1. All agricultural, timber, and mineral lands of the public domain, waters,
minerals, coal, petroleum, and other mineral oils, all forces of potential energy, and other
natural resources of the Philippines belong to the State, and their disposition, exploitation,
development, or utilization shall be limited to citizens of the Philippines, or to corporations or
associations at least sixty per centum of the capital of which is owned by such citizens,
subject to any existing right, grant, lease, or concession at the time of the inauguration of the
Government established under this Constitution. Natural resources, with the exception of
public agricultural land, shall not be alienated, and no license, concession, or lease for the
exploitation, development, or utilization of any of the natural resources shall be granted for a
period exceeding twenty-five years, except as to water rights for irrigation, water supply,
fisheries, or industrial uses other than the development of water power, in which cases
beneficial use may be the measure and the limit of the grant.

The nationalization and conservation of the natural resources of the country was one of the fixed and
dominating objectives of the 1935 Constitutional Convention.109 One delegate relates:

There was an overwhelming sentiment in the Convention in favor of the principle of state ownership
of natural resources and the adoption of the Regalian doctrine. State ownership of natural resources
was seen as a necessary starting point to secure recognition of the state's power to control their
disposition, exploitation, development, or utilization. The delegates of the Constitutional Convention
very well knew that the concept of State ownership of land and natural resources was introduced by
the Spaniards, however, they were not certain whether it was continued and applied by the
Americans. To remove all doubts, the Convention approved the provision in the Constitution
affirming the Regalian doctrine.

The adoption of the principle of state ownership of the natural resources and of the Regalian
doctrine was considered to be a necessary starting point for the plan of nationalizing and conserving
the natural resources of the country. For with the establishment of the principle of state ownership of
the natural resources, it would not be hard to secure the recognition of the power of the State to
control their disposition, exploitation, development or utilization.110

The nationalization of the natural resources was intended (1) to insure their conservation for Filipino
posterity; (2) to serve as an instrument of national defense, helping prevent the extension to the
country of foreign control through peaceful economic penetration; and (3) to avoid making the
Philippines a source of international conflicts with the consequent danger to its internal security and
independence.111

The same Section 1, Article XIII also adopted the concession system, expressly permitting the State
to grant licenses, concessions, or leases for the exploitation, development, or utilization of any of the
natural resources. Grants, however, were limited to Filipinos or entities at least 60% of the capital of
which is owned by Filipinos. lawph!l.ne+

The swell of nationalism that suffused the 1935 Constitution was radically diluted when on
November 1946, the Parity Amendment, which came in the form of an "Ordinance Appended to the
Constitution," was ratified in a plebiscite. 112 The Amendment extended, from July 4, 1946 to July 3,
1974, the right to utilize and exploit our natural resources to citizens of the United States and
business enterprises owned or controlled, directly or indirectly, by citizens of the United States:113

Notwithstanding the provision of section one, Article Thirteen, and section eight, Article Fourteen, of
the foregoing Constitution, during the effectivity of the Executive Agreement entered into by the
President of the Philippines with the President of the United States on the fourth of July, nineteen
hundred and forty-six, pursuant to the provisions of Commonwealth Act Numbered Seven hundred
and thirty-three, but in no case to extend beyond the third of July, nineteen hundred and seventy-
four, the disposition, exploitation, development, and utilization of all agricultural, timber, and mineral
lands of the public domain, waters, minerals, coals, petroleum, and other mineral oils, all forces and
sources of potential energy, and other natural resources of the Philippines, and the operation of
public utilities, shall, if open to any person, be open to citizens of the United States and to all forms
of business enterprise owned or controlled, directly or indirectly, by citizens of the United States in
the same manner as to, and under the same conditions imposed upon, citizens of the Philippines or
corporations or associations owned or controlled by citizens of the Philippines.
The Parity Amendment was subsequently modified by the 1954 Revised Trade Agreement, also
known as the Laurel-Langley Agreement, embodied in Republic Act No. 1355.114

THE PETROLEUM ACT OF 1949 AND THE CONCESSION SYSTEM

In the meantime, Republic Act No. 387,115 also known as the Petroleum Act of 1949, was approved
on June 18, 1949.

The Petroleum Act of 1949 employed the concession system for the exploitation of the nation's
petroleum resources. Among the kinds of concessions it sanctioned were exploration and
exploitation concessions, which respectively granted to the concessionaire the exclusive right to
explore for116 or develop117 petroleum within specified areas.

Concessions may be granted only to duly qualified persons 118 who have sufficient finances,
organization, resources, technical competence, and skills necessary to conduct the operations to be
undertaken.119

Nevertheless, the Government reserved the right to undertake such work itself. 120 This proceeded
from the theory that all natural deposits or occurrences of petroleum or natural gas in public and/or
private lands in the Philippines belong to the State.121 Exploration and exploitation concessions did
not confer upon the concessionaire ownership over the petroleum lands and petroleum
deposits.122 However, they did grant concessionaires the right to explore, develop, exploit, and utilize
them for the period and under the conditions determined by the law.123

Concessions were granted at the complete risk of the concessionaire; the Government did not
guarantee the existence of petroleum or undertake, in any case, title warranty.124

Concessionaires were required to submit information as maybe required by the Secretary of


Agriculture and Natural Resources, including reports of geological and geophysical examinations, as
well as production reports.125Exploration126 and exploitation127 concessionaires were also required to
submit work programs. lavvphi1.net

Exploitation concessionaires, in particular, were obliged to pay an annual exploitation tax,128 the


object of which is to induce the concessionaire to actually produce petroleum, and not simply to sit
on the concession without developing or exploiting it.129 These concessionaires were also bound to
pay the Government royalty, which was not less than 12½% of the petroleum produced and saved,
less that consumed in the operations of the concessionaire. 130 Under Article 66, R.A. No. 387, the
exploitation tax may be credited against the royalties so that if the concessionaire shall be actually
producing enough oil, it would not actually be paying the exploitation tax.131

Failure to pay the annual exploitation tax for two consecutive years,132 or the royalty due to the
Government within one year from the date it becomes due, 133 constituted grounds for the cancellation
of the concession. In case of delay in the payment of the taxes or royalty imposed by the law or by
the concession, a surcharge of 1% per month is exacted until the same are paid.134

As a rule, title rights to all equipment and structures that the concessionaire placed on the land
belong to the exploration or exploitation concessionaire.135 Upon termination of such concession, the
concessionaire had a right to remove the same.136

The Secretary of Agriculture and Natural Resources was tasked with carrying out the provisions of
the law, through the Director of Mines, who acted under the Secretary's immediate supervision and
control.137 The Act granted the Secretary the authority to inspect any operation of the concessionaire
and to examine all the books and accounts pertaining to operations or conditions related to payment
of taxes and royalties.138

The same law authorized the Secretary to create an Administration Unit and a Technical
Board.139 The Administration Unit was charged, inter alia, with the enforcement of the provisions of
the law.140 The Technical Board had, among other functions, the duty to check on the performance of
concessionaires and to determine whether the obligations imposed by the Act and its implementing
regulations were being complied with.141

Victorio Mario A. Dimagiba, Chief Legal Officer of the Bureau of Energy Development, analyzed the
benefits and drawbacks of the concession system insofar as it applied to the petroleum industry:

Advantages of Concession. Whether it emphasizes income tax or royalty, the most positive aspect of
the concession system is that the State's financial involvement is virtually risk free and administration
is simple and comparatively low in cost. Furthermore, if there is a competitive allocation of the
resource leading to substantial bonuses and/or greater royalty coupled with a relatively high level of
taxation, revenue accruing to the State under the concession system may compare favorably with
other financial arrangements.

Disadvantages of Concession. There are, however, major negative aspects to this system. Because
the Government's role in the traditional concession is passive, it is at a distinct disadvantage in
managing and developing policy for the nation's petroleum resource. This is true for several reasons.
First, even though most concession agreements contain covenants requiring diligence in operations
and production, this establishes only an indirect and passive control of the host country in resource
development. Second, and more importantly, the fact that the host country does not directly
participate in resource management decisions inhibits its ability to train and employ its nationals in
petroleum development. This factor could delay or prevent the country from effectively engaging in
the development of its resources. Lastly, a direct role in management is usually necessary in order
to obtain a knowledge of the international petroleum industry which is important to an appreciation of
the host country's resources in relation to those of other countries.142

Other liabilities of the system have also been noted:

x x x there are functional implications which give the concessionaire great economic power arising
from its exclusive equity holding. This includes, first, appropriation of the returns of the undertaking,
subject to a modest royalty; second, exclusive management of the project; third, control of
production of the natural resource, such as volume of production, expansion, research and
development; and fourth, exclusive responsibility for downstream operations, like processing,
marketing, and distribution. In short, even if nominally, the state is the sovereign and owner of the
natural resource being exploited, it has been shorn of all elements of control over such natural
resource because of the exclusive nature of the contractual regime of the concession. The
concession system, investing as it does ownership of natural resources, constitutes a consistent
inconsistency with the principle embodied in our Constitution that natural resources belong to the
state and shall not be alienated, not to mention the fact that the concession was the bedrock of the
colonial system in the exploitation of natural resources.143

Eventually, the concession system failed for reasons explained by Dimagiba:

Notwithstanding the good intentions of the Petroleum Act of 1949, the concession system could not
have properly spurred sustained oil exploration activities in the country, since it assumed that such a
capital-intensive, high risk venture could be successfully undertaken by a single individual or a small
company. In effect, concessionaires' funds were easily exhausted. Moreover, since the concession
system practically closed its doors to interested foreign investors, local capital was stretched to the
limits. The old system also failed to consider the highly sophisticated technology and expertise
required, which would be available only to multinational companies.144

A shift to a new regime for the development of natural resources thus seemed imminent.

PRESIDENTIAL DECREE NO. 87, THE 1973 CONSTITUTION AND THE SERVICE CONTRACT
SYSTEM

The promulgation on December 31, 1972 of Presidential Decree No. 87,145 otherwise known as The
Oil Exploration and Development Act of 1972 signaled such a transformation. P.D. No. 87 permitted
the government to explore for and produce indigenous petroleum through "service contracts."146

"Service contracts" is a term that assumes varying meanings to different people, and it has carried
many names in different countries, like "work contracts" in Indonesia, "concession agreements" in
Africa, "production-sharing agreements" in the Middle East, and "participation agreements" in Latin
America.147 A functional definition of "service contracts" in the Philippines is provided as follows:

A service contract is a contractual arrangement for engaging in the exploitation and development of
petroleum, mineral, energy, land and other natural resources by which a government or its agency,
or a private person granted a right or privilege by the government authorizes the other party (service
contractor) to engage or participate in the exercise of such right or the enjoyment of the privilege, in
that the latter provides financial or technical resources, undertakes the exploitation or production of a
given resource, or directly manages the productive enterprise, operations of the exploration and
exploitation of the resources or the disposition of marketing or resources.148

In a service contract under P.D. No. 87, service and technology are furnished by the service
contractor for which it shall be entitled to the stipulated service fee. 149 The contractor must be
technically competent and financially capable to undertake the operations required in the contract.150

Financing is supposed to be provided by the Government to which all petroleum produced


belongs.151 In case the Government is unable to finance petroleum exploration operations, the
contractor may furnish services, technology and financing, and the proceeds of sale of the petroleum
produced under the contract shall be the source of funds for payment of the service fee and the
operating expenses due the contractor.152 The contractor shall undertake, manage and execute
petroleum operations, subject to the government overseeing the management of the
operations.153 The contractor provides all necessary services and technology and the requisite
financing, performs the exploration work obligations, and assumes all exploration risks such that if
no petroleum is produced, it will not be entitled to reimbursement.154 Once petroleum in commercial
quantity is discovered, the contractor shall operate the field on behalf of the government.155

P.D. No. 87 prescribed minimum terms and conditions for every service contract. 156 It also granted
the contractor certain privileges, including exemption from taxes and payment of tariff duties,157 and
permitted the repatriation of capital and retention of profits abroad.158

Ostensibly, the service contract system had certain advantages over the concession regime. 159 It has
been opined, though, that, in the Philippines, our concept of a service contract, at least in the
petroleum industry, was basically a concession regime with a production-sharing element.160
On January 17, 1973, then President Ferdinand E. Marcos proclaimed the ratification of a new
Constitution.161Article XIV on the National Economy and Patrimony contained provisions similar to
the 1935 Constitution with regard to Filipino participation in the nation's natural resources. Section 8,
Article XIV thereof provides:

Sec. 8. All lands of the public domain, waters, minerals, coal, petroleum and other mineral oils, all
forces of potential energy, fisheries, wildlife, and other natural resources of the Philippines belong to
the State. With the exception of agricultural, industrial or commercial, residential and resettlement
lands of the public domain, natural resources shall not be alienated, and no license, concession, or
lease for the exploration, development, exploitation, or utilization of any of the natural resources
shall be granted for a period exceeding twenty-five years, renewable for not more than twenty-five
years, except as to water rights for irrigation, water supply, fisheries, or industrial uses other than the
development of water power, in which cases beneficial use may be the measure and the limit of the
grant.

While Section 9 of the same Article maintained the Filipino-only policy in the enjoyment of natural
resources, it also allowed Filipinos, upon authority of the Batasang Pambansa, to enter into service
contracts with any person or entity for the exploration or utilization of natural resources.

Sec. 9. The disposition, exploration, development, exploitation, or utilization of any of the natural
resources of the Philippines shall be limited to citizens, or to corporations or associations at least
sixty per centum of which is owned by such citizens. The Batasang Pambansa, in the national
interest, may allow such citizens, corporations or associations to enter into service contracts for
financial, technical, management, or other forms of assistance with any person or entity for the
exploration, or utilization of any of the natural resources. Existing valid and binding service contracts
for financial, technical, management, or other forms of assistance are hereby recognized as such.
[Emphasis supplied.]

The concept of service contracts, according to one delegate, was borrowed from the methods
followed by India, Pakistan and especially Indonesia in the exploration of petroleum and mineral
oils.162 The provision allowing such contracts, according to another, was intended to "enhance the
proper development of our natural resources since Filipino citizens lack the needed capital and
technical know-how which are essential in the proper exploration, development and exploitation of
the natural resources of the country."163

The original idea was to authorize the government, not private entities, to enter into service contracts
with foreign entities.164 As finally approved, however, a citizen or private entity could be allowed by
the National Assembly to enter into such service contract.165 The prior approval of the National
Assembly was deemed sufficient to protect the national interest. 166 Notably, none of the laws allowing
service contracts were passed by the Batasang Pambansa. Indeed, all of them were enacted by
presidential decree.

On March 13, 1973, shortly after the ratification of the new Constitution, the President promulgated
Presidential Decree No. 151.167 The law allowed Filipino citizens or entities which have acquired
lands of the public domain or which own, hold or control such lands to enter into service contracts for
financial, technical, management or other forms of assistance with any foreign persons or entity for
the exploration, development, exploitation or utilization of said lands.168

Presidential Decree No. 463,169 also known as The Mineral Resources Development Decree of 1974,
was enacted on May 17, 1974. Section 44 of the decree, as amended, provided that a lessee of a
mining claim may enter into a service contract with a qualified domestic or foreign contractor for the
exploration, development and exploitation of his claims and the processing and marketing of the
product thereof.

Presidential Decree No. 704170 (The Fisheries Decree of 1975), approved on May 16, 1975, allowed
Filipinos engaged in commercial fishing to enter into contracts for financial, technical or other forms
of assistance with any foreign person, corporation or entity for the production, storage, marketing
and processing of fish and fishery/aquatic products.171

Presidential Decree No. 705172 (The Revised Forestry Code of the Philippines), approved on May 19,
1975, allowed "forest products licensees, lessees, or permitees to enter into service contracts for
financial, technical, management, or other forms of assistance . . . with any foreign person or entity
for the exploration, development, exploitation or utilization of the forest resources."173

Yet another law allowing service contracts, this time for geothermal resources, was Presidential
Decree No. 1442,174 which was signed into law on June 11, 1978. Section 1 thereof authorized the
Government to enter into service contracts for the exploration, exploitation and development of
geothermal resources with a foreign contractor who must be technically and financially capable of
undertaking the operations required in the service contract.

Thus, virtually the entire range of the country's natural resources –from petroleum and minerals to
geothermal energy, from public lands and forest resources to fishery products – was well covered by
apparent legal authority to engage in the direct participation or involvement of foreign persons or
corporations (otherwise disqualified) in the exploration and utilization of natural resources through
service contracts.175

THE 1987 CONSTITUTION AND TECHNICAL OR FINANCIAL ASSISTANCE AGREEMENTS

After the February 1986 Edsa Revolution, Corazon C. Aquino took the reins of power under a
revolutionary government. On March 25, 1986, President Aquino issued Proclamation No.
3,176 promulgating the Provisional Constitution, more popularly referred to as the Freedom
Constitution. By authority of the same Proclamation, the President created a Constitutional
Commission (CONCOM) to draft a new constitution, which took effect on the date of its ratification
on February 2, 1987.177

The 1987 Constitution retained the Regalian doctrine. The first sentence of Section 2, Article XII
states: "All lands of the public domain, waters, minerals, coal, petroleum, and other mineral oils, all
forces of potential energy, fisheries, forests or timber, wildlife, flora and fauna, and other natural
resources are owned by the State."

Like the 1935 and 1973 Constitutions before it, the 1987 Constitution, in the second sentence of the
same provision, prohibits the alienation of natural resources, except agricultural lands.

The third sentence of the same paragraph is new: "The exploration, development and utilization of
natural resources shall be under the full control and supervision of the State." The constitutional
policy of the State's "full control and supervision" over natural resources proceeds from the concept
of jura regalia, as well as the recognition of the importance of the country's natural resources, not
only for national economic development, but also for its security and national defense.178 Under this
provision, the State assumes "a more dynamic role" in the exploration, development and utilization
of natural resources.179
Conspicuously absent in Section 2 is the provision in the 1935 and 1973 Constitutions authorizing
the State to grant licenses, concessions, or leases for the exploration, exploitation, development, or
utilization of natural resources. By such omission, the utilization of inalienable lands of public domain
through "license, concession or lease" is no longer allowed under the 1987 Constitution.180

Having omitted the provision on the concession system, Section 2 proceeded to introduce
"unfamiliar language":181

The State may directly undertake such activities or it may enter into co-production, joint venture, or
production-sharing agreements with Filipino citizens, or corporations or associations at least sixty
per centum of whose capital is owned by such citizens.

Consonant with the State's "full supervision and control" over natural resources, Section 2 offers the
State two "options."182 One, the State may directly undertake these activities itself; or two, it may
enter into co-production, joint venture, or production-sharing agreements with Filipino citizens, or
entities at least 60% of whose capital is owned by such citizens.

A third option is found in the third paragraph of the same section:

The Congress may, by law, allow small-scale utilization of natural resources by Filipino citizens, as
well as cooperative fish farming, with priority to subsistence fishermen and fish-workers in rivers,
lakes, bays, and lagoons.

While the second and third options are limited only to Filipino citizens or, in the case of the former, to
corporations or associations at least 60% of the capital of which is owned by Filipinos, a fourth
allows the participation of foreign-owned corporations. The fourth and fifth paragraphs of Section 2
provide:

The President may enter into agreements with foreign-owned corporations involving either technical
or financial assistance for large-scale exploration, development, and utilization of minerals,
petroleum, and other mineral oils according to the general terms and conditions provided by law,
based on real contributions to the economic growth and general welfare of the country. In such
agreements, the State shall promote the development and use of local scientific and technical
resources.

The President shall notify the Congress of every contract entered into in accordance with this
provision, within thirty days from its execution.

Although Section 2 sanctions the participation of foreign-owned corporations in the exploration,


development, and utilization of natural resources, it imposes certain limitations or conditions to
agreements with such corporations.

First, the parties to FTAAs. Only the President, in behalf of the State, may enter into these
agreements, and only with corporations. By contrast, under the 1973 Constitution, a Filipino
citizen, corporation or association may enter into a service contract with a "foreign person or
entity."

Second, the size of the activities: only large-scale exploration, development, and utilization is
allowed. The term "large-scale usually refers to very capital-intensive activities."183
Third, the natural resources subject of the activities is restricted to minerals, petroleum and
other mineral oils, the intent being to limit service contracts to those areas where Filipino
capital may not be sufficient.184

Fourth, consistency with the provisions of statute. The agreements must be in accordance
with the terms and conditions provided by law.

Fifth, Section 2 prescribes certain standards for entering into such agreements. The
agreements must be based on real contributions to economic growth and general welfare of
the country.

Sixth, the agreements must contain rudimentary stipulations for the promotion of the
development and use of local scientific and technical resources.

Seventh, the notification requirement. The President shall notify Congress of every financial
or technical assistance agreement entered into within thirty days from its execution.

Finally, the scope of the agreements. While the 1973 Constitution referred to "service
contracts for financial, technical, management, or other forms of assistance" the 1987
Constitution provides for "agreements. . . involving either financial or technical assistance." It
bears noting that the phrases "service contracts" and "management or other forms of
assistance" in the earlier constitution have been omitted.

By virtue of her legislative powers under the Provisional Constitution, 185 President Aquino, on July 10,
1987, signed into law E.O. No. 211 prescribing the interim procedures in the processing and
approval of applications for the exploration, development and utilization of minerals. The omission in
the 1987 Constitution of the term "service contracts" notwithstanding, the said E.O. still referred to
them in Section 2 thereof:

Sec. 2. Applications for the exploration, development and utilization of mineral resources, including
renewal applications and applications for approval of operating agreements and mining service
contracts, shall be accepted and processed and may be approved x x x. [Emphasis supplied.]

The same law provided in its Section 3 that the "processing, evaluation and approval of all mining
applications . . . operating agreements and service contracts . . . shall be governed by Presidential
Decree No. 463, as amended, other existing mining laws, and their implementing rules and
regulations. . . ."

As earlier stated, on the 25th also of July 1987, the President issued E.O. No. 279 by authority of
which the subject WMCP FTAA was executed on March 30, 1995.

On March 3, 1995, President Ramos signed into law R.A. No. 7942. Section 15 thereof declares that
the Act "shall govern the exploration, development, utilization, and processing of all mineral
resources." Such declaration notwithstanding, R.A. No. 7942 does not actually cover all the modes
through which the State may undertake the exploration, development, and utilization of natural
resources.

The State, being the owner of the natural resources, is accorded the primary power and
responsibility in the exploration, development and utilization thereof. As such, it may undertake these
activities through four modes:
The State may directly undertake such activities.

(2) The State may enter into co-production, joint venture or production-sharing agreements
with Filipino citizens or qualified corporations.

(3) Congress may, by law, allow small-scale utilization of natural resources by Filipino
citizens.

(4) For the large-scale exploration, development and utilization of minerals, petroleum and
other mineral oils, the President may enter into agreements with foreign-owned corporations
involving technical or financial assistance.186

Except to charge the Mines and Geosciences Bureau of the DENR with performing researches and
surveys,187 and a passing mention of government-owned or controlled corporations, 188 R.A. No. 7942
does not specify how the State should go about the first mode. The third mode, on the other hand, is
governed by Republic Act No. 7076189(the People's Small-Scale Mining Act of 1991) and other
pertinent laws.190 R.A. No. 7942 primarily concerns itself with the second and fourth modes.

Mineral production sharing, co-production and joint venture agreements are collectively classified by
R.A. No. 7942 as "mineral agreements."191 The Government participates the least in a mineral
production sharing agreement (MPSA). In an MPSA, the Government grants the contractor192 the
exclusive right to conduct mining operations within a contract area 193 and shares in the gross
output.194 The MPSA contractor provides the financing, technology, management and personnel
necessary for the agreement's implementation.195 The total government share in an MPSA is the
excise tax on mineral products under Republic Act No. 7729,196 amending Section 151(a) of the
National Internal Revenue Code, as amended.197

In a co-production agreement (CA),198 the Government provides inputs to the mining operations other
than the mineral resource,199 while in a joint venture agreement (JVA), where the Government enjoys
the greatest participation, the Government and the JVA contractor organize a company with both
parties having equity shares.200 Aside from earnings in equity, the Government in a JVA is also
entitled to a share in the gross output. 201The Government may enter into a CA202 or JVA203 with one or
more contractors. The Government's share in a CA or JVA is set out in Section 81 of the law:

The share of the Government in co-production and joint venture agreements shall be negotiated by
the Government and the contractor taking into consideration the: (a) capital investment of the
project, (b) the risks involved, (c) contribution of the project to the economy, and (d) other factors
that will provide for a fair and equitable sharing between the Government and the contractor. The
Government shall also be entitled to compensations for its other contributions which shall be agreed
upon by the parties, and shall consist, among other things, the contractor's income tax, excise tax,
special allowance, withholding tax due from the contractor's foreign stockholders arising from
dividend or interest payments to the said foreign stockholders, in case of a foreign national and all
such other taxes, duties and fees as provided for under existing laws.

All mineral agreements grant the respective contractors the exclusive right to conduct mining
operations and to extract all mineral resources found in the contract area. 204 A "qualified person" may
enter into any of the mineral agreements with the Government.205 A "qualified person" is

any citizen of the Philippines with capacity to contract, or a corporation, partnership, association, or
cooperative organized or authorized for the purpose of engaging in mining, with technical and
financial capability to undertake mineral resources development and duly registered in accordance
with law at least sixty per centum (60%) of the capital of which is owned by citizens of the Philippines
x x x.206

The fourth mode involves "financial or technical assistance agreements." An FTAA is defined as "a
contract involving financial or technical assistance for large-scale exploration, development, and
utilization of natural resources."207 Any qualified person with technical and financial capability to
undertake large-scale exploration, development, and utilization of natural resources in the
Philippines may enter into such agreement directly with the Government through the DENR.208 For
the purpose of granting an FTAA, a legally organized foreign-owned corporation (any corporation,
partnership, association, or cooperative duly registered in accordance with law in which less than
50% of the capital is owned by Filipino citizens)209 is deemed a "qualified person."210

Other than the difference in contractors' qualifications, the principal distinction between mineral
agreements and FTAAs is the maximum contract area to which a qualified person may hold or be
granted.211 "Large-scale" under R.A. No. 7942 is determined by the size of the contract area, as
opposed to the amount invested (US $50,000,000.00), which was the standard under E.O. 279.

Like a CA or a JVA, an FTAA is subject to negotiation.212 The Government's contributions, in the form


of taxes, in an FTAA is identical to its contributions in the two mineral agreements, save that in an
FTAA:

The collection of Government share in financial or technical assistance agreement shall commence
after the financial or technical assistance agreement contractor has fully recovered its pre-operating
expenses, exploration, and development expenditures, inclusive.213

III

Having examined the history of the constitutional provision and statutes enacted pursuant thereto, a
consideration of the substantive issues presented by the petition is now in order.

THE EFFECTIVITY OF EXECUTIVE ORDER NO. 279

Petitioners argue that E.O. No. 279, the law in force when the WMC FTAA was executed, did not
come into effect.

E.O. No. 279 was signed into law by then President Aquino on July 25, 1987, two days before the
opening of Congress on July 27, 1987.214 Section 8 of the E.O. states that the same "shall take effect
immediately." This provision, according to petitioners, runs counter to Section 1 of E.O. No.
200,215 which provides:

SECTION 1. Laws shall take effect after fifteen days following the completion of their publication
either in the Official Gazette or in a newspaper of general circulation in the Philippines, unless it is
otherwise provided.216 [Emphasis supplied.]

On that premise, petitioners contend that E.O. No. 279 could have only taken effect fifteen days after
its publication at which time Congress had already convened and the President's power to legislate
had ceased.

Respondents, on the other hand, counter that the validity of E.O. No. 279 was settled in Miners
Association of the Philippines v. Factoran, supra. This is of course incorrect for the issue in Miners
Association was not the validity of E.O. No. 279 but that of DAO Nos. 57 and 82 which were issued
pursuant thereto.

Nevertheless, petitioners' contentions have no merit.

It bears noting that there is nothing in E.O. No. 200 that prevents a law from taking effect on a date
other than – even before – the 15-day period after its publication. Where a law provides for its own
date of effectivity, such date prevails over that prescribed by E.O. No. 200. Indeed, this is the very
essence of the phrase "unless it is otherwise provided" in Section 1 thereof. Section 1, E.O. No. 200,
therefore, applies only when a statute does not provide for its own date of effectivity.

What is mandatory under E.O. No. 200, and what due process requires, as this Court held in Tañada
v. Tuvera,217is the publication of the law for without such notice and publication, there would be no
basis for the application of the maxim "ignorantia legis n[eminem] excusat." It would be the height of
injustice to punish or otherwise burden a citizen for the transgression of a law of which he had no
notice whatsoever, not even a constructive one.

While the effectivity clause of E.O. No. 279 does not require its publication, it is not a ground for its
invalidation since the Constitution, being "the fundamental, paramount and supreme law of the
nation," is deemed written in the law.218 Hence, the due process clause,219 which, so Tañada held,
mandates the publication of statutes, is read into Section 8 of E.O. No. 279. Additionally, Section 1
of E.O. No. 200 which provides for publication "either in the Official Gazette or in a newspaper of
general circulation in the Philippines," finds suppletory application. It is significant to note that E.O.
No. 279 was actually published in the Official Gazette220 on August 3, 1987.

From a reading then of Section 8 of E.O. No. 279, Section 1 of E.O. No. 200, and Tañada v. Tuvera,
this Court holds that E.O. No. 279 became effective immediately upon its publication in the Official
Gazette on August 3, 1987.

That such effectivity took place after the convening of the first Congress is irrelevant. At the time
President Aquino issued E.O. No. 279 on July 25, 1987, she was still validly exercising legislative
powers under the Provisional Constitution. 221 Article XVIII (Transitory Provisions) of the 1987
Constitution explicitly states:

Sec. 6. The incumbent President shall continue to exercise legislative powers until the first Congress
is convened.

The convening of the first Congress merely precluded the exercise of legislative powers by President
Aquino; it did not prevent the effectivity of laws she had previously enacted.

There can be no question, therefore, that E.O. No. 279 is an effective, and a validly enacted, statute.

THE CONSTITUTIONALITY OF THE WMCP FTAA

Petitioners submit that, in accordance with the text of Section 2, Article XII of the Constitution,
FTAAs should be limited to "technical or financial assistance" only. They observe, however, that,
contrary to the language of the Constitution, the WMCP FTAA allows WMCP, a fully foreign-owned
mining corporation, to extend more than mere financial or technical assistance to the State, for it
permits WMCP to manage and operate every aspect of the mining activity. 222
Petitioners' submission is well-taken. It is a cardinal rule in the interpretation of constitutions that the
instrument must be so construed as to give effect to the intention of the people who adopted
it.223 This intention is to be sought in the constitution itself, and the apparent meaning of the words is
to be taken as expressing it, except in cases where that assumption would lead to absurdity,
ambiguity, or contradiction.224 What the Constitution says according to the text of the provision,
therefore, compels acceptance and negates the power of the courts to alter it, based on the
postulate that the framers and the people mean what they say.225 Accordingly, following the literal
text of the Constitution, assistance accorded by foreign-owned corporations in the large-scale
exploration, development, and utilization of petroleum, minerals and mineral oils should be limited to
"technical" or "financial" assistance only.

WMCP nevertheless submits that the word "technical" in the fourth paragraph of Section 2 of E.O.
No. 279 encompasses a "broad number of possible services," perhaps, "scientific and/or
technological in basis."226 It thus posits that it may also well include "the area of management or
operations . . . so long as such assistance requires specialized knowledge or skills, and are related
to the exploration, development and utilization of mineral resources."227

This Court is not persuaded. As priorly pointed out, the phrase "management or other forms of
assistance" in the 1973 Constitution was deleted in the 1987 Constitution, which allows only
"technical or financial assistance." Casus omisus pro omisso habendus est. A person, object or thing
omitted from an enumeration must be held to have been omitted intentionally. 228 As will be shown
later, the management or operation of mining activities by foreign contractors, which is the primary
feature of service contracts, was precisely the evil that the drafters of the 1987 Constitution sought to
eradicate.

Respondents insist that "agreements involving technical or financial assistance" is just another term
for service contracts. They contend that the proceedings of the CONCOM indicate "that although the
terminology 'service contract' was avoided [by the Constitution], the concept it represented was not."
They add that "[t]he concept is embodied in the phrase 'agreements involving financial or technical
assistance.'"229 And point out how members of the CONCOM referred to these agreements as
"service contracts." For instance:

SR. TAN. Am I correct in thinking that the only difference between these future service
contracts and the past service contracts under Mr. Marcos is the general law to be enacted
by the legislature and the notification of Congress by the President? That is the only
difference, is it not?

MR. VILLEGAS. That is right.

SR. TAN. So those are the safeguards[?]

MR. VILLEGAS. Yes. There was no law at all governing service contracts before.

SR. TAN. Thank you, Madam President.230 [Emphasis supplied.]

WMCP also cites the following statements of Commissioners Gascon, Garcia, Nolledo and
Tadeo who alluded to service contracts as they explained their respective votes in the
approval of the draft Article:

MR. GASCON. Mr. Presiding Officer, I vote no primarily because of two reasons: One, the
provision on service contracts. I felt that if we would constitutionalize any provision on
service contracts, this should always be with the concurrence of Congress and not guided
only by a general law to be promulgated by Congress. x x x.231 [Emphasis supplied.]

x x x.

MR. GARCIA. Thank you.

I vote no. x x x.

Service contracts are given constitutional legitimization in Section 3, even when they have
been proven to be inimical to the interests of the nation, providing as they do the legal
loophole for the exploitation of our natural resources for the benefit of foreign interests. They
constitute a serious negation of Filipino control on the use and disposition of the nation's
natural resources, especially with regard to those which are nonrenewable. 232[Emphasis
supplied.]

xxx

MR. NOLLEDO. While there are objectionable provisions in the Article on National Economy
and Patrimony, going over said provisions meticulously, setting aside prejudice and
personalities will reveal that the article contains a balanced set of provisions. I hope the
forthcoming Congress will implement such provisions taking into account that Filipinos
should have real control over our economy and patrimony, and if foreign equity is permitted,
the same must be subordinated to the imperative demands of the national interest.

x x x.

It is also my understanding that service contracts involving foreign corporations or entities


are resorted to only when no Filipino enterprise or Filipino-controlled enterprise could
possibly undertake the exploration or exploitation of our natural resources and that
compensation under such contracts cannot and should not equal what should pertain to
ownership of capital. In other words, the service contract should not be an instrument to
circumvent the basic provision, that the exploration and exploitation of natural resources
should be truly for the benefit of Filipinos.

Thank you, and I vote yes.233 [Emphasis supplied.]

x x x.

MR. TADEO. Nais ko lamang ipaliwanag ang aking boto.

Matapos suriin ang kalagayan ng Pilipinas, ang saligang suliranin, pangunahin ang salitang
"imperyalismo." Ang ibig sabihin nito ay ang sistema ng lipunang pinaghaharian ng iilang
monopolyong kapitalista at ang salitang "imperyalismo" ay buhay na buhay sa National
Economy and Patrimony na nating ginawa. Sa pamamagitan ng salitang "based on,"
naroroon na ang free trade sapagkat tayo ay mananatiling tagapagluwas ng hilaw na
sangkap at tagaangkat ng yaring produkto. Pangalawa, naroroon pa rin ang parity rights,
ang service contract, ang 60-40 equity sa natural resources. Habang naghihirap ang
sambayanang Pilipino, ginagalugad naman ng mga dayuhan ang ating likas na yaman.
Kailan man ang Article on National Economy and Patrimony ay hindi nagpaalis sa
pagkaalipin ng ating ekonomiya sa kamay ng mga dayuhan. Ang solusyon sa suliranin ng
bansa ay dalawa lamang: ang pagpapatupad ng tunay na reporma sa lupa at ang national
industrialization. Ito ang tinatawag naming pagsikat ng araw sa Silangan. Ngunit ang mga
landlords and big businessmen at ang mga komprador ay nagsasabi na ang free trade na
ito, ang kahulugan para sa amin, ay ipinipilit sa ating sambayanan na ang araw ay sisikat sa
Kanluran. Kailan man hindi puwedeng sumikat ang araw sa Kanluran. I vote no.234 [Emphasis
supplied.]

This Court is likewise not persuaded.

As earlier noted, the phrase "service contracts" has been deleted in the 1987 Constitution's Article
on National Economy and Patrimony. If the CONCOM intended to retain the concept of service
contracts under the 1973 Constitution, it could have simply adopted the old terminology ("service
contracts") instead of employing new and unfamiliar terms ("agreements . . . involving either
technical or financial assistance"). Such a difference between the language of a provision in a
revised constitution and that of a similar provision in the preceding constitution is viewed as
indicative of a difference in purpose. 235 If, as respondents suggest, the concept of "technical or
financial assistance" agreements is identical to that of "service contracts," the CONCOM would not
have bothered to fit the same dog with a new collar. To uphold respondents' theory would reduce the
first to a mere euphemism for the second and render the change in phraseology meaningless.

An examination of the reason behind the change confirms that technical or financial assistance
agreements are not synonymous to service contracts.

[T]he Court in construing a Constitution should bear in mind the object sought to be accomplished by
its adoption, and the evils, if any, sought to be prevented or remedied. A doubtful provision will be
examined in light of the history of the times, and the condition and circumstances under which the
Constitution was framed. The object is to ascertain the reason which induced the framers of the
Constitution to enact the particular provision and the purpose sought to be accomplished thereby, in
order to construe the whole as to make the words consonant to that reason and calculated to effect
that purpose.236

As the following question of Commissioner Quesada and Commissioner Villegas' answer shows the
drafters intended to do away with service contracts which were used to circumvent the capitalization
(60%-40%) requirement:

MS. QUESADA. The 1973 Constitution used the words "service contracts." In this particular
Section 3, is there a safeguard against the possible control of foreign interests if the Filipinos
go into coproduction with them?

MR. VILLEGAS. Yes. In fact, the deletion of the phrase "service contracts" was our first
attempt to avoid some of the abuses in the past regime in the use of service contracts to go
around the 60-40 arrangement. The safeguard that has been introduced – and this, of
course can be refined – is found in Section 3, lines 25 to 30, where Congress will have to
concur with the President on any agreement entered into between a foreign-owned
corporation and the government involving technical or financial assistance for large-scale
exploration, development and utilization of natural resources.237 [Emphasis supplied.]

In a subsequent discussion, Commissioner Villegas allayed the fears of Commissioner


Quesada regarding the participation of foreign interests in Philippine natural resources,
which was supposed to be restricted to Filipinos.
MS. QUESADA. Another point of clarification is the phrase "and utilization of natural
resources shall be under the full control and supervision of the State." In the 1973
Constitution, this was limited to citizens of the Philippines; but it was removed and
substituted by "shall be under the full control and supervision of the State." Was the concept
changed so that these particular resources would be limited to citizens of the Philippines? Or
would these resources only be under the full control and supervision of the State; meaning,
noncitizens would have access to these natural resources? Is that the understanding?

MR. VILLEGAS. No, Mr. Vice-President, if the Commissioner reads the next sentence, it
states:

Such activities may be directly undertaken by the State, or it may enter into co-production, joint
venture, production-sharing agreements with Filipino citizens.

So we are still limiting it only to Filipino citizens.

x x x.

MS. QUESADA. Going back to Section 3, the section suggests that:

The exploration, development, and utilization of natural resources… may be directly undertaken by
the State, or it may enter into co-production, joint venture or production-sharing agreement with . . .
corporations or associations at least sixty per cent of whose voting stock or controlling interest is
owned by such citizens.

Lines 25 to 30, on the other hand, suggest that in the large-scale exploration, development and
utilization of natural resources, the President with the concurrence of Congress may enter into
agreements with foreign-owned corporations even for technical or financial assistance.

I wonder if this part of Section 3 contradicts the second part. I am raising this point for fear that
foreign investors will use their enormous capital resources to facilitate the actual exploitation or
exploration, development and effective disposition of our natural resources to the detriment of
Filipino investors. I am not saying that we should not consider borrowing money from foreign
sources. What I refer to is that foreign interest should be allowed to participate only to the extent that
they lend us money and give us technical assistance with the appropriate government permit. In this
way, we can insure the enjoyment of our natural resources by our own people.

MR. VILLEGAS. Actually, the second provision about the President does not permit foreign investors
to participate. It is only technical or financial assistance – they do not own anything – but on
conditions that have to be determined by law with the concurrence of Congress. So, it is very
restrictive.

If the Commissioner will remember, this removes the possibility for service contracts which we said
yesterday were avenues used in the previous regime to go around the 60-40
requirement.238 [Emphasis supplied.]

The present Chief Justice, then a member of the CONCOM, also referred to this limitation in scope
in proposing an amendment to the 60-40 requirement:

MR. DAVIDE. May I be allowed to explain the proposal?


MR. MAAMBONG. Subject to the three-minute rule, Madam President.

MR. DAVIDE. It will not take three minutes.

The Commission had just approved the Preamble. In the Preamble we clearly stated that the Filipino
people are sovereign and that one of the objectives for the creation or establishment of a
government is to conserve and develop the national patrimony. The implication is that the national
patrimony or our natural resources are exclusively reserved for the Filipino people. No alien must be
allowed to enjoy, exploit and develop our natural resources. As a matter of fact, that principle
proceeds from the fact that our natural resources are gifts from God to the Filipino people and it
would be a breach of that special blessing from God if we will allow aliens to exploit our natural
resources.

I voted in favor of the Jamir proposal because it is not really exploitation that we granted to the alien
corporations but only for them to render financial or technical assistance. It is not for them to enjoy
our natural resources. Madam President, our natural resources are depleting; our population is
increasing by leaps and bounds. Fifty years from now, if we will allow these aliens to exploit our
natural resources, there will be no more natural resources for the next generations of Filipinos. It
may last long if we will begin now. Since 1935 the aliens have been allowed to enjoy to a certain
extent the exploitation of our natural resources, and we became victims of foreign dominance and
control. The aliens are interested in coming to the Philippines because they would like to enjoy the
bounty of nature exclusively intended for Filipinos by God.

And so I appeal to all, for the sake of the future generations, that if we have to pray in the Preamble
"to preserve and develop the national patrimony for the sovereign Filipino people and for the
generations to come," we must at this time decide once and for all that our natural resources must
be reserved only to Filipino citizens.

Thank you.239 [Emphasis supplied.]

The opinion of another member of the CONCOM is persuasive240 and leaves no doubt as to the
intention of the framers to eliminate service contracts altogether. He writes:

Paragraph 4 of Section 2 specifies large-scale, capital-intensive, highly technological undertakings


for which the President may enter into contracts with foreign-owned corporations, and enunciates
strict conditions that should govern such contracts. x x x.

This provision balances the need for foreign capital and technology with the need to maintain the
national sovereignty. It recognizes the fact that as long as Filipinos can formulate their own terms in
their own territory, there is no danger of relinquishing sovereignty to foreign interests.

Are service contracts allowed under the new Constitution? No. Under the new Constitution, foreign
investors (fully alien-owned) can NOT participate in Filipino enterprises except to provide: (1)
Technical Assistance for highly technical enterprises; and (2) Financial Assistance for large-scale
enterprises.

The intent of this provision, as well as other provisions on foreign investments, is to prevent the
practice (prevalent in the Marcos government) of skirting the 60/40 equation using the cover of
service contracts.241 [Emphasis supplied.]
Furthermore, it appears that Proposed Resolution No. 496,242 which was the draft Article on National
Economy and Patrimony, adopted the concept of "agreements . . . involving either technical or
financial assistance" contained in the "Draft of the 1986 U.P. Law Constitution Project" (U.P. Law
draft) which was taken into consideration during the deliberation of the CONCOM.243 The former, as
well as Article XII, as adopted, employed the same terminology, as the comparative table below
shows:

DRAFT OF THE UP LAW PROPOSED ARTICLE XII OF THE


CONSTITUTION RESOLUTION NO. 496 1987 CONSTITUTION
PROJECT OF THE
CONSTITUTIONAL
COMMISSION

Sec. 1. All lands of the Sec. 3. All lands of the Sec. 2. All lands of the
public domain, waters, public domain, waters, public domain, waters,
minerals, coal, petroleum minerals, coal, petroleum minerals, coal, petroleum,
and other mineral oils, all and other mineral oils, all and other mineral oils, all
forces of potential energy, forces of potential energy, forces of potential energy,
fisheries, flora and fauna fisheries, forests, flora and fisheries, forests or timber,
and other natural fauna, and other natural wildlife, flora and fauna,
resources of the resources are owned by and other natural
Philippines are owned by the State. With the resources are owned by
the State. With the exception of agricultural the State. With the
exception of agricultural lands, all other natural exception of agricultural
lands, all other natural resources shall not be lands, all other natural
resources shall not be alienated. The exploration, resources shall not be
alienated. The exploration, development, and alienated. The exploration,
development and utilization of natural development, and
utilization of natural resources shall be under utilization of natural
resources shall be under the full control and resources shall be under
the full control and supervision of the State. the full control and
supervision of the State. Such activities may be supervision of the State.
Such activities may be directly undertaken by the The State may directly
directly undertaken by the State, or it may enter into undertake such activities
state, or it may enter into co-production, joint or it may enter into co-
co-production, joint venture, production- production, joint venture,
venture, production sharing agreements with or production-sharing
sharing agreements with Filipino citizens or agreements with Filipino
Filipino citizens or corporations or citizens, or corporations or
corporations or associations at least sixty associations at least sixty
associations sixty per cent per cent of whose voting per centum of whose
of whose voting stock or stock or controlling interest capital is owned by such
controlling interest is is owned by such citizens. citizens. Such agreements
owned by such citizens for Such agreements shall be may be for a period not
a period of not more than for a period of twenty-five exceeding twenty-five
twenty-five years, years, renewable for not years, renewable for not
renewable for not more more than twenty-five more than twenty-five
than twenty-five years and years, and under such years, and under such
under such terms and term and conditions as terms and conditions as
conditions as may be may be provided by law. In may be provided by law. In
provided by law. In case cases of water rights for case of water rights for
as to water rights for irrigation, water supply, irrigation, water supply,
irrigation, water supply, fisheries or industrial uses fisheries, or industrial uses
fisheries, or industrial uses other than the other than the
other than the development for water development of water
development of water power, beneficial use may power, beneficial use may
power, beneficial use may be the measure and limit be the measure and limit
be the measure and limit of the grant. of the grant.
of the grant.
The Congress may by law The State shall protect the
The National Assembly allow small-scale nation's marine wealth in
may by law allow small utilization of natural its archipelagic waters,
scale utilization of natural resources by Filipino territorial sea, and
resources by Filipino citizens, as well as exclusive economic zone,
citizens. cooperative fish farming in and reserve its use and
rivers, lakes, bays, and enjoyment exclusively to
The National Assembly, lagoons. Filipino citizens.
may, by two-thirds vote of
all its members by special The President with the The Congress may, by
law provide the terms and concurrence of Congress, law, allow small-scale
conditions under which a by special law, shall utilization of natural
foreign-owned corporation provide the terms and resources by Filipino
may enter into agreements conditions under which a citizens, as well as
with the government foreign-owned corporation cooperative fish farming,
involving either technical may enter into agreements with priority to subsistence
or financial with the government fishermen and fish-
assistance for large-scale involving either technical workers in rivers, lakes,
exploration, development, or financial bays, and lagoons.
or utilization of natural assistance for large-scale
resources. [Emphasis exploration, development, The President may enter
supplied.] and utilization of natural into agreements with
resources. [Emphasis foreign-owned
supplied.] corporations
involving either technical
or financial
assistance for large-scale
exploration, development,
and utilization of minerals,
petroleum, and other
mineral oils according to
the general terms and
conditions provided by
law, based on real
contributions to the
economic growth and
general welfare of the
country. In such
agreements, the State
shall promote the
development and use of
local scientific and
technical resources.
[Emphasis supplied.]

The President shall notify


the Congress of every
contract entered into in
accordance with this
provision, within thirty days
from its execution.

The insights of the proponents of the U.P. Law draft are, therefore, instructive in interpreting the
phrase "technical or financial assistance."

In his position paper entitled Service Contracts: Old Wine in New Bottles?, Professor Pacifico A.
Agabin, who was a member of the working group that prepared the U.P. Law draft, criticized service
contracts for they "lodge exclusive management and control of the enterprise to the service
contractor, which is reminiscent of the old concession regime. Thus, notwithstanding the provision of
the Constitution that natural resources belong to the State, and that these shall not be alienated, the
service contract system renders nugatory the constitutional provisions cited."244 He elaborates:

Looking at the Philippine model, we can discern the following vestiges of the concession regime,
thus:

1. Bidding of a selected area, or leasing the choice of the area to the interested party and
then negotiating the terms and conditions of the contract; (Sec. 5, P.D. 87)

2. Management of the enterprise vested on the contractor, including operation of the field if
petroleum is discovered; (Sec. 8, P.D. 87)

3. Control of production and other matters such as expansion and development; (Sec. 8)

4. Responsibility for downstream operations – marketing, distribution, and processing may


be with the contractor (Sec. 8);

5. Ownership of equipment, machinery, fixed assets, and other properties remain with
contractor (Sec. 12, P.D. 87);

6. Repatriation of capital and retention of profits abroad guaranteed to the contractor (Sec.
13, P.D. 87); and

7. While title to the petroleum discovered may nominally be in the name of the government,
the contractor has almost unfettered control over its disposition and sale, and even the
domestic requirements of the country is relegated to a pro rata basis (Sec. 8).
In short, our version of the service contract is just a rehash of the old concession regime x x x. Some
people have pulled an old rabbit out of a magician's hat, and foisted it upon us as a new and
different animal.

The service contract as we know it here is antithetical to the principle of sovereignty over our natural
resources restated in the same article of the [1973] Constitution containing the provision for service
contracts. If the service contractor happens to be a foreign corporation, the contract would also run
counter to the constitutional provision on nationalization or Filipinization, of the exploitation of our
natural resources.245 [Emphasis supplied. Underscoring in the original.]

Professor Merlin M. Magallona, also a member of the working group, was harsher in his reproach of
the system:

x x x the nationalistic phraseology of the 1935 [Constitution] was retained by the [1973] Charter, but
the essence of nationalism was reduced to hollow rhetoric. The 1973 Charter still provided that the
exploitation or development of the country's natural resources be limited to Filipino citizens or
corporations owned or controlled by them. However, the martial-law Constitution allowed them, once
these resources are in their name, to enter into service contracts with foreign investors for financial,
technical, management, or other forms of assistance. Since foreign investors have the capital
resources, the actual exploitation and development, as well as the effective disposition, of the
country's natural resources, would be under their direction, and control, relegating the Filipino
investors to the role of second-rate partners in joint ventures.

Through the instrumentality of the service contract, the 1973 Constitution had legitimized at the
highest level of state policy that which was prohibited under the 1973 Constitution, namely: the
exploitation of the country's natural resources by foreign nationals. The drastic impact of [this]
constitutional change becomes more pronounced when it is considered that the active party to any
service contract may be a corporation wholly owned by foreign interests. In such a case, the
citizenship requirement is completely set aside, permitting foreign corporations to obtain actual
possession, control, and [enjoyment] of the country's natural resources.246 [Emphasis supplied.]

Accordingly, Professor Agabin recommends that:

Recognizing the service contract for what it is, we have to expunge it from the Constitution and
reaffirm ownership over our natural resources. That is the only way we can exercise effective control
over our natural resources.

This should not mean complete isolation of the country's natural resources from foreign investment.
Other contract forms which are less derogatory to our sovereignty and control over natural resources
– like technical assistance agreements, financial assistance [agreements], co-production
agreements, joint ventures, production-sharing – could still be utilized and adopted without violating
constitutional provisions. In other words, we can adopt contract forms which recognize and assert
our sovereignty and ownership over natural resources, and where the foreign entity is just a pure
contractor instead of the beneficial owner of our economic resources.247 [Emphasis supplied.]

Still another member of the working group, Professor Eduardo Labitag, proposed that:

2. Service contracts as practiced under the 1973 Constitution should be discouraged, instead the
government may be allowed, subject to authorization by special law passed by an extraordinary
majority to enter into either technical or financial assistance. This is justified by the fact that as
presently worded in the 1973 Constitution, a service contract gives full control over the contract area
to the service contractor, for him to work, manage and dispose of the proceeds or production. It was
a subterfuge to get around the nationality requirement of the constitution.248[Emphasis supplied.]

In the annotations on the proposed Article on National Economy and Patrimony, the U.P. Law draft
summarized the rationale therefor, thus:

5. The last paragraph is a modification of the service contract provision found in Section 9, Article
XIV of the 1973 Constitution as amended. This 1973 provision shattered the framework of
nationalism in our fundamental law (see Magallona, "Nationalism and its Subversion in the
Constitution"). Through the service contract, the 1973 Constitution had legitimized that which was
prohibited under the 1935 constitution—the exploitation of the country's natural resources by foreign
nationals. Through the service contract, acts prohibited by the Anti-Dummy Law were recognized as
legitimate arrangements. Service contracts lodge exclusive management and control of the
enterprise to the service contractor, not unlike the old concession regime where the concessionaire
had complete control over the country's natural resources, having been given exclusive and plenary
rights to exploit a particular resource and, in effect, having been assured of ownership of that
resource at the point of extraction (see Agabin, "Service Contracts: Old Wine in New Bottles").
Service contracts, hence, are antithetical to the principle of sovereignty over our natural resources,
as well as the constitutional provision on nationalization or Filipinization of the exploitation of our
natural resources.

Under the proposed provision, only technical assistance or financial assistance agreements may be
entered into, and only for large-scale activities. These are contract forms which recognize and assert
our sovereignty and ownership over natural resources since the foreign entity is just a pure
contractor and not a beneficial owner of our economic resources. The proposal recognizes the need
for capital and technology to develop our natural resources without sacrificing our sovereignty and
control over such resources by the safeguard of a special law which requires two-thirds vote of all
the members of the Legislature. This will ensure that such agreements will be debated upon
exhaustively and thoroughly in the National Assembly to avert prejudice to the nation. 249 [Emphasis
supplied.]

The U.P. Law draft proponents viewed service contracts under the 1973 Constitution as grants of
beneficial ownership of the country's natural resources to foreign owned corporations. While, in
theory, the State owns these natural resources – and Filipino citizens, their beneficiaries – service
contracts actually vested foreigners with the right to dispose, explore for, develop, exploit, and utilize
the same. Foreigners, not Filipinos, became the beneficiaries of Philippine natural resources. This
arrangement is clearly incompatible with the constitutional ideal of nationalization of natural
resources, with the Regalian doctrine, and on a broader perspective, with Philippine sovereignty.

The proponents nevertheless acknowledged the need for capital and technical know-how in the
large-scale exploitation, development and utilization of natural resources – the second paragraph of
the proposed draft itself being an admission of such scarcity. Hence, they recommended a
compromise to reconcile the nationalistic provisions dating back to the 1935 Constitution, which
reserved all natural resources exclusively to Filipinos, and the more liberal 1973 Constitution, which
allowed foreigners to participate in these resources through service contracts. Such a compromise
called for the adoption of a new system in the exploration, development, and utilization of natural
resources in the form of technical agreements or financial agreements which, necessarily, are
distinct concepts from service contracts.

The replacement of "service contracts" with "agreements… involving either technical or financial
assistance," as well as the deletion of the phrase "management or other forms of assistance,"
assumes greater significance when note is taken that the U.P. Law draft proposed other equally
crucial changes that were obviously heeded by the CONCOM. These include the abrogation of the
concession system and the adoption of new "options" for the State in the exploration, development,
and utilization of natural resources. The proponents deemed these changes to be more consistent
with the State's ownership of, and its "full control and supervision" (a phrase also employed by the
framers) over, such resources. The Project explained:

3. In line with the State ownership of natural resources, the State should take a more active role in
the exploration, development, and utilization of natural resources, than the present practice of
granting licenses, concessions, or leases – hence the provision that said activities shall be under the
full control and supervision of the State. There are three major schemes by which the State could
undertake these activities: first, directly by itself; second, by virtue of co-production, joint venture,
production sharing agreements with Filipino citizens or corporations or associations sixty per cent
(60%) of the voting stock or controlling interests of which are owned by such citizens; or third, with a
foreign-owned corporation, in cases of large-scale exploration, development, or utilization of natural
resources through agreements involving either technical or financial assistance only. x x x.

At present, under the licensing concession or lease schemes, the government benefits from such
benefits only through fees, charges, ad valorem taxes and income taxes of the exploiters of our
natural resources. Such benefits are very minimal compared with the enormous profits reaped by
theses licensees, grantees, concessionaires. Moreover, some of them disregard the conservation of
natural resources and do not protect the environment from degradation. The proposed role of the
State will enable it to a greater share in the profits – it can also actively husband its natural
resources and engage in developmental programs that will be beneficial to them.

4. Aside from the three major schemes for the exploration, development, and utilization of our
natural resources, the State may, by law, allow Filipino citizens to explore, develop, utilize natural
resources in small-scale. This is in recognition of the plight of marginal fishermen, forest dwellers,
gold panners, and others similarly situated who exploit our natural resources for their daily
sustenance and survival.250

Professor Agabin, in particular, after taking pains to illustrate the similarities between the two
systems, concluded that the service contract regime was but a "rehash" of the concession system.
"Old wine in new bottles," as he put it. The rejection of the service contract regime, therefore, is in
consonance with the abolition of the concession system.

In light of the deliberations of the CONCOM, the text of the Constitution, and the adoption of other
proposed changes, there is no doubt that the framers considered and shared the intent of the U.P.
Law proponents in employing the phrase "agreements . . . involving either technical or financial
assistance."

While certain commissioners may have mentioned the term "service contracts" during the CONCOM
deliberations, they may not have been necessarily referring to the concept of service contracts under
the 1973 Constitution. As noted earlier, "service contracts" is a term that assumes different
meanings to different people.251 The commissioners may have been using the term loosely, and not
in its technical and legal sense, to refer, in general, to agreements concerning natural resources
entered into by the Government with foreign corporations. These loose statements do not
necessarily translate to the adoption of the 1973 Constitution provision allowing service contracts.

It is true that, as shown in the earlier quoted portions of the proceedings in CONCOM, in response to
Sr. Tan's question, Commissioner Villegas commented that, other than congressional notification,
the only difference between "future" and "past" "service contracts" is the requirement of a general
law as there were no laws previously authorizing the same.252 However, such remark is far
outweighed by his more categorical statement in his exchange with Commissioner Quesada that the
draft article "does not permit foreign investors to participate" in the nation's natural resources – which
was exactly what service contracts did – except to provide "technical or financial assistance."253

In the case of the other commissioners, Commissioner Nolledo himself clarified in his work that the
present charter prohibits service contracts.254 Commissioner Gascon was not totally averse to foreign
participation, but favored stricter restrictions in the form of majority congressional concurrence. 255 On
the other hand, Commissioners Garcia and Tadeo may have veered to the extreme side of the
spectrum and their objections may be interpreted as votes against any foreign participation in our
natural resources whatsoever.

WMCP cites Opinion No. 75, s. 1987,256 and Opinion No. 175, s. 1990257 of the Secretary of Justice,
expressing the view that a financial or technical assistance agreement "is no different in concept"
from the service contract allowed under the 1973 Constitution. This Court is not, however, bound by
this interpretation. When an administrative or executive agency renders an opinion or issues a
statement of policy, it merely interprets a pre-existing law; and the administrative interpretation of the
law is at best advisory, for it is the courts that finally determine what the law means.258

In any case, the constitutional provision allowing the President to enter into FTAAs with foreign-
owned corporations is an exception to the rule that participation in the nation's natural resources is
reserved exclusively to Filipinos. Accordingly, such provision must be construed strictly against their
enjoyment by non-Filipinos. As Commissioner Villegas emphasized, the provision is "very
restrictive."259 Commissioner Nolledo also remarked that "entering into service contracts is an
exception to the rule on protection of natural resources for the interest of the nation and, therefore,
being an exception, it should be subject, whenever possible, to stringent rules."260 Indeed, exceptions
should be strictly but reasonably construed; they extend only so far as their language fairly warrants
and all doubts should be resolved in favor of the general provision rather than the exception.261

With the foregoing discussion in mind, this Court finds that R.A. No. 7942 is invalid insofar as said
Act authorizes service contracts. Although the statute employs the phrase "financial and technical
agreements" in accordance with the 1987 Constitution, it actually treats these agreements as service
contracts that grant beneficial ownership to foreign contractors contrary to the fundamental law.

Section 33, which is found under Chapter VI (Financial or Technical Assistance Agreement) of R.A.
No. 7942 states:

SEC. 33. Eligibility.—Any qualified person with technical and financial capability to undertake large-
scale exploration, development, and utilization of mineral resources in the Philippines may enter into
a financial or technical assistance agreement directly with the Government through the Department.
[Emphasis supplied.]

"Exploration," as defined by R.A. No. 7942,

means the searching or prospecting for mineral resources by geological, geochemical or


geophysical surveys, remote sensing, test pitting, trending, drilling, shaft sinking, tunneling or any
other means for the purpose of determining the existence, extent, quantity and quality thereof and
the feasibility of mining them for profit.262

A legally organized foreign-owned corporation may be granted an exploration permit, 263 which vests it
with the right to conduct exploration for all minerals in specified areas,264 i.e., to enter, occupy and
explore the same.265Eventually, the foreign-owned corporation, as such permittee, may apply for a
financial and technical assistance agreement.266
"Development" is the work undertaken to explore and prepare an ore body or a mineral deposit for
mining, including the construction of necessary infrastructure and related facilities.267

"Utilization" "means the extraction or disposition of minerals."268 A stipulation that the proponent shall
dispose of the minerals and byproducts produced at the highest price and more advantageous terms
and conditions as provided for under the implementing rules and regulations is required to be
incorporated in every FTAA.269

A foreign-owned/-controlled corporation may likewise be granted a mineral processing


permit.270 "Mineral processing" is the milling, beneficiation or upgrading of ores or minerals and rocks
or by similar means to convert the same into marketable products.271

An FTAA contractor makes a warranty that the mining operations shall be conducted in accordance
with the provisions of R.A. No. 7942 and its implementing rules 272 and for work programs and
minimum expenditures and commitments.273 And it obliges itself to furnish the Government records of
geologic, accounting, and other relevant data for its mining operation.274

"Mining operation," as the law defines it, means mining activities involving exploration, feasibility,
development, utilization, and processing.275

The underlying assumption in all these provisions is that the foreign contractor manages the mineral
resources, just like the foreign contractor in a service contract.

Furthermore, Chapter XII of the Act grants foreign contractors in FTAAs the same auxiliary mining
rights that it grants contractors in mineral agreements (MPSA, CA and JV).276 Parenthetically,
Sections 72 to 75 use the term "contractor," without distinguishing between FTAA and mineral
agreement contractors. And so does "holders of mining rights" in Section 76. A foreign contractor
may even convert its FTAA into a mineral agreement if the economic viability of the contract area is
found to be inadequate to justify large-scale mining operations, 277 provided that it reduces its equity
in the corporation, partnership, association or cooperative to forty percent (40%).278

Finally, under the Act, an FTAA contractor warrants that it "has or has access to all the financing,
managerial, and technical expertise. . . ."279 This suggests that an FTAA contractor is bound to
provide some management assistance – a form of assistance that has been eliminated and,
therefore, proscribed by the present Charter.

By allowing foreign contractors to manage or operate all the aspects of the mining operation, the
above-cited provisions of R.A. No. 7942 have in effect conveyed beneficial ownership over the
nation's mineral resources to these contractors, leaving the State with nothing but bare title thereto.

Moreover, the same provisions, whether by design or inadvertence, permit a circumvention of the
constitutionally ordained 60%-40% capitalization requirement for corporations or associations
engaged in the exploitation, development and utilization of Philippine natural resources.

In sum, the Court finds the following provisions of R.A. No. 7942 to be violative of Section 2, Article
XII of the Constitution:

(1) The proviso in Section 3 (aq), which defines "qualified person," to wit:
Provided, That a legally organized foreign-owned corporation shall be deemed a qualified
person for purposes of granting an exploration permit, financial or technical assistance
agreement or mineral processing permit.

(2) Section 23,280 which specifies the rights and obligations of an exploration permittee,
insofar as said section applies to a financial or technical assistance agreement,

(3) Section 33, which prescribes the eligibility of a contractor in a financial or technical
assistance agreement;

(4) Section 35,281 which enumerates the terms and conditions for every financial or technical
assistance agreement;

(5) Section 39,282 which allows the contractor in a financial and technical assistance
agreement to convert the same into a mineral production-sharing agreement;

(6) Section 56,283 which authorizes the issuance of a mineral processing permit to a


contractor in a financial and technical assistance agreement;

The following provisions of the same Act are likewise void as they are dependent on the foregoing
provisions and cannot stand on their own:

(1) Section 3 (g),284 which defines the term "contractor," insofar as it applies to a financial or
technical assistance agreement.

Section 34,285 which prescribes the maximum contract area in a financial or technical


assistance agreements;

Section 36,286 which allows negotiations for financial or technical assistance agreements;

Section 37,287 which prescribes the procedure for filing and evaluation of financial or technical
assistance agreement proposals;

Section 38,288 which limits the term of financial or technical assistance agreements;

Section 40,289 which allows the assignment or transfer of financial or technical assistance


agreements;

Section 41,290 which allows the withdrawal of the contractor in an FTAA;

The second and third paragraphs of Section 81,291 which provide for the Government's share
in a financial and technical assistance agreement; and

Section 90,292 which provides for incentives to contractors in FTAAs insofar as it applies to


said contractors;

When the parts of the statute are so mutually dependent and connected as conditions,
considerations, inducements, or compensations for each other, as to warrant a belief that the
legislature intended them as a whole, and that if all could not be carried into effect, the legislature
would not pass the residue independently, then, if some parts are unconstitutional, all the provisions
which are thus dependent, conditional, or connected, must fall with them.293
There can be little doubt that the WMCP FTAA itself is a service contract.

Section 1.3 of the WMCP FTAA grants WMCP "the exclusive right to explore, exploit, utilise[,]
process and dispose of all Minerals products and by-products thereof that may be produced from the
Contract Area."294 The FTAA also imbues WMCP with the following rights:

(b) to extract and carry away any Mineral samples from the Contract area for the purpose of
conducting tests and studies in respect thereof;

(c) to determine the mining and treatment processes to be utilised during the
Development/Operating Period and the project facilities to be constructed during the
Development and Construction Period;

(d) have the right of possession of the Contract Area, with full right of ingress and egress and
the right to occupy the same, subject to the provisions of Presidential Decree No. 512 (if
applicable) and not be prevented from entry into private ands by surface owners and/or
occupants thereof when prospecting, exploring and exploiting for minerals therein;

xxx

(f) to construct roadways, mining, drainage, power generation and transmission facilities and
all other types of works on the Contract Area;

(g) to erect, install or place any type of improvements, supplies, machinery and other
equipment relating to the Mining Operations and to use, sell or otherwise dispose of, modify,
remove or diminish any and all parts thereof;

(h) enjoy, subject to pertinent laws, rules and regulations and the rights of third Parties,
easement rights and the use of timber, sand, clay, stone, water and other natural resources
in the Contract Area without cost for the purposes of the Mining Operations;

xxx

(i) have the right to mortgage, charge or encumber all or part of its interest and obligations
under this Agreement, the plant, equipment and infrastructure and the Minerals produced
from the Mining Operations;

x x x. 295

All materials, equipment, plant and other installations erected or placed on the Contract Area remain
the property of WMCP, which has the right to deal with and remove such items within twelve months
from the termination of the FTAA.296

Pursuant to Section 1.2 of the FTAA, WMCP shall provide "[all] financing, technology, management
and personnel necessary for the Mining Operations." The mining company binds itself to "perform all
Mining Operations . . . providing all necessary services, technology and financing in connection
therewith,"297 and to "furnish all materials, labour, equipment and other installations that may be
required for carrying on all Mining Operations."298> WMCP may make expansions, improvements and
replacements of the mining facilities and may add such new facilities as it considers necessary for
the mining operations.299
These contractual stipulations, taken together, grant WMCP beneficial ownership over natural
resources that properly belong to the State and are intended for the benefit of its citizens. These
stipulations are abhorrent to the 1987 Constitution. They are precisely the vices that the fundamental
law seeks to avoid, the evils that it aims to suppress. Consequently, the contract from which they
spring must be struck down.

In arguing against the annulment of the FTAA, WMCP invokes the Agreement on the Promotion and
Protection of Investments between the Philippine and Australian Governments, which was signed in
Manila on January 25, 1995 and which entered into force on December 8, 1995.

x x x. Article 2 (1) of said treaty states that it applies to investments whenever made and thus the
fact that [WMCP's] FTAA was entered into prior to the entry into force of the treaty does not preclude
the Philippine Government from protecting [WMCP's] investment in [that] FTAA. Likewise, Article 3
(1) of the treaty provides that "Each Party shall encourage and promote investments in its area by
investors of the other Party and shall [admit] such investments in accordance with its Constitution,
Laws, regulations and investment policies" and in Article 3 (2), it states that "Each Party shall ensure
that investments are accorded fair and equitable treatment." The latter stipulation indicates that it
was intended to impose an obligation upon a Party to afford fair and equitable treatment to the
investments of the other Party and that a failure to provide such treatment by or under the laws of
the Party may constitute a breach of the treaty. Simply stated, the Philippines could not, under said
treaty, rely upon the inadequacies of its own laws to deprive an Australian investor (like [WMCP]) of
fair and equitable treatment by invalidating [WMCP's] FTAA without likewise nullifying the service
contracts entered into before the enactment of RA 7942 such as those mentioned in PD 87 or EO
279.

This becomes more significant in the light of the fact that [WMCP's] FTAA was executed not by a
mere Filipino citizen, but by the Philippine Government itself, through its President no less, which, in
entering into said treaty is assumed to be aware of the existing Philippine laws on service contracts
over the exploration, development and utilization of natural resources. The execution of the FTAA by
the Philippine Government assures the Australian Government that the FTAA is in accordance with
existing Philippine laws.300 [Emphasis and italics by private respondents.]

The invalidation of the subject FTAA, it is argued, would constitute a breach of said treaty which, in
turn, would amount to a violation of Section 3, Article II of the Constitution adopting the generally
accepted principles of international law as part of the law of the land. One of these generally
accepted principles is pacta sunt servanda, which requires the performance in good faith of treaty
obligations.

Even assuming arguendo that WMCP is correct in its interpretation of the treaty and its assertion
that "the Philippines could not . . . deprive an Australian investor (like [WMCP]) of fair and equitable
treatment by invalidating [WMCP's] FTAA without likewise nullifying the service contracts entered
into before the enactment of RA 7942 . . .," the annulment of the FTAA would not constitute a breach
of the treaty invoked. For this decision herein invalidating the subject FTAA forms part of the legal
system of the Philippines.301 The equal protection clause302 guarantees that such decision shall apply
to all contracts belonging to the same class, hence, upholding rather than violating, the "fair and
equitable treatment" stipulation in said treaty.

One other matter requires clarification. Petitioners contend that, consistent with the provisions of
Section 2, Article XII of the Constitution, the President may enter into agreements involving "either
technical or financial assistance" only. The agreement in question, however, is a technical and
financial assistance agreement.
Petitioners' contention does not lie. To adhere to the literal language of the Constitution would lead
to absurd consequences.303 As WMCP correctly put it:

x x x such a theory of petitioners would compel the government (through the President) to enter into
contract with two (2) foreign-owned corporations, one for financial assistance agreement and with
the other, for technical assistance over one and the same mining area or land; or to execute two (2)
contracts with only one foreign-owned corporation which has the capability to provide both financial
and technical assistance, one for financial assistance and another for technical assistance, over the
same mining area. Such an absurd result is definitely not sanctioned under the canons of
constitutional construction.304 [Underscoring in the original.]

Surely, the framers of the 1987 Charter did not contemplate such an absurd result from their use of
"either/or." A constitution is not to be interpreted as demanding the impossible or the impracticable;
and unreasonable or absurd consequences, if possible, should be avoided. 305 Courts are not to give
words a meaning that would lead to absurd or unreasonable consequences and a literal
interpretation is to be rejected if it would be unjust or lead to absurd results.306 That is a strong
argument against its adoption.307 Accordingly, petitioners' interpretation must be rejected.

The foregoing discussion has rendered unnecessary the resolution of the other issues raised by the
petition.

WHEREFORE, the petition is GRANTED. The Court hereby declares unconstitutional and void:

(1) The following provisions of Republic Act No. 7942:

(a) The proviso in Section 3 (aq),

(b) Section 23,

(c) Section 33 to 41,

(d) Section 56,

(e) The second and third paragraphs of Section 81, and

(f) Section 90.

(2) All provisions of Department of Environment and Natural Resources Administrative Order
96-40, s. 1996 which are not in conformity with this Decision, and

(3) The Financial and Technical Assistance Agreement between the Government of the
Republic of the Philippines and WMC Philippines, Inc.

SO ORDERED.

G.R. No. 208566               November 19, 2013


GRECO ANTONIOUS BEDA B. BELGICA JOSE M. VILLEGAS JR. JOSE L. GONZALEZ
REUBEN M. ABANTE and QUINTIN PAREDES SAN DIEGO, Petitioners, 
vs.
HONORABLE EXECUTIVE SECRETARY PAQUITO N. OCHOA JR. SECRETARY OF BUDGET
AND MANAGEMENT FLORENCIO B. ABAD, NATIONAL TREASURER ROSALIA V. DE LEON
SENATE OF THE PHILIPPINES represented by FRANKLIN M. DRILON m his capacity as
SENATE PRESIDENT and HOUSE OF REPRESENTATIVES represented by FELICIANO S.
BELMONTE, JR. in his capacity as SPEAKER OF THE HOUSE, Respondents.

x-----------------------x

G.R. No. 208493

SOCIAL JUSTICE SOCIETY (SJS) PRESIDENT SAMSON S. ALCANTARA, Petitioner, 


vs.
HONORABLE FRANKLIN M. DRILON in his capacity as SENATE PRESIDENT and
HONORABLE FELICIANO S. BELMONTE, JR., in his capacity as SPEAKER OF THE HOUSE
OF REPRESENTATIVES, Respondents.

x-----------------------x

G.R. No. 209251

PEDRITO M. NEPOMUCENO, Former Mayor-Boac, Marinduque Former Provincial Board


Member -Province of Marinduque, Petitioner, 
vs.
PRESIDENT BENIGNO SIMEON C. AQUINO III* and SECRETARY FLORENCIO BUTCH ABAD,
DEPARTMENT OF BUDGET AND MANAGEMENT, Respondents.

DECISION

PERLAS-BERNABE, J.:

"Experience is the oracle of truth." 1

-James Madison

Before the Court are consolidated petitions  taken under Rule 65 of the Rules of Court, all of which
2

assail the constitutionality of the Pork Barrel System. Due to the complexity of the subject matter, the
Court shall heretofore discuss the system‘s conceptual underpinnings before detailing the particulars
of the constitutional challenge.

The Facts

I. Pork Barrel: General Concept.

"Pork Barrel" is political parlance of American -English origin.  Historically, its usage may be
3

traced to the degrading ritual of rolling out a barrel stuffed with pork to a multitude of black
slaves who would cast their famished bodies into the porcine feast to assuage their hunger
with morsels coming from the generosity of their well-fed master.  This practice was later
4

compared to the actions of American legislators in trying to direct federal budgets in favor of
their districts.  While the advent of refrigeration has made the actual pork barrel obsolete, it
5

persists in reference to political bills that "bring home the bacon" to a legislator‘s district and
constituents.  In a more technical sense, "Pork Barrel" refers to an appropriation of
6

government spending meant for localized projects and secured solely or primarily to bring
money to a representative's district. Some scholars on the subject further use it to refer to
7

legislative control of local appropriations. 8

In the Philippines, "Pork Barrel" has been commonly referred to as lump-sum, discretionary
funds of Members of the Legislature,  although, as will be later discussed, its usage would
9

evolve in reference to certain funds of the Executive.

II. History of Congressional Pork Barrel in the Philippines.

A. Pre-Martial Law Era (1922-1972).

Act 3044,  or the Public Works Act of 1922, is considered  as the earliest form of
10 11

"Congressional Pork Barrel" in the Philippines since the utilization of the funds
appropriated therein were subjected to post-enactment legislator approval.
Particularly, in the area of fund release, Section 3  provides that the sums
12

appropriated for certain public works projects  "shall be distributed x x x subject to


13

the approval of a joint committee elected by the Senate and the House of
Representatives. "The committee from each House may also authorize one of its
members to approve the distribution made by the Secretary of Commerce and
Communications."  Also, in the area of fund realignment, the same section provides
14

that the said secretary, "with the approval of said joint committee, or of the
authorized members thereof, may, for the purposes of said distribution, transfer
unexpended portions of any item of appropriation under this Act to any other item
hereunder."

In 1950, it has been documented  that post-enactment legislator participation


15

broadened from the areas of fund release and realignment to the area of project
identification. During that year, the mechanics of the public works act was modified to
the extent that the discretion of choosing projects was transferred from the Secretary
of Commerce and Communications to legislators. "For the first time, the law carried a
list of projects selected by Members of Congress, they ‘being the representatives of
the people, either on their own account or by consultation with local officials or civil
leaders.‘"  During this period, the pork barrel process commenced with local
16

government councils, civil groups, and individuals appealing to Congressmen or


Senators for projects. Petitions that were accommodated formed part of a legislator‘s
allocation, and the amount each legislator would eventually get is determined in a
caucus convened by the majority. The amount was then integrated into the
administration bill prepared by the Department of Public Works and Communications.
Thereafter, the Senate and the House of Representatives added their own provisions
to the bill until it was signed into law by the President – the Public Works Act.  In the
17

1960‘s, however, pork barrel legislation reportedly ceased in view of the stalemate
between the House of Representatives and the Senate. 18

B. Martial Law Era (1972-1986).

While the previous" Congressional Pork Barrel" was apparently discontinued in 1972
after Martial Law was declared, an era when "one man controlled the
legislature,"  the reprieve was only temporary. By 1982, the Batasang Pambansa
19
had already introduced a new item in the General Appropriations Act (GAA) called
the" Support for Local Development Projects" (SLDP) under the article on "National
Aid to Local Government Units". Based on reports,  it was under the SLDP that the
20

practice of giving lump-sum allocations to individual legislators began, with each


assemblyman receiving ₱500,000.00. Thereafter, assemblymen would communicate
their project preferences to the Ministry of Budget and Management for approval.
Then, the said ministry would release the allocation papers to the Ministry of Local
Governments, which would, in turn, issue the checks to the city or municipal
treasurers in the assemblyman‘s locality. It has been further reported that
"Congressional Pork Barrel" projects under the SLDP also began to cover not only
public works projects, or so- called "hard projects", but also "soft projects",  or non-
21

public works projects such as those which would fall under the categories of, among
others, education, health and livelihood. 22

C. Post-Martial Law Era:

Corazon Cojuangco Aquino Administration (1986-1992).

After the EDSA People Power Revolution in 1986 and the restoration of Philippine
democracy, "Congressional Pork Barrel" was revived in the form of the "Mindanao
Development Fund" and the "Visayas Development Fund" which were created with
lump-sum appropriations of ₱480 Million and ₱240 Million, respectively, for the
funding of development projects in the Mindanao and Visayas areas in 1989. It has
been documented  that the clamor raised by the Senators and the Luzon legislators
23

for a similar funding, prompted the creation of the "Countrywide Development Fund"
(CDF) which was integrated into the 1990 GAA  with an initial funding of ₱2.3 Billion
24

to cover "small local infrastructure and other priority community projects."

Under the GAAs for the years 1991 and 1992,  CDF funds were, with the approval of
25

the President, to be released directly to the implementing agencies but "subject to


the submission of the required list of projects and activities."Although the GAAs from
1990 to 1992 were silent as to the amounts of allocations of the individual legislators,
as well as their participation in the identification of projects, it has been reported  that
26

by 1992, Representatives were receiving ₱12.5 Million each in CDF funds, while
Senators were receiving ₱18 Million each, without any limitation or qualification, and
that they could identify any kind of project, from hard or infrastructure projects such
as roads, bridges, and buildings to "soft projects" such as textbooks, medicines, and
scholarships. 27

D. Fidel Valdez Ramos (Ramos) Administration (1992-1998).

The following year, or in 1993,  the GAA explicitly stated that the release of CDF
28

funds was to be made upon the submission of the list of projects and activities
identified by, among others, individual legislators. For the first time, the 1993 CDF
Article included an allocation for the Vice-President.  As such, Representatives were
29

allocated ₱12.5 Million each in CDF funds, Senators, ₱18 Million each, and the Vice-
President, ₱20 Million.

In 1994,  1995,  and 1996,  the GAAs contained the same provisions on project
30 31 32

identification and fund release as found in the 1993 CDF Article. In addition,
however, the Department of Budget and Management (DBM) was directed to submit
reports to the Senate Committee on Finance and the House Committee on
Appropriations on the releases made from the funds. 33

Under the 1997  CDF Article, Members of Congress and the Vice-President, in
34

consultation with the implementing agency concerned, were directed to submit to the
DBM the list of 50% of projects to be funded from their respective CDF allocations
which shall be duly endorsed by (a) the Senate President and the Chairman of the
Committee on Finance, in the case of the Senate, and (b) the Speaker of the House
of Representatives and the Chairman of the Committee on Appropriations, in the
case of the House of Representatives; while the list for the remaining 50% was to be
submitted within six (6) months thereafter. The same article also stated that the
project list, which would be published by the DBM,  "shall be the basis for the release
35

of funds" and that "no funds appropriated herein shall be disbursed for projects not
included in the list herein required."

The following year, or in 1998,  the foregoing provisions regarding the required lists
36

and endorsements were reproduced, except that the publication of the project list
was no longer required as the list itself sufficed for the release of CDF Funds.

The CDF was not, however, the lone form of "Congressional Pork Barrel" at that
time. Other forms of "Congressional Pork Barrel" were reportedly fashioned and
inserted into the GAA (called "Congressional Insertions" or "CIs") in order to
perpetuate the ad ministration‘s political agenda.  It has been articulated that since
37

CIs "formed part and parcel of the budgets of executive departments, they were not
easily identifiable and were thus harder to monitor." Nonetheless, the lawmakers
themselves as well as the finance and budget officials of the implementing agencies,
as well as the DBM, purportedly knew about the insertions.  Examples of these CIs
38

are the Department of Education (DepEd) School Building Fund, the Congressional
Initiative Allocations, the Public Works Fund, the El Niño Fund, and the Poverty
Alleviation Fund.  The allocations for the School Building Fund, particularly, ―shall
39

be made upon prior consultation with the representative of the legislative district
concerned.”  Similarly, the legislators had the power to direct how, where and when
40

these appropriations were to be spent. 41

E. Joseph Ejercito Estrada (Estrada) Administration (1998-2001).

In 1999,  the CDF was removed in the GAA and replaced by three (3) separate
42

forms of CIs, namely, the "Food Security Program Fund,"  the "Lingap Para Sa
43

Mahihirap Program Fund," and the "Rural/Urban Development Infrastructure


44

Program Fund,"  all of which contained a special provision requiring "prior


45

consultation" with the Member s of Congress for the release of the funds.

It was in the year 2000  that the "Priority Development Assistance Fund" (PDAF)
46

appeared in the GAA. The requirement of "prior consultation with the respective
Representative of the District" before PDAF funds were directly released to the
implementing agency concerned was explicitly stated in the 2000 PDAF Article.
Moreover, realignment of funds to any expense category was expressly allowed, with
the sole condition that no amount shall be used to fund personal services and other
personnel benefits.  The succeeding PDAF provisions remained the same in view of
47

the re-enactment  of the 2000 GAA for the year 2001.
48

F. Gloria Macapagal-Arroyo (Arroyo) Administration (2001-2010).


The 2002  PDAF Article was brief and straightforward as it merely contained a single
49

special provision ordering the release of the funds directly to the implementing
agency or local government unit concerned, without further qualifications. The
following year, 2003,  the same single provision was present, with simply an
50

expansion of purpose and express authority to realign. Nevertheless, the provisions


in the 2003 budgets of the Department of Public Works and Highways  (DPWH) and 51

the DepEd  required prior consultation with Members of Congress on the aspects of
52

implementation delegation and project list submission, respectively. In 2004, the


2003 GAA was re-enacted. 53

In 2005,  the PDAF Article provided that the PDAF shall be used "to fund priority
54

programs and projects under the ten point agenda of the national government and
shall be released directly to the implementing agencies." It also introduced the
program menu concept,  which is essentially a list of general programs and
55

implementing agencies from which a particular PDAF project may be subsequently


chosen by the identifying authority. The 2005 GAA was re-enacted  in 2006 and 56

hence, operated on the same bases. In similar regard, the program menu concept
was consistently integrated into the 2007,  2008,  2009,  and 2010  GAAs.
57 58 59 60

Textually, the PDAF Articles from 2002 to 2010 were silent with respect to the
specific amounts allocated for the individual legislators, as well as their participation
in the proposal and identification of PDAF projects to be funded. In contrast to the
PDAF Articles, however, the provisions under the DepEd School Building Program
and the DPWH budget, similar to its predecessors, explicitly required prior
consultation with the concerned Member of Congress anent certain aspects of
61

project implementation.

Significantly, it was during this era that provisions which allowed formal participation
of non-governmental organizations (NGO) in the implementation of government
projects were introduced. In the Supplemental Budget for 2006, with respect to the
appropriation for school buildings, NGOs were, by law, encouraged to participate.
For such purpose, the law stated that "the amount of at least ₱250 Million of the
₱500 Million allotted for the construction and completion of school buildings shall be
made available to NGOs including the Federation of Filipino-Chinese Chambers of
Commerce and Industry, Inc. for its "Operation Barrio School" program, with
capability and proven track records in the construction of public school buildings x x
x."  The same allocation was made available to NGOs in the 2007 and 2009 GAAs
62

under the DepEd Budget.  Also, it was in 2007 that the Government Procurement
63

Policy Board  (GPPB) issued Resolution No. 12-2007 dated June 29, 2007 (GPPB
64

Resolution 12-2007), amending the implementing rules and regulations  of RA 65

9184,  the Government Procurement Reform Act, to include, as a form of negotiated


66

procurement,  the procedure whereby the Procuring Entity (the implementing


67 68

agency) may enter into a memorandum of agreement with an NGO, provided that
"an appropriation law or ordinance earmarks an amount to be specifically contracted
out to NGOs." 69

G. Present Administration (2010-Present).

Differing from previous PDAF Articles but similar to the CDF Articles, the
2011  PDAF Article included an express statement on lump-sum amounts allocated
70

for individual legislators and the Vice-President: Representatives were given ₱70
Million each, broken down into ₱40 Million for "hard projects" and ₱30 Million for "soft
projects"; while ₱200 Million was given to each Senator as well as the Vice-
President, with a ₱100 Million allocation each for "hard" and "soft projects." Likewise,
a provision on realignment of funds was included, but with the qualification that it
may be allowed only once. The same provision also allowed the Secretaries of
Education, Health, Social Welfare and Development, Interior and Local Government,
Environment and Natural Resources, Energy, and Public Works and Highways to
realign PDAF Funds, with the further conditions that: (a) realignment is within the
same implementing unit and same project category as the original project, for
infrastructure projects; (b) allotment released has not yet been obligated for the
original scope of work, and (c) the request for realignment is with the concurrence of
the legislator concerned. 71

In the 2012  and 2013  PDAF Articles, it is stated that the "identification of projects
72 73

and/or designation of beneficiaries shall conform to the priority list, standard or


design prepared by each implementing agency (priority list requirement) x x x."
However, as practiced, it would still be the individual legislator who would choose
and identify the project from the said priority list.
74

Provisions on legislator allocations  as well as fund realignment  were included in


75 76

the 2012 and 2013 PDAF Articles; but the allocation for the Vice-President, which
was pegged at ₱200 Million in the 2011 GAA, had been deleted. In addition, the
2013 PDAF Article now allowed LGUs to be identified as implementing agencies if
they have the technical capability to implement the projects.  Legislators were also
77

allowed to identify programs/projects, except for assistance to indigent patients and


scholarships, outside of his legislative district provided that he secures the written
concurrence of the legislator of the intended outside-district, endorsed by the
Speaker of the House.  Finally, any realignment of PDAF funds, modification and
78

revision of project identification, as well as requests for release of funds, were all
required to be favorably endorsed by the House Committee on Appropriations and
the Senate Committee on Finance, as the case may be. 79

III. History of Presidential Pork Barrel in the Philippines.

While the term "Pork Barrel" has been typically associated with lump-sum, discretionary
funds of Members of Congress, the present cases and the recent controversies on the
matter have, however, shown that the term‘s usage has expanded to include certain funds of
the President such as the Malampaya Funds and the Presidential Social Fund.

On the one hand, the Malampaya Funds was created as a special fund under Section 8  of 80

Presidential Decree No. (PD) 910,  issued by then President Ferdinand E. Marcos (Marcos)
81

on March 22, 1976. In enacting the said law, Marcos recognized the need to set up a special
fund to help intensify, strengthen, and consolidate government efforts relating to the
exploration, exploitation, and development of indigenous energy resources vital to economic
growth.  Due to the energy-related activities of the government in the Malampaya natural
82

gas field in Palawan, or the "Malampaya Deep Water Gas-to-Power Project",  the special
83

fund created under PD 910 has been currently labeled as Malampaya Funds.

On the other hand the Presidential Social Fund was created under Section 12, Title IV  of 84

PD 1869,  or the Charter of the Philippine Amusement and Gaming Corporation (PAGCOR).
85

PD 1869 was similarly issued by Marcos on July 11, 1983. More than two (2) years after, he
amended PD 1869 and accordingly issued PD 1993 on October 31, 1985,  amending 86

Section 12  of the former law. As it stands, the Presidential Social Fund has been described
87
as a special funding facility managed and administered by the Presidential Management
Staff through which the President provides direct assistance to priority programs and projects
not funded under the regular budget. It is sourced from the share of the government in the
aggregate gross earnings of PAGCOR. 88

IV. Controversies in the Philippines.

Over the decades, "pork" funds in the Philippines have increased tremendously,  owing in no
89

small part to previous Presidents who reportedly used the "Pork Barrel" in order to gain
congressional support.  It was in 1996 when the first controversy surrounding the "Pork
90

Barrel" erupted. Former Marikina City Representative Romeo Candazo (Candazo), then an
anonymous source, "blew the lid on the huge sums of government money that regularly went
into the pockets of legislators in the form of kickbacks."  He said that "the kickbacks were
91

‘SOP‘ (standard operating procedure) among legislators and ranged from a low 19 percent to
a high 52 percent of the cost of each project, which could be anything from dredging, rip
rapping, sphalting, concreting, and construction of school buildings."  "Other sources of
92

kickbacks that Candazo identified were public funds intended for medicines and textbooks. A
few days later, the tale of the money trail became the banner story of the Philippine Daily
Inquirer issue of August 13, 1996, accompanied by an illustration of a roasted pig."  "The 93

publication of the stories, including those about congressional initiative allocations of certain
lawmakers, including ₱3.6 Billion for a Congressman, sparked public outrage." 94

Thereafter, or in 2004, several concerned citizens sought the nullification of the PDAF as
enacted in the 2004 GAA for being unconstitutional. Unfortunately, for lack of "any pertinent
evidentiary support that illegal misuse of PDAF in the form of kickbacks has become a
common exercise of unscrupulous Members of Congress," the petition was dismissed. 95

Recently, or in July of the present year, the National Bureau of Investigation (NBI) began its
probe into allegations that "the government has been defrauded of some ₱10 Billion over the
past 10 years by a syndicate using funds from the pork barrel of lawmakers and various
government agencies for scores of ghost projects."  The investigation was spawned by
96

sworn affidavits of six (6) whistle-blowers who declared that JLN Corporation – "JLN"
standing for Janet Lim Napoles (Napoles) – had swindled billions of pesos from the public
coffers for "ghost projects" using no fewer than 20 dummy NGOs for an entire decade. While
the NGOs were supposedly the ultimate recipients of PDAF funds, the whistle-blowers
declared that the money was diverted into Napoles‘ private accounts.  Thus, after its
97

investigation on the Napoles controversy, criminal complaints were filed before the Office of
the Ombudsman, charging five (5) lawmakers for Plunder, and three (3) other lawmakers for
Malversation, Direct Bribery, and Violation of the Anti-Graft and Corrupt Practices Act. Also
recommended to be charged in the complaints are some of the lawmakers‘ chiefs -of-staff or
representatives, the heads and other officials of three (3) implementing agencies, and the
several presidents of the NGOs set up by Napoles. 98

On August 16, 2013, the Commission on Audit (CoA) released the results of a three-year
audit investigation covering the use of legislators' PDAF from 2007 to 2009, or during the
99

last three (3) years of the Arroyo administration. The purpose of the audit was to determine
the propriety of releases of funds under PDAF and the Various Infrastructures including
Local Projects (VILP)  by the DBM, the application of these funds and the implementation of
100

projects by the appropriate implementing agencies and several government-owned-and-


controlled corporations (GOCCs).  The total releases covered by the audit amounted to
101

₱8.374 Billion in PDAF and ₱32.664 Billion in VILP, representing 58% and 32%,
respectively, of the total PDAF and VILP releases that were found to have been made
nationwide during the audit period.  Accordingly, the Co A‘s findings contained in its Report
102

No. 2012-03 (CoA Report), entitled "Priority Development Assistance Fund (PDAF) and
Various Infrastructures including Local Projects (VILP)," were made public, the highlights of
which are as follows:103

● Amounts released for projects identified by a considerable number of legislators


significantly exceeded their respective allocations.

● Amounts were released for projects outside of legislative districts of sponsoring


members of the Lower House.

● Total VILP releases for the period exceeded the total amount appropriated under
the 2007 to 2009 GAAs.

● Infrastructure projects were constructed on private lots without these having been
turned over to the government.

● Significant amounts were released to implementing agencies without the latter‘s


endorsement and without considering their mandated functions, administrative and
technical capabilities to implement projects.

● Implementation of most livelihood projects was not undertaken by the


implementing agencies themselves but by NGOs endorsed by the proponent
legislators to which the Funds were transferred.

● The funds were transferred to the NGOs in spite of the absence of any
appropriation law or ordinance.

● Selection of the NGOs were not compliant with law and regulations.

● Eighty-Two (82) NGOs entrusted with implementation of seven hundred seventy


two (772) projects amount to ₱6.156 Billion were either found questionable, or
submitted questionable/spurious documents, or failed to liquidate in whole or in part
their utilization of the Funds.

● Procurement by the NGOs, as well as some implementing agencies, of goods and


services reportedly used in the projects were not compliant with law.

As for the "Presidential Pork Barrel", whistle-blowers alleged that" at least ₱900 Million from
royalties in the operation of the Malampaya gas project off Palawan province intended for
agrarian reform beneficiaries has gone into a dummy NGO."  According to incumbent CoA
104

Chairperson Maria Gracia Pulido Tan (CoA Chairperson), the CoA is, as of this writing, in the
process of preparing "one consolidated report" on the Malampaya Funds. 105

V. The Procedural Antecedents.

Spurred in large part by the findings contained in the CoA Report and the Napoles
controversy, several petitions were lodged before the Court similarly seeking that the "Pork
Barrel System" be declared unconstitutional. To recount, the relevant procedural
antecedents in these cases are as follows:
On August 28, 2013, petitioner Samson S. Alcantara (Alcantara), President of the Social Justice
Society, filed a Petition for Prohibition of even date under Rule 65 of the Rules of Court (Alcantara
Petition), seeking that the "Pork Barrel System" be declared unconstitutional, and a writ of prohibition
be issued permanently restraining respondents Franklin M. Drilon and Feliciano S. Belmonte, Jr., in
their respective capacities as the incumbent Senate President and Speaker of the House of
Representatives, from further taking any steps to enact legislation appropriating funds for the "Pork
Barrel System," in whatever form and by whatever name it may be called, and from approving further
releases pursuant thereto.  The Alcantara Petition was docketed as G.R. No. 208493.
106

On September 3, 2013, petitioners Greco Antonious Beda B. Belgica, Jose L. Gonzalez, Reuben M.
Abante, Quintin Paredes San Diego (Belgica, et al.), and Jose M. Villegas, Jr. (Villegas) filed an
Urgent Petition For Certiorari and Prohibition With Prayer For The Immediate Issuance of Temporary
Restraining Order (TRO) and/or Writ of Preliminary Injunction dated August 27, 2013 under Rule 65
of the Rules of Court (Belgica Petition), seeking that the annual "Pork Barrel System," presently
embodied in the provisions of the GAA of 2013 which provided for the 2013 PDAF, and the
Executive‘s lump-sum, discretionary funds, such as the Malampaya Funds and the Presidential
Social Fund,  be declared unconstitutional and null and void for being acts constituting grave abuse
107

of discretion. Also, they pray that the Court issue a TRO against respondents Paquito N. Ochoa, Jr.,
Florencio B. Abad (Secretary Abad) and Rosalia V. De Leon, in their respective capacities as the
incumbent Executive Secretary, Secretary of the Department of Budget and Management (DBM),
and National Treasurer, or their agents, for them to immediately cease any expenditure under the
aforesaid funds. Further, they pray that the Court order the foregoing respondents to release to the
CoA and to the public: (a) "the complete schedule/list of legislators who have availed of their PDAF
and VILP from the years 2003 to 2013, specifying the use of the funds, the project or activity and the
recipient entities or individuals, and all pertinent data thereto"; and (b) "the use of the Executive‘s
lump-sum, discretionary funds, including the proceeds from the x x x Malampaya Funds and
remittances from the PAGCOR x x x from 2003 to 2013, specifying the x x x project or activity and
the recipient entities or individuals, and all pertinent data thereto."  Also, they pray for the "inclusion
108

in budgetary deliberations with the Congress of all presently off-budget, lump-sum, discretionary
funds including, but not limited to, proceeds from the Malampaya Funds and remittances from the
PAGCOR."  The Belgica Petition was docketed as G.R. No. 208566.
109 110

Lastly, on September 5, 2013, petitioner Pedrito M. Nepomuceno (Nepomuceno), filed a Petition


dated August 23, 2012 (Nepomuceno Petition), seeking that the PDAF be declared unconstitutional,
and a cease and desist order be issued restraining President Benigno Simeon S. Aquino III
(President Aquino) and Secretary Abad from releasing such funds to Members of Congress and,
instead, allow their release to fund priority projects identified and approved by the Local
Development Councils in consultation with the executive departments, such as the DPWH, the
Department of Tourism, the Department of Health, the Department of Transportation, and
Communication and the National Economic Development Authority.  The Nepomuceno Petition was
111

docketed as UDK-14951. 112

On September 10, 2013, the Court issued a Resolution of even date (a) consolidating all cases; (b)
requiring public respondents to comment on the consolidated petitions; (c) issuing a TRO
(September 10, 2013 TRO) enjoining the DBM, National Treasurer, the Executive Secretary, or any
of the persons acting under their authority from releasing (1) the remaining PDAF allocated to
Members of Congress under the GAA of 2013, and (2) Malampaya Funds under the phrase "for
such other purposes as may be hereafter directed by the President" pursuant to Section 8 of PD 910
but not for the purpose of "financing energy resource development and exploitation programs and
projects of the government‖ under the same provision; and (d) setting the consolidated cases for
Oral Arguments on October 8, 2013.
On September 23, 2013, the Office of the Solicitor General (OSG) filed a Consolidated Comment
(Comment) of even date before the Court, seeking the lifting, or in the alternative, the partial lifting
with respect to educational and medical assistance purposes, of the Court‘s September 10, 2013
TRO, and that the consolidated petitions be dismissed for lack of merit. 113

On September 24, 2013, the Court issued a Resolution of even date directing petitioners to reply to
the Comment.

Petitioners, with the exception of Nepomuceno, filed their respective replies to the Comment: (a) on
September 30, 2013, Villegas filed a separate Reply dated September 27, 2013 (Villegas Reply); (b)
on October 1, 2013, Belgica, et al. filed a Reply dated September 30, 2013 (Belgica Reply); and (c)
on October 2, 2013, Alcantara filed a Reply dated October 1, 2013.

On October 1, 2013, the Court issued an Advisory providing for the guidelines to be observed by the
parties for the Oral Arguments scheduled on October 8, 2013. In view of the technicality of the
issues material to the present cases, incumbent Solicitor General Francis H. Jardeleza (Solicitor
General) was directed to bring with him during the Oral Arguments representative/s from the DBM
and Congress who would be able to competently and completely answer questions related to,
among others, the budgeting process and its implementation. Further, the CoA Chairperson was
appointed as amicus curiae and thereby requested to appear before the Court during the Oral
Arguments.

On October 8 and 10, 2013, the Oral Arguments were conducted. Thereafter, the Court directed the
parties to submit their respective memoranda within a period of seven (7) days, or until October 17,
2013, which the parties subsequently did.

The Issues Before the Court

Based on the pleadings, and as refined during the Oral Arguments, the following are the main issues
for the Court‘s resolution:

I. Procedural Issues.

Whether or not (a) the issues raised in the consolidated petitions involve an actual and justiciable
controversy; (b) the issues raised in the consolidated petitions are matters of policy not subject to
judicial review; (c) petitioners have legal standing to sue; and (d) the Court‘s Decision dated August
19, 1994 in G.R. Nos. 113105, 113174, 113766, and 113888, entitled "Philippine Constitution
Association v. Enriquez"  (Philconsa) and Decision dated April 24, 2012 in G.R. No. 164987,
114

entitled "Lawyers Against Monopoly and Poverty v. Secretary of Budget and Management"  (LAMP) 115

bar the re-litigatio n of the issue of constitutionality of the "Pork Barrel System" under the principles
of res judicata and stare decisis.

II. Substantive Issues on the "Congressional Pork Barrel."

Whether or not the 2013 PDAF Article and all other Congressional Pork Barrel Laws similar thereto
are unconstitutional considering that they violate the principles of/constitutional provisions on (a)
separation of powers; (b) non-delegability of legislative power; (c) checks and balances; (d)
accountability; (e) political dynasties; and (f) local autonomy.

III. Substantive Issues on the "Presidential Pork Barrel."


Whether or not the phrases (a) "and for such other purposes as may be hereafter directed by the
President" under Section 8 of PD 910,  relating to the Malampaya Funds, and (b) "to finance the
116

priority infrastructure development projects and to finance the restoration of damaged or destroyed
facilities due to calamities, as may be directed and authorized by the Office of the President of the
Philippines" under Section 12 of PD 1869, as amended by PD 1993, relating to the Presidential
Social Fund, are unconstitutional insofar as they constitute undue delegations of legislative power.

These main issues shall be resolved in the order that they have been stated. In addition, the Court
shall also tackle certain ancillary issues as prompted by the present cases.

The Court’s Ruling

The petitions are partly granted.

I. Procedural Issues.

The prevailing rule in constitutional litigation is that no question involving the constitutionality or
validity of a law or governmental act may be heard and decided by the Court unless there is
compliance with the legal requisites for judicial inquiry,  namely: (a) there must be an actual case or
117

controversy calling for the exercise of judicial power; (b) the person challenging the act must have
the standing to question the validity of the subject act or issuance; (c) the question of constitutionality
must be raised at the earliest opportunity ; and (d) the issue of constitutionality must be the very lis
mota of the case.  Of these requisites, case law states that the first two are the most important and,
118 119

therefore, shall be discussed forthwith.

A. Existence of an Actual Case or Controversy.

By constitutional fiat, judicial power operates only when there is an actual case or controversy.  This
120

is embodied in Section 1, Article VIII of the 1987 Constitution which pertinently states that "judicial
power includes the duty of the courts of justice to settle actual controversies involving rights which
are legally demandable and enforceable x x x." Jurisprudence provides that an actual case or
controversy is one which "involves a conflict of legal rights, an assertion of opposite legal claims,
susceptible of judicial resolution as distinguished from a hypothetical or abstract difference or
dispute.  In other words, "there must be a contrariety of legal rights that can be interpreted and
121

enforced on the basis of existing law and jurisprudence."  Related to the requirement of an actual
122

case or controversy is the requirement of "ripeness," meaning that the questions raised for
constitutional scrutiny are already ripe for adjudication. "A question is ripe for adjudication when the
act being challenged has had a direct adverse effect on the individual challenging it. It is a
prerequisite that something had then been accomplished or performed by either branch before a
court may come into the picture, and the petitioner must allege the existence of an immediate or
threatened injury to itself as a result of the challenged action."  "Withal, courts will decline to pass
123

upon constitutional issues through advisory opinions, bereft as they are of authority to resolve
hypothetical or moot questions." 124

Based on these principles, the Court finds that there exists an actual and justiciable controversy in
these cases.

The requirement of contrariety of legal rights is clearly satisfied by the antagonistic positions of the
parties on the constitutionality of the "Pork Barrel System." Also, the questions in these consolidated
cases are ripe for adjudication since the challenged funds and the provisions allowing for their
utilization – such as the 2013 GAA for the PDAF, PD 910 for the Malampaya Funds and PD 1869,
as amended by PD 1993, for the Presidential Social Fund – are currently existing and operational;
hence, there exists an immediate or threatened injury to petitioners as a result of the unconstitutional
use of these public funds.

As for the PDAF, the Court must dispel the notion that the issues related thereto had been rendered
moot and academic by the reforms undertaken by respondents. A case becomes moot when there is
no more actual controversy between the parties or no useful purpose can be served in passing upon
the merits.  Differing from this description, the Court observes that respondents‘ proposed line-item
125

budgeting scheme would not terminate the controversy nor diminish the useful purpose for its
resolution since said reform is geared towards the 2014 budget, and not the 2013 PDAF Article
which, being a distinct subject matter, remains legally effective and existing. Neither will the
President‘s declaration that he had already "abolished the PDAF" render the issues on PDAF moot
precisely because the Executive branch of government has no constitutional authority to nullify or
annul its legal existence. By constitutional design, the annulment or nullification of a law may be
done either by Congress, through the passage of a repealing law, or by the Court, through a
declaration of unconstitutionality. Instructive on this point is the following exchange between
Associate Justice Antonio T. Carpio (Justice Carpio) and the Solicitor General during the Oral
Arguments: 126

Justice Carpio: The President has taken an oath to faithfully execute the law,  correct? Solicitor
127

General Jardeleza: Yes, Your Honor.

Justice Carpio: And so the President cannot refuse to implement the General Appropriations Act,
correct?

Solicitor General Jardeleza: Well, that is our answer, Your Honor. In the case, for example of the
PDAF, the President has a duty to execute the laws but in the face of the outrage over PDAF, the
President was saying, "I am not sure that I will continue the release of the soft projects," and that
started, Your Honor. Now, whether or not that … (interrupted)

Justice Carpio: Yeah. I will grant the President if there are anomalies in the project, he has the
power to stop the releases in the meantime, to investigate, and that is Section 38 of Chapter 5 of
Book 6 of the Revised Administrative Code  x x x. So at most the President can suspend, now if the
128

President believes that the PDAF is unconstitutional, can he just refuse to implement it?

Solicitor General Jardeleza: No, Your Honor, as we were trying to say in the specific case of the
PDAF because of the CoA Report, because of the reported irregularities and this Court can take
judicial notice, even outside, outside of the COA Report, you have the report of the whistle-blowers,
the President was just exercising precisely the duty ….

xxxx

Justice Carpio: Yes, and that is correct. You‘ve seen the CoA Report, there are anomalies, you stop
and investigate, and prosecute, he has done that. But, does that mean that PDAF has been
repealed?

Solicitor General Jardeleza: No, Your Honor x x x.

xxxx

Justice Carpio: So that PDAF can be legally abolished only in two (2) cases. Congress passes a law
to repeal it, or this Court declares it unconstitutional, correct?
Solictor General Jardeleza: Yes, Your Honor.

Justice Carpio: The President has no power to legally abolish PDAF. (Emphases supplied)

Even on the assumption of mootness, jurisprudence, nevertheless, dictates that "the moot and
academic‘ principle is not a magical formula that can automatically dissuade the Court in resolving a
case." The Court will decide cases, otherwise moot, if: first, there is a grave violation of the
Constitution; second, the exceptional character of the situation and the paramount public interest is
involved; third, when the constitutional issue raised requires formulation of controlling principles to
guide the bench, the bar, and the public; and fourth, the case is capable of repetition yet evading
review.129

The applicability of the first exception is clear from the fundamental posture of petitioners – they
essentially allege grave violations of the Constitution with respect to, inter alia, the principles of
separation of powers, non-delegability of legislative power, checks and balances, accountability and
local autonomy.

The applicability of the second exception is also apparent from the nature of the interests involved

– the constitutionality of the very system within which significant amounts of public funds have been
and continue to be utilized and expended undoubtedly presents a situation of exceptional character
as well as a matter of paramount public interest. The present petitions, in fact, have been lodged at a
time when the system‘s flaws have never before been magnified. To the Court‘s mind, the
coalescence of the CoA Report, the accounts of numerous whistle-blowers, and the government‘s
own recognition that reforms are needed "to address the reported abuses of the
PDAF"  demonstrates a prima facie pattern of abuse which only underscores the importance of the
130

matter. It is also by this finding that the Court finds petitioners‘ claims as not merely theorized,
speculative or hypothetical. Of note is the weight accorded by the Court to the findings made by the
CoA which is the constitutionally-mandated audit arm of the government. In Delos Santos v. CoA,  a 131

recent case wherein the Court upheld the CoA‘s disallowance of irregularly disbursed PDAF funds, it
was emphasized that:

The COA is endowed with enough latitude to determine, prevent, and disallow irregular,
unnecessary, excessive, extravagant or unconscionable expenditures of government funds. It is
tasked to be vigilant and conscientious in safeguarding the proper use of the government's, and
ultimately the people's, property. The exercise of its general audit power is among the constitutional
mechanisms that gives life to the check and balance system inherent in our form of government.

It is the general policy of the Court to sustain the decisions of administrative authorities, especially
one which is constitutionally-created, such as the CoA, not only on the basis of the doctrine of
separation of powers but also for their presumed expertise in the laws they are entrusted to enforce.
Findings of administrative agencies are accorded not only respect but also finality when the decision
and order are not tainted with unfairness or arbitrariness that would amount to grave abuse of
discretion. It is only when the CoA has acted without or in excess of jurisdiction, or with grave abuse
of discretion amounting to lack or excess of jurisdiction, that this Court entertains a petition
questioning its rulings. x x x. (Emphases supplied)

Thus, if only for the purpose of validating the existence of an actual and justiciable controversy in
these cases, the Court deems the findings under the CoA Report to be sufficient.

The Court also finds the third exception to be applicable largely due to the practical need for a
definitive ruling on the system‘s constitutionality. As disclosed during the Oral Arguments, the CoA
Chairperson estimates that thousands of notices of disallowances will be issued by her office in
connection with the findings made in the CoA Report. In this relation, Associate Justice Marvic Mario
Victor F. Leonen (Justice Leonen) pointed out that all of these would eventually find their way to the
courts.  Accordingly, there is a compelling need to formulate controlling principles relative to the
132

issues raised herein in order to guide the bench, the bar, and the public, not just for the expeditious
resolution of the anticipated disallowance cases, but more importantly, so that the government may
be guided on how public funds should be utilized in accordance with constitutional principles.

Finally, the application of the fourth exception is called for by the recognition that the preparation and
passage of the national budget is, by constitutional imprimatur, an affair of annual occurrence.  The
133

relevance of the issues before the Court does not cease with the passage of a "PDAF -free budget
for 2014."  The evolution of the "Pork Barrel System," by its multifarious iterations throughout the
134

course of history, lends a semblance of truth to petitioners‘ claim that "the same dog will just
resurface wearing a different collar."  In Sanlakas v. Executive Secretary,  the government had
135 136

already backtracked on a previous course of action yet the Court used the "capable of repetition but
evading review" exception in order "to prevent similar questions from re- emerging."  The situation
137

similarly holds true to these cases. Indeed, the myriad of issues underlying the manner in which
certain public funds are spent, if not resolved at this most opportune time, are capable of repetition
and hence, must not evade judicial review.

B. Matters of Policy: the Political Question Doctrine.

The "limitation on the power of judicial review to actual cases and controversies‖ carries the
assurance that "the courts will not intrude into areas committed to the other branches of
government."  Essentially, the foregoing limitation is a restatement of the political question doctrine
138

which, under the classic formulation of Baker v. Carr, applies when there is found, among others, "a
139

textually demonstrable constitutional commitment of the issue to a coordinate political department,"


"a lack of judicially discoverable and manageable standards for resolving it" or "the impossibility of
deciding without an initial policy determination of a kind clearly for non- judicial discretion." Cast
against this light, respondents submit that the "the political branches are in the best position not only
to perform budget-related reforms but also to do them in response to the specific demands of their
constituents" and, as such, "urge the Court not to impose a solution at this stage." 140

The Court must deny respondents‘ submission.

Suffice it to state that the issues raised before the Court do not present political but legal questions
which are within its province to resolve. A political question refers to "those questions which, under
the Constitution, are to be decided by the people in their sovereign capacity, or in regard to which full
discretionary authority has been delegated to the Legislature or executive branch of the
Government. It is concerned with issues dependent upon the wisdom, not legality, of a particular
measure."  The intrinsic constitutionality of the "Pork Barrel System" is not an issue dependent upon
141

the wisdom of the political branches of government but rather a legal one which the Constitution
itself has commanded the Court to act upon. Scrutinizing the contours of the system along
constitutional lines is a task that the political branches of government are incapable of rendering
precisely because it is an exercise of judicial power. More importantly, the present Constitution has
not only vested the Judiciary the right to exercise judicial power but essentially makes it a duty to
proceed therewith. Section 1, Article VIII of the 1987 Constitution cannot be any clearer: "The judicial
power shall be vested in one Supreme Court and in such lower courts as may be established by law.
It includes the duty of the courts of justice to settle actual controversies involving rights which are
legally demandable and enforceable, and to determine whether or not there has been a grave abuse
of discretion amounting to lack or excess of jurisdiction on the part of any branch or instrumentality
of the Government." In Estrada v. Desierto,  the expanded concept of judicial power under the 1987
142

Constitution and its effect on the political question doctrine was explained as follows:143

To a great degree, the 1987 Constitution has narrowed the reach of the political question doctrine
when it expanded the power of judicial review of this court not only to settle actual controversies
involving rights which are legally demandable and enforceable but also to determine whether or not
there has been a grave abuse of discretion amounting to lack or excess of jurisdiction on the part of
any branch or instrumentality of government. Heretofore, the judiciary has focused on the "thou shalt
not's" of the Constitution directed against the exercise of its jurisdiction. With the new provision,
however, courts are given a greater prerogative to determine what it can do to prevent grave abuse
of discretion amounting to lack or excess of jurisdiction on the part of any branch or instrumentality
of government. Clearly, the new provision did not just grant the Court power of doing nothing. x x x
(Emphases supplied)

It must also be borne in mind that ― when the judiciary mediates to allocate constitutional
boundaries, it does not assert any superiority over the other departments; does not in reality nullify
or invalidate an act of the legislature or the executive, but only asserts the solemn and sacred
obligation assigned to it by the Constitution."  To a great extent, the Court is laudably cognizant of
144

the reforms undertaken by its co-equal branches of government. But it is by constitutional force that
the Court must faithfully perform its duty. Ultimately, it is the Court‘s avowed intention that a
resolution of these cases would not arrest or in any manner impede the endeavors of the two other
branches but, in fact, help ensure that the pillars of change are erected on firm constitutional
grounds. After all, it is in the best interest of the people that each great branch of government, within
its own sphere, contributes its share towards achieving a holistic and genuine solution to the
problems of society. For all these reasons, the Court cannot heed respondents‘ plea for judicial
restraint.

C. Locus Standi.

"The gist of the question of standing is whether a party alleges such personal stake in the outcome
of the controversy as to assure that concrete adverseness which sharpens the presentation of
issues upon which the court depends for illumination of difficult constitutional questions. Unless a
person is injuriously affected in any of his constitutional rights by the operation of statute or
ordinance, he has no standing." 145

Petitioners have come before the Court in their respective capacities as citizen-taxpayers and
accordingly, assert that they "dutifully contribute to the coffers of the National Treasury."  Clearly, as
146

taxpayers, they possess the requisite standing to question the validity of the existing "Pork Barrel
System" under which the taxes they pay have been and continue to be utilized. It is undeniable that
petitioners, as taxpayers, are bound to suffer from the unconstitutional usage of public funds, if the
Court so rules. Invariably, taxpayers have been allowed to sue where there is a claim that public
funds are illegally disbursed or that public money is being deflected to any improper purpose, or that
public funds are wasted through the enforcement of an invalid or unconstitutional law,  as in these
147

cases.

Moreover, as citizens, petitioners have equally fulfilled the standing requirement given that the
issues they have raised may be classified as matters "of transcendental importance, of overreaching
significance to society, or of paramount public interest."  The CoA Chairperson‘s statement during
148

the Oral Arguments that the present controversy involves "not merely a systems failure" but a
"complete breakdown of controls"  amplifies, in addition to the matters above-discussed, the
149

seriousness of the issues involved herein. Indeed, of greater import than the damage caused by the
illegal expenditure of public funds is the mortal wound inflicted upon the fundamental law by the
enforcement of an invalid statute.  All told, petitioners have sufficient locus standi to file the instant
150

cases.

D. Res Judicata and Stare Decisis.

Res judicata (which means a "matter adjudged") and stare decisis non quieta et movere (or simply,
stare decisis which means "follow past precedents and do not disturb what has been settled") are
general procedural law principles which both deal with the effects of previous but factually similar
dispositions to subsequent cases. For the cases at bar, the Court examines the applicability of these
principles in relation to its prior rulings in Philconsa and LAMP.

The focal point of res judicata is the judgment. The principle states that a judgment on the merits in a
previous case rendered by a court of competent jurisdiction would bind a subsequent case if,
between the first and second actions, there exists an identity of parties, of subject matter, and of
causes of action.  This required identity is not, however, attendant hereto since Philconsa and
151

LAMP, respectively involved constitutional challenges against the 1994 CDF Article and 2004 PDAF
Article, whereas the cases at bar call for a broader constitutional scrutiny of the entire "Pork Barrel
System." Also, the ruling in LAMP is essentially a dismissal based on a procedural technicality –
and, thus, hardly a judgment on the merits – in that petitioners therein failed to present any
"convincing proof x x x showing that, indeed, there were direct releases of funds to the Members of
Congress, who actually spend them according to their sole discretion" or "pertinent evidentiary
support to demonstrate the illegal misuse of PDAF in the form of kickbacks and has become a
common exercise of unscrupulous Members of Congress." As such, the Court up held, in view of the
presumption of constitutionality accorded to every law, the 2004 PDAF Article, and saw "no need to
review or reverse the standing pronouncements in the said case." Hence, for the foregoing reasons,
the res judicata principle, insofar as the Philconsa and LAMP cases are concerned, cannot apply.

On the other hand, the focal point of stare decisis is the doctrine created. The principle, entrenched
under Article 8  of the Civil Code, evokes the general rule that, for the sake of certainty, a
152

conclusion reached in one case should be doctrinally applied to those that follow if the facts are
substantially the same, even though the parties may be different. It proceeds from the first principle
of justice that, absent any powerful countervailing considerations, like cases ought to be decided
alike. Thus, where the same questions relating to the same event have been put forward by the
parties similarly situated as in a previous case litigated and decided by a competent court, the rule of
stare decisis is a bar to any attempt to re-litigate the same issue.
153

Philconsa was the first case where a constitutional challenge against a Pork Barrel provision, i.e.,
the 1994 CDF Article, was resolved by the Court. To properly understand its context, petitioners‘
posturing was that "the power given to the Members of Congress to propose and identify projects
and activities to be funded by the CDF is an encroachment by the legislature on executive power,
since said power in an appropriation act is in implementation of the law" and that "the proposal and
identification of the projects do not involve the making of laws or the repeal and amendment thereof,
the only function given to the Congress by the Constitution."  In deference to the foregoing
154

submissions, the Court reached the following main conclusions: one, under the Constitution, the
power of appropriation, or the "power of the purse," belongs to Congress; two, the power of
appropriation carries with it the power to specify the project or activity to be funded under the
appropriation law and it can be detailed and as broad as Congress wants it to be; and, three, the
proposals and identifications made by Members of Congress are merely recommendatory. At once,
it is apparent that the Philconsa resolution was a limited response to a separation of powers
problem, specifically on the propriety of conferring post-enactment identification authority to
Members of Congress. On the contrary, the present cases call for a more holistic examination of (a)
the inter-relation between the CDF and PDAF Articles with each other, formative as they are of the
entire "Pork Barrel System" as well as (b) the intra-relation of post-enactment measures contained
within a particular CDF or PDAF Article, including not only those related to the area of project
identification but also to the areas of fund release and realignment. The complexity of the issues and
the broader legal analyses herein warranted may be, therefore, considered as a powerful
countervailing reason against a wholesale application of the stare decisis principle.

In addition, the Court observes that the Philconsa ruling was actually riddled with inherent
constitutional inconsistencies which similarly countervail against a full resort to stare decisis. As may
be deduced from the main conclusions of the case, Philconsa‘s fundamental premise in allowing
Members of Congress to propose and identify of projects would be that the said identification
authority is but an aspect of the power of appropriation which has been constitutionally lodged in
Congress. From this premise, the contradictions may be easily seen. If the authority to identify
projects is an aspect of appropriation and the power of appropriation is a form of legislative power
thereby lodged in Congress, then it follows that: (a) it is Congress which should exercise such
authority, and not its individual Members; (b) such authority must be exercised within the prescribed
procedure of law passage and, hence, should not be exercised after the GAA has already been
passed; and (c) such authority, as embodied in the GAA, has the force of law and, hence, cannot be
merely recommendatory. Justice Vitug‘s Concurring Opinion in the same case sums up the
Philconsa quandary in this wise: "Neither would it be objectionable for Congress, by law, to
appropriate funds for such specific projects as it may be minded; to give that authority, however, to
the individual members of Congress in whatever guise, I am afraid, would be constitutionally
impermissible." As the Court now largely benefits from hindsight and current findings on the matter,
among others, the CoA Report, the Court must partially abandon its previous ruling in Philconsa
insofar as it validated the post-enactment identification authority of Members of Congress on the
guise that the same was merely recommendatory. This postulate raises serious constitutional
inconsistencies which cannot be simply excused on the ground that such mechanism is "imaginative
as it is innovative." Moreover, it must be pointed out that the recent case of Abakada Guro Party List
v. Purisima (Abakada) has effectively overturned Philconsa‘s allowance of post-enactment legislator
155

participation in view of the separation of powers principle. These constitutional inconsistencies and
the Abakada rule will be discussed in greater detail in the ensuing section of this Decision.

As for LAMP, suffice it to restate that the said case was dismissed on a procedural technicality and,
hence, has not set any controlling doctrine susceptible of current application to the substantive
issues in these cases. In fine, stare decisis would not apply.

II. Substantive Issues.

A. Definition of Terms.

Before the Court proceeds to resolve the substantive issues of these cases, it must first define the
terms "Pork Barrel System," "Congressional Pork Barrel," and "Presidential Pork Barrel" as they are
essential to the ensuing discourse.

Petitioners define the term "Pork Barrel System" as the "collusion between the Legislative and
Executive branches of government to accumulate lump-sum public funds in their offices with
unchecked discretionary powers to determine its distribution as political largesse."  They assert that
156

the following elements make up the Pork Barrel System: (a) lump-sum funds are allocated through
the appropriations process to an individual officer; (b) the officer is given sole and broad discretion in
determining how the funds will be used or expended; (c) the guidelines on how to spend or use the
funds in the appropriation are either vague, overbroad or inexistent; and (d) projects funded are
intended to benefit a definite constituency in a particular part of the country and to help the political
careers of the disbursing official by yielding rich patronage benefits.  They further state that the Pork
157
Barrel System is comprised of two (2) kinds of discretionary public funds: first, the Congressional (or
Legislative) Pork Barrel, currently known as the PDAF;  and, second, the Presidential (or Executive)
158

Pork Barrel, specifically, the Malampaya Funds under PD 910 and the Presidential Social Fund
under PD 1869, as amended by PD 1993. 159

Considering petitioners‘ submission and in reference to its local concept and legal history, the Court
defines the Pork Barrel System as the collective body of rules and practices that govern the manner
by which lump-sum, discretionary funds, primarily intended for local projects, are utilized through the
respective participations of the Legislative and Executive branches of government, including its
members. The Pork Barrel System involves two (2) kinds of lump-sum discretionary funds:

First, there is the Congressional Pork Barrel which is herein defined as a kind of lump-sum,
discretionary fund wherein legislators, either individually or collectively organized into committees,
are able to effectively control certain aspects of the fund’s utilization through various post-enactment
measures and/or practices. In particular, petitioners consider the PDAF, as it appears under the
2013 GAA, as Congressional Pork Barrel since it is, inter alia, a post-enactment measure that allows
individual legislators to wield a collective power;  and
160

Second, there is the Presidential Pork Barrel which is herein defined as a kind of lump-sum,
discretionary fund which allows the President to determine the manner of its utilization. For reasons
earlier stated,  the Court shall delimit the use of such term to refer only to the Malampaya Funds
161

and the Presidential Social Fund.

With these definitions in mind, the Court shall now proceed to discuss the substantive issues of
these cases.

B. Substantive Issues on the Congressional Pork Barrel.

1. Separation of Powers.

a. Statement of Principle.

The principle of separation of powers refers to the constitutional demarcation of the three
fundamental powers of government. In the celebrated words of Justice Laurel in Angara v. Electoral
Commission,  it means that the "Constitution has blocked out with deft strokes and in bold lines,
162

allotment of power to the executive, the legislative and the judicial departments of the
government."  To the legislative branch of government, through Congress, belongs the power to
163 164

make laws; to the executive branch of government, through the President,  belongs the power to
165

enforce laws; and to the judicial branch of government, through the Court,  belongs the power to
166

interpret laws. Because the three great powers have been, by constitutional design, ordained in this
respect, "each department of the government has exclusive cognizance of matters within its
jurisdiction, and is supreme within its own sphere."  Thus, "the legislature has no authority to
167

execute or construe the law, the executive has no authority to make or construe the law, and the
judiciary has no power to make or execute the law."  The principle of separation of powers and its
168

concepts of autonomy and independence stem from the notion that the powers of government must
be divided to avoid concentration of these powers in any one branch; the division, it is hoped, would
avoid any single branch from lording its power over the other branches or the citizenry.  To achieve
169

this purpose, the divided power must be wielded by co-equal branches of government that are
equally capable of independent action in exercising their respective mandates. Lack of
independence would result in the inability of one branch of government to check the arbitrary or self-
interest assertions of another or others.
170
Broadly speaking, there is a violation of the separation of powers principle when one branch of
government unduly encroaches on the domain of another. US Supreme Court decisions instruct that
the principle of separation of powers may be violated in two (2) ways: firstly, "one branch may
interfere impermissibly with the other’s performance of its constitutionally assigned function";  and
171

"alternatively, the doctrine may be violated when one branch assumes a function that more properly
is entrusted to another."  In other words, there is a violation of the principle when there is
172

impermissible (a) interference with and/or (b) assumption of another department‘s functions.

The enforcement of the national budget, as primarily contained in the GAA, is indisputably a function
both constitutionally assigned and properly entrusted to the Executive branch of government. In
Guingona, Jr. v. Hon. Carague  (Guingona, Jr.), the Court explained that the phase of budget
173

execution "covers the various operational aspects of budgeting" and accordingly includes "the
evaluation of work and financial plans for individual activities," the "regulation and release of funds"
as well as all "other related activities" that comprise the budget execution cycle.  This is rooted in
174

the principle that the allocation of power in the three principal branches of government is a grant of
all powers inherent in them.  Thus, unless the Constitution provides otherwise, the Executive
175

department should exclusively exercise all roles and prerogatives which go into the implementation
of the national budget as provided under the GAA as well as any other appropriation law.

In view of the foregoing, the Legislative branch of government, much more any of its members,
should not cross over the field of implementing the national budget since, as earlier stated, the same
is properly the domain of the Executive. Again, in Guingona, Jr., the Court stated that "Congress
enters the picture when it deliberates or acts on the budget proposals of the President. Thereafter,
Congress, "in the exercise of its own judgment and wisdom, formulates an appropriation act
precisely following the process established by the Constitution, which specifies that no money may
be paid from the Treasury except in accordance with an appropriation made by law." Upon approval
and passage of the GAA, Congress‘ law -making role necessarily comes to an end and from there
the Executive‘s role of implementing the national budget begins. So as not to blur the constitutional
boundaries between them, Congress must "not concern it self with details for implementation by the
Executive."176

The foregoing cardinal postulates were definitively enunciated in Abakada where the Court held that
"from the moment the law becomes effective, any provision of law that empowers Congress or any
of its members to play any role in the implementation or enforcement of the law violates the principle
of separation of powers and is thus unconstitutional."  It must be clarified, however, that since the
177

restriction only pertains to "any role in the implementation or enforcement of the law," Congress may
still exercise its oversight function which is a mechanism of checks and balances that the
Constitution itself allows. But it must be made clear that Congress‘ role must be confined to mere
oversight. Any post-enactment-measure allowing legislator participation beyond oversight is bereft of
any constitutional basis and hence, tantamount to impermissible interference and/or assumption of
executive functions. As the Court ruled in Abakada: 178

Any post-enactment congressional measure x x x should be limited to scrutiny and investigation.  In 1âwphi1

particular, congressional oversight must be confined to the following:

(1) scrutiny based primarily on Congress‘ power of appropriation and the budget hearings
conducted in connection with it, its power to ask heads of departments to appear before and
be heard by either of its Houses on any matter pertaining to their departments and its power
of confirmation; and

(2) investigation and monitoring of the implementation of laws pursuant to the power of
Congress to conduct inquiries in aid of legislation.
Any action or step beyond that will undermine the separation of powers guaranteed by the
Constitution. (Emphases supplied)

b. Application.

In these cases, petitioners submit that the Congressional Pork Barrel – among others, the 2013
PDAF Article – "wrecks the assignment of responsibilities between the political branches" as it is
designed to allow individual legislators to interfere "way past the time it should have ceased" or,
particularly, "after the GAA is passed."  They state that the findings and recommendations in the
179

CoA Report provide "an illustration of how absolute and definitive the power of legislators wield over
project implementation in complete violation of the constitutional principle of separation of
powers."  Further, they point out that the Court in the Philconsa case only allowed the CDF to exist
180

on the condition that individual legislators limited their role to recommending projects and not if they
actually dictate their implementation.181

For their part, respondents counter that the separations of powers principle has not been violated
since the President maintains "ultimate authority to control the execution of the GAA‖ and that he
"retains the final discretion to reject" the legislators‘ proposals.  They maintain that the Court, in
182

Philconsa, "upheld the constitutionality of the power of members of Congress to propose and identify
projects so long as such proposal and identification are recommendatory."  As such, they claim that
183

"everything in the Special Provisions [of the 2013 PDAF Article follows the Philconsa framework, and
hence, remains constitutional." 184

The Court rules in favor of petitioners.

As may be observed from its legal history, the defining feature of all forms of Congressional Pork
Barrel would be the authority of legislators to participate in the post-enactment phases of project
implementation.

At its core, legislators – may it be through project lists,  prior consultations  or program menus  –
185 186 187

have been consistently accorded post-enactment authority to identify the projects they desire to be
funded through various Congressional Pork Barrel allocations. Under the 2013 PDAF Article, the
statutory authority of legislators to identify projects post-GAA may be construed from the import of
Special Provisions 1 to 3 as well as the second paragraph of Special Provision 4. To elucidate,
Special Provision 1 embodies the program menu feature which, as evinced from past PDAF Articles,
allows individual legislators to identify PDAF projects for as long as the identified project falls under a
general program listed in the said menu. Relatedly, Special Provision 2 provides that the
implementing agencies shall, within 90 days from the GAA is passed, submit to Congress a more
detailed priority list, standard or design prepared and submitted by implementing agencies from
which the legislator may make his choice. The same provision further authorizes legislators to
identify PDAF projects outside his district for as long as the representative of the district concerned
concurs in writing. Meanwhile, Special Provision 3 clarifies that PDAF projects refer to "projects to be
identified by legislators"  and thereunder provides the allocation limit for the total amount of projects
188

identified by each legislator. Finally, paragraph 2 of Special Provision 4 requires that any
modification and revision of the project identification "shall be submitted to the House Committee on
Appropriations and the Senate Committee on Finance for favorable endorsement to the DBM or the
implementing agency, as the case may be." From the foregoing special provisions, it cannot be
seriously doubted that legislators have been accorded post-enactment authority to identify PDAF
projects.

Aside from the area of project identification, legislators have also been accorded post-enactment
authority in the areas of fund release and realignment. Under the 2013 PDAF Article, the statutory
authority of legislators to participate in the area of fund release through congressional committees is
contained in Special Provision 5 which explicitly states that "all request for release of funds shall be
supported by the documents prescribed under Special Provision No. 1 and favorably endorsed by
House Committee on Appropriations and the Senate Committee on Finance, as the case may be";
while their statutory authority to participate in the area of fund realignment is contained in: first ,
paragraph 2, Special Provision 4  which explicitly state s, among others, that "any realignment of
189

funds shall be submitted to the House Committee on Appropriations and the Senate Committee on
Finance for favorable endorsement to the DBM or the implementing agency, as the case may be‖ ;
and, second , paragraph 1, also of Special Provision 4 which authorizes the "Secretaries of
Agriculture, Education, Energy, Interior and Local Government, Labor and Employment, Public
Works and Highways, Social Welfare and Development and Trade and Industry  x x x to approve
190

realignment from one project/scope to another within the allotment received from this Fund, subject
to among others (iii) the request is with the concurrence of the legislator concerned."

Clearly, these post-enactment measures which govern the areas of project identification, fund
release and fund realignment are not related to functions of congressional oversight and, hence,
allow legislators to intervene and/or assume duties that properly belong to the sphere of budget
execution. Indeed, by virtue of the foregoing, legislators have been, in one form or another,
authorized to participate in – as Guingona, Jr. puts it – "the various operational aspects of
budgeting," including "the evaluation of work and financial plans for individual activities" and the
"regulation and release of funds" in violation of the separation of powers principle. The fundamental
rule, as categorically articulated in Abakada, cannot be overstated – from the moment the law
becomes effective, any provision of law that empowers Congress or any of its members to play any
role in the implementation or enforcement of the law violates the principle of separation of powers
and is thus unconstitutional.  That the said authority is treated as merely recommendatory in nature
191

does not alter its unconstitutional tenor since the prohibition, to repeat, covers any role in the
implementation or enforcement of the law. Towards this end, the Court must therefore abandon its
ruling in Philconsa which sanctioned the conduct of legislator identification on the guise that the
same is merely recommendatory and, as such, respondents‘ reliance on the same falters altogether.

Besides, it must be pointed out that respondents have nonetheless failed to substantiate their
position that the identification authority of legislators is only of recommendatory import. Quite the
contrary, respondents – through the statements of the Solicitor General during the Oral Arguments –
have admitted that the identification of the legislator constitutes a mandatory requirement before his
PDAF can be tapped as a funding source, thereby highlighting the indispensability of the said act to
the entire budget execution process: 192

Justice Bernabe: Now, without the individual legislator’s identification of the project, can the PDAF of
the legislator be utilized?

Solicitor General Jardeleza: No, Your Honor.

Justice Bernabe: It cannot?

Solicitor General Jardeleza: It cannot… (interrupted)

Justice Bernabe: So meaning you should have the identification of the project by the individual
legislator?

Solicitor General Jardeleza: Yes, Your Honor.

xxxx
Justice Bernabe: In short, the act of identification is mandatory?

Solictor General Jardeleza: Yes, Your Honor. In the sense that if it is not done and then there is no
identification.

xxxx

Justice Bernabe: Now, would you know of specific instances when a project was implemented
without the identification by the individual legislator?

Solicitor General Jardeleza: I do not know, Your Honor; I do not think so but I have no specific
examples. I would doubt very much, Your Honor, because to implement, there is a need for a SARO
and the NCA. And the SARO and the NCA are triggered by an identification from the legislator.

xxxx

Solictor General Jardeleza: What we mean by mandatory, Your Honor, is we were replying to a
question, "How can a legislator make sure that he is able to get PDAF Funds?" It is mandatory in the
sense that he must identify, in that sense, Your Honor. Otherwise, if he does not identify, he cannot
avail of the PDAF Funds and his district would not be able to have PDAF Funds, only in that sense,
Your Honor. (Emphases supplied)

Thus, for all the foregoing reasons, the Court hereby declares the 2013 PDAF Article as well as all
other provisions of law which similarly allow legislators to wield any form of post-enactment authority
in the implementation or enforcement of the budget, unrelated to congressional oversight, as
violative of the separation of powers principle and thus unconstitutional. Corollary thereto, informal
practices, through which legislators have effectively intruded into the proper phases of budget
execution, must be deemed as acts of grave abuse of discretion amounting to lack or excess of
jurisdiction and, hence, accorded the same unconstitutional treatment. That such informal practices
do exist and have, in fact, been constantly observed throughout the years has not been substantially
disputed here. As pointed out by Chief Justice Maria Lourdes P.A. Sereno (Chief Justice Sereno)
during the Oral Arguments of these cases:193

Chief Justice Sereno:

Now, from the responses of the representative of both, the DBM and two (2) Houses of Congress, if
we enforces the initial thought that I have, after I had seen the extent of this research made by my
staff, that neither the Executive nor Congress frontally faced the question of constitutional
compatibility of how they were engineering the budget process. In fact, the words you have been
using, as the three lawyers of the DBM, and both Houses of Congress has also been using is
surprise; surprised that all of these things are now surfacing. In fact, I thought that what the 2013
PDAF provisions did was to codify in one section all the past practice that had been done since
1991. In a certain sense, we should be thankful that they are all now in the PDAF Special Provisions.
x x x (Emphasis and underscoring supplied)

Ultimately, legislators cannot exercise powers which they do not have, whether through formal
measures written into the law or informal practices institutionalized in government agencies, else the
Executive department be deprived of what the Constitution has vested as its own.

2. Non-delegability of Legislative Power.

a. Statement of Principle.
As an adjunct to the separation of powers principle,  legislative power shall be exclusively exercised
194

by the body to which the Constitution has conferred the same. In particular, Section 1, Article VI of
the 1987 Constitution states that such power shall be vested in the Congress of the Philippines
which shall consist of a Senate and a House of Representatives, except to the extent reserved to the
people by the provision on initiative and referendum.  Based on this provision, it is clear that only
195

Congress, acting as a bicameral body, and the people, through the process of initiative and
referendum, may constitutionally wield legislative power and no other. This premise embodies the
principle of non-delegability of legislative power, and the only recognized exceptions thereto would
be: (a) delegated legislative power to local governments which, by immemorial practice, are allowed
to legislate on purely local matters;  and (b) constitutionally-grafted exceptions such as the authority
196

of the President to, by law, exercise powers necessary and proper to carry out a declared national
policy in times of war or other national emergency, or fix within specified limits, and subject to such
197

limitations and restrictions as Congress may impose, tariff rates, import and export quotas, tonnage
and wharfage dues, and other duties or imposts within the framework of the national development
program of the Government. 198

Notably, the principle of non-delegability should not be confused as a restriction to delegate rule-
making authority to implementing agencies for the limited purpose of either filling up the details of
the law for its enforcement (supplementary rule-making) or ascertaining facts to bring the law into
actual operation (contingent rule-making). The conceptual treatment and limitations of delegated
199

rule-making were explained in the case of People v. Maceren  as follows:


200

The grant of the rule-making power to administrative agencies is a relaxation of the principle of
separation of powers and is an exception to the nondelegation of legislative powers. Administrative
regulations or "subordinate legislation" calculated to promote the public interest are necessary
because of "the growing complexity of modern life, the multiplication of the subjects of governmental
regulations, and the increased difficulty of administering the law."

xxxx

Nevertheless, it must be emphasized that the rule-making power must be confined to details for
regulating the mode or proceeding to carry into effect the law as it has been enacted. The power
cannot be extended to amending or expanding the statutory requirements or to embrace matters not
covered by the statute. Rules that subvert the statute cannot be sanctioned. (Emphases supplied)

b. Application.

In the cases at bar, the Court observes that the 2013 PDAF Article, insofar as it confers post-
enactment identification authority to individual legislators, violates the principle of non-delegability
since said legislators are effectively allowed to individually exercise the power of appropriation,
which – as settled in Philconsa – is lodged in Congress.  That the power to appropriate must be
201

exercised only through legislation is clear from Section 29(1), Article VI of the 1987 Constitution
which states that: "No money shall be paid out of the Treasury except in pursuance of an
appropriation made by law." To understand what constitutes an act of appropriation, the Court, in
Bengzon v. Secretary of Justice and Insular Auditor  (Bengzon), held that the power of appropriation
202

involves (a) the setting apart by law of a certain sum from the public revenue for (b) a specified
purpose. Essentially, under the 2013 PDAF Article, individual legislators are given a personal lump-
sum fund from which they are able to dictate (a) how much from such fund would go to (b) a specific
project or beneficiary that they themselves also determine. As these two (2) acts comprise the
exercise of the power of appropriation as described in Bengzon, and given that the 2013 PDAF
Article authorizes individual legislators to perform the same, undoubtedly, said legislators have been
conferred the power to legislate which the Constitution does not, however, allow. Thus, keeping with
the principle of non-delegability of legislative power, the Court hereby declares the 2013 PDAF
Article, as well as all other forms of Congressional Pork Barrel which contain the similar legislative
identification feature as herein discussed, as unconstitutional.

3. Checks and Balances.

a. Statement of Principle; Item-Veto Power.

The fact that the three great powers of government are intended to be kept separate and distinct
does not mean that they are absolutely unrestrained and independent of each other. The
Constitution has also provided for an elaborate system of checks and balances to secure
coordination in the workings of the various departments of the government. 203

A prime example of a constitutional check and balance would be the President’s power to veto an
item written into an appropriation, revenue or tariff bill submitted to him by Congress for approval
through a process known as "bill presentment." The President‘s item-veto power is found in Section
27(2), Article VI of the 1987 Constitution which reads as follows:

Sec. 27. x x x.

xxxx

(2) The President shall have the power to veto any particular item or items in an appropriation,
revenue, or tariff bill, but the veto shall not affect the item or items to which he does not object.

The presentment of appropriation, revenue or tariff bills to the President, wherein he may exercise
his power of item-veto, forms part of the "single, finely wrought and exhaustively considered,
procedures" for law-passage as specified under the Constitution.  As stated in Abakada, the final
204

step in the law-making process is the "submission of the bill to the President for approval. Once
approved, it takes effect as law after the required publication."205

Elaborating on the President‘s item-veto power and its relevance as a check on the legislature, the
Court, in Bengzon, explained that: 206

The former Organic Act and the present Constitution of the Philippines make the Chief Executive an
integral part of the law-making power. His disapproval of a bill, commonly known as a veto, is
essentially a legislative act. The questions presented to the mind of the Chief Executive are precisely
the same as those the legislature must determine in passing a bill, except that his will be a broader
point of view.

The Constitution is a limitation upon the power of the legislative department of the government, but
in this respect it is a grant of power to the executive department. The Legislature has the affirmative
power to enact laws; the Chief Executive has the negative power by the constitutional exercise of
which he may defeat the will of the Legislature. It follows that the Chief Executive must find his
authority in the Constitution. But in exercising that authority he may not be confined to rules of strict
construction or hampered by the unwise interference of the judiciary. The courts will indulge every
intendment in favor of the constitutionality of a veto in the same manner as they will presume the
constitutionality of an act as originally passed by the Legislature. (Emphases supplied)

The justification for the President‘s item-veto power rests on a variety of policy goals such as to
prevent log-rolling legislation,  impose fiscal restrictions on the legislature, as well as to fortify the
207
executive branch‘s role in the budgetary process.  In Immigration and Naturalization Service v.
208

Chadha, the US Supreme Court characterized the President‘s item-power as "a salutary check upon
the legislative body, calculated to guard the community against the effects of factions, precipitancy,
or of any impulse unfriendly to the public good, which may happen to influence a majority of that
body"; phrased differently, it is meant to "increase the chances in favor of the community against the
passing of bad laws, through haste, inadvertence, or design." 209

For the President to exercise his item-veto power, it necessarily follows that there exists a proper
"item" which may be the object of the veto. An item, as defined in the field of appropriations, pertains
to "the particulars, the details, the distinct and severable parts of the appropriation or of the bill." In
the case of Bengzon v. Secretary of Justice of the Philippine Islands,  the US Supreme Court
210

characterized an item of appropriation as follows:

An item of an appropriation bill obviously means an item which, in itself, is a specific appropriation of
money, not some general provision of law which happens to be put into an appropriation bill.
(Emphases supplied)

On this premise, it may be concluded that an appropriation bill, to ensure that the President may be
able to exercise his power of item veto, must contain "specific appropriations of money" and not only
"general provisions" which provide for parameters of appropriation.

Further, it is significant to point out that an item of appropriation must be an item characterized by
singular correspondence – meaning an allocation of a specified singular amount for a specified
singular purpose, otherwise known as a "line-item."  This treatment not only allows the item to be
211

consistent with its definition as a "specific appropriation of money" but also ensures that the
President may discernibly veto the same. Based on the foregoing formulation, the existing Calamity
Fund, Contingent Fund and the Intelligence Fund, being appropriations which state a specified
amount for a specific purpose, would then be considered as "line- item" appropriations which are
rightfully subject to item veto. Likewise, it must be observed that an appropriation may be validly
apportioned into component percentages or values; however, it is crucial that each percentage or
value must be allocated for its own corresponding purpose for such component to be considered as
a proper line-item. Moreover, as Justice Carpio correctly pointed out, a valid appropriation may even
have several related purposes that are by accounting and budgeting practice considered as one
purpose, e.g., MOOE (maintenance and other operating expenses), in which case the related
purposes shall be deemed sufficiently specific for the exercise of the President‘s item veto power.
Finally, special purpose funds and discretionary funds would equally square with the constitutional
mechanism of item-veto for as long as they follow the rule on singular correspondence as herein
discussed. Anent special purpose funds, it must be added that Section 25(4), Article VI of the 1987
Constitution requires that the "special appropriations bill shall specify the purpose for which it is
intended, and shall be supported by funds actually available as certified by the National Treasurer,
or t o be raised by a corresponding revenue proposal therein." Meanwhile, with respect to
discretionary funds, Section 2 5(6), Article VI of the 1987 Constitution requires that said funds "shall
be disbursed only for public purposes to be supported by appropriate vouchers and subject to such
guidelines as may be prescribed by law."

In contrast, what beckons constitutional infirmity are appropriations which merely provide for a
singular lump-sum amount to be tapped as a source of funding for multiple purposes. Since such
appropriation type necessitates the further determination of both the actual amount to be expended
and the actual purpose of the appropriation which must still be chosen from the multiple purposes
stated in the law, it cannot be said that the appropriation law already indicates a "specific
appropriation of money‖ and hence, without a proper line-item which the President may veto. As a
practical result, the President would then be faced with the predicament of either vetoing the entire
appropriation if he finds some of its purposes wasteful or undesirable, or approving the entire
appropriation so as not to hinder some of its legitimate purposes. Finally, it may not be amiss to
state that such arrangement also raises non-delegability issues considering that the implementing
authority would still have to determine, again, both the actual amount to be expended and the actual
purpose of the appropriation. Since the foregoing determinations constitute the integral aspects of
the power to appropriate, the implementing authority would, in effect, be exercising legislative
prerogatives in violation of the principle of non-delegability.

b. Application.

In these cases, petitioners claim that "in the current x x x system where the PDAF is a lump-sum
appropriation, the legislator‘s identification of the projects after the passage of the GAA denies the
President the chance to veto that item later on."  Accordingly, they submit that the "item veto power
212

of the President mandates that appropriations bills adopt line-item budgeting" and that "Congress
cannot choose a mode of budgeting which effectively renders the constitutionally-given power of the
President useless."213

On the other hand, respondents maintain that the text of the Constitution envisions a process which
is intended to meet the demands of a modernizing economy and, as such, lump-sum appropriations
are essential to financially address situations which are barely foreseen when a GAA is enacted.
They argue that the decision of the Congress to create some lump-sum appropriations is
constitutionally allowed and textually-grounded.214

The Court agrees with petitioners.

Under the 2013 PDAF Article, the amount of ₱24.79 Billion only appears as a collective allocation
limit since the said amount would be further divided among individual legislators who would then
receive personal lump-sum allocations and could, after the GAA is passed, effectively appropriate
PDAF funds based on their own discretion. As these intermediate appropriations are made by
legislators only after the GAA is passed and hence, outside of the law, it necessarily means that the
actual items of PDAF appropriation would not have been written into the General Appropriations Bill
and thus effectuated without veto consideration. This kind of lump-sum/post-enactment legislative
identification budgeting system fosters the creation of a budget within a budget" which subverts the
prescribed procedure of presentment and consequently impairs the President‘s power of item veto.
As petitioners aptly point out, the above-described system forces the President to decide between
(a) accepting the entire ₱24.79 Billion PDAF allocation without knowing the specific projects of the
legislators, which may or may not be consistent with his national agenda and (b) rejecting the whole
PDAF to the detriment of all other legislators with legitimate projects.
215

Moreover, even without its post-enactment legislative identification feature, the 2013 PDAF Article
would remain constitutionally flawed since it would then operate as a prohibited form of lump-sum
appropriation above-characterized. In particular, the lump-sum amount of ₱24.79 Billion would be
treated as a mere funding source allotted for multiple purposes of spending, i.e., scholarships,
medical missions, assistance to indigents, preservation of historical materials, construction of roads,
flood control, etc. This setup connotes that the appropriation law leaves the actual amounts and
purposes of the appropriation for further determination and, therefore, does not readily indicate a
discernible item which may be subject to the President‘s power of item veto.

In fact, on the accountability side, the same lump-sum budgeting scheme has, as the CoA
Chairperson relays, "limited state auditors from obtaining relevant data and information that would
aid in more stringently auditing the utilization of said Funds."  Accordingly, she recommends the
216
adoption of a "line by line budget or amount per proposed program, activity or project, and per
implementing agency." 217

Hence, in view of the reasons above-stated, the Court finds the 2013 PDAF Article, as well as all
Congressional Pork Barrel Laws of similar operation, to be unconstitutional. That such budgeting
system provides for a greater degree of flexibility to account for future contingencies cannot be an
excuse to defeat what the Constitution requires. Clearly, the first and essential truth of the matter is
that unconstitutional means do not justify even commendable ends. 218

c. Accountability.

Petitioners further relate that the system under which various forms of Congressional Pork Barrel
operate defies public accountability as it renders Congress incapable of checking itself or its
Members. In particular, they point out that the Congressional Pork Barrel "gives each legislator a
direct, financial interest in the smooth, speedy passing of the yearly budget" which turns them "from
fiscalizers" into "financially-interested partners."  They also claim that the system has an effect on
219

re- election as "the PDAF excels in self-perpetuation of elective officials." Finally, they add that the
"PDAF impairs the power of impeachment" as such "funds are indeed quite useful, ‘to well,
accelerate the decisions of senators.‘" 220

The Court agrees in part.

The aphorism forged under Section 1, Article XI of the 1987 Constitution, which states that "public
office is a public trust," is an overarching reminder that every instrumentality of government should
exercise their official functions only in accordance with the principles of the Constitution which
embodies the parameters of the people‘s trust. The notion of a public trust connotes
accountability,  hence, the various mechanisms in the Constitution which are designed to exact
221

accountability from public officers.

Among others, an accountability mechanism with which the proper expenditure of public funds may
be checked is the power of congressional oversight. As mentioned in Abakada,  congressional
222

oversight may be performed either through: (a) scrutiny based primarily on Congress‘ power of
appropriation and the budget hearings conducted in connection with it, its power to ask heads of
departments to appear before and be heard by either of its Houses on any matter pertaining to their
departments and its power of confirmation;  or (b) investigation and monitoring of the
223

implementation of laws pursuant to the power of Congress to conduct inquiries in aid of legislation. 224

The Court agrees with petitioners that certain features embedded in some forms of Congressional
Pork Barrel, among others the 2013 PDAF Article, has an effect on congressional oversight. The fact
that individual legislators are given post-enactment roles in the implementation of the budget makes
it difficult for them to become disinterested "observers" when scrutinizing, investigating or monitoring
the implementation of the appropriation law. To a certain extent, the conduct of oversight would be
tainted as said legislators, who are vested with post-enactment authority, would, in effect, be
checking on activities in which they themselves participate. Also, it must be pointed out that this very
same concept of post-enactment authorization runs afoul of Section 14, Article VI of the 1987
Constitution which provides that:

Sec. 14. No Senator or Member of the House of Representatives may personally appear as counsel
before any court of justice or before the Electoral Tribunals, or quasi-judicial and other administrative
bodies. Neither shall he, directly or indirectly, be interested financially in any contract with, or in any
franchise or special privilege granted by the Government, or any subdivision, agency, or
instrumentality thereof, including any government-owned or controlled corporation, or its subsidiary,
during his term of office. He shall not intervene in any matter before any office of the Government for
his pecuniary benefit or where he may be called upon to act on account of his office. (Emphasis
supplied)

Clearly, allowing legislators to intervene in the various phases of project implementation – a matter
before another office of government – renders them susceptible to taking undue advantage of their
own office.

The Court, however, cannot completely agree that the same post-enactment authority and/or the
individual legislator‘s control of his PDAF per se would allow him to perpetuate himself in office.
Indeed, while the Congressional Pork Barrel and a legislator‘s use thereof may be linked to this area
of interest, the use of his PDAF for re-election purposes is a matter which must be analyzed based
on particular facts and on a case-to-case basis.

Finally, while the Court accounts for the possibility that the close operational proximity between
legislators and the Executive department, through the former‘s post-enactment participation, may
affect the process of impeachment, this matter largely borders on the domain of politics and does not
strictly concern the Pork Barrel System‘s intrinsic constitutionality. As such, it is an improper subject
of judicial assessment.

In sum, insofar as its post-enactment features dilute congressional oversight and violate Section 14,
Article VI of the 1987 Constitution, thus impairing public accountability, the 2013 PDAF Article and
other forms of Congressional Pork Barrel of similar nature are deemed as unconstitutional.

4. Political Dynasties.

One of the petitioners submits that the Pork Barrel System enables politicians who are members of
political dynasties to accumulate funds to perpetuate themselves in power, in contravention of
Section 26, Article II of the 1987 Constitution  which states that:
225

Sec. 26. The State shall guarantee equal access to opportunities for public service, and prohibit
political dynasties as may be defined by law. (Emphasis and underscoring supplied)

At the outset, suffice it to state that the foregoing provision is considered as not self-executing due to
the qualifying phrase "as may be defined by law." In this respect, said provision does not, by and of
itself, provide a judicially enforceable constitutional right but merely specifies guideline for legislative
or executive action.  Therefore, since there appears to be no standing law which crystallizes the
226

policy on political dynasties for enforcement, the Court must defer from ruling on this issue.

In any event, the Court finds the above-stated argument on this score to be largely speculative since
it has not been properly demonstrated how the Pork Barrel System would be able to propagate
political dynasties.

5. Local Autonomy.

The State‘s policy on local autonomy is principally stated in Section 25, Article II and Sections 2 and
3, Article X of the 1987 Constitution which read as follows:

ARTICLE II

Sec. 25. The State shall ensure the autonomy of local governments.
ARTICLE X

Sec. 2. The territorial and political subdivisions shall enjoy local autonomy.

Sec. 3. The Congress shall enact a local government code which shall provide for a more
responsive and accountable local government structure instituted through a system of
decentralization with effective mechanisms of recall, initiative, and referendum, allocate among the
different local government units their powers, responsibilities, and resources, and provide for the
qualifications, election, appointment and removal, term, salaries, powers and functions and duties of
local officials, and all other matters relating to the organization and operation of the local units.

Pursuant thereto, Congress enacted RA 7160,  otherwise known as the "Local Government Code of
227

1991" (LGC), wherein the policy on local autonomy had been more specifically explicated as follows:

Sec. 2. Declaration of Policy. – (a) It is hereby declared the policy of the State that the territorial and
political subdivisions of the State shall enjoy genuine and meaningful local autonomy to enable them
to attain their fullest development as self-reliant communities and make them more effective partners
in the attainment of national goals. Toward this end, the State shall provide for a more responsive
and accountable local government structure instituted through a system of decentralization whereby
local government units shall be given more powers, authority, responsibilities, and resources. The
process of decentralization shall proceed from the National Government to the local government
units.

xxxx

(c) It is likewise the policy of the State to require all national agencies and offices to conduct periodic
consultations with appropriate local government units, nongovernmental and people‘s organizations,
and other concerned sectors of the community before any project or program is implemented in their
respective jurisdictions. (Emphases and underscoring supplied)

The above-quoted provisions of the Constitution and the LGC reveal the policy of the State to
empower local government units (LGUs) to develop and ultimately, become self-sustaining and
effective contributors to the national economy. As explained by the Court in Philippine Gamefowl
Commission v. Intermediate Appellate Court: 228

This is as good an occasion as any to stress the commitment of the Constitution to the policy of local
autonomy which is intended to provide the needed impetus and encouragement to the development
of our local political subdivisions as "self - reliant communities." In the words of Jefferson, "Municipal
corporations are the small republics from which the great one derives its strength." The vitalization of
local governments will enable their inhabitants to fully exploit their resources and more important,
imbue them with a deepened sense of involvement in public affairs as members of the body politic.
This objective could be blunted by undue interference by the national government in purely local
affairs which are best resolved by the officials and inhabitants of such political units. The decision we
reach today conforms not only to the letter of the pertinent laws but also to the spirit of the
Constitution.  (Emphases and underscoring supplied)
229

In the cases at bar, petitioners contend that the Congressional Pork Barrel goes against the
constitutional principles on local autonomy since it allows district representatives, who are national
officers, to substitute their judgments in utilizing public funds for local development.  The Court
230

agrees with petitioners.


Philconsa described the 1994 CDF as an attempt "to make equal the unequal" and that "it is also a
recognition that individual members of Congress, far more than the President and their
congressional colleagues, are likely to be knowledgeable about the needs of their respective
constituents and the priority to be given each project."  Drawing strength from this pronouncement,
231

previous legislators justified its existence by stating that "the relatively small projects implemented
under the Congressional Pork Barrel complement and link the national development goals to the
countryside and grassroots as well as to depressed areas which are overlooked by central agencies
which are preoccupied with mega-projects.  Similarly, in his August 23, 2013 speech on the
232

"abolition" of PDAF and budgetary reforms, President Aquino mentioned that the Congressional
Pork Barrel was originally established for a worthy goal, which is to enable the representatives to
identify projects for communities that the LGU concerned cannot afford. 233

Notwithstanding these declarations, the Court, however, finds an inherent defect in the system which
actually belies the avowed intention of "making equal the unequal." In particular, the Court observes
that the gauge of PDAF and CDF allocation/division is based solely on the fact of office, without
taking into account the specific interests and peculiarities of the district the legislator represents. In
this regard, the allocation/division limits are clearly not based on genuine parameters of equality,
wherein economic or geographic indicators have been taken into consideration. As a result, a district
representative of a highly-urbanized metropolis gets the same amount of funding as a district
representative of a far-flung rural province which would be relatively "underdeveloped" compared to
the former. To add, what rouses graver scrutiny is that even Senators and Party-List
Representatives – and in some years, even the Vice-President – who do not represent any locality,
receive funding from the Congressional Pork Barrel as well. These certainly are anathema to the
Congressional Pork Barrel‘s original intent which is "to make equal the unequal." Ultimately, the
PDAF and CDF had become personal funds under the effective control of each legislator and given
unto them on the sole account of their office.

The Court also observes that this concept of legislator control underlying the CDF and PDAF
conflicts with the functions of the various Local Development Councils (LDCs) which are already
legally mandated to "assist the corresponding sanggunian in setting the direction of economic and
social development, and coordinating development efforts within its territorial
jurisdiction."  Considering that LDCs are instrumentalities whose functions are essentially geared
234

towards managing local affairs,  their programs, policies and resolutions should not be overridden
235

nor duplicated by individual legislators, who are national officers that have no law-making authority
except only when acting as a body. The undermining effect on local autonomy caused by the post-
enactment authority conferred to the latter was succinctly put by petitioners in the following wise: 236

With PDAF, a Congressman can simply bypass the local development council and initiate projects
on his own, and even take sole credit for its execution. Indeed, this type of personality-driven project
identification has not only contributed little to the overall development of the district, but has even
contributed to "further weakening infrastructure planning and coordination efforts of the government."

Thus, insofar as individual legislators are authorized to intervene in purely local matters and thereby
subvert genuine local autonomy, the 2013 PDAF Article as well as all other similar forms of
Congressional Pork Barrel is deemed unconstitutional.

With this final issue on the Congressional Pork Barrel resolved, the Court now turns to the
substantive issues involving the Presidential Pork Barrel.

C. Substantive Issues on the Presidential Pork Barrel.

1. Validity of Appropriation.
Petitioners preliminarily assail Section 8 of PD 910 and Section 12 of PD1869 (now, amended by PD
1993), which respectively provide for the Malampaya Funds and the Presidential Social Fund, as
invalid appropriations laws since they do not have the "primary and specific" purpose of authorizing
the release of public funds from the National Treasury. Petitioners submit that Section 8 of PD 910 is
not an appropriation law since the "primary and specific‖ purpose of PD 910 is the creation of an
Energy Development Board and Section 8 thereof only created a Special Fund incidental
thereto.  In similar regard, petitioners argue that Section 12 of PD 1869 is neither a valid
237

appropriations law since the allocation of the Presidential Social Fund is merely incidental to the
"primary and specific" purpose of PD 1869 which is the amendment of the Franchise and Powers of
PAGCOR.  In view of the foregoing, petitioners suppose that such funds are being used without any
238

valid law allowing for their proper appropriation in violation of Section 29(1), Article VI of the 1987
Constitution which states that: "No money shall be paid out of the Treasury except in pursuance of
an appropriation made by law." 239

The Court disagrees.

"An appropriation made by law‖ under the contemplation of Section 29(1), Article VI of the 1987
Constitution exists when a provision of law (a) sets apart a determinate or determinable  amount of
240

money and (b) allocates the same for a particular public purpose. These two minimum designations
of amount and purpose stem from the very definition of the word "appropriation," which means "to
allot, assign, set apart or apply to a particular use or purpose," and hence, if written into the law,
demonstrate that the legislative intent to appropriate exists. As the Constitution "does not provide or
prescribe any particular form of words or religious recitals in which an authorization or appropriation
by Congress shall be made, except that it be ‘made by law,‘" an appropriation law may – according
to Philconsa – be "detailed and as broad as Congress wants it to be" for as long as the intent to
appropriate may be gleaned from the same. As held in the case of Guingona, Jr.: 241

There is no provision in our Constitution that provides or prescribes any particular form of words or
religious recitals in which an authorization or appropriation by Congress shall be made, except that it
be "made by law," such as precisely the authorization or appropriation under the questioned
presidential decrees. In other words, in terms of time horizons, an appropriation may be made
impliedly (as by past but subsisting legislations) as well as expressly for the current fiscal year (as by
enactment of laws by the present Congress), just as said appropriation may be made in general as
well as in specific terms. The Congressional authorization may be embodied in annual laws, such as
a general appropriations act or in special provisions of laws of general or special application which
appropriate public funds for specific public purposes, such as the questioned decrees. An
appropriation measure is sufficient if the legislative intention clearly and certainly appears from the
language employed (In re Continuing Appropriations, 32 P. 272), whether in the past or in the
present. (Emphases and underscoring supplied)

Likewise, as ruled by the US Supreme Court in State of Nevada v. La Grave: 242

To constitute an appropriation there must be money placed in a fund applicable to the designated
purpose. The word appropriate means to allot, assign, set apart or apply to a particular use or
purpose. An appropriation in the sense of the constitution means the setting apart a portion of the
public funds for a public purpose. No particular form of words is necessary for the purpose, if the
intention to appropriate is plainly manifested. (Emphases supplied)

Thus, based on the foregoing, the Court cannot sustain the argument that the appropriation must be
the "primary and specific" purpose of the law in order for a valid appropriation law to exist. To
reiterate, if a legal provision designates a determinate or determinable amount of money and
allocates the same for a particular public purpose, then the legislative intent to appropriate becomes
apparent and, hence, already sufficient to satisfy the requirement of an "appropriation made by law"
under contemplation of the Constitution.

Section 8 of PD 910 pertinently provides:

Section 8. Appropriations. x x x

All fees, revenues and receipts of the Board from any and all sources including receipts from service
contracts and agreements such as application and processing fees, signature bonus, discovery
bonus, production bonus; all money collected from concessionaires, representing unspent work
obligations, fines and penalties under the Petroleum Act of 1949; as well as the government share
representing royalties, rentals, production share on service contracts and similar payments on the
exploration, development and exploitation of energy resources, shall form part of a Special Fund to
be used to finance energy resource development and exploitation programs and projects of the
government and for such other purposes as may be hereafter directed by the President. (Emphases
supplied)

Whereas Section 12 of PD 1869, as amended by PD 1993, reads:

Sec. 12. Special Condition of Franchise. — After deducting five (5%) percent as Franchise Tax, the
Fifty (50%) percent share of the Government in the aggregate gross earnings of the Corporation
from this Franchise, or 60% if the aggregate gross earnings be less than ₱150,000,000.00 shall be
set aside and shall accrue to the General Fund to finance the priority infrastructure development
projects and to finance the restoration of damaged or destroyed facilities due to calamities, as may
be directed and authorized by the Office of the President of the Philippines. (Emphases supplied)

Analyzing the legal text vis-à-vis the above-mentioned principles, it may then be concluded that (a)
Section 8 of PD 910, which creates a Special Fund comprised of "all fees, revenues, and receipts of
the Energy Development Board from any and all sources" (a determinable amount) "to be used to
finance energy resource development and exploitation programs and projects of the government and
for such other purposes as may be hereafter directed by the President" (a specified public purpose),
and (b) Section 12 of PD 1869, as amended by PD 1993, which similarly sets aside, "after deducting
five (5%) percent as Franchise Tax, the Fifty (50%) percent share of the Government in the
aggregate gross earnings of PAGCOR, or 60%, if the aggregate gross earnings be less than
₱150,000,000.00" (also a determinable amount) "to finance the priority infrastructure development
projects and x x x the restoration of damaged or destroyed facilities due to calamities, as may be
directed and authorized by the Office of the President of the Philippines" (also a specified public
purpose), are legal appropriations under Section 29(1), Article VI of the 1987 Constitution.

In this relation, it is apropos to note that the 2013 PDAF Article cannot be properly deemed as a
legal appropriation under the said constitutional provision precisely because, as earlier stated, it
contains post-enactment measures which effectively create a system of intermediate appropriations.
These intermediate appropriations are the actual appropriations meant for enforcement and since
they are made by individual legislators after the GAA is passed, they occur outside the law. As such,
the Court observes that the real appropriation made under the 2013 PDAF Article is not the ₱24.79
Billion allocated for the entire PDAF, but rather the post-enactment determinations made by the
individual legislators which are, to repeat, occurrences outside of the law. Irrefragably, the 2013
PDAF Article does not constitute an "appropriation made by law" since it, in its truest sense, only
authorizes individual legislators to appropriate in violation of the non-delegability principle as afore-
discussed.

2. Undue Delegation.
On a related matter, petitioners contend that Section 8 of PD 910 constitutes an undue delegation of
legislative power since the phrase "and for such other purposes as may be hereafter directed by the
President" gives the President "unbridled discretion to determine for what purpose the funds will be
used."  Respondents, on the other hand, urged the Court to apply the principle of ejusdem generis
243

to the same section and thus, construe the phrase "and for such other purposes as may be hereafter
directed by the President" to refer only to other purposes related "to energy resource development
and exploitation programs and projects of the government." 244

The Court agrees with petitioners‘ submissions.

While the designation of a determinate or determinable amount for a particular public purpose is
sufficient for a legal appropriation to exist, the appropriation law must contain adequate legislative
guidelines if the same law delegates rule-making authority to the Executive  either for the purpose
245

of (a) filling up the details of the law for its enforcement, known as supplementary rule-making, or (b)
ascertaining facts to bring the law into actual operation, referred to as contingent rule-
making.  There are two (2) fundamental tests to ensure that the legislative guidelines for delegated
246

rule-making are indeed adequate. The first test is called the "completeness test." Case law states
that a law is complete when it sets forth therein the policy to be executed, carried out, or
implemented by the delegate. On the other hand, the second test is called the "sufficient standard
test." Jurisprudence holds that a law lays down a sufficient standard when it provides adequate
guidelines or limitations in the law to map out the boundaries of the delegate‘s authority and prevent
the delegation from running riot.  To be sufficient, the standard must specify the limits of the
247

delegate‘s authority, announce the legislative policy, and identify the conditions under which it is to
be implemented. 248

In view of the foregoing, the Court agrees with petitioners that the phrase "and for such other
purposes as may be hereafter directed by the President" under Section 8 of PD 910 constitutes an
undue delegation of legislative power insofar as it does not lay down a sufficient standard to
adequately determine the limits of the President‘s authority with respect to the purpose for which the
Malampaya Funds may be used. As it reads, the said phrase gives the President wide latitude to use
the Malampaya Funds for any other purpose he may direct and, in effect, allows him to unilaterally
appropriate public funds beyond the purview of the law. That the subject phrase may be confined
only to "energy resource development and exploitation programs and projects of the government"
under the principle of ejusdem generis, meaning that the general word or phrase is to be construed
to include – or be restricted to – things akin to, resembling, or of the same kind or class as those
specifically mentioned,  is belied by three (3) reasons: first, the phrase "energy resource
249

development and exploitation programs and projects of the government" states a singular and
general class and hence, cannot be treated as a statutory reference of specific things from which the
general phrase "for such other purposes" may be limited; second, the said phrase also exhausts the
class it represents, namely energy development programs of the government;  and, third, the
250

Executive department has, in fact, used the Malampaya Funds for non-energy related purposes
under the subject phrase, thereby contradicting respondents‘ own position that it is limited only to
"energy resource development and exploitation programs and projects of the government."  Thus, 251

while Section 8 of PD 910 may have passed the completeness test since the policy of energy
development is clearly deducible from its text, the phrase "and for such other purposes as may be
hereafter directed by the President" under the same provision of law should nonetheless be stricken
down as unconstitutional as it lies independently unfettered by any sufficient standard of the
delegating law. This notwithstanding, it must be underscored that the rest of Section 8, insofar as it
allows for the use of the Malampaya Funds "to finance energy resource development and
exploitation programs and projects of the government," remains legally effective and subsisting.
Truth be told, the declared unconstitutionality of the aforementioned phrase is but an assurance that
the Malampaya Funds would be used – as it should be used – only in accordance with the avowed
purpose and intention of PD 910.
As for the Presidential Social Fund, the Court takes judicial notice of the fact that Section 12 of PD
1869 has already been amended by PD 1993 which thus moots the parties‘ submissions on the
same.  Nevertheless, since the amendatory provision may be readily examined under the current
252

parameters of discussion, the Court proceeds to resolve its constitutionality.

Primarily, Section 12 of PD 1869, as amended by PD 1993, indicates that the Presidential Social
Fund may be used "to first, finance the priority infrastructure development projects and second, to
finance the restoration of damaged or destroyed facilities due to calamities, as may be directed and
authorized by the Office of the President of the Philippines." The Court finds that while the second
indicated purpose adequately curtails the authority of the President to spend the Presidential Social
Fund only for restoration purposes which arise from calamities, the first indicated purpose, however,
gives him carte blanche authority to use the same fund for any infrastructure project he may so
determine as a "priority". Verily, the law does not supply a definition of "priority in frastructure
development projects" and hence, leaves the President without any guideline to construe the same.
To note, the delimitation of a project as one of "infrastructure" is too broad of a classification since
the said term could pertain to any kind of facility. This may be deduced from its lexicographic
definition as follows: "the underlying framework of a system, especially public services and facilities
(such as highways, schools, bridges, sewers, and water-systems) needed to support commerce as
well as economic and residential development."  In fine, the phrase "to finance the priority
253

infrastructure development projects" must be stricken down as unconstitutional since – similar to the
above-assailed provision under Section 8 of PD 910 – it lies independently unfettered by any
sufficient standard of the delegating law. As they are severable, all other provisions of Section 12 of
PD 1869, as amended by PD 1993, remains legally effective and subsisting.

D. Ancillary Prayers. 1.

Petitioners’ Prayer to be Furnished Lists and Detailed Reports.

Aside from seeking the Court to declare the Pork Barrel System unconstitutional – as the Court did
so in the context of its pronouncements made in this Decision – petitioners equally pray that the
Executive Secretary and/or the DBM be ordered to release to the CoA and to the public: (a) "the
complete schedule/list of legislators who have availed of their PDAF and VILP from the years 2003
to 2013, specifying the use of the funds, the project or activity and the recipient entities or
individuals, and all pertinent data thereto" (PDAF Use Schedule/List);  and (b) "the use of the
254

Executive‘s lump-sum, discretionary funds, including the proceeds from the x x x Malampaya Funds
and remittances from the PAGCOR x x x from 2003 to 2013, specifying the x x x project or activity
and the recipient entities or individuals, and all pertinent data thereto"  (Presidential Pork Use
255

Report). Petitioners‘ prayer is grounded on Section 28, Article II and Section 7, Article III of the 1987
Constitution which read as follows:

ARTICLE II

Sec. 28. Subject to reasonable conditions prescribed by law, the State adopts and implements a
policy of full public disclosure of all its transactions involving public interest.

ARTICLE III Sec. 7.

The right of the people to information on matters of public concern shall be recognized. Access to
official records, and to documents and papers pertaining to official acts, transactions, or decisions,
as well as to government research data used as basis for policy development, shall be afforded the
citizen, subject to such limitations as may be provided by law.
The Court denies petitioners‘ submission.

Case law instructs that the proper remedy to invoke the right to information is to file a petition for
mandamus. As explained in the case of Legaspi v. Civil Service Commission: 256

While the manner of examining public records may be subject to reasonable regulation by the
government agency in custody thereof, the duty to disclose the information of public concern, and to
afford access to public records cannot be discretionary on the part of said agencies. Certainly, its
performance cannot be made contingent upon the discretion of such agencies. Otherwise, the
enjoyment of the constitutional right may be rendered nugatory by any whimsical exercise of agency
discretion. The constitutional duty, not being discretionary, its performance may be compelled by a
writ of mandamus in a proper case.

But what is a proper case for Mandamus to issue? In the case before Us, the public right to be
enforced and the concomitant duty of the State are unequivocably set forth in the Constitution.

The decisive question on the propriety of the issuance of the writ of mandamus in this case is,
whether the information sought by the petitioner is within the ambit of the constitutional guarantee.
(Emphases supplied)

Corollarily, in the case of Valmonte v. Belmonte Jr.  (Valmonte), it has been clarified that the right to
257

information does not include the right to compel the preparation of "lists, abstracts, summaries and
the like." In the same case, it was stressed that it is essential that the "applicant has a well -defined,
clear and certain legal right to the thing demanded and that it is the imperative duty of defendant to
perform the act required." Hence, without the foregoing substantiations, the Court cannot grant a
particular request for information. The pertinent portions of Valmonte are hereunder quoted: 258

Although citizens are afforded the right to information and, pursuant thereto, are entitled to "access
to official records," the Constitution does not accord them a right to compel custodians of official
records to prepare lists, abstracts, summaries and the like in their desire to acquire information on
matters of public concern.

It must be stressed that it is essential for a writ of mandamus to issue that the applicant has a well-
defined, clear and certain legal right to the thing demanded and that it is the imperative duty of
defendant to perform the act required. The corresponding duty of the respondent to perform the
required act must be clear and specific Lemi v. Valencia, G.R. No. L-20768, November 29,1968,126
SCRA 203; Ocampo v. Subido, G.R. No. L-28344, August 27, 1976, 72 SCRA 443.

The request of the petitioners fails to meet this standard, there being no duty on the part of
respondent to prepare the list requested. (Emphases supplied)

In these cases, aside from the fact that none of the petitions are in the nature of mandamus actions,
the Court finds that petitioners have failed to establish a "a well-defined, clear and certain legal right"
to be furnished by the Executive Secretary and/or the DBM of their requested PDAF Use
Schedule/List and Presidential Pork Use Report. Neither did petitioners assert any law or
administrative issuance which would form the bases of the latter‘s duty to furnish them with the
documents requested. While petitioners pray that said information be equally released to the CoA, it
must be pointed out that the CoA has not been impleaded as a party to these cases nor has it filed
any petition before the Court to be allowed access to or to compel the release of any official
document relevant to the conduct of its audit investigations. While the Court recognizes that the
information requested is a matter of significant public concern, however, if only to ensure that the
parameters of disclosure are properly foisted and so as not to unduly hamper the equally important
interests of the government, it is constrained to deny petitioners‘ prayer on this score, without
prejudice to a proper mandamus case which they, or even the CoA, may choose to pursue through a
separate petition.

It bears clarification that the Court‘s denial herein should only cover petitioners‘ plea to be furnished
with such schedule/list and report and not in any way deny them, or the general public, access to
official documents which are already existing and of public record. Subject to reasonable regulation
and absent any valid statutory prohibition, access to these documents should not be proscribed.
Thus, in Valmonte, while the Court denied the application for mandamus towards the preparation of
the list requested by petitioners therein, it nonetheless allowed access to the documents sought for
by the latter, subject, however, to the custodian‘s reasonable regulations,viz.:259

In fine, petitioners are entitled to access to the documents evidencing loans granted by the GSIS,
subject to reasonable regulations that the latter may promulgate relating to the manner and hours of
examination, to the end that damage to or loss of the records may be avoided, that undue
interference with the duties of the custodian of the records may be prevented and that the right of
other persons entitled to inspect the records may be insured Legaspi v. Civil Service Commission,
supra at p. 538, quoting Subido v. Ozaeta, 80 Phil. 383, 387. The petition, as to the second and third
alternative acts sought to be done by petitioners, is meritorious.

However, the same cannot be said with regard to the first act sought by petitioners, i.e.,

"to furnish petitioners the list of the names of the Batasang Pambansa members belonging to the
UNIDO and PDP-Laban who were able to secure clean loans immediately before the February 7
election thru the intercession/marginal note of the then First Lady Imelda Marcos."

The Court, therefore, applies the same treatment here.

2. Petitioners’ Prayer to Include Matters in Congressional Deliberations.

Petitioners further seek that the Court "order the inclusion in budgetary deliberations with the
Congress of all presently, off-budget, lump sum, discretionary funds including but not limited to,
proceeds from the x x x Malampaya Fund, remittances from the PAGCOR and the PCSO or the
Executive‘s Social Funds." 260

Suffice it to state that the above-stated relief sought by petitioners covers a matter which is generally
left to the prerogative of the political branches of government. Hence, lest the Court itself overreach,
it must equally deny their prayer on this score.

3. Respondents’ Prayer to Lift TRO; Consequential Effects of Decision.

The final issue to be resolved stems from the interpretation accorded by the DBM to the concept of
released funds. In response to the Court‘s September 10, 2013 TRO that enjoined the release of the
remaining PDAF allocated for the year 2013, the DBM issued Circular Letter No. 2013-8 dated
September 27, 2013 (DBM Circular 2013-8) which pertinently reads as follows:

3.0 Nonetheless, PDAF projects funded under the FY 2013 GAA, where a Special Allotment
Release Order (SARO) has been issued by the DBM and such SARO has been obligated by the
implementing agencies prior to the issuance of the TRO, may continually be implemented and
disbursements thereto effected by the agencies concerned.
Based on the text of the foregoing, the DBM authorized the continued implementation and
disbursement of PDAF funds as long as they are: first, covered by a SARO; and, second, that said
SARO had been obligated by the implementing agency concerned prior to the issuance of the
Court‘s September 10, 2013 TRO.

Petitioners take issue with the foregoing circular, arguing that "the issuance of the SARO does not
yet involve the release of funds under the PDAF, as release is only triggered by the issuance of a
Notice of Cash Allocation [(NCA)]."  As such, PDAF disbursements, even if covered by an obligated
261

SARO, should remain enjoined.

For their part, respondents espouse that the subject TRO only covers "unreleased and unobligated
allotments." They explain that once a SARO has been issued and obligated by the implementing
agency concerned, the PDAF funds covered by the same are already "beyond the reach of the TRO
because they cannot be considered as ‘remaining PDAF.‘" They conclude that this is a reasonable
interpretation of the TRO by the DBM. 262

The Court agrees with petitioners in part.

At the outset, it must be observed that the issue of whether or not the Court‘s September 10, 2013
TRO should be lifted is a matter rendered moot by the present Decision. The unconstitutionality of
the 2013 PDAF Article as declared herein has the consequential effect of converting the temporary
injunction into a permanent one. Hence, from the promulgation of this Decision, the release of the
remaining PDAF funds for 2013, among others, is now permanently enjoined.

The propriety of the DBM‘s interpretation of the concept of "release" must, nevertheless, be resolved
as it has a practical impact on the execution of the current Decision. In particular, the Court must
resolve the issue of whether or not PDAF funds covered by obligated SAROs, at the time this
Decision is promulgated, may still be disbursed following the DBM‘s interpretation in DBM Circular
2013-8.

On this score, the Court agrees with petitioners‘ posturing for the fundamental reason that funds
covered by an obligated SARO are yet to be "released" under legal contemplation. A SARO, as
defined by the DBM itself in its website, is "aspecific authority issued to identified agencies to incur
obligations not exceeding a given amount during a specified period for the purpose indicated. It shall
cover expenditures the release of which is subject to compliance with specific laws or regulations, or
is subject to separate approval or clearance by competent authority." 263

Based on this definition, it may be gleaned that a SARO only evinces the existence of an obligation
and not the directive to pay. Practically speaking, the SARO does not have the direct and immediate
effect of placing public funds beyond the control of the disbursing authority. In fact, a SARO may
even be withdrawn under certain circumstances which will prevent the actual release of funds. On
the other hand, the actual release of funds is brought about by the issuance of the NCA,  which is
264

subsequent to the issuance of a SARO. As may be determined from the statements of the DBM
representative during the Oral Arguments: 265

Justice Bernabe: Is the notice of allocation issued simultaneously with the SARO?

xxxx

Atty. Ruiz: It comes after. The SARO, Your Honor, is only the go signal for the agencies to obligate
or to enter into commitments. The NCA, Your Honor, is already the go signal to the treasury for us to
be able to pay or to liquidate the amounts obligated in the SARO; so it comes after. x x x The NCA,
Your Honor, is the go signal for the MDS for the authorized government-disbursing banks to,
therefore, pay the payees depending on the projects or projects covered by the SARO and the NCA.

Justice Bernabe: Are there instances that SAROs are cancelled or revoked?

Atty. Ruiz: Your Honor, I would like to instead submit that there are instances that the SAROs issued
are withdrawn by the DBM.

Justice Bernabe: They are withdrawn?

Atty. Ruiz: Yes, Your Honor x x x. (Emphases and underscoring supplied)

Thus, unless an NCA has been issued, public funds should not be treated as funds which have been
"released." In this respect, therefore, the disbursement of 2013 PDAF funds which are only covered
by obligated SAROs, and without any corresponding NCAs issued, must, at the time of this
Decision’s promulgation, be enjoined and consequently reverted to the unappropriated surplus of the
general fund. Verily, in view of the declared unconstitutionality of the 2013 PDAF Article, the funds
appropriated pursuant thereto cannot be disbursed even though already obligated, else the Court
sanctions the dealing of funds coming from an unconstitutional source.

This same pronouncement must be equally applied to (a) the Malampaya Funds which have been
obligated but not released – meaning, those merely covered by a SARO – under the phrase "and for
such other purposes as may be hereafter directed by the President" pursuant to Section 8 of PD
910; and (b) funds sourced from the Presidential Social Fund under the phrase "to finance the
priority infrastructure development projects" pursuant to Section 12 of PD 1869, as amended by PD
1993, which were altogether declared by the Court as unconstitutional. However, these funds should
not be reverted to the general fund as afore-stated but instead, respectively remain under the
Malampaya Funds and the Presidential Social Fund to be utilized for their corresponding special
purposes not otherwise declared as unconstitutional.

E. Consequential Effects of Decision.

As a final point, it must be stressed that the Court‘s pronouncement anent the unconstitutionality of
(a) the 2013 PDAF Article and its Special Provisions, (b) all other Congressional Pork Barrel
provisions similar thereto, and (c) the phrases (1) "and for such other purposes as may be hereafter
directed by the President" under Section 8 of PD 910, and (2) "to finance the priority infrastructure
development projects" under Section 12 of PD 1869, as amended by PD 1993, must only be treated
as prospective in effect in view of the operative fact doctrine.

To explain, the operative fact doctrine exhorts the recognition that until the judiciary, in an
appropriate case, declares the invalidity of a certain legislative or executive act, such act is
presumed constitutional and thus, entitled to obedience and respect and should be properly
enforced and complied with. As explained in the recent case of Commissioner of Internal Revenue v.
San Roque Power Corporation,  the doctrine merely "reflects awareness that precisely because the
266

judiciary is the governmental organ which has the final say on whether or not a legislative or
executive measure is valid, a period of time may have elapsed before it can exercise the power of
judicial review that may lead to a declaration of nullity. It would be to deprive the law of its quality of
fairness and justice then, if there be no recognition of what had transpired prior to such
adjudication."  "In the language of an American Supreme Court decision: ‘The actual existence of a
267

statute, prior to such a determination of unconstitutionality, is an operative fact and may have
consequences which cannot justly be ignored.‘" 268
For these reasons, this Decision should be heretofore applied prospectively.

Conclusion

The Court renders this Decision to rectify an error which has persisted in the chronicles of our
history. In the final analysis, the Court must strike down the Pork Barrel System as unconstitutional
in view of the inherent defects in the rules within which it operates. To recount, insofar as it has
allowed legislators to wield, in varying gradations, non-oversight, post-enactment authority in vital
areas of budget execution, the system has violated the principle of separation of powers; insofar as it
has conferred unto legislators the power of appropriation by giving them personal, discretionary
funds from which they are able to fund specific projects which they themselves determine, it has
similarly violated the principle of non-delegability of legislative power ; insofar as it has created a
system of budgeting wherein items are not textualized into the appropriations bill, it has flouted the
prescribed procedure of presentment and, in the process, denied the President the power to veto
items ; insofar as it has diluted the effectiveness of congressional oversight by giving legislators a
stake in the affairs of budget execution, an aspect of governance which they may be called to
monitor and scrutinize, the system has equally impaired public accountability ; insofar as it has
authorized legislators, who are national officers, to intervene in affairs of purely local nature, despite
the existence of capable local institutions, it has likewise subverted genuine local autonomy ; and
again, insofar as it has conferred to the President the power to appropriate funds intended by law for
energy-related purposes only to other purposes he may deem fit as well as other public funds under
the broad classification of "priority infrastructure development projects," it has once more
transgressed the principle of non-delegability.

For as long as this nation adheres to the rule of law, any of the multifarious unconstitutional methods
and mechanisms the Court has herein pointed out should never again be adopted in any system of
governance, by any name or form, by any semblance or similarity, by any influence or effect.
Disconcerting as it is to think that a system so constitutionally unsound has monumentally endured,
the Court urges the people and its co-stewards in government to look forward with the optimism of
change and the awareness of the past. At a time of great civic unrest and vociferous public debate,
the Court fervently hopes that its Decision today, while it may not purge all the wrongs of society nor
bring back what has been lost, guides this nation to the path forged by the Constitution so that no
one may heretofore detract from its cause nor stray from its course. After all, this is the Court‘s
bounden duty and no other‘s.

WHEREFORE, the petitions are PARTLY GRANTED. In view of the constitutional violations
discussed in this Decision, the Court hereby declares as UNCONSTITUTIONAL: (a) the entire 2013
PDAF Article; (b) all legal provisions of past and present Congressional Pork Barrel Laws, such as
the previous PDAF and CDF Articles and the various Congressional Insertions, which authorize/d
legislators – whether individually or collectively organized into committees – to intervene, assume or
participate in any of the various post-enactment stages of the budget execution, such as but not
limited to the areas of project identification, modification and revision of project identification, fund
release and/or fund realignment, unrelated to the power of congressional oversight; (c) all legal
provisions of past and present Congressional Pork Barrel Laws, such as the previous PDAF and
CDF Articles and the various Congressional Insertions, which confer/red personal, lump-sum
allocations to legislators from which they are able to fund specific projects which they themselves
determine; (d) all informal practices of similar import and effect, which the Court similarly deems to
be acts of grave abuse of discretion amounting to lack or excess of jurisdiction; and (e) the phrases
(1) "and for such other purposes as may be hereafter directed by the President" under Section 8 of
Presidential Decree No. 910 and (2) "to finance the priority infrastructure development projects"
under Section 12 of Presidential Decree No. 1869, as amended by Presidential Decree No. 1993, for
both failing the sufficient standard test in violation of the principle of non-delegability of legislative
power.

Accordingly, the Court‘s temporary injunction dated September 10, 2013 is hereby declared to be
PERMANENT. Thus, the disbursement/release of the remaining PDAF funds allocated for the year
2013, as well as for all previous years, and the funds sourced from (1) the Malampaya Funds under
the phrase "and for such other purposes as may be hereafter directed by the President" pursuant to
Section 8 of Presidential Decree No. 910, and (2) the Presidential Social Fund under the phrase "to
finance the priority infrastructure development projects" pursuant to Section 12 of Presidential
Decree No. 1869, as amended by Presidential Decree No. 1993, which are, at the time this Decision
is promulgated, not covered by Notice of Cash Allocations (NCAs) but only by Special Allotment
Release Orders (SAROs), whether obligated or not, are hereby ENJOINED. The remaining PDAF
funds covered by this permanent injunction shall not be disbursed/released but instead reverted to
the unappropriated surplus of the general fund, while the funds under the Malampaya Funds and the
Presidential Social Fund shall remain therein to be utilized for their respective special purposes not
otherwise declared as unconstitutional.

On the other hand, due to improper recourse and lack of proper substantiation, the Court hereby
DENIES petitioners‘ prayer seeking that the Executive Secretary and/or the Department of Budget
and Management be ordered to provide the public and the Commission on Audit complete
lists/schedules or detailed reports related to the availments and utilization of the funds subject of
these cases. Petitioners‘ access to official documents already available and of public record which
are related to these funds must, however, not be prohibited but merely subjected to the custodian‘s
reasonable regulations or any valid statutory prohibition on the same. This denial is without prejudice
to a proper mandamus case which they or the Commission on Audit may choose to pursue through
a separate petition.

The Court also DENIES petitioners prayer to order the inclusion of the funds subject of these cases
in the budgetary deliberations of Congress as the same is a matter left to the prerogative of the
political branches of government.

Finally, the Court hereby DIRECTS all prosecutorial organs of the government to, within the bounds
of reasonable dispatch, investigate and accordingly prosecute all government officials and/or private
individuals for possible criminal offenses related to the irregular, improper and/or unlawful
disbursement/utilization of all funds under the Pork Barrel System.

This Decision is immediately executory but prospective in effect.

SO ORDERED.

G.R. No. 173034             October 9, 2007

PHARMACEUTICAL AND HEALTH CARE ASSOCIATION OF THE PHILIPPINES, petitioner, 


vs.
HEALTH SECRETARY FRANCISCO T. DUQUE III; HEALTH UNDER SECRETARIES DR.
ETHELYN P. NIETO, DR. MARGARITA M. GALON, ATTY. ALEXANDER A. PADILLA, & DR.
JADE F. DEL MUNDO; and ASSISTANT SECRETARIES DR. MARIO C. VILLAVERDE, DR.
DAVID J. LOZADA, AND DR. NEMESIO T. GAKO,respondents.
DECISION

AUSTRIA-MARTINEZ, J.:

The Court and all parties involved are in agreement that the best nourishment for an infant is
mother's milk. There is nothing greater than for a mother to nurture her beloved child straight from
her bosom. The ideal is, of course, for each and every Filipino child to enjoy the unequaled benefits
of breastmilk. But how should this end be attained?

Before the Court is a petition for certiorari under Rule 65 of the Rules of Court, seeking to nullify
Administrative Order (A.O.) No. 2006-0012 entitled, Revised Implementing Rules and
Regulations of Executive Order No. 51, Otherwise Known as The "Milk Code," Relevant
International Agreements, Penalizing Violations Thereof, and for Other Purposes (RIRR).
Petitioner posits that the RIRR is not valid as it contains provisions that are not constitutional and go
beyond the law it is supposed to implement.

Named as respondents are the Health Secretary, Undersecretaries, and Assistant Secretaries of the
Department of Health (DOH). For purposes of herein petition, the DOH is deemed impleaded as a
co-respondent since respondents issued the questioned RIRR in their capacity as officials of said
executive agency.1

Executive Order No. 51 (Milk Code) was issued by President Corazon Aquino on October 28, 1986
by virtue of the legislative powers granted to the president under the Freedom Constitution. One of
the preambular clauses of the Milk Code states that the law seeks to give effect to Article 112 of the
International Code of Marketing of Breastmilk Substitutes (ICMBS), a code adopted by the World
Health Assembly (WHA) in 1981. From 1982 to 2006, the WHA adopted several Resolutions to the
effect that breastfeeding should be supported, promoted and protected, hence, it should be ensured
that nutrition and health claims are not permitted for breastmilk substitutes.

In 1990, the Philippines ratified the International Convention on the Rights of the Child. Article 24 of
said instrument provides that State Parties should take appropriate measures to diminish infant and
child mortality, and ensure that all segments of society, specially parents and children, are informed
of the advantages of breastfeeding.

On May 15, 2006, the DOH issued herein assailed RIRR which was to take effect on July 7, 2006.

However, on June 28, 2006, petitioner, representing its members that are manufacturers of
breastmilk substitutes, filed the present Petition for Certiorari and Prohibition with Prayer for the
Issuance of a Temporary Restraining Order (TRO) or Writ of Preliminary Injunction.

The main issue raised in the petition is whether respondents officers of the DOH acted without or in
excess of jurisdiction, or with grave abuse of discretion amounting to lack or excess of jurisdiction,
and in violation of the provisions of the Constitution in promulgating the RIRR.3

On August 15, 2006, the Court issued a Resolution granting a TRO enjoining respondents from
implementing the questioned RIRR.

After the Comment and Reply had been filed, the Court set the case for oral arguments on June 19,
2007. The Court issued an Advisory (Guidance for Oral Arguments) dated June 5, 2007, to wit:

The Court hereby sets the following issues:


1. Whether or not petitioner is a real party-in-interest;

2. Whether Administrative Order No. 2006-0012 or the Revised Implementing Rules and
Regulations (RIRR) issued by the Department of Health (DOH) is not constitutional;

2.1 Whether the RIRR is in accord with the provisions of Executive Order No. 51 (Milk Code);

2.2 Whether pertinent international agreements1 entered into by the Philippines are part of
the law of the land and may be implemented by the DOH through the RIRR; If in the
affirmative, whether the RIRR is in accord with the international agreements;

2.3 Whether Sections 4, 5(w), 22, 32, 47, and 52 of the RIRR violate the due process clause
and are in restraint of trade; and

2.4 Whether Section 13 of the RIRR on Total Effect provides sufficient standards.

_____________

1 (1) United Nations Convention on the Rights of the Child; (2) the WHO and Unicef "2002
Global Strategy on Infant and Young Child Feeding;" and (3) various World Health Assembly
(WHA) Resolutions.

The parties filed their respective memoranda.

The petition is partly imbued with merit.

On the issue of petitioner's standing

With regard to the issue of whether petitioner may prosecute this case as the real party-in-interest,
the Court adopts the view enunciated in Executive Secretary v. Court of Appeals,4 to wit:

The modern view is that an association has standing to complain of injuries to its members.
This view fuses the legal identity of an association with that of its members. An association
has standing to file suit for its workers despite its lack of direct interest if its members
are affected by the action. An organization has standing to assert the concerns of its
constituents.

xxxx

x x x We note that, under its Articles of Incorporation, the respondent was organized x x x to
act as the representative of any individual, company, entity or association on matters related
to the manpower recruitment industry, and to perform other acts and activities necessary to
accomplish the purposes embodied therein. The respondent is, thus, the appropriate
party to assert the rights of its members, because it and its members are in every
practical sense identical. x x x The respondent [association] is but the medium
through which its individual members seek to make more effective the expression of
their voices and the redress of their grievances. 5 (Emphasis supplied)

which was reasserted in Purok Bagong Silang Association, Inc. v. Yuipco,6 where the Court ruled
that an association has the legal personality to represent its members because the results of the
case will affect their vital interests.7
Herein petitioner's Amended Articles of Incorporation contains a similar provision just like in
Executive Secretary, that the association is formed "to represent directly or through approved
representatives the pharmaceutical and health care industry before the Philippine Government and
any of its agencies, the medical professions and the general public."8 Thus, as an organization,
petitioner definitely has an interest in fulfilling its avowed purpose of representing members who are
part of the pharmaceutical and health care industry. Petitioner is duly authorized 9 to take the
appropriate course of action to bring to the attention of government agencies and the courts any
grievance suffered by its members which are directly affected by the RIRR. Petitioner, which is
mandated by its Amended Articles of Incorporation to represent the entire industry, would be remiss
in its duties if it fails to act on governmental action that would affect any of its industry members, no
matter how few or numerous they are. Hence, petitioner, whose legal identity is deemed fused with
its members, should be considered as a real party-in-interest which stands to be benefited or injured
by any judgment in the present action.

On the constitutionality of the provisions of the RIRR

First, the Court will determine if pertinent international instruments adverted to by respondents are
part of the law of the land.

Petitioner assails the RIRR for allegedly going beyond the provisions of the Milk Code, thereby
amending and expanding the coverage of said law. The defense of the DOH is that the RIRR
implements not only the Milk Code but also various international instruments10 regarding infant and
young child nutrition. It is respondents' position that said international instruments are deemed part
of the law of the land and therefore the DOH may implement them through the RIRR.

The Court notes that the following international instruments invoked by respondents, namely: (1) The
United Nations Convention on the Rights of the Child; (2) The International Covenant on Economic,
Social and Cultural Rights; and (3) the Convention on the Elimination of All Forms of Discrimination
Against Women, only provide in general terms that steps must be taken by State Parties to diminish
infant and child mortality and inform society of the advantages of breastfeeding, ensure the health
and well-being of families, and ensure that women are provided with services and nutrition in
connection with pregnancy and lactation. Said instruments do not contain specific provisions
regarding the use or marketing of breastmilk substitutes.

The international instruments that do have specific provisions regarding breastmilk substitutes are
the ICMBS and various WHA Resolutions.

Under the 1987 Constitution, international law can become part of the sphere of domestic law either
by transformation or incorporation.11 The transformation method requires that an international law
be transformed into a domestic law through a constitutional mechanism such as local legislation.
The incorporation method applies when, by mere constitutional declaration, international law is
deemed to have the force of domestic law.12

Treaties become part of the law of the land through transformation pursuant to Article VII, Section
21 of the Constitution which provides that "[n]o treaty or international agreement shall be valid and
effective unless concurred in by at least two-thirds of all the members of the Senate." Thus, treaties
or conventional international law must go through a process prescribed by the Constitution for it to
be transformed into municipal law that can be applied to domestic conflicts.13

The ICMBS and WHA Resolutions are not treaties as they have not been concurred in by at least
two-thirds of all members of the Senate as required under Section 21, Article VII of the 1987
Constitution.
However, the ICMBS which was adopted by the WHA in 1981 had been transformed into domestic
law through local legislation, the Milk Code. Consequently, it is the Milk Code that has the force and
effect of law in this jurisdiction and not the ICMBS per se.

The Milk Code is almost a verbatim reproduction of the ICMBS, but it is well to emphasize at this
point that the Code did not adopt the provision in the ICMBS absolutely prohibiting advertising or
other forms of promotion to the general public of products within the scope of the ICMBS.
Instead, the Milk Code expressly provides that advertising, promotion, or other marketing
materials may be allowed if such materials are duly authorized and approved by the Inter-
Agency Committee (IAC).

On the other hand, Section 2, Article II of the 1987 Constitution, to wit:

SECTION 2. The Philippines renounces war as an instrument of national policy, adopts the


generally accepted principles of international law as part of the law of the land and
adheres to the policy of peace, equality, justice, freedom, cooperation and amity with all
nations. (Emphasis supplied)

embodies the incorporation method.14

In Mijares v. Ranada,15 the Court held thus:

[G]enerally accepted principles of international law, by virtue of the incorporation clause of


the Constitution, form part of the laws of the land even if they do not derive from treaty
obligations. The classical formulation in international law sees those customary rules
accepted as binding result from the combination [of] two elements: the established,
widespread, and consistent practice on the part of States; and a psychological element
known as the opinion juris sive necessitates (opinion as to law or necessity). Implicit in the
latter element is a belief that the practice in question is rendered obligatory by the existence
of a rule of law requiring it.16 (Emphasis supplied)

"Generally accepted principles of international law" refers to norms of general or customary


international law which are binding on all states,17 i.e., renunciation of war as an instrument of
national policy, the principle of sovereign immunity,18 a person's right to life, liberty and due
process,19 and pacta sunt servanda,20 among others. The concept of "generally accepted principles
of law" has also been depicted in this wise:

Some legal scholars and judges look upon certain "general principles of law" as a primary source of
international law because they have the "character of jus rationale" and are "valid through all
kinds of human societies."(Judge Tanaka in his dissenting opinion in the 1966 South West Africa
Case, 1966 I.C.J. 296). O'Connell holds that certain priniciples are part of international law
because they are "basic to legal systems generally" and hence part of the jus gentium. These
principles, he believes, are established by a process of reasoning based on the common identity of
all legal systems. If there should be doubt or disagreement, one must look to state practice and
determine whether the municipal law principle provides a just and acceptable solution. x x
x 21 (Emphasis supplied)

Fr. Joaquin G. Bernas defines customary international law as follows:

Custom or customary international law means "a general and consistent practice of states
followed by them from a sense of legal obligation [opinio juris]." (Restatement) This
statement contains the two basic elements of custom: the material factor, that is, how
states behave, and the psychological or subjective factor, that is, why they behave the
way they do.

xxxx

The initial factor for determining the existence of custom is the actual behavior of states. This
includes several elements: duration, consistency, and generality of the practice of states.

The required duration can be either short or long. x x x

xxxx

Duration therefore is not the most important element. More important is the consistency and
the generality of the practice. x x x

xxxx

Once the existence of state practice has been established, it becomes necessary to
determine why states behave the way they do. Do states behave the way they do
because they consider it obligatory to behave thus or do they do it only as a matter of
courtesy? Opinio juris, or the belief that a certain form of behavior is obligatory, is
what makes practice an international rule. Without it, practice is not law.22(Underscoring
and Emphasis supplied)

Clearly, customary international law is deemed incorporated into our domestic system.23

WHA Resolutions have not been embodied in any local legislation. Have they attained the status of
customary law and should they then be deemed incorporated as part of the law of the land?

The World Health Organization (WHO) is one of the international specialized agencies allied with the
United Nations (UN) by virtue of Article 57,24 in relation to Article 6325 of the UN Charter. Under the
1946 WHO Constitution, it is the WHA which determines the policies of the WHO, 26 and has the
power to adopt regulations concerning "advertising and labeling of biological, pharmaceutical and
similar products moving in international commerce,"27and to "make recommendations to members
with respect to any matter within the competence of the Organization."28 The legal effect of its
regulations, as opposed to recommendations, is quite different.

Regulations, along with conventions and agreements, duly adopted by the WHA bind member
states thus:

Article 19. The Health Assembly shall have authority to adopt conventions or agreements
with respect to any matter within the competence of the Organization. A two-thirds vote of
the Health Assembly shall be required for the adoption of such conventions or
agreements, which shall come into force for each Member when accepted by it in
accordance with its constitutional processes.

Article 20. Each Member undertakes that it will, within eighteen months after the adoption
by the Health Assembly of a convention or agreement, take action relative to the
acceptance of such convention or agreement. Each Member shall notify the Director-
General of the action taken, and if it does not accept such convention or agreement within
the time limit, it will furnish a statement of the reasons for non-acceptance. In case of
acceptance, each Member agrees to make an annual report to the Director-General in
accordance with Chapter XIV.

Article 21. The Health Assembly shall have authority to adopt regulations concerning: (a)
sanitary and quarantine requirements and other procedures designed to prevent the
international spread of disease; (b) nomenclatures with respect to diseases, causes of death
and public health practices; (c) standards with respect to diagnostic procedures for
international use; (d) standards with respect to the safety, purity and potency of biological,
pharmaceutical and similar products moving in international commerce; (e) advertising and
labeling of biological, pharmaceutical and similar products moving in international commerce.

Article 22. Regulations adopted pursuant to Article 21 shall come into force for all
Members after due notice has been given of their adoption by the Health Assembly except
for such Members as may notify the Director-General of rejection or reservations within the
period stated in the notice. (Emphasis supplied)

On the other hand, under Article 23, recommendations of the WHA do not come into force for
members, in the same way that conventions or agreements under Article 19 and regulations under
Article 21 come into force. Article 23 of the WHO Constitution reads:

Article 23. The Health Assembly shall have authority to make recommendations to


Members with respect to any matter within the competence of the Organization. (Emphasis
supplied)

The absence of a provision in Article 23 of any mechanism by which the recommendation would
come into force for member states is conspicuous.

The former Senior Legal Officer of WHO, Sami Shubber, stated that WHA recommendations are
generally not binding, but they "carry moral and political weight, as they constitute the judgment on a
health issue of the collective membership of the highest international body in the field of
health."29 Even the ICMBS itself was adopted as a mere recommendation, as WHA Resolution No.
34.22 states:

"The Thirty-Fourth World Health Assembly x x x adopts, in the sense of Article 23 of the
Constitution, the International Code of Marketing of Breastmilk Substitutes annexed to the
present resolution." (Emphasis supplied)

The Introduction to the ICMBS also reads as follows:

In January 1981, the Executive Board of the World Health Organization at its sixty-seventh
session, considered the fourth draft of the code, endorsed it, and unanimously
recommended to the Thirty-fourth World Health Assembly the text of a resolution by which it
would adopt the code in the form of a recommendation rather than a regulation. x x x
(Emphasis supplied)

The legal value of WHA Resolutions as recommendations is summarized in Article 62 of the WHO
Constitution, to wit:
Art. 62. Each member shall report annually on the action taken with respect to
recommendations made to it by the Organization, and with respect to conventions,
agreements and regulations.

Apparently, the WHA Resolution adopting the ICMBS and subsequent WHA Resolutions urging
member states to implement the ICMBS are merely recommendatory and legally non-binding. Thus,
unlike what has been done with the ICMBS whereby the legislature enacted most of the
provisions into law which is the Milk Code, the subsequent WHA Resolutions, 30 specifically
providing for exclusive breastfeeding from 0-6 months, continued breastfeeding up to 24
months, and absolutely prohibiting advertisements and promotions of breastmilk substitutes,
have not been adopted as a domestic law.

It is propounded that WHA Resolutions may constitute "soft law" or non-binding norms, principles
and practices that influence state behavior.31

"Soft law" does not fall into any of the categories of international law set forth in Article 38, Chapter
III of the 1946 Statute of the International Court of Justice. 32 It is, however, an expression of non-
binding norms, principles, and practices that influence state behavior. 33 Certain declarations and
resolutions of the UN General Assembly fall under this category.34 The most notable is the UN
Declaration of Human Rights, which this Court has enforced in various cases,
specifically, Government of Hongkong Special Administrative Region v. Olalia,35 Mejoff v. Director of
Prisons,36 Mijares v. Rañada37 and Shangri-la International Hotel Management, Ltd. v. Developers
Group of Companies, Inc..38

The World Intellectual Property Organization (WIPO), a specialized agency attached to the UN with
the mandate to promote and protect intellectual property worldwide, has resorted to soft law as a
rapid means of norm creation, in order "to reflect and respond to the changing needs and demands
of its constituents."39 Other international organizations which have resorted to soft law include the
International Labor Organization and the Food and Agriculture Organization (in the form of
the Codex Alimentarius).40

WHO has resorted to soft law. This was most evident at the time of the Severe Acute Respiratory
Syndrome (SARS) and Avian flu outbreaks.

Although the IHR Resolution does not create new international law binding on WHO
member states, it provides an excellent example of the power of "soft law" in
international relations. International lawyers typically distinguish binding rules of
international law-"hard law"-from non-binding norms, principles, and practices that
influence state behavior-"soft law." WHO has during its existence generated many
soft law norms, creating a "soft law regime" in international governance for public
health.

The "soft law" SARS and IHR Resolutions represent significant steps in laying the political
groundwork for improved international cooperation on infectious diseases. These resolutions
clearly define WHO member states' normative duty to cooperate fully with other countries
and with WHO in connection with infectious disease surveillance and response to outbreaks.

This duty is neither binding nor enforceable, but, in the wake of the SARS epidemic,
the duty is powerful politically for two reasons. First, the SARS outbreak has taught the
lesson that participating in, and enhancing, international cooperation on infectious disease
controls is in a country's self-interest x x x if this warning is heeded, the "soft law" in the
SARS and IHR Resolution could inform the development of general and consistent state
practice on infectious disease surveillance and outbreak response, perhaps crystallizing
eventually into customary international law on infectious disease prevention and control.41

In the Philippines, the executive department implemented certain measures recommended by WHO
to address the outbreaks of SARS and Avian flu by issuing Executive Order (E.O.) No. 201 on April
26, 2003 and E.O. No. 280 on February 2, 2004, delegating to various departments broad powers to
close down schools/establishments, conduct health surveillance and monitoring, and ban importation
of poultry and agricultural products.

It must be emphasized that even under such an international emergency, the duty of a state to
implement the IHR Resolution was still considered not binding or enforceable, although said
resolutions had great political influence.

As previously discussed, for an international rule to be considered as customary law, it must be


established that such rule is being followed by states because they consider it obligatory to comply
with such rules (opinio juris). Respondents have not presented any evidence to prove that the WHA
Resolutions, although signed by most of the member states, were in fact enforced or practiced by at
least a majority of the member states; neither have respondents proven that any compliance by
member states with said WHA Resolutions was obligatory in nature.

Respondents failed to establish that the provisions of pertinent WHA Resolutions are customary
international law that may be deemed part of the law of the land.

Consequently, legislation is necessary to transform the provisions of the WHA Resolutions into
domestic law. The provisions of the WHA Resolutions cannot be considered as part of the law
of the land that can be implemented by executive agencies without the need of a law enacted
by the legislature.

Second, the Court will determine whether the DOH may implement the provisions of the WHA
Resolutions by virtue of its powers and functions under the Revised Administrative Code even in the
absence of a domestic law.

Section 3, Chapter 1, Title IX of the Revised Administrative Code of 1987 provides that the DOH
shall define the national health policy and implement a national health plan within the framework
of the government's general policies and plans, and issue orders and regulations concerning the
implementation of established health policies.

It is crucial to ascertain whether the absolute prohibition on advertising and other forms of promotion
of breastmilk substitutes provided in some WHA Resolutions has been adopted as part of the
national health policy.

Respondents submit that the national policy on infant and young child feeding is embodied in A.O.
No. 2005-0014, dated May 23, 2005. Basically, the Administrative Order declared the following
policy guidelines: (1) ideal breastfeeding practices, such as early initiation of breastfeeding,
exclusive breastfeeding for the first six months, extended breastfeeding up to two years and beyond;
(2) appropriate complementary feeding, which is to start at age six months; (3) micronutrient
supplementation; (4) universal salt iodization; (5) the exercise of other feeding options; and (6)
feeding in exceptionally difficult circumstances. Indeed, the primacy of breastfeeding for children is
emphasized as a national health policy. However, nowhere in A.O. No. 2005-0014 is it declared
that as part of such health policy, the advertisement or promotion of breastmilk substitutes
should be absolutely prohibited.
The national policy of protection, promotion and support of breastfeeding cannot automatically be
equated with a total ban on advertising for breastmilk substitutes.

In view of the enactment of the Milk Code which does not contain a total ban on the advertising and
promotion of breastmilk substitutes, but instead, specifically creates an IAC which will regulate said
advertising and promotion, it follows that a total ban policy could be implemented only pursuant to a
law amending the Milk Code passed by the constitutionally authorized branch of government, the
legislature.

Thus, only the provisions of the Milk Code, but not those of subsequent WHA Resolutions, can
be validly implemented by the DOH through the subject RIRR.

Third, the Court will now determine whether the provisions of the RIRR are in accordance with those
of the Milk Code.

In support of its claim that the RIRR is inconsistent with the Milk Code, petitioner alleges the
following:

1. The Milk Code limits its coverage to children 0-12 months old, but the RIRR extended its
coverage to "young children" or those from ages two years old and beyond:

MILK CODE RIRR


WHEREAS, in order to ensure that safe and Section 2. Purpose – These Revised Rules
adequate nutrition for infants is provided, there and Regulations are hereby promulgated to
is a need to protect and promote breastfeeding ensure the provision of safe and adequate
and to inform the public about the proper use of nutrition for infants and young children by the
breastmilk substitutes and supplements and promotion, protection and support of
related products through adequate, consistent breastfeeding and by ensuring the proper use
and objective information and appropriate of breastmilk substitutes, breastmilk
regulation of the marketing and distribution of supplements and related products when these
the said substitutes, supplements and related are medically indicated and only when
products; necessary, on the basis of adequate
information and through appropriate marketing
SECTION 4(e). "Infant" means a person falling and distribution.
within the age bracket of 0-12 months.
Section 5(ff). "Young Child" means a person
from the age of more than twelve (12) months
up to the age of three (3) years (36 months).

2. The Milk Code recognizes that infant formula may be a proper and possible substitute for
breastmilk in certain instances; but the RIRR provides "exclusive breastfeeding for infants
from 0-6 months" and declares that "there is no substitute nor replacement for breastmilk":

MILK CODE RIRR


WHEREAS, in order to ensure that safe and Section 4. Declaration of Principles – The
adequate nutrition for infants is provided, there following are the underlying principles from
is a need to protect and promote breastfeeding which the revised rules and regulations are
and to inform the public about the proper use of premised upon:
breastmilk substitutes and supplements and
related products through adequate, consistent a. Exclusive breastfeeding is for infants from 0
and objective information and appropriate
regulation of the marketing and distribution of
the said substitutes, supplements and related to six (6) months.
products;
b. There is no substitute or replacement for
breastmilk.

3. The Milk Code only regulates and does not impose unreasonable requirements for
advertising and promotion; RIRR imposes an absolute ban on such activities for breastmilk
substitutes intended for infants from 0-24 months old or beyond, and forbids the use of
health and nutritional claims. Section 13 of the RIRR, which provides for a "total effect" in the
promotion of products within the scope of the Code, is vague:

MILK CODE RIRR


SECTION 6. The General Public and Section 4. Declaration of Principles – The
Mothers. – following are the underlying principles from
which the revised rules and regulations are
(a) No advertising, promotion or other premised upon:
marketing materials, whether written, audio or
visual, for products within the scope of this x x x x
Code shall be printed, published, distributed,
exhibited and broadcast unless such materials f. Advertising, promotions, or sponsor-ships of
are duly authorized and approved by an inter- infant formula, breastmilk substitutes and other
agency committee created herein pursuant to related products are prohibited.
the applicable standards provided for in this
Code. Section 11. Prohibition – No advertising,
promotions, sponsorships, or marketing
materials and activities for breastmilk
substitutes intended for infants and young
children up to twenty-four (24) months, shall be
allowed, because they tend to convey or give
subliminal messages or impressions that
undermine breastmilk and breastfeeding or
otherwise exaggerate breastmilk substitutes
and/or replacements, as well as related
products covered within the scope of this Code.

Section 13. "Total Effect" - Promotion of


products within the scope of this Code must be
objective and should not equate or make the
product appear to be as good or equal to
breastmilk or breastfeeding in the advertising
concept. It must not in any case undermine
breastmilk or breastfeeding. The "total effect"
should not directly or indirectly suggest that
buying their product would produce better
individuals, or resulting in greater love,
intelligence, ability, harmony or in any manner
bring better health to the baby or other such
exaggerated and unsubstantiated claim.

Section 15. Content of Materials. - The


following shall not be included in advertising,
promotional and marketing materials:

a. Texts, pictures, illustrations or information


which discourage or tend to undermine the
benefits or superiority of breastfeeding or which
idealize the use of breastmilk substitutes and
milk supplements. In this connection, no
pictures of babies and children together with
their mothers, fathers, siblings, grandparents,
other relatives or caregivers (or yayas) shall be
used in any advertisements for infant formula
and breastmilk supplements;

b. The term "humanized," "maternalized,"


"close to mother's milk" or similar words in
describing breastmilk substitutes or milk
supplements;

c. Pictures or texts that idealize the use of


infant and milk formula.

Section 16. All health and nutrition claims for


products within the scope of the Code are
absolutely prohibited. For this purpose, any
phrase or words that connotes to increase
emotional, intellectual abilities of the infant and
young child and other like phrases shall not be
allowed.

4. The RIRR imposes additional labeling requirements not found in the Milk Code:

MILK CODE RIRR


SECTION 10. Containers/Label. – Section 26. Content – Each container/label
shall contain such message, in both Filipino
(a) Containers and/or labels shall be designed and English languages, and which message
to provide the necessary information about the cannot be readily separated therefrom, relative
appropriate use of the products, and in such a the following points:
way as not to discourage breastfeeding.
(a) The words or phrase "Important Notice" or
(b) Each container shall have a clear, "Government Warning" or their equivalent;
conspicuous and easily readable and
understandable message in Pilipino or English (b) A statement of the superiority of
printed on it, or on a label, which message can breastfeeding;
not readily become separated from it, and
which shall include the following points: (c) A statement that there is no substitute for
breastmilk;
(i) the words "Important Notice" or their
equivalent; (d) A statement that the product shall be used
only on the advice of a health worker as to the
(ii) a statement of the superiority of need for its use and the proper methods of use;
breastfeeding;
(e) Instructions for appropriate prepara-tion,
(iii) a statement that the product shall be used and a warning against the health hazards of
only on the advice of a health worker as to the inappropriate preparation; and
need for its use and the proper methods of use;
and (f) The health hazards of unnecessary or
improper use of infant formula and other
(iv) instructions for appropriate preparation, related products including information that
and a warning against the health hazards of powdered infant formula may contain
inappropriate preparation. pathogenic microorganisms and must be
prepared and used appropriately.

5. The Milk Code allows dissemination of information on infant formula to health


professionals; the RIRR totally prohibits such activity:

MILK CODE RIRR


SECTION 7. Health Care System. – Section 22. No manufacturer, distributor, or
representatives of products covered by the
(b) No facility of the health care system shall Code shall be allowed to conduct or be
be used for the purpose of promoting infant involved in any activity on breastfeeding
formula or other products within the scope of promotion, education and production of
this Code. This Code does not, however, Information, Education and Communication
preclude the dissemination of information to (IEC) materials on breastfeeding, holding of or
health professionals as provided in Section participating as speakers in classes or
8(b). seminars for women and children activities and
to avoid the use of these venues to market
SECTION 8. Health Workers. - their brands or company names.

(b) Information provided by manufacturers and SECTION 16. All health and nutrition claims for
distributors to health professionals regarding products within the scope of the Code are
products within the scope of this Code shall be absolutely prohibited. For this purpose, any
restricted to scientific and factual matters and phrase or words that connotes to increase
such information shall not imply or create a emotional, intellectual abilities of the infant and
belief that bottle-feeding is equivalent or young child and other like phrases shall not be
superior to breastfeeding. It shall also include allowed.
the information specified in Section 5(b).

6. The Milk Code permits milk manufacturers and distributors to extend assistance in
research and continuing education of health professionals; RIRR absolutely forbids the
same.

MILK CODE RIRR


SECTION 8. Health Workers – Section 4. Declaration of Principles –

(e) Manufacturers and distributors of products The following are the underlying principles from
within the scope of this Code may assist in the which the revised rules and regulations are
research, scholarships and continuing premised upon:
education, of health professionals, in
accordance with the rules and regulations i. Milk companies, and their
promulgated by the Ministry of Health. representatives, should not form part of any
policymaking body or entity in relation to the
advancement of breasfeeding.

SECTION 22. No manufacturer, distributor, or


representatives of products covered by the
Code shall be allowed to conduct or be
involved in any activity on breastfeeding
promotion, education and production of
Information, Education and Communication
(IEC) materials on breastfeeding, holding of or
participating as speakers in classes or
seminars for women and children activitiesand
to avoid the use of these venues to market
their brands or company names.

SECTION 32. Primary Responsibility of


Health Workers - It is the primary
responsibility of the health workers to promote,
protect and support breastfeeding and
appropriate infant and young child feeding.
Part of this responsibility is to continuously
update their knowledge and skills on
breastfeeding. No assistance, support, logistics
or training from milk companies shall be
permitted.

7. The Milk Code regulates the giving of donations; RIRR absolutely prohibits it.

MILK CODE RIRR


SECTION 6. The General Public and Section 51. Donations Within the Scope of
Mothers. – This Code - Donations of products, materials,
defined and covered under the Milk Code and
(f) Nothing herein contained shall prevent these implementing rules and regulations, shall
donations from manufacturers and distributors be strictly prohibited.
of products within the scope of this Code upon
request by or with the approval of the Ministry Section 52. Other Donations By Milk
of Health. Companies Not Covered by this Code. -
Donations of products, equipments, and the
like, not otherwise falling within the scope of
this Code or these Rules, given by milk
companies and their agents, representatives,
whether in kind or in cash, may only be
coursed through the Inter Agency Committee
(IAC), which shall determine whether such
donation be accepted or otherwise.

8. The RIRR provides for administrative sanctions not imposed by the Milk Code.

MILK CODE RIRR


  Section 46. Administrative Sanctions. – The
following administrative sanctions shall be
imposed upon any person, juridical or natural,
found to have violated the provisions of the
Code and its implementing Rules and
Regulations:

a) 1st violation – Warning;

b) 2nd violation – Administrative fine of a


minimum of Ten Thousand (P10,000.00) to
Fifty Thousand (P50,000.00) Pesos, depending
on the gravity and extent of the violation,
including the recall of the offending product;

c) 3rd violation – Administrative Fine of a


minimum of Sixty Thousand (P60,000.00) to
One Hundred Fifty Thousand (P150,000.00)
Pesos, depending on the gravity and extent of
the violation, and in addition thereto, the recall
of the offending product, and suspension of the
Certificate of Product Registration (CPR);

d) 4th violation –Administrative Fine of a


minimum of Two Hundred Thousand
(P200,000.00) to Five Hundred (P500,000.00)
Thousand Pesos, depending on the gravity and
extent of the violation; and in addition thereto,
the recall of the product, revocation of the
CPR, suspension of the License to Operate
(LTO) for one year;

e) 5th and succeeding repeated violations –


Administrative Fine of One Million
(P1,000,000.00) Pesos, the recall of the
offending product, cancellation of the CPR,
revocation of the License to Operate (LTO) of
the company concerned, including the
blacklisting of the company to be furnished the
Department of Budget and Management
(DBM) and the Department of Trade and
Industry (DTI);

f) An additional penalty of Two Thou-sand Five


Hundred (P2,500.00) Pesos per day shall be
made for every day the violation continues
after having received the order from the IAC or
other such appropriate body, notifying and
penalizing the company for the infraction.

For purposes of determining whether or not


there is "repeated" violation, each product
violation belonging or owned by a company,
including those of their subsidiaries, are
deemed to be violations of the concerned milk
company and shall not be based on the
specific violating product alone.

9. The RIRR provides for repeal of existing laws to the contrary.

The Court shall resolve the merits of the allegations of petitioner seriatim.

1. Petitioner is mistaken in its claim that the Milk Code's coverage is limited only to children 0-12
months old. Section 3 of the Milk Code states:

SECTION 3. Scope of the Code – The Code applies to the marketing, and practices related
thereto, of the following products: breastmilk substitutes, including infant formula; other milk
products, foods and beverages, including bottle-fed complementary foods, when marketed or
otherwise represented to be suitable, with or without modification, for use as a partial or total
replacement of breastmilk; feeding bottles and teats. It also applies to their quality and
availability, and to information concerning their use.

Clearly, the coverage of the Milk Code is not dependent on the age of the child but on the kind of
product being marketed to the public. The law treats infant formula, bottle-fed complementary food,
and breastmilk substitute as separate and distinct product categories.

Section 4(h) of the Milk Code defines infant formula as "a breastmilk substitute x x x to satisfy the
normal nutritional requirements of infants up to between four to six months of age, and adapted to
their physiological characteristics"; while under Section 4(b), bottle-fed complementary food refers to
"any food, whether manufactured or locally prepared, suitable as a complement to breastmilk or
infant formula, when either becomes insufficient to satisfy the nutritional requirements of the infant."
An infant under Section 4(e) is a person falling within the age bracket 0-12 months. It is the
nourishment of this group of infants or children aged 0-12 months that is sought to be promoted and
protected by the Milk Code.

But there is another target group. Breastmilk substitute is defined under Section 4(a) as "any food
being marketed or otherwise presented as a partial or total replacement for breastmilk, whether or
not suitable for that purpose." This section conspicuously lacks reference to any particular age-
group of children. Hence, the provision of the Milk Code cannot be considered exclusive for
children aged 0-12 months. In other words, breastmilk substitutes may also be intended for young
children more than 12 months of age. Therefore, by regulating breastmilk substitutes, the Milk Code
also intends to protect and promote the nourishment of children more than 12 months old.

Evidently, as long as what is being marketed falls within the scope of the Milk Code as provided in
Section 3, then it can be subject to regulation pursuant to said law, even if the product is to be used
by children aged over 12 months.

There is, therefore, nothing objectionable with Sections 242 and 5(ff)43 of the RIRR.

2. It is also incorrect for petitioner to say that the RIRR, unlike the Milk Code, does not recognize that
breastmilk substitutes may be a proper and possible substitute for breastmilk.
The entirety of the RIRR, not merely truncated portions thereof, must be considered and construed
together. As held in De Luna v. Pascual,44 "[t]he particular words, clauses and phrases in the Rule
should not be studied as detached and isolated expressions, but the whole and every part thereof
must be considered in fixing the meaning of any of its parts and in order to produce a harmonious
whole."

Section 7 of the RIRR provides that "when medically indicated and only when necessary, the use of
breastmilk substitutes is proper if based on complete and updated information." Section 8 of the
RIRR also states that information and educational materials should include information on the proper
use of infant formula when the use thereof is needed.

Hence, the RIRR, just like the Milk Code, also recognizes that in certain cases, the use of
breastmilk substitutes may be proper.

3. The Court shall ascertain the merits of allegations 345 and 446 together as they are interlinked with
each other.

To resolve the question of whether the labeling requirements and advertising regulations under the
RIRR are valid, it is important to deal first with the nature, purpose, and depth of the regulatory
powers of the DOH, as defined in general under the 1987 Administrative Code,47 and as delegated in
particular under the Milk Code.

Health is a legitimate subject matter for regulation by the DOH (and certain other administrative
agencies) in exercise of police powers delegated to it. The sheer span of jurisprudence on that
matter precludes the need to further discuss it..48 However, health information, particularly advertising
materials on apparently non-toxic products like breastmilk substitutes and supplements, is a
relatively new area for regulation by the DOH.49

As early as the 1917 Revised Administrative Code of the Philippine Islands, 50 health information was
already within the ambit of the regulatory powers of the predecessor of DOH.51 Section 938 thereof
charged it with the duty to protect the health of the people, and vested it with such powers as "(g) the
dissemination of hygienic information among the people and especially the inculcation of
knowledge as to the proper care of infants and the methods of preventing and combating
dangerous communicable diseases."

Seventy years later, the 1987 Administrative Code tasked respondent DOH to carry out the state
policy pronounced under Section 15, Article II of the 1987 Constitution, which is "to protect and
promote the right to health of the people and instill health consciousness among them."52 To that
end, it was granted under Section 3 of the Administrative Code the power to "(6) propagate health
information and educate the population on important health, medical and environmental matters
which have health implications."53

When it comes to information regarding nutrition of infants and young children, however, the Milk
Code specifically delegated to the Ministry of Health (hereinafter referred to as DOH) the power to
ensure that there is adequate, consistent and objective information on breastfeeding and use of
breastmilk substitutes, supplements and related products; and the power to control such
information. These are expressly provided for in Sections 12 and 5(a), to wit:

SECTION 12. Implementation and Monitoring –

xxxx
(b) The Ministry of Health shall be principally responsible for the implementation and
enforcement of the provisions of this Code. For this purpose, the Ministry of Health shall
have the following powers and functions:

(1) To promulgate such rules and regulations as are necessary or proper for the
implementation of this Code and the accomplishment of its purposes and objectives.

xxxx

(4) To exercise such other powers and functions as may be necessary for or
incidental to the attainment of the purposes and objectives of this Code.

SECTION 5. Information and Education –

(a) The government shall ensure that objective and consistent information is provided on


infant feeding, for use by families and those involved in the field of infant nutrition. This
responsibility shall cover the planning, provision, design and dissemination of information,
and the control thereof, on infant nutrition. (Emphasis supplied)

Further, DOH is authorized by the Milk Code to control the content of any information on
breastmilk vis-à-visbreastmilk substitutes, supplement and related products, in the following manner:

SECTION 5. x x x

(b) Informational and educational materials, whether written, audio, or visual, dealing with the
feeding of infants and intended to reach pregnant women and mothers of infants, shall
include clear information on all the following points: (1) the benefits and superiority of
breastfeeding; (2) maternal nutrition, and the preparation for and maintenance of
breastfeeding; (3) the negative effect on breastfeeding of introducing partial bottlefeeding; (4)
the difficulty of reversing the decision not to breastfeed; and (5) where needed, the proper
use of infant formula, whether manufactured industrially or home-prepared. When such
materials contain information about the use of infant formula, they shall include the
social and financial implications of its use; the health hazards of inappropriate foods
or feeding methods; and, in particular, the health hazards of unnecessary or improper
use of infant formula and other breastmilk substitutes. Such materials shall not use
any picture or text which may idealize the use of breastmilk substitutes.

SECTION 8. Health Workers –

xxxx

(b) Information provided by manufacturers and distributors to health professionals regarding


products within the scope of this Code shall be restricted to scientific and factual
matters, and such information shall not imply or create a belief that bottlefeeding is
equivalent or superior to breastfeeding. It shall also include the information specified
in Section 5(b).

SECTION 10. Containers/Label –
(a) Containers and/or labels shall be designed to provide the necessary information about
the appropriate use of the products, and in such a way as not to discourage
breastfeeding.

xxxx

(d) The term "humanized," "maternalized" or similar terms shall not be used. (Emphasis
supplied)

The DOH is also authorized to control the purpose of the information and to whom such information
may be disseminated under Sections 6 through 9 of the Milk Code 54 to ensure that the information
that would reach pregnant women, mothers of infants, and health professionals and workers in the
health care system is restricted to scientific and factual matters and shall not imply or create a belief
that bottlefeeding is equivalent or superior to breastfeeding.

It bears emphasis, however, that the DOH's power under the Milk Code to control information
regarding breastmilk vis-a-vis breastmilk substitutes is not absolute as the power to control does
not encompass the power to absolutely prohibit the advertising, marketing, and promotion of
breastmilk substitutes.

The following are the provisions of the Milk Code that unequivocally indicate that the control over
information given to the DOH is not absolute and that absolute prohibition is not contemplated by the
Code:

a) Section 2 which requires adequate information and appropriate marketing and distribution
of breastmilk substitutes, to wit:

SECTION 2. Aim of the Code – The aim of the Code is to contribute to the provision
of safe and adequate nutrition for infants by the protection and promotion of
breastfeeding and by ensuring the proper use of breastmilk substitutes and
breastmilk supplements when these are necessary, on the basis of adequate
information and through appropriate marketing and distribution.

b) Section 3 which specifically states that the Code applies to the marketing of and practices
related to breastmilk substitutes, including infant formula, and to information concerning their
use;

c) Section 5(a) which provides that the government shall ensure that objective and consistent
information is provided on infant feeding;

d) Section 5(b) which provides that written, audio or visual informational and educational
materials shall not use any picture or text which may idealize the use of breastmilk
substitutes and should include information on the health hazards of unnecessary or improper
use of said product;

e) Section 6(a) in relation to Section 12(a) which creates and empowers the IAC to review
and examine advertising, promotion, and other marketing materials;

f) Section 8(b) which states that milk companies may provide information to health
professionals but such information should be restricted to factual and scientific matters and
shall not imply or create a belief that bottlefeeding is equivalent or superior to breastfeeding;
and

g) Section 10 which provides that containers or labels should not contain information that
would discourage breastfeeding and idealize the use of infant formula.

It is in this context that the Court now examines the assailed provisions of the RIRR regarding
labeling and advertising.

Sections 1355 on "total effect" and 2656 of Rule VII of the RIRR contain some labeling requirements,
specifically: a) that there be a statement that there is no substitute to breastmilk; and b) that there be
a statement that powdered infant formula may contain pathogenic microorganisms and must be
prepared and used appropriately. Section 1657of the RIRR prohibits all health and nutrition claims for
products within the scope of the Milk Code, such as claims of increased emotional and intellectual
abilities of the infant and young child.

These requirements and limitations are consistent with the provisions of Section 8 of the Milk Code,
to wit:

SECTION 8. Health workers -

xxxx

(b) Information provided by manufacturers and distributors to health professionals regarding


products within the scope of this Code shall be restricted to scientific and factual matters,
and such information shall notimply or create a belief that bottlefeeding
is equivalent or superior to breastfeeding. It shall also include the information specified in
Section 5.58 (Emphasis supplied)

and Section 10(d)59 which bars the use on containers and labels of the terms "humanized,"
"maternalized," or similar terms.

These provisions of the Milk Code expressly forbid information that would imply or create a belief
that there is any milk product equivalent to breastmilk or which is humanized or maternalized, as
such information would be inconsistent with the superiority of breastfeeding.

It may be argued that Section 8 of the Milk Code refers only to information given to health workers
regarding breastmilk substitutes, not to containers and labels thereof. However, such restrictive
application of Section 8(b) will result in the absurd situation in which milk companies and distributors
are forbidden to claim to health workers that their products are substitutes or equivalents of
breastmilk, and yet be allowed to display on the containers and labels of their products the exact
opposite message. That askewed interpretation of the Milk Code is precisely what Section 5(a)
thereof seeks to avoid by mandating that all information regarding breastmilk vis-a-vis breastmilk
substitutes be consistent, at the same time giving the government control over planning, provision,
design, and dissemination of information on infant feeding.

Thus, Section 26(c) of the RIRR which requires containers and labels to state that the product
offered is not a substitute for breastmilk, is a reasonable means of enforcing Section 8(b) of the Milk
Code and deterring circumvention of the protection and promotion of breastfeeding as embodied in
Section 260 of the Milk Code.
Section 26(f)61 of the RIRR is an equally reasonable labeling requirement. It implements Section 5(b)
of the Milk Code which reads:

SECTION 5. x x x

xxxx

(b) Informational and educational materials, whether written, audio, or visual, dealing with the
feeding of infants and intended to reach pregnant women and mothers of infants, shall
include clear information on all the following points: x x x (5) where needed, the proper use
of infant formula, whether manufactured industrially or home-prepared. When such materials
contain information about the use of infant formula, they shall include the social and financial
implications of its use; the health hazards of inappropriate foods or feeding methods;
and, in particular, the health hazards of unnecessary or improper use of infant
formula and other breastmilk substitutes. Such materials shall not use any picture or text
which may idealize the use of breastmilk substitutes. (Emphasis supplied)

The label of a product contains information about said product intended for the buyers thereof. The
buyers of breastmilk substitutes are mothers of infants, and Section 26 of the RIRR merely adds a
fair warning about the likelihood of pathogenic microorganisms being present in infant formula and
other related products when these are prepared and used inappropriately.

Petitioner’s counsel has admitted during the hearing on June 19, 2007 that formula milk is prone to
contaminations and there is as yet no technology that allows production of powdered infant formula
that eliminates all forms of contamination.62

Ineluctably, the requirement under Section 26(f) of the RIRR for the label to contain the message
regarding health hazards including the possibility of contamination with pathogenic microorganisms
is in accordance with Section 5(b) of the Milk Code.

The authority of DOH to control information regarding breastmilk vis-a-vis breastmilk substitutes and


supplements and related products cannot be questioned. It is its intervention into the area of
advertising, promotion, and marketing that is being assailed by petitioner.

In furtherance of Section 6(a) of the Milk Code, to wit:

SECTION 6. The General Public and Mothers. –

(a) No advertising, promotion or other marketing materials, whether written, audio or visual,
for products within the scope of this Code shall be printed, published, distributed, exhibited
and broadcast unless such materials are duly authorized and approved by an inter-agency
committee created herein pursuant to the applicable standards provided for in this Code.

the Milk Code invested regulatory authority over advertising, promotional and marketing materials to
an IAC, thus:

SECTION 12. Implementation and Monitoring -

(a) For purposes of Section 6(a) of this Code, an inter-agency committee composed of the
following members is hereby created:
Minister of Health ------------------- Chairman

Minister of Trade and Industry ------------------- Member

Minister of Justice ------------------- Member

Minister of Social Services and Development ------------------- Member

The members may designate their duly authorized representative to every meeting of the
Committee.

The Committee shall have the following powers and functions:

(1) To review and examine all advertising. promotion or other marketing materials,
whether written, audio or visual, on products within the scope of this Code;

(2) To approve or disapprove, delete objectionable portions from and prohibit the
printing, publication, distribution, exhibition and broadcast of, all advertising
promotion or other marketing materials, whether written, audio or visual, on products
within the scope of this Code;

(3) To prescribe the internal and operational procedure for the exercise of its powers
and functions as well as the performance of its duties and responsibilities; and

(4) To promulgate such rules and regulations as are necessary or proper for
the implementation of Section 6(a) of this Code. x x x (Emphasis supplied)

However, Section 11 of the RIRR, to wit:

SECTION 11. Prohibition – No advertising, promotions, sponsorships, or marketing materials


and activities for breastmilk substitutes intended for infants and young children up to twenty-
four (24) months, shall be allowed, because they tend to convey or give subliminal messages
or impressions that undermine breastmilk and breastfeeding or otherwise exaggerate
breastmilk substitutes and/or replacements, as well as related products covered within the
scope of this Code.

prohibits advertising, promotions, sponsorships or marketing materials and activities for breastmilk
substitutes in line with the RIRR’s declaration of principle under Section 4(f), to wit:

SECTION 4. Declaration of Principles –

xxxx

(f) Advertising, promotions, or sponsorships of infant formula, breastmilk substitutes and


other related products are prohibited.

The DOH, through its co-respondents, evidently arrogated to itself not only the regulatory authority
given to the IAC but also imposed absolute prohibition on advertising, promotion, and marketing.
Yet, oddly enough, Section 12 of the RIRR reiterated the requirement of the Milk Code in Section 6
thereof for prior approval by IAC of all advertising, marketing and promotional materials prior to
dissemination.

Even respondents, through the OSG, acknowledged the authority of IAC, and repeatedly insisted,
during the oral arguments on June 19, 2007, that the prohibition under Section 11 is not actually
operational, viz:

SOLICITOR GENERAL DEVANADERA:

xxxx

x x x Now, the crux of the matter that is being questioned by Petitioner is whether or not
there is an absolute prohibition on advertising making AO 2006-12 unconstitutional. We
maintained that what AO 2006-12 provides is not an absolute prohibition because Section 11
while it states and it is entitled prohibition it states that no advertising, promotion,
sponsorship or marketing materials and activities for breast milk substitutes intended for
infants and young children up to 24 months shall be allowed because this is the standard
they tend to convey or give subliminal messages or impression undermine that breastmilk or
breastfeeding x x x.

We have to read Section 11 together with the other Sections because the other Section,
Section 12, provides for the inter agency committee that is empowered to process and
evaluate all the advertising and promotion materials.

xxxx

What AO 2006-12, what it does, it does not prohibit the sale and manufacture, it simply
regulates the advertisement and the promotions of breastfeeding milk substitutes.

xxxx

Now, the prohibition on advertising, Your Honor, must be taken together with the provision
on the Inter-Agency Committee that processes and evaluates because there may be some
information dissemination that are straight forward information dissemination. What the AO
2006 is trying to prevent is any material that will undermine the practice of breastfeeding,
Your Honor.

xxxx

ASSOCIATE JUSTICE SANTIAGO:

Madam Solicitor General, under the Milk Code, which body has authority or power to
promulgate Rules and Regulations regarding the Advertising, Promotion and Marketing of
Breastmilk Substitutes?

SOLICITOR GENERAL DEVANADERA:

Your Honor, please, it is provided that the Inter-Agency Committee, Your Honor.

xxxx
ASSOCIATE JUSTICE SANTIAGO:

x x x Don't you think that the Department of Health overstepped its rule making authority
when it totally banned advertising and promotion under Section 11 prescribed the total effect
rule as well as the content of materials under Section 13 and 15 of the rules and regulations?

SOLICITOR GENERAL DEVANADERA:

Your Honor, please, first we would like to stress that there is no total absolute ban. Second,
the Inter-Agency Committee is under the Department of Health, Your Honor.

xxxx

ASSOCIATE JUSTICE NAZARIO:

x x x Did I hear you correctly, Madam Solicitor, that there is no absolute ban on advertising of
breastmilk substitutes in the Revised Rules?

SOLICITOR GENERAL DEVANADERA:

Yes, your Honor.

ASSOCIATE JUSTICE NAZARIO:

But, would you nevertheless agree that there is an absolute ban on advertising of breastmilk
substitutes intended for children two (2) years old and younger?

SOLICITOR GENERAL DEVANADERA:

It's not an absolute ban, Your Honor, because we have the Inter-Agency Committee that can
evaluate some advertising and promotional materials, subject to the standards that we have
stated earlier, which are- they should not undermine breastfeeding, Your Honor.

xxxx

x x x Section 11, while it is titled Prohibition, it must be taken in relation with the other
Sections, particularly 12 and 13 and 15, Your Honor, because it is recognized that the Inter-
Agency Committee has that power to evaluate promotional materials, Your Honor.

ASSOCIATE JUSTICE NAZARIO:

So in short, will you please clarify there's no absolute ban on advertisement regarding milk
substitute regarding infants two (2) years below?

SOLICITOR GENERAL DEVANADERA:

We can proudly say that the general rule is that there is a prohibition, however, we take
exceptions and standards have been set. One of which is that, the Inter-Agency Committee
can allow if the advertising and promotions will not undermine breastmilk and breastfeeding,
Your Honor.63
Sections 11 and 4(f) of the RIRR are clearly violative of the Milk Code.

However, although it is the IAC which is authorized to promulgate rules and regulations for the
approval or rejection of advertising, promotional, or other marketing materials under Section 12(a) of
the Milk Code, said provision must be related to Section 6 thereof which in turn provides that the
rules and regulations must be "pursuant to the applicable standards provided for in this Code." Said
standards are set forth in Sections 5(b), 8(b), and 10 of the Code, which, at the risk of being
repetitious, and for easy reference, are quoted hereunder:

SECTION 5. Information and Education –

xxxx

(b) Informational and educational materials, whether written, audio, or visual, dealing with the
feeding of infants and intended to reach pregnant women and mothers of infants, shall
include clear information on all the following points: (1) the benefits and superiority of
breastfeeding; (2) maternal nutrition, and the preparation for and maintenance of
breastfeeding; (3) the negative effect on breastfeeding of introducing partial bottlefeeding; (4)
the difficulty of reversing the decision not to breastfeed; and (5) where needed, the proper
use of infant formula, whether manufactured industrially or home-prepared. When such
materials contain information about the use of infant formula, they shall include the social
and financial implications of its use; the health hazards of inappropriate foods of feeding
methods; and, in particular, the health hazards of unnecessary or improper use of infant
formula and other breastmilk substitutes. Such materials shall not use any picture or text
which may idealize the use of breastmilk substitutes.

xxxx

SECTION 8. Health Workers. –

xxxx

(b) Information provided by manufacturers and distributors to health professionals regarding


products within the scope of this Code shall be restricted to scientific and factual matters and
such information shall not imply or create a belief that bottle feeding is equivalent or superior
to breastfeeding. It shall also include the information specified in Section 5(b).

xxxx

SECTION 10. Containers/Label –

(a) Containers and/or labels shall be designed to provide the necessary information about
the appropriate use of the products, and in such a way as not to discourage breastfeeding.

(b) Each container shall have a clear, conspicuous and easily readable and understandable
message in Pilipino or English printed on it, or on a label, which message can not readily
become separated from it, and which shall include the following points:

(i) the words "Important Notice" or their equivalent;

(ii) a statement of the superiority of breastfeeding;


(iii) a statement that the product shall be used only on the advice of a health worker
as to the need for its use and the proper methods of use; and

(iv) instructions for appropriate preparation, and a warning against the health hazards
of inappropriate preparation.

Section 12(b) of the Milk Code designates the DOH as the principal implementing agency for the
enforcement of the provisions of the Code. In relation to such responsibility of the DOH, Section 5(a)
of the Milk Code states that:

SECTION 5. Information and Education –

(a) The government shall ensure that objective and consistent information is provided on


infant feeding, for use by families and those involved in the field of infant nutrition. This
responsibility shall cover the planning, provision, design and dissemination of information,
and the control thereof, on infant nutrition. (Emphasis supplied)

Thus, the DOH has the significant responsibility to translate into operational terms the
standards set forth in Sections 5, 8, and 10 of the Milk Code, by which the IAC shall screen
advertising, promotional, or other marketing materials.

It is pursuant to such responsibility that the DOH correctly provided for Section 13 in the RIRR which
reads as follows:

SECTION 13. "Total Effect" - Promotion of products within the scope of this Code must be
objective and should not equate or make the product appear to be as good or equal to
breastmilk or breastfeeding in the advertising concept. It must not in any case undermine
breastmilk or breastfeeding. The "total effect" should not directly or indirectly suggest that
buying their product would produce better individuals, or resulting in greater love,
intelligence, ability, harmony or in any manner bring better health to the baby or other such
exaggerated and unsubstantiated claim.

Such standards bind the IAC in formulating its rules and regulations on advertising, promotion, and
marketing. Through that single provision, the DOH exercises control over the information content of
advertising, promotional and marketing materials on breastmilk vis-a-vis breastmilk substitutes,
supplements and other related products. It also sets a viable standard against which the IAC may
screen such materials before they are made public.

In Equi-Asia Placement, Inc. vs. Department of Foreign Affairs,64 the Court held:

x x x [T]his Court had, in the past, accepted as sufficient standards the following: "public
interest," "justice and equity," "public convenience and welfare," and "simplicity, economy
and welfare."65

In this case, correct information as to infant feeding and nutrition is infused with public interest and
welfare.

4. With regard to activities for dissemination of information to health professionals, the Court also
finds that there is no inconsistency between the provisions of the Milk Code and the RIRR. Section
7(b)66 of the Milk Code, in relation to Section 8(b)67 of the same Code, allows dissemination of
information to health professionals but such information is restricted to scientific and factual
matters.

Contrary to petitioner's claim, Section 22 of the RIRR does not prohibit the giving of information to
health professionals on scientific and factual matters. What it prohibits is the involvement of the
manufacturer and distributor of the products covered by the Code in activities for the promotion,
education and production of Information, Education and Communication (IEC) materials regarding
breastfeeding that are intended for women and children. Said provision cannot be construed to
encompass even the dissemination of information to health professionals, as restricted by the
Milk Code.

5. Next, petitioner alleges that Section 8(e)68 of the Milk Code permits milk manufacturers and
distributors to extend assistance in research and in the continuing education of health professionals,
while Sections 22 and 32 of the RIRR absolutely forbid the same. Petitioner also assails Section
4(i)69 of the RIRR prohibiting milk manufacturers' and distributors' participation in any policymaking
body in relation to the advancement of breastfeeding.

Section 4(i) of the RIRR provides that milk companies and their representatives should not form part
of any policymaking body or entity in relation to the advancement of breastfeeding. The Court finds
nothing in said provisions which contravenes the Milk Code. Note that under Section 12(b) of the
Milk Code, it is the DOH which shall be principally responsible for the implementation and
enforcement of the provisions of said Code. It is entirely up to the DOH to decide which entities to
call upon or allow to be part of policymaking bodies on breastfeeding. Therefore, the RIRR's
prohibition on milk companies’ participation in any policymaking body in relation to the advancement
of breastfeeding is in accord with the Milk Code.

Petitioner is also mistaken in arguing that Section 22 of the RIRR prohibits milk companies from
giving reasearch assistance and continuing education to health professionals. Section 2270 of the
RIRR does not pertain to research assistance to or the continuing education of health
professionals; rather, it deals with breastfeeding promotion and education for women and
children. Nothing in Section 22 of the RIRR prohibits milk companies from giving assistance for
research or continuing education to health professionals; hence, petitioner's argument against this
particular provision must be struck down.

It is Sections 971 and 1072 of the RIRR which govern research assistance. Said sections of the RIRR
provide that research assistance for health workers and researchers may be allowed upon
approval of an ethics committee, and with certain disclosure requirements imposed on the
milk company and on the recipient of the research award.

The Milk Code endows the DOH with the power to determine how such research or educational
assistance may be given by milk companies or under what conditions health workers may accept the
assistance. Thus, Sections 9 and 10 of the RIRR imposing limitations on the kind of research done
or extent of assistance given by milk companies are completely in accord with the Milk Code.

Petitioner complains that Section 3273 of the RIRR prohibits milk companies from giving assistance,
support, logistics or training to health workers. This provision is within the prerogative given to the
DOH under Section 8(e)74of the Milk Code, which provides that manufacturers and distributors of
breastmilk substitutes may assist in researches, scholarships and the continuing education, of health
professionals in accordance with the rules and regulations promulgated by the Ministry of Health,
now DOH.
6. As to the RIRR's prohibition on donations, said provisions are also consistent with the Milk Code.
Section 6(f) of the Milk Code provides that donations may be made by manufacturers and
distributors of breastmilk substitutes upon the request or with the approval of the DOH. The law
does not proscribe the refusal of donations. The Milk Code leaves it purely to the discretion of the
DOH whether to request or accept such donations. The DOH then appropriately exercised its
discretion through Section 5175 of the RIRR which sets forth its policy not to request or approve
donations from manufacturers and distributors of breastmilk substitutes.

It was within the discretion of the DOH when it provided in Section 52 of the RIRR that any donation
from milk companies not covered by the Code should be coursed through the IAC which shall
determine whether such donation should be accepted or refused. As reasoned out by respondents,
the DOH is not mandated by the Milk Code to accept donations. For that matter, no person or entity
can be forced to accept a donation. There is, therefore, no real inconsistency between the RIRR and
the law because the Milk Code does not prohibit the DOH from refusing donations.

7. With regard to Section 46 of the RIRR providing for administrative sanctions that are not found in
the Milk Code, the Court upholds petitioner's objection thereto.

Respondent's reliance on Civil Aeronautics Board v. Philippine Air Lines, Inc. 76 is misplaced. The
glaring difference in said case and the present case before the Court is that, in the Civil Aeronautics
Board, the Civil Aeronautics Administration (CAA) was expressly granted by the law (R.A. No.
776) the power to impose fines and civil penalties, while the Civil Aeronautics Board (CAB) was
granted by the same law the power to review on appeal the order or decision of the CAA and to
determine whether to impose, remit, mitigate, increase or compromise such fine and civil penalties.
Thus, the Court upheld the CAB's Resolution imposing administrative fines.

In a more recent case, Perez v. LPG Refillers Association of the Philippines, Inc.,77 the Court upheld
the Department of Energy (DOE) Circular No. 2000-06-10
implementing Batas Pambansa (B.P.) Blg. 33. The circular provided for fines for the commission of
prohibited acts. The Court found that nothing in the circular contravened the law because the DOE
was expressly authorized by B.P. Blg. 33 and R.A. No. 7638 to impose fines or penalties.

In the present case, neither the Milk Code nor the Revised Administrative Code grants the DOH the
authority to fix or impose administrative fines. Thus, without any express grant of power to fix or
impose such fines, the DOH cannot provide for those fines in the RIRR. In this regard, the DOH
again exceeded its authority by providing for such fines or sanctions in Section 46 of the RIRR. Said
provision is, therefore, null and void.

The DOH is not left without any means to enforce its rules and regulations. Section 12(b) (3) of the
Milk Code authorizes the DOH to "cause the prosecution of the violators of this Code and other
pertinent laws on products covered by this Code." Section 13 of the Milk Code provides for the
penalties to be imposed on violators of the provision of the Milk Code or the rules and regulations
issued pursuant to it, to wit:

SECTION 13. Sanctions –

(a) Any person who violates the provisions of this Code or the rules and regulations
issued pursuant to this Code shall, upon conviction, be punished by a penalty of two (2)
months to one (1) year imprisonment or a fine of not less than One Thousand Pesos
(P1,000.00) nor more than Thirty Thousand Pesos (P30,000.00) or both. Should the offense
be committed by a juridical person, the chairman of the Board of Directors, the president,
general manager, or the partners and/or the persons directly responsible therefor, shall be
penalized.

(b) Any license, permit or authority issued by any government agency to any health worker,
distributor, manufacturer, or marketing firm or personnel for the practice of their profession or
occupation, or for the pursuit of their business, may, upon recommendation of the Ministry of
Health, be suspended or revoked in the event of repeated violations of this Code, or of the
rules and regulations issued pursuant to this Code. (Emphasis supplied)

8. Petitioner’s claim that Section 57 of the RIRR repeals existing laws that are contrary to the RIRR
is frivolous.

Section 57 reads:

SECTION 57. Repealing Clause - All orders, issuances, and rules and regulations or parts
thereof inconsistent with these revised rules and implementing regulations are hereby
repealed or modified accordingly.

Section 57 of the RIRR does not provide for the repeal of laws but only orders, issuances and rules
and regulations. Thus, said provision is valid as it is within the DOH's rule-making power.

An administrative agency like respondent possesses quasi-legislative or rule-making power or the


power to make rules and regulations which results in delegated legislation that is within the confines
of the granting statute and the Constitution, and subject to the doctrine of non-delegability and
separability of powers.78 Such express grant of rule-making power necessarily includes the power to
amend, revise, alter, or repeal the same.79 This is to allow administrative agencies flexibility in
formulating and adjusting the details and manner by which they are to implement the provisions of a
law,80 in order to make it more responsive to the times. Hence, it is a standard provision in
administrative rules that prior issuances of administrative agencies that are inconsistent therewith
are declared repealed or modified.

In fine, only Sections 4(f), 11 and 46 are ultra vires, beyond the authority of the DOH to promulgate
and in contravention of the Milk Code and, therefore, null and void. The rest of the provisions of the
RIRR are in consonance with the Milk Code.

Lastly, petitioner makes a "catch-all" allegation that:

x x x [T]he questioned RIRR sought to be implemented by the Respondents is unnecessary


and oppressive, and is offensive to the due process clause of the Constitution, insofar
as the same is in restraint of trade and because a provision therein is inadequate to
provide the public with a comprehensible basis to determine whether or not they have
committed a violation.81 (Emphasis supplied)

Petitioner refers to Sections 4(f),82 4(i),83 5(w),84 11,85 22,86 32,87 46,88 and 5289 as the provisions that


suppress the trade of milk and, thus, violate the due process clause of the Constitution.

The framers of the constitution were well aware that trade must be subjected to some form of
regulation for the public good. Public interest must be upheld over business interests. 90 In Pest
Management Association of the Philippines v. Fertilizer and Pesticide Authority,91 it was held thus:
x x x Furthermore, as held in Association of Philippine Coconut Desiccators v. Philippine
Coconut Authority, despite the fact that "our present Constitution enshrines free
enterprise as a policy, it nonetheless reserves to the government the power to
intervene whenever necessary to promote the general welfare." There can be no
question that the unregulated use or proliferation of pesticides would be hazardous to our
environment. Thus, in the aforecited case, the Court declared that "free enterprise does not
call for removal of ‘protective regulations’." x x x It must be clearly explained and
proven by competent evidence just exactly how such protective regulation would
result in the restraint of trade. [Emphasis and underscoring supplied]

In this case, petitioner failed to show that the proscription of milk manufacturers’ participation in any
policymaking body (Section 4(i)), classes and seminars for women and children (Section 22); the
giving of assistance, support and logistics or training (Section 32); and the giving of donations
(Section 52) would unreasonably hamper the trade of breastmilk substitutes. Petitioner has not
established that the proscribed activities are indispensable to the trade of breastmilk substitutes.
Petitioner failed to demonstrate that the aforementioned provisions of the RIRR are unreasonable
and oppressive for being in restraint of trade.

Petitioner also failed to convince the Court that Section 5(w) of the RIRR is unreasonable and
oppressive. Said section provides for the definition of the term "milk company," to wit:

SECTION 5 x x x. (w) "Milk Company" shall refer to the owner, manufacturer, distributor of
infant formula, follow-up milk, milk formula, milk supplement, breastmilk substitute or
replacement, or by any other description of such nature, including their representatives who
promote or otherwise advance their commercial interests in marketing those products;

On the other hand, Section 4 of the Milk Code provides:

(d) "Distributor" means a person, corporation or any other entity in the public or private sector
engaged in the business (whether directly or indirectly) of marketing at the wholesale or retail
level a product within the scope of this Code. A "primary distributor" is a manufacturer's sales
agent, representative, national distributor or broker.

xxxx

(j) "Manufacturer" means a corporation or other entity in the public or private sector engaged
in the business or function (whether directly or indirectly or through an agent or and entity
controlled by or under contract with it) of manufacturing a products within the scope of this
Code.

Notably, the definition in the RIRR merely merged together under the term "milk company" the
entities defined separately under the Milk Code as "distributor" and "manufacturer." The RIRR also
enumerated in Section 5(w) the products manufactured or distributed by an entity that would qualify
it as a "milk company," whereas in the Milk Code, what is used is the phrase "products within the
scope of this Code." Those are the only differences between the definitions given in the Milk Code
and the definition as re-stated in the RIRR.

Since all the regulatory provisions under the Milk Code apply equally to both manufacturers and
distributors, the Court sees no harm in the RIRR providing for just one term to encompass both
entities. The definition of "milk company" in the RIRR and the definitions of "distributor" and
"manufacturer" provided for under the Milk Code are practically the same.
The Court is not convinced that the definition of "milk company" provided in the RIRR would bring
about any change in the treatment or regulation of "distributors" and "manufacturers" of breastmilk
substitutes, as defined under the Milk Code.

Except Sections 4(f), 11 and 46, the rest of the provisions of the RIRR are in consonance with the
objective, purpose and intent of the Milk Code, constituting reasonable regulation of an industry
which affects public health and welfare and, as such, the rest of the RIRR do not constitute illegal
restraint of trade nor are they violative of the due process clause of the Constitution.

WHEREFORE, the petition is PARTIALLY GRANTED. Sections 4(f), 11 and 46 of Administrative


Order No. 2006-0012 dated May 12, 2006 are declared NULL and VOID for being ultra vires. The
Department of Health and respondents are PROHIBITED from implementing said provisions.

The Temporary Restraining Order issued on August 15, 2006 is LIFTED insofar as the rest of the
provisions of Administrative Order No. 2006-0012 is concerned.

SO ORDERED.

G.R. No. 151445      April 11, 2002

ARTHUR D. LIM and PAULINO R. ERSANDO, petitioners, 


vs.
HONORABLE EXECUTIVE SECRETARY as alter ego of HER EXCELLENCEY GLORIA
MACAPAGAL-ARROYO, and HONORABLE ANGELO REYES in his capacity as Secretary of
National Defense, respondents.

----------------------------------------

SANLAKAS and PARTIDO NG MANGGAGAWA, petitioners-intervenors, 


vs.
GLORIA MACAPAGA-ARROYO, ALBERTO ROMULO, ANGELO REYES, respondents.

DISSENTING OPINION

SEPARATE OPINION

DE LEON, JR., J.:

This case involves a petition for certiorari and prohibition as well as a petition-in-intervention, praying
that respondents be restrained from proceeding with the so-called "Balikatan 02-1" and that after
due notice and hearing, that judgment be rendered issuing a permanent writ of injunction and/or
prohibition against the deployment of U.S. troops in Basilan and Mindanao for being illegal and in
violation of the Constitution.

The facts are as follows:


Beginning January of this year 2002, personnel from the armed forces of the United States of
America started arriving in Mindanao to take part, in conjunction with the Philippine military, in
"Balikatan 02-1." These so-called "Balikatan" exercises are the largest combined training operations
involving Filipino and American troops. In theory, they are a simulation of joint military maneuvers
pursuant to the Mutual Defense Treaty, a bilateral defense agreement entered into by the

Philippines and the United States in 1951.

Prior to the year 2002, the last "Balikatan" was held in 1995. This was due to the paucity of any
formal agreement relative to the treatment of United States personnel visiting the Philippines. In the
meantime, the respective governments of the two countries agreed to hold joint exercises on a
reduced scale. The lack of consensus was eventually cured when the two nations concluded the
Visiting Forces Agreement (V FA) in 1999.

The entry of American troops into Philippine soil is proximately rooted in the international anti-
terrorism campaign declared by President George W. Bush in reaction to the tragic events that
occurred on September 11, 2001. On that day, three (3) commercial aircrafts were hijacked, flown
and smashed into the twin towers of the World Trade Center in New York City and the Pentagon
building in Washington, D.C. by terrorists with alleged links to the al-Qaeda ("the Base"), a Muslim
extremist organization headed by the infamous Osama bin Laden. Of no comparable historical
parallels, these acts caused billions of dollars worth of destruction of property and incalculable loss
of hundreds of lives.

On February 1, 2002, petitioners Arthur D. Lim and Paulino P. Ersando filed this petition for certiorari
and prohibition, attacking the constitutionality of the joint exercise. They were joined subsequently by

SANLAKAS and PARTIDO NG MANGGAGAWA, both party-Iist organizations, who filed a petition-
in-intervention on February 11, 2002.

Lim and Ersando filed suit in their capacities as citizens, lawyers and taxpayers. SANLAKAS and
PARTIDO, on the other hand, aver that certain members of their organization are residents of
Zamboanga and Sulu, and hence will be directly affected by the operations being conducted in
Mindanao. They likewise pray for a relaxation on the rules relative to locus standi citing the
unprecedented importance of the issue involved.

On February 71 2002 the Senate conducted a hearing on the "Balikatan" exercise wherein Vice-
President Teofisto T. Guingona, Jr., who is concurrently Secretary of Foreign. Affairs, presented the
Draft Terms of Reference (TOR). Five days later, he approved the TOR, which we quote hereunder:
3

I. POLICY LEVEL

1. The Exercise shall be consistent with the Philippine Constitution and all its activities shall
be in consonance with the laws of the land and the provisions of the RP-US Visiting Forces
Agreement (VFA).

2. The conduct of this training Exercise is in accordance with pertinent United Nations
resolutions against global terrorism as understood by the respective parties.

3. No permanent US basing and support facilities shall be established. Temporary structures


such as those for troop billeting, classroom instruction and messing may be set up for use by
RP and US Forces during the Exercise.
4. The Exercise shall be implemented jointly by RP and US Exercise Co-Directors under the
authority of the Chief of Staff, AFP. In no instance will US Forces operate independently
during field training exercises (FTX). AFP and US Unit Commanders will retain command
over their respective forces under the overall authority of the Exercise Co-Directors. RP and
US participants shall comply with operational instructions of the AFP during the FTX.

5. The exercise shall be conducted and completed within a period of not more than six
months, with the projected participation of 660 US personnel and 3,800 RP Forces. The
Chief of Staff, AFP shall direct the Exercise Co-Directors to wind up and terminate the
Exercise and other activities within the six month Exercise period.

6. The Exercise is a mutual counter-terrorism advising, assisting and training Exercise


relative to Philippine efforts against the ASG, and will be conducted on the Island of Basilan.
Further advising, assisting and training exercises shall be conducted in Malagutay and the
Zamboanga area. Related activities in Cebu will be for support of the Exercise.

7. Only 160 US Forces organized in 12-man Special Forces Teams shall be deployed with
AFP field, commanders. The US teams shall remain at the Battalion Headquarters and,
when approved, Company Tactical headquarters where they can observe and assess the
performance of the AFP Forces.

8. US exercise participants shall not engage in combat, without prejudice to their right of self-
defense.

9. These terms of Reference are for purposes of this Exercise only and do not create
additional legal obligations between the US Government and the Republic of the Philippines.

II. EXERCISE LEVEL

1. TRAINING

a. The Exercise shall involve the conduct of mutual military assisting, advising and
training of RP and US Forces with the primary objective of enhancing the operational
capabilities of both forces to combat terrorism.

b. At no time shall US Forces operate independently within RP territory.

c. Flight plans of all aircraft involved in the exercise will comply with the local air
traffic regulations.

2. ADMINISTRATION & LOGISTICS

a. RP and US participants shall be given a country and area briefing at the start of
the Exercise. This briefing shall acquaint US Forces on the culture and sensitivities of
the Filipinos and the provisions of the VF A. The briefing shall also promote the full
cooperation on the part of the RP and US participants for the successful conduct of
the Exercise.

b. RP and US participating forces may share, in accordance with their respective


laws and regulations, in the use of their resources, equipment and other assets. They
will use their respective logistics channels.
c. Medical evaluation shall be jointly planned and executed utilizing RP and US
assets and resources.

d. Legal liaison officers from each respective party shall be appointed by the
Exercise Directors.

3. PUBLIC AFFAIRS

a. Combined RP-US Information Bureaus shall be established at the Exercise


Directorate in Zamboanga City and at GHQ, AFP in Camp Aguinaldo, Quezon City.

b. Local media relations will be the concern of the AFP and all public affairs
guidelines shall be jointly developed by RP and US Forces.

c. Socio-Economic Assistance Projects shall be planned and executed jointly by RP


and US Forces in accordance with their respective laws and regulations, and in
consultation with community and local government officials.

Contemporaneously, Assistant Secretary for American Affairs Minerva Jean A. Falcon and United
States Charge d' Affaires Robert Fitts signed the Agreed Minutes of the discussion between the
Vice-President and Assistant Secretary Kelly. 4

Petitioners Lim and Ersando present the following arguments:

THE PHILIPPINES AND THE UNITED STATES SIGNED THE MUTUAL DEFENSE
TREATY (MDT) in 1951 TO PROVIDE MUTUAL MILITARY ASSIST ANCE IN
ACCORDANCE WITH THE 'CONSTITUTIONAL PROCESSE-S' OF EACH COUNTRY
ONLY IN THE CASE OF AN ARMED ATTACK BY AN EXTERNAL AGGRESSOR,
MEANING A THIRD COUNTRY AGAINST ONE OF THEM.

BY NO STRETCH OF THE IMAGINA TION CAN IT BE SAID THAT THE ABU SAYYAF
BANDITS IN BASILAN CONSTITUTE AN EXTERNAL ARMED FORCE THAT HAS
SUBJECT THE PHILIPPINES TO AN ARMED EXTERNAL ATTACK TO WARRANT U.S.
MILITARY ASSISTANCE UNDER THE MDT OF 1951.

II

NEITHER DOES THE VFA OF 1999 AUTHORIZE AMERICAN SOLDIERS TO ENGAGE IN


COMBAT OPERATIONS IN PHILIPPINE TERRITORY, NOT EVEN TO FIRE BACK "IF
FIRED UPON".

Substantially the same points are advanced by petitioners SANLAKAS and PARTIDO.

In his Comment, the Solicitor General points to infirmities in the petitions regarding, inter alia, Lim
and Ersando's standing to file suit, the prematurity of the action, as well as the impropriety of availing
of certiorari to ascertain a question of fact. Anent their locus standi, the Solicitor General argues
that first, they may not file suit in their capacities as, taxpayers inasmuch as it has not been shown
that "Balikatan 02-1 " involves the exercise of Congress' taxing or spending powers. Second, their
being lawyers does not invest them with sufficient personality to initiate the case, citing our ruling
in Integrated Bar of the Philippines v. Zamora. Third, Lim and Ersando have failed to demonstrate

the requisite showing of direct personal injury. We agree.

It is also contended that the petitioners are indulging in speculation. The Solicitor General is of the
view that since the Terms of Reference are clear as to the extent and duration of "Balikatan 02-1,"
the issues raised by petitioners are premature, as they are based only on a fear of future violation of
the Terms of Reference. Even petitioners' resort to a special civil action for certiorari is assailed on
the ground that the writ may only issue on the basis of established facts.

Apart from these threshold issues, the Solicitor General claims that there is actually no question of
constitutionality involved. The true object of the instant suit, it is said, is to obtain an interpretation of
the V FA. The Solicitor General asks that we accord due deference to the executive determination
that "Balikatan 02-1" is covered by the VFA, considering the President's monopoly in the field of
foreign relations and her role as commander-in-chief of the Philippine armed forces.

Given the primordial importance of the issue involved, it will suffice to reiterate our view on this point
in a related case:

Notwithstanding, in view of the paramount importance and the constitutional


significance of the issues raised in the petitions, this Court, in the exercise of its
sound discretion, brushes aside the procedural barrier and takes cognizance of the
petitions, as we have done in the early Emergency Powers Cases, where we had
occasion to rule:

'x x x ordinary citizens and taxpayers were allowed to question the constitutionality of
several executive orders issued by President Quirino although they were involving
only an indirect and general interest shared in common with the public. The Court
dismissed the objection that they were not proper parties and ruled
that 'transcendental importance to the public of these cases demands that they
be settled promptly and definitely, brushing aside, if we must, technicalities of
procedure.' We have since then applied the exception in many other cases. [citation
omitted]

This principle was reiterated in the subsequent cases of Gonzales vs. COMELEC, Daza vs.
Singson, and Basco vs. Phil, Amusement and Gaming Corporation, where we
emphatically held:

Considering however the importance to the public of the case at bar, and in keeping
with the Court's duty, under the 1987 Constitution, to determine whether or not the
other branches of the government have kept themselves within the limits of the
Constitution and the laws that they have not abused the discretion given to them, the
Court has brushed aside technicalities of procedure and has taken cognizance of this
petition. xxx'

Again, in the more recent case of Kilosbayan vs. Guingona, Jr., this Court ruled that in
cases of transcendental importance, the Court may relax the standing requirements and
allow a suit to prosper even where there is no direct injury to the party claiming the
right of judicial review.

Although courts generally avoid having to decide a constitutional question based on the
doctrine of separation of powers, which enjoins upon the department of the government a
becoming respect for each other's act, this Court nevertheless resolves to take cognizance
of the instant petition. 6

Hence, we treat with similar dispatch the general objection to the supposed prematurity of the action.
At any rate, petitioners' concerns on the lack of any specific regulation on the latitude of activity US
personnel may undertake and the duration of their stay has been addressed in the Terms of
Reference.

The holding of "Balikatan 02-1" must be studied in the framework of the treaty antecedents to which
the Philippines bound itself. The first of these is the Mutual Defense Treaty (MDT, for brevity). The
MDT has been described as the "core" of the defense relationship between the Philippines and its
traditional ally, the United States. Its aim is to enhance the strategic and technological capabilities of
our armed forces through joint training with its American counterparts; the "Balikatan" is the largest
such training exercise directly supporting the MDT's objectives. It is this treaty to which the V FA
adverts and the obligations thereunder which it seeks to reaffirm.

The lapse of the US-Philippine Bases Agreement in 1992 and the decision not to renew it created a
vacuum in US-Philippine defense relations, that is, until it was replaced by the Visiting Forces
Agreement. It should be recalled that on October 10, 2000, by a vote of eleven to three, this Court
upheld the validity of the VFA. The V FA provides the "regulatory mechanism" by which "United

States military and civilian personnel [may visit] temporarily in the Philippines in connection with
activities approved by the Philippine Government." It contains provisions relative to entry and
departure of American personnel, driving and vehicle registration, criminal jurisdiction, claims,
importation and exportation, movement of vessels and aircraft, as well as the duration of the
agreement and its termination. It is the VFA which gives continued relevance to the MDT despite the
passage of years. Its primary goal is to facilitate the promotion of optimal cooperation between
American and Philippine military forces in the event of an attack by a common foe.

The first question that should be addressed is whether "Balikatan 02-1" is covered by the Visiting
Forces Agreement. To resolve this, it is necessary to refer to the V FA itself: Not much help can be
had therefrom, unfortunately, since the terminology employed is itself the source of the problem. The
VFA permits United States personnel to engage, on an impermanent basis, in "activities," the exact
meaning of which was left undefined. The expression is ambiguous, permitting a wide scope of
undertakings subject only to the approval of the Philippine government. The sole encumbrance

placed on its definition is couched in the negative, in that United States personnel must "abstain from
any activity inconsistent with the spirit of this agreement, and in particular, from any political
activity." All other activities, in other words, are fair game.

We are not left completely unaided, however. The Vienna Convention on the Law of Treaties, which
contains provisos governing interpretations of international agreements, state:

SECTION 3. INTERPRETATION OF TREATIES

Article 31

General rule of interpretation

1. A treaty shall be interpreted in good faith ill accordance with the ordinary meaning to be
given to the tenus of the treaty in their context and in the light of its object and purpose.
2. The context for the purpose of the interpretation of a treaty shall comprise, in addition to
the text, including its preamble and annexes:

(a) any agreement relating to the treaty which was made between all the parties in
connexion with the conclusion of the treaty;

(b) any instrument which was made by one or more parties in connexion with the
conclusion of the treaty and accepted by the other parties as an instrument related to
the party .

3. There shall be taken into account, together with the context:

(a) any subsequent agreement between the parties regarding the interpretation of the
treaty or the application of its provisions;

(b) any subsequent practice in the application of the treaty which establishes the
agreement of the parties regarding its interpretation;

(c) any relevant rules of international law applicable in the relations between the
parties.

4. A special meaning shall be given to a term if it is established that the parties so intended.

Article 32

Supplementary means of interpretation

Recourse may be had to supplementary means of interpretation, including the preparatory


work of the treaty and the circumstances of its conclusion, in order to confirm the meaning
resulting from the application of article 31, or to determine the meaning when the
interpretation according to article 31 :

(a) leaves the meaning ambiguous or obscure; or

(b) leads to a result which is manifestly absurd unreasonable.

It is clear from the foregoing that the cardinal rule of interpretation must involve an examination of
the text, which is presumed to verbalize the parties' intentions. The Convention likewise dictates
what may be used as aids to deduce the meaning of terms, which it refers to as the context of the
treaty, as well as other elements may be taken into account alongside the aforesaid context. As
explained by a writer on the Convention ,

[t]he Commission's proposals (which were adopted virtually without change by the
conference and are now reflected in Articles 31 and 32 of the Convention) were clearly
based on the view that the text of a treaty must be presumed to be the authentic expression
of the intentions of the parties; the Commission accordingly came down firmly in favour of the
view that 'the starting point of interpretation is the elucidation of the meaning of the text, not
an investigation ab initio into the intentions of the parties'. This is not to say that
the travauxpreparatoires of a treaty , or the circumstances of its conclusion, are relegated to
a subordinate, and wholly ineffective, role. As Professor Briggs points out, no rigid temporal
prohibition on resort to travaux preparatoires of a treaty was intended by the use of the
phrase 'supplementary means of interpretation' in what is now Article 32 of the Vienna
Convention. The distinction between the general rule of interpretation and the supplementary
means of interpretation is intended rather to ensure that the supplementary means do not
constitute an alternative, autonomous method of interpretation divorced from the general
rule.10

The Terms of Reference rightly fall within the context of the VFA.

After studied reflection, it appeared farfetched that the ambiguity surrounding the meaning of the
word .'activities" arose from accident. In our view, it was deliberately made that way to give both
parties a certain leeway in negotiation. In this manner, visiting US forces may sojourn in Philippine
territory for purposes other than military. As conceived, the joint exercises may include training on
new techniques of patrol and surveillance to protect the nation's marine resources, sea search-and-
rescue operations to assist vessels in distress, disaster relief operations, civic action projects such
as the building of school houses, medical and humanitarian missions, and the like.

Under these auspices, the VFA gives legitimacy to the current Balikatan exercises. It is only logical
to assume that .'Balikatan 02-1," a "mutual anti- terrorism advising, assisting and training exercise,"
falls under the umbrella of sanctioned or allowable activities in the context of the agreement. Both
the history and intent of the Mutual Defense Treaty and the V FA support the conclusion
that combat-related activities -as opposed to combat itself -such as the one subject of the instant
petition, are indeed authorized.

That is not the end of the matter, though. Granted that "Balikatan 02-1" is permitted under the terms
of the VFA, what may US forces legitimately do in furtherance of their aim to provide advice,
assistance and training in the global effort against terrorism? Differently phrased, may American
troops actually engage in combat in Philippine territory? The Terms of Reference are explicit
enough. Paragraph 8 of section I stipulates that US exercise participants may not engage
in combat "except in self-defense." We wryly note that this sentiment is admirable in the abstract
but difficult in implementation. The target of "Balikatan 02-1 I" the Abu Sayyaf, cannot reasonably be
expected to sit idly while the battle is brought to their very doorstep. They cannot be expected to pick
and choose their targets for they will not have the luxury of doing so. We state this point if only to
signify our awareness that the parties straddle a fine line, observing the honored legal maxim "Nemo
potest facere per alium quod non potest facere per directum." The indirect violation is actually
11 

petitioners' worry, that in reality, "Balikatan 02-1 " is actually a war principally conducted by the
United States government, and that the provision on self-defense serves only as camouflage to
conceal the true nature of the exercise. A clear pronouncement on this matter thereby becomes
crucial.

In our considered opinion, neither the MDT nor the V FA allow foreign troops to engage in an
offensive war on Philippine territory. We bear in mind the salutary proscription stated in the Charter
of the United Nations, to wit:

Article 2

The Organization and its Members, in pursuit of the Purposes stated in Article 1, shall act in
accordance with the following Principles.

xxx      xxx      xxx      xxx
4. All Members shall refrain in their international relations from the threat or use of force
against the territorial integrity or political independence of any state, or in any other manner
inconsistent with the Purposes of the United Nations.

xxx      xxx      xxx      xxx

In the same manner, both the Mutual Defense Treaty and the Visiting Forces Agreement, as in all
other treaties and international agreements to which the Philippines is a party, must be read in the
context of the 1987 Constitution. In particular, the Mutual Defense Treaty was concluded way before
the present Charter, though it nevertheless remains in effect as a valid source of international
obligation. The present Constitution contains key provisions useful in determining the extent to which
foreign military troops are allowed in Philippine territory. Thus, in the Declaration of Principles and
State Policies, it is provided that:

xxx      xxx      xxx      xxx

SEC. 2. The Philippines renounces war as an instrument of national policy, adopts the
generally accepted principles of international law as part of the law of the land and adheres
to the policy of peace, equality, justice, freedom, cooperation, and amity with all nations.

xxx      xxx      xxx      xxx

SEC. 7. The State shall pursue an independent foreign policy. In its relations with other
states the paramount consideration shall be national sovereignty, territorial integrity, national
interest, and the right to self- determination.

SEC. 8. The Philippines, consistent with the national interest, adopts and pursues a policy of
freedom from nuclear weapons in the country.

xxx      xxx      xxx      xxx

The Constitution also regulates the foreign relations powers of the Chief Executive when it provides
that "[n]o treaty or international agreement shall be valid and effective unless concurred in by at least
two-thirds of all the members of the Senate." Even more pointedly, the Transitory Provisions state:
12 

Sec. 25. After the expiration in 1991 of the Agreement between the Republic of the
Philippines and the United States of America concerning Military Bases, foreign military
bases, troops or facilities shall not be allowed in the Philippines except under a treaty duly
concurred in by the Senate and, when the Congress so requires, ratified by a majority of the
votes cast by the people in a national referendum held for that purpose, and recognized as a
treaty by the other contracting state.

The aforequoted provisions betray a marked antipathy towards foreign military presence in the
country, or of foreign influence in general. Hence, foreign troops are allowed entry into the
Philippines only by way of direct exception. Conflict arises then between the fundamental law and
our obligations arising from international agreements.

A rather recent formulation of the relation of international law vis-a-vis municipal law was expressed
in Philip Morris, Inc. v. Court of Appeals, to wit:
13 
xxx Withal, the fact that international law has been made part of the law of the land does not
by any means imply the primacy of international law over national law in the municipal
sphere. Under the doctrine of incorporation as applied in most countries, rules of
international law are given a standing equal, not superior, to national legislation.

This is not exactly helpful in solving the problem at hand since in trying to find a middle ground, it
favors neither one law nor the other, which only leaves the hapless seeker with an unsolved
dilemma. Other more traditional approaches may offer valuable insights.

From the perspective of public international law, a treaty is favored over municipal law pursuant to
the principle of pacta sunt servanda. Hence, "[e]very treaty in force is binding upon the parties to it
and must be performed by them in good faith." Further, a party to a treaty is not allowed to "invoke
14 

the provisions of its internal law as justification for its failure to perform a treaty."
15

Our Constitution espouses the opposing view. Witness our jurisdiction as I stated in section 5 of
Article VIII:

The Supreme Court shall have the following powers:

xxx      xxx      xxx      xxx

(2) Review, revise, reverse, modify, or affirm on appeal or certiorari, as the law or the Rules
of Court may provide, final judgments and order of lower courts in:

(A) All cases in which the constitutionality or validity of any treaty, international or executive
agreement, law, presidential decree, proclamation, order, instruction, ordinance, or
regulation is in question.

xxx      xxx      xxx      xxx

In Ichong v. Hernandez, we ruled that the provisions of a treaty are always subject to qualification
16 

or amendment by a subsequent law, or that it is subject to the police power of the State.
In Gonzales v. Hechanova, 17

xxx As regards the question whether an international agreement may be invalidated by our
courts, suffice it to say that the Constitution of the Philippines has clearly settled it in the
affirmative, by providing, in Section 2 of Article VIII thereof, that the Supreme Court may not
be deprived "of its jurisdiction to review, revise, reverse, modify, or affirm on appeal,
certiorari, or writ of error as the law or the rules of court may provide, final judgments and
decrees of inferior courts in -( I) All cases in which the constitutionality or validity of
any treaty, law, ordinance, or executive order or regulation is in question." In other words,
our Constitution authorizes the nullification of a treaty, not only when it conflicts with the
fundamental law, but, also, when it runs counter to an act of Congress.

The foregoing premises leave us no doubt that US forces are prohibited / from engaging in an
offensive war on Philippine territory.

Yet a nagging question remains: are American troops actively engaged in combat alongside Filipino
soldiers under the guise of an alleged training and assistance exercise? Contrary to what petitioners
would have us do, we cannot take judicial notice of the events transpiring down south,18 as reported
from the saturation coverage of the media. As a rule, we do not take cognizance of newspaper or
electronic reports per se, not because of any issue as to their truth, accuracy, or impartiality, but for
the simple reason that facts must be established in accordance with the rules of evidence. As a
result, we cannot accept, in the absence of concrete proof, petitioners' allegation that the Arroyo
government is engaged in "doublespeak" in trying to pass off as a mere training exercise an
offensive effort by foreign troops on native soil. The petitions invite us to speculate on what is really
happening in Mindanao, to issue I make factual findings on matters well beyond our immediate
perception, and this we are understandably loath to do.

It is all too apparent that the determination thereof involves basically a question of fact. On this point,
we must concur with the Solicitor General that the present subject matter is not a fit topic for a
special civil action for certiorari. We have held in too many instances that questions of fact are not
entertained in such a remedy. The sole object of the writ is to correct errors of jurisdiction or grave
abuse of discretion: The phrase "grave abuse of discretion" has a precise meaning in law, denoting
abuse of discretion "too patent and gross as to amount to an evasion of a positive duty, or a virtual
refusal to perform the duty enjoined or act in contemplation of law, or where the power is exercised
in an arbitrary and despotic manner by reason of passion and personal hostility." 19

In this connection, it will not be amiss to add that the Supreme Court is not a trier of facts. 20

Under the expanded concept of judicial power under the Constitution, courts are charged with the
duty "to determine whether or not there has been a grave abuse of discretion amounting to lack or
excess of jurisdiction on the part of any branch or instrumentality of the government." From the facts
21 

obtaining, we find that the holding of "Balikatan 02-1" joint military exercise has not intruded into that
penumbra of error that would otherwise call for correction on our part. In other words, respondents in
the case at bar have not committed grave abuse of discretion amounting to lack or excess of
jurisdiction.

WHEREFORE, the petition and the petition-in-intervention are hereby DISMISSED without prejudice


to the filing of a new petition sufficient in form and substance in the proper Regional Trial Court.

SO ORDERED.

G.R. No. 141284               August 15, 2000

INTEGRATED BAR OF THE PHILIPPINES, petitioner, 


vs.
HON. RONALDO B. ZAMORA, GEN. PANFILO M. LACSON, GEN. EDGAR B. AGLIPAY, and
GEN. ANGELO REYES, respondents.

DECISION

KAPUNAN, J.:

At bar is a special civil action for certiorari and prohibition with prayer for issuance of a temporary
restraining order seeking to nullify on constitutional grounds the order of President Joseph Ejercito
Estrada commanding the deployment of the Philippine Marines (the "Marines") to join the Philippine
National Police (the "PNP") in visibility patrols around the metropolis.

In view of the alarming increase in violent crimes in Metro Manila, like robberies, kidnappings and
carnappings, the President, in a verbal directive, ordered the PNP and the Marines to conduct joint
visibility patrols for the purpose of crime prevention and suppression. The Secretary of National
Defense, the Chief of Staff of the Armed Forces of the Philippines (the "AFP"), the Chief of the PNP
and the Secretary of the Interior and Local Government were tasked to execute and implement the
said order. In compliance with the presidential mandate, the PNP Chief, through Police Chief
Superintendent Edgar B. Aglipay, formulated Letter of Instruction 02/2000 (the "LOI") which detailed

the manner by which the joint visibility patrols, called Task Force Tulungan, would be
conducted. Task Force Tulungan was placed under the leadership of the Police Chief of Metro

Manila.

Subsequently, the President confirmed his previous directive on the deployment of the Marines in a
Memorandum, dated 24 January 2000, addressed to the Chief of Staff of the AFP and the PNP
Chief. In the Memorandum, the President expressed his desire to improve the peace and order

situation in Metro Manila through a more effective crime prevention program including increased
police patrols. The President further stated that to heighten police visibility in the metropolis,

augmentation from the AFP is necessary. Invoking his powers as Commander-in-Chief under

Section 18, Article VII of the Constitution, the President directed the AFP Chief of Staff and PNP
Chief to coordinate with each other for the proper deployment and utilization of the Marines to assist
the PNP in preventing or suppressing criminal or lawless violence. Finally, the President declared

that the services of the Marines in the anti-crime campaign are merely temporary in nature and for a
reasonable period only, until such time when the situation shall have improved. 7

The LOI explains the concept of the PNP-Philippine Marines joint visibility patrols as follows:

xxx

2. PURPOSE:

The Joint Implementing Police Visibility Patrols between the PNP NCRPO and the Philippine
Marines partnership in the conduct of visibility patrols in Metro Manila for the suppression of crime
prevention and other serious threats to national security.

3. SITUATION:

Criminal incidents in Metro Manila have been perpetrated not only by ordinary criminals but also by
organized syndicates whose members include active and former police/military personnel whose
training, skill, discipline and firepower prove well-above the present capability of the local police
alone to handle. The deployment of a joint PNP NCRPO-Philippine Marines in the conduct of police
visibility patrol in urban areas will reduce the incidence of crimes specially those perpetrated by
active or former police/military personnel.

4. MISSION:

The PNP NCRPO will organize a provisional Task Force to conduct joint NCRPO-PM visibility
patrols to keep Metro Manila streets crime-free, through a sustained street patrolling to minimize or
eradicate all forms of high-profile crimes especially those perpetrated by organized crime syndicates
whose members include those that are well-trained, disciplined and well-armed active or former
PNP/Military personnel.

5. CONCEPT IN JOINT VISIBILITY PATROL OPERATIONS:

a. The visibility patrols shall be conducted jointly by the NCRPO [National Capital Regional
Police Office] and the Philippine Marines to curb criminality in Metro Manila and to preserve
the internal security of the state against insurgents and other serious threat to national
security, although the primary responsibility over Internal Security Operations still rests upon
the AFP.

b. The principle of integration of efforts shall be applied to eradicate all forms of high-profile
crimes perpetrated by organized crime syndicates operating in Metro Manila. This concept
requires the military and police to work cohesively and unify efforts to ensure a focused,
effective and holistic approach in addressing crime prevention. Along this line, the role of the
military and police aside from neutralizing crime syndicates is to bring a wholesome
atmosphere wherein delivery of basic services to the people and development is achieved.
Hand-in-hand with this joint NCRPO-Philippine Marines visibility patrols, local Police Units
are responsible for the maintenance of peace and order in their locality.

c. To ensure the effective implementation of this project, a provisional Task Force


"TULUNGAN" shall be organized to provide the mechanism, structure, and procedures for
the integrated planning, coordinating, monitoring and assessing the security situation.

xxx. 8

The selected areas of deployment under the LOI are: Monumento Circle, North Edsa (SM City),
Araneta Shopping Center, Greenhills, SM Megamall, Makati Commercial Center, LRT/MRT Stations
and the NAIA and Domestic Airport. 9

On 17 January 2000, the Integrated Bar of the Philippines (the "IBP") filed the instant petition to
annul LOI 02/2000 and to declare the deployment of the Philippine Marines, null and void and
unconstitutional, arguing that:

THE DEPLOYMENT OF THE PHILIPPINE MARINES IN METRO MANILA IS VIOLATIVE OF THE


CONSTITUTION, IN THAT:

A) NO EMERGENCY SITUATION OBTAINS IN METRO MANILA AS WOULD JUSTIFY,


EVEN ONLY REMOTELY, THE DEPLOYMENT OF SOLDIERS FOR LAW ENFORCEMENT
WORK; HENCE, SAID DEPLOYMENT IS IN DEROGATION OF ARTICLE II, SECTION 3
OF THE CONSTITUTION;

B) SAID DEPLOYMENT CONSTITUTES AN INSIDIOUS INCURSION BY THE MILITARY IN


A CIVILIAN FUNCTION OF GOVERNMENT (LAW ENFORCEMENT) IN DEROGATION OF
ARTICLE XVI, SECTION 5 (4), OF THE CONSTITUTION;

C) SAID DEPLOYMENT CREATES A DANGEROUS TENDENCY TO RELY ON THE


MILITARY TO PERFORM THE CIVILIAN FUNCTIONS OF THE GOVERNMENT.

II

IN MILITARIZING LAW ENFORCEMENT IN METRO MANILA, THE ADMINISTRATION IS


UNWITTINGLY MAKING THE MILITARY MORE POWERFUL THAN WHAT IT SHOULD REALLY
BE UNDER THE CONSTITUTION. 10
Asserting itself as the official organization of Filipino lawyers tasked with the bounden duty to uphold
the rule of law and the Constitution, the IBP questions the validity of the deployment and utilization of
the Marines to assist the PNP in law enforcement.

Without granting due course to the petition, the Court in a Resolution, dated 25 January 2000,
11 

required the Solicitor General to file his Comment on the petition. On 8 February 2000, the Solicitor
General submitted his Comment.

The Solicitor General vigorously defends the constitutionality of the act of the President in deploying
the Marines, contending, among others, that petitioner has no legal standing; that the question of
deployment of the Marines is not proper for judicial scrutiny since the same involves a political
question; that the organization and conduct of police visibility patrols, which feature the team-up of
one police officer and one Philippine Marine soldier, does not violate the civilian supremacy clause in
the Constitution.

The issues raised in the present petition are: (1) Whether or not petitioner has legal standing; (2)
Whether or not the President’s factual determination of the necessity of calling the armed forces is
subject to judicial review; and, (3) Whether or not the calling of the armed forces to assist the PNP in
joint visibility patrols violates the constitutional provisions on civilian supremacy over the military and
the civilian character of the PNP.

The petition has no merit.

First, petitioner failed to sufficiently show that it is in possession of the requisites of standing to raise
the issues in the petition. Second, the President did not commit grave abuse of discretion amounting
to lack or excess of jurisdiction nor did he commit a violation of the civilian supremacy clause of the
Constitution.

The power of judicial review is set forth in Section 1, Article VIII of the Constitution, to wit:

Section 1. The judicial power shall be vested in one Supreme Court and in such lower courts as may
be established by law.

Judicial power includes the duty of the courts of justice to settle actual controversies involving rights
which are legally demandable and enforceable, and to determine whether or not there has been
grave abuse of discretion amounting to lack or excess of jurisdiction on the part of any branch or
instrumentality of the Government.

When questions of constitutional significance are raised, the Court can exercise its power of judicial
review only if the following requisites are complied with, namely: (1) the existence of an actual and
appropriate case; (2) a personal and substantial interest of the party raising the constitutional
question; (3) the exercise of judicial review is pleaded at the earliest opportunity; and (4) the
constitutional question is the lis mota of the case. 12

The IBP has not sufficiently complied with the requisites of standing in this case.

"Legal standing" or locus standi has been defined as a personal and substantial interest in the case
such that the party has sustained or will sustain direct injury as a result of the governmental act that
is being challenged. The term "interest" means a material interest, an interest in issue affected by
13 

the decree, as distinguished from mere interest in the question involved, or a mere incidental
interest. The gist of the question of standing is whether a party alleges "such personal stake in the
14 
outcome of the controversy as to assure that concrete adverseness which sharpens the presentation
of issues upon which the court depends for illumination of difficult constitutional questions."15

In the case at bar, the IBP primarily anchors its standing on its alleged responsibility to uphold the
rule of law and the Constitution. Apart from this declaration, however, the IBP asserts no other basis
in support of its locus standi. The mere invocation by the IBP of its duty to preserve the rule of law
and nothing more, while undoubtedly true, is not sufficient to clothe it with standing in this case. This
is too general an interest which is shared by other groups and the whole citizenry. Based on the
standards above-stated, the IBP has failed to present a specific and substantial interest in the
resolution of the case. Its fundamental purpose which, under Section 2, Rule 139-A of the Rules of
Court, is to elevate the standards of the law profession and to improve the administration of justice is
alien to, and cannot be affected by the deployment of the Marines. It should also be noted that the
interest of the National President of the IBP who signed the petition, is his alone, absent a formal
board resolution authorizing him to file the present action. To be sure, members of the BAR, those in
the judiciary included, have varying opinions on the issue. Moreover, the IBP, assuming that it has
duly authorized the National President to file the petition, has not shown any specific injury which it
has suffered or may suffer by virtue of the questioned governmental act. Indeed, none of its
members, whom the IBP purportedly represents, has sustained any form of injury as a result of the
operation of the joint visibility patrols. Neither is it alleged that any of its members has been arrested
or that their civil liberties have been violated by the deployment of the Marines. What the IBP
projects as injurious is the supposed "militarization" of law enforcement which might threaten
Philippine democratic institutions and may cause more harm than good in the long run. Not only is
the presumed "injury" not personal in character, it is likewise too vague, highly speculative and
uncertain to satisfy the requirement of standing. Since petitioner has not successfully established a
direct and personal injury as a consequence of the questioned act, it does not possess the
personality to assail the validity of the deployment of the Marines. This Court, however, does not
categorically rule that the IBP has absolutely no standing to raise constitutional issues now or in the
future. The IBP must, by way of allegations and proof, satisfy this Court that it has sufficient stake to
obtain judicial resolution of the controversy.

Having stated the foregoing, it must be emphasized that this Court has the discretion to take
cognizance of a suit which does not satisfy the requirement of legal standing when paramount
interest is involved. In not a few cases, the Court has adopted a liberal attitude on the locus
16 

standi of a petitioner where the petitioner is able to craft an issue of transcendental significance to
the people. Thus, when the issues raised are of paramount importance to the public, the Court may
17 

brush aside technicalities of procedure. In this case, a reading of the petition shows that the IBP has
18 

advanced constitutional issues which deserve the attention of this Court in view of their seriousness,
novelty and weight as precedents. Moreover, because peace and order are under constant threat
and lawless violence occurs in increasing tempo, undoubtedly aggravated by the Mindanao
insurgency problem, the legal controversy raised in the petition almost certainly will not go away. It
will stare us in the face again. It, therefore, behooves the Court to relax the rules on standing and to
resolve the issue now, rather than later.

The President did not commit grave abuse of discretion in calling out the Marines.

In the case at bar, the bone of contention concerns the factual determination of the President of the
necessity of calling the armed forces, particularly the Marines, to aid the PNP in visibility patrols. In
this regard, the IBP admits that the deployment of the military personnel falls under the Commander-
in-Chief powers of the President as stated in Section 18, Article VII of the Constitution, specifically,
the power to call out the armed forces to prevent or suppress lawless violence, invasion or rebellion.
What the IBP questions, however, is the basis for the calling of the Marines under the aforestated
provision. According to the IBP, no emergency exists that would justify the need for the calling of the
military to assist the police force. It contends that no lawless violence, invasion or rebellion exist to
warrant the calling of the Marines. Thus, the IBP prays that this Court "review the sufficiency of the
factual basis for said troop [Marine] deployment." 19

The Solicitor General, on the other hand, contends that the issue pertaining to the necessity of
calling the armed forces is not proper for judicial scrutiny since it involves a political question and the
resolution of factual issues which are beyond the review powers of this Court.

As framed by the parties, the underlying issues are the scope of presidential powers and limits, and
the extent of judicial review. But, while this Court gives considerable weight to the parties’
formulation of the issues, the resolution of the controversy may warrant a creative approach that
goes beyond the narrow confines of the issues raised. Thus, while the parties are in agreement that
the power exercised by the President is the power to call out the armed forces, the Court is of the
view that the power involved may be no more than the maintenance of peace and order and
promotion of the general welfare. For one, the realities on the ground do not show that there exist a
20 

state of warfare, widespread civil unrest or anarchy. Secondly, the full brunt of the military is not
brought upon the citizenry, a point discussed in the latter part of this decision. In the words of the
late Justice Irene Cortes in Marcos v. Manglapus:

More particularly, this case calls for the exercise of the President’s powers as protector of the peace.
[Rossiter, The American Presidency]. The power of the President to keep the peace is not limited
merely to exercising the commander-in-chief powers in times of emergency or to leading the State
against external and internal threats to its existence. The President is not only clothed with
extraordinary powers in times of emergency, but is also tasked with attending to the day-to-day
problems of maintaining peace and order and ensuring domestic tranquility in times when no foreign
foe appears on the horizon. Wide discretion, within the bounds of law, in fulfilling presidential duties
in times of peace is not in any way diminished by the relative want of an emergency specified in the
commander-in-chief provision. For in making the President commander-in-chief the enumeration of
powers that follow cannot be said to exclude the President’s exercising as Commander-in-Chief
powers short of the calling of the armed forces, or suspending the privilege of the writ of habeas
corpus or declaring martial law, in order to keep the peace, and maintain public order and security.

xxx 21

Nonetheless, even if it is conceded that the power involved is the President’s power to call out the
armed forces to prevent or suppress lawless violence, invasion or rebellion, the resolution of the
controversy will reach a similar result.

We now address the Solicitor General’s argument that the issue involved is not susceptible to review
by the judiciary because it involves a political question, and thus, not justiciable.

As a general proposition, a controversy is justiciable if it refers to a matter which is appropriate for


court review. It pertains to issues which are inherently susceptible of being decided on grounds
22 

recognized by law. Nevertheless, the Court does not automatically assume jurisdiction over actual
constitutional cases brought before it even in instances that are ripe for resolution. One class of
cases wherein the Court hesitates to rule on are "political questions." The reason is that political
questions are concerned with issues dependent upon the wisdom, not the legality, of a particular act
or measure being assailed. Moreover, the political question being a function of the separation of
powers, the courts will not normally interfere with the workings of another co-equal branch unless the
case shows a clear need for the courts to step in to uphold the law and the Constitution.
As Tañada v. Cuenco puts it, political questions refer "to those questions which, under the
23 

Constitution, are to be decided by the people in their sovereign capacity, or in regard to which full
discretionary authority has been delegated to the legislative or executive branch of government."
Thus, if an issue is clearly identified by the text of the Constitution as matters for discretionary action
by a particular branch of government or to the people themselves then it is held to be a political
question. In the classic formulation of Justice Brennan in Baker v. Carr, "[p]rominent on the surface
24 

of any case held to involve a political question is found a textually demonstrable constitutional
commitment of the issue to a coordinate political department; or a lack of judicially discoverable and
manageable standards for resolving it; or the impossibility of deciding without an initial policy
determination of a kind clearly for nonjudicial discretion; or the impossibility of a court’s undertaking
independent resolution without expressing lack of the respect due coordinate branches of
government; or an unusual need for unquestioning adherence to a political decision already made;
or the potentiality of embarassment from multifarious pronouncements by various departments on
the one question."

The 1987 Constitution expands the concept of judicial review by providing that "(T)he Judicial power
shall be vested in one Supreme Court and in such lower courts as may be established by law.
Judicial power includes the duty of the courts of justice to settle actual controversies involving rights
which are legally demandable and enforceable, and to determine whether or not there has been a
grave abuse of discretion amounting to lack or excess of jurisdiction on the part of any branch or
instrumentality of the Government." Under this definition, the Court cannot agree with the Solicitor
25 

General that the issue involved is a political question beyond the jurisdiction of this Court to review.
When the grant of power is qualified, conditional or subject to limitations, the issue of whether the
prescribed qualifications or conditions have been met or the limitations respected, is justiciable - the
problem being one of legality or validity, not its wisdom. Moreover, the jurisdiction to delimit
26 

constitutional boundaries has been given to this Court. When political questions are involved, the
27 

Constitution limits the determination as to whether or not there has been a grave abuse of discretion
amounting to lack or excess of jurisdiction on the part of the official whose action is being
questioned. 28

By grave abuse of discretion is meant simply capricious or whimsical exercise of judgment that is
patent and gross as to amount to an evasion of positive duty or a virtual refusal to perform a duty
enjoined by law, or to act at all in contemplation of law, as where the power is exercised in an
arbitrary and despotic manner by reason of passion or hostility. Under this definition, a court is
29 

without power to directly decide matters over which full discretionary authority has been delegated.
But while this Court has no power to substitute its judgment for that of Congress or of the President,
it may look into the question of whether such exercise has been made in grave abuse of
discretion. A showing that plenary power is granted either department of government, may not be an
30

obstacle to judicial inquiry, for the improvident exercise or abuse thereof may give rise to justiciable
controversy. 31

When the President calls the armed forces to prevent or suppress lawless violence, invasion or
rebellion, he necessarily exercises a discretionary power solely vested in his wisdom. This is clear
from the intent of the framers and from the text of the Constitution itself. The Court, thus, cannot be
called upon to overrule the President’s wisdom or substitute its own. However, this does not prevent
an examination of whether such power was exercised within permissible constitutional limits or
whether it was exercised in a manner constituting grave abuse of discretion. In view of the
constitutional intent to give the President full discretionary power to determine the necessity of
calling out the armed forces, it is incumbent upon the petitioner to show that the President’s decision
is totally bereft of factual basis. The present petition fails to discharge such heavy burden as there is
no evidence to support the assertion that there exist no justification for calling out the armed forces.
There is, likewise, no evidence to support the proposition that grave abuse was committed because
the power to call was exercised in such a manner as to violate the constitutional provision on civilian
supremacy over the military. In the performance of this Court’s duty of "purposeful hesitation" before
32 

declaring an act of another branch as unconstitutional, only where such grave abuse of discretion is
clearly shown shall the Court interfere with the President’s judgment. To doubt is to sustain.

There is a clear textual commitment under the Constitution to bestow on the President full
discretionary power to call out the armed forces and to determine the necessity for the exercise of
such power. Section 18, Article VII of the Constitution, which embodies the powers of the President
as Commander-in-Chief, provides in part:

The President shall be the Commander-in-Chief of all armed forces of the Philippines and whenever
it becomes necessary, he may call out such armed forces to prevent or suppress lawless violence,
invasion or rebellion. In case of invasion or rebellion, when the public safety requires it, he may, for a
period not exceeding sixty days, suspend the privilege of the writ of habeas corpus, or place the
Philippines or any part thereof under martial law.

xxx

The full discretionary power of the President to determine the factual basis for the exercise of the
calling out power is also implied and further reinforced in the rest of Section 18, Article VII which
reads, thus:

xxx

Within forty-eight hours from the proclamation of martial law or the suspension of the privilege of the
writ of habeas corpus, the President shall submit a report in person or in writing to the Congress.
The Congress, voting jointly, by a vote of at least a majority of all its Members in regular or special
session, may revoke such proclamation or suspension, which revocation shall not be set aside by
the President. Upon the initiative of the President, the Congress may, in the same manner, extend
such proclamation or suspension for a period to be determined by the Congress, if the invasion or
rebellion shall persist and public safety requires it.

The Congress, if not in session, shall within twenty-four hours following such proclamation or
suspension, convene in accordance with its rules without need of a call.

The Supreme Court may review, in an appropriate proceeding filed by any citizen, the sufficiency of
the factual basis of the proclamation of martial law or the suspension of the privilege of the writ or
the extension thereof, and must promulgate its decision thereon within thirty days from its filing.

A state of martial law does not suspend the operation of the Constitution, nor supplant the
functioning of the civil courts or legislative assemblies, nor authorize the conferment of jurisdiction on
military courts and agencies over civilians where civil courts are able to function, nor automatically
suspend the privilege of the writ.

The suspension of the privilege of the writ shall apply only to persons judicially charged for rebellion
or offenses inherent in or directly connected with invasion.

During the suspension of the privilege of the writ, any person thus arrested or detained shall be
judicially charged within three days, otherwise he shall be released.

Under the foregoing provisions, Congress may revoke such proclamation or suspension and the
Court may review the sufficiency of the factual basis thereof. However, there is no such equivalent
provision dealing with the revocation or review of the President’s action to call out the armed forces.
The distinction places the calling out power in a different category from the power to declare martial
law and the power to suspend the privilege of the writ of habeas corpus, otherwise, the framers of
the Constitution would have simply lumped together the three powers and provided for their
revocation and review without any qualification. Expressio unius est exclusio alterius. Where the
terms are expressly limited to certain matters, it may not, by interpretation or construction, be
extended to other matters. That the intent of the Constitution is exactly what its letter says, i.e., that
33 

the power to call is fully discretionary to the President, is extant in the deliberation of the
Constitutional Commission, to wit:

FR. BERNAS. It will not make any difference. I may add that there is a graduated power of the
President as Commander-in-Chief. First, he can call out such Armed Forces as may be necessary to
suppress lawless violence; then he can suspend the privilege of the writ of habeas corpus, then he
can impose martial law. This is a graduated sequence.

When he judges that it is necessary to impose martial law or suspend the privilege of the writ
of habeas corpus, his judgment is subject to review. We are making it subject to review by the
Supreme Court and subject to concurrence by the National Assembly. But when he exercises this
lesser power of calling on the Armed Forces, when he says it is necessary, it is my opinion that his
judgment cannot be reviewed by anybody.

xxx

FR. BERNAS. Let me just add that when we only have imminent danger, the matter can be handled
by the first sentence: "The President may call out such armed forces to prevent or suppress lawless
violence, invasion or rebellion." So we feel that that is sufficient for handling imminent danger.

MR. DE LOS REYES. So actually, if a President feels that there is imminent danger, the matter can
be handled by the First Sentence: "The President....may call out such Armed Forces to prevent or
suppress lawless violence, invasion or rebellion." So we feel that that is sufficient for handling
imminent danger, of invasion or rebellion, instead of imposing martial law or suspending the writ
of habeas corpus, he must necessarily have to call the Armed Forces of the Philippines as their
Commander-in-Chief. Is that the idea?

MR. REGALADO. That does not require any concurrence by the legislature nor is it subject to
judicial review.
34

The reason for the difference in the treatment of the aforementioned powers highlights the intent to
grant the President the widest leeway and broadest discretion in using the power to call out because
it is considered as the lesser and more benign power compared to the power to suspend the
privilege of the writ of habeas corpus and the power to impose martial law, both of which involve the
curtailment and suppression of certain basic civil rights and individual freedoms, and thus
necessitating safeguards by Congress and review by this Court.

Moreover, under Section 18, Article VII of the Constitution, in the exercise of the power to suspend
the privilege of the writ of habeas corpus or to impose martial law, two conditions must concur: (1)
there must be an actual invasion or rebellion and, (2) public safety must require it. These conditions
are not required in the case of the power to call out the armed forces. The only criterion is that
"whenever it becomes necessary," the President may call the armed forces "to prevent or suppress
lawless violence, invasion or rebellion." The implication is that the President is given full discretion
and wide latitude in the exercise of the power to call as compared to the two other powers.
If the petitioner fails, by way of proof, to support the assertion that the President acted without factual
basis, then this Court cannot undertake an independent investigation beyond the pleadings. The
factual necessity of calling out the armed forces is not easily quantifiable and cannot be objectively
established since matters considered for satisfying the same is a combination of several factors
which are not always accessible to the courts. Besides the absence of textual standards that the
court may use to judge necessity, information necessary to arrive at such judgment might also prove
unmanageable for the courts. Certain pertinent information might be difficult to verify, or wholly
unavailable to the courts. In many instances, the evidence upon which the President might decide
that there is a need to call out the armed forces may be of a nature not constituting technical proof.

On the other hand, the President as Commander-in-Chief has a vast intelligence network to gather
information, some of which may be classified as highly confidential or affecting the security of the
state. In the exercise of the power to call, on-the-spot decisions may be imperatively necessary in
emergency situations to avert great loss of human lives and mass destruction of property. Indeed,
the decision to call out the military to prevent or suppress lawless violence must be done swiftly and
decisively if it were to have any effect at all. Such a scenario is not farfetched when we consider the
present situation in Mindanao, where the insurgency problem could spill over the other parts of the
country. The determination of the necessity for the calling out power if subjected to unfettered
judicial scrutiny could be a veritable prescription for disaster, as such power may be unduly
straitjacketed by an injunction or a temporary restraining order every time it is exercised.

Thus, it is the unclouded intent of the Constitution to vest upon the President, as Commander-in-
Chief of the Armed Forces, full discretion to call forth the military when in his judgment it is
necessary to do so in order to prevent or suppress lawless violence, invasion or rebellion. Unless the
petitioner can show that the exercise of such discretion was gravely abused, the President’s
exercise of judgment deserves to be accorded respect from this Court.

The President has already determined the necessity and factual basis for calling the armed forces.
In his Memorandum, he categorically asserted that, "[V]iolent crimes like bank/store robberies,
holdups, kidnappings and carnappings continue to occur in Metro Manila..." We do not doubt the
35 

veracity of the President’s assessment of the situation, especially in the light of present
developments. The Court takes judicial notice of the recent bombings perpetrated by lawless
elements in the shopping malls, public utilities, and other public places. These are among the areas
of deployment described in the LOI 2000. Considering all these facts, we hold that the President has
sufficient factual basis to call for military aid in law enforcement and in the exercise of this
constitutional power.

The deployment of the Marines does not violate the civilian supremacy clause nor does it infringe
the civilian character of the police force.

Prescinding from its argument that no emergency situation exists to justify the calling of the Marines,
the IBP asserts that by the deployment of the Marines, the civilian task of law enforcement is
"militarized" in violation of Section 3, Article II of the Constitution.
36 

We disagree. The deployment of the Marines does not constitute a breach of the civilian supremacy
clause. The calling of the Marines in this case constitutes permissible use of military assets for
civilian law enforcement. The participation of the Marines in the conduct of joint visibility patrols is
appropriately circumscribed. The limited participation of the Marines is evident in the provisions of
the LOI itself, which sufficiently provides the metes and bounds of the Marines’ authority. It is
noteworthy that the local police forces are the ones in charge of the visibility patrols at all times, the
real authority belonging to the PNP. In fact, the Metro Manila Police Chief is the overall leader of the
PNP-Philippine Marines joint visibility patrols. Under the LOI, the police forces are tasked to brief or
37 
orient the soldiers on police patrol procedures. It is their responsibility to direct and manage the
38 

deployment of the Marines. It is, likewise, their duty to provide the necessary equipment to the
39 

Marines and render logistical support to these soldiers. In view of the foregoing, it cannot be
40 

properly argued that military authority is supreme over civilian authority. Moreover, the deployment
of the Marines to assist the PNP does not unmake the civilian character of the police force. Neither
does it amount to an "insidious incursion" of the military in the task of law enforcement in violation of
Section 5(4), Article XVI of the Constitution. 41

In this regard, it is not correct to say that General Angelo Reyes, Chief of Staff of the AFP, by his
alleged involvement in civilian law enforcement, has been virtually appointed to a civilian post in
derogation of the aforecited provision. The real authority in these operations, as stated in the LOI, is
lodged with the head of a civilian institution, the PNP, and not with the military. Such being the case,
it does not matter whether the AFP Chief actually participates in the Task Force Tulungan since he
does not exercise any authority or control over the same. Since none of the Marines was
incorporated or enlisted as members of the PNP, there can be no appointment to civilian position to
speak of. Hence, the deployment of the Marines in the joint visibility patrols does not destroy the
civilian character of the PNP.

Considering the above circumstances, the Marines render nothing more than assistance required in
conducting the patrols. As such, there can be no "insidious incursion" of the military in civilian affairs
nor can there be a violation of the civilian supremacy clause in the Constitution.

It is worth mentioning that military assistance to civilian authorities in various forms persists in
Philippine jurisdiction. The Philippine experience reveals that it is not averse to requesting the
assistance of the military in the implementation and execution of certain traditionally "civil" functions.
As correctly pointed out by the Solicitor General, some of the multifarious activities wherein military
aid has been rendered, exemplifying the activities that bring both the civilian and the military together
in a relationship of cooperation, are:

1. Elections; 42

2. Administration of the Philippine National Red Cross; 43

3. Relief and rescue operations during calamities and disasters; 44

4. Amateur sports promotion and development; 45

5. Development of the culture and the arts; 46

6. Conservation of natural resources; 47

7. Implementation of the agrarian reform program; 48

8. Enforcement of customs laws; 49

9. Composite civilian-military law enforcement activities; 50

10. Conduct of licensure examinations; 51

11. Conduct of nationwide tests for elementary and high school students; 52
12. Anti-drug enforcement activities; 53

13. Sanitary inspections; 54

14. Conduct of census work; 55

15. Administration of the Civil Aeronautics Board; 56

16. Assistance in installation of weather forecasting devices; 57

17. Peace and order policy formulation in local government units. 58

This unquestionably constitutes a gloss on executive power resulting from a systematic, unbroken,
executive practice, long pursued to the knowledge of Congress and, yet, never before
questioned. What we have here is mutual support and cooperation between the military and civilian
59 

authorities, not derogation of civilian supremacy.

In the United States, where a long tradition of suspicion and hostility towards the use of military force
for domestic purposes has persisted, and whose Constitution, unlike ours, does not expressly
60 

provide for the power to call, the use of military personnel by civilian law enforcement officers is
allowed under circumstances similar to those surrounding the present deployment of the Philippine
Marines. Under the Posse Comitatus Act of the US, the use of the military in civilian law
61 

enforcement is generally prohibited, except in certain allowable circumstances. A provision of the Act
states:

§ 1385. Use of Army and Air Force as posse comitatus

Whoever, except in cases and under circumstances expressly authorized by the Constitution or Act
of Congress, willfully uses any part of the Army or the Air Force as posse comitatus or otherwise to
execute the laws shall be fined not more than $10,000 or imprisoned not more than two years, or
both. 62

To determine whether there is a violation of the Posse Comitatus Act in the use of military personnel,
the US courts apply the following standards, to wit:
63 

Were Army or Air Force personnel used by the civilian law enforcement officers at Wounded Knee in
such a manner that the military personnel subjected the citizens to the exercise of military power
which was regulatory, proscriptive, or compulsory George Washington Law Review, pp. 404-433
64 

(1986), which discusses the four divergent standards for assessing acceptable involvement of
military personnel in civil law enforcement. See likewise HONORED IN THE BREECH:
PRESIDENTIAL AUTHORITY TO EXECUTE THE LAWS WITH MILITARY FORCE, 83 Yale Law
Journal, pp. 130-152, 1973. 64 in nature, either presently or prospectively?

xxx

When this concept is transplanted into the present legal context, we take it to mean that military
involvement, even when not expressly authorized by the Constitution or a statute, does not violate
the Posse Comitatus Act unless it actually regulates, forbids or compels some conduct on the part of
those claiming relief.  A mere threat of some future injury would be insufficient. (emphasis supplied)
1âwphi1
Even if the Court were to apply the above rigid standards to the present case to determine whether
there is permissible use of the military in civilian law enforcement, the conclusion is inevitable that no
violation of the civilian supremacy clause in the Constitution is committed. On this point, the Court
agrees with the observation of the Solicitor General:

3. The designation of tasks in Annex A does not constitute the exercise of regulatory, proscriptive,
65 

or compulsory military power. First, the soldiers do not control or direct the operation. This is evident
from Nos. 6, 8(k) and 9(a) of Annex A. These soldiers, second, also have no power to prohibit or
66  67  68

condemn. In No. 9(d) of Annex A, all arrested persons are brought to the nearest police stations for
69 

proper disposition. And last, these soldiers apply no coercive force. The materials or equipment
issued to them, as shown in No. 8(c) of Annex A, are all low impact and defensive in character. The
70 

conclusion is that there being no exercise of regulatory, proscriptive or compulsory military power,
the deployment of a handful of Philippine Marines constitutes no impermissible use of military power
for civilian law enforcement. 71

It appears that the present petition is anchored on fear that once the armed forces are deployed, the
military will gain ascendancy, and thus place in peril our cherished liberties. Such apprehensions,
however, are unfounded. The power to call the armed forces is just that - calling out the armed
forces. Unless, petitioner IBP can show, which it has not, that in the deployment of the Marines, the
President has violated the fundamental law, exceeded his authority or jeopardized the civil liberties
of the people, this Court is not inclined to overrule the President’s determination of the factual basis
for the calling of the Marines to prevent or suppress lawless violence.

One last point. Since the institution of the joint visibility patrol in January, 2000, not a single citizen
has complained that his political or civil rights have been violated as a result of the deployment of the
Marines. It was precisely to safeguard peace, tranquility and the civil liberties of the people that the
joint visibility patrol was conceived. Freedom and democracy will be in full bloom only when people
feel secure in their homes and in the streets, not when the shadows of violence and anarchy
constantly lurk in their midst.

WHEREFORE, premises considered, the petition is hereby DISMISSED.

SO ORDERED.

G.R. No. 187298               July 03, 2012

JAMAR M. KULAYAN, TEMOGEN S. TULAWIE, HJI. MOH. YUSOP ISMI, JULHAJAN AWADI,
and SPO1 SATTAL H. JADJULI, Petitioners, 
vs.
GOV. ABDUSAKUR M. TAN, in his capacity as Governor of Sulu; GEN. JUANCHO SABAN,
COL. EUGENIO CLEMEN PN, P/SUPT. JULASIRIM KASIM and P/SUPT. BIENVENIDO G.
LATAG, in their capacity as officers of the Phil. Marines and Phil. National Police,
respectively, Respondents.

DECISION

SERENO, J.:
On 15 January 2009, three members from the International Committee of the Red Cross (ICRC)
were kidnapped in the vicinity of the Provincial Capitol in Patikul, Sulu. Andres Notter, a Swiss

national and head of the ICRC in Zamboanga City, Eugenio Vagni, an Italian national and ICRC
delegate, and Marie Jean Lacaba, a Filipino engineer, were purportedly inspecting a water and
sanitation project for the Sulu Provincial Jail when inspecting a water and sanitation project for the
Sulu Provincial Jail when they were seized by three armed men who were later confirmed to be
members of the Abu Sayyaf Group (ASG). The leader of the alleged kidnappers was identified as

Raden Abu, a former guard at the Sulu Provincial Jail. News reports linked Abu to Albader Parad,
one of the known leaders of the Abu Sayyaf.

On 21 January 2009, a task force was created by the ICRC and the Philippine National Police
(PNP), which then organized a parallel local group known as the Local Crisis Committee. The local

group, later renamed Sulu Crisis Management Committee, convened under the leadership of
respondent Abdusakur Mahail Tan, the Provincial Governor of Sulu. Its armed forces component
was headed by respondents General Juancho Saban, and his deputy, Colonel Eugenio Clemen. The
PNP component was headed by respondent Police Superintendent Bienvenido G. Latag, the Police
Deputy Director for Operations of the Autonomous Region of Muslim Mindanao (ARMM). 4

Governor Tan organized the Civilian Emergency Force (CEF), a group of armed male civilians
coming from different municipalities, who were redeployed to surrounding areas of Patikul. The 5 

organization of the CEF was embodied in a "Memorandum of Understanding" entered into


between three parties: the provincial government of Sulu, represented by Governor Tan; the Armed
Forces of the Philippines, represented by Gen. Saban; and the Philippine National Police,
represented by P/SUPT. Latag. The Whereas clauses of the Memorandum alluded to the
extraordinary situation in Sulu, and the willingness of civilian supporters of the municipal mayors to
offer their services in order that "the early and safe rescue of the hostages may be achieved." 7

This Memorandum, which was labeled ‘secret’ on its all pages, also outlined the responsibilities of
each of the party signatories, as follows:

Responsibilities of the Provincial Government:

1) The Provincial Government shall source the funds and logistics needed for the activation
of the CEF;

2) The Provincial Government shall identify the Local Government Units which shall
participate in the operations and to propose them for the approval of the parties to this
agreement;

3) The Provincial Government shall ensure that there will be no unilateral action(s) by the
CEF without the knowledge and approval by both parties.

Responsibilities of AFP/PNP/ TF ICRC (Task Force ICRC):

1) The AFP/PNP shall remain the authority as prescribed by law in military operations and
law enforcement;

2) The AFP/PNP shall ensure the orderly deployment of the CEF in the performance of their
assigned task(s);
3) The AFP/PNP shall ensure the safe movements of the CEF in identified areas of
operation(s);

4) The AFP/PNP shall provide the necessary support and/or assistance as called for in the
course of operation(s)/movements of the CEF. 8

Meanwhile, Ronaldo Puno, then Secretary of the Department of Interior and Local Government,
announced to the media that government troops had cornered some one hundred and twenty (120)
Abu Sayyaf members along with the three (3) hostages. However, the ASG made

contact with the authorities and demanded that the military pull its troops back from the jungle
area. The government troops yielded and went back to their barracks; the Philippine Marines
10 

withdrew to their camp, while police and civilian forces pulled back from the terrorists’ stronghold by
ten (10) to fifteen (15) kilometers. Threatening that one of the hostages will be beheaded, the ASG
further demanded the evacuation of the military camps and bases in the different barangays in
Jolo. The authorities were given no later than 2:00 o’clock in the afternoon of 31 March 2009 to
11 

comply. 12

On 31 March 2009, Governor Tan issued Proclamation No. 1, Series of 2009 (Proclamation 1-09),
declaring a state of emergency in the province of Sulu. It cited the kidnapping incident as a ground
13 

for the said declaration, describing it as a terrorist act pursuant to the Human Security

Act (R.A. 9372). It also invoked Section 465 of the Local Government Code of 1991 (R.A. 7160),
which bestows on the Provincial Governor the power to carry out emergency measures during man-
made and natural disasters and calamities, and to call upon the appropriate national law
enforcement agencies to suppress disorder and lawless violence.

In the same Proclamation, respondent Tan called upon the PNP and the CEF to set up checkpoints
and chokepoints, conduct general search and seizures including arrests, and other actions
necessary to ensure public safety. The pertinent portion of the proclamation states:

NOW, THEREFORE, BY VIRTUE OF THE POWERS VESTED IN ME BY LAW, I, ABDUSAKUR


MAHAIL TAN, GOVERNOR OF THE PROVINCE OF SULU, DO HEREBY DECLARE A STATE OF
EMERGENCY IN THE PROVINCE OF SULU, AND CALL ON THE PHILIPPINE NATIONAL
POLICE WITH THE ASSISTANCE OF THE ARMED FORCES OF THE PHILIPPINES AND THE
CIVILIAN EMERGENCY FORCE TO IMPLEMENT THE FOLLOWING:

1. The setting-up of checkpoints and chokepoints in the province;

2. The imposition of curfew for the entire province subject to such Guidelines as may be
issued by proper authorities;

3. The conduct of General Search and Seizure including arrests in the pursuit of the
kidnappers and their supporters; and

4. To conduct such other actions or police operations as may be necessary to ensure public
safety.

DONE AT THE PROVINCIAL CAPITOL, PROVINCE OF SULU THIS

31STDAY OF MARCH 2009. Sgd. Abdusakur M. Tan Governor. 14


On 1 April 2009, SPO1 Sattal Jadjuli was instructed by his superior to report to respondent P/SUPT.
Julasirim Kasim. Upon arriving at the police station, he was booked, and interviewed about his
15 

relationship to Musin, Jaiton, and Julamin, who were all his deceased relatives. Upon admitting that
he was indeed related to the three, he was detained. After a few hours, former Punong Barangay
Juljahan Awadi, Hadji Hadjirul Bambra, Abdugajir Hadjirul, as well as PO2 Marcial Hajan, SPO3
Muhilmi Ismula, Punong Barangay Alano Mohammad and jeepney driver Abduhadi Sabdani, were
also arrested. The affidavit of the apprehending officer alleged that they were suspected ASG
16  17 

supporters and were being arrested under Proclamation 1-09. The following day, 2 April 2009, the
hostage Mary Jane Lacaba was released by the ASG.

On 4 April 2009, the office of Governor Tan distributed to civic organizations, copies of the
"Guidelines for the Implementation of Proclamation No. 1, Series of 2009 Declaring a State of
Emergency in the Province of Sulu." These Guidelines suspended all Permits to Carry
18

Firearms Outside of Residence (PTCFORs) issued by the Chief of the PNP, and allowed civilians to
seek exemption from the gun ban only by applying to the Office of the Governor and obtaining the
appropriate identification cards. The said guidelines also allowed general searches and seizures in
designated checkpoints and chokepoints.

On 16 April 2009, Jamar M. Kulayan, Temogen S. Tulawie, Hadji Mohammad Yusop Ismi, Ahajan
Awadi, and SPO1 Sattal H. Jadjuli, residents of Patikul, Sulu, filed the present Petition for Certiorari
and Prohibition, claiming that Proclamation 1-09 was issued with grave abuse of discretion
19 

amounting to lack or excess of jurisdiction, as it threatened fundamental freedoms guaranteed under


Article III of the 1987 Constitution.

Petitioners contend that Proclamation No. 1 and its Implementing Guidelines were issued ultra vires,
and thus null and void, for violating Sections 1 and 18, Article VII of the Constitution, which grants
the President sole authority to exercise emergency powers and calling-out powers as the chief
executive of the Republic and commander-in-chief of the armed forces. Additionally, petitioners
20 

claim that the Provincial Governor is not authorized by any law to create civilian armed forces under
his command, nor regulate and limit the issuances of PTCFORs to his own private army.

In his Comment, Governor Tan contended that petitioners violated the doctrine on hierarchy of
courts when they filed the instant petition directly in the court of last resort, even if both the Court of
Appeals (CA) and the Regional Trial Courts (RTC) possessed concurrent jurisdiction with the

Supreme Court under Rule 65. This is the only procedural defense raised by respondent Tan.
21 

Respondents Gen. Juancho Saban, Col. Eugenio Clemen, P/SUPT. Julasirim Kasim, and P/SUPT.
Bienvenido Latag did not file their respective Comments. 1âwphi1

On the substantive issues, respondents deny that Proclamation 1-09 was issued ultra vires, as
Governor Tan allegedly acted pursuant to Sections 16 and 465 of the Local Government Code,
which empowers the Provincial Governor to carry out emergency measures during calamities and
disasters, and to call upon the appropriate national law enforcement agencies to suppress disorder,
riot, lawless violence, rebellion or sedition. Furthermore, the Sangguniang Panlalawigan of Sulu
22 

authorized the declaration of a state of emergency as evidenced by Resolution No. 4, Series of 2009
issued on 31 March 2009 during its regular session. 23

The threshold issue in the present case is whether or not Section 465, in relation to Section 16, of
the Local Government Code authorizes the respondent governor to declare a state of emergency,
and exercise the powers enumerated under Proclamation 1-09, specifically the conduct of general
searches and seizures. Subsumed herein is the secondary question of whether or not the provincial
governor is similarly clothed with authority to convene the CEF under the said provisions.

We grant the petition.

I. Transcendental public Importance warrants a relaxation of the Doctrine of Hierarchy of Courts

We first dispose of respondents’ invocation of the doctrine of hierarchy of courts which allegedly
prevents judicial review by this Court in the present case, citing for this specific purpose, Montes v.
Court of Appeals and Purok Bagong Silang Association, Inc. v. Yuipco. Simply put, the
24 

doctrine provides that where the issuance of an extraordinary writ is also within the competence of
the CA or the RTC, it is in either of these courts and not in the Supreme Court, that the specific
action for the issuance of such writ must be sought unless special and important laws are clearly and
specifically set forth in the petition. The reason for this is that this Court is a court of last resort and
must so remain if it is to perform the functions assigned to it by the Constitution and immemorial
tradition. It cannot be burdened with deciding cases in the first instance. 25

The said rule, however, is not without exception. In Chavez v. PEA-Amari, the Court stated:
26 

PEA and AMARI claim petitioner ignored the judicial hierarchy by seeking relief directly from the
Court. The principle of hierarchy of courts applies generally to cases involving factual questions. As
it is not a trier of facts, the Court cannot entertain cases involving factual issues. The instant case,
however, raises constitutional questions of transcendental importance to the public. The Court can
resolve this case without determining any factual issue related to the case. Also, the instant case is a
petition for mandamus which falls under the original jurisdiction of the Court under Section 5, Article
VIII of the Constitution. We resolve to exercise primary jurisdiction over the instant case. 27

The instant case stems from a petition for certiorari and prohibition, over which the Supreme Court
possesses original jurisdiction. More crucially, this case involves acts of a public official which
28 

pertain to restrictive custody, and is thus impressed with transcendental public importance that
would warrant the relaxation of the general rule. The Court would be remiss in its constitutional
duties were it to dismiss the present petition solely due to claims of judicial hierarchy.

In David v. Macapagal-Arroyo, the Court highlighted the transcendental public importance involved
29 

in cases that concern restrictive custody, because judicial review in these cases serves as "a
manifestation of the crucial defense of civilians ‘in police power’ cases due to the diminution of their
basic liberties under the guise of a state of emergency." Otherwise, the importance of the high
30 

tribunal as the court of last resort would be put to naught, considering the nature of "emergency"
cases, wherein the proclamations and issuances are inherently short-lived. In finally disposing of the
claim that the issue had become moot and academic, the Court also cited transcendental public
importance as an exception, stating:

Sa kabila ng pagiging akademiko na lamang ng mga isyu tungkol sa mahigpit na pangangalaga


(restrictive custody) at pagmonitor ng galaw (monitoring of movements) ng nagpepetisyon,
dedesisyunan namin ito (a) dahil sa nangingibabaw na interes ng madla na nakapaloob dito,

(b) dahil sa posibilidad na maaaring maulit ang pangyayari at (c) dahil kailangang maturuan ang
kapulisan tungkol dito.
The moot and academic principle is not a magical formula that can automatically dissuade the courts
in resolving a case. Courts will decide cases, otherwise moot and academic, if: first, there is a grave
violation of the Constitution; second, the exceptional character of the situation and the paramount
public interest is involved; third, when [the] constitutional issue raised requires formulation of
controlling principles to guide the bench, the bar, and the public; and fourth, the case is capable of
repetition yet evading review.

…There is no question that the issues being raised affect the public interest, involving as they do the
people’s basic rights to freedom of expression, of assembly and of the press. Moreover, the

Court has the duty to formulate guiding and controlling constitutional precepts, doctrines or rules. It
has the symbolic function of educating the bench and the bar, and in the present petitions, the
military and the police, on the extent of the protection given by constitutional guarantees. And lastly,
respondents contested actions are capable of repetition. Certainly, the petitions are subject to
judicial review.

Evidently, the triple reasons We advanced at the start of Our ruling are justified under the foregoing
exceptions. Every bad, unusual incident where police officers figure in generates public interest and
people watch what will be done or not done to them. Lack of disciplinary steps taken against them
erode public confidence in the police institution. As petitioners themselves assert, the restrictive
custody of policemen under investigation is an existing practice, hence, the issue is bound to crop up
every now and then. The matter is capable of repetition or susceptible of recurrence. It better be
resolved now for the education and guidance of all concerned. (Emphasis supplied)
31 

Hence, the instant petition is given due course, impressed as it is with transcendental public
importance.

II. Only the President is vested with calling-out powers, as the commander-in-chief of the Republic

i. One executive, one commander-in-chief

As early as Villena v. Secretary of Interior, it has already been established that there is one
32 

repository of executive powers, and that is the President of the Republic. This means that when
Section 1, Article VII of the Constitution speaks of executive power, it is granted to the President and
no one else. As emphasized by Justice Jose P. Laurel, in his ponencia in Villena:
33 

With reference to the Executive Department of the government, there is one purpose which is
crystal-clear and is readily visible without the projection of judicial searchlight, and that is the
establishment of a single, not plural, Executive. The first section of Article VII of the Constitution,
dealing with the Executive Department, begins with the enunciation of the principle that "The
executive power shall be vested in a President of the Philippines." This means that the President of
the Philippines is the Executive of the Government of the Philippines, and no other. 34

Corollarily, it is only the President, as Executive, who is authorized to exercise emergency powers as
provided under Section 23, Article VI, of the Constitution, as well as what became known as the
calling-out powers under Section 7, Article VII thereof.

ii. The exceptional character of Commander-in-Chief powers dictate that they are exercised by one
president
Springing from the well-entrenched constitutional precept of One President is the notion that there
are certain acts which, by their very nature, may only be performed by the president as the Head of
the State. One of these acts or prerogatives is the bundle of Commander-in-Chief powers to which
the "calling-out" powers constitutes a portion. The President’s Emergency Powers, on the other
hand, is balanced only by the legislative act of Congress, as embodied in the second paragraph of
Section 23, Article 6 of the Constitution:

Article 6, Sec 23(2). In times of war or other national emergency, the Congress may, by law,
authorize the President, for a limited period and subject to such restrictions as it may prescribe, to
exercise powers necessary and proper to carry out a declared national policy. Unless sooner
withdrawn by resolution of the Congress, such powers shall cease upon the next adjournment
thereof.35

Article 7, Sec 18. The President shall be the Commander-in-Chief of all armed forces of the
Philippines and whenever it becomes necessary, he may call out such armed forces to prevent or
suppress lawless violence, invasion or rebellion. In case of invasion or rebellion, when the public
safety requires it, he may, for a period not exceeding sixty days, suspend the privilege of the writ of
habeas corpus or place the Philippines or any part thereof under martial law. Within forty-eight hours
from the proclamation of martial law or the suspension of the privilege of the writ of habeas corpus,
the President shall submit a report in person or in writing to the Congress. The Congress, voting
jointly, by a vote of at least a majority of all its Members in regular or special session, may revoke
such proclamation or suspension, which revocation shall not be set aside by the President. Upon the
initiative of the President, the Congress may, in the same manner, extend such proclamation or
suspension for a period to be determined by the Congress, if the invasion or rebellion shall persist
and public safety requires it.

The Congress, if not in session, shall, within twenty-four hours following such proclamation or
suspension, convene in accordance with its rules without need of a call. 36

The power to declare a state of martial law is subject to the Supreme Court’s authority to review the
factual basis thereof.  By constitutional fiat, the calling-out powers, which is of lesser gravity than the
37 

power to declare martial law, is bestowed upon the President alone. As noted in Villena, "(t)here are
certain constitutional powers and prerogatives of the Chief Executive of the Nation which must be
exercised by him in person and no amount of approval or ratification will validate the exercise of any
of those powers by any other person. Such, for instance, is his power to suspend the writ of habeas
corpus and proclaim martial law x x x. 38

Indeed, while the President is still a civilian, Article II, Section 3 of the Constitution mandates that
39 

civilian authority is, at all times, supreme over the military, making the civilian president the nation’s
supreme military leader. The net effect of Article II, Section 3, when read with Article VII,

Section 18, is that a civilian President is the ceremonial, legal and administrative head of the armed
forces. The Constitution does not require that the President must be possessed of military training
and talents, but as Commander-in-Chief, he has the power to direct military operations and to
determine military strategy. Normally, he would be expected to delegate the actual command of the
armed forces to military experts; but the ultimate power is his. As Commander-in-Chief, he is
40 

authorized to direct the movements of the naval and military forces placed by law at his command,
and to employ them in the manner he may deem most effectual. 41

In the case of Integrated Bar of the Philippines v. Zamora, the Court had occasion to rule that the
42 

calling-out powers belong solely to the President as commander-in-chief:


When the President calls the armed forces to prevent or suppress lawless violence, invasion or
rebellion, he necessarily exercises a discretionary power solely vested in his wisdom. This is clear
from the intent of the framers and from the text of the Constitution itself. The Court, thus, cannot be
called upon to overrule the President’s wisdom or substitute its own. However, this does not prevent
an examination of whether such power was exercised within permissible constitutional limits or
whether it was exercised in a manner constituting grave abuse of discretion. In view of the
constitutional intent to give the President full discretionary power to determine the necessity of
calling out the armed forces, it is incumbent upon the petitioner to show that the President’s decision
is totally bereft of factual basis.

There is a clear textual commitment under the Constitution to bestow on the President full
discretionary power to call out the armed forces and to determine the necessity for the exercise of
such power. (Emphasis supplied)
43

Under the foregoing provisions, Congress may revoke such proclamation or suspension and the
Court may review the sufficiency of the factual basis thereof. However, there is no such equivalent
provision dealing with the revocation or review of the President’s action to call out the armed forces.
The distinction places the calling out power in a different category from the power to declare martial
law and the power to suspend the privilege of the writ of habeas corpus, otherwise, the framers of
the Constitution would have simply lumped together the three powers and provided for their
revocation and review without any qualification.44

That the power to call upon the armed forces is discretionary on the president is clear from the
deliberation of the Constitutional Commission:

FR. BERNAS. It will not make any difference. I may add that there is a graduated power of the
President as Commander-in-Chief. First, he can call out such Armed Forces as may be necessary to
suppress lawless violence; then he can suspend the privilege of the writ of habeas corpus, then he
can impose martial law. This is a graduated sequence.

When he judges that it is necessary to impose martial law or suspend the privilege of the writ of
habeas corpus, his judgment is subject to review. We are making it subject to review by the
Supreme Court and subject to concurrence by the National Assembly. But when he exercises this
lesser power of calling on the Armed Forces, when he says it is necessary, it is my opinion that his
judgment cannot be reviewed by anybody.

x x x           x x x          x x x

MR. REGALADO. That does not require any concurrence by the legislature nor is it subject to
judicial review.

The reason for the difference in the treatment of the aforementioned powers highlights the intent to
grant the President the widest leeway and broadest discretion in using the power to call out because
it is considered as the lesser and more benign power compared to the power to suspend the
privilege of the writ of habeas corpus and the power to impose martial law, both of which involve the
curtailment and suppression of certain basic civil rights and individual freedoms, and thus
necessitating safeguards by Congress and review by this Court.

x x x Thus, it is the unclouded intent of the Constitution to vest upon the President, as Commander-
in-Chief of the Armed Forces, full discretion to call forth the military when in his judgment it is
necessary to do so in order to prevent or suppress lawless violence, invasion or
rebellion. (Emphasis Supplied)
45 
In the more recent case of Constantino, Jr. v. Cuisia, the Court characterized these powers as
46 

exclusive to the President, precisely because they are of exceptional import:

These distinctions hold true to this day as they remain embodied in our fundamental law. There are
certain presidential powers which arise out of exceptional circumstances, and if exercised, would
involve the suspension of fundamental freedoms, or at least call for the supersedence of executive
prerogatives over those exercised by co-equal branches of government. The declaration of martial
law, the suspension of the writ of habeas corpus, and the exercise of the pardoning power,
notwithstanding the judicial determination of guilt of the accused, all fall within this special class that
demands the exclusive exercise by the President of the constitutionally vested power. The list is by
no means exclusive, but there must be a showing that the executive power in question is of
similar gravitas and exceptional import. 47

In addition to being the commander-in-chief of the armed forces, the President also acts as the
leader of the country’s police forces, under the mandate of Section 17, Article VII of the Constitution,
which provides that, "The President shall have control of all the executive departments, bureaus, and
offices. He shall ensure that the laws be faithfully executed." During the deliberations of the
Constitutional Commission on the framing of this provision, Fr. Bernas defended the retention of the
word "control," employing the same rationale of singularity of the office of the president, as the only
Executive under the presidential form of government. 48

Regarding the country’s police force, Section 6, Article XVI of the Constitution states that: "The State
shall establish and maintain one police force, which shall be national in scope and civilian in
character, to be administered and controlled by a national police commission. The authority of local
executives over the police units in their jurisdiction shall be provided by law."49

A local chief executive, such as the provincial governor, exercises operational supervision over the
police, and may exercise control only in day-to-day operations, viz:
50 

Mr. Natividad: By experience, it is not advisable to provide either in our Constitution or by law full
control of the police by the local chief executive and local executives, the mayors. By our
experience, this has spawned warlordism, bossism and sanctuaries for vices and abuses. If the
national government does not have a mechanism to supervise these 1,500 legally, technically
separate police forces, plus 61 city police forces, fragmented police system, we will have a lot of
difficulty in presenting a modern professional police force. So that a certain amount of supervision
and control will have to be exercised by the national government.

For example, if a local government, a town cannot handle its peace and order problems or police
problems, such as riots, conflagrations or organized crime, the national government may come in,
especially if requested by the local executives. Under that situation, if they come in under such an
extraordinary situation, they will be in control. But if the day-to-day business of police investigation of
crime, crime prevention, activities, traffic control, is all lodged in the mayors, and if they are in
complete operational control of the day-to-day business of police service, what the national
government would control would be the administrative aspect.

x x x           x x x          x x x

Mr. de los Reyes: so the operational control on a day-to-day basis, meaning, the usual duties being
performed by the ordinary policemen, will be under the supervision of the local executives?

Mr. Natividad: Yes, Madam President.


x x x           x x x          x x x

Mr. de los Reyes: But in exceptional cases, even the operational control can be taken over by the
National Police Commission?

Mr. Natividad: If the situation is beyond the capacity of the local governments. (Emphases supplied)
51 

Furthermore according to the framers, it is still the President who is authorized to exercise
supervision and control over the police, through the National Police Commission:

Mr. Rodrigo: Just a few questions. The President of the Philippines is the Commander-in-Chief of all
the armed forces.

Mr. Natividad: Yes, Madam President.

Mr. Rodrigo: Since the national police is not integrated with the armed forces, I do not suppose they
come under the Commander-in-Chief powers of the President of the Philippines.

Mr. Natividad: They do, Madam President. By law, they are under the supervision and control of the
President of the Philippines.

Mr. Rodrigo: Yes, but the President is not the Commander-in-Chief of the national police.

Mr. Natividad: He is the President.

Mr. Rodrigo: Yes, the Executive. But they do not come under that specific provision that the
President is the Commander-in-Chief of all the armed forces.

Mr. Natividad: No, not under the Commander-in-Chief provision.

Mr. Rodrigo: There are two other powers of the President. The

President has control over ministries, bureaus and offices, and supervision over local governments.
Under which does the police fall, under control or under supervision?

Mr. Natividad: Both, Madam President.

Mr. Rodrigo: Control and supervision.

Mr. Natividad: Yes, in fact, the National Police Commission is under the Office of the President. 52

In the discussions of the Constitutional Commission regarding the above provision it is clear that the
framers never intended for local chief executives to exercise unbridled control over the police in
emergency situations. This is without prejudice to their authority over police units in their jurisdiction
as provided by law, and their prerogative to seek assistance from the police in day to day situations,
as contemplated by the Constitutional Commission. But as a civilian agency of the government, the
police, through the NAPOLCOM, properly comes within, and is subject to, the exercise by the
President of the power of executive control. 53

iii. The provincial governor does not possess the same calling-out powers as the President
Given the foregoing, respondent provincial governor is not endowed with the power to call upon the
armed forces at his own bidding. In issuing the assailed proclamation, Governor Tan exceeded his
authority when he declared a state of emergency and called upon the Armed Forces, the police, and
his own Civilian Emergency Force. The calling-out powers contemplated under the Constitution is
exclusive to the President. An exercise by another official, even if he is the local chief executive, is
ultra vires, and may not be justified by the invocation of Section 465 of the Local Government Code,
as will be discussed subsequently.

Respondents, however, justify this stance by stating that nowhere in the seminal case of David v.
Arroyo, which dealt squarely with the issue of the declaration of a state of emergency, does it limit
the said authority to the President alone. Respondents contend that the ruling in David expressly
limits the authority to declare a national emergency, a condition which covers the entire country, and
does not include emergency situations in local government units. This claim is belied by the clear
54 

intent of the framers that in all situations involving threats to security, such as lawless violence,
invasion or rebellion, even in localized areas, it is still the President who possesses the sole
authority to exercise calling-out powers. As reflected in the Journal of the Constitutional
Commission:

Thereafter, Mr. Padilla proposed on line 29 to insert the phrase OR PUBLIC DISORDER in lieu of
"invasion or rebellion." Mr. Sumulong stated that the committee could not accept the amendment
because under the first section of Section 15, the President may call out and make use of the armed
forces to prevent or suppress not only lawless violence but even invasion or rebellion without
declaring martial law. He observed that by deleting "invasion or rebellion" and substituting PUBLIC
DISORDER, the President would have to declare martial law before he can make use of the armed
forces to prevent or suppress lawless invasion or rebellion.

Mr. Padilla, in reply thereto, stated that the first sentence contemplates a lighter situation where
there is some lawless violence in a small portion of the country or public disorder in another at which
times, the armed forces can be called to prevent or suppress these incidents. He noted that the
Commander-in-Chief can do so in a minor degree but he can also exercise such powers should the
situation worsen. The words "invasion or rebellion" to be eliminated on line 14 are covered by the
following sentence which provides for "invasion or rebellion." He maintained that the proposed
amendment does not mean that under such circumstances, the President cannot call on the armed
forces to prevent or suppress the same. (Emphasis supplied)
55 

III. Section 465 of the Local

Government Code cannot be invoked to justify the powers enumerated under Proclamation 1-09

Respondent governor characterized the kidnapping of the three ICRC workers as a terroristic act,
and used this incident to justify the exercise of the powers enumerated under Proclamation 1-09. He56 

invokes Section 465, in relation to Section 16, of the Local Government Code, which purportedly
allows the governor to carry out emergency measures and call upon the appropriate national law
enforcement agencies for assistance. But a closer look at the said proclamation shows that there is
no provision in the Local Government Code nor in any law on which the broad and unwarranted
powers granted to the Governor may be based.

Petitioners cite the implementation of "General Search and Seizure including arrests in the pursuit of
the kidnappers and their supporters," as being violative of the constitutional proscription on general
57 

search warrants and general seizures. Petitioners rightly assert that this alone would be sufficient to
render the proclamation void, as general searches and seizures are proscribed, for being violative of
the rights enshrined in the Bill of Rights, particularly:
The right of the people to be secure in their persons, houses, papers, and effects against
unreasonable searches and seizures of whatever nature and for any purpose shall be inviolable, and
no search warrant or warrant of arrest shall issue except upon probable cause to be determined
personally by the judge after examination under oath or affirmation of the complainant and the
witnesses he may produce, and particularly describing the place to be searched and the persons or
things to be seized. 58

In fact, respondent governor has arrogated unto himself powers exceeding even the martial law
powers of the President, because as the Constitution itself declares, "A state of martial law does not
suspend the operation of the Constitution, nor supplant the functioning of the civil courts or
legislative assemblies, nor authorize the conferment of the jurisdiction on military courts and
agencies over civilians where civil courts are able to function, nor automatically suspend the
privilege of the writ." 59

We find, and so hold, that there is nothing in the Local Government Code which justifies the acts
sanctioned under the said Proclamation. Not even Section 465 of the said Code, in relation to
Section 16, which states:

Section 465. The Chief Executive: Powers, Duties, Functions, and Compensation.

x x x           x x x          x x x

(b) For efficient, effective and economical governance the purpose of which is the general welfare of
the province and its inhabitants pursuant to Section 16 of this Code, the provincial governor shall:

(1) Exercise general supervision and control over all programs, projects, services, and activities of
the provincial government, and in this connection, shall:

x x x           x x x          x x x

(vii) Carry out such emergency measures as may be necessary during and in the aftermath of man-
made and natural disasters and calamities;

(2) Enforce all laws and ordinances relative to the governance of the province and the exercise of
the appropriate corporate powers provided for under Section 22 of this Code, implement all
approved policies, programs, projects, services and activities of the province and, in addition to the
foregoing, shall:

x x x           x x x          x x x

(vi) Call upon the appropriate national law enforcement agencies to suppress disorder, riot, lawless
violence, rebellion or sedition or to apprehend violators of the law when public interest so requires
and the police forces of the component city or municipality where the disorder or violation is
happening are inadequate to cope with the situation or the violators.

Section 16. General Welfare. - Every local government unit shall exercise the powers expressly
granted, those necessarily implied therefrom, as well as powers necessary, appropriate, or incidental
for its efficient and effective governance, and those which are essential to the promotion of the
general welfare. Within their respective territorial jurisdictions, local government units shall ensure
and support, among other things, the preservation and enrichment of culture, promote health and
safety, enhance the right of the people to a balanced ecology, encourage and support the
development of appropriate and self-reliant scientific and technological capabilities, improve public
morals, enhance economic prosperity and social justice, promote full employment among their
residents, maintain peace and order, and preserve the comfort and convenience of their inhabitants.
(Emphases supplied)

Respondents cannot rely on paragraph 1, subparagraph (vii) of Article 465 above, as the said
provision expressly refers to calamities and disasters, whether man-made or natural. The governor,
as local chief executive of the province, is certainly empowered to enact and implement emergency
measures during these occurrences. But the kidnapping incident in the case at bar cannot be
considered as a calamity or a disaster. Respondents cannot find any legal mooring under this
provision to justify their actions.

Paragraph 2, subparagraph (vi) of the same provision is equally inapplicable for two reasons. First,
the Armed Forces of the Philippines does not fall under the category of a "national law enforcement
agency," to which the National Police Commission (NAPOLCOM) and its departments belong.

Its mandate is to uphold the sovereignty of the Philippines, support the Constitution, and defend the
Republic against all enemies, foreign and domestic. Its aim is also to secure the integrity of the
national territory.
60

Second, there was no evidence or even an allegation on record that the local police forces were
inadequate to cope with the situation or apprehend the violators. If they were inadequate, the
recourse of the provincial governor was to ask the assistance of the Secretary of Interior and Local
Government, or such other authorized officials, for the assistance of national law enforcement
agencies.

The Local Government Code does not involve the diminution of central powers inherently vested in
the National Government, especially not the prerogatives solely granted by the Constitution to the
President in matters of security and defense.

The intent behind the powers granted to local government units is fiscal, economic, and
administrative in nature. The Code is concerned only with powers that would make the delivery of
1âwphi1

basic services more effective to the constituents, and should not be unduly stretched to confer
61 

calling-out powers on local executives.

In the sponsorship remarks for Republic Act 7160, it was stated that the devolution of powers is a
step towards the autonomy of local government units (LGUs), and is actually an experiment whose
success heavily relies on the power of taxation of the LGUs. The underpinnings of the Code can be
found in Section 5, Article II of the 1973 Constitution, which allowed LGUs to create their own
sources of revenue. During the interpellation made by Mr. Tirol addressed to Mr. de Pedro, the latter
62 

emphasized that "Decentralization is an administrative concept and the process of shifting and
delegating power from a central point to subordinate levels to promote independence, responsibility,
and quicker decision-making. … (I)t does not involve any transfer of final authority from the national
to field levels, nor diminution of central office powers and responsibilities. Certain government
agencies, including the police force, are exempted from the decentralization process because their
functions are not inherent in local government units." 63

IV. Provincial governor is not authorized to convene CEF

Pursuant to the national policy to establish one police force, the organization of private citizen armies
is proscribed. Section 24 of Article XVIII of the Constitution mandates that:
Private armies and other armed groups not recognized by duly constituted authority shall be
dismantled. All paramilitary forces including Civilian Home Defense Forces (CHDF) not consistent
with the citizen armed force established in this Constitution, shall be dissolved or, where appropriate,
converted into the regular force.

Additionally, Section 21of Article XI states that, "The preservation of peace and order within the
regions shall be the responsibility of the local police agencies which shall be organized, maintained,
supervised, and utilized in accordance with applicable laws. The defense and security of the regions
shall be the responsibility of the National Government."

Taken in conjunction with each other, it becomes clear that the Constitution does not authorize the
organization of private armed groups similar to the CEF convened by the respondent Governor. The
framers of the Constitution were themselves wary of armed citizens’ groups, as shown in the
following proceedings:

MR. GARCIA: I think it is very clear that the problem we have here is a paramilitary force operating
under the cloak, under the mantle of legality is creating a lot of problems precisely by being able to
operate as an independent private army for many regional warlords. And at the same time, this I
think has been the thrust, the intent of many of the discussions and objections to the paramilitary
units and the armed groups.

MR. PADILLA: My proposal covers two parts: the private armies of political warlords and other
armed torces not recognized by constituted authority which shall be dismantled and dissolved. In my
trips to the provinces, I heard of many abuses committed by the CHDF (Civilian Home Defense
Forces), specially in Escalante, Negros Occidental. But I do not know whether a particular CHDF is
approved or authorized by competent authority. If it is not authorized, then the CHDF will have to be
dismantled. If some CHDFs, say in other provinces, are authorized by constituted authority, by the
Armed Forces of the Philippines, through the Chief of Staff or the Minister of National Defense, if
they are recognized and authorized, then they will not be dismantled. But I cannot give a categorical
answer to any specific CHDF unit, only the principle that if they are armed forces which are not
authorized, then they should be dismantled.  (Emphasis supplied)
64 

Thus, with the discussions in the Constitutional Commission as guide, the creation of the Civilian
Emergency Force (CEF) in the present case, is also invalid.

WHEREFORE, the instant petition is GRANTED. Judgment is rendered commanding respondents to


desist from further proceedings m implementing Proclamation No. 1, Series of 2009, and its
Implementing Guidelines. The said proclamation and guidelines are hereby declared NULL and
VOID for having been issued in grave abuse of discretion, amounting to lack or excess of
jurisdiction.

SO ORDERED.

G.R. No. 170165               August 15, 2006

B/GEN. (RET.) FRANCISCO V. GUDANI AND LT. COL. ALEXANDER F. BALUTAN Petitioners, 


vs.
LT./GEN. GENEROSO S. SENGA CORONA, AS CHIEF OF STAFF OF THE CARPIO-MORALES,
ARMED FORCES OF THE CALLEJO, SR., PHILIPPINES, COL. GILBERTO AZCUNA, JOSE C.
ROA AS THE PRE-TRIAL TINGA, INVESTIGATING OFFICER, THE CHICO-NAZARIO, PROVOST
MARSHALL GENERAL GARCIA, and OF THE ARMED FORCES OF THE PHILIPPINES AND
THE GENERAL COURT-MARTIAL, Respondents.

DECISION

TINGA, J.:

A most dangerous general proposition is foisted on the Court — that soldiers who defy orders of
their superior officers are exempt

from the strictures of military law and discipline if such defiance is predicated on an act otherwise
valid under civilian law. Obedience and deference to the military chain of command and the
President as commander-in-chief are the cornerstones of a professional military in the firm cusp of
civilian control. These values of obedience and deference expected of military officers are content-
neutral, beyond the sway of the officer’s own sense of what is prudent or rash, or more elementally,
of right or wrong. A self-righteous military invites itself as the scoundrel’s activist solution to the "ills"
of participatory democracy.

Petitioners seek the annulment of a directive from President Gloria Macapagal-Arroyo1 enjoining


them and other military officers from testifying before Congress without the President’s consent.
Petitioners also pray for injunctive relief against a pending preliminary investigation against them, in
preparation for possible court-martial proceedings, initiated within the military justice system in
connection with petitioners’ violation of the aforementioned directive.

The Court is cognizant that petitioners, in their defense, invoke weighty constitutional principles that
center on fundamental freedoms enshrined in the Bill of Rights. Although these concerns will not be

addressed to the satisfaction of petitioners, the Court recognizes these values as of paramount
importance to our civil society, even if not determinative of the resolution of this petition. Had the
relevant issue before us been the right of the Senate to compel the testimony of petitioners, the
constitutional questions raised by them would have come to fore. Such a scenario could have very
well been presented to the Court in such manner, without the petitioners having had to violate a
direct order from their commanding officer. Instead, the Court has to resolve whether petitioners may
be subjected to military discipline on account of their defiance of a direct order of the AFP Chief of
Staff.

The solicited writs of certiorari and prohibition do not avail; the petition must be denied.

I.

The petitioners are high-ranking officers of the Armed Forces of the Philippines (AFP). Both
petitioners, Brigadier General Francisco Gudani (Gen. Gudani) and Lieutenant Colonel Alexander
Balutan (Col. Balutan), belonged to the Philippine Marines. At the time of the subject incidents, both
Gen. Gudani and Col. Balutan were assigned to the Philippine Military Academy (PMA) in Baguio
City, the former as the PMA Assistant Superintendent, and the latter as the Assistant Commandant
of Cadets.2

On 22 September 2005, Senator Rodolfo Biazon (Sen. Biazon) invited several senior officers of the
AFP to appear at a public hearing before the Senate Committee on National Defense and Security
(Senate Committee) scheduled on 28 September 2005. The hearing was scheduled after topics
concerning the conduct of the 2004 elections emerged in the public eye, particularly allegations of
massive cheating and the surfacing of copies of an audio excerpt purportedly of a phone
conversation between President Gloria Macapagal Arroyo and an official of the Commission on
Elections (COMELEC) widely reputed as then COMELEC Commissioner Virgilio Garcillano. At the
time of the 2004 elections, Gen. Gudani had been designated as commander, and Col. Balutan a
member, of "Joint Task Force Ranao" by the AFP Southern Command. "Joint Task Force Ranao"
was tasked with the maintenance of peace and order during the 2004 elections in the provinces of
Lanao del Norte and Lanao del Sur.3 `

Gen. Gudani, Col. Balutan, and AFP Chief of Staff Lieutenant General Generoso Senga (Gen.
Senga) were among the several AFP officers who received a letter invitation from Sen. Biazon to
attend the 28 September 2005 hearing. On 23 September 2005, Gen. Senga replied through a letter
to Sen. Biazon that he would be unable to attend the hearing due to a previous commitment in
Brunei, but he nonetheless "directed other officers from the AFP who were invited to attend the
hearing."4

On 26 September 2005, the Office of the Chief of Staff of the AFP issued a Memorandum addressed
to the Superintendent of the PMA Gen. Cristolito P. Baloing (Gen. Baloing). It was signed by Lt. Col.
Hernando DCA Iriberri in behalf of Gen. Senga.5 Noting that Gen. Gudani and Col. Balutan had been
invited to attend the Senate Committee hearing on 28 September 2005, the Memorandum directed
the two officers to attend the hearing. 6Conformably, Gen. Gudani and Col. Balutan filed their
respective requests for travel authority addressed to the PMA Superintendent.

On 27 September 2005, Gen. Senga wrote a letter to Sen. Biazon, requesting the postponement of
the hearing scheduled for the following day, since the AFP Chief of Staff was himself unable to
attend said hearing, and that some of the invited officers also could not attend as they were
"attending to other urgent operational matters." By this time, both Gen. Gudani and Col. Balutan had
already departed Baguio for Manila to attend the hearing.

Then on the evening of 27 September 2005, at around 10:10 p.m., a message was transmitted to
the PMA Superintendent from the office of Gen. Senga, stating as follows:

PER INSTRUCTION OF HER EXCELLENCY PGMA, NO AFP PERSONNEL SHALL


APPEAR BEFORE ANY CONGRESSIONAL OR SENATE HEARING WITHOUT HER
APPROVAL. INFORM BGEN FRANCISCO F GUDANI AFP AND LTC ALEXANDER
BALUTAN PA (GSC) ACCORDINGLY.7

The following day, Gen. Senga sent another letter to Sen. Biazon, this time informing the senator
that "no approval has been granted by the President to any AFP officer to appear" before the
hearing scheduled on that day. Nonetheless, both Gen. Gudani and Col. Balutan were present as
the hearing started, and they both testified as to the conduct of the 2004 elections.

The Office of the Solicitor General (OSG), representing the respondents before this Court, has
offered additional information surrounding the testimony of Gen. Gudani and Col. Balutan. The OSG
manifests that the couriers of the AFP Command Center had attempted to deliver the radio message
to Gen. Gudani’s residence in a subdivision in Parañaque City late in the night of 27 September
2005, but they were not permitted entry by the subdivision guards. The next day, 28 September
2005, shortly before the start of the hearing, a copy of Gen. Senga’s letter to Sen. Biazon sent
earlier that day was handed at the Senate by Commodore Amable B. Tolentino of the AFP Office for
Legislative Affairs to Gen. Gudani, who replied that he already had a copy. Further, Gen. Senga
called Commodore Tolentino on the latter’s cell phone and asked to talk to Gen. Gudani, but Gen.
Gudani refused. In response, Gen. Senga instructed Commodore Tolentino to inform Gen. Gudani
that "it was an order," yet Gen. Gudani still refused to take Gen. Senga’s call.8

A few hours after Gen. Gudani and Col. Balutan had concluded their testimony, the office of Gen.
Senga issued a statement which noted that the two had appeared before the Senate Committee "in
spite of the fact that a guidance has been given that a Presidential approval should be sought prior
to such an appearance;" that such directive was "in keeping with the time[-]honored principle of the
Chain of Command;" and that the two officers "disobeyed a legal order, in violation of A[rticles of]
W[ar] 65 (Willfully Disobeying Superior Officer), hence they will be subjected to General Court
Martial proceedings x x x" Both Gen. Gudani and Col. Balutan were likewise relieved of their
assignments then.9

On the very day of the hearing, 28 September 2005, President Gloria-Macapagal-Arroyo issued
Executive Order No. 464 (E.O. 464). The OSG notes that the E.O. "enjoined officials of the executive
department including the military establishment from appearing in any legislative inquiry without her
approval."10 This Court subsequently ruled on the constitutionality of the said executive order
in Senate v. Ermita.11 The relevance of E.O. 464 and Senate to the present petition shall be
discussed forthwith.

In the meantime, on 30 September 2005, petitioners were directed by General Senga, through Col.
Henry A. Galarpe of the AFP Provost Marshal General, to appear before the Office of the Provost
Marshal General (OPMG) on 3 October 2005 for investigation. During their appearance before Col.
Galarpe, both petitioners invoked their right to remain silent.12 The following day, Gen. Gudani was
compulsorily retired from military service, having reached the age of 56.13

In an Investigation Report dated 6 October 2005, the OPMG recommended that petitioners be
charged with violation of Article of War 65, on willfully disobeying a superior officer, in relation to
Article of War 97, on conduct prejudicial to the good order and military discipline. 14 As recommended,
the case was referred to a Pre-Trial Investigation Officer (PTIO) preparatory to trial by the General
Court Martial (GCM).15 Consequently, on 24 October 2005, petitioners were separately served with
Orders respectively addressed to them and signed by respondent Col. Gilbert Jose C. Roa, the Pre-
Trial Investigating Officer of the PTIO. The Orders directed petitioners to appear in person before
Col. Roa at the Pre-Trial Investigation of the Charges for violation of Articles 65 16 and 9717 of
Commonwealth Act No. 408,18 and to submit their counter-affidavits and affidavits of witnesses at the
Office of the Judge Advocate General. 19 The Orders were accompanied by respective charge sheets
against petitioners, accusing them of violating Articles of War 65 and 97.

It was from these premises that the present petition for certiorari and prohibition was filed,
particularly seeking that (1) the order of President Arroyo coursed through Gen. Senga preventing
petitioners from testifying before Congress without her prior approval be declared unconstitutional;
(2) the charges stated in the charge sheets against petitioners be quashed; and (3) Gen. Senga,
Col. Galarpe, Col. Roa, and their successors-in-interest or persons acting for and on their behalf or
orders, be permanently enjoined from proceeding against petitioners, as a consequence of their
having testified before the Senate on 28 September 2005.20

Petitioners characterize the directive from President Arroyo requiring her prior approval before any
AFP personnel appear before Congress as a "gag order," which violates the principle of separation
of powers in government as it interferes with the investigation of the Senate Committee conducted in
aid of legislation. They also equate the "gag order" with culpable violation of the Constitution,
particularly in relation to the public’s constitutional right to information and transparency in matters of
public concern. Plaintively, petitioners claim that "the Filipino people have every right to hear the
[petitioners’] testimonies," and even if the "gag order" were unconstitutional, it still was tantamount to
"the crime of obstruction of justice." Petitioners further argue that there was no law prohibiting them
from testifying before the Senate, and in fact, they were appearing in obeisance to the authority of
Congress to conduct inquiries in aid of legislation.

Finally, it is stressed in the petition that Gen. Gudani was no longer subject to military jurisdiction on
account of his compulsory retirement on 4 October 2005. It is pointed out that Article 2, Title I of the
Articles of War defines persons subject to military law as "all officers and soldiers in the active
service" of the AFP.

II.

We first proceed to define the proper litigable issues. Notably, the guilt or innocence of petitioners in
violating Articles 65 and 97 of the Articles of War is not an issue before this Court, especially
considering that per records, petitioners have not yet been subjected to court martial proceedings.
Owing to the absence of such proceedings, the correct inquiry should be limited to whether
respondents could properly initiate such proceedings preparatory to a formal court-martial, such as
the aforementioned preliminary investigation, on the basis of petitioners’ acts surrounding their
testimony before the Senate on 28 September 2005. Yet this Court, consistent with the principle that
it is not a trier of facts at first instance, 21 is averse to making any authoritative findings of fact, for that
function is first for the court-martial court to fulfill.

Thus, we limit ourselves to those facts that are not controverted before the Court, having been
commonly alleged by petitioners and the OSG (for respondents). Petitioners were called by the
Senate Committee to testify in its 28 September 2005 hearing. Petitioners attended such hearing
and testified before the Committee, despite the fact that the day before, there was an order from
Gen. Senga (which in turn was sourced "per instruction" from President Arroyo) prohibiting them
from testifying without the prior approval of the President. Petitioners do not precisely admit before
this Court that they had learned of such order prior to their testimony, although the OSG asserts that
at the very least, Gen. Gudani already knew of such order before he testified. 22 Yet while this fact
may be ultimately material in the court-martial proceedings, it is not determinative of this petition,
which as stated earlier, does not proffer as an issue whether petitioners are guilty of violating the
Articles of War.

What the Court has to consider though is whether the violation of the aforementioned order of Gen.
Senga, which emanated from the President, could lead to any investigation for court-martial of
petitioners. It has to be acknowledged as a general principle23 that AFP personnel of whatever rank
are liable under military law for violating a direct order of an officer superior in rank. Whether
petitioners did violate such an order is not for the Court to decide, but it will be necessary to assume,
for the purposes of this petition, that petitioners did so.

III.

Preliminarily, we must discuss the effect of E.O. 464 and the Court’s ruling in Senate on the present
petition. Notably, it is not alleged that petitioners were in any way called to task for violating
E.O. 464, but instead, they were charged for violating the direct order of Gen. Senga not to
appear before the Senate Committee, an order that stands independent of the executive
order. Distinctions are called for, since Section 2(b) of E.O. 464 listed "generals and flag officers of
the Armed Forces of the Philippines and such other officers who in the judgment of the Chief of Staff
are covered by the executive privilege," as among those public officials required in Section 3 of E.O.
464 "to secure prior consent of the President prior to appearing before either House of Congress."
The Court in Senate declared both Section 2(b) and Section 3 void,24 and the impression may have
been left following Senate that it settled as doctrine, that the President is prohibited from requiring
military personnel from attending congressional hearings without having first secured prior
presidential consent. That impression is wrong.

Senate turned on the nature of executive privilege, a presidential prerogative which is encumbered


by significant limitations. Insofar as E.O. 464 compelled officials of the executive branch to seek prior
presidential approval before appearing before Congress, the notion of executive control also comes
into consideration.25 However, the ability of the President to require a military official to secure prior
consent before appearing before Congress pertains to a wholly different and independent specie of
presidential authority—the commander-in-chief powers of the President. By tradition and
jurisprudence, the commander-in-chief powers of the President are not encumbered by the same
degree of restriction as that which may attach to executive privilege or executive control.

During the deliberations in Senate, the Court was very well aware of the pendency of this petition as
well as the issues raised herein. The decision in Senate was rendered with the comfort that the
nullification of portions of E.O. 464 would bear no impact on the present petition since petitioners
herein were not called to task for violating the executive order. Moreover, the Court was then
cognizant that Senate and this case would ultimately hinge on disparate legal issues.
Relevantly, Senate purposely did not touch upon or rule on the faculty of the President, under the
aegis of the commander-in-chief powers26 to require military officials from securing prior consent
before appearing before Congress. The pertinent factors in considering that question are markedly
outside of those which did become relevant in adjudicating the issues raised in Senate. It is in this
petition that those factors come into play.

At this point, we wish to dispose of another peripheral issue before we strike at the heart of the
matter. General Gudani argues that he can no longer fall within the jurisdiction of the court-martial,
considering his retirement last 4 October 2005. He cites Article 2, Title I of Commonwealth Act No.
408, which defines persons subject to military law as, among others, "all officers and soldiers in the
active service of the [AFP]," and points out that he is no longer in the active service.

This point was settled against Gen. Gudani’s position in Abadilla v. Ramos,27 where the Court
declared that an officer whose name was dropped from the roll of officers cannot be considered to
be outside the jurisdiction of military authorities when military justice proceedings were initiated
against him before the termination of his service. Once jurisdiction has been acquired over the
officer, it continues until his case is terminated. Thus, the Court held:

The military authorities had jurisdiction over the person of Colonel Abadilla at the time of the alleged
offenses. This jurisdiction having been vested in the military authorities, it is retained up to the end of
the proceedings against Colonel Abadilla. Well-settled is the rule that jurisdiction once acquired is
not lost upon the instance of the parties but continues until the case is terminated.28

Citing Colonel Winthrop’s treatise on Military Law, the Court further stated:

We have gone through the treatise of Colonel Winthrop and We find the following passage which
goes against the contention of the petitioners, viz —

3. Offenders in general — Attaching of jurisdiction. It has further been held, and is now settled law, in
regard to military offenders in general, that if the military jurisdiction has once duly attached to them
previous to the date of the termination of their legal period of service, they may be brought to trial by
court-martial after that date, their discharge being meanwhile withheld. This principle has mostly
been applied to cases where the offense was committed just prior to the end of the term. In such
cases the interests of discipline clearly forbid that the offender should go unpunished. It is held
therefore that if before the day on which his service legally terminates and his right to a
discharge is complete, proceedings with a view to trial are commenced against him — as by
arrest or the service of charges, — the military jurisdiction will fully attach and once attached
may be continued by a trial by court-martial ordered and held after the end of the term of the
enlistment of the accused x x x 29

Thus, military jurisdiction has fully attached to Gen. Gudani inasmuch as both the acts complained of
and the initiation of the proceedings against him occurred before he compulsorily retired on 4
October 2005. We see no reason to unsettle the Abadilla doctrine. The OSG also points out that
under Section 28 of Presidential Decree No. 1638, as amended, "[a]n officer or enlisted man carried
in the retired list [of the Armed Forces of the Philippines] shall be subject to the Articles of War x x
x"30 To this citation, petitioners do not offer any response, and in fact have excluded the matter of
Gen. Gudani’s retirement as an issue in their subsequent memorandum.

IV.

We now turn to the central issues.

Petitioners wish to see annulled the "gag order" that required them to secure presidential consent
prior to their appearance before the Senate, claiming that it violates the constitutional right to
information and transparency in matters of public concern; or if not, is tantamount at least to the
criminal acts of obstruction of justice and grave coercion. However, the proper perspective from
which to consider this issue entails the examination of the basis and authority of the President to
issue such an order in the first place to members of the AFP and the determination of whether such
an order is subject to any limitations.

The vitality of the tenet that the President is the commander-in-chief of the Armed Forces is most
crucial to the democratic way of life, to civilian supremacy over the military, and to the general
stability of our representative system of government. The Constitution reposes final authority, control
and supervision of the AFP to the President, a civilian who is not a member of the armed forces, and
whose duties as commander-in-chief represent only a part of the organic duties imposed upon the
office, the other functions being clearly civil in nature.31 Civilian supremacy over the military also
countermands the notion that the military may bypass civilian authorities, such as civil courts, on
matters such as conducting warrantless searches and seizures.32

Pursuant to the maintenance of civilian supremacy over the military, the Constitution has allocated
specific roles to the legislative and executive branches of government in relation to military affairs.
Military appropriations, as with all other appropriations, are determined by Congress, as is the power
to declare the existence of a state of war.33Congress is also empowered to revoke a proclamation of
martial law or the suspension of the writ of habeas corpus.34 The approval of the Commission on
Appointments is also required before the President can promote military officers from the rank of
colonel or naval captain.35 Otherwise, on the particulars of civilian dominance and administration
over the military, the Constitution is silent, except for the commander-in-chief clause which is fertile
in meaning and

implication as to whatever inherent martial authority the President may possess.36

The commander-in-chief provision in the Constitution is denominated as Section 18, Article VII,
which begins with the simple declaration that "[t]he President shall be the Commander-in-Chief of all
armed forces of the Philippines x x x"37 Outside explicit constitutional limitations, such as those found
in Section 5, Article XVI, the commander-in-chief clause vests on the President, as commander-in-
chief, absolute authority over the persons and actions of the members of the armed forces. Such
authority includes the ability of the President to restrict the travel, movement and speech of military
officers, activities which may otherwise be sanctioned under civilian law.

Reference to Kapunan, Jr. v. De Villa38 is useful in this regard. Lt. Col. Kapunan was ordered
confined under "house arrest" by then Chief of Staff (later President) Gen. Fidel Ramos. Kapunan
was also ordered, as a condition for his house arrest, that he may not issue any press statements or
give any press conference during his period of detention. The Court unanimously upheld such
restrictions, noting:

[T]he Court is of the view that such is justified by the requirements of military discipline. It cannot be
gainsaid that certain liberties of persons in the military service, including the freedom of
speech, may be circumscribed by rules of military discipline. Thus, to a certain degree,
individual rights may be curtailed, because the effectiveness of the military in fulfilling its
duties under the law depends to a large extent on the maintenance of discipline within its
ranks. Hence, lawful orders must be followed without question and rules must be faithfully
complied with, irrespective of a soldier's personal views on the matter. It is from this viewpoint
that the restrictions imposed on petitioner Kapunan, an officer in the AFP, have to be considered.39

Any good soldier, or indeed any ROTC cadet, can attest to the fact that the military way of life
circumscribes several of the cherished freedoms of civilian life. It is part and parcel of the military
package. Those who cannot abide by these limitations normally do not pursue a military career and
instead find satisfaction in other fields; and in fact many of those discharged from the service are
inspired in their later careers precisely by their rebellion against the regimentation of military life.
Inability or unwillingness to cope with military discipline is not a stain on character, for the military
mode is a highly idiosyncratic path which persons are not generally conscripted into, but volunteer
themselves to be part of. But for those who do make the choice to be a soldier, significant
concessions to personal freedoms are expected. After all, if need be, the men and women of the
armed forces may be commanded upon to die for country, even against their personal inclinations.

It may be so that military culture is a remnant of a less democratic era, yet it has been fully
integrated into the democratic system of governance. The constitutional role of the armed forces is
as protector of the people and of the State.40 Towards this end, the military must insist upon a
respect for duty and a discipline without counterpart in civilian life. 41 The laws and traditions
governing that discipline have a long history; but they are founded on unique military exigencies as
powerful now as in the past.42 In the end, it must be borne in mind that the armed forces has a
distinct subculture with unique needs, a specialized society separate from civilian society. 43 In the
elegant prose of the eminent British military historian, John Keegan:

[Warriors who fight wars have] values and skills [which] are not those of politicians and diplomats.
They are those of a world apart, a very ancient world, which exists in parallel with the everyday world
but does not belong to it. Both worlds change over time, and the warrior world adopts in step to the
civilian. It follows it, however, at a distance. The distance can never be closed, for the culture of the
warrior can never be that of civilization itself….44

Critical to military discipline is obeisance to the military chain of command. Willful disobedience of a
superior officer is punishable by court-martial under Article 65 of the Articles of War.45 "An individual
soldier is not free to ignore the lawful orders or duties assigned by his immediate superiors. For
there would be an end of all discipline if the seaman and marines on board a ship of war [or soldiers
deployed in the field], on a distant service, were permitted to act upon their own opinion of their
rights [or their opinion of the
President’s intent], and to throw off the authority of the commander whenever they supposed it to
be unlawfully exercised."46

Further traditional restrictions on members of the armed forces are those imposed on free speech
and mobility. Kapunan is ample precedent in justifying that a soldier may be restrained by a superior
1âwphi1

officer from speaking out on certain matters. As a general rule, the discretion of a military officer to
restrain the speech of a soldier under his/her command will be accorded deference, with minimal
regard if at all to the reason for such restraint. It is integral to military discipline that the soldier’s
speech be with the consent and approval of the military commander.

The necessity of upholding the ability to restrain speech becomes even more imperative if the soldier
desires to speak freely on political matters. The Constitution requires that "[t]he armed forces shall
be insulated from partisan politics," and that ‘[n]o member of the military shall engage directly or
indirectly in any partisan political activity, except to vote." 47 Certainly, no constitutional provision or
military indoctrination will eliminate a soldier’s ability to form a personal political opinion, yet it is vital
that such opinions be kept out of the public eye. For one, political belief is a potential source of
discord among people, and a military torn by political strife is incapable of fulfilling its constitutional
function as protectors of the people and of the State. For another, it is ruinous to military discipline to
foment an atmosphere that promotes an active dislike of or dissent against the President, the
commander-in-chief of the armed forces. Soldiers are constitutionally obliged to obey a President
they may dislike or distrust. This fundamental principle averts the country from going the way of
banana republics.

Parenthetically, it must be said that the Court is well aware that our country’s recent past is marked
by regime changes wherein active military dissent from the chain of command formed a key, though
not exclusive, element. The Court is not blind to history, yet it is a judge not of history but of the
Constitution. The Constitution, and indeed our modern democratic order, frown in no uncertain terms
on a politicized military, informed as they are on the trauma of absolute martial rule. Our history
might imply that a political military is part of the natural order, but this view cannot be affirmed by the
legal order. The evolutionary path of our young democracy necessitates a reorientation from this
view, reliant as our socio-political culture has become on it. At the same time, evolution mandates a
similar demand that our system of governance be more responsive to the needs and aspirations of
the citizenry, so as to avoid an environment vulnerable to a military apparatus able at will to exert an
undue influence in our polity.

Of possibly less gravitas, but of equal importance, is the principle that mobility of travel is another
necessary restriction on members of the military. A soldier cannot leave his/her post without the
consent of the commanding officer. The reasons are self-evident. The commanding officer has to be
aware at all times of the location of the troops under command, so as to be able to appropriately
respond to any exigencies. For the same reason, commanding officers have to be able to restrict the
movement or travel of their soldiers, if in their judgment, their presence at place of call of duty is
necessary. At times, this may lead to unsentimental, painful consequences, such as a soldier being
denied permission to witness the birth of his first-born, or to attend the funeral of a parent. Yet again,
military life calls for considerable personal sacrifices during the period of conscription, wherein the
higher duty is not to self but to country.

Indeed, the military practice is to require a soldier to obtain permission from the commanding officer
before he/she may leave his destination. A soldier who goes from the properly appointed place of
duty or absents from his/her command, guard, quarters, station, or camp without proper leave is
subject to punishment by court-martial.48 It is even clear from the record that petitioners had actually
requested for travel authority from the PMA in Baguio City to Manila, to attend the Senate
Hearing.49 Even petitioners are well aware that it was necessary for them to obtain permission from
their superiors before they could travel to Manila to attend the Senate Hearing.

It is clear that the basic position of petitioners impinges on these fundamental principles we have
discussed. They seek to be exempted from military justice for having traveled to the Senate to testify
before the Senate Committee against the express orders of Gen. Senga, the AFP Chief of Staff. If
petitioners’ position is affirmed, a considerable exception would be carved from the unimpeachable
right of military officers to restrict the speech and movement of their juniors. The ruinous
consequences to the chain of command and military discipline simply cannot warrant the
Court’s imprimatur on petitioner’s position.

V.

Still, it would be highly myopic on our part to resolve the issue solely on generalities surrounding
military discipline. After all, petitioners seek to impress on us that their acts are justified as they were
responding to an invitation from the Philippine Senate, a component of the legislative branch of
government. At the same time, the order for them not to testify ultimately came from the President,
the head of the executive branch of government and the commander-in-chief of the armed forces.

Thus, we have to consider the question: may the President prevent a member of the armed forces
from testifying before a legislative inquiry? We hold that the President has constitutional authority to
do so, by virtue of her power as commander-in-chief, and that as a consequence a military officer
who defies such injunction is liable under military justice. At the same time, we also hold that any
chamber of Congress which seeks the appearance before it of a military officer against the consent
of the President has adequate remedies under law to compel such attendance. Any military official
whom Congress summons to testify before it may be compelled to do so by the President. If the
President is not so inclined, the President may be commanded by judicial order to compel the
attendance of the military officer. Final judicial orders have the force of the law of the land which the
President has the duty to faithfully execute.50

Explication of these principles is in order.

As earlier noted, we ruled in Senate that the President may not issue a blanket requirement of prior
consent on executive officials summoned by the legislature to attend a congressional hearing. In
doing so, the Court recognized the considerable limitations on executive privilege, and affirmed that
the privilege must be formally invoked on specified grounds. However, the ability of the President
to prevent military officers from testifying before Congress does not turn on executive
privilege, but on the Chief Executive’s power as commander-in-chief to control the actions
and speech of members of the armed forces. The President’s prerogatives as commander-in-
chief are not hampered by the same limitations as in executive privilege.

Our ruling that the President could, as a general rule, require military officers to seek presidential
approval before appearing before Congress is based foremost on the notion that a contrary rule
unduly diminishes the prerogatives of the President as commander-in-chief. Congress holds
significant control over the armed forces in matters such as budget appropriations and the approval
of higher-rank promotions,51 yet it is on the President that the Constitution vests the title as
commander-in-chief and all the prerogatives and functions appertaining to the position. Again, the
exigencies of military discipline and the chain of command mandate that the President’s ability to
control the individual members of the armed forces be accorded the utmost respect. Where a military
officer is torn between obeying the President and obeying the Senate, the Court will without
hesitation affirm that the officer has to choose the President. After all, the Constitution prescribes
that it is the President, and not the Senate, who is the commander-in-chief of the armed forces.52
At the same time, the refusal of the President to allow members of the military to appear before
Congress is still subject to judicial relief. The Constitution itself recognizes as one of the legislature’s
functions is the conduct of inquiries in aid of legislation. 53 Inasmuch as it is ill-advised for Congress to
interfere with the President’s power as commander-in-chief, it is similarly detrimental for the
President to unduly interfere with Congress’s right to conduct legislative inquiries. The impasse did
not come to pass in this petition, since petitioners testified anyway despite the presidential
prohibition. Yet the Court is aware that with its pronouncement today that the President has the right
to require prior consent from members of the armed forces, the clash may soon loom or actualize.

We believe and hold that our constitutional and legal order sanctions a modality by which members
of the military may be compelled to attend legislative inquiries even if the President desires
otherwise, a modality which does not offend the Chief Executive’s prerogatives as commander-in-
chief. The remedy lies with the courts.

The fact that the executive branch is an equal, coordinate branch of government to the legislative
creates a wrinkle to any basic rule that persons summoned to testify before Congress must do so.
There is considerable interplay between the legislative and executive branches, informed by due
deference and respect as to their various constitutional functions. Reciprocal courtesy idealizes this
relationship; hence, it is only as a last resort that one branch seeks to compel the other to a
particular mode of behavior. The judiciary, the third coordinate branch of government, does not enjoy
a similar dynamic with either the legislative or executive branches. Whatever weakness inheres on
judicial power due to its inability to originate national policies and legislation, such is balanced by the
fact that it is the branch empowered by the Constitution to compel obeisance to its rulings by the
other branches of government.

As evidenced by Arnault v. Nazareno54 and Bengzon v. Senate Blue Ribbon Committee,55 among


others, the Court has not shirked from reviewing the exercise by Congress of its power of legislative
inquiry.56 Arnault recognized that the legislative power of inquiry and the process to enforce it, "is an
essential and appropriate auxiliary to the legislative function."57 On the other
hand, Bengzon acknowledged that the power of both houses of Congress to conduct inquiries in aid
of legislation is not "absolute or unlimited", and its exercise is circumscribed by Section 21, Article VI
of the Constitution.58 From these premises, the Court enjoined the Senate Blue Ribbon Committee
from requiring the petitioners in Bengzon from testifying and producing evidence before the
committee, holding that the inquiry in question did not involve any intended legislation.

Senate affirmed both the Arnault and Bengzon rulings. It elucidated on the constitutional scope and


limitations on the constitutional power of congressional inquiry. Thus:

As discussed in Arnault, the power of inquiry, "with process to enforce it," is grounded on the
necessity of information in the legislative process. If the information possessed by executive officials
on the operation of their offices is necessary for wise legislation on that subject, by parity of
reasoning, Congress has the right to that information and the power to compel the disclosure
thereof.

As evidenced by the American experience during the so-called "McCarthy era", however, the right of
Congress to conduct inquirites in aid of legislation is, in theory, no less susceptible to abuse than
executive or judicial power. It may thus be subjected to judicial review pursuant to the Court’s
certiorari powers under Section 1, Article VIII of the Constitution.

For one, as noted in Bengzon v. Senate Blue Ribbon Committee, the inquiry itself might not properly
be in aid of legislation, and thus beyond the constitutional power of Congress. Such inquiry could not
usurp judicial functions. Parenthetically, one possible way for Congress to avoid such result as
occurred in Bengzon is to indicate in its invitations to the public officials concerned, or to any person
for that matter, the possible needed statute which prompted the need for the inquiry. Given such
statement in its invitations, along with the usual indication of the subject of inquiry and the questions
relative to and in furtherance thereof, there would be less room for speculation on the part of the
person invited on whether the inquiry is in aid of legislation.

Section 21, Article VI likewise establishes critical safeguards that proscribe the legislative power of
inquiry. The provision requires that the inquiry be done in accordance with the Senate or House’s
duly published rules of procedure, necessarily implying the constitutional infirmity of an inquiry
conducted without duly published rules of procedure. Section 21 also mandates that the rights of
persons appearing in or affected by such inquiries be respected, an imposition that obligates
Congress to adhere to the guarantees in the Bill of Rights.

These abuses are, of course, remediable before the courts, upon the proper suit filed by the persons
affected, even if they belong to the executive branch. Nonetheless, there may be exceptional
circumstances… wherein a clear pattern of abuse of the legislative power of inquiry might be
established, resulting in palpable violations of the rights guaranteed to members of the executive
department under the Bill of Rights. In such instances, depending on the particulars of each case,
attempts by the Executive Branch to forestall these abuses may be accorded judicial sanction59 .

In Senate, the Court ruled that the President could not impose a blanket prohibition barring executive
officials from testifying before Congress without the President’s consent notwithstanding the
invocation of executive privilege to justify such prohibition. The Court did not rule that the power to
conduct legislative inquiry ipso facto superseded the claim of executive privilege, acknowledging
instead that the viability of executive privilege stood on a case to case basis. Should neither branch
yield to the other branch’s assertion, the constitutional recourse is to the courts, as the final arbiter if
the dispute. It is only the courts that can compel, with conclusiveness, attendance or non-attendance
in legislative inquiries.

Following these principles, it is clear that if the President or the Chief of Staff refuses to allow a
member of the AFP to appear before Congress, the legislative body seeking such testimony may
seek judicial relief to compel the attendance. Such judicial action should be directed at the heads of
the executive branch or the armed forces, the persons who wield authority and control over the
actions of the officers concerned. The legislative purpose of such testimony, as well as any defenses
against the same — whether grounded on executive privilege, national security or similar concerns
— would be accorded due judicial evaluation. All the constitutional considerations pertinent to either
branch of government may be raised, assessed, and ultimately weighed against each other. And
once the courts speak with finality, both branches of government have no option but to comply with
the decision of the courts, whether the effect of the decision is to their liking or disfavor.

Courts are empowered, under the constitutional principle of judicial review, to arbitrate disputes
between the legislative and executive branches of government on the proper constitutional
parameters of power.60 This is the fair and workable solution implicit in the constitutional allocation of
powers among the three branches of government. The judicial filter helps assure that the
particularities of each case would ultimately govern, rather than any overarching principle unduly
inclined towards one branch of government at the expense of the other. The procedure may not
move as expeditiously as some may desire, yet it ensures thorough deliberation of all relevant and
cognizable issues before one branch is compelled to yield to the other. Moreover, judicial review
does not preclude the legislative and executive branches from negotiating a mutually acceptable
solution to the impasse. After all, the two branches, exercising as they do functions and
responsibilities that are political in nature, are free to smooth over the thorns in their relationship with
a salve of their own choosing.
And if emphasis be needed, if the courts so rule, the duty falls on the shoulders of the
President, as commander-in-chief, to authorize the appearance of the military officers before
Congress. Even if the President has earlier disagreed with the notion of officers appearing
before the legislature to testify, the Chief Executive is nonetheless obliged to comply with the
final orders of the courts.

Petitioners have presented several issues relating to the tenability or wisdom of the President’s order
on them and other military officers not to testify before Congress without the President’s consent.
Yet these issues ultimately detract from the main point — that they testified before the Senate
despite an order from their commanding officer and their commander-in-chief for them not to do
so,61 in contravention of the traditions of military discipline which we affirm today.  The issues raised
1âwphi1

by petitioners could have very well been raised and properly adjudicated if the proper procedure was
observed. Petitioners could have been appropriately allowed to testify before the Senate without
having to countermand their Commander-in-chief and superior officer under the setup we have
prescribed.

We consider the other issues raised by petitioners unnecessary to the resolution of this petition.

Petitioners may have been of the honest belief that they were defying a direct order of their
Commander-in-Chief and Commanding General in obeisance to a paramount idea formed within
their consciences, which could not be lightly ignored. Still, the Court, in turn, is guided by the
superlative principle that is the Constitution, the embodiment of the national conscience. The
Constitution simply does not permit the infraction which petitioners have allegedly committed, and
moreover, provides for an orderly manner by which the same result could have been achieved
without offending constitutional principles.

WHEREFORE, the petition is DENIED. No pronouncement as to costs.

SO ORDERED.

G.R. No. 164007 August 10, 2006

LT. (SG) EUGENE GONZALES, LT. (SG) ANDY TORRATO, LT. (SG) ANTONIO TRILLANES IV,
CPT. GARY ALEJANO, LT. (SG) JAMES LAYUG, CPT. GERARDO GAMBALA, CPT. NICANOR
FAELDON, LT. (SG) MANUEL CABOCHAN, ENS. ARMAND PONTEJOS, LT. (JG) ARTURO
PASCUA, and 1LT. JONNEL SANGGALANG, Petitioners,
vs.
GEN. NARCISO ABAYA, in his capacity as Chief of Staff of the Armed Forces of the
Philippines, and B. GEN. MARIANO M. SARMIENTO, JR., in his capacity as the Judge
Advocate General of the Judge Advocate General’s Office (JAGO), Respondents.

DECISION

SANDOVAL-GUTIERREZ, J.:

For our resolution is the Petition for Prohibition (with prayer for a temporary restraining order) filed by
the above-named members of the Armed Forces of the Philippines (AFP), herein petitioners, against
the AFP Chief of Staff and the Judge Advocate General, respondents.
The facts are:

On July 26, 2003, President Gloria Macapagal Arroyo received intelligence reports that some
members of the AFP, with high-powered weapons, had abandoned their designated places of
assignment. Their aim was to destabilize the government. The President then directed the AFP and
the Philippine National Police (PNP) to track and arrest them.

On July 27, 2003 at around 1:00 a.m., more than 300 heavily armed junior officers and enlisted men
of the AFP – mostly from the elite units of the Army’s Scout Rangers and the Navy’s Special Warfare
Group – entered the premises of the Oakwood Premier Luxury Apartments on Ayala Avenue, Makati
City. They disarmed the security guards and planted explosive devices around the building.

Led by Navy Lt. (SG) Antonio Trillanes IV, the troops sported red armbands emblazoned with the
emblem of the "Magdalo" faction of the Katipunan. 1 The troops then, through broadcast media,
announced their grievances against the administration of President Gloria Macapagal Arroyo, such
as the graft and corruption in the military, the illegal sale of arms and ammunition to the "enemies" of
the State, and the bombings in Davao City intended to acquire more military assistance from the US
government. They declared their withdrawal of support from their Commander-in-Chief and
demanded that she resign as President of the Republic. They also called for the resignation of her
cabinet members and the top brass of the AFP and PNP.

About noontime of the same day, President Arroyo issued Proclamation No. 427 declaring a state of
rebellion, followed by General Order No. 4 directing the AFP and PNP to take all necessary
measures to suppress the rebellion then taking place in Makati City. She then called the soldiers to
surrender their weapons at five o’clock in the afternoon of that same day.

In order to avoid a bloody confrontation, the government sent negotiators to dialogue with the
soldiers. The aim was to persuade them to peacefully return to the fold of the law. After several
hours of negotiation, the government panel succeeded in convincing them to lay down their arms
and defuse the explosives placed around the premises of the Oakwood Apartments. Eventually, they
returned to their barracks.

A total of 321 soldiers, including petitioners herein, surrendered to the authorities.

The National Bureau of Investigation (NBI) investigated the incident and recommended that the
military personnel involved be charged with coup d’etat defined and penalized under Article 134-A of
the Revised Penal Code, as amended. On July 31, 2003, the Chief State Prosecutor of the
Department of Justice (DOJ) recommended the filing of the corresponding Information against them.

Meanwhile, on August 2, 2003, pursuant to Article 70 of the Articles of War, respondent General
Narciso Abaya, then AFP Chief of Staff, ordered the arrest and detention of the soldiers involved in
the Oakwood incident and directed the AFP to conduct its own separate investigation.

On August 5, 2003, the DOJ filed with the Regional Trial Court (RTC), Makati City an Information
for coup d’etat 2against those soldiers, docketed as Criminal Case No. 03-2784 and eventually
raffled off to Branch 61, presided by Judge Romeo F. Barza. 3 Subsequently, this case was
consolidated with Criminal Case No. 03-2678, involving the other accused, pending before Branch
148 of the RTC, Makati City, presided by Judge Oscar B. Pimentel.

On August 13, 2003, the RTC directed the DOJ to conduct a reinvestigation of Criminal Case No.
03-2784.
On the same date, respondent Chief of Staff issued Letter Order No. 625 creating a Pre-Trial
Investigation Panel tasked to determine the propriety of filing with the military tribunal charges for
violations of the Articles of War under Commonwealth Act No. 408, 4 as amended, against the same
military personnel. Specifically, the charges are: (a) violation of Article 63 for disrespect toward the
President, the Secretary of National Defense, etc., (b) violation of Article 64 for disrespect toward a
superior officer, (c) violation of Article 67 for mutiny or sedition, (d) violation of Article 96 for conduct
unbecoming an officer and a gentleman, and (e) violation of Article 97 for conduct prejudicial to good
order and military discipline.

Of the original 321 accused in Criminal Case No. 03-2784, only 243 (including petitioners herein)
filed with the RTC, Branch 148 an Omnibus Motion praying that the said trial court assume
jurisdiction over all the charges filed with the military tribunal. They invoked Republic Act (R.A.) No.
7055. 5

On September 15, 2003, petitioners filed with the Judge Advocate General’s Office (JAGO) a motion
praying for the suspension of its proceedings until after the RTC shall have resolved their motion to
assume jurisdiction.

On October 29, 2003, the Pre-Trial Investigation Panel submitted its Initial Report to the AFP Chief
of Staff recommending that the military personnel involved in the Oakwood incident be charged
before a general court martial with violations of Articles 63, 64, 67, 96, and 97 of the Articles of War.

Meanwhile, on November 11, 2003, the DOJ, after conducting a reinvestigation, found probable
cause against only 31 (petitioners included) of the 321 accused in Criminal Case No. 03-2784.
Accordingly, the prosecution filed with the RTC an Amended Information. 6

In an Order dated November 14, 2003, the RTC admitted the Amended Information and dropped the
charge of coup d’etat against the 290 accused.

Subsequently, or on December 12, 2003, the Pre-Trial Investigation Panel submitted its Final Pre-
Trial Investigation Report 7 to the JAGO, recommending that, following the "doctrine of absorption,"
those charged with coup d’etatbefore the RTCshould not be charged before the military tribunal for
violation of the Articles of War.

For its part, the RTC, on February 11, 2004, issued an Order 8 stating that "all charges before the
court martial against the accused…are hereby declared not service-connected, but rather absorbed
and in furtherance of the alleged crime of coup d’etat." The trial court then proceeded to hear
petitioners’ applications for bail.

In the meantime, Colonel Julius A. Magno, in his capacity as officer-in-charge of the JAGO, reviewed
the findings of the Pre-Trial Investigation Panel. He recommended that 29 of the officers involved in
the Oakwood incident, including petitioners, be prosecuted before a general court martial for
violation of Article 96 (conduct unbecoming an officer and a gentleman) of the Articles of War.

On June 17, 2004, Colonel Magno’s recommendation was approved by the AFP top brass. The AFP
Judge Advocate General then directed petitioners to submit their answer to the charge. Instead of
complying, they filed with this Court the instant Petition for Prohibition praying that respondents be
ordered to desist from charging them with violation of Article 96 of the Articles of War in relation to
the Oakwood incident. 9
Petitioners maintain that since the RTC has made a determination in its Order of February 11, 2004
that the offense for violation of Article 96 (conduct unbecoming an officer and a gentleman) of the
Articles of War is not service-connected, but is absorbed in the crime of coup d’etat, the military
tribunal cannot compel them to submit to its jurisdiction.

The Solicitor General, representing the respondents, counters that R.A. No. 7055 specifies which
offenses covered by the Articles of War areservice-connected. These are violations of Articles 54 to
70, 72 to 92, and 95 to 97. The law provides that violations of these Articles are properly cognizable
by the court martial. As the charge against petitioners is violation of Article 96 which, under R.A. No.
7055 is a service-connected offense, then it falls under the jurisdiction of the court martial.

Subsequently, petitioners filed with this Court a Supplemental Petition raising the additional issue
that the offense charged before the General Court Martial has prescribed. Petitioners alleged therein
that during the pendency of their original petition, respondents proceeded with the Pre-Trial
Investigation for purposes of charging them with violation of Article 96 (conduct unbecoming an
officer and a gentleman) of the Articles of War; that the Pre-Trial Investigation Panel then referred
the case to the General Court Martial; that "almost two years since the Oakwood incident on July 27,
2003, only petitioner Lt. (SG) Antonio Trillanes was arraigned, and this was done under questionable
circumstances;" 10 that in the hearing of July 26, 2005, herein petitioners moved for the dismissal of
the case on the ground that they were not arraigned within the prescribed period of two (2) years
from the date of the commission of the alleged offense, in violation of Article 38 of the Articles of
War; 11 that "the offense charged prescribed on July 25, 2005;" 12 that the General Court Martial
ruled, however, that "the prescriptive period shall end only at 12:00 midnight of July 26, 2005;" 13 that
"(a)s midnight of July 26, 2005 was approaching and it was becoming apparent that the accused
could not be arraigned, the prosecution suddenly changed its position and asserted that 23 of the
accused have already been arraigned;" 14 and that petitioners moved for a reconsideration but it was
denied by the general court martial in its Order dated September 14, 2005. 15

In his Comment, the Solicitor General prays that the Supplemental Petition be denied for lack of
merit. He alleges that "contrary to petitioners’ pretensions, all the accused were duly arraigned on
July 13 and 18, 2005." 16 The "(r)ecords show that in the hearing on July 13, 2005, all the 29 accused
were present" and, "(o)n that day, Military Prosecutor Captain Karen Ong Jags read the Charges
and Specifications from the Charge Sheet in open court (pp. 64, TSN, July 13, 2005)." 17

The sole question for our resolution is whether the petitioners are entitled to the writ of prohibition.

There is no dispute that petitioners, being officers of the AFP, are subject to military law. Pursuant to
Article 1 (a) of Commonwealth Act No. 408, as amended, otherwise known as the Articles of War,
the term "officer" is "construed to refer to a commissioned officer." Article 2 provides:

Art. 2. Persons Subject to Military Law. – The following persons are subject to these articles and
shall be understood as included in the term "any person subject to military law" or "persons subject
to military law," whenever used in these articles:

(a) All officers and soldiers in the active service of the Armed Forces of the Philippines or of
the Philippine Constabulary, all members of the reserve force, from the dates of their call to active
duty and while on such active duty; all trainees undergoing military instructions; and all other
persons lawfully called, drafted, or ordered into, or to duty or for training in the said service, from the
dates they are required by the terms of the call, draft, or order to obey the same.

Upon the other hand, Section 1 of R.A. No. 7055 reads:


SEC. 1. Members of the Armed Forces of the Philippines and other persons subject to military law,
including members of the Citizens Armed Forces Geographical Units, who commit crimes or
offenses penalized under the Revised Penal Code, other special penal laws, or local government
ordinances, regardless of whether or not civilians are co-accused, victims, or offended parties, which
may be natural or juridical persons, shall be tried by the proper civil court, except when the offense,
as determined before arraignment by the civil court, is service-connected, in which case, the offense
shall be tried by court-martial, Provided, That the President of the Philippines may, in the interest of
justice, order or direct at any time before arraignment that any such crimes or offenses be tried by
the proper civil courts.

As used in this Section, service-connected crimes or offenses shall be limited to those defined in
Articles 54 to 70, Articles 72 to 92, and Articles 95 to 97 of Commonwealth Act No. 408, as
amended.

In imposing the penalty for such crimes or offenses, the court-martial may take into consideration the
penalty prescribed therefor in the Revised Penal Code, other special laws, or local government
ordinances.

Section 1 of R.A. No. 7055, quoted above, is clear and unambiguous. First, it lays down the general
rule that members of the AFP and other persons subject to military law, including members of the
Citizens Armed Forces Geographical Units, who commit crimes or offenses penalized under the
Revised Penal Code (like coup d’etat), other special penal laws, or local ordinances shall be tried by
the proper civil court. Next, it provides the exception to the general rule, i.e., where the civil court,
before arraignment, has determined the offense to be service-connected, then the offending soldier
shall be tried by a court martial. Lastly, the law states an exception to the exception, i.e., where the
President of the Philippines, in the interest of justice, directs before arraignment that any such
crimes or offenses be tried by the proper civil court.

The second paragraph of the same provision further identifies the "service-connected crimes or
offenses" as "limited to those defined in Articles 54 to 70, Articles 72 to 92, and Articles 95 to 97"
of the Articles of War. Violations of these specified Articles are triable by court martial. This
delineates the jurisdiction between the civil courts and the court martial over crimes or offenses
committed by military personnel.

Such delineation of jurisdiction by R.A. No. 7055 is necessary to preserve the peculiar nature of
military justice system over military personnel charged with service-connected offenses. The military
justice system is disciplinary in nature, aimed at achieving the highest form of discipline in order to
ensure the highest degree of military efficiency. 18 Military law is established not merely to enforce
discipline in times of war, but also to preserve the tranquility and security of the State in time of
peace; for there is nothing more dangerous to the public peace and safety than a licentious and
undisciplined military body. 19 The administration of military justice has been universally practiced.
Since time immemorial, all the armies in almost all countries of the world look upon the power of
military law and its administration as the most effective means of enforcing discipline. For this
reason, the court martial has become invariably an indispensable part of any organized armed
forces, it being the most potent agency in enforcing discipline both in peace and in war. 20

Here, petitioners are charged for violation of Article 96 (conduct unbecoming an officer and a
gentleman) of the Articles of War before the court martial, thus:

All persons subject to military law, did on or about 27 July 2003 at Oakwood Hotel, Makati City,
Metro Manila, willfully, unlawfully and feloniously violate their solemn oath as officers to defend
the Constitution, the law and the duly-constituted authorities and abused their constitutional
duty to protect the people and the State by, among others, attempting to oust the incumbent duly-
elected and legitimate President by force and violence, seriously disturbing the peace and tranquility
of the people and the nation they are sworn to protect, thereby causing dishonor and disrespect
to the military profession, conduct unbecoming an officer and a gentleman, in violation of AW
96 of the Articles of War.

CONTRARY TO LAW. (Underscoring ours)

Article 96 of the Articles of War 21 provides:

ART. 96. Conduct Unbecoming an Officer and Gentleman. – Any officer, member of the Nurse
Corps, cadet, flying cadet, or probationary second lieutenant, who is convicted of conduct
unbecoming an officer and a gentleman shall be dismissed from the service. (Underscoring ours)

We hold that the offense for violation of Article 96 of the Articles of War is service-connected. This is
expressly provided in Section 1 (second paragraph) of R.A. No. 7055. It bears stressing that the
charge against the petitioners concerns the alleged violation of their solemn oath as officers to
defend the Constitution and the duly-constituted authorities.Such violation allegedly caused
dishonor and disrespect to the military profession. In short, the charge has a bearing on
their professional conduct or behavior as military officers. Equally indicative of the "service-
connected" nature of the offense is the penalty prescribed for the same – dismissal from the
service – imposable only by the military court.Such penalty is purely disciplinary in character,
evidently intended to cleanse the military profession of misfits and to preserve the stringent standard
of military discipline.

Obviously, there is no merit in petitioners’ argument that they can no longer be charged before the
court martial for violation of Article 96 of the Articles of War because the same has been declared by
the RTC in its Order of February 11, 2004 as "not service-connected, but rather absorbed and in
furtherance of the alleged crime of coup d’etat," hence, triable by said court (RTC). The RTC, in
making such declaration, practically amended the law which expressly vests in the court martial the
jurisdiction over "service-connected crimes or offenses." What the law has conferred the court
should not take away. It is only the Constitution or the law that bestows jurisdiction on the court,
tribunal, body or officer over the subject matter or nature of an action which can do so. 22 And it is
only through a constitutional amendment or legislative enactment that such act can be done. The
first and fundamental duty of the courts is merely to apply the law "as they find it, not as they like it to
be." 23 Evidently, such declaration by the RTC constitutes grave abuse of discretion tantamount to
lack or excess of jurisdiction and is, therefore, void.

In Navales v. Abaya., 24 this Court, through Mr. Justice Romeo J. Callejo, Sr., held:

We agree with the respondents that the sweeping declaration made by the RTC (Branch 148) in the
dispositive portion of its Order dated February 11, 2004 that all charges before the court-martial
against the accused were not service-connected, but absorbed and in furtherance of the crime of
coup d’etat, cannot be given effect. x x x, such declaration was made without or in excess of
jurisdiction; hence, a nullity.

The second paragraph of the above provision (referring to Section 1 of R.A. No. 7055) explicitly
specifies what are considered "service-connected crimes or offenses" under Commonwealth Act No.
408, as amended, also known as the Articles of War, to wit:

Articles 54 to 70:
Art. 54. Fraudulent Enlistment.

Art. 55. Officer Making Unlawful Enlistment.

Art. 56. False Muster.

Art. 57. False Returns.

Art. 58. Certain Acts to Constitute Desertion.

Art. 59. Desertion.

Art. 60. Advising or Aiding Another to Desert.

Art. 61. Entertaining a Deserter.

Art. 62. Absence Without Leave.

Art. 63. Disrespect Toward the President, Vice-President,

Congress of the Philippines, or Secretary of National

Defense.

Art. 64. Disrespect Toward Superior Officer.

Art. 65. Assaulting or Willfully Disobeying Superior Officer.

Art. 66. Insubordinate Conduct Toward Non-Commissioned Officer.

Art. 67. Mutiny or Sedition.

Art. 68. Failure to Suppress Mutiny or Sedition.

Art. 69. Quarrels; Frays; Disorders.

Art. 70. Arrest or Confinement.

Articles 72 to 92:

Art. 72. Refusal to Receive and Keep Prisoners.

Art. 73. Report of Prisoners Received.

Art. 74. Releasing Prisoner Without Authority.

Art. 75. Delivery of Offenders to Civil Authorities.

Art. 76. Misbehavior Before the Enemy.


Art. 77. Subordinates Compelling Commander to Surrender.

Art. 78. Improper Use of Countersign.

Art. 79. Forcing a Safeguard.

Art. 80. Captured Property to be Secured for Public Service.

Art. 81. Dealing in Captured or Abandoned Property.

Art. 82. Relieving, Corresponding With, or Aiding the Enemy.

Art. 83. Spies.

Art. 84. Military Property.–Willful or Negligent Loss, Damage

or wrongful Disposition.

Art. 85. Waste or Unlawful Disposition of Military Property

Issued to Soldiers.

Art. 86. Drunk on Duty.

Art. 87. Misbehavior of Sentinel.

Art. 88. Personal Interest in Sale of Provisions.

Art. 88-A. Unlawful Influencing Action of Court.

Art. 89. Intimidation of Persons Bringing Provisions.

Art. 90. Good Order to be Maintained and Wrongs Redressed.

Art. 91. Provoking Speeches or Gestures.

Art. 92. Dueling.

Articles 95 to 97:

Art. 95. Frauds Against the Government.

Art. 96. Conduct Unbecoming an Officer and Gentleman.

Art. 97. General Article.

Further, Section 1 of Rep. Act No. 7055 vests on the military courts the jurisdiction over the
foregoing offenses. x x x.
It is clear from the foregoing that Rep. Act No. 7055 did not divest the military courts of jurisdiction to
try cases involving violations of Articles 54 to 70, Articles 72 to 92, and Articles 95 to 97 of the
Articles of War as these are considered "service-connected crimes or offenses." In fact, it mandates
that these shall be tried by the court-martial.

Moreover, the observation made by Mr. Justice Antonio T. Carpio during the deliberation of this case
is worth quoting, thus:

The trial court aggravated its error when it justified its ruling by holding that the charge of Conduct
Unbecoming an Officer and a Gentleman is ‘absorbed and in furtherance to the alleged crime of
coup d’etat.’ Firstly, the doctrine of ‘absorption of crimes’ is peculiar to criminal law and generally
applies to crimes punished by the same statute, 25unlike here where different statutes are involved.
Secondly, the doctrine applies only if the trial court has jurisdiction over both offenses. Here, Section
1 of R.A. 7055 deprives civil courts of jurisdiction over service-connected offenses, including Article
96 of the Articles of War. Thus, the doctrine of absorption of crimes is not applicable to this case.

Military law is sui generis (Calley v. Callaway, 519 F.2d 184 [1975]), applicable only to military
personnel because the military constitutes an armed organization requiring a system of discipline
separate from that of civilians (see Orloff v. Willoughby, 345 U.S. 83 [1953]). Military personnel carry
high-powered arms and other lethal weapons not allowed to civilians. History, experience, and the
nature of a military organization dictate that military personnel must be subjected to a separate
disciplinary system not applicable to unarmed civilians or unarmed government personnel.

A civilian government employee reassigned to another place by his superior may question his
reassignment by asking a temporary restraining order or injunction from a civil court. However, a
soldier cannot go to a civil court and ask for a restraining or injunction if his military commander
reassigns him to another area of military operations. If this is allowed, military discipline will collapse.

xxx

This Court has recognized that courts-martial are instrumentalities of the Executive to enable the
President, as Commander-in-Chief, to effectively command, control, and discipline the armed forces
(see Ruffy v. Chief of Staff, 75 Phil. 875 [1946], citing Winthrop’s Military Law and Precedents, 2nd
edition, p. 49). In short, courts-martial form part of the disciplinary system that ensures the
President’s control, and thus civilian supremacy, over the military. At the apex of this disciplinary
system is the President who exercises review powers over decisions of courts-martial (citing Article
50 of the Articles of War; quoted provisions omitted).

xxx

While the Court had intervened before in courts-martial or similar proceedings, it did so sparingly
and only to release a military personnel illegally detained (Ognir v. Director of Prisons, 80 Phil. 401
[1948] or to correct objectionable procedures (Yamashita v. Styer, 75 Phil. 563 [1945]). The Court
has never suppressed court-martial proceedings on the ground that the offense charged ‘is
absorbed and in furtherance of’ another criminal charge pending with the civil courts. The Court may
now do so only if the offense charged is not one of the service-connected offenses specified in
Section 1 of RA 7055. Such is not the situation in the present case.

With respect to the issue of prescription raised by petitioners in their Supplemental Petition, suffice it
to say that we cannot entertain the same. The contending parties are at loggerheads as to (a) who
among the petitioners were actually arraigned, and (b) the dates of their arraignment. These are
matters involving questions of fact, not within our power of review, as we are not a trier of facts. In a
petition for prohibition, such as the one at bar, only legal issues affecting the jurisdiction of the
tribunal, board or officer involved may be resolved on the basis of the undisputed facts. 26

Clearly, the instant petition for prohibition must fail. The office of prohibition is to prevent the unlawful
and oppressive exercise of authority and is directed against proceedings that are done without or in
excess of jurisdiction, or with grave abuse of discretion, there being no appeal or other plain,
speedy, and adequate remedy in the ordinary course of law. 27 Stated differently, prohibition is the
remedy to prevent inferior courts, corporations, boards, or persons from usurping or exercising a
jurisdiction or power with which they have not been vested by law. 28

In fine, this Court holds that herein respondents have the authority in convening a court martial and
in charging petitioners with violation of Article 96 of the Articles of War.

WHEREFORE, the instant petition for prohibition is DISMISSED.

SO ORDERED.

[A.M. NO. P-02-1651 : June 22, 2006]


(Formerly OCA I.P.I. No. 00-1021-P)

ALEJANDRO ESTRADA, Complainant, v. SOLEDAD S. ESCRITOR, Respondent.

RESOLUTION

PUNO, J.:

While man is finite, he seeks and subscribes to the Infinite. Respondent Soledad Escritor once again stands
before the Court invoking her religious freedom and her Jehovah God in a bid to save her family - united
without the benefit of legal marriage - and livelihood. The State, on the other hand, seeks to wield its power
to regulate her behavior and protect its interest in marriage and family and the integrity of the courts where
respondent is an employee. How the Court will tilt the scales of justice in the case at bar will decide not only
the fate of respondent Escritor but of other believers coming to Court bearing grievances on their free
exercise of religion. This case comes to us from our remand to the Office of the Court Administrator on
August 4, 2003.1

I. THE PAST PROCEEDINGS

In a sworn-letter complaint dated July 27, 2000, complainant Alejandro Estrada requested Judge Jose F.
Caoibes, Jr., presiding judge of Branch 253, Regional Trial Court of Las Piñas City, for an investigation of
respondent Soledad Escritor, court interpreter in said court, for living with a man not her husband, and
having borne a child within this live-in arrangement. Estrada believes that Escritor is committing an immoral
act that tarnishes the image of the court, thus she should not be allowed to remain employed therein as it
might appear that the court condones her act.2 Consequently, respondent was charged with committing
"disgraceful and immoral conduct" under Book V, Title I, Chapter VI, Sec. 46(b)(5) of the Revised
Administrative Code.3

Respondent Escritor testified that when she entered the judiciary in 1999, she was already a widow, her
husband having died in 1998.4 She admitted that she started living with Luciano Quilapio, Jr. without the
benefit of marriage more than twenty years ago when her husband was still alive but living with another
woman. She also admitted that she and Quilapio have a son. 5 But as a member of the religious sect known
as the Jehovah's Witnesses and the Watch Tower and Bible Tract Society, respondent asserted that their
conjugal arrangement is in conformity with their religious beliefs and has the approval of her
congregation.6 In fact, after ten years of living together, she executed on July 28, 1991, a "Declaration of
Pledging Faithfulness."7

For Jehovah's Witnesses, the Declaration allows members of the congregation who have been abandoned by
their spouses to enter into marital relations. The Declaration thus makes the resulting union moral and
binding within the congregation all over the world except in countries where divorce is allowed. As laid out
by the tenets of their faith, the Jehovah's congregation requires that at the time the declarations are
executed, the couple cannot secure the civil authorities' approval of the marital relationship because of legal
impediments. Only couples who have been baptized and in good standing may execute the Declaration,
which requires the approval of the elders of the congregation. As a matter of practice, the marital status of
the declarants and their respective spouses' commission of adultery are investigated before the declarations
are executed.8 Escritor and Quilapio's declarations were executed in the usual and approved form prescribed
by the Jehovah's Witnesses,9 approved by elders of the congregation where the declarations were
executed,10 and recorded in the Watch Tower Central Office.11

Moreover, the Jehovah's congregation believes that once all legal impediments for the couple are lifted, the
validity of the declarations ceases, and the couple should legalize their union. In Escritor's case, although
she was widowed in 1998, thereby lifting the legal impediment to marry on her part, her mate was still not
capacitated to remarry. Thus, their declarations remained valid. 12 In sum, therefore, insofar as the
congregation is concerned, there is nothing immoral about the conjugal arrangement between Escritor and
Quilapio and they remain members in good standing in the congregation.

By invoking the religious beliefs, practices and moral standards of her congregation, in asserting that her
conjugal arrangement does not constitute disgraceful and immoral conduct for which she should be held
administratively liable,13 the Court had to determine the contours of religious freedom under Article III,
Section 5 of the Constitution, which provides, viz:

Sec. 5. No law shall be made respecting an establishment of religion, or prohibiting the free exercise
thereof. The free exercise and enjoyment of religious profession and worship, without discrimination or
preference, shall forever be allowed. No religious test shall be required for the exercise of civil or political
rights.

A. Ruling

In our decision dated August 4, 2003, after a long and arduous scrutiny into the origins and development of
the religion clauses in the United States (U.S.) and the Philippines, we held that in resolving claims involving
religious freedom (1) benevolent neutrality or accommodation, whether mandatory or permissive, is the
spirit, intent and framework underlying the religion clauses in our Constitution; and (2) in deciding
respondent's plea of exemption based on the Free Exercise Clause (from the law with which she is
administratively charged), it is the compelling state interest test, the strictest test, which must be applied.14

Notwithstanding the above rulings, the Court could not, at that time, rule definitively on the ultimate issue
of whether respondent was to be held administratively liable for there was need to give the State the
opportunity to adduce evidence that it has a more "compelling interest" to defeat the claim of the
respondent to religious freedom. Thus, in the decision dated August 4, 2003, we remanded the complaint to
the Office of the Court Administrator (OCA), and ordered the Office of the Solicitor General (OSG) to
intervene in the case so it can:

(a) examine the sincerity and centrality of respondent's claimed religious belief and practice;

(b) present evidence on the state's "compelling interest" to override respondent's religious belief and
practice; andcralawlibrary

(c) show that the means the state adopts in pursuing its interest is the least restrictive to respondent's
religious freedom.15

It bears stressing, therefore, that the residual issues of the case pertained NOT TO WHAT APPROACH THIS
COURT SHOULD TAKE IN CONSTRUING THE RELIGION CLAUSES, NOR TO THE PROPER TEST APPLICABLE IN
DETERMINING CLAIMS OF EXEMPTION BASED ON FREEDOM OF RELIGION. These issues have already been
ruled upon prior to the remand, and constitute "the law of the case" insofar as they resolved the issues of
which framework and test are to be applied in this case, and no motion for its reconsideration having been
filed.16 The only task that the Court is left to do is to determine whether the evidence adduced by the State
proves its more compelling interest. This issue involves a pure question of fact.

B. Law of the case

Mr. Justice Carpio's insistence, in his dissent, in attacking the ruling of this case interpreting the religious
clauses of the Constitution, made more than two years ago, is misplaced to say the least. Since neither the
complainant, respondent nor the government has filed a motion for reconsideration assailing this ruling, the
same has attained finality and constitutes the law of the case. Any attempt to reopen this final ruling
constitutes a crass contravention of elementary rules of procedure. Worse, insofar as it would overturn the
parties' right to rely upon our interpretation which has long attained finality, it also runs counter to
substantive due process.

Be that as it may, even assuming that there were no procedural and substantive infirmities in Mr. Justice
Carpio's belated attempts to disturb settled issues, and that he had timely presented his arguments, the
results would still be the same.

We review the highlights of our decision dated August 4, 2003.

1. Old World Antecedents

In our August 4, 2003 decision, we made a painstaking review of Old World antecedents of the religion
clauses, because "one cannot understand, much less intelligently criticize the approaches of the courts and
the political branches to religious freedom in the recent past in the United States without a deep
appreciation of the roots of these controversies in the ancient and medieval world and in the American
experience."17 We delved into the conception of religion from primitive times, when it started out as the
state

itself, when the authority and power of the state were ascribed to God. 18 Then, religion developed on its own
and became superior to the state,19 its subordinate,20 and even becoming an engine of state policy.21

We ascertained two salient features in the review of religious history: First, with minor exceptions, the
history of church-state relationships was characterized by persecution, oppression, hatred, bloodshed, and
war, all in the name of the God of Love and of the Prince of Peace. Second, likewise with minor exceptions,
this history witnessed the unscrupulous use of religion by secular powers to promote secular purposes and
policies, and the willing acceptance of that role by the vanguards of religion in exchange for the favors and
mundane benefits conferred by ambitious princes and emperors in exchange for religion's invaluable service.
This was the context in which the unique experiment of the principle of religious freedom and separation of
church and state saw its birth in American constitutional democracy and in human history.22

Strictly speaking, the American experiment of freedom and separation was not translated in the First
Amendment. That experiment had been launched four years earlier, when the founders of the republic
carefully withheld from the new national government any power to deal with religion. As James Madison
said, the national government had no "jurisdiction" over religion or any "shadow of right to intermeddle"
with it.23

The omission of an express guaranty of religious freedom and other natural rights, however, nearly
prevented the ratification of the Constitution. The restriction had to be made explicit with the adoption of
the religion clauses in the First Amendment as they are worded to this day. Thus, the First Amendment did
not take away or abridge any power of the national government; its intent was to make express the absence
of power.24 It commands, in two parts (with the first part usually referred to as the Establishment Clause
and the second part, the Free Exercise Clause), viz:

Congress shall make no law respecting an establishment of religion or prohibiting the free exercise thereof.25
The Establishment and Free Exercise Clauses, it should be noted, were not designed to serve contradictory
purposes. They have a single goal to promote freedom of individual religious beliefs and practices. In
simplest terms, the Free Exercise Clause prohibits government from inhibiting religious beliefs with penalties
for religious beliefs and practice, while the Establishment Clause prohibits government from inhibiting
religious belief with rewards for religious beliefs and practices. In other words, the two religion clauses were
intended to deny government the power to use either the carrot or the stick to influence individual religious
beliefs and practices.26

In sum, a review of the Old World antecedents of religion shows the movement of establishment of religion
as an engine to promote state interests, to the principle of non-establishment to allow the free exercise of
religion.

2. Religion Clauses in the U.S. Context

The Court then turned to the religion clauses' interpretation and construction in the United States, not
because we are bound by their interpretation, but because the U.S. religion clauses are the precursors to the
Philippine religion clauses, although we have significantly departed from the U.S. interpretation as will be
discussed later on.

At the outset, it is worth noting that American jurisprudence in this area has been volatile and fraught with
inconsistencies whether within a Court decision or across decisions. For while there is widespread agreement
regarding the value of the First Amendment religion clauses, there is an equally broad disagreement as to
what these clauses specifically require, permit and forbid. No agreement has been reached by those who
have studied the religion clauses as regards its exact meaning and the paucity of records in the U.S.
Congress renders it difficult to ascertain its meaning.27

U.S. history has produced two identifiably different, even opposing, strains of jurisprudence on the religion
clauses. First is the standard of separation, which may take the form of either (a) strict separation or (b) the
tamer version of strict neutrality or separation, or what Mr. Justice Carpio refers to as the second theory of
governmental neutrality. Although the latter form is not as hostile to religion as the former, both are
anchored on the Jeffersonian premise that a "wall of separation" must exist between the state and the
Church to protect the state from the church.28 Both protect the principle of church-state separation with a
rigid reading of the principle. On the other hand, the second standard, the benevolent neutrality or
accommodation, is buttressed by the view that the wall of separation is meant to protect the church from
the state. A brief review of each theory is in order.

A. Strict Separation and Strict Neutrality/Separation

The Strict Separationist believes that the Establishment Clause was meant to protect the state from the
church, and the state's hostility towards religion allows no interaction between the two. According to this
Jeffersonian view, an absolute barrier to formal interdependence of religion and state needs to be erected.
Religious institutions could not receive aid, whether direct or indirect, from the state. Nor could the state
adjust its secular programs to alleviate burdens the programs placed on believers. 29 Only the complete
separation of religion from politics would eliminate the formal influence of religious institutions and provide
for a free choice among political views, thus a strict "wall of separation" is necessary.30

Strict separation faces difficulties, however, as it is deeply embedded in American history and contemporary
practice that enormous amounts of aid, both direct and indirect, flow to religion from government in return
for huge amounts of mostly indirect aid from religion.31 For example, less than twenty-four hours after
Congress adopted the First Amendment's prohibition on laws respecting an establishment of religion,
Congress decided to express its thanks to God Almighty for the many blessings enjoyed by the nation with a
resolution in favor of a presidential proclamation declaring a national day of Thanksgiving and
Prayer.32 Thus, strict separationists are caught in an awkward position of claiming a constitutional principle
that has never existed and is never likely to.33

The tamer version of the strict separationist view, the strict neutrality or separationist view, (or, the
governmental neutrality theory) finds basis in Everson v. Board of Education,34 where the Court declared
that Jefferson's "wall of separation" encapsulated the meaning of the First Amendment. However, unlike the
strict separationists, the strict neutrality view believes that the "wall of separation" does not require the
state to be their adversary. Rather, the state must be neutral in its relations with groups of religious
believers and non-believers. "State power is no more to be used so as to handicap religions than it is to
favor them."35 The strict neutrality approach is not hostile to religion, but it is strict in holding that religion
may not be used as a basis for classification for purposes of governmental action, whether the action confers
rights or privileges or imposes duties or obligations. Only secular criteria may be the basis of government
action. It does not permit, much less require, accommodation of secular programs to religious belief.36

The problem with the strict neutrality approach, however, is if applied in interpreting the Establishment
Clause, it could lead to a de facto voiding of religious expression in the Free Exercise Clause. As pointed out
by Justice Goldberg in his concurring opinion in Abington School District v. Schempp,37 strict neutrality could
lead to "a brooding and pervasive devotion to the secular and a passive, or even active, hostility to the
religious" which is prohibited by the Constitution. 38 Professor Laurence Tribe commented in his authoritative
treatise, viz:

To most observers. . . strict neutrality has seemed incompatible with the very idea of a free exercise clause.
The Framers, whatever specific applications they may have intended, clearly envisioned religion as
something special; they enacted that vision into law by guaranteeing the free exercise of religion but not,
say, of philosophy or science. The strict neutrality approach all but erases this distinction. Thus it is not
surprising that the [U.S.] Supreme Court has rejected strict neutrality, permitting and sometimes mandating
religious classifications.39

Thus, the dilemma of the separationist approach, whether in the form of strict separation or strict neutrality,
is that while the Jeffersonian wall of separation "captures the spirit of the American ideal of church-state
separation," in real life, church and state are not and cannot be totally separate. This is all the more true in
contemporary times when both the government and religion are growing and expanding their spheres of
involvement and activity, resulting in the intersection of government and religion at many points.40

b. Benevolent Neutrality/Accommodation

The theory of benevolent neutrality or accommodation is premised on a different view of the "wall of
separation," associated with Williams, founder of the Rhode Island colony. Unlike the Jeffersonian wall that
is meant to protect the state from the church, the wall is meant to protect the church from the state. 41 This
doctrine was expressed in Zorach v. Clauson,42 which held, viz:

The First Amendment, however, does not say that in every and all respects there shall be a separation of
Church and State. Rather, it studiously defines the manner, the specific ways, in which there shall be no
concert or union or dependency one or the other. That is the common sense of the matter. Otherwise, the
state and religion would be aliens to each other - hostile, suspicious, and even unfriendly. Churches could
not be required to pay even property taxes. Municipalities would not be permitted to render police or fire
protection to religious groups. Policemen who helped parishioners into their places of worship would violate
the Constitution. Prayers in our legislative halls; the appeals to the Almighty in the messages of the Chief
Executive; the proclamations making Thanksgiving Day a holiday; "so help me God" in our courtroom oaths
- these and all other references to the Almighty that run through our laws, our public rituals, our ceremonies
would be flouting the First Amendment. A fastidious atheist or agnostic could even object to the supplication
with which the Court opens each session: "God save the United States and this Honorable Court."

xxx xxx xxx

We are a religious people whose institutions presuppose a Supreme Being. We guarantee the freedom to
worship as one chooses. . . When the state encourages religious instruction or cooperates with religious
authorities by adjusting the schedule of public events, it follows the best of our traditions. For it then
respects the religious nature of our people and accommodates the public service to their spiritual needs. To
hold that it may not would be to find in the Constitution a requirement that the government show a callous
indifference to religious groups. . . But we find no constitutional requirement which makes it necessary for
government to be hostile to religion and to throw its weight against efforts to widen their effective scope of
religious influence.43

Benevolent neutrality recognizes that religion plays an important role in the public life of the United States
as shown by many traditional government practices which, to strict neutrality, pose Establishment Clause
questions. Among these are the inscription of "In God We Trust" on American currency; the recognition of
America as "one nation under God" in the official pledge of allegiance to the flag; the Supreme Court's time-
honored practice of opening oral argument with the invocation "God save the United States and this
Honorable Court"; and the practice of Congress and every state legislature of paying a chaplain, usually of a
particular Protestant denomination, to lead representatives in prayer. These practices clearly show the
preference for one theological viewpoint the existence of and potential for intervention by a god over the
contrary theological viewpoint of atheism. Church and government agencies also cooperate in the building of
low-cost housing and in other forms of poor relief, in the treatment of alcoholism and drug addiction, in
foreign aid and other government activities with strong moral dimension.44

Examples of accommodations in American jurisprudence also abound, including, but not limited to the U.S.
Court declaring the following acts as constitutional: a state hiring a Presbyterian minister to lead the
legislature in daily prayers,45 or requiring employers to pay workers compensation when the resulting
inconsistency between work and Sabbath leads to discharge; 46 for government to give money to religiously-
affiliated organizations to teach adolescents about proper sexual behavior;47 or to provide religious school
pupils with books;48 or bus rides to religious schools;49 or with cash to pay for state-mandated standardized
tests.50

(1) Legislative Acts and the Free Exercise Clause

As with the other rights under the Constitution, the rights embodied in the Religion clauses are invoked in
relation to governmental action, almost invariably in the form of legislative acts.

Generally speaking, a legislative act that purposely aids or inhibits religion will be challenged as
unconstitutional, either because it violates the Free Exercise Clause or the Establishment Clause or both.
This is true whether one subscribes to the separationist approach or the benevolent neutrality or
accommodationist approach.

But the more difficult religion cases involve legislative acts which have a secular purpose and general
applicability, but may incidentally or inadvertently aid or burden religious exercise. Though the government
action is not religiously motivated, these laws have a "burdensome effect" on religious exercise.

The benevolent neutrality theory believes that with respect to these governmental actions, accommodation
of religion may be allowed, not to promote the government's favored form of religion, but to allow
individuals and groups to exercise their religion without hindrance. The purpose of accommodations is to
remove a burden on, or facilitate the exercise of, a person's or institution's religion. As Justice Brennan
explained, the "government [may] take religion into account to exempt, when possible, from generally
applicable governmental regulation individuals whose religious beliefs and practices would otherwise thereby
be infringed, or to create without state involvement an atmosphere in which voluntary religious exercise
may flourish."51 In the ideal world, the legislature would recognize the religions and their practices and
would consider them, when practical, in enacting laws of general application. But when the legislature fails
to do so, religions that are threatened and burdened may turn to the courts for protection.52

Thus, what is sought under the theory of accommodation is not a declaration of unconstitutionality of a
facially neutral law, but an exemption from its application or its "burdensome effect," whether by the
legislature or the courts.53 Most of the free exercise claims brought to the U.S. Court are for exemption, not
invalidation of the facially neutral law that has a "burdensome" effect.54

(2) Free Exercise Jurisprudence: Sherbert, Yoder and Smith

The pinnacle of free exercise protection and the theory of accommodation in the U.S. blossomed in the case
of Sherbert v. Verner,55 which ruled that state regulation that indirectly restrains or punishes religious belief
or conduct must be subjected to strict scrutiny under the Free Exercise Clause. 56According to Sherbert, when
a law of general application infringes religious exercise, albeit incidentally, the state interest sought to be
promoted must be so paramount and compelling as to override the free exercise claim. Otherwise, the Court
itself will carve out the exemption.

In this case, Sherbert, a Seventh Day Adventist, claimed unemployment compensation under the law as her
employment was terminated for refusal to work on Saturdays on religious grounds. Her claim was denied.
She sought recourse in the Supreme Court. In laying down the standard for determining whether the denial
of benefits could withstand constitutional scrutiny, the Court ruled, viz:

Plainly enough, appellee's conscientious objection to Saturday work constitutes no conduct prompted by
religious principles of a kind within the reach of state legislation. If, therefore, the decision of the South
Carolina Supreme Court is to withstand appellant's constitutional challenge, it must be either because her
disqualification as a beneficiary represents no infringement by the State of her constitutional right of free
exercise, or because any incidental burden on the free exercise of appellant's religion may be justified by a
"compelling state interest in the regulation of a subject within the State's constitutional power to
regulate. . . ."57 (emphasis supplied)

The Court stressed that in the area of religious liberty, it is basic that it is not sufficient to merely show a
rational relationship of the substantial infringement to the religious right and a colorable state interest. "(I)n
this highly sensitive constitutional area, '[o]nly the gravest abuses, endangering paramount interests, give
occasion for permissible limitation.' "58 The Court found that there was no such compelling state interest to
override Sherbert's religious liberty. It added that even if the state could show that Sherbert's exemption
would pose serious detrimental effects to the unemployment compensation fund and scheduling of work, it
was incumbent upon the state to show that no alternative means of regulations would address such
detrimental effects without infringing religious liberty. The state, however, did not discharge this burden.
The Court thus carved out for Sherbert an exemption from the Saturday work requirement that caused her
disqualification from claiming the unemployment benefits. The Court reasoned that upholding the denial of
Sherbert's benefits would force her to choose between receiving benefits and following her religion. This
choice placed "the same kind of burden upon the free exercise of religion as would a fine imposed against
(her) for her Saturday worship." This germinal case of Sherbert firmly established the exemption
doctrine, 59 viz:

It is certain that not every conscience can be accommodated by all the laws of the land; but when general
laws conflict with scruples of conscience, exemptions ought to be granted unless some "compelling state
interest" intervenes.

Thus, Sherbert and subsequent cases held that when government action burdens, even inadvertently, a
sincerely held religious belief or practice, the state must justify the burden by demonstrating that the law
embodies a compelling interest, that no less restrictive alternative exists, and that a religious exemption
would impair the state's ability to effectuate its compelling interest. As in other instances of state action
affecting fundamental rights, negative impacts on those rights demand the highest level of judicial scrutiny.
After Sherbert, this strict scrutiny balancing test resulted in court-mandated religious exemptions from
facially-neutral laws of general application whenever unjustified burdens were found.60

Then, in the 1972 case of Wisconsin v. Yoder,61 the U.S. Court again ruled that religious exemption was in
order, notwithstanding that the law of general application had a criminal penalty. Using heightened scrutiny,
the Court overturned the conviction of Amish parents for violating Wisconsin compulsory school-attendance
laws. The Court, in effect, granted exemption from a neutral, criminal statute that punished religiously
motivated conduct. Chief Justice Burger, writing for the majority, held, viz:

It follows that in order for Wisconsin to compel school attendance beyond the eighth grade against a claim
that such attendance interferes with the practice of a legitimate religious belief, it must appear either that
the State does not deny the free exercise of religious belief by its requirement, or that there is a state
interest of sufficient magnitude to override the interest claiming protection under the Free Exercise Clause.
Long before there was general acknowledgement of the need for universal education, the Religion Clauses
had specially and firmly fixed the right of free exercise of religious beliefs, and buttressing this fundamental
right was an equally firm, even if less explicit, prohibition against the establishment of any religion. The
values underlying these two provisions relating to religion have been zealously protected, sometimes even
at the expense of other interests of admittedly high social importance. . .

The essence of all that has been said and written on the subject is that only those interests of the highest
order and those not otherwise served can overbalance legitimate claims to the free exercise of religion. . .

. . . our decisions have rejected the idea that religiously grounded conduct is always outside the protection
of the Free Exercise Clause. It is true that activities of individuals, even when religiously based, are often
subject to regulation by the States in the exercise of their undoubted power to promote the health, safety,
and general welfare, or the Federal government in the exercise of its delegated powers . . . But to agree
that religiously grounded conduct must often be subject to the broad police power of the State is not to deny
that there are areas of conduct protected by the Free Exercise Clause of the First Amendment and thus
beyond the power of the State to control, even under regulations of general applicability. . . .This case,
therefore, does not become easier because respondents were convicted for their "actions" in refusing to
send their children to the public high school; in this context belief and action cannot be neatly confined in
logic-tight compartments. . .62

The cases of Sherbert and Yoder laid out the following doctrines: (a) free exercise clause claims were
subject to heightened scrutiny or compelling interest test if government substantially burdened the exercise
of religion; (b) heightened scrutiny or compelling interest test governed cases where the burden was direct,
i.e., the exercise of religion triggered a criminal or civil penalty, as well as cases where the burden was
indirect, i.e., the exercise of religion resulted in the forfeiture of a government benefit; 63 and (c) the Court
could carve out accommodations or exemptions from a facially neutral law of general application, whether
general or criminal.

The Sherbert-Yoder doctrine had five main components. First, action was protected conduct beyond speech,
press, or worship was included in the shelter of freedom of religion. Neither Sherbert's refusal to work on
the Sabbath nor the Amish parents' refusal to let their children attend ninth and tenth grades can be
classified as conduct protected by the other clauses of the First Amendment. Second, indirect impositions on
religious conduct, such as the denial of twenty-six weeks of unemployment insurance benefits to Adel
Sherbert, as well as direct restraints, such as the criminal prohibition at issue in Yoder, were prohibited.
Third, as the language in the two cases indicate, the protection granted was extensive. Only extremely
strong governmental interests justified impingement on religious conduct, as the absolute language of the
test of the Free Exercise Clause suggests.64

Fourth, the strong language was backed by a requirement that the government provide proof of the
important interest at stake and of the dangers to that interest presented by the religious conduct at issue.
Fifth, in determining the injury to the government's interest, a court was required to focus on the effect that
exempting religious claimants from the regulation would have, rather than on the value of the regulation in
general. Thus, injury to governmental interest had to be measured at the margin: assuming the law still
applied to all others, what would be the effect of exempting the religious claimant in this case and other
similarly situated religious claimants in the future? Together, the fourth and fifth elements required that
facts, rather than speculation, had to be presented concerning how the government's interest would be
harmed by excepting religious conduct from the law being challenged.65

Sherbert and Yoder adopted a balancing test for free exercise jurisprudence which would impose a discipline
to prevent manipulation in the balancing of interests. The fourth and the fifth elements prevented the
likelihood of exaggeration of the weight on the governmental interest side of the balance, by not allowing
speculation about the effects of a decision adverse to those interests nor accepting that those interests
would be defined at a higher level of generality than the constitutional interests on the other side of the
balance.66

Thus, the strict scrutiny and compelling state interest test significantly increased the degree of protection
afforded to religiously motivated conduct. While not affording absolute immunity to religious activity, a
compelling secular justification was necessary to uphold public policies that collided with religious practices.
Although the members of the U.S. Court often disagreed over which governmental interests should be
considered compelling, thereby producing dissenting and separate opinions in religious conduct cases, this
general test established a strong presumption in favor of the free exercise of religion. 67 Most scholars and
courts agreed that under Sherbert and Yoder, the Free Exercise Clause provided individuals some form of
heightened scrutiny protection, if not always a compelling interest one.68 The 1990 case of Employment
Division, Oregon Department of Human Resources v. Smith,69 drastically changed all that.

Smith involved a challenge by Native Americans to an Oregon law prohibiting use of peyote, a hallucinogenic
substance. Specifically, individuals challenged the state's determination that their religious use of peyote,
which resulted in their dismissal from employment, was misconduct disqualifying them from receipt of
unemployment compensation benefits.70
Justice Scalia, writing for the majority, rejected the claim that free exercise of religion required an
exemption from an otherwise valid law. Scalia said that "[w]e have never held that an individual's religious
beliefs excuse him from compliance with an otherwise valid law prohibiting conduct that the State is free to
regulate. On the contrary, the record of more than a century of our free exercise jurisprudence contradicts
that proposition." 71 Scalia thus declared "that the right of free exercise does not relieve an individual of the
obligation to comply with a 'valid and neutral law of general applicability of the ground that the law
proscribes (or prescribes) conduct that his religion prescribes (or proscribes).' " 72

Justice Scalia's opinion then reviewed the cases where free exercise challenges had been upheld'such as
Cantwell, Murdock, Follet, Pierce, and Yoder and said that none involved the free exercise clause claims
alone. All involved "the Free Exercise Clause in conjunction with other constitutional protections, such as
freedom of speech and of the press, or the right of parents to direct the education of their children." 73The
Court said that Smith was distinguishable because it did not involve such a "hybrid situation," but was a free
exercise claim "unconnected with any communicative activity or parental right." 74

Moreover, the Court said that the Sherbert line of cases applied only in the context of the denial of
unemployment benefits; it did not create a basis for an exemption from criminal laws. Scalia wrote that
"[e]ven if we were inclined to breathe into Sherbert some life beyond the unemployment compensation field,
we would not apply it to require exemptions from a generally applicable criminal law." 75

The Court expressly rejected the use of strict scrutiny for challenges to neutral laws of general applicability
that burden religion. Justice Scalia said that "[p]recisely because 'we are a cosmopolitan nation made up of
people of almost conceivable religious preference,' and precisely because we value and protect that religious
divergence, we cannot afford the luxury of deeming presumptively invalid, as applied to the religious
objector, every regulation of conduct that does not protect an interest of the highest order." The Court said
that those seeking religious exemptions from laws should look to the democratic process for protection, not
the courts.76

Smith thus changed the test for the free exercise clause. Strict or heightened scrutiny and the compelling
justification approach were abandoned for evaluating laws burdening religion; neutral laws of general
applicability only have to meet the rational basis test, no matter how much they burden religion.77

Justice O Connor wrote a concurring opinion sharply criticizing the rejection of the compelling state interest
test, asserting that "(t)he compelling state interest test effectuates the First Amendment's command that
religious liberty is an independent liberty, that it occupies a preferred position, and that the Court will not
permit encroachments upon this liberty, whether direct or indirect, unless required by clear and compelling
government interest 'of the highest order.' "78 She said that strict scrutiny is appropriate for free exercise
challenges because "[t]he compelling interest test reflects the First Amendment's mandate of preserving
religious liberty to the fullest extent possible in a pluralistic society." 79

Justice O Connor also disagreed with the majority's description of prior cases and especially its leaving the
protection of minority religions to the political process. She said that, "First Amendment was enacted
precisely to protect the rights of those whose religious practice are not shared by the majority and may be
viewed with hostility." 80

Justice Blackmun wrote a dissenting opinion that was joined by Justices Brennan and Marshall. The
dissenting Justices agreed with Justice O Connor that the majority had mischaracterized precedents, such as
in describing Yoder as a "hybrid" case rather than as one under the free exercise clause. The dissent also
argued that strict scrutiny should be used in evaluating government laws burdening religion.81

Criticism of Smith was intense and widespread.82 Academics, Justices, and a bipartisan majority of Congress
noisily denounced the decision.83 Smith has the rather unusual distinction of being one case that is almost
universally despised (and this is not too strong a word) by both the liberals and conservatives. 84 Liberals
chasten the Court for its hostility to minority faiths which, in light of Smith's general applicability rule, will
allegedly suffer at the hands of the majority faith whether through outright hostility or neglect.
Conservatives bemoan the decision as an assault on religious belief leaving religion, more than ever, subject
to the caprice of an ever more secular nation that is increasingly hostile to religious belief as an oppressive
and archaic anachronism.85
The Smith doctrine is highly unsatisfactory in several respects and has been criticized as exhibiting a shallow
understanding of free exercise jurisprudence. 86 First, the First amendment was intended to protect minority
religions from the tyranny of the religious and political majority.87 Critics of Smith have worried about
religious minorities, who can suffer disproportionately from laws that enact majoritarian mores. 88 Smith, in
effect would allow discriminating in favor of mainstream religious groups against smaller, more peripheral
groups who lack legislative clout,89 contrary to the original theory of the First Amendment.90 Undeniably,
claims for judicial exemption emanate almost invariably from relatively politically powerless minority
religions and Smith virtually wiped out their judicial recourse for exemption. 91 Second, Smith leaves too
much leeway for pervasive welfare-state regulation to burden religion while satisfying neutrality. After all,
laws not aimed at religion can hinder observance just as effectively as those that target
religion.92 Government impairment of religious liberty would most often be of the inadvertent kind as in
Smith considering the political culture where direct and deliberate regulatory imposition of religious
orthodoxy is nearly inconceivable. If the Free Exercise Clause could not afford protection to inadvertent
interference, it would be left almost meaningless.93 Third, the Reynolds-Gobitis-Smith94 doctrine simply
defies common sense. The state should not be allowed to interfere with the most deeply held fundamental
religious convictions of an individual in order to pursue some trivial state economic or bureaucratic
objective. This is especially true when there are alternative approaches for the state to effectively pursue its
objective without serious inadvertent impact on religion.95

At bottom, the Court's ultimate concern in Smith appeared to be two-fold: (1) the difficulty in defining and
limiting the term "religion" in today's pluralistic society, and (2) the belief that courts have no business
determining the significance of an individual's religious beliefs. For the Smith Court, these two concerns
appear to lead to the conclusion that the Free Exercise Clause must protect everything or it must protect
virtually nothing. As a result, the Court perceives its only viable options are to leave free exercise protection
to the political process or to allow a "system in which each conscience is a law unto itself." 96 The Court's
characterization of its choices have been soundly rejected as false, viz:

If one accepts the Court's assumption that these are the only two viable options, then admittedly, the Court
has a stronger argument. But the Free Exercise Clause cannot be summarily dismissed as too difficult to
apply and this should not be applied at all. The Constitution does not give the judiciary the option of simply
refusing to interpret its provisions. The First Amendment dictates that free exercise of "religion" must be
protected. Accordingly, the Constitution compels the Court to struggle with the contours of what constitutes
"religion." There is no constitutional opt-out provision for constitutional words that are difficult to apply.

Nor does the Constitution give the Court the option of simply ignoring constitutional mandates. A large area
of middle ground exists between the Court's two opposing alternatives for free exercise jurisprudence.
Unfortunately, this middle ground requires the Court to tackle difficult issues such as defining religion and
possibly evaluating the significance of a religious belief against the importance of a specific law. The Court
describes the results of this middle ground where "federal judges will regularly balance against the
importance of general laws the significance of religious practice," and then dismisses it as a "parade of
horribles" that is too "horrible to contemplate."

It is not clear whom the Court feels would be most hurt by this "parade of horribles." Surely not religious
individuals; they would undoubtedly prefer their religious beliefs to be probed for sincerity and significance
rather than acquiesce to the Court's approach of simply refusing to grant any constitutional significance to
their beliefs at all. If the Court is concerned about requiring lawmakers at times constitutionally to exempt
religious individuals from statutory provisions, its concern is misplaced. It is the lawmakers who have sought
to prevent the Court from dismantling the Free Exercise Clause through such legislation as the [Religious
Freedom Restoration Act of 1993], and in any case, the Court should not be overly concerned about hurting
legislature's feelings by requiring their laws to conform to constitutional dictates. Perhaps the Court is
concerned about putting such burden on judges. If so, it would truly be odd to say that

requiring the judiciary to perform its appointed role as constitutional interpreters is a burden no judge
should be expected to fulfill.97

Parenthetically, Smith's characterization that the U.S. Court has "never held that an individual's religious
beliefs excuse him from compliance with an otherwise valid law prohibiting conduct that the state is free to
regulate" an assertion which Mr. Justice Carpio adopted unequivocally in his dissent has been sharply
criticized even implicitly by its supporters, as blatantly untrue. Scholars who supported Smith frequently did
not do so by opposing the arguments that the Court was wrong as a matter of original meaning [of the
religion clauses] or that the decision conflicted with precedent [i.e. the Smith decision made shocking use of
precedent] those points were often conceded.98

To justify its perversion of precedent, the Smith Court attempted to distinguish the exemption made in
Yoder, by asserting that these were premised on two constitutional rights combined the right of parents to
direct the education of their children and the right of free exercise of religion. Under the Court's opinion in
Smith, the right of free exercise of religion standing alone would not allow Amish parents to disregard the
compulsory school attendance law, and under the Court's opinion in Yoder, parents whose objection to the
law was not religious would also have to obey it. The fatal flaw in this argument, however, is that if two
constitutional claims will fail on its own, how would it prevail if combined?99 As for Sherbert, the Smith Court
attempted to limit its doctrine as applicable only to denials of unemployment compensation benefits where
the religiously-compelled conduct that leads to job loss is not a violation of criminal law. And yet, this is
precisely why the rejection of Sherbert was so damaging in its effect: the religious person was more likely to
be entitled to constitutional protection when forced to choose between religious conscience and going to jail
than when forced to choose between religious conscience and financial loss.100

Thus, the Smith decision elicited much negative public reaction especially from the religious community, and
commentaries insisted that the Court was allowing the Free Exercise Clause to disappear.101 So much was
the uproar that a majority in Congress was convinced to enact the Religious Freedom Restoration Act (RFRA)
of 1993.102 The RFRA was adopted to negate the Smith test and require strict scrutiny for free exercise
claims. Indeed, the findings section of the Act notes that Smith "virtually eliminated the requirement that
the government justify burdens on religious exercise imposed by laws neutral toward religion."103 The Act
declares that its purpose is to restore the compelling interest test as set forth in Sherbert v. Verner and
Wisconsin v. Yoder, and to guarantee its application in all cases where free exercise of religion is
substantially burdened; and to provide a claim of defense to a person whose religious exercise is
substantially burdened by government.104 The RFRA thus sought to overrule Smith and make strict scrutiny
the test for all free exercise clause claims.105

In the City of Boerne v. Flores, 106 the U.S. Supreme Court declared the RFRA unconstitutional, ruling that
Congress had exceeded its power under the Fourteenth Amendment in enacting the law. The Court ruled
that Congress is empowered to enact laws "to enforce the amendment," but Congress is not "enforcing"
when it creates new constitutional rights or expands the scope of rights.107

City of Boerne also drew public backlash as the U.S. Supreme Court was accused of lack of judicial respect
for the constitutional decision-making by a coordinate branch of government. In Smith, Justice Scalia wrote:

"Values that are protected against governmental interference through enshrinement in the Bill of Rights are
not thereby banished from the political process. Just as society believes in the negative protection accorded
to the press by the First Amendment is likely to enact laws that affirmatively foster the dissemination of the
printed word, so also a society that believes in the negative protection accorded to religious belief can be
expected to be solicitous of that value in its legislation as well."

By invalidating RFRA, the Court showed a marked disrespect of the solicitude of a nearly unanimous
Congress. Contrary to the Court's characterization of the RFRA as a kind of usurpation of the judicial power
to say what the Constitution means, the law offered no definition of Free Exercise, and on its face appeared
to be a procedural measure establishing a standard of proof and allocating the duty of meeting it. In effect,
the Court ruled that Congress had no power in the area of religion. And yet, Free Exercise exists in the First
Amendment as a negative on Congress. The power of Congress to act towards the states in matters of
religion arises from the Fourteenth Amendment.108

From the foregoing, it can be seen that Smith, while expressly recognizing the power of legislature to give
accommodations, is in effect contrary to the benevolent neutrality or accommodation approach. Moreover, if
we consider the history of the incorporation of the religion clauses in the U.S., the decision in Smith is
grossly inconsistent with the importance placed by the framers on religious faith. Smith is dangerous
precedent because it subordinates fundamental rights of religious belief and practice to all neutral, general
legislation. Sherbert recognized the need to protect religious exercise in light of the massive increase in the
size of government, the concerns within its reach, and the number of laws administered by it. However,
Smith abandons the protection of religious exercise at a time when the scope and reach of government has
never been greater. It has been pointed out that Smith creates the legal framework for persecution: through
general, neutral laws, legislatures are now able to force conformity on religious minorities whose practice
irritate or frighten an intolerant majority.109

The effect of Smith is to erase entirely the concept of mandatory accommodations, thereby emasculating the
Free Exercise Clause. Smith left religious freedom for many in the hands of the political process, exactly
where it would be if the religion clauses did not exist in the Bill of Rights. Like most protections found in the
Bill of Rights, the religion clauses of the First Amendment are most important to those who cannot prevail in
the political process. The Court in Smith ignores the fact that the protections found in the Bill of Rights were
deemed too important to leave to the political process. Because mainstream religions generally have been
successful in protecting their interests through the political process, it is the non-mainstream religions that
are adversely affected by Smith. In short, the U.S. Supreme Court has made it clear to such religions that
they should not look to the First Amendment for religious freedom.110

(3) Accommodation under the Religion Clauses

A free exercise claim could result to three kinds of accommodation: (a) those which are found to be
constitutionally compelled, i.e., required by the Free Exercise Clause; (b) those which are discretionary or
legislative, i.e., not required by the Free Exercise Clause but nonetheless permitted by the Establishment
Clause; and (c) those which the religion clauses prohibit.111

Mandatory accommodation results when the Court finds that accommodation is required by the Free
Exercise Clause, i.e, when the Court itself carves out an exemption. This accommodation occurs when all
three conditions of the compelling interest test are met, i.e, a statute or government action has burdened
claimant's free exercise of religion, and there is no doubt as to the sincerity of the religious belief; the state
has failed to demonstrate a particularly important or compelling governmental goal in preventing an
exemption; and that the state has failed to demonstrate that it used the least restrictive means. In these
cases, the Court finds that the injury to religious conscience is so great and the advancement of public
purposes is incomparable that only indifference or hostility could explain a refusal to make exemptions.
Thus, if the state's objective could be served as well or almost as well by granting an exemption to those
whose religious beliefs are burdened by the regulation, the Court must grant the exemption. The Yoder case
is an example where the Court held that the state must accommodate the religious beliefs of the Amish who
objected to enrolling their children in high school as required by law. The Sherbert case is another example
where the Court held that the state unemployment compensation plan must accommodate the religious
convictions of Sherbert.112

In permissive accommodation, the Court finds that the State may, but is not required to, accommodate
religious interests. The U.S. Walz case illustrates this situation where the U.S. Supreme Court upheld the
constitutionality of tax exemption given by New York to church properties, but did not rule that the state
was required to provide tax exemptions. The Court declared that "(t)he limits of permissible state
accommodation to religion are by no means co-extensive with the noninterference mandated by the Free
Exercise Clause."113 Other examples are Zorach v. Clauson,114 allowing released time in public schools and
Marsh v. Chambers,115 allowing payment of legislative chaplains from public funds. Parenthetically, the Court
in Smith has ruled that this is the only accommodation allowed by the Religion Clauses.

Finally, when the Court finds no basis for a mandatory accommodation, or it determines that the legislative
accommodation runs afoul of the establishment or the free exercise clause, it results to a prohibited
accommodation. In this case, the Court finds that establishment concerns prevail over potential
accommodation interests. To say that there are valid exemptions buttressed by the Free Exercise Clause
does not mean that all claims for free exercise exemptions are valid. 116 An example where accommodation
was prohibited is McCollum v. Board of Education,117 where the Court ruled against optional religious
instruction in the public school premises.118

Given that a free exercise claim could lead to three different results, the question now remains as to how
the Court should determine which action to take. In this regard, it is the strict scrutiny-compelling state
interest test which is most in line with the benevolent neutrality-accommodation approach.

Under the benevolent-neutrality theory, the principle underlying the First Amendment is that freedom to
carry out one's duties to a Supreme Being is an inalienable right, not one dependent on the grace of
legislature. Religious freedom is seen as a substantive right and not merely a privilege against
discriminatory legislation. With religion looked upon with benevolence and not hostility, benevolent
neutrality allows accommodation of religion under certain circumstances.

Considering that laws nowadays are rarely enacted specifically to disable religious belief or practice, free
exercise disputes arise commonly when a law that is religiously neutral and generally applicable on its face
is argued to prevent or burden what someone's religious faith requires, or alternatively, requires someone to
undertake an act that faith would preclude. In essence, then, free exercise arguments contemplate religious
exemptions from otherwise general laws.119

Strict scrutiny is appropriate for free exercise challenges because "[t]he compelling interest test reflects the
First Amendment's mandate of preserving religious liberty to the fullest extent possible in a pluralistic
society.120 Underlying the compelling state interest test is the notion that free exercise is a fundamental right
and that laws burdening it should be subject to strict scrutiny.121

In its application, the compelling state interest test follows a three-step process, summarized as follows:

If the plaintiff can show that a law or government practice inhibits the free exercise of his religious beliefs,
the burden shifts to the government to demonstrate that the law or practice is necessary to the
accomplishment of some important (or 'compelling' ) secular objective and that it is the least restrictive
means of achieving that objective. If the plaintiff meets this burden and the government does not, the
plaintiff is entitled to exemption from the law or practice at issue. In order to be protected, the claimant's
beliefs must be 'sincere', but they need not necessarily be consistent, coherent, clearly articulated, or
congruent with those of the claimant's religious denomination. 'Only beliefs rooted in religion are protected
by the Free Exercise Clause'; secular beliefs, however sincere and conscientious, do not suffice.122

In sum, the U.S. Court has invariably decided claims based on the religion clauses using either the
separationist approach, or the benevolent neutrality approach. The benevolent neutrality approach has also
further been split by the view that the First Amendment requires accommodation, or that it only allows
permissible legislative accommodations. The current prevailing view as pronounced in Smith, however, is
that that there are no required accommodation under the First Amendment, although it permits of legislative
accommodations.

3. Religion Clauses in the Philippine Context: Constitution, Jurisprudence and Practice

A. US Constitution and jurisprudence vis - à-vis Philippine Constitution

By juxtaposing the American Constitution and jurisprudence against that of the Philippines, it is immediately
clear that one cannot simply conclude that we have adopted lock, stock and barrel the religion clauses as
embodied in the First Amendment, and therefore, the U.S. Court's interpretation of the same. Unlike in the
U.S. where legislative exemptions of religion had to be upheld by the U.S. Supreme Court as constituting
permissive accommodations, similar exemptions for religion are mandatory accommodations under our own
constitutions. Thus, our 1935, 1973 and 1987 Constitutions contain provisions on tax exemption of church
property,123 salary of religious officers in government institutions, 124 and optional religious instruction.125 Our
own preamble also invokes the aid of a divine being.126 These constitutional provisions are wholly ours and
have no counterpart in the U.S. Constitution or its amendments. They all reveal without doubt that the
Filipino people, in adopting these constitutions, manifested their adherence to the benevolent neutrality
approach that requires accommodations in interpreting the religion clauses.127

The argument of Mr. Justice Carpio that the August 4, 2003 ponencia was erroneous insofar as it asserted
that the 1935 Constitution incorporates the Walz ruling as this case was decided subsequent to the 1935
Constitution is a misreading of the ponencia. What the ponencia pointed out was that even as early as 1935,
or more than three decades before the U.S. Court could validate the exemption in Walz as a form or
permissible accommodation, we have already incorporated the same in our Constitution, as a mandatory
accommodation.

There is no ambiguity with regard to the Philippine Constitution's departure from the U.S. Constitution,
insofar as religious accommodations are concerned. It is indubitable that benevolent neutrality-
accommodation, whether mandatory or permissive, is the spirit, intent and framework underlying the
Philippine Constitution.128 As stated in our Decision, dated August 4, 2003:
The history of the religion clauses in the 1987 Constitution shows that these clauses were largely adopted
from the First Amendment of the U.S. Constitution xxxx Philippine jurisprudence and commentaries on the
religious clauses also continued to borrow authorities from U.S. jurisprudence without articulating the stark
distinction between the two streams of U.S. jurisprudence [i.e., separation and benevolent neutrality]. One
might simply conclude that the Philippine Constitutions and jurisprudence also inherited the disarray of U.S.
religion clause jurisprudence and the two identifiable streams; thus, when a religion clause case comes
before the Court, a separationist approach or a benevolent neutrality approach might be adopted and each
will have U.S. authorities to support it. Or, one might conclude that as the history of the First Amendment as
narrated by the Court in Everson supports the separationist approach, Philippine jurisprudence should also
follow this approach in light of the Philippine religion clauses' history. As a result, in a case where the party
claims religious liberty in the face of a general law that inadvertently burdens his religious exercise, he faces
an almost insurmountable wall in convincing the Court that the wall of separation would not be breached if
the Court grants him an exemption. These conclusions, however, are not and were never warranted by the
1987, 1973 and 1935 Constitutions as shown by other provisions on religion in all three constitutions. It is a
cardinal rule in constitutional construction that the constitution must be interpreted as a whole and
apparently conflicting provisions should be reconciled and harmonized in a manner that will give to all of
them full force and effect. From this construction, it will be ascertained that the intent of the framers was to
adopt a benevolent neutrality approach in interpreting the religious clauses in the Philippine constitutions,
and the enforcement of this intent is the goal of construing the constitution.129 [citations omitted]

We therefore reject Mr. Justice Carpio's total adherence to the U.S. Court's interpretation of the religion
clauses to effectively deny accommodations on the sole basis that the law in question is neutral and of
general application. For even if it were true that "an unbroken line of U.S. Supreme Court decisions" has
never held that "an individual's religious beliefs [do not] excuse him from compliance with an otherwise valid
law prohibiting conduct that the State is free to regulate," our own Constitutions have made significant
changes to accommodate and exempt religion. Philippine jurisprudence shows that the Court has allowed
exemptions from a law of general application, in effect, interpreting our religion clauses to cover both
mandatory and permissive accommodations.130

To illustrate, in American Bible Society v. City of Manila, 131 the Court granted to plaintiff exemption from a
law of general application based on the Free Exercise Clause. In this case, plaintiff was required by an
ordinance to secure a mayor's permit and a municipal license as ordinarily required of those engaged in the
business of general merchandise under the city's ordinances. Plaintiff argued that this amounted to
"religious censorship and restrained the free exercise and enjoyment of religious profession, to wit: the
distribution and sale of bibles and other religious literature to the people of the Philippines." Although the
Court categorically held that the questioned ordinances were not applicable to plaintiff as it was not engaged
in the business or occupation of selling said "merchandise" for profit, it also ruled that applying the
ordinance to plaintiff and requiring it to secure a license and pay a license fee or tax would impair its free
exercise of religious profession and worship and its right of dissemination of religious beliefs "as the power
to tax the exercise of a privilege is the power to control or suppress its enjoyment." The decision states in
part, viz:

The constitutional guaranty of the free exercise and enjoyment of religious profession and worship carries
with it the right to disseminate religious information. Any restraint of such right can only be justified like
other restraints of freedom of expression on the grounds that there is a clear and present danger of any
substantive evil which the State has the right to prevent. (citations omitted, emphasis supplied)

Another case involving mandatory accommodation is Ebralinag v. The Division Superintendent of


Schools.132 The case involved several Jehovah's Witnesses who were expelled from school for refusing to
salute the flag, sing the national anthem and recite the patriotic pledge, in violation of the Administrative
Code of 1987. In resolving the religious freedom issue, a unanimous Court overturned an earlier ruling
denying such exemption,133 using the "grave and imminent danger" test, viz:

The sole justification for a prior restraint or limitation on the exercise of religious freedom (according to the
late Chief Justice Claudio Teehankee in his dissenting opinion in German v. Barangan, 135 SCRA 514, 517)
is the existence of a grave and present danger of a character both grave and imminent, of a serious evil to
public safety, public morals, public health or any other legitimate public interest, that the State has a right
(and duty) to prevent. Absent such a threat to public safety, the expulsion of the petitioners from the
schools is not justified.134 (emphases supplied)
In these two cases, the Court itself carved out an exemption from a law of general application, on the
strength directly of the Free Exercise Clause.

We also have jurisprudence that supports permissive accommodation. The case of Victoriano v. Elizalde
Rope Workers Union135 is an example of the application of Mr. Justice Carpio's theory of permissive
accommodation, where religious exemption is granted by a legislative act. In Victoriano, the constitutionality
of Republic Act No. 3350 was questioned. The said R.A. exempt employees from the application and
coverage of a closed shop agreement mandated in another law based on religious objections. A unanimous
Court upheld the constitutionality of the law, holding that "government is not precluded from pursuing valid
objectives secular in character even if the incidental result would be favorable to a religion or sect."
Interestingly, the secular purpose of the challenged law which the Court upheld was the advancement of
"the constitutional right to the free exercise of religion."136

Having established that benevolent neutrality-accommodation is the framework by which free exercise cases
must be decided, the next question then turned to the test that should be used in ascertaining the limits of
the exercise of religious freedom. In our Decision dated August 4, 2003, we reviewed our jurisprudence, and
ruled that in cases involving purely conduct based on religious belief, as in the case at bar, the compelling
state interest test, is proper, viz:

Philippine jurisprudence articulates several tests to determine these limits. Beginning with the first case on
the Free Exercise Clause, American Bible Society, the Court mentioned the "clear and present danger" test
but did not employ it. Nevertheless, this test continued to be cited in subsequent cases on religious liberty.
The Gerona case then pronounced that the test of permissibility of religious freedom is whether it violates
the established institutions of society and law. The Victoriano case mentioned the "immediate and grave
danger" test as well as the doctrine that a law of general applicability may burden religious exercise
provided the law is the least restrictive means to accomplish the goal of the law. The case also used, albeit
inappropriately, the "compelling state interest" test. After Victoriano, German went back to the Gerona rule.
Ebralinag then employed the "grave and immediate danger" test and overruled the Gerona test. The fairly
recent case of Iglesia ni Cristo went back to the "clear and present danger" test in the maiden case of
American Bible Society. Not surprisingly, all the cases which employed the "clear and present danger" or
"grave and immediate danger" test involved, in one form or another, religious speech as this test is often
used in cases on freedom of expression. On the other hand, the Gerona and German cases set the rule that
religious freedom will not prevail over established institutions of society and law. Gerona, however, which
was the authority cited by German has been overruled by Ebralinag which employed the "grave and
immediate danger" test. Victoriano was the only case that employed the "compelling state interest" test, but
as explained previously, the use of the test was inappropriate to the facts of the case.

The case at bar does not involve speech as in American Bible Society, Ebralinag and Iglesia ni Cristo where
the "clear and present danger" and "grave and immediate danger" tests were appropriate as speech has
easily discernible or immediate effects. The Gerona and German doctrine, aside from having been overruled,
is not congruent with the benevolent neutrality approach, thus not appropriate in this jurisdiction. Similar to
Victoriano, the present case involves purely conduct arising from religious belief. The "compelling state
interest" test is proper where conduct is involved for the whole gamut of human conduct has different
effects on the state's interests: some effects may be immediate and short-term while others delayed and
far-reaching. A test that would protect the interests of the state in preventing a substantive evil, whether
immediate or delayed, is therefore necessary. However, not any interest of the state would suffice to prevail
over the right to religious freedom as this is a fundamental right that enjoys a preferred position in the
hierarchy of rights - "the most inalienable and sacred of all human rights", in the words of Jefferson. This
right is sacred for an invocation of the Free Exercise Clause is an appeal to a higher sovereignty. The entire
constitutional order of limited government is premised upon an acknowledgment of such higher sovereignty,
thus the Filipinos implore the "aid of Almighty God in order to build a just and humane society and establish
a government." As held in Sherbert, only the gravest abuses, endangering paramount interests can limit this
fundamental right. A mere balancing of interests which balances a right with just a colorable state interest is
therefore not appropriate. Instead, only a compelling interest of the state can prevail over the fundamental
right to religious liberty. The test requires the state to carry a heavy burden, a compelling one, for to do
otherwise would allow the state to batter religion, especially the less powerful ones until they are destroyed.
In determining which shall prevail between the state's interest and religious liberty, reasonableness shall be
the guide. The "compelling state interest" serves the purpose of revering religious liberty while at the same
time affording protection to the paramount interests of the state. This was the test used in Sherbert which
involved conduct, i.e. refusal to work on Saturdays. In the end, the "compelling state interest" test, by
upholding the paramount interests of the state, seeks to protect the very state, without which, religious
liberty will not be preserved.137 (citations omitted)

At this point, we take note of Mr. Justice Carpio's dissent, which, while loosely disputing the applicability of
the benevolent neutrality framework and compelling state interest test, states that "[i]t is true that a test
needs to be applied by the Court in determining the validity of a free exercise claim of exemption as made
here by Escritor." This assertion is inconsistent with the position negating the benevolent neutrality or
accommodation approach. If it were true, indeed, that the religion clauses do not require accommodations
based on the free exercise of religion, then there would be no need for a test to determine the validity of a
free exercise claim, as any and all claims for religious exemptions from a law of general application would
fail.

Mr. Justice Carpio also asserts that "[m]aking a distinction between permissive accommodation and
mandatory accommodation is more critically important in analyzing free exercise exemption claims because
it forces the Court to confront how far it can validly set the limits of religious liberty under the Free Exercise
Clause, rather than presenting the separation theory and accommodation theory as opposite concepts, and
then rejecting relevant and instructive American jurisprudence (such as the Smith case) just because it does
not espouse the theory selected." He then asserts that the Smith doctrine cannot be dismissed because it
does not really espouse the strict neutrality approach, but more of permissive accommodation.

Mr. Justice Carpio's assertion misses the point. Precisely because the doctrine in Smith is that only
legislative accommodations are allowed under the Free Exercise Clause, it cannot be used in determining a
claim of religion exemption directly anchored on the Free Exercise Clause. Thus, even assuming that the
Smith doctrine actually espouses the theory of accommodation or benevolent neutrality, the accommodation
is limited to the permissive, or legislative exemptions. It, therefore, cannot be used as a test in determining
the claims of religious exemptions directly under the Free Exercise Clause because Smith does not recognize
such exemption. Moreover, Mr. Justice Carpio's advocacy of the Smith doctrine would effectively render the
Free Exercise protection a fundamental right under our Constitution nugatory because he would deny its
status as an independent source of right.

b. The Compelling State Interest Test

As previously stated, the compelling state interest test involves a three-step process. We explained this
process in detail, by showing the questions which must be answered in each step, viz:

'First, "[H]as the statute or government action created a burden on the free exercise of religion?" The courts
often look into the sincerity of the religious belief, but without inquiring into the truth of the belief because
the Free Exercise Clause prohibits inquiring about its truth as held in Ballard and Cantwell. The sincerity of
the claimant's belief is ascertained to avoid the mere claim of religious beliefs to escape a mandatory
regulation. xxx

xxx xxx xxx

Second, the court asks: "[I]s there a sufficiently compelling state interest to justify this infringement of
religious liberty?" In this step, the government has to establish that its purposes are legitimate for the state
and that they are compelling. Government must do more than assert the objectives at risk if exemption is
given; it must precisely show how and to what extent those objectives will be undermined if exemptions are
granted. xxx

xxx xxx xxx

Third, the court asks: "[H]as the state in achieving its legitimate purposes used the least intrusive means
possible so that the free exercise is not infringed any more than necessary to achieve the legitimate goal of
the state?" The analysis requires the state to show that the means in which it is achieving its legitimate
state objective is the least intrusive means, i.e., it has chosen a way to achieve its legitimate state end that
imposes as little as possible on religious liberties xxx.138 [citations omitted]
Again, the application of the compelling state interest test could result to three situations of
accommodation: First, mandatory accommodation would result if the Court finds that accommodation is
required by the Free Exercise Clause. Second, if the Court finds that the State may, but is not required to,
accommodate religious interests, permissive accommodation results. Finally, if the Court finds that that
establishment concerns prevail over potential accommodation interests, then it must rule that the
accommodation is prohibited.

One of the central arguments in Mr. Justice Carpio's dissent is that only permissive accommodation can
carve out an exemption from a law of general application. He posits the view that the law should prevail in
the absence of a legislative exemption, and the Court cannot make the accommodation or exemption.

Mr. Justice Carpio's position is clearly not supported by Philippine jurisprudence. The cases of American Bible
Society, Ebralinag, and Victoriano demonstrate that our application of the doctrine of benevolent neutrality-
accommodation covers not only the grant of permissive, or legislative accommodations, but also mandatory
accommodations. Thus, an exemption from a law of general application is possible, even if anchored directly
on an invocation of the Free Exercise Clause alone, rather than a legislative exemption.

Moreover, it should be noted that while there is no Philippine case as yet wherein the Court granted an
accommodation/exemption to a religious act from the application of general penal laws, permissive
accommodation based on religious freedom has been granted with respect to one of the crimes penalized
under the Revised Penal Code, that of bigamy.

In the U.S. case of Reynolds v. United States, 139 the U.S. Court expressly denied to Mormons an exemption
from a general federal law criminalizing polygamy, even if it was proven that the practice constituted a
religious duty under their faith.140 In contradistinction, Philippine law accommodates the same practice
among Moslems, through a legislative act. For while the act of marrying more than one still constitutes
bigamy under the Revised Penal Code, Article 180 of P.D. No. 1083, otherwise known as the Code of Muslim
Personal Laws of the Philippines, provides that the penal laws relative to the crime of bigamy "shall not
apply to a person married under Muslim law." Thus, by legislative action, accommodation is granted of a
Muslim practice which would otherwise violate a valid and general criminal law. Mr. Justice Carpio
recognized this accommodation when, in his dissent in our Decision dated August 4, 2003 and citing Sulu
Islamic Association of Masjid Lambayong v. Malik,141 he stated that a Muslim Judge "is not criminally liable
for bigamy because Shari a law allows a Muslim to have more than one wife."

From the foregoing, the weakness of Mr. Justice Carpio's "permissive-accommodation only" advocacy in this
jurisdiction becomes manifest. Having anchored his argument on the Smith doctrine that "the guaranty of
religious liberty as embodied in the Free Exercise Clause does not require the grant of exemptions from
generally applicable laws to individuals whose religious practice conflict with those laws," his theory is
infirmed by the showing that the benevolent neutrality approach which allows for both mandatory and
permissive accommodations was unequivocally adopted by our framers in the Philippine Constitution, our
legislature, and our jurisprudence.

Parenthetically, it should be pointed out that a "permissive accommodation-only" stance is the antithesis to
the notion that religion clauses, like the other fundamental liberties found in the Bill or Rights, is a preferred
right and an independent source of right.

What Mr. Justice Carpio is left with is the argument, based on Smith, that the test in Sherbert is not
applicable when the law in question is a generally applicable criminal law. Stated differently, even if Mr.
Justice Carpio conceded that there is no question that in the Philippine context, accommodations are made,
the question remains as to how far the exemptions will be made and who would make these exemptions.

On this point, two things must be clarified: first, in relation to criminal statutes, only the question of
mandatory accommodation is uncertain, for Philippine law and jurisprudence have, in fact, allowed
legislative accommodation. Second, the power of the Courts to grant exemptions in general (i.e., finding
that the Free Exercise Clause required the accommodation, or mandatory accommodations) has already
been decided, not just once, but twice by the Court. Thus, the crux of the matter is whether this Court can
make exemptions as in Ebralinag and the American Bible Society, in cases involving criminal laws of general
application.
We hold that the Constitution itself mandates the Court to do so for the following reasons.

First, as previously discussed, while the U.S. religion clauses are the precursors to the Philippine religion
clauses, the benevolent neutrality-accommodation approach in Philippine jurisdiction is more pronounced
and given leeway than in the U.S.

Second, the whole purpose of the accommodation theory, including the notion of mandatory
accommodations, was to address the "inadvertent burdensome effect" that an otherwise facially neutral law
would have on religious exercise. Just because the law is criminal in nature, therefore, should not bring it
out of the ambit of the Free Exercise Clause. As stated by Justice O Connor in her concurring opinion in
Smith, "[t]here is nothing talismanic about neutral laws of general applicability or general criminal
prohibitions, for laws neutral towards religion can coerce a person to violate his religious conscience or
intrude upon his religious duties just as effectively as laws aimed at religion."142

Third, there is wisdom in accommodation made by the Court as this is the recourse of minority religions who
are likewise protected by the Free Exercise Clause. Mandatory accommodations are particularly necessary to
protect adherents of minority religions from the inevitable effects of majoritarianism, which include
ignorance and indifference and overt hostility to the minority. As stated in our Decision, dated August 4,
2003:

....In a democratic republic, laws are inevitably based on the presuppositions of the majority, thus not
infrequently, they come into conflict with the religious scruples of those holding different world views, even
in the absence of a deliberate intent to interfere with religious practice. At times, this effect is unavoidable
as a practical matter because some laws are so necessary to the common good that exceptions are
intolerable. But in other instances, the injury to religious conscience is so great and the advancement of
public purposes so small or incomparable that only indifference or hostility could explain a refusal to make
exemptions. Because of plural traditions, legislators and executive officials are frequently willing to make
such exemptions when the need is brought to their attention, but this may not always be the case when the
religious practice is either unknown at the time of enactment or is for some reason unpopular. In these
cases, a constitutional interpretation that allows accommodations prevents needless injury to the religious
consciences of those who can have an influence in the legislature; while a constitutional interpretation that
requires accommodations extends this treatment to religious faiths that are less able to protect themselves
in the political arena.

Fourth, exemption from penal laws on account of religion is not entirely an alien concept, nor will it be
applied for the first time, as an exemption of such nature, albeit by legislative act, has already been granted
to Moslem polygamy and the criminal law of bigamy.

Finally, we must consider the language of the Religion Clauses vis - à-vis the other fundamental rights in
the Bill of Rights. It has been noted that unlike other fundamental rights like the right to life, liberty or
property, the Religion Clauses are stated in absolute terms, unqualified by the requirement of "due process,"
"unreasonableness," or "lawful order." Only the right to free speech is comparable in its absolute grant.
Given the unequivocal and unqualified grant couched in the language, the Court cannot simply dismiss a
claim of exemption based on the Free Exercise Clause, solely on the premise that the law in question is a
general criminal law.143 If the burden is great and the sincerity of the religious belief is not in question,
adherence to the benevolent neutrality-accommodation approach require that the Court make an individual
determination and not dismiss the claim outright.

At this point, we must emphasize that the adoption of the benevolent neutrality-accommodation approach
does not mean that the Court ought to grant exemptions every time a free exercise claim comes before it.
This is an erroneous reading of the framework which the dissent of Mr. Justice Carpio seems to entertain.
Although benevolent neutrality is the lens with which the Court ought to view religion clause cases, the
interest of the state should also be afforded utmost protection. This is precisely the purpose of the test to
draw the line between mandatory, permissible and forbidden religious exercise. Thus, under the framework,
the Court cannot simply dismiss a claim under the Free Exercise Clause because the conduct in question
offends a law or the orthodox view, as proposed by Mr. Justice Carpio, for this precisely is the protection
afforded by the religion clauses of the Constitution.144 As stated in the Decision:
xxx While the Court cannot adopt a doctrinal formulation that can eliminate the difficult questions of
judgment in determining the degree of burden on religious practice or importance of the state interest or the
sufficiency of the means adopted by the state to pursue its interest, the Court can set a doctrine on the ideal
towards which religious clause jurisprudence should be directed. We here lay down the doctrine that in
Philippine jurisdiction, we adopt the benevolent neutrality approach not only because of its merits as
discussed above, but more importantly, because our constitutional history and interpretation indubitably
show that benevolent neutrality is the launching pad from which the Court should take off in interpreting
religion clause cases. The ideal towards which this approach is directed is the protection of religious liberty
"not only for a minority, however small - not only for a majority, however large but for each of us" to the
greatest extent possible within flexible constitutional limits.145

II. THE CURRENT PROCEEDINGS

We now resume from where we ended in our August 4, 2003 Decision. As mentioned, what remained to be
resolved, upon which remand was necessary, pertained to the final task of subjecting this case to the careful
application of the compelling state interest test, i.e., determining whether respondent is entitled to
exemption, an issue which is essentially factual or evidentiary in nature.

After the termination of further proceedings with the OCA, and with the transmittal of the Hearing Officer's
report,146 along with the evidence submitted by the OSG, this case is once again with us, to resolve the
penultimate question of whether respondent should be found guilty of the administrative charge of
"disgraceful and immoral conduct." It is at this point then that we examine the report and documents
submitted by the hearing officer of this case, and apply the three-step process of the compelling state
interest test based on the evidence presented by the parties, especially the government.

On the sincerity of religious belief, the Solicitor General categorically concedes that the sincerity and
centrality of respondent's claimed religious belief and practice are beyond serious doubt.147 Thus, having
previously established the preliminary conditions required by the compelling state interest test, i.e., that a
law or government practice inhibits the free exercise of respondent's religious beliefs, and there being no
doubt as to the sincerity and centrality of her faith to claim the exemption based on the free exercise clause,
the burden shifted to the government to demonstrate that the law or practice justifies a compelling secular
objective and that it is the least restrictive means of achieving that objective.

A look at the evidence that the OSG has presented fails to demonstrate "the gravest abuses, endangering
paramount interests" which could limit or override respondent's fundamental right to religious freedom.
Neither did the government exert any effort to show that the means it seeks to achieve its legitimate state
objective is the least intrusive means.

The OSG merely offered the following as exhibits and their purposes:

1. Exhibit "A-OSG" and submarking - The September 30, 2003 Letter to the OSG of Bro. Raymond B. Leach,
Legal Representative of the Watch Tower Bible and Tract Society of the Philippines, Inc.

Purpose: To show that the OSG exerted efforts to examine the sincerity and centrality of respondent's
claimed religious belief and practice.

2. Exhibit "B-OSG" and submarking - The duly notarized certification dated September 30, 2003 issued and
signed by Bro. Leach.

PURPOSES: (1) To substantiate the sincerity and centrality of respondent's claimed religious belief and
practice; and (2) to prove that the Declaration of Pledging Faithfulness, being a purely internal arrangement
within the congregation of the Jehovah's Witnesses, cannot be a source of any legal protection for
respondent.

In its Memorandum-In-Intervention, the OSG contends that the State has a compelling interest to override
respondent's claimed religious belief and practice, in order to protect marriage and the family as basic social
institutions. The Solicitor General, quoting the Constitution 148 and the Family Code,149argues that marriage
and the family are so crucial to the stability and peace of the nation that the conjugal arrangement
embraced in the Declaration of Pledging Faithfulness should not be recognized or given effect, as "it is
utterly destructive of the avowed institutions of marriage and the family for it reduces to a mockery these
legally exalted and socially significant institutions which in their purity demand respect and dignity."150

Parenthetically, the dissenting opinion of Mr. Justice Carpio echoes the Solicitor General in so far as he
asserts that the State has a compelling interest in the preservation of marriage and the family as basic
social institutions, which is ultimately the public policy underlying the criminal sanctions against concubinage
and bigamy. He also argues that in dismissing the administrative complaint against respondent, "the
majority opinion effectively condones and accords a semblance of legitimacy to her patently unlawful
cohabitation..." and "facilitates the circumvention of the Revised Penal Code." According to Mr. Justice
Carpio, by choosing to turn a blind eye to respondent's criminal conduct, the majority is in fact recognizing a
practice, custom or agreement that subverts marriage. He argues in a similar fashion as regards the state's
interest in the sound administration of justice.

There has never been any question that the state has an interest in protecting the institutions of marriage
and the family, or even in the sound administration of justice. Indeed, the provisions by which respondent's
relationship is said to have impinged, e.g., Book V, Title I, Chapter VI, Sec. 46(b)(5) of the Revised
Administrative Code, Articles 334 and 349 of the Revised Penal Code, and even the provisions on marriage
and family in the Civil Code and Family Code, all clearly demonstrate the State's need to protect these
secular interests.

Be that as it may, the free exercise of religion is specifically articulated as one of the fundamental rights in
our Constitution. It is a fundamental right that enjoys a preferred position in the hierarchy of rights - "the
most inalienable and sacred of human rights," in the words of Jefferson. Hence, it is not enough to contend
that the state's interest is important, because our Constitution itself holds the right to religious freedom
sacred. The State must articulate in specific terms the state interest involved in preventing the exemption,
which must be compelling, for only the gravest abuses, endangering paramount interests can limit the
fundamental right to religious freedom. To rule otherwise would be to emasculate the Free Exercise Clause
as a source of right by itself.

Thus, it is not the State's broad interest in "protecting the institutions of marriage and the family," or even
"in the sound administration of justice" that must be weighed against respondent's claim, but the State's
narrow interest in refusing to make an exception for the cohabitation which respondent's faith finds moral.
In other words, the government must do more than assert the objectives at risk if exemption is given; it
must precisely show how and to what extent those objectives will be undermined if exemptions are
granted.151 This, the Solicitor General failed to do.

To paraphrase Justice Blackmun's application of the compelling interest test, the State's interest in enforcing
its prohibition, in order to be sufficiently compelling to outweigh a free exercise claim, cannot be merely
abstract or symbolic. The State cannot plausibly assert that unbending application of a criminal prohibition is
essential to fulfill any compelling interest, if it does not, in fact, attempt to enforce that prohibition. In the
case at bar, the State has not evinced any concrete interest in enforcing the concubinage or bigamy charges
against respondent or her partner. The State has never sought to prosecute respondent nor her partner. The
State's asserted interest thus amounts only to the symbolic preservation of an unenforced prohibition.
Incidentally, as echoes of the words of Messrs. J. Bellosillo and Vitug, in their concurring opinions in our
Decision, dated August 4, 2003, to deny the exemption would effectively break up "an otherwise ideal union
of two individuals who have managed to stay together as husband and wife [approximately twenty-five
years]" and have the effect of defeating the very substance of marriage and the family.

The Solicitor General also argued against respondent's religious freedom on the basis of morality, i.e., that
"the conjugal arrangement of respondent and her live-in partner should not be condoned because adulterous
relationships are constantly frowned upon by society";152 and "that State laws on marriage, which are moral
in nature, take clear precedence over the religious beliefs and practices of any church, religious sect or
denomination on marriage. Verily, religious beliefs and practices should not be permitted to override laws
relating to public policy such as those of marriage."153

The above arguments are mere reiterations of the arguments raised by Mme. Justice Ynares-Santiago in her
dissenting opinion to our Decision dated August 4, 2003, which she offers again in toto. These arguments
have already been addressed in our decision dated August 4, 2003.154 In said Decision, we noted that Mme.
Justice Ynares-Santiago's dissenting opinion dwelt more on the standards of morality, without categorically
holding that religious freedom is not in issue.155 We, therefore, went into a discussion on morality, in order
to show that:

(a) The public morality expressed in the law is necessarily secular for in our constitutional order, the religion
clauses prohibit the state from establishing a religion, including the morality it sanctions. 156 Thus, when the
law speaks of "immorality" in the Civil Service Law or "immoral" in the Code of Professional Responsibility
for lawyers,157 or "public morals" in the Revised Penal Code,158 or "morals" in the New Civil Code,159 or "moral
character" in the Constitution,160 the distinction between public and secular morality on the one hand, and
religious morality, on the other, should be kept in mind;161

(b) Although the morality contemplated by laws is secular, benevolent neutrality could allow for
accommodation of morality based on religion, provided it does not offend compelling state interests;162

(c) The jurisdiction of the Court extends only to public and secular morality. Whatever pronouncement the
Court makes in the case at bar should be understood only in this realm where it has authority.163

(d) Having distinguished between public and secular morality and religious morality, the more difficult task
is determining which immoral acts under this public and secular morality fall under the phrase "disgraceful
and immoral conduct" for which a government employee may be held administratively liable. 164 Only one
conduct is in question before this Court, i.e., the conjugal arrangement of a government employee whose
partner is legally married to another which Philippine law and jurisprudence consider both immoral and
illegal.165

(e) While there is no dispute that under settled jurisprudence, respondent's conduct constitutes "disgraceful
and immoral conduct," the case at bar involves the defense of religious freedom, therefore none of the cases
cited by Mme. Justice Ynares-Santiago apply.166 There is no jurisprudence in Philippine jurisdiction holding
that the defense of religious freedom of a member of the Jehovah's Witnesses under the same
circumstances as respondent will not prevail over the laws on adultery, concubinage or some other law. We
cannot summarily conclude therefore

that her conduct is likewise so "odious" and "barbaric" as to be immoral and punishable by law.167

Again, we note the arguments raised by Mr. Justice Carpio with respect to charging respondent with conduct
prejudicial to the best interest of the service, and we reiterate that the dissent offends due process as
respondent was not given an opportunity to defend herself against the charge of "conduct prejudicial to the
best interest of the service." Indeed, there is no evidence of the alleged prejudice to the best interest of the
service.168

Mr. Justice Carpio's slippery slope argument, on the other hand, is non-sequitur. If the Court grants
respondent exemption from the laws which respondent Escritor has been charged to have violated, the
exemption would not apply to Catholics who have secured church annulment of their marriage even without
a final annulment from a civil court. First, unlike Jehovah's Witnesses, the Catholic faith considers
cohabitation without marriage as immoral. Second, but more important, the Jehovah's Witnesses have
standards and procedures which must be followed before cohabitation without marriage is given the blessing
of the congregation. This includes an investigative process whereby the elders of the congregation verify the
circumstances of the declarants. Also, the Declaration is not a blanket authority to cohabit without marriage
because once all legal impediments for the couple are lifted, the validity of the Declaration ceases, and the
congregation requires that the couple legalize their union.

At bottom, the slippery slope argument of Mr. Justice Carpio is speculative. Nevertheless, insofar as he
raises the issue of equality among religions, we look to the words of the Religion Clauses, which clearly
single out religion for both a benefit and a burden: "No law shall be made respecting an establishment of
religion, or prohibiting the free exercise thereof' " On its face, the language grants a unique advantage to
religious conduct, protecting it from governmental imposition; and imposes a unique disadvantage,
preventing the government from supporting it. To understand this as a provision which puts religion on an
equal footing with other bases for action seems to be a curious reading. There are no "free exercise" of
"establishment" provisions for science, sports, philosophy, or family relations. The language itself thus
seems to answer whether we have a paradigm of equality or liberty; the language of the Clause is clearly in
the form of a grant of liberty.169
In this case, the government's conduct may appear innocent and nondiscriminatory but in effect, it is
oppressive to the minority. In the interpretation of a document, such as the Bill of Rights, designed to
protect the minority from the majority, the question of which perspective is appropriate would seem easy to
answer. Moreover, the text, history, structure and values implicated in the interpretation of the clauses, all
point toward this perspective. Thus, substantive equality a reading of the religion clauses which leaves both
politically dominant and the politically weak religious groups equal in their inability to use the government
(law) to assist their own religion or burden others'makes the most sense in the interpretation of the Bill of
Rights, a document designed to protect minorities and individuals from mobocracy in a democracy (the
majority or a coalition of minorities).170

As previously discussed, our Constitution adheres to the benevolent neutrality approach that gives room for
accommodation of religious exercises as required by the Free Exercise Clause. 171 Thus, in arguing that
respondent should be held administratively liable as the arrangement she had was "illegal per se because,
by universally recognized standards, it is inherently or by its very nature bad, improper, immoral and
contrary to good conscience,"172 the Solicitor General failed to appreciate that benevolent neutrality could
allow for accommodation of morality based on religion, provided it does not offend compelling state
interests.173

Finally, even assuming that the OSG has proved a compelling state interest, it has to further demonstrate
that the state has used the least intrusive means possible so that the free exercise is not infringed any more
than necessary to achieve the legitimate goal of the state, i.e., it has chosen a way to achieve its legitimate
state end that imposes as little as possible on religious liberties. 174 Again, the Solicitor General utterly failed
to prove this element of the test. Other than the two documents offered as cited above which established
the sincerity of respondent's religious belief and the fact that the agreement was an internal arrangement
within respondent's congregation, no iota of evidence was offered. In fact, the records are bereft of even a
feeble attempt to procure any such evidence to show that the means the state adopted in pursuing this
compelling interest is the least restrictive to respondent's religious freedom.

Thus, we find that in this particular case and under these distinct circumstances, respondent Escritor's
conjugal arrangement cannot be penalized as she has made out a case for exemption from the law based on
her fundamental right to freedom of religion. The Court recognizes that state interests must be upheld in
order that freedoms - including religious freedom - may be enjoyed. In the area of religious exercise as a
preferred freedom, however, man stands accountable to an authority higher than the state, and so the state
interest sought to be upheld must be so compelling that its violation will erode the very fabric of the state
that will also protect the freedom. In the absence of a showing that such state interest exists, man must be
allowed to subscribe to the Infinite.

IN VIEW WHEREOF, the instant administrative complaint is dismissed.

SO ORDERED.

EN BANC

G. R. No. 153888 - July 9, 2003

ISLAMIC DA'WAH COUNCIL OF THE PHILIPPINES, INC., herein represented by PROF.


ABDULRAFIH H. SAYEDY, Petitioner, vs. OFFICE OF THE EXECUTIVE SECRETARY of the Office
of the President of the Philippines, herein represented by HON. ALBERTO G. ROMULO,
Executive Secretary, and the OFFICE ON MUSLIM AFFAIRS, herein represented by its
Executive Director, HABIB MUJAHAB HASHIM, Respondents.

CORONA, J.:

Before us is a petition for prohibition filed by petitioner Islamic Da'wah Council of the Philippines, Inc.
(IDCP) praying for the declaration of nullity of Executive Order (EO) 46, s. 2001 and the prohibition of
herein respondents Office of the Executive Secretary and Office of Muslim Affairs (OMA) from
implementing the subject EO.

Petitioner IDCP, a corporation that operates under Department of Social Welfare and Development
License No. SB-01-085, is a non-governmental organization that extends voluntary services to the
Filipino people, especially to Muslim communities. It claims to be a federation of national Islamic
organizations and an active member of international organizations such as the Regional Islamic
Da'wah Council of Southeast Asia and the Pacific (RISEAP)1 and The World Assembly of Muslim Youth.
The RISEAP accredited petitioner to issue halal2 certifications in the Philippines. Thus, among the
functions petitioner carries out is to conduct seminars, orient manufacturers on halal food and issue
halal certifications to qualified products and manufacturers.

Petitioner alleges that, on account of the actual need to certify food products as halal and also due to
halal food producers' request, petitioner formulated in 1995 internal rules and procedures based on
the Qur'an3 and the Sunnah4 for the analysis of food, inspection thereof and issuance of halal
certifications. In that same year, petitioner began to issue, for a fee, certifications to qualified
products and food manufacturers. Petitioner even adopted for use on its halal certificates a distinct
sign or logo registered in the Philippine Patent Office under Patent No. 4-2000-03664.

On October 26, 2001, respondent Office of the Executive Secretary issued EO 46 5 creating the
Philippine Halal Certification Scheme and designating respondent OMA to oversee its implementation.
Under the EO, respondent OMA has the exclusive authority to issue halal certificates and perform
other related regulatory activities.

On May 8, 2002, a news article entitled "OMA Warns NGOs Issuing Illegal 'Halal' Certification" was
published in the Manila Bulletin, a newspaper of general circulation. In said article, OMA warned
Muslim consumers to buy only products with its official halal certification since those without said
certification had not been subjected to careful analysis and therefore could contain pork or its
derivatives. Respondent OMA also sent letters to food manufacturers asking them to secure the halal
certification only from OMA lest they violate EO 46 and RA 4109.6 As a result, petitioner lost revenues
after food manufacturers stopped securing certifications from it.

Hence, this petition for prohibition.

Petitioner contends that the subject EO violates the constitutional provision on the separation of
Church and State.7 It is unconstitutional for the government to formulate policies and guidelines on
the halal certification scheme because said scheme is a function only religious organizations, entity or
scholars can lawfully and validly perform for the Muslims. According to petitioner, a food product
becomes halal only after the performance of Islamic religious ritual and prayer. Thus, only practicing
Muslims are qualified to slaughter animals for food. A government agency like herein respondent OMA
cannot therefore perform a religious function like certifying qualified food products as halal.

Petitioner also maintains that the respondents violated Section 10, Article III of the 1987 Constitution
which provides that "(n)o law impairing the obligation of contracts, shall be passed." After the subject
EO was implemented, food manufacturers with existing contracts with petitioner ceased to obtain
certifications from the latter.

Moreover, petitioner argues that the subject EO violates Sections 15 and 16 of Article XIII of the 1987
Constitution which respectively provide:

ROLE AND RIGHTS OF PEOPLE'S ORGANIZATIONS

Sec. 15. The State shall respect the role of independent people's organizations to enable the people to
pursue and protect, within the democratic framework, their legitimate and collective interests and
aspirations through peaceful and lawful means.
People's organizations are bona fide associations of citizens with demonstrated capacity to promote
the public interest and with identifiable leadership, membership, and structure.

Sec. 16. The rights of the people and their organizations to effective and reasonable participation at all
levels of social, political, and economic decision-making shall not be abridged. The State shall, by law,
facilitate, the establishment of adequate consultation mechanisms.

According to petitioner, the subject EO was issued with utter haste and without even consulting
Muslim people's organizations like petitioner before it became effective.

We grant the petition.

OMA was created in 1981 through Executive Order No. 697 (EO 697) "to ensure the integration of
Muslim Filipinos into the mainstream of Filipino society with due regard to their beliefs, customs,
traditions, and institutions."8 OMA deals with the societal, legal, political and economic concerns of the
Muslim community as a "national cultural community" and not as a religious group. Thus, bearing in
mind the constitutional barrier between the Church and State, the latter must make sure that OMA
does not intrude into purely religious matters lest it violate the non-establishment clause and the "free
exercise of religion" provision found in Article III, Section 5 of the 1987 Constitution.9

Freedom of religion was accorded preferred status by the framers of our fundamental law. And this
Court has consistently affirmed this preferred status, well aware that it is "designed to protect the
broadest possible liberty of conscience, to allow each man to believe as his conscience directs, to
profess his beliefs, and to live as he believes he ought to live, consistent with the liberty of others and
with the common good."10

Without doubt, classifying a food product as halal is a religious function because the standards used
are drawn from the Qur'an and Islamic beliefs. By giving OMA the exclusive power to classify food
products as halal, EO 46 encroached on the religious freedom of Muslim organizations like herein
petitioner to interpret for Filipino Muslims what food products are fit for Muslim consumption. Also, by
arrogating to itself the task of issuing halal certifications, the State has in effect forced Muslims to
accept its own interpretation of the Qur'an and Sunnah on halal food.

To justify EO 46's intrusion into the subject religious activity, the Solicitor General argues that the
freedom of religion is subservient to the police power of the State. By delegating to OMA the authority
to issue halal certifications, the government allegedly seeks to protect and promote the muslim
Filipinos' right to health, and to instill health consciousness in them.

We disagree.

Only the prevention of an immediate and grave danger to the security and welfare of the community
can justify the infringement of religious freedom.11 If the government fails to show the seriousness
and immediacy of the threat, State intrusion is constitutionally unacceptable. In a society with a
democratic framework like ours, the State must minimize its interference with the affairs of its citizens
and instead allow them to exercise reasonable freedom of personal and religious activity.

In the case at bar, we find no compelling justification for the government to deprive muslim
organizations, like herein petitioner, of their religious right to classify a product as halal, even on the
premise that the health of muslim Filipinos can be effectively protected by assigning to OMA the
exclusive power to issue halal certifications. The protection and promotion of the muslim Filipinos'
right to health are already provided for in existing laws and ministered to by government agencies
charged with ensuring that food products released in the market are fit for human consumption,
properly labeled and safe. Unlike EO 46, these laws do not encroach on the religious freedom of
muslims.
Section 48(4) of the Administrative Code of 1987 gives to the National Meat Inspection Commission
(NMIC) of the Department of Agriculture (DOA) the power to inspect slaughtered animals intended for
human consumption to ensure the safety of the meat released in the market. Another law, RA 7394,
otherwise known as "The Consumer Act of 1992," gives to certain government departments the duty
to protect the interests of the consumer, promote his general welfare and to establish standards of
conduct for business and industry. 12 To this end, a food product, before its distribution to the market,
is required to secure the Philippine Standard Certification Mark after the concerned department
inspects and certifies its compliance with quality and safety standards.13

One such government agency designated by RA 7394 is the Bureau of Food and Drugs (BFD) of the
Department of Health (DOH). Under Article 22 of said law, BFD has the duty to promulgate and
enforce rules and regulations fixing and establishing a reasonable definition and standard of identity, a
standard of quality and a standard of fill of containers for food. The BFD also ensures that food
products released in the market are not adulterated.14

Furthermore, under Article 48 of RA 7394, the Department of Trade and Industry (DTI) is tasked to
protect the consumer against deceptive, unfair and unconscionable sales acts or practices as defined
in Article 50.15 DTI also enforces compulsory labeling and fair packaging to enable the consumer to
obtain accurate information as to the nature, quality and quantity of the contents of consumer
products and to facilitate his comparison of the value of such products.16

With these regulatory bodies given detailed functions on how to screen and check the quality and
safety of food products, the perceived danger against the health of muslim and non-muslim Filipinos
alike is totally avoided. Of great help are the provisions on labeling of food products (Articles 74 to
85)17 of RA 7394. In fact, through these labeling provisions, the State ably informs the consuming
public of the contents of food products released in the market. Stiff sanctions are imposed on violators
of said labeling requirements.

Through the laws on food safety and quality, therefore, the State indirectly aids muslim consumers in
differentiating food from non-food products. The NMIC guarantees that the meat sold in the market
has been thoroughly inspected and fit for consumption. Meanwhile, BFD ensures that food products
are properly categorized and have passed safety and quality standards. Then, through the labeling
provisions enforced by the DTI, muslim consumers are adequately apprised of the products that
contain substances or ingredients that, according to their Islamic beliefs, are not fit for human intake.
These are the non-secular steps put in place by the State to ensure that the muslim consumers' right
to health is protected. The halal certifications issued by petitioner and similar organizations come
forward as the official religious approval of a food product fit for muslim consumption.

We do not share respondents' apprehension that the absence of a central administrative body to
regulate halal certifications might give rise to schemers who, for profit, will issue certifications for
products that are not actually halal. Aside from the fact that muslim consumers can actually verify
through the labels whether a product contains non-food substances, we believe that they are
discerning enough to know who the reliable and competent certifying organizations in their community
are. Before purchasing a product, they can easily avert this perceived evil by a diligent inquiry on the
reliability of the concerned certifying organization.

WHEREFORE, the petition is GRANTED. Executive Order 46, s. 2000, is hereby declared NULL AND
VOID. Consequently, respondents are prohibited from enforcing the same.

SO ORDERED.

G.R. No. 162224              June 7, 2007


2nd LT. SALVADOR PARREÑO represented by his daughter Myrna P. Caintic, petitioner, 
vs.
COMMISSION ON AUDIT and CHIEF OF STAFF, ARMED FORCES OF THE
PHILIPPINES, respondents.

DECISION

CARPIO, J.:

The Case

Before the Court is a petition for certiorari1 assailing the 9 January 2003 Decision2 and 13 January
2004 Resolution3 of the Commission on Audit (COA).

The Antecedent Facts

Salvador Parreño (petitioner) served in the Armed Forces of the Philippines (AFP) for 32 years. On 5
January 1982, petitioner retired from the Philippine Constabulary with the rank of 2nd Lieutenant.
Petitioner availed, and received payment, of a lump sum pension equivalent to three years pay. In
1985, petitioner started receiving his monthly pension amounting to ₱13,680.

Petitioner migrated to Hawaii and became a naturalized American citizen. In January 2001, the AFP
stopped petitioner’s monthly pension in accordance with Section 27 of Presidential Decree No.
16384 (PD 1638), as amended by Presidential Decree No. 1650.5 Section 27 of PD 1638, as
amended, provides that a retiree who loses his Filipino citizenship shall be removed from the retired
list and his retirement benefits terminated upon loss of Filipino citizenship. Petitioner requested for
reconsideration but the Judge Advocate General of the AFP denied the request.

Petitioner filed a claim before the COA for the continuance of his monthly pension.

The Ruling of the Commission on Audit

In its 9 January 2003 Decision, the COA denied petitioner’s claim for lack of jurisdiction. The COA
ruled:

It becomes immediately noticeable that the resolution of the issue at hand hinges upon the validity of
Section 27 of P.D. No. 1638, as amended. Pursuant to the mandate of the Constitution, whenever a
dispute involves the validity of laws, "the courts, as guardians of the Constitution, have the inherent
authority to determine whether a statute enacted by the legislature transcends the limit imposed by
the fundamental law. Where the statute violates the Constitution, it is not only the right but the duty
of the judiciary to declare such act as unconstitutional and void." (Tatad vs. Secretary of Department
of Energy, 281 SCRA 330) That being so, prudence dictates that this Commission defer to the
authority and jurisdiction of the judiciary to rule in the first instance upon the constitutionality of the
provision in question.

Premises considered, the request is denied for lack of jurisdiction to adjudicate the same. Claimant
is advised to file his claim with the proper court of original jurisdiction.6

Petitioner filed a motion for reconsideration. Petitioner alleged that the COA has the power and
authority to incidentally rule on the constitutionality of Section 27 of PD 1638, as amended. Petitioner
alleged that a direct recourse to the court would be dismissed for failure to exhaust administrative
remedies. Petitioner further alleged that since his monthly pension involves government funds, the
reason for the termination of the pension is subject to COA’s authority and jurisdiction.

In its 13 January 2004 Resolution, the COA denied the motion. The COA ruled that the doctrine of
exhaustion of administrative remedies does not apply if the administrative body has, in the first
place, no jurisdiction over the case. The COA further ruled that even if it assumed jurisdiction over
the claim, petitioner’s entitlement to the retirement benefits he was previously receiving must
necessarily cease upon the loss of his Filipino citizenship in accordance with Section 27 of PD 1638,
as amended.

Hence, the petition before this Court.

The Issues

Petitioner raises the following issues:

1. Whether Section 27 of PD 1638, as amended, is constitutional;

2. Whether the COA has jurisdiction to rule on the constitutionality of Section 27 of PD 1638, as
amended; and

3. Whether PD 1638, as amended, has retroactive or prospective effect.7

The Ruling of this Court

The petition has no merit.

Jurisdiction of the COA

Petitioner filed his money claim before the COA. A money claim is "a demand for payment of a sum
of money, reimbursement or compensation arising from law or contract due from or owing to a
government agency."8 Under Commonwealth Act No. 327,9 as amended by Presidential Decree No.
1445,10 money claims against the government shall be filed before the COA.11

Section 2(1), Article IX(D) of the 1987 Constitution prescribes the powers of the COA, as follows:

Sec. 2. (1) The Commission on Audit shall have the power, authority, and duty to examine, audit,
and settle all accounts pertaining to the revenue and receipts of, and expenditures or uses of funds
and property, owned or held in trust by, or pertaining to, the Government, or any of its subdivisions,
agencies, or instrumentalities, including government-owned or controlled corporations with original
charters, and on a post-audit basis; (a) constitutional bodies, commissions and offices that have
been granted fiscal autonomy under this Constitution; (b) autonomous state colleges and
universities; (c) other government-owned or controlled corporations and their subsidiaries; and (d)
such non-governmental entities receiving subsidy or equity, directly or indirectly, from or through the
Government, which are required by law or the granting institution to submit such audit as a condition
of subsidy or equity. However, where the internal control system of the audited agencies is
inadequate, the Commission may adopt such measures, including temporary or special pre-audit, as
are necessary and appropriate to correct the deficiencies. It shall keep the general accounts of the
Government and, for such period as may be provided by law, preserve the vouchers and other
supporting papers pertaining thereto.
The jurisdiction of the COA over money claims against the government does not include the power
to rule on the constitutionality or validity of laws. The 1987 Constitution vests the power of judicial
review or the power to declare unconstitutional a law, treaty, international or executive agreement,
presidential decree, order, instruction, ordinance, or regulation in this Court and in all Regional Trial
Courts.12 Petitioner’s money claim essentially involved the constitutionality of Section 27 of PD 1638,
as amended. Hence, the COA did not commit grave abuse of discretion in dismissing petitioner’s
money claim.

Petitioner submits that the COA has the authority to order the restoration of his pension even without
ruling on the constitutionality of Section 27 of PD 1638, as amended. The COA actually ruled on the
matter in its 13 January 2004 Resolution, thus:

Furthermore, assuming arguendo that this Commission assumed jurisdiction over the instant case,
claimant’s entitlement to the retirement benefits he was previously receiving must necessarily be
severed or stopped upon the loss of his Filipino citizenship as prescribed in Section 27, P.D. No.
1638, as amended by P.D. No. 1650.13

The COA effectively denied petitioner’s claim because of the loss of his Filipino citizenship.

Application of PD 1638, as amended

Petitioner alleges that PD 1638, as amended, should apply prospectively. The Office of the Solicitor
General (OSG) agrees with petitioner. The OSG argues that PD 1638, as amended, should apply
only to those who joined the military service after its effectivity, citing Sections 33 and 35, thus:

Section 33. Nothing in this Decree shall be construed in any manner to reduce whatever retirement
and separation pay or gratuity or other monetary benefits which any person is heretofore receiving or
is entitled to receive under the provisions of existing law.

xxxx

Section. 35. Except those necessary to give effect to the provisions of this Decree and to preserve
the rights granted to retired or separated military personnel, all laws, rules and regulations
inconsistent with the provisions of this Decree are hereby repealed or modified accordingly.

The OSG further argues that retirement laws are liberally construed in favor of the retirees. Article 4
of the Civil Code provides: "Laws shall have no retroactive effect, unless the contrary is provided."
Section 36 of PD 1638, as amended, provides that it shall take effect upon its approval. It was
signed on 10 September 1979. PD 1638, as amended, does not provide for its retroactive
application. There is no question that PD 1638, as amended, applies prospectively.

However, we do not agree with the interpretation of petitioner and the OSG that PD 1638, as
amended, should apply only to those who joined the military after its effectivity. Since PD 1638, as
amended, is about the new system of retirement and separation from service of military personnel, it
should apply to those who were in the service at the time of its approval. In fact, Section 2 of PD
1638, as amended, provides that "th[e] Decree shall apply to all military personnel in the service of
the Armed Forces of the Philippines." PD 1638, as amended, was signed on 10 September 1979.
Petitioner retired in 1982, long after the approval of PD 1638, as amended. Hence, the provisions of
PD 1638, as amended, apply to petitioner.

Petitioner Has No Vested Right to his


Retirement Benefits

Petitioner alleges that Section 27 of PD 1638, as amended, deprives him of his property which the
Constitution and statutes vest in him. Petitioner alleges that his pension, being a property vested by
the Constitution, cannot be removed or taken from him just because he became a naturalized
American citizen. Petitioner further alleges that the termination of his monthly pension is a penalty
equivalent to deprivation of his life.

The allegations have no merit. PD 1638, as amended, does not impair any vested right or interest of
petitioner. Where the employee retires and meets the eligibility requirements, he acquires a vested
right to the benefits that is protected by the due process clause.14 At the time of the approval of PD
1638 and at the time of its amendment, petitioner was still in active service. Hence, petitioner’s
retirement benefits were only future benefits and did not constitute a vested right. Before a right to
retirement benefits or pension vests in an employee, he must have met the stated conditions of
eligibility with respect to the nature of employment, age, and length of service.15 It is only upon
retirement that military personnel acquire a vested right to retirement benefits. Retirees enjoy a
protected property interest whenever they acquire a right to immediate payment under pre-existing
law.16

Further, the retirement benefits of military personnel are purely gratuitous in nature. They are not
similar to pension plans where employee participation is mandatory, hence, the employees have
contractual or vested rights in the pension which forms part of the compensation.17

Constitutionality of Section 27 of PD 1638

Section 27 of PD 1638, as amended, provides:

Section 27. Military personnel retired under Sections 4, 5, 10, 11 and 12 shall be carried in the
retired list of the Armed Forces of the Philippines. The name of a retiree who loses his Filipino
citizenship shall be removed from the retired list and his retirement benefits terminated upon such
loss.

The OSG agrees with petitioner that Section 27 of PD 1638, as amended, is unconstitutional. The
OSG argues that the obligation imposed on petitioner to retain his Filipino citizenship as a condition
for him to remain in the AFP retired list and receive his retirement benefit is contrary to public policy
and welfare, oppressive, discriminatory, and violative of the due process clause of the Constitution.
The OSG argues that the retirement law is in the nature of a contract between the government and
its employees. The OSG further argues that Section 27 of PD 1638, as amended, discriminates
against AFP retirees who have changed their nationality.

We do not agree.

The constitutional right to equal protection of the laws is not absolute but is subject to reasonable
classification.18 To be reasonable, the classification (a) must be based on substantial distinctions
which make real differences; (b) must be germane to the purpose of the law; (c) must not be limited
to existing conditions only; and (d) must apply equally to each member of the class.19

There is compliance with all these conditions. There is a substantial difference between retirees who
are citizens of the Philippines and retirees who lost their Filipino citizenship by naturalization in
another country, such as petitioner in the case before us. The constitutional right of the state to
require all citizens to render personal and military service20 necessarily includes not only private
citizens but also citizens who have retired from military service. A retiree who had lost his Filipino
citizenship already renounced his allegiance to the state. Thus, he may no longer be compelled by
the state to render compulsory military service when the need arises. Petitioner’s loss of Filipino
citizenship constitutes a substantial distinction that distinguishes him from other retirees who retain
their Filipino citizenship. If the groupings are characterized by substantial distinctions that make real
differences, one class may be treated and regulated differently from another.21

Republic Act No. 707722 (RA 7077) affirmed the constitutional right of the state to a Citizen Armed
Forces. Section 11 of RA 7077 provides that citizen soldiers or reservists include ex-servicemen and
retired officers of the AFP. Hence, even when a retiree is no longer in the active service, he is still a
part of the Citizen Armed Forces. Thus, we do not find the requirement imposed by Section 27 of PD
1638, as amended, oppressive, discriminatory, or contrary to public policy. The state has the right to
impose a reasonable condition that is necessary for national defense. To rule otherwise would be
detrimental to the interest of the state.

There was no denial of due process in this case. When petitioner lost his Filipino citizenship, the
AFP had no choice but to stop his monthly pension in accordance with Section 27 of PD 1638, as
amended. Petitioner had the opportunity to contest the termination of his pension when he requested
for reconsideration of the removal of his name from the list of retirees and the termination of his
pension. The Judge Advocate General denied the request pursuant to Section 27 of PD 1638, as
amended.

Petitioner argues that he can reacquire his Filipino citizenship under Republic Act No. 9225 23 (RA
9225), in which case he will still be considered a natural-born Filipino. However, petitioner alleges
that if he reacquires his Filipino citizenship under RA 9225, he will still not be entitled to his pension
because of its prior termination. This situation is speculative. In the first place, petitioner has not
shown that he has any intention of reacquiring, or has done anything to reacquire, his Filipino
citizenship. Secondly, in response to the request for opinion of then AFP Chief of Staff, General
Efren L. Abu, the Department of Justice (DOJ) issued DOJ Opinion No. 12, series of 2005, dated 19
January 2005, thus:

[T]he AFP uniformed personnel retirees, having re-acquired Philippine citizenship pursuant to R.A.
No. 9225 and its IRR, are entitled to pension and gratuity benefits reckoned from the date they have
taken their oath of allegiance to the Republic of the Philippines. It goes without saying that these
retirees have no right to receive such pension benefits during the time that they have ceased to be
Filipinos pursuant to the aforequoted P.D. No. 1638, as amended, and any payment made to them
should be returned to the AFP. x x x.24

Hence, petitioner has other recourse if he desires to continue receiving his monthly pension. Just
recently, in AASJS Member-Hector Gumangan Calilung v. Simeon Datumanong,25 this Court upheld
the constitutionality of RA 9225. If petitioner reacquires his Filipino citizenship, he will even recover
his natural-born citizenship.26 In Tabasa v. Court of Appeals,27 this Court reiterated that "[t]he
repatriation of the former Filipino will allow him to recover his natural-born citizenship x x x."

Petitioner will be entitled to receive his monthly pension should he reacquire his Filipino citizenship
since he will again be entitled to the benefits and privileges of Filipino citizenship reckoned from the
time of his reacquisition of Filipino citizenship. There is no legal obstacle to the resumption of his
retirement benefits from the time he complies again with the condition of the law, that is, he can
receive his retirement benefits provided he is a Filipino citizen.

We acknowledge the service rendered to the country by petitioner and those similarly situated.
However, petitioner failed to overcome the presumption of constitutionality of Section 27 of PD 1638,
as amended. Unless the provision is amended or repealed in the future, the AFP has to apply
Section 27 of PD 1638, as amended.

WHEREFORE, we DISMISS the petition. We AFFIRM the 9 January 2003 Decision and 13 January


2004 Resolution of the Commission on Audit.

SO ORDERED.

G.R. No. 118910 November 16, 1995

KILOSBAYAN, INCORPORATED, JOVITO R. SALONGA, CIRILO A. RIGOS, ERME CAMBA,


EMILIO C. CAPULONG, JR., JOSE T. APOLO, EPHRAIM TENDERO, FERNANDO SANTIAGO,
JOSE ABCEDE, CHRISTINE TAN, RAFAEL G. FERNANDO, RAOUL V. VICTORINO, JOSE
CUNANAN, QUINTIN S. DOROMAL, SEN. FREDDIE WEBB, SEN. WIGBERTO TAÑADA, REP.
JOKER P. ARROYO, petitioners, 
vs.
MANUEL L. MORATO, in his capacity as Chairman of the Philippine Charity Sweepstakes
Office, and the PHILIPPINE GAMING MANAGEMENT CORPORATION, respondents.

RESOLUTION

MENDOZA, J.:

Petitioners seek reconsideration of our decision in this case. They insist that the decision in the first
case has already settled (1) whether petitioner Kilosbayan, Inc. has a standing to sue and (2)
whether under its charter (R.A. No. 1169, as amended) the Philippine Charity Sweepstakes Office
can enter into any form of association or collaboration with any party in operating an on-line lottery.
Consequently, petitioners contend, these questions can no longer be reopened.

Because two members of the Court did not consider themselves bound by the decision in the first
case, petitioners suggest that the two, in joining the dissenters in the first case in reexamining the
questions in the present case, acted otherwise than according to law. They cite the following
statement in the opinion of the Court:

The voting on petitioners' standing in the previous case was a narrow one, with
seven (7) members sustaining petitioners' standing and six (6) denying petitioners'
right to bring the suit. The majority was thus a tenuous one that is not likely to be
maintained in any subsequent litigation. In addition, there have been changes in the
membership of the Court, with the retirement of Justices Cruz and Bidin and the
appointment of the writer of this opinion and Justice Francisco. Given this fact it is
hardly tenable to insist on the maintenance of the ruling as to petitioners' standing.

Petitioners claim that this statement "conveys a none too subtle suggestion, perhaps a
Freudian slip, that the two new appointees, regardless of the merit of the Decision in the first
Kilosbayan case against the lotto (Kilosbayan, et al. v. Guingona, 232 SCRA 110 (1994))
must of necessity align themselves with all the Ramos appointees who were dissenters in
the first case and constitute the new majority in the second lotto case." And petitioners ask,
"why should it be so?"

Petitioners ask a question to which they have made up an answer. Their attempt at psychoanalysis,
detecting a Freudian slip where none exists, may be more revealing of their own unexpressed wish
to find motives where there are none which they can impute to some members of the Court.

For the truth is that the statement is no more than an effort to explain — rather than to justify — the
majority's decision to overrule the ruling in the previous case. It is simply meant to explain that
because the five members of the Court who dissented in the first case (Melo, Quiason, Puno, Vitug
and Kapunan, JJ.) and the two new members (Mendoza and Francisco, JJ.) thought the previous
ruling to be erroneous and its reexamination not to be barred by stare decisis, res judicata or
conclusiveness of judgment, or law of the case, it was hardly tenable for petitioners to insist on the
first ruling.

Consequently to petitioners' question "What is the glue that holds them together," implying some
ulterior motives on the part of the new majority in reexamining the two questions, the answer is:
None, except a conviction on the part of the five, who had been members of the Court at the time
they dissented in the first case, and the two new members that the previous ruling was erroneous.
The eighth Justice (Padilla, J.) on the other hand agrees with the seven Justices that the ELA is in a
real sense a lease agreement and therefore does not violate R.A. No. 1169.

The decision in the first case was a split decision: 7-6. With the retirement of one of the original
majority (Cruz, J.) and one of the dissenters (Bidin, J.) it was not surprising that the first decision in
the first case was later reversed.

It is argued that, in any case, a reexamination of the two questions is barred because the PCSO and
the Philippine Gaming Management Corporation made a " formal commitment not to ask for a
reconsideration of the Decision in the first lotto case and instead submit a new agreement that would
be in conformity with the PCSO Charter (R.A. No. 1169, as amended) and with the Decision of the
Supreme Court in the first Kilosbayan case against on-line, hi-tech lotto."

To be sure, a new contract was entered into which the majority of the Court finds has been purged of
the features which made the first contract objectionable. Moreover, what the PCSO said in its
manifestation in the first case was the following:

1. They are no longer filing a motion for reconsideration of the Decision of this
Honorable Court dated May 5, 1994, a copy of which was received on May 6, 1994.

2. Respondents PCSO and PGMC are presently negotiating a new lease agreement
consistent with the authority of PCSO under its charter (R.A. No. 1169, as amended
by B.P. Blg. 42) and conformable with the pronouncements of this Honorable Court
in its Decision of May 5, 1995.

The PGMC made substantially the same manifestation as the PCSO.

There was thus no "formal commitment" — but only a manifestation — that the parties were not filing
a motion for reconsideration. Even if the parties made a "formal commitment," the six (6) dissenting
Justices certainly could not be bound thereby not to insist on their contrary view on the question of
standing. Much less were the two new members bound by any "formal commitment" made by the
parties. They believed that the ruling in the first case was erroneous. Since in their view
reexamination was not barred by the doctrine of stare decisis, res judicata or conclusiveness of
judgment or law of the case, they voted the way they did with the remaining five (5) dissenters in the
first case to form a new majority of eight.

Petitioners ask, "Why should this be so?" Because, as explained in the decision, the first decision
was erroneous and no legal doctrine stood in the way of its reexamination. It can, therefore, be
asked "with equal candor": "Why should this not be so?"

Nor is this the first time a split decision was tested, if not reversed, in a subsequent case because of
change in the membership of a court. In 1957, this Court, voting 6-5, held in Feliciano v. Aquinas,
G.R. No. L-10201, Sept. 23, 1957 that the phrase "at the time of the election" in §2174 of the
Revised Administrative Code of 1917 meant that a candidate for municipal elective position must be
at least 23 years of age on the date of the election. On the other hand, the dissenters argued that it
was enough if he attained that age on the day he assumed office.

Less than three years later, the same question was before the Court again, as a candidate for
municipal councilor stated under oath in her certificate of candidacy that she was eligible for that
position although she attained the requisite age (23 years) only when she assumed office. The
question was whether she could be prosecuted for falsification. In People v. Yang, 107 Phi. 888
(1960), the Court ruled she could not. Justice, later Chief Justice, Benison, who dissented in the first
case, Feliciano v. Aquinas, supra, wrote the opinion of the Court, holding that while the statement
that the accused was eligible was "inexact or erroneous, according to the majority in the Feliciano
case," the accused could not be held liable for falsification, because

the question [whether the law really required candidates to have the required age on
the day of the election or whether it was sufficient that they attained it at the
beginning of the term of office] has not been discussed anew, despite the presence
of new members; we simply assume for the purpose of this decision that the doctrine
stands.

Thus because in the meantime there had been a change in the membership of the Court with the
retirement of two members (Recess and Flex, JJ.) who had taken part in the decision in the first
case and their replacement by new members (Barrera and Gutierrez-David, JJ.) and the fact that the
vote in the first case was a narrow one (6 to 5), the Court allowed that the continuing validity of its
ruling in the first case might well be doubted. For this reason it gave the accused the benefit of the
doubt that she had acted in the good faith belief that it was sufficient that she was 23 years of age
when she assumed office.

In that case, the change in the membership of the Court and the possibility of change in the ruling
were noted without anyone — much less would-be psychoanalysts — finding in the statement of the
Court any Freudian slip. The possibility of change in the rule as a result of change in membership
was accepted as a sufficient reason for finding good faith and lack of criminal intent on the part of
the accused.

Indeed, a change in the composition of the Court could prove the means of undoing an erroneous
decision. This was the lesson of Knox v. Lee, 12 Wall. 457 (1871). The Legal Tender Acts, which
were passed during the Civil War, made U.S. notes (greenbacks) legal tender for the payment of
debts, public or private, with certain exceptions. The validity of the acts, as applied to preexisting
debts, was challenged in Hepburn v. Griswold, 8 Wall. 603 (1869). The Court was then composed of
only eight (8) Justices because of Congressional effort to limit the appointing power of President
Johnson. Voting 5-3, the Court declared the acts void. Chief Justice Chase wrote the opinion of the
Court in which four others, including Justice Grier, concurred. Justices Miller, Swayne and Davis
dissented. A private memorandum left by the dissenting Justices described how an effort was made
"to convince an aged and infirm member of the court [Justice Grier] that he had not understood the
question on which he voted," with the result that what was originally a 4-4 vote was converted into a
majority (5-3) for holding the acts invalid.

On the day the decision was announced, President Grant nominated to the Court William Strong and
Joseph P. Bradley to fill the vacancy caused by the resignation of Justice Grier and to restore the
membership of the Court to nine. In 1871, Hepburn v. Griswold was overruled in the Legal Tender
Cases, as Knox v. Lee came to be known, in an opinion by Justice Strong, with a dissenting opinion
by Chief Justice Chase and the three other surviving members of the former majority. There were
allegations that the new Justices were appointed for their known views on the validity of the Legal
Tender Acts, just as there were others who defended the character and independence of the new
Justices. History has vindicated the overruling of the Hepburn case by the new majority. The Legal
Tender Cases proved to be the Court's means of salvation from what Chief Justice Hughes later
described as one of the Court's "self-inflicted wounds." 1

We now consider the specific grounds for petitioners' motion for reconsideration.

I. We have held that because there are no genuine issues of constitutionality in this case, the rule
concerning real party in interest, applicable to private litigation rather than the more liberal rule
on standing, applies to petitioners. Two objections are made against that ruling: (1) that the
constitutional policies and principles invoked by petitioners, while not supplying the basis for
affirmative relief from the courts, may nonetheless be resorted to for striking down laws or official
actions which are inconsistent with them and (2) that the Constitution, by guaranteeing to
independent people's organizations "effective and reasonable participation at all levels of social,
political and economic decision-making" (Art. XIII, §16), grants them standing to sue on
constitutional grounds.

The policies and principles of the Constitution invoked by petitioner read:

Art. II, §5. The maintenance of peace and order, the protection life, liberty, and
property, and thepromotion of the general welfare are essential for the enjoyment by
all the people of the blessings of democracy.

Id., §12. The natural and primary right and duty of parents in the rearing of the youth
for civic efficiency and the development of moral character shall receive the support
of the Government.

Id., §13. The State recognizes the vital role of the youth in nation-building and shall
promote and protect their physical, moral, spiritual, intellectual, and social well-being.
It shall inculcate in the youth patriotism and nationalism, and encourage their
involvement in public and civic affairs.

Id., §17. The State shall give priority to education, science and technology, arts,
culture, and sports to foster patriotism and nationalism, accelerate social progress,
and promote total human liberation and development.

As already stated, however, these provisions are not self-executing. They do not confer rights which
can be enforced in the courts but only provide guidelines for legislative or executive action. By
authorizing the holding of lottery for charity, Congress has in effect determined that consistently with
these policies and principles of the Constitution, the PCSO may be given this authority. That is why
we said with respect to the opening by the PAGCOR of a casino in Cagayan de Oro, "the morality of
gambling is not a justiciable issue. Gambling is not illegalper se. . . . It is left to Congress to deal with
the activity as it sees fit." (Magtajas v. Pryce Properties Corp., Inc., 234 SCRA 255, 268 [1994]).

It is noteworthy that petitioners do not question the validity of the law allowing lotteries. It is the
contract entered into by the PCSO and the PGMC which they are assailing. This case, therefore,
does not raise issues of constitutionality but only of contract law, which petitioners, not being privies
to the agreement, cannot raise.

Nor does Kilosbayan's status as a people's organization give it the requisite personality to question
the validity of the contract in this case. The Constitution provides that "the State shall respect the
role of independent people's organizations to enable the people to pursue and protect, within the
democratic framework, their legitimate and collective interests and aspirations through peaceful and
lawful means," that their right to "effective and reasonable participation at all levels of social, political,
and economic decision-making shall not be abridged." (Art. XIII, §§ 15-16)

These provisions have not changed the traditional rule that only real parties in interest or those with
standing, as the case may be, may invoke the judicial power. The jurisdiction of this Court, even in
cases involving constitutional questions, is limited by the "case and controversy" requirement of Art.
VIII, §5. This requirement lies at the very heart of the judicial function. It is what differentiates
decision-making in the courts from decision-making in the political departments of the government
and bars the bringing of suits by just any party.

Petitioners quote extensively from the speech of Commissioner Garcia before the Constitutional
Commission, explaining the provisions on independent people's organizations. There is nothing in
the speech, however, which supports their claim of standing. On the contrary, the speech points the
way to the legislative and executive branches of the government, rather than to the courts, as the
appropriate fora for the advocacy of petitioners' views.  Indeed, the provisions on independent
2

people's organizations may most usefully be read in connection with the provision on initiative and
referendum as a means whereby the people may propose or enact laws or reject any of those
passed by Congress. For the fact is that petitioners' opposition to the contract in question is nothing
more than an opposition to the government policy on lotteries.

It is nevertheless insisted that this Court has in the past accorded standing to taxpayers and
concerned citizens in cases involving "paramount public interest." Taxpayers, voters, concerned
citizens and legislators have indeed been allowed to sue but then only (1) in cases involving
constitutional issues and 
(2) under certain conditions. Petitioners do not meet these requirements on standing.

Taxpayers are allowed to sue, for example, where there is a claim of illegal disbursement of public
funds. (Pascual v. Secretary of Public Works, 110 Phi. 331 (1960); Sanidad v. Comelec, 73 SCRA
333 (1976); Bugnay Const. & Dev. v. Laron, 176 SCRA 240 (1989); City Council of Cebu v. Cuizon,
47 SCRA 325 [1972]) or where a tax measure is assailed as unconstitutional. (VAT Cases [Tolentino
v. Secretary of Finance], 235 SCRA 630 [1994]) Voters are allowed to question the validity of
election laws because of their obvious interest in the validity of such laws. (Gonzales v. Comelec, 21
SCRA 774 [1967]) Concerned citizens can bring suits if the constitutional question they raise is of
"transcendental importance" which must be settled early. (Emergency Powers Cases [Araneta v.
Dinglasan], 84 Phi. 368 (1949); Iloilo Palay and Corn Planters Ass'n v. Feliciano, 121 Phi. 358
(1965); Philconsa v. Gimenez, 122 Phi. 894 (1965); CLU v. Executive Secretary, 194 SCRA 317
[1991]) Legislators are allowed to sue to question the validity of any official action which they claim
infringes their prerogatives qua legislators. (Philconsa v. Enriquez, 235 506 (1994); Guingona v.
PCGG, 207 SCRA 659 (1992); Gonzales v. Macaraig, 191 SCRA 452 (1990); Tolentino v. Comelec,
41 SCRA 702 (1971); Tatad v. Garcia, G.R. No. 114222, April 16, 1995 (Mendoza, J., concurring))
Petitioners do not have the same kind of interest that these various litigants have. Petitioners assert
an interest as taxpayers, but they do not meet the standing requirement for bringing taxpayer's suits
as set forth in Dumlao v. Comelec, 95 SCRA 392, 403 (1980), to wit:

While, concededly, the elections to be held involve the expenditure of public


moneys, nowhere in their Petition do said petitioners allege that their tax money
is "being extracted and spent in violation of specific constitutional protections against
abuses of legislative power" (Flast v. Cohen, 392 U.S., 83 [1960]), or that there is a
misapplication of such funds by respondent COMELEC (see Pascual vs. Secretary of
Public Works, 110 Phil. 331 [1960]), or that public money is being deflected to any
improper purpose. Neither do petitioners seek to restrain respondent from wasting
public funds through the enforcement of an invalid or unconstitutional law. (Philippine
Constitution Association vs. Mathay, 18 SCRA 300 [1966]), citing Philippine
Constitution Association vs. Gimenez, 15 SCRA 479 [1965]). Besides, the institution
of a taxpayer's suit, per se, is no assurance of judicial review. As held by this Court in
Tan vs. Macapagal (43 SCRA 677 [1972]), speaking through our present Chief
Justice, this Court is vested with discretion as to whether or not a taxpayer's suit
should be entertained. (Emphasis added)

Petitioners' suit does not fall under any of these categories of taxpayers' suits.

Neither do the other cases cited by petitioners support their contention that taxpayers have standing
to question government contracts regardless of whether public funds are involved or not.
In Gonzales v. National Housing, Corp., 94 SCRA 786 (1979), petitioner filed a taxpayer's suit
seeking the annulment of a contract between the NHC and a foreign corporation. The case was
dismissed by the trial court. The dismissal was affirmed by this Court on the grounds of res
judicata and pendency of a prejudicial question, thus avoiding the question of petitioner's standing.

On the other hand, in Gonzales v. Raquiza, 180 SCRA 254 (1989), petitioner sought the annulment
of a contract made by the government with a foreign corporation for the purchase of road
construction equipment. The question of standing was not discussed, but even if it was, petitioner's
standing could be sustained because he was a minority stockholder of the Philippine National Bank,
which was one of the defendants in the case.

In the other case cited by petitioners, City Council of Cebu v. Cuizon, 47 SCRA 325 (1972),
members of the city council were allowed to sue to question the validity of a contract entered into by
the city government for the purchase of road construction equipment because their contention was
that the contract had been made without their authority. In addition, as taxpayers they had an
interest in seeing to it that public funds were spent pursuant to an appropriation made by law.

But, in the case at bar, there is an allegation that public funds are being misapplied or
misappropriated. The controlling doctrine is that of Gonzales v. Marcos, 65 SCRA 624 (1975) where
it was held that funds raised from contributions for the benefit of the Cultural Center of the
Philippines were not public funds and petitioner had no standing to bring a taxpayer's suit to question
their disbursement by the President of the Philippines.

Thus, petitioners' right to sue as taxpayers cannot be sustained. Nor as concerned citizens can they
bring this suit because no specific injury suffered by them is alleged. As for the petitioners, who are
members of Congress, their right to sue as legislators cannot be invoked because they do not
complain of any infringement of their rights as legislators.
Finally, in Valmonte v. PCSO, G.R. No. 78716, September 22, 1987, we threw out a petition
questioning another form of lottery conducted by the PCSO on the ground that petitioner, who
claimed to be a "citizen, lawyer, taxpayer and father of three minor children," had no direct and
personal interest in the lottery. We said: "He must be able to show, not only that the law is invalid,
but also that he has sustained or is in immediate danger of sustaining some direct injury as a result
of its enforcement, and not merely that he suffers thereby in some indefinite way. It must appear that
the person complaining has been or is about to be denied some right or privilege to which he is
lawfully entitled or that he is about to be subjected to some burdens or penalties by reason of the
statute complained of." In the case at bar, petitioners have not shown why, unlike petitioner in the
Valmonte case, they should be accorded standing to bring this suit.

The case of Oposa v. Factoran, Jr. 224 SCRA 792 (1993) is different. Citizens' standing to bring a
suit seeking the cancellation of timber licenses was sustained in that case because the Court
considered Art. II, §16 a right-conferring provision which can be enforced in the courts. That
provision states:

The State shall protect and advance the right of the people to a balanced and
healthful ecology in accord with the rhythm and harmony of nature. (Emphasis)

In contrast, the policies and principles invoked by petitioners in this case do not permit of
such categorization.

Indeed, as already stated, petitioners' opposition is not really to the validity of the ELA but to lotteries
which they regard to be immoral. This is not, however, a legal issue, but a policy matter for Congress
to decide and Congress has permitted lotteries for charity.

Nevertheless, although we have concluded that petitioners do not have standing, we have not
stopped there and dismissed their case. For in the view we take, whether a party has a cause of
action and, therefore, is a real party in interest or one with standing to raise a constitutional question
must turn on whether he has a right which has been violated. For this reason the Court has not
ducked the substantive issues raised by petitioners.

II. R.A. No. 1169, as amended by B.P No . 42, states:

§1. The Philippine Charity Sweepstakes Office. — The Philippine Charity


Sweepstakes Office, hereinafter designated the Office, shall be the principal
government agency for raising and providing for funds for health programs, medical
assistance and services and charities of national character, and as such shall have
the general powers conferred in section thirteen of Act Numbered One Thousand
Four Hundred Fifty-Nine, as amended, and shall have the authority:

A. To hold and conduct charity sweepstakes races, lotteries and other similar
activities, in such frequency and manner, as shall be determined, and subject to such
rules and regulations as shall be promulgated by the Board of Directors.

B. Subject to the approval of the Minister of Human Settlements, to engage in health


and welfare-related investments, programs, projects and activities which may be
profit-oriented, by itself or in collaboration, association or joint venture with any
person, association, company or entity, whether domestic or foreign, except for the
activities mentioned in the preceding paragraph (A), for the purpose of providing for
permanent and continuing sources of funds for health programs, including the
expansion of existing ones, medical assistance and services, and/or charitable
grants: Provided, That such investments will not compete with the private sector in
areas where investments are adequate as may be determined by the National
Economic and Development Authority.

Petitioners insist on the ruling in the previous case that the PCSO cannot hold and conduct charity
sweepstakes, lotteries and other similar activities in collaboration, association or joint venture with
any other party because of the clause "except for the activities mentioned in the preceding
paragraph (A)" in paragraph (B) of §1. Petitioners contend that the ruling is the law of this case
because the parties are the same and the case involves the same issue, i.e., the meaning of this
statutory provision.

The "law of the case" doctrine is inapplicable, because this case is not a continuation of the first one.
Petitioners also say that inquiry into the same question as to the meaning of the statutory provision
is barred by the doctrine of res judicata. The general rule on the "conclusiveness of judgment,"
however, is subject to the exception that a question may be reopened if it is a legal question and the
two actions involve substantially different claims. This is generally accepted in American law from
which our Rules of Court was adopted. (Montana v. United States, 440 U.S. 59 L.Ed.2d 147, 210
(1979); RESTATEMENT OF THE LAW 2d, ON JUDGMENTS, §28; P. BATOR, D. MELTZER, P.
MISHKIN AND D. SHAPIRO, THE FEDERAL COURTS AND THE FEDERAL SYSTEM 1058, n.2
[3rd Ed., 1988]) There is nothing in the record of this case to suggest that this exception is
inapplicable in this jurisdiction.

Indeed, the questions raised in this case are legal questions and the claims involved are
substantially different from those involved in the prior case between the parties. As already stated,
the ELA is substantially different from the Contract of Lease declared void in the first case.

Borrowing from the dissenting opinion of Justice Feliciano, petitioners argue that the phrase "by itself
or in collaboration, association or joint venture with any other party" qualifies not only §1 (B) but also
§1 (A), because the exception clause ("except for the activities mentioned in the preceding
paragraph [A]") "operates, as it were, as a renvoi clause which refers back to Section 1(A) and in
this manner avoids the necessity of simultaneously amending the text of Section 1(A)."

This interpretation, however, fails to take into account not only the location of the phrase in
paragraph (B), when it should be in paragraph (A) had that been the intention of the lawmaking
authority, but also the phrase "by itself." In other words, under paragraph (B), the PCSO is prohibited
from "engag[ing] in . . . investments, programs, projects and activities" if these involve sweepstakes
races, lotteries and other similar activities not only "in collaboration, association or joint venture" with
any other party but also "by itself." Obviously, this prohibition cannot apply when the PCSO conducts
these activities itself. Otherwise, what paragraph (A) authorizes the PCSO to do, paragraph (B)
would prohibit.

The fact is that the phrase in question does not qualify the authority of the PCSO under paragraph
(A), but rather the authority granted to it by paragraph (B). The amendment of paragraph (B) by B.P.
Blg. 42 was intended to enable the PCSO to engage in certain investments, programs, projects and
activities for the purpose of raising funds for health programs and charity. That is why the law
provides that such investments by the PCSO should "not compete with the private sector in areas
where investments are adequate as may be determined by the National Economic and Development
Authority." Justice Davide, then an Assemblyman, made a proposal which was accepted, reflecting
the understanding that the bill they were discussing concerned the authority of the PCSO to invest in
the business of others. The following excerpt from the Record of the Batasan Pambansa shows this
to be the subject of the discussion:
MR. DAVIDE. May I introduce an amendment after "adequate". The intention of the
amendment is not to leave the determination of whether it is adequate or not to
anybody. And my amendment is to add after "adequate" the words AS MAY BE
DETERMINED BY THE NATIONAL ECONOMIC AND DEVELOPMENT
AUTHORITY. As a mater of fact, it will strengthen the authority to invest in these
areas, provided that the determination of whether the private sector's activity is
already adequate must be determined by the National Economic and Development
Authority.

Mr. ZAMORA. Mr. Speaker, the committee accepts the proposed amendment.

MR. DAVIDE. Thank you, Mr. Speaker.

(2 RECORD OF THE BATASAN PAMBANSA, Sept. 6, 1979,


p. 1007)

Thus what the PCSO is prohibited from doing is from investing in a business engaged in
sweepstakes races, lotteries and other similar activities. It is prohibited from doing so whether "in
collaboration, association or joint venture" with others or "by itself." This seems to be the only
possible interpretation of §1 (A) and (B) in light of its text and its legislative history. That there is
today no other entity engaged in sweepstakes races, lotteries and the like does not detract from the
validity of this interpretation.

III. The Court noted in its decision that the provisions of the first contract, which were considered to
be features of a joint venture agreement, had been removed in the new contract. For instance, §5 of
the ELA provides that in the operation of the on-line lottery, the PCSO must employ "its own
competent and qualified personnel." Petitioners claim, however, that the "contemporaneous
interpretation" of PGMC officials of this provision is otherwise. They cite the testimony of Glen
Barroga of the PGMC before a Senate committee to the effect that under the ELA the PGMC would
be operating the lottery system "side by side" with PCSO personnel as part of the transfer of
technology.

Whether the transfer of technology would result in a violation of PCSO's franchise should be
determined by facts and not by what some officials of the PGMC state by way of opinion. In the
absence of proof to the contrary, it must be presumed that §5 reflects the true intention of the
parties. Thus, Art. 1370 of the Civil Code says that "If the terms of a contract are clear and leave no
doubt upon the intention of the contracting parties, the literal meaning of its stipulations shall
control." The intention of the parties must be ascertained from their "contemporaneous and
subsequent acts." (Art. 1371; Atlantic Gulf Co. v. Insular Government, 10 Phil. 166 [1908]) It cannot
simply be judged from what one of them says. On the other hand, the claim of third parties, like
petitioners, that the clause on upgrading of equipment would enable the parties after a while to
change the contract and enter into something else in violation of the law is mere speculation and
cannot be a basis for judging the validity of the contract.

IV. It is contended that §1 of E.O. No. 301 covers all types of "contract[s] for public services or for
furnishing of supplies, materials and equipment to the government or to any of its branches,
agencies or instrumentalities" and not only contracts of purchase and sale. Consequently, a lease of
equipment, like the ELA, must be submitted to public bidding in order to be valid. This contention is
based on two premises: (1) that §1 of E.O. No. 301 applies to any contract whereby the government
acquires title to or the use of the equipment and (2) that the words "supplies," "materials," and
"equipment" are distinct from each other so that when an exception in §1 speaks of "supplies," it
cannot be construed to mean "equipment."
Petitioners' contention will not bear analysis. For example, the term "supplies" is used in paragraph
(a), which provides that a contract for the furnishing of "supplies" in order to meet an emergency is
exempt from public bidding. Unless "supplies" is construed to include "equipment," however, the
lease of heavy equipment needed for rescue operations in case of a calamity will have to be
submitted to public bidding before it can be entered into by the government.

In dissent Justice Feliciano says that in such a situation the government can simply resort to
expropriation, paying compensation afterward. This is just like purchasing the equipment through
negotiation when the question is whether the purchase should be by public bidding, not to mention
the fact that the power to expropriate may not be exercised when the government can very well
negotiate with private owners.

Indeed, there are fundamental difficulties in simultaneously contending (1) that E.O. No. 301, §1
covers both contracts of sale and lease agreements and (2) that the words "supplies,"
"materials" and "equipment" can not be interchanged. Thus, under paragraph (b) of §1, public
bidding is not required "whenever the supplies are to be used in connection with a project or activity
which cannot be delayed without causing detriment to the public service." Following petitioners'
theory, there should be a public bidding before the government can enter into a contract for the
lease of bulldozers and dredging equipment even if these are urgently needed in areas ravaged by
lahar because, first, lease contracts are covered by the general rule and, second, the exception to
public bidding in paragraph (b) covers only "supplies" but not equipment.

To take still another example. Paragraph (d), which does away with the requirement of public bidding
"whenever the supplies under procurement have been unsuccessfully placed on bid for at least two
consecutive times, either due to lack of bidders or the offers received in each instance were
exorbitant or nonconforming to specifications." Again, following the theory of the petitioners, a
contract for the lease of equipment cannot be entered into even if there are no bids because, first,
lease contracts are governed by the general rule on public bidding and, second, the exception to
public bidding in paragraph (d) applies only to contracts for the furnishing of "supplies."

Other examples can be given to show the absurdity of interpreting §1 as applicable to any contract
for the furnishing of supplies, materials and equipment and of considering the words "supplies,"
"materials" and "equipment" to be not interchangeable. Our ruling that §1 of E.O. No. 301 does not
cover the lease of equipment avoids these fundamental difficulties and is supported by the text of
§1, which is entitled "Guidelines for Negotiated Contracts" and by the fact that the only provisions of
E.O. No. 301 on leases, namely, §§6 and 7, concern the lease of buildings by or to the government.
Thus the text of §1 reads:

§1. Guidelines for Negotiated Contracts. — Any provision of law, decree, executive
order or other issuances to the contrary notwithstanding, no contract for public
services or for furnishing supplies, materials and equipment to the government or
any of its branches, agencies or instrumentalities shall be renewed or entered into
without public bidding, except under any of the following situations:

a. Whenever the supplies are urgently needed to meet an emergency


which may involve the loss of, or danger to, life and/or property;

b. Whenever the supplies are to be used in connection with a project


or activity which cannot be delayed without causing detriment to the
public service;
c. Whenever the materials are sold by an exclusive distributor or
manufacturer who does not have subdealers selling at lower prices
and for which no suitable substitute can be obtained elsewhere at
more advantageous terms to the government;

d. Whenever the supplies under procurement have been


unsuccessfully placed on bid for at least two consecutive times, either
due to lack of bidders or the offers received in each instance were
exhorbitant or non-conforming to specifications;

e. In cases where it is apparent that the requisition of the needed


supplies through negotiated purchase is most advantageous to the
government to be determined by the Department Head concerned;
and

f. Whenever the purchase is made from an agency of the


government.

Indeed, the purpose for promulgating E.O. No. 301 was merely to decentralize the system of
reviewing negotiated contracts of purchase for the furnishing of supplies, materials and equipment
as well as lease contracts of buildings. Theretofore, E.O. No. 298, promulgated on August 12, 1940,
required consultation with the Secretary of Justice and the Department Head concerned and the
approval of the President of the Philippines before contracts for the furnishing of supplies, materials
and equipment could be made on a negotiated basis, without public bidding. E.O. No. 301 changed
this by providing as follows:

§2. Jurisdiction over Negotiated Contracts. — In line with the principles of


decentralization and accountability, negotiated contracts for public services or for
furnishing supplies, materials or equipment may be entered into by the department or
agency head or the governing board of the government-owned or controlled
corporation concerned, without need of prior approval by higher authorities, subject
to availability of funds, compliance with the standards or guidelines prescribed in
Section 1 hereof, and to the audit jurisdiction of the commission on Audit in
accordance with existing rules and regulations.

Negotiated contracts involving P2,000,000 up to P10,000,000 shall be signed by the


Secretary and two other Undersecretaries.

xxx xxx xxx

§7. Jurisdiction Over Lease Contracts. — The heads of agency intending to rent
privately-owned buildings or spaces for their use, or to lease out government-owned
buildings or spaces for private use, shall have authority to determine the
reasonableness of the terms of the lease and the rental rates thereof, and to enter
into such lease contracts without need of prior approval by higher authorities, subject
to compliance with the uniform standards or guidelines established pursuant to
Section 6 hereof by the DPWH and to the audit jurisdiction of COA or its duly
authorized representative in accordance with existing rules and regulations.

In sum, E.O. No. 301 applies only to contracts for the purchase of supplies, materials and
equipment, and it was merely to change the system of administrative review of emergency
purchases, as theretofore prescribed by E.O. No. 298, that E.O. No. 301 was issued on July 26,
1987. Part B of this Executive Order applies to leases of buildings, not of equipment, and therefore
does not govern the lease contract in this case. Even if it applies, it does not require public bidding
for entering into it.

Our holding that E.O. No. 301, §1 applies only to contracts of purchase and sale is conformable to
P.D. No. 526, promulgated on August 2, 1974, which is in pari materia. P.D. No. 526 requires local
governments to hold public bidding in the "procurement of supplies." By specifying "procurement of
supplies" and excepting from the general rule "purchases" when made under certain circumstances,
P.D. No. 526, §12 indicates quite clearly that it applies only to contracts of purchase and sale. This
provision reads:

§12. Procurement without public bidding. — Procurement of supplies may be made


without the benefit of public bidding in the following modes:

(1) Personal canvass of responsible merchants;

(2) Emergency purchases;

(3) Direct purchases from manufacturers or exclusive distributors;

(4) Thru the Bureau of Supply Coordination; and

(5) Purchase from other government entities or foreign governments.

Sec. 3 broadly defines the term "supplies" as including —

everything except real estate, which may be needed in the


transaction of public business, or in the pursuit of any undertaking,
project, or activity, whether of the nature of equipment, furniture,
stationery, materials for construction, or personal property of any sort,
including non-personal or contractual services such as the repair and
maintenance of equipment and furniture, as well as trucking, hauling,
janitorial, security, and related or analogous services.

Thus, the texts of both E.O. No. 301, §1 and of P.D. No. 526, §§1 and 12, make it clear that only
contracts for the purchase and sale of supplies, materials and equipment are contemplated by the
rule concerning public biddings.

Finally, it is contended that equipment leases are attractive and commonly used in place of contracts
of purchase and sale because of "multifarious credit and tax constraints" and therefore could not
have been left out from the requirement of public bidding. Obviously these credit and tax constraints
can have no attraction to the government when considering the advantages of sale over lease of
equipment. The fact that lease contracts are in common use is not a reason for implying that the rule
on public bidding applies not only to government purchases but also to lease contracts. For the fact
also is that the government leases equipment, such as copying machines, personal computers and
the like, without going through public bidding.

FOR THE FOREGOING REASONS, the motion for reconsideration of petitioners is DENIED with
finality.

SO ORDERED.
 

A.M. No. 10-4-19-SC

RE: LETTER OF TONY Q. V ALENCIANO, HOLDING OF RELIGIOUS RITUALS AT THE HALL


OF JUSTICE BUILDING IN QUEZON CITY

RESOLUTION

MENDOZA, J.:

One of our fundamental differences lies in our chosen religion. Some put their faith in a god different
from ours, while some may not believe in a god at all. Nevertheless, despite the inconveniences this
difference may cause us, we must accept it unconditionally for only upon acceptance of the fact that
we are different from each other will we learn to respect one another.

This controversy originated from a series of letters, written by Tony Q. Valenciano (Valenciano) and


addressed to then Chief Justice Reynato S.

Puno (Chief Justice Puno).

In his first Letter,  dated January 6, 2009, Valenciano reported that the basement of the Hall of
1

Justice of Quezon City (QC) had been converted into a Roman Catholic Chapel, complete with
offertory table, images of Catholic religious icons, a canopy, an electric organ, and a projector. He
believed that such practice violated the constitutional provision on the separation of Church and
State and the constitutional prohibition against the appropriation of public money or property for the
benefit of a sect, church, denomination, or any other system of religion.

Valenciano further averred that the holding of masses at the basement of the QC Hall of Justice
showed that it tended to favor Catholic litigants; that the rehearsals of the choir caused great
disturbance to other employees; that the public could no longer use the basement as resting place;
that the employees and litigants of the Public Attorney's Office (PAO), Branches 82 and 83 of the
Regional Trial Court (RTC), Legal Library, Philippine Mediation Center, and Records Section of the
Office of the Clerk of Court (OCC) could not attend to their personal necessities such as going to the
lavatories because they could not traverse the basement between 12:00 o'clock noontime and 1: 15
o'clock in the afternoon; that the court employees became hostile toward each other as they vied for
the right to read the epistle; and that the water supply in the entire building was cut off during the
mass because the generator was turned off to ensure silence.

In his 1st Indorsement,  dated February 6, 2009, Chief Justice Puno referred Valenciano 's letter to
2

then Deputy Court Administrator (DCA) and Officer-in-Charge of the Office on Halls of Justice,


Antonio H. Dujua (DCA Dujua).

In turn, DCA Dujua, in his 1st Indorsement,  dated February 11, 2009, referred the letter to Executive
3

Judge Teodoro A. Bay (Judge Bay) of the RTC and to Executive Judge Luis Zenon Q.
Maceren (Judge Maceren) of the Metropolitan Trial Court (MeTC) for their respective comments.

In his March 6, 2009 Letter,  addressed to DCA Dujua, Judge Maceren clarified that the basement of
4

the QC Hall of Justice was known as the prayer corner. He opined that the use of the said area for
holding masses did not violate the constitutional prohibition against the use of public property for
religious purposes because the religious character of such use was merely incidental to a temporary
use.

In his Memorandum,  dated March 10, 2009, Judge Bay manifested that he was due to compulsorily
5

retire on April 29, 2009, and he was taking a leave of absence prior to such date to concentrate in
resolving cases submitted for decision before his sala and requested that then Vice-Executive Judge
Jaime N. Salazar (Judge Salazar) be assigned to further investigate, study, and make
recommendations on the matter raised by Valenciana.

In the meantime, Judge Bay recommended that, pending the final resolution of the case, daily
masses be permitted to continue, provided that: (1) the mass be limited to thirty (30) minutes; (2) no
loud singing be allowed so as not to disturb others; and (3) the inconveniences caused by the mass
be addressed.

In his 1st Indorsement,  dated May 27, 2009, Chief Justice Puno referred another letter of
6

Valenciano, dated May 13, 2009, to DCA Dujua for appropriate action, as he complained that
masses continued to be held at the basement of the QC Hall of Justice.

On March 23, 2010, Valenciano wrote another letter,  praying that rules be promulgated by the Court
7

to put a stop to the holding of Catholic masses, or any other religious rituals, at the QC Hall of
Justice and in all other halls of justice in the country.

In its June 22, 2010 Resolution,  the Court noted the March 23, 2010 letter of Valenciano and
8

referred the matter to the Office of the Court Administrator (OCA) for evaluation, report and
recommendation.

Thus, in its 1st Indorsement,  dated September 6, 2010, the OCA, through then Assistant Court
9

Administrator (ACA) Jenny Lind R. AldecoaDelorino (now Deputy Court Administrator), referred the


letters of Valenciano to the incumbent RTC Executive Judge Fernando T. Sagun, Jr. (Judge Sagun,
Jr.) and incumbent MeTC Executive Judge Caridad M. WalseLutero (Judge Lutero).

In his Letter-Comment,  dated September 9, 2010, Judge Sagun, Jr. informed the Court that his
10

office had already implemented measures to address Valenciano's complaints. He reported that
masses were shortened to a little over thirty (30) minutes; that it was only during special holy days of
obligation when the celebration of mass went beyond one (1) o'clock in the afternoon; that the
pathways leading to the lavatories were open and could be used without obstruction; that there was
never an instance where the actions of court personnel, who were vying to read the epistle during
mass, caused back-biting and irritation among themselves; that the water generator had been
broken beyond repair and decommissioned since December 2009; and that the court employees
prepared for the mass before the day officially started, so that the performance of their official duties
in court was not hampered.

In her letter,  Judge Lutero reported that Catholic masses were being held only during lunch breaks
11

and did not disturb court proceedings; that the basement of the QC Hall of Justice could still be used
as waiting area for the public; that court personnel and the public were never physically prevented
from reaching the lavatories during mass as there was a clear path from the public offices leading to
the comfort rooms; that water service interruptions were caused by maintenance problems and not
because the water pump was being shut off during mass; and that the elevators could not be used
during mass because elevator attendants took their lunch break from twelve (12) o'clock to one (1)
o'clock in the afternoon.
Judge Lutero opined that it is not the conduct of masses in public places which the Constitution
prohibited, but the passage of laws or the use of public funds for the purpose of establishing a
religion or prohibiting the free exercise thereof. She conveyed the fact that no law or rule had been
passed and that no public funds had been appropriated or used to support the celebration of
masses. She added that the holding of Catholic masses did not mean that Catholics had better
chances of obtaining favorable resolutions from the court.

Accordingly, Judge Lutero recommended that the holding of masses at the basement of the QC Hall
of Justice be allowed to continue considering that it was not inimical to the interests of the court
employees and the public.

The OCA Report


and Recommendation

In its Memorandum,  dated August 7, 2014, the OCA believed that the practical inconveniences
12

cited by Valenciano were unfounded. It, thus, recommended that his letter-complaints, dated
January 6, 2009, May 13, 2009 and March 23, 2010, be dismissed for lack of merit and that the RTC
and MeTC Executive Judges of QC be directed to closely regulate and monitor the holding of
masses and other religious practices within the premises of the QC Hall of Justice. 1âwphi1

The OCA opined that the principle of separation of Church and State, particularly with reference to
the Establishment Clause, ought not to be interpreted according to the rigid standards of separation;
that the neutrality of the State on religion should be benevolent because religion was an ingrained
part of society and played an important role in it; and that the State, therefore, instead of being
belligerent (in the case of Strict Separation) or being aloof (in the case of Strict Neutrality) towards
religion should instead interact and forbear.
13

The OCA advanced the view that the standard of Benevolent Neutrality/Accommodation was
espoused because the principal religion clauses in our Constitution were not limited to the
Establishment Clause, which created a wall between the Church and the State, but was quickly
followed by the declaration of the Free Exercise Clause, which protected the right of the people to
practice their religion. In effect, the standard of Benevolent Neutrality/Accommodation balanced the
interest of the State through the Establishment Clause, and the interest and right of the individual to
freely exercise his religion as guaranteed by the Free Exercise Clause. 14

The OCA observed that the present controversy did not involve a national or local law or regulation
in conflict with the Free Exercise Clause. On the contrary, Valenciano was merely questioning the
propriety of holding religious masses at the basement of the QC Hall of Justice, which was nothing
more than an issue of whether the said religious practice could be accommodated or not. It ended
up concluding that based on prevailing jurisprudence, as well as the interpretations given to the
religion clauses of the 1987 Constitution, there was nothing constitutionally abhorrent in allowing the
continuation of the masses. 15

The OCA added that by allowing or accommodating the celebration of Catholic masses within the
premises of the QC Hall of Justice, the Court could not be said to have established Roman
Catholicism as an official religion or to have endorsed the said religion, for the reason that it also
allowed other religious denominations to practice their religion within the courthouses. 16

ISSUE

WHETHER THE HOLDING OF MASSES AT THE BASEMENT OF THE QUEZON CITY HALL OF
JUSTICE VIOLATES THE CONSTITUTIONAL PRINCIPLE OF SEPARATION OF CHURCH AND
STATE AS WELL AS THE CONSTITUTIONAL PROHIBITION AGAINST APPROPRIATION OF
PUBLIC MONEY OR PROPERTY FOR THE BENEFIT OF ANY SECT, CHURCH,
DENOMINATION, SECTARIAN INSTITUTION, OR SYSTEM OF RELIGION.

The Court's Ruling

The Court agrees with the findings and recommendation of the OCA and denies the prayer of
Valenciano that the holding of religious rituals of any of the world's religions in the QC Hall of Justice
or any halls of justice all over the country be prohibited.

The Holding of Religious


Rituals in the Halls of Justice
does not Amount to a Union of
Church and State

As earlier stated, Valenciano is against the holding of religious rituals in the halls of justice on the
ground that it violates the constitutional provision on the separation of Church and State and the
constitutional prohibition against the appropriation of public money or property for the benefit of a
sect, church, denomination, or any other system of religion. Indeed, Section 6, Article II of the 1987
Constitution provides:

The separation of Church and State shall be inviolable. 17

The Court once pronounced that "our history, not to speak of the history of mankind, has taught us
that the union of church and state is prejudicial to both, for occasions might arise when the state will
use the church, and the church the state, as a weapon in the furtherance of their respective ends
and aims." 18

Justice Isagani Cruz expounded on this doctrine, viz.:

The rationale of the rule is summed up in the familiar saying, "Strong fences make good neighbors."
The idea is to delineate the boundaries between the two institutions and, thus, avoid encroachments
by one against the other because of a misunderstanding of the limits of their respective exclusive
jurisdictions. The demarcation line calls on the entities to "render therefore unto Caesar the things
that are Caesar's and unto God the things that are God's." 19

This, notwithstanding, the State still recognizes the inherent right of the people to have some form of
belief system, whether such may be belief in a Supreme Being, a certain way of life, or even an
outright rejection of religion. Our very own Constitution recognizes the heterogeneity and religiosity
of our people as reflected in lmbong v. Ochoa, as follows:
20

At the outset, it cannot be denied that we all live in a heterogeneous society. It is made up of people
of diverse ethnic, cultural and religious beliefs and backgrounds. History has shown us that our
government, in law and in practice, has allowed these various religious, cultural, social and racial
groups to thrive in a single society together. It has embraced minority groups and is tolerant towards
all - the religious people of different sects and the non-believers. The undisputed fact is that our
people generally believe in a deity, whatever they conceived Him to be, and to Whom they called for
guidance and enlightenment in crafting our fundamental law. Thus, the preamble of the present
Constitution reads:
We, the sovereign Filipino people, imploring the aid of Almighty God, in order to build a just and
humane society, and establish a Government that shall embody our ideals and aspirations, promote
the common good, conserve and develop our patrimony, and secure to ourselves and our posterity,
the blessings of independence and democracy under the rule of law and a regime of truth, justice,
freedom, love, equality, and peace, do ordain and promulgate this Constitution.

The Filipino people in "imploring the aid of Almighty God" manifested their spirituality innate in our
nature and consciousness as a people, shaped by tradition and historical experience. As this is
embodied in the preamble, it means that the State recognizes with respect the influence of religion in
so far as it instills into the mind the purest principles of morality. Moreover, in recognition of the
contributions of religion to society, the 1935, 1973 and 1987 Constitutions contain benevolent and
accommodating provisions towards religions such as tax exemption of church property, salary of
religious officers in government institutions, and optional religious instructions in public schools.
[Emphases supplied]

In Aglipay v. Ruiz  (Aglipay), the Court acknowledged how religion could serve as a motivating force
21

behind each person's actions:

Religious freedom, however, as a constitutional mandate is not inhibition of profound reverence for
religion and is not a denial of its influence in human affairs. Religion as a profession of faith to an
active power that binds and elevates man to his Creator is recognized. And, in so far as it instills into
the minds the purest principles of morality, its influence is deeply felt and highly appreciated. When
the Filipino people, in the preamble of their Constitution, implored "the aid of Divine Providence, in
order to establish a government that shall embody their ideals, conserve and develop the patrimony
of the nation, promote the general welfare, and secure to themselves and their posterity the
blessings of independence under a regime of justice, liberty and democracy," they thereby
manifested their intense religious nature and placed unfaltering reliance upon Him who guides the
destinies of men and nations. The elevating influence of religion in human society is recognized here
as elsewhere. In fact, certain general concessions are indiscriminately accorded to religious sects
and denominations. Our Constitution and laws exempt from taxation properties devoted exclusively
to religious purposes (sec. 14, subsec. 3, Art. VI, Constitution of the Philippines and sec. 1, subsec.
Ordinance appended thereto; Assessment Law, sec. 344, par [c], Adm. Code) sectarian aid is not
prohibited when a priest, preacher, minister or other religious teacher or dignitary as such is
assigned to the armed forces or to any penal institution, orphanage or leprosarium xxx. Optional
religious instruction in the public schools is by constitutional mandate allowed xxx. Thursday and
Friday of Holy Week, Thanksgiving Day, Christmas Day, and Sundays are made legal holidays (sec.
29, Adm. Code) because of the secular idea that their observance is conducive to beneficial moral
results. The law allows divorce but punishes polygamy and bigamy; and certain crimes against
religious worship are considered crimes against the fundamental laws of the state xxx.  [Emphasis
22

supplied]

Thus, the right to believe or not to believe has again been enshrined in Section 5, Article III of the
1987 Constitution:

Section 5. xxx. The free exercise and enjoyment of religious profession and worship, without
discrimination or preference, shall forever be allowed. xxx.

Free Exercise Clause

Freedom of religion was accorded preferred status by the framers of our fundamental law. And this
Court has consistently affirmed this preferred status, well aware that it is "designed to protect the
broadest possible liberty of conscience, to allow each man to believe as his conscience directs, to
profess his beliefs, and to live as he believes he ought to live, consistent with the liberty of others
and with the common good." 23

"The right to religious profession and worship has a two-fold aspect - freedom to believe and
freedom to act on one's beliefs. The first is absolute as long as the belief is confined within the realm
of thought. The second is subject to regulation where the belief is translated into external acts that
affect the public welfare."  Justice Isagani A. Cruz explained these two (2) concepts in this wise:
24

(1) Freedom to Believe

The individual is free to believe (or disbelieve) as he pleases concerning the hereafter. He may
indulge his own theories about life and death; worship any god he chooses, or none at all; embrace
or reject any religion; acknowledge the divinity of God or of any being that appeals to his reverence;
recognize or deny the immortality of his soul - in fact, cherish any religious conviction as he and he
alone sees fit. However absurd his beliefs may be to others, even if they be hostile and heretical to
the majority, he has full freedom to believe as he pleases. He may not be required to prove his
beliefs. He may not be punished for his inability to do so. Religion, after all, is a matter of faith. "Men
may believe what they cannot prove." Every one has a right to his beliefs and he may not be called
to account because he cannot prove what he believes.

(2) Freedom to Act on One's Beliefs

But where the individual externalizes his beliefs in acts or omissions that affect the public, his
freedom to do so becomes subject to the authority of the State. As great as this liberty may be,
religious freedom, like all other rights guaranteed in the Constitution, can be enjoyed only with a
proper regard for the rights of others.

It is error to think that the mere invocation of religious freedom will stalemate the State and render it
impotent in protecting the general welfare. The inherent police power can be exercised to prevent
religious practices inimical to society. And this is true even if such practices are pursued out of
sincere religious conviction and not merely for the purpose of evading the reasonable requirements
or prohibitions of the law.

Justice Frankfurter put it succinctly: "The constitutional provision on religious freedom terminated
disabilities, it did not create new privileges. It gave religious liberty, not civil immunity. Its essence is
freedom from conformity to religious dogma, not freedom from conformity to law because of religious
dogma." 25

Allowing religion to flourish is not contrary to the principle of separation of Church and State. In fact,
these two principles are in perfect harmony with each other.

The State is aware of the existence of religious movements whose members believe in the divinity of
Jose Rizal. Yet, it does not implement measures to suppress the said religious sects. Such inaction
or indifference on the part of the State gives meaning to the separation of Church and State, and at
the same time, recognizes the religious freedom of the members of these sects to worship their own
Supreme Being.

As pointed out by Judge Lutero, "the Roman Catholics express their worship through the holy mass
and to stop these would be tantamount to repressing the right to the free exercise of their religion.
Our Muslim brethren, who are government employees, are allowed to worship their Allah even
during office hours inside their own offices. The Seventh Day Adventists are exempted from
rendering Saturday duty because their religion prohibits them from working on a Saturday. Even
Christians have been allowed to conduct their own bible studies in their own offices. All these have
been allowed in respect of the workers' right to the free exercise of their religion. xxx"
26

Clearly, allowing the citizens to practice their religion is not equivalent to a fusion of Church and
State.

No Compelling State Interest

Religious freedom, however, is not absolute. It cannot have its way if there is a compelling state
interest. To successfully invoke compelling state interest, it must be demonstrated that the masses
in the QC Hall of Justice unduly disrupt the delivery of public services or affect the judges and
employees in the performance of their official functions. In Estrada v. Escritor,  the Court expounded
27

on the test as follows:

The "compelling state interest" test is proper where conduct is involved for the whole gamut of
human conduct has different effects on the state's interests: some effects may be immediate and
short-term while others delayed and far-reaching. A test that would protect the interests of the state
in preventing a substantive evil, whether immediate or delayed, is therefore necessary. However, not
any interest of the state would suffice to prevail over the right to religious freedom as this is a
fundamental .right that enjoys a preferred position in the hierarchy of rights - "the most inalienable
and sacred of all human rights", in the words of Jefferson. This right is sacred for an invocation of the
Free Exercise Clause is an appeal to a higher sovereignty. The entire constitutional order of limited
government is premised upon an acknowledgment of such higher sovereignty, thus the Filipinos
implore the "aid of Almighty God in order to build a just and humane society and establish a
government." As held in Sherbert, only the gravest abuses, endangering paramount interests can
limit this fundamental right. A mere balancing of interests which balances a right with just a colorable
state interest is therefore not appropriate. Instead, only a compelling interest of the state can prevail
over the fundamental right to religious liberty. The test requires the state to carry a heavy burden, a
compelling one, for to do otherwise would allow the state to batter religion, especially the less
powerful ones until they are destroyed. In determining which shall prevail between the state's
interest and religious liberty, reasonableness shall be the guide. The "compelling state interest"
serves the purpose of revering religious liberty while at the same time affording protection to the
paramount interests of the state. This was the test used in Sherbert which involved conduct, i.e.
refusal to work on Saturdays. In the end, the "compelling state interest" test, by upholding the
paramount interests of the state, seeks to protect the very state, without which, religious liberty will
not be preserved.137 [Citations omitted] [Emphases supplied]

As reported by the Executive Judges of Quezon City, the masses were being conducted only during
noon breaks and were not disruptive of public services. The court proceedings were not being
distracted or interrupted and that the performance of the judiciary employees were not being
adversely affected. Moreover, no Civil Service rules were being violated. As there has been no
detrimental effect on the public service or prejudice to the State, there is simply no state interest
compelling enough to prohibit the exercise of religious freedom in the halls of justice.

In fact, the Civil Service Commission (CSC) was more lenient or tolerant. On November 13, 1981,
the CSC came out with Resolution No. 81-1277, which provided, among others, that "during Friday,
the Muslim pray day, Muslims are excused from work from 10:00 o'clock in the morning to 2:00
o'clock in the afternoon." The Court struck this down  as not sanctioned by the law. It wrote:
28

To allow the Muslim employees in the Judiciary to be excused from work from 10:00 a.m. to 2:00
p.m. every Friday (Muslim Prayer Day) during the entire calendar year would mean a diminution of
the prescribed government working hours. For then, they would be rendering service twelve (12)
hours less than that required by the civil service rules for each month. Further, this would encourage
other religious denominations to request for similar treatment.

The performance of religious practices, whether by the Muslim employees or those belonging to
other religious denominations, should not prejudice the courts and the public. Indeed, the exercise of
religious freedom does not exempt anyone from compliance with reasonable requirements of the
law, including civil service laws.

Accommodation, Not Establishment of Religion

In order to give life to the constitutional right of freedom of religion, the State adopts a policy of
accommodation. Accommodation is a recognition of the reality that some governmental measures
may not be imposed on a certain portion of the population for the reason that these measures are
contrary to their religious beliefs. As long as it can be shown that the exercise of the right does not
impair the public welfare, the attempt of the State to regulate or prohibit such right would be an
unconstitutional encroachment. 29

In Estrada v. Escritor,  the Court adopted a policy of benevolent neutrality:


30

With religion looked upon with benevolence and not hostility, benevolent neutrality allows
accommodation of religion under certain circumstances. Accommodations are government policies
that take religion specifically into account not to promote the government's favored form of religion,
but to allow individuals and groups to exercise their religion without hindrance. Their purpose or
effect therefore is to remove a burden on, or facilitate the exercise of, a person's or institution's
religion. As Justice Brennan explained, the "government [may] take religion into account ... to
exempt, when possible, from generally applicable governmental regulation individuals whose
religious beliefs and practices would otherwise thereby be infringed, or to create without state
involvement an atmosphere in which voluntary religious exercise may flourish." [Emphases supplied]

In Victoriano v. Elizalde Rope Workers Union,  the Court upheld the exemption of members
31

of Iglesia ni Cristo from the coverage of a closed shop agreement between their employer and a
union, because it would violate the teaching of their church not to affiliate with a labor organization.

In Ebralinag v. Division Superintendent of Schools of Cebu,  the petitioners, who were members of


32

the Jehovah 's Witnesses, refused to salute the flag, sing the national anthem, and recite the
patriotic pledge for it is their belief that those were acts of worship or religious devotion, which they
could not conscientiously give to anyone or anything except God. The Court accommodated them
and granted them an exemption from observing the flag ceremony out of respect for their religious
beliefs.

Further, several laws have been enacted to accommodate religion. The Revised Administrative
Code of 1987 has declared Maundy Thursday, Good Friday, and Christmas Day as regular holidays.
Republic Act (R.A.) No. 9177 proclaimed the FIRST Day of Shawwal, the tenth month of the Islamic
Calendar, a national holiday for the observance of Eidul Fitr (the end of Ramadan). R.A. No. 9849
declared the tenth day of Zhu/ Hijja, the twelfth month of the Islamic Calendar, a national holiday for
the observance of Eidul Adha. Presidential Decree (P.D.) No. 1083, otherwise known as the Code of
Muslim Personal Laws of the Philippines, expressly allows a Filipino Muslim to have more than one
(1) wife and exempts him from the crime of bigamy punishable under Revised Penal
Code (RPC). The same Code allows Muslims to have divorce. 33
As to Muslims in government offices, Section 3 of P.D. No. 291, as amended by P.D. No. 322,
provides:

Sec. 3. (a) During the fasting season on the month of Ramadan, all Muslim employees in the
national government, government-owned or controlled corporations, provinces, cities, municipalities
and other instrumentalities shall observe office hours from seven-thirty in the morning (7:30 a.m.) to
three-thirty in the afternoon (3:30 p.m.) without lunch break or coffee breaks, and that there shall be
no diminution of salary or wages, provided, that the employee who is not fasting is not entitled to the
benefit of this provision.

Pursuant thereto, the CSC promulgated Resolution No. 81-1277, dated November 13, 1981, which
reads in part:

2. During "Ramadan" the Fasting month (30 days) of the Muslims, the Civil Service official time of 8
o'clock to 12 o'clock and 1 o'clock to 5 o'clock is hereby modified to 7:30 AM. to 3:30 P.M. without
noon break and the difference of 2 hours is not counted as undertime.

Following the decree, in Re: Request of Muslim Employees in the Different Courts in Iligan City (Re:
Office Hours), the Court recognized that the observance of Ramadan as integral to the Islamic faith
34

and allowed Muslim employees in the Judiciary to hold flexible office hours from 7:30 o'clock in the
morning to 3:30 o'clock in the afternoon without any break during the period. This is a clear case of
accommodation because Section 5, Rule XVII of the Omnibus Rules Implementing Book V of E.0.
No. 292, enjoins all civil servants, of whatever religious denomination, to render public service of no
less than eight (8) hours a day or forty (40) hours a week.

Non-Establishment Clause

On the opposite side of the spectrum is the constitutional mandate that "no law shall be made
respecting an establishment of religion,"  otherwise known as the non-establishment clause. Indeed,
35

there is a thin line between accommodation and establishment, which makes it even more
imperative to understand each of these concepts by placing them in the Filipino society's
perspective.

The non-establishment clause reinforces the wall of separation between Church and State. It simply
means that the State cannot set up a Church; nor pass laws which aid one religion, aid all religion, or
prefer one religion over another nor force nor influence a person to go to or remain away from
church against his will or force him to profess a belief or disbelief in any religion; that the state
cannot punish a person for entertaining or professing religious beliefs or disbeliefs, for church
attendance or nonattendance; that no tax in any amount, large or small, can be levied to support any
religious activity or institution whatever they may be called or whatever form they may adopt or teach
or practice religion; that the state cannot openly or secretly participate in the affairs of any religious
organization or group and vice versa.  Its minimal sense is that the state cannot establish or sponsor
36

an official religion.
37

In the same breath that the establishment clause restricts what the government can do with religion,
it also limits what religious sects can or cannot do. They can neither cause the government to adopt
their particular doctrines as policy for everyone, nor can they cause the government to restrict other
groups. To do so, in simple terms, would cause the State to adhere to a particular religion and, thus,
establish a state religion.
38

Father Bernas further elaborated on this matter, as follows:


"In effect, what non-establishment calls for is government neutrality in religious matters. Such
government neutrality may be summarized in four general propositions: (1) Government must not
prefer one religion over another or religion over irreligion because such preference would violate
voluntarism and breed dissension; (2) Government funds must not be applied to religious purposes
because this too would violate voluntarism and breed interfaith dissension; (3) Government action
must not aid religion because this too can violate voluntarism and breed interfaith dissension; [and]
(4) Government action must not result in excessive entanglement with religion because this too can
violate voluntarism and breed interfaith dissension."39

Establishment entails a positive action on the part of the State. Accommodation, on the other hand,
is passive. In the former, the State becomes involved through the use of government resources with
the primary intention of setting up a state religion. In the latter, the State, without being entangled,
merely gives consideration to its citizens who want to freely exercise their religion.

In a September 12, 2003 Memorandum for Chief Justice Hilario G. Davide, Jr., the Office of the
Chief Attorney recommended to deny, on constitutional grounds, the request of Rev. Fr. Carlo M.
Ilagan to hold a oneday vigil in honor of the Our Lady of Caysasay within the premises of the Court.
Such controversy must be distinguished from the present issue in that with respect to the former, a
Catholic priest was the one who requested for the vigil. Moreover, in that case, the vigil would take
one (1) whole working day; whereas in this case, the masses are held at the initiative of Catholic
employees and only during the thirty-minute lunch break.

Guided by the foregoing, it is our considered view that the holding of Catholic masses at the
basement of the QC Hall of Justice is not a case of establishment, but merely accommodation.
First, there is no law, ordinance or circular issued by any duly constitutive authorities expressly
mandating that judiciary employees attend the Catholic masses at the basement. Second, when
judiciary employees attend the masses to profess their faith, it is at their own initiative as they are
there on their own free will and volition, without any coercion from the judges or administrative
officers. Third, no government funds are being spent because the lightings and airconditioning
continue to be operational even if there are no religious rituals there. Fourth, the basement has
neither been converted into a Roman Catholic chapel nor has it been permanently appropriated for
the exclusive use of its faithful. Fifth, the allowance of the masses has not prejudiced other religions.

No Appropriation of Public
Money or Property for the
Benefit of any Church

Section 29 (2), Article VI of the 1987 Constitution provides, "No public money or property shall be
appropriated, applied, paid, or employed, directly or indirectly, for the use, benefit, or support of any
sect, church, denomination, sectarian institution, or system of religion, or of any priest, preacher,
minister, or other religious teacher, or dignitary as such, except when such priest, preacher, minister,
or dignitary is assigned to the armed forces, or to any penal institution, or government orphanage or
leprosarium."

The word "apply" means "to use or employ for a particular purpose."  "Appropriate" means "to
40

prescribe a particular use for particular moneys or to designate or destine a fund or property for a
distinct use, or for the payment of a particular demand." 41

Under the principle of noscitur a sociis, where a particular word or phrase is ambiguous in itself or is
equally susceptible of various meanings, its correct construction may be made clear and specific by
considering the company of words in which it is found or with which it is associated. This is because
a word or phrase in a statute is always used in association with other words or phrases, and its
meaning may, thus, be modified or restricted by the latter. The particular words, clauses and
phrases should not be studied as detached and isolated expressions, but the whole and every part
of the statute must be considered in fixing the meaning of any of its parts and in order to produce a
harmonious whole. A statute must be so construed as to harmonize and give effect to all its
provisions whenever possible. 42

Thus, the words "pay" and "employ" should be understood to mean that what is prohibited is the use
of public money or property for the sole purpose of benefiting or supporting any church. The
prohibition contemplates a scenario where the appropriation is primarily intended for the furtherance
of a particular church.

It has also been held that the aforecited constitutional provision "does not inhibit the use of public
property for religious purposes when the religious character of such use is merely incidental to a
temporary use which is available indiscriminately to the public in general." Hence, a public street
may be used for a religious procession even as it is available for a civic parade, in the same way that
a public plaza is not barred to a religious rally if it may also be used for a political assemblage.
43

In relation thereto, the phrase "directly or indirectly" refers to the manner of appropriation of public
money or property, not as to whether a particular act involves a direct or a mere incidental benefit to
any church. Otherwise, the framers of the Constitution would have placed it before "use, benefit or
support" to describe the same. Even the exception to the same provision bolsters this interpretation.
The exception contemplates a situation wherein public funds are paid to a priest, preacher, minister,
or other religious teacher, or dignitary because they rendered service in the armed forces, or to any
penal institution, or government orphanage or leprosarium. That a priest belongs to a particular
church and the latter may have benefited from the money he received is of no moment, for the
purpose of the payment of public funds is merely to compensate the priest for services rendered and
for which other persons, who will perform the same services will also be compensated in the same
manner.

Ut magis valeat quam pereat. The Constitution is to be interpreted as a whole.  As such, the
44

foregoing interpretation finds support in the

Establishment Clause, which is as clear as daylight in stating that what is proscribed is the passage
of any law which tends to establish a religion, not merely to accommodate the free exercise thereof.

The Constitution even grants tax exemption to properties actually, directly and exclusively devoted to
religious purposes.  Certainly, this benefits the religious sects for a portion of what could have been
45

collected for the benefit of the public is surrendered in their favor.

In Manosca v. CA,  a parcel of land located in Taguig was determined by the National Historical
46

Institute to be the birthsite of Felix Y. Manalo, the founder of Iglesia ni Cristo. The Republic then
sought to expropriate the said property. The exercise of the power of eminent domain was
questioned on the ground that it would only benefit members of Iglesia ni Cristo. The Court upheld
the legality of the expropriation, viz.:

The practical reality that greater benefit may be derived by members of the Iglesia ni Cristo than by
most others could well be true but such a peculiar advantage still remains to be merely incidental
and secondary in nature.  [Emphasis supplied]
47

Again, in Aglipay, the issuing and selling of postage stamps commemorative of the Thirty-third
International Eucharistic Congress was assailed on the ground that it violated the constitutional
prohibition against the appropriation of public money or property for the benefit of any church. In
ruling that there was no such violation, the Court held:

It is obvious that while the issuance and sale of the stamps in question may be said to be
inseparably linked with an event of a religious character, the resulting propaganda, if any, received
by the Roman Catholic Church, was not the aim and purpose of the Government. We are of the
opinion that the Government should not be embarrassed in its activities simply because of incidental
results, more or less religious in character, if the purpose had in view is one which could legitimately
be undertaken by appropriate legislation. The main purpose should not be frustrated by its
subordination to mere incidental results not contemplated.  [Emphasis supplied]
48

Here, the basement of the QC Hall of Justice is not appropriated, applied or employed for the sole
purpose of supporting the Roman Catholics.

Further, it has not been converted into a Roman Catholic chapel for the exclusive use of its faithful
contrary to the claim of Valenciana. Judge

Maceren reported that the basement is also being used as a public waiting area for most of the day
and a meeting place for different employee organizations. The use of the area for holding masses is
limited to lunch break period from twelve (12) o'clock to one (1) o'clock in the afternoon. Further,
Judge Sagun, Jr. related that masses run for just a little over thirty (30) minutes. It is, therefore, clear
that no undue religious bias is being committed when the subject basement is allowed to be
temporarily used by the Catholics to celebrate mass, as the same area can be used by other groups
of people and for other purposes.  Thus, the basement of the QC Hall of Justice has remained to be
49

a public property devoted for public use because the holding of Catholic masses therein is a mere
incidental consequence of its primary purpose.

Conclusion

Directing the Executive Judges of the RTC and MeTC to regulate and closely monitor the holding of
masses and other religious practices within the courts does not promote excessive collaboration
between courts and various religions. On the contrary, this is necessary to ensure that there would
be no excessive entanglement.

To disallow the holding of religious rituals within halls of justice would set a dangerous precedent
and commence a domino effect. Strict separation, rather than benevolent neutrality/accommodation,
would be the norm. Thus, the establishment of Shari'a courts, the National Commission for Muslim
Filipinos, and the exception of Muslims from the provisions of the RPC relative to the crime of
bigamy would all be rendered nugatory because of strict separation. The exception of members
of Iglesia ni Cristo from joining a union or the non-compulsion recognized in favor of members of the
Jehovah's Witnesses from doing certain gestures during the flag ceremony, will all go down the drain
simply because we insist on strict separation.

That the holding of masses at the basement of the QC Hall of Justice may offend non-Catholics is no
reason to proscribe it. Our Constitution ensures and mandates an unconditional tolerance, without
regard to whether those who seek to profess their faith belong to the majority or to the minority. It is
emphatic in saying that "the free exercise and enjoyment of religious profession and worship shall be
without discrimination or preference." Otherwise, accommodation or tolerance would just be mere lip
service.
One cannot espouse that the constitutional freedom of religion ensures tolerance, but, in reality,
refuses to practice what he preaches. One cannot ask for tolerance when he refuses to do the same
for others.

In fine, the Court denies the plea that the holding of Catholic masses at the basement of the QC Hall
of Justice be prohibited because the said practice does not violate the constitutional principle of
separation of Church and State and the constitutional prohibition against appropriation of public
money or property for the benefit of a sect, church, denomination, or any other system of religion.

WHEREFORE, the Court resolves to:

1. NOTE the letter-complaints of Mr. Tony Q. Valenciano, dated January 6, 2009, May 13, 2009, and
March 23, 2010;

2. NOTE the 1st Indorsement, dated September 21, 2010, by the Office on Halls of Justice,
containing photocopies and certified photocopies of previous actions made relative to the complaint;

3. NOTE the Letter-Comment, dated September 9, 2010, of Quezon City Regional Trial Court
Executive Judge Fernando T. Sagun, Jr.;

4. NOTE the undated Letter-Comment of Quezon City Metropolitan Trial Court Executive Judge
Caridad M. Walse-Lutero;

5. DENY the prayer of Tony Q. Valenciano to prohibit the holding of religious rituals in the QC Hall of
Justice and in all halls of justice in the country; and

6. DIRECT the Executive Judges of Quezon City to REGULATE and CLOSELY MONITOR the


holding of masses and other religious practices within the Quezon City Hall of Justice by ensuring,
among others, that:

(a) it does not disturb or interrupt court proceedings;

(b) it does not adversely affect and interrupt the delivery of public service; and

(c) it does not unduly inconvenience the public.

In no case shall a particular part of a public building be a permanent place for worship for the benefit
of any and all religious groups. There shall also be no permanent display of religious icons in all halls
of justice in the country. In case of religious rituals, religious icons and images may be displayed but
their presentation is limited only during the celebration of such activities so as not to offend the
sensibilities of members of other religious denominations or the non-religious public. After any
religious affair, the icons and images shall be hidden or concealed from public view.

The disposition in this administrative matter shall apply to all halls of justice in the country. Other
churches, religious denominations or sects are entitled to the same rights, privileges, and practices
in every hall of justice. In other buildings not owned or controlled by the Judiciary, the Executive
Judges should coordinate and seek approval of the building owners/administrators accommodating
their courts.

SO ORDERED.
G.R. No. 204819               April 8, 2014

JAMES M. IMBONG and LOVELY-ANN C. IMBONG, for themselves and in behalf of their minor
children, LUCIA CARLOS IMBONG and BERNADETTE CARLOS IMBONG and MAGNIFICAT
CHILD DEVELOPMENT CENTER, INC., Petitioners, 
vs.
HON. PAQUITO N. OCHOA, JR., Executive Secretary, HON. FLORENCIO B. ABAD, Secretary,
Department of Budget and Management, HON. ENRIQUE T. ONA, Secretary, Department of
Health, HON. ARMIN A. LUISTRO, Secretary, Department of Education, Culture and Sports
and HON. MANUELA. ROXAS II, Secretary, Department of Interior and Local
Government, Respondents.

x---------------------------------x

G.R. No. 204934

ALLIANCE FOR THE FAMILY FOUNDATION PHILIPPINES, INC. [ALFI], represented by its
President, Maria Concepcion S. Noche, Spouses Reynaldo S. Luistro & Rosie B . Luistro,
Jose S. Sandejas & Elenita S.A. Sandejas, Arturo M. Gorrez & Marietta C. Gorrez, Salvador S.
Mante, Jr. & Hazeleen L. Mante, Rolando M. Bautista & Maria Felisa S. Bautista, Desiderio
Racho & Traquilina Racho, F emand Antonio A. Tansingco & Carol Anne C. Tansingco for
themselves and on behalf of their minor children, Therese Antonette C. Tansingco, Lorenzo
Jose C. Tansingco, Miguel F emando C. Tangsingco, Carlo Josemaria C. Tansingco & Juan
Paolo C. Tansingco, Spouses Mariano V. Araneta & Eileen Z. Araneta for themselves and on
behalf of their minor children, Ramon Carlos Z. Araneta & Maya Angelica Z. Araneta, Spouses
Renato C. Castor & Mildred C. Castor for themselves and on behalf of their minor children,
Renz Jeffrey C. Castor, Joseph Ramil C. Castor, John Paul C. Castor & Raphael C. Castor,
Spouses Alexander R. Racho & Zara Z. Racho for themselves and on behalf of their minor
children Margarita Racho, Mikaela Racho, Martin Racho, Mari Racho & Manolo Racho,
Spouses Alfred R. Racho & Francine V. Racho for themselves and on behalf of their minor
children Michael Racho, Mariana Racho, Rafael Racho, Maxi Racho, Chessie Racho & Laura
Racho, Spouses David R. Racho & Armilyn A. Racho for themselves and on behalf of their
minor child Gabriel Racho, Mindy M. Juatas and on behalf of her minor children Elijah Gerald
Juatas and Elian Gabriel Juatas, Salvacion M. Monteiro, Emily R. Laws, Joseph R . Laws &
Katrina R. Laws, Petitioners, 
vs.
HON. PAQUITO N. OCHOA, JR., Executive Secretary, HON. ENRIQUE T. ONA, Secretary,
Department of Health, HON. ARMIN A. LUISTRO, Secretary, Department of Education, Culture
and Sports, HON. CORAZON SOLIMAN, Secretary, Department of Social Welfare and
Development, HON. MANUELA. ROXAS II, Secretary, Department of Interior and Local
Government, HON. FLORENCIO B. ABAD, Secretary, Department of Budget and Management,
HON. ARSENIO M. BALISACAN, Socio-Economic Planning Secretary and NEDA Director-
General, THE PHILIPPINE COMMISSION ON WOMEN, represented by its Chairperson,
Remedios lgnacio-Rikken, THE PHILIPPINE HEALTH INSURANCE CORPORATION,
represented by its President Eduardo Banzon, THE LEAGUE OF PROVINCES OF THE
PHILIPPINES, represented by its President Alfonso Umali, THE LEAGUE OF CITIES OF THE
PHILIPPINES, represented by its President Oscar Rodriguez, and THE LEAGUE OF
MUNICIPALITIES OF THE PHILIPPINES, represented by its President Donato
Marcos,Respondents.

x---------------------------------x
G.R. No. 204957

TASK FORCE FOR FAMILY AND LIFE VISAYAS, INC. and VALERIANO S. AVILA, Petitioners, 
vs.
HON. PAQUITO N. OCHOA, JR., Executive Secretary; HON. FLORENCIO B. ABAD, Secretary,
Department of Budget and Management; HON. ENRIQUE T. ONA, Secretary, Department of
Education; and HON. MANUELA. ROXAS II, Secretary, Department of Interior and Local
Government, Respondents.

x---------------------------------x

G.R. No. 204988

SERVE LIFE CAGAYAN DE ORO CITY, INC., represented by Dr. Nestor B. Lumicao, M.D., as
President and in his personal capacity, ROSEVALE FOUNDATION INC., represented by Dr.
Rodrigo M. Alenton, M.D., as member of the school board and in his personal capacity,
ROSEMARIE R. ALENTON, IMELDA G. IBARRA, CPA, LOVENIAP. NACES, Phd., ANTHONY G.
NAGAC, EARL ANTHONY C. GAMBE and MARLON I. YAP,Petitioners, 
vs.
OFFICE OF THE PRESIDENT, SENATE OF THE PHILIPPINES, HOUSE OF
REPRESENTATIVES, HON. PAQUITO N. OCHOA, JR., Executive Secretary, HON. FLORENCIO
B. ABAD, Secretary, Department of Budget and Management; HON. ENRIQUE T. ONA,
Secretary, Department of Health; HON. ARMIN A. LUISTRO, Secretary, Department of
Education and HON. MANUELA. ROXAS II, Secretary, Department of Interior and Local
Government, Respondents.

x---------------------------------x

G.R. No. 205003

EXPEDITO A. BUGARIN, JR., Petitioner, 


vs.
OFFICE OF THE PRESIDENT OF THE REPUBLIC OF THE PHILIPPINES, HON. SENATE
PRESIDENT, HON. SPEAKER OF THE HOUSE OF REPRESENTATIVES and HON. SOLICITOR
GENERAL, Respondents.

x---------------------------------x

G.R. No. 205043

EDUARDO B. OLAGUER and THE CATHOLIC XYBRSPACE APOSTOLATE OF THE


PHILIPPINES, Petitioners, 
vs.
DOH SECRETARY ENRIQUE T. ONA, FDA DIRECTOR SUZETTE H. LAZO, DBM SECRETARY
FLORENCIO B. ABAD, DILG SECRETARY MANUELA. ROXAS II, DECS SECRETARY ARMIN A.
LUISTRO, Respondents.

x---------------------------------x

G.R. No. 205138


PHILIPPINE ALLIANCE OF XSEMINARIANS, INC. (PAX), herein represented by its National
President, Atty. Ricardo M . Ribo, and in his own behalf, Atty. Lino E.A. Dumas, Romeo B.
Almonte, Osmundo C. Orlanes, Arsenio Z. Menor, Samuel J. Yap, Jaime F. Mateo, Rolly
Siguan, Dante E. Magdangal, Michael Eugenio O. Plana, Bienvenido C. Miguel, Jr., Landrito M.
Diokno and Baldomero Falcone, Petitioners, 
vs.
HON. PAQUITO N. OCHOA, JR., Executive Secretary, HON. FLORENCIO B. ABAD, Secretary,
Department of Budget and Management, HON. ENRIQUE T. ONA, Secretary, Department of
Health, HON. ARMIN A. LUISTRO, Secretary, Department of Education, HON. MANUELA.
ROXAS II, Secretary, Department of Interior and Local Government, HON. CORAZON J.
SOLIMAN, Secretary, Department of Social Welfare and Development, HON. ARSENIO
BALISACAN, Director-General, National Economic and Development Authority, HON.
SUZETTE H. LAZO, Director-General, Food and Drugs Administration, THE BOARD OF
DIRECTORS, Philippine Health Insurance Corporation, and THE BOARD OF
COMMISSIONERS, Philippine Commission on Women, Respondents.

x---------------------------------x

G.R. No. 205478

REYNALDO J. ECHAVEZ, M.D., JACQUELINE H. KING, M.D., CYNTHIA T. DOMINGO, M.D.,


AND JOSEPHINE MILLADO-LUMITAO, M.D., collectively known as Doctors For Life, and
ANTHONY PEREZ, MICHAEL ANTHONY G. MAPA, CARLOS ANTONIO PALAD, WILFREDO
JOSE, CLAIRE NAVARRO, ANNA COSIO, and GABRIEL DY LIACCO collectively known as
Filipinos For Life, Petitioners, 
vs.
HON. PAQUITO N. OCHOA, JR., Executive Secretary; HON. FLORENCIO B. ABAD, Secretary
of the Department of Budget and Management; HON. ENRIQUE T. ONA, Secretary of the
Department of Health; HON. ARMIN A. LUISTRO, Secretary of the Department of Education;
and HON. MANUELA. ROXAS II, Secretary of the Department of Interior and Local
Government, Respondents.

x---------------------------------x

G.R. No. 205491

SPOUSES FRANCISCO S. TATAD AND MARIA FENNY C. TATAD & ALA F. PAGUIA, for
themselves, their Posterity, and the rest of Filipino posterity, Petitioners, 
vs.
OFFICE OF THE PRESIDENT of the Republic of the Philippines, Respondent.

x---------------------------------x

G.R. No. 205720

PRO-LIFE PHILIPPINES FOUNDATION, Inc., represented by Loma Melegrito, as Executive


Director, and in her personal capacity, JOSELYN B. BASILIO, ROBERT Z. CORTES, ARIEL A.
CRISOSTOMO, JEREMY I. GATDULA, CRISTINA A. MONTES, RAUL ANTONIO A. NIDOY,
WINSTON CONRAD B. PADOJINOG, RUFINO L. POLICARPIO III, Petitioners, 
vs.
OFFICE OF THE PRESIDENT, SENATE OF THE PHILIPPINES, HOUSE OF
REPRESENTATIVES, HON. PAQUITO N. OCHOA, JR., Executive Secretary, HON. FLORENCIO
B. ABAD, Secretary, Department of Budget and Management, HON. ENRIQUE T. ONA,
Secretary, Department of Health, HON. ARMIN A. LUISTRO, Secretary, Department of
Education and HON. MANUEL A. ROXAS II, Secretary, Department of Interior and Local
Government, Respondents.

x---------------------------------x

G.R. No. 206355

MILLENNIUM SAINT FOUNDATION, INC., ATTY. RAMON PEDROSA, ATTY. CITA BORROMEO-
GARCIA, STELLAACEDERA, ATTY. BERTENI CATALUNA CAUSING, Petitioners, 
vs.
OFFICE OF THE PRESIDENT, OFFICE OF THE EXECUTIVE SECRETARY, DEPARTMENT OF
HEALTH, DEPARTMENT OF EDUCATION, Respondents.

x---------------------------------x

G.R. No. 207111

JOHN WALTER B. JUAT, MARY M. IMBONG, ANTHONY VICTORIO B. LUMICAO, JOSEPH


MARTIN Q. VERDEJO, ANTONIA EMMA R. ROXAS and LOTA LAT-GUERRERO, Petitioners, 
vs.
HON. PAQUITO N. OCHOA, JR., Executive Secretary, HON. FLORENCIO ABAD, Secretary,
Department of Budget and Management, HON. ENRIQUE T. ONA, Secretary, Department of
Health, HON. ARMIN A. LUISTRO, Secretary, Department of Education, Culture and Sports
and HON. MANUEL A. ROXAS II, Secretary, Department of Interior and Local
Government, Respondents.

x---------------------------------x

G.R. No. 207172

COUPLES FOR CHRIST FOUNDATION, INC., SPOUSES JUAN CARLOS ARTADI SARMIENTO
AND FRANCESCA ISABELLE BESINGA-SARMIENTO, AND SPOUSES LUIS FRANCIS A.
RODRIGO, JR. and DEBORAH MARIE VERONICA N. RODRIGO, Petitioners, 
vs.
HON. PAQUITO N. OCHOA, JR., Executive Secretary, HON. FLORENCIO B. ABAD, Secretary,
Department of Budget and Management, HON. ENRIQUE T. ONA, Secretary, Department of
Health, HON. ARMIN A. LUISTRO, Secretary, Department of Education, Culture and Sports
and HON. MANUELA. ROXAS II, Secretary, Department of Interior and Local
Government, Respondents.

x---------------------------------x

G.R. No. 207563

ALMARIM CENTI TILLAH and ABDULHUSSEIN M. KASHIM, Petitioners, 


vs.
HON. PAQUITO N. OCHOA, JR., Executive Secretary, HON. ENRIQUE T. ONA, Secretary of the
Department of Health, and HON. ARMIN A. LUISTRO,Secretary of the Department of Budget
and Management,Respondents.
DECISION

MENDOZA, J.:

Freedom of religion was accorded preferred status by the framers of our fundamental law. And this
Court has consistently affirmed this preferred status, well aware that it is "designed to protect the
broadest possible liberty of conscience, to allow each man to believe as his conscience directs, to
profess his beliefs , and to live as he believes he ought to live, consistent with the liberty of others
and with the common good." 1

To this day, poverty is still a major stumbling block to the nation's emergence as a developed
country, leaving our people beleaguered in a state of hunger, illiteracy and unemployment. While
governmental policies have been geared towards the revitalization of the economy, the bludgeoning
dearth in social services remains to be a problem that concerns not only the poor, but every member
of society. The government continues to tread on a trying path to the realization of its very purpose,
that is, the general welfare of the Filipino people and the development of the country as a whole. The
legislative branch, as the main facet of a representative government, endeavors to enact laws and
policies that aim to remedy looming societal woes, while the executive is closed set to fully
implement these measures and bring concrete and substantial solutions within the reach of Juan
dela Cruz. Seemingly distant is the judicial branch, oftentimes regarded as an inert governmental
body that merely casts its watchful eyes on clashing stakeholders until it is called upon to adjudicate.
Passive, yet reflexive when called into action, the Judiciary then willingly embarks on its solemn duty
to interpret legislation vis-a-vis the most vital and enduring principle that holds Philippine society
together - the supremacy of the Philippine Constitution.

Nothing has polarized the nation more in recent years than the issues of population growth control,
abortion and contraception. As in every democratic society, diametrically opposed views on the
subjects and their perceived consequences freely circulate in various media. From television
debates  to sticker campaigns,  from rallies by socio-political activists to mass gatherings organized
2 3

by members of the clergy  - the clash between the seemingly antithetical ideologies of the religious
4

conservatives and progressive liberals has caused a deep division in every level of the society.
Despite calls to withhold support thereto, however, Republic Act (R.A.) No. 10354, otherwise known
as the Responsible Parenthood and Reproductive Health Act of 2012 (RH Law), was enacted by
Congress on December 21, 2012.

Shortly after the President placed his imprimatur on the said law, challengers from various sectors of
society came knocking on the doors of the Court, beckoning it to wield the sword that strikes down
constitutional disobedience. Aware of the profound and lasting impact that its decision may produce,
the Court now faces the iuris controversy, as presented in fourteen (14) petitions and two (2)
petitions- in-intervention, to wit:

(1) Petition for Certiorari and Prohibition,  filed by spouses Attys. James M. Imbong and
5

Lovely Ann C. Imbong, in their personal capacities as citizens, lawyers and taxpayers and on
behalf of their minor children; and the Magnificat Child Leaming Center, Inc., a domestic,
privately-owned educational institution (Jmbong);

(2) Petition for Prohibition,  filed by the Alliance for the Family Foundation Philippines, Inc.,
6

through its president, Atty. Maria Concepcion S. Noche  and several others  in their personal
7 8

capacities as citizens and on behalf of the generations unborn (ALFI);

(3) Petition for Certiorari,  filed by the Task Force for Family and Life Visayas, Inc., and
9

Valeriano S. Avila, in their capacities as citizens and taxpayers (Task Force Family);
(4) Petition for Certiorari and Prohibition,  filed by Serve Life Cagayan De Oro City,
10

Inc.,  Rosevale Foundation, Inc.,  a domestic, privately-owned educational institution, and


11 12

several others,  in their capacities as citizens (Serve Life);


13

(5) Petition,  filed by Expedito A. Bugarin, Jr. in his capacity as a citizen (Bugarin);
14

(6) Petition for Certiorari and Prohibition,  filed by Eduardo Olaguer and the Catholic
15

Xybrspace Apostolate of the Philippines,  in their capacities as a citizens and taxpayers
16

(Olaguer);

(7) Petition for Certiorari and Prohibition,  filed by the Philippine Alliance of Xseminarians
17

Inc.,  and several others  in their capacities as citizens and taxpayers (PAX);
18 19

(8) Petition,  filed by Reynaldo J. Echavez, M.D. and several others,  in their capacities as
20 21

citizens and taxpayers (Echavez);

(9) Petition for Certiorari and Prohibition,  filed by spouses Francisco and Maria Fenny C.
22

Tatad and Atty. Alan F. Paguia, in their capacities as citizens, taxpayers and on behalf of
those yet unborn. Atty. Alan F. Paguia is also proceeding in his capacity as a member of the
Bar (Tatad);

(10) Petition for Certiorari and Prohibition,  filed by Pro-Life Philippines Foundation Inc.  and
23 24

several others,  in their capacities as citizens and taxpayers and on behalf of its associates
25

who are members of the Bar (Pro-Life);

(11) Petition for Prohibition,  filed by Millennium Saint Foundation, Inc.,  Attys. Ramon
26 27

Pedrosa, Cita Borromeo-Garcia, Stella Acedera, and Berteni Catalufia Causing, in their
capacities as citizens, taxpayers and members of the Bar (MSF);

(12) Petition for Certiorari and Prohibition,  filed by John Walter B. Juat and several
28

others,  in their capacities as citizens (Juat) ;


29

(13) Petition for Certiorari and Prohibition,  filed by Couples for Christ Foundation, Inc. and
30

several others, in their capacities as citizens (CFC);


31

(14) Petition for Prohibition  filed by Almarim Centi Tillah and Abdulhussein M. Kashim in
32

their capacities as citizens and taxpayers (Tillah); and

(15) Petition-In-Intervention,  filed by Atty. Samson S. Alcantara in his capacity as a citizen


33

and a taxpayer (Alcantara); and

(16) Petition-In-Intervention,  filed by Buhay Hayaang Yumabong (B UHAY) , an accredited


34

political party.

A perusal of the foregoing petitions shows that the petitioners are assailing the constitutionality of
RH Law on the following GROUNDS:

• The RH Law violates the right to life of the unborn. According to the petitioners,
notwithstanding its declared policy against abortion, the implementation of the RH Law would
authorize the purchase of hormonal contraceptives, intra-uterine devices and injectables
which are abortives, in violation of Section 12, Article II of the Constitution which guarantees
protection of both the life of the mother and the life of the unborn from conception.
35

• The RH Law violates the right to health and the right to protection against hazardous
products. The petitioners posit that the RH Law provides universal access to contraceptives
which are hazardous to one's health, as it causes cancer and other health problems. 36

• The RH Law violates the right to religious freedom. The petitioners contend that the RH
Law violates the constitutional guarantee respecting religion as it authorizes the use of public
funds for the procurement of contraceptives. For the petitioners, the use of public funds for
purposes that are believed to be contrary to their beliefs is included in the constitutional
mandate ensuring religious freedom. 37

It is also contended that the RH Law threatens conscientious objectors of criminal prosecution,
imprisonment and other forms of punishment, as it compels medical practitioners 1] to refer patients
who seek advice on reproductive health programs to other doctors; and 2] to provide full and correct
information on reproductive health programs and service, although it is against their religious beliefs
and convictions. 38

In this connection, Section 5 .23 of the Implementing Rules and Regulations of the RH Law (RH-
IRR),  provides that skilled health professionals who are public officers such as, but not limited to,
39

Provincial, City, or Municipal Health Officers, medical officers, medical specialists, rural health
physicians, hospital staff nurses, public health nurses, or rural health midwives, who are specifically
charged with the duty to implement these Rules, cannot be considered as conscientious objectors. 40

It is also argued that the RH Law providing for the formulation of mandatory sex education in schools
should not be allowed as it is an affront to their religious beliefs.
41

While the petit10ners recognize that the guarantee of religious freedom is not absolute, they argue
that the RH Law fails to satisfy the "clear and present danger test" and the "compelling state interest
test" to justify the regulation of the right to free exercise of religion and the right to free speech. 42

• The RH Law violates the constitutional provision on involuntary servitude. According to the
petitioners, the RH Law subjects medical practitioners to involuntary servitude because, to
be accredited under the PhilHealth program, they are compelled to provide forty-eight (48)
hours of pro bona services for indigent women, under threat of criminal prosecution,
imprisonment and other forms of punishment. 43

The petitioners explain that since a majority of patients are covered by PhilHealth, a medical
practitioner would effectively be forced to render reproductive health services since the lack of
PhilHealth accreditation would mean that the majority of the public would no longer be able to avail
of the practitioners services. 44

• The RH Law violates the right to equal protection of the law. It is claimed that the RH Law
discriminates against the poor as it makes them the primary target of the government
program that promotes contraceptive use. The petitioners argue that, rather than promoting
reproductive health among the poor, the RH Law seeks to introduce contraceptives that
would effectively reduce the number of the poor. 45

• The RH Law is "void-for-vagueness" in violation of the due process clause of the


Constitution. In imposing the penalty of imprisonment and/or fine for "any violation," it is
vague because it does not define the type of conduct to be treated as "violation" of the RH
Law. 46

In this connection, it is claimed that "Section 7 of the RH Law violates the right to due process by
removing from them (the people) the right to manage their own affairs and to decide what kind of
health facility they shall be and what kind of services they shall offer."  It ignores the management
47

prerogative inherent in corporations for employers to conduct their affairs in accordance with their
own discretion and judgment.

• The RH Law violates the right to free speech. To compel a person to explain a full range of
family planning methods is plainly to curtail his right to expound only his own preferred way
of family planning. The petitioners note that although exemption is granted to institutions
owned and operated by religious groups, they are still forced to refer their patients to another
healthcare facility willing to perform the service or procedure.48

• The RH Law intrudes into the zone of privacy of one's family protected by the Constitution.
It is contended that the RH Law providing for mandatory reproductive health education
intrudes upon their constitutional right to raise their children in accordance with their beliefs.
49

It is claimed that, by giving absolute authority to the person who will undergo reproductive health
procedure, the RH Law forsakes any real dialogue between the spouses and impedes the right of
spouses to mutually decide on matters pertaining to the overall well-being of their family. In the same
breath, it is also claimed that the parents of a child who has suffered a miscarriage are deprived of
parental authority to determine whether their child should use contraceptives. 50

• The RH Law violates the constitutional principle of non-delegation of legislative authority.


The petitioners question the delegation by Congress to the FDA of the power to determine
whether a product is non-abortifacient and to be included in the Emergency Drugs List
(EDL). 51

• The RH Law violates the one subject/one bill rule provision under Section 26( 1 ), Article VI
of the Constitution.52

• The RH Law violates Natural Law. 53

• The RH Law violates the principle of Autonomy of Local Government Units (LGUs) and the
Autonomous Region of Muslim Mindanao {ARMM). It is contended that the RH Law,
providing for reproductive health measures at the local government level and the ARMM,
infringes upon the powers devolved to LGUs and the ARMM under the Local Government
Code and R.A . No. 9054. 54

Various parties also sought and were granted leave to file their respective comments-in-intervention
in defense of the constitutionality of the RH Law. Aside from the Office of the Solicitor General
(OSG) which commented on the petitions in behalf of the respondents,  Congressman Edcel C.
55

Lagman,  former officials of the Department of Health Dr. Esperanza I. Cabral, Jamie Galvez-Tan,
56

and Dr. Alberto G. Romualdez,  the Filipino Catholic Voices for Reproductive Health (C4RH),  Ana
57 58

Theresa "Risa" Hontiveros,  and Atty. Joan De Venecia  also filed their respective Comments-in-
59 60

Intervention in conjunction with several others. On June 4, 2013, Senator Pia Juliana S. Cayetano
was also granted leave to intervene. 61
The respondents, aside from traversing the substantive arguments of the petitioners, pray for the
dismissal of the petitions for the principal reasons that 1] there is no actual case or controversy and,
therefore, the issues are not yet ripe for judicial determination.; 2] some petitioners lack standing to
question the RH Law; and 3] the petitions are essentially petitions for declaratory relief over which
the Court has no original jurisdiction.

Meanwhile, on March 15, 2013, the RH-IRR for the enforcement of the assailed legislation took
effect.

On March 19, 2013, after considering the issues and arguments raised, the Court issued the Status
Quo Ante Order (SQAO), enjoining the effects and implementation of the assailed legislation for a
period of one hundred and twenty (120) days, or until July 17, 2013. 62

On May 30, 2013, the Court held a preliminary conference with the counsels of the parties to
determine and/or identify the pertinent issues raised by the parties and the sequence by which these
issues were to be discussed in the oral arguments. On July 9 and 23, 2013, and on August 6, 13,
and 27, 2013, the cases were heard on oral argument. On July 16, 2013, the SQAO was ordered
extended until further orders of the Court.
63

Thereafter, the Court directed the parties to submit their respective memoranda within sixty (60)
days and, at the same time posed several questions for their clarification on some contentions of the
parties.
64

The Status Quo Ante

(Population, Contraceptive and Reproductive Health Laws

Prior to the RH Law

Long before the incipience of the RH Law, the country has allowed the sale, dispensation and
distribution of contraceptive drugs and devices. As far back as June 18, 1966, the country enacted
R.A. No. 4729 entitled "An Act to Regu,late the Sale, Dispensation, and/or Distribution of
Contraceptive Drugs and Devices." Although contraceptive drugs and devices were allowed, they
could not be sold, dispensed or distributed "unless such sale, dispensation and distribution is by a
duly licensed drug store or pharmaceutical company and with the prescription of a qualified medical
practitioner."
65

In addition, R.A. No. 5921,  approved on June 21, 1969, contained provisions relative to "dispensing
66

of abortifacients or anti-conceptional substances and devices." Under Section 37 thereof, it was


provided that "no drug or chemical product or device capable of provoking abortion or preventing
conception as classified by the Food and Drug Administration shall be delivered or sold to any
person without a proper prescription by a duly licensed physician."

On December 11, 1967, the Philippines, adhering to the UN Declaration on Population, which
recognized that the population problem should be considered as the principal element for long-term
economic development, enacted measures that promoted male vasectomy and tubal ligation to
mitigate population growth.  Among these measures included R.A. No. 6365, approved on August
67

16, 1971, entitled "An Act Establishing a National Policy on Population, Creating the Commission on
Population and for Other Purposes. " The law envisioned that "family planning will be made part of a
broad educational program; safe and effective means will be provided to couples desiring to space
or limit family size; mortality and morbidity rates will be further reduced."
To further strengthen R.A. No. 6365, then President Ferdinand E . Marcos issued Presidential
Decree. (P.D.) No. 79,  dated December 8, 1972, which, among others, made "family planning a
68

part of a broad educational program," provided "family planning services as a part of over-all health
care," and made "available all acceptable methods of contraception, except abortion, to all Filipino
citizens desirous of spacing, limiting or preventing pregnancies."

Through the years, however, the use of contraceptives and family planning methods evolved from
being a component of demographic management, to one centered on the promotion of public health,
particularly, reproductive health.  Under that policy, the country gave priority to one's right to freely
69

choose the method of family planning to be adopted, in conformity with its adherence to the
commitments made in the International Conference on Population and Development.  Thus, on 70

August 14, 2009, the country enacted R.A. No. 9710 or "The Magna Carta for Women, " which,
among others, mandated the State to provide for comprehensive health services and programs for
women, including family planning and sex education. 71

The RH Law

Despite the foregoing legislative measures, the population of the country kept on galloping at an
uncontrollable pace. From a paltry number of just over 27 million Filipinos in 1960, the population of
the country reached over 76 million in the year 2000 and over 92 million in 2010.  The executive and
72

the legislative, thus, felt that the measures were still not adequate. To rein in the problem, the RH
Law was enacted to provide Filipinos, especially the poor and the marginalized, access and
information to the full range of modem family planning methods, and to ensure that its objective to
provide for the peoples' right to reproductive health be achieved. To make it more effective, the RH
Law made it mandatory for health providers to provide information on the full range of modem family
planning methods, supplies and services, and for schools to provide reproductive health education.
To put teeth to it, the RH Law criminalizes certain acts of refusals to carry out its mandates.

Stated differently, the RH Law is an enhancement measure to fortify and make effective the current
laws on contraception, women's health and population control.

Prayer of the Petitioners - Maintain the Status Quo

The petitioners are one in praying that the entire RH Law be declared unconstitutional. Petitioner
ALFI, in particular, argues that the government sponsored contraception program, the very essence
of the RH Law, violates the right to health of women and the sanctity of life, which the State is
mandated to protect and promote. Thus, ALFI prays that "the status quo ante - the situation prior to
the passage of the RH Law - must be maintained."  It explains:
73

x x x. The instant Petition does not question contraception and contraceptives per se. As provided
under Republic Act No. 5921 and Republic Act No. 4729, the sale and distribution of contraceptives
are prohibited unless dispensed by a prescription duly licensed by a physician. What the Petitioners
find deplorable and repugnant under the RH Law is the role that the State and its agencies - the
entire bureaucracy, from the cabinet secretaries down to the barangay officials in the remotest areas
of the country - is made to play in the implementation of the contraception program to the fullest
extent possible using taxpayers' money. The State then will be the funder and provider of all forms of
family planning methods and the implementer of the program by ensuring the widespread
dissemination of, and universal access to, a full range of family planning methods, devices and
supplies.74

ISSUES
After a scrutiny of the various arguments and contentions of the parties, the Court has synthesized
and refined them to the following principal issues:

I. PROCEDURAL: Whether the Court may exercise its power of judicial review over the controversy.

1] Power of Judicial Review

2] Actual Case or Controversy

3] Facial Challenge

4] Locus Standi

5] Declaratory Relief

6] One Subject/One Title Rule

II. SUBSTANTIVE: Whether the RH law is unconstitutional:

1] Right to Life

2] Right to Health

3] Freedom of Religion and the Right to Free Speech

4] The Family

5] Freedom of Expression and Academic Freedom

6] Due Process

7] Equal Protection

8] Involuntary Servitude

9] Delegation of Authority to the FDA

10] Autonomy of Local Govemments/ARMM

DISCUSSION

Before delving into the constitutionality of the RH Law and its implementing rules, it behooves the
Court to resolve some procedural impediments.

I. PROCEDURAL ISSUE: Whether the Court can exercise its power of judicial review over the
controversy.

The Power of Judicial Review


In its attempt to persuade the Court to stay its judicial hand, the OSG asserts that it should submit to
the legislative and political wisdom of Congress and respect the compromises made in the crafting of
the RH Law, it being "a product of a majoritarian democratic process"  and "characterized by an
75

inordinate amount of transparency."  The OSG posits that the authority of the Court to review social
76

legislation like the RH Law by certiorari is "weak," since the Constitution vests the discretion to
implement the constitutional policies and positive norms with the political departments, in particular,
with Congress.  It further asserts that in view of the Court's ruling in Southern Hemisphere v. Anti-
77

Terrorism Council,  the remedies of certiorari and prohibition utilized by the petitioners are improper
78

to assail the validity of the acts of the legislature.


79

Moreover, the OSG submits that as an "as applied challenge," it cannot prosper considering that the
assailed law has yet to be enforced and applied to the petitioners, and that the government has yet
to distribute reproductive health devices that are abortive. It claims that the RH Law cannot be
challenged "on its face" as it is not a speech-regulating measure. 80

In many cases involving the determination of the constitutionality of the actions of the Executive and
the Legislature, it is often sought that the Court temper its exercise of judicial power and accord due
respect to the wisdom of its co-equal branch on the basis of the principle of separation of powers. To
be clear, the separation of powers is a fundamental principle in our system of government, which
obtains not through express provision but by actual division in our Constitution. Each department of
the government has exclusive cognizance of matters within its jurisdiction and is supreme within its
own sphere. 81

Thus, the 1987 Constitution provides that: (a) the legislative power shall be vested in the Congress
of the Philippines;  (b) the executive power shall be vested in the President of the Philippines;  and
82 83

(c) the judicial power shall be vested in one Supreme Court and in such lower courts as may be
established by law.  The Constitution has truly blocked out with deft strokes and in bold lines, the
84

allotment of powers among the three branches of government. 85

In its relationship with its co-equals, the Judiciary recognizes the doctrine of separation of powers
which imposes upon the courts proper restraint, born of the nature of their functions and of their
respect for the other branches of government, in striking down the acts of the Executive or the
Legislature as unconstitutional. Verily, the policy is a harmonious blend of courtesy and caution. 86

It has also long been observed, however, that in times of social disquietude or political instability, the
great landmarks of the Constitution are apt to be forgotten or marred, if not entirely obliterated.  In 87

order to address this, the Constitution impresses upon the Court to respect the acts performed by a
co-equal branch done within its sphere of competence and authority, but at the same time, allows it
to cross the line of separation - but only at a very limited and specific point - to determine whether
the acts of the executive and the legislative branches are null because they were undertaken with
grave abuse of discretion.  Thus, while the Court may not pass upon questions of wisdom, justice or
88

expediency of the RH Law, it may do so where an attendant unconstitutionality or grave abuse of


discretion results.  The Court must demonstrate its unflinching commitment to protect those
89

cherished rights and principles embodied in the Constitution.

In this connection, it bears adding that while the scope of judicial power of review may be limited, the
Constitution makes no distinction as to the kind of legislation that may be subject to judicial scrutiny,
be it in the form of social legislation or otherwise. The reason is simple and goes back to the earlier
point. The Court may pass upon the constitutionality of acts of the legislative and the executive
branches, since its duty is not to review their collective wisdom but, rather, to make sure that they
have acted in consonance with their respective authorities and rights as mandated of them by the
Constitution. If after said review, the Court finds no constitutional violations of any sort, then, it has
no more authority of proscribing the actions under review.  This is in line with Article VIII, Section 1
90

of the Constitution which expressly provides:

Section 1. The judicial power shall be vested in one Supreme Court and in such lower courts as may
be established by law.

Judicial power includes the duty of the courts of justice to settle actual controversies involving rights
which are legally demandable and enforceable, and to determine whether or not there has been a
grave abuse of discretion amounting to lack or excess of jurisdiction on the part of any branch or
instrumentality of the Government. [Emphases supplied]

As far back as Tanada v. Angara,  the Court has unequivocally declared that certiorari, prohibition
91

and mandamus are appropriate remedies to raise constitutional issues and to review and/or
prohibit/nullify, when proper, acts of legislative and executive officials, as there is no other plain,
speedy or adequate remedy in the ordinary course of law. This ruling was later on applied in
Macalintal v. COMELEC,  Aldaba v. COMELEC,  Magallona v. Ermita,  and countless others. In
92 93 94

Tanada, the Court wrote:

In seeking to nullify an act of the Philippine Senate on the ground that it contravenes the
Constitution, the petition no doubt raises a justiciable controversy. Where an action of the legislative
branch is seriously alleged to have infringed the Constitution, it becomes not only the right but in fact
the duty of the judiciary to settle the dispute. "The question thus posed is judicial rather than political.
The duty (to adjudicate) remains to assure that the supremacy of the Constitution is upheld. " Once a
"controversy as to the application or interpretation of constitutional provision is raised before this
Court (as in the instant case), it becomes a legal issue which the Court is bound by constitutional
mandate to decide. [Emphasis supplied]

In the scholarly estimation of former Supreme Court Justice Florentino Feliciano, "judicial review is
essential for the maintenance and enforcement of the separation of powers and the balancing of
powers among the three great departments of government through the definition and maintenance of
the boundaries of authority and control between them. To him, judicial review is the chief, indeed the
only, medium of participation - or instrument of intervention - of the judiciary in that balancing
operation.95

Lest it be misunderstood, it bears emphasizing that the Court does not have the unbridled authority
to rule on just any and every claim of constitutional violation. Jurisprudence is replete with the rule
that the power of judicial review is limited by four exacting requisites, viz : (a) there must be an
actual case or controversy; (b) the petitioners must possess locus standi; (c) the question of
constitutionality must be raised at the earliest opportunity; and (d) the issue of constitutionality must
be the lis mota of the case. 96

Actual Case or Controversy

Proponents of the RH Law submit that the subj ect petitions do not present any actual case or
controversy because the RH Law has yet to be implemented.  They claim that the questions raised
97

by the petitions are not yet concrete and ripe for adjudication since no one has been charged with
violating any of its provisions and that there is no showing that any of the petitioners' rights has been
adversely affected by its operation.  In short, it is contended that judicial review of the RH Law is
98

premature.

An actual case or controversy means an existing case or controversy that is appropriate or ripe for
determination, not conjectural or anticipatory, lest the decision of the court would amount to an
advisory opinion.  The rule is that courts do not sit to adjudicate mere academic questions to satisfy
99

scholarly interest, however intellectually challenging. The controversy must be justiciable-definite


and concrete, touching on the legal relations of parties having adverse legal interests. In other
words, the pleadings must show an active antagonistic assertion of a legal right, on the one hand,
and a denial thereof, on the other; that is, it must concern a real, tangible and not merely a
theoretical question or issue. There ought to be an actual and substantial controversy admitting of
specific relief through a decree conclusive in nature, as distinguished from an opinion advising what
the law would be upon a hypothetical state of facts. 100

Corollary to the requirement of an actual case or controversy is the requirement of ripeness.  A 101

question is ripe for adjudication when the act being challenged has had a direct adverse effect on
the individual challenging it. For a case to be considered ripe for adjudication, it is a prerequisite that
something has then been accomplished or performed by either branch before a court may come into
the picture, and the petitioner must allege the existence of an immediate or threatened injury to
himself as a result of the challenged action. He must show that he has sustained or is immediately in
danger of sustaining some direct injury as a result of the act complained of 102

In The Province of North Cotabato v. The Government of the Republic of the Philippines,  where the
103

constitutionality of an unimplemented Memorandum of Agreement on the Ancestral Domain (MOA-


AD) was put in question, it was argued that the Court has no authority to pass upon the issues
raised as there was yet no concrete act performed that could possibly violate the petitioners' and the
intervenors' rights. Citing precedents, the Court ruled that the fact of the law or act in question being
not yet effective does not negate ripeness. Concrete acts under a law are not necessary to render
the controversy ripe. Even a singular violation of the Constitution and/or the law is enough to awaken
judicial duty.

In this case, the Court is of the view that an actual case or controversy exists and that the same is
ripe for judicial determination. Considering that the RH Law and its implementing rules have already
taken effect and that budgetary measures to carry out the law have already been passed, it is
evident that the subject petitions present a justiciable controversy. As stated earlier, when an action
of the legislative branch is seriously alleged to have infringed the Constitution, it not only becomes a
right, but also a duty of the Judiciary to settle the dispute.
104

Moreover, the petitioners have shown that the case is so because medical practitioners or medical
providers are in danger of being criminally prosecuted under the RH Law for vague violations
thereof, particularly public health officers who are threatened to be dismissed from the service with
forfeiture of retirement and other benefits. They must, at least, be heard on the matter NOW.

Facial Challenge

The OSG also assails the propriety of the facial challenge lodged by the subject petitions,
contending that the RH Law cannot be challenged "on its face" as it is not a speech regulating
measure. 105

The Court is not persuaded.

In United States (US) constitutional law, a facial challenge, also known as a First Amendment
Challenge, is one that is launched to assail the validity of statutes concerning not only protected
speech, but also all other rights in the First Amendment.  These include religious freedom, freedom
106

of the press, and the right of the people to peaceably assemble, and to petition the Government for a
redress of grievances.  After all, the fundamental right to religious freedom, freedom of the press
107
and peaceful assembly are but component rights of the right to one's freedom of expression, as they
are modes which one's thoughts are externalized.

In this jurisdiction, the application of doctrines originating from the U.S. has been generally
maintained, albeit with some modifications. While this Court has withheld the application of facial
challenges to strictly penal statues,  it has expanded its scope to cover statutes not only regulating
108

free speech, but also those involving religious freedom, and other fundamental rights.  The 109

underlying reason for this modification is simple. For unlike its counterpart in the U.S., this Court,
under its expanded jurisdiction, is mandated by the Fundamental Law not only to settle actual
controversies involving rights which are legally demandable and enforceable, but also to determine
whether or not there has been a grave abuse of discretion amounting to lack or excess of jurisdiction
on the part of any branch or instrumentality of the Government.  Verily, the framers of Our
110

Constitution envisioned a proactive Judiciary, ever vigilant with its duty to maintain the supremacy of
the Constitution.

Consequently, considering that the foregoing petitions have seriously alleged that the constitutional
human rights to life, speech and religion and other fundamental rights mentioned above have been
violated by the assailed legislation, the Court has authority to take cognizance of these kindred
petitions and to determine if the RH Law can indeed pass constitutional scrutiny. To dismiss these
petitions on the simple expedient that there exist no actual case or controversy, would diminish this
Court as a reactive branch of government, acting only when the Fundamental Law has been
transgressed, to the detriment of the Filipino people.

Locus Standi

The OSG also attacks the legal personality of the petitioners to file their respective petitions. It
contends that the "as applied challenge" lodged by the petitioners cannot prosper as the assailed
law has yet to be enforced and applied against them,  and the government has yet to distribute
111

reproductive health devices that are abortive. 112

The petitioners, for their part, invariably invoke the "transcendental importance" doctrine and their
status as citizens and taxpayers in establishing the requisite locus standi.

Locus standi or legal standing is defined as a personal and substantial interest in a case such that
the party has sustained or will sustain direct injury as a result of the challenged governmental act.  It
113

requires a personal stake in the outcome of the controversy as to assure the concrete adverseness
which sharpens the presentation of issues upon which the court so largely depends for illumination
of difficult constitutional questions.
114

In relation to locus standi, the "as applied challenge" embodies the rule that one can challenge the
constitutionality of a statute only if he asserts a violation of his own rights. The rule prohibits one
from challenging the constitutionality of the statute grounded on a violation of the rights of third
persons not before the court. This rule is also known as the prohibition against third-party standing. 115

Transcendental Importance

Notwithstanding, the Court leans on the doctrine that "the rule on standing is a matter of procedure,
hence, can be relaxed for non-traditional plaintiffs like ordinary citizens, taxpayers, and legislators
when the public interest so requires, such as when the matter is of transcendental importance, of
overreaching significance to society, or of paramount public interest."116
In Coconut Oil Refiners Association, Inc. v. Torres,  the Court held that in cases of paramount
117

importance where serious constitutional questions are involved, the standing requirement may be
relaxed and a suit may be allowed to prosper even where there is no direct injury to the party
claiming the right of judicial review. In the first Emergency Powers Cases,  ordinary citizens and
118

taxpayers were allowed to question the constitutionality of several executive orders although they
had only an indirect and general interest shared in common with the public.

With these said, even if the constitutionality of the RH Law may not be assailed through an "as-
applied challenge, still, the Court has time and again acted liberally on the locus s tandi requirement.
It has accorded certain individuals standing to sue, not otherwise directly injured or with material
interest affected by a Government act, provided a constitutional issue of transcendental importance
is invoked. The rule on locus standi is, after all, a procedural technicality which the Court has, on
more than one occasion, waived or relaxed, thus allowing non-traditional plaintiffs, such as
concerned citizens, taxpayers, voters or legislators, to sue in the public interest, albeit they may not
have been directly injured by the operation of a law or any other government act. As held in Jaworski
v. PAGCOR: 119

Granting arguendo that the present action cannot be properly treated as a petition for prohibition, the
transcendental importance of the issues involved in this case warrants that we set aside the
technical defects and take primary jurisdiction over the petition at bar. One cannot deny that the
issues raised herein have potentially pervasive influence on the social and moral well being of this
nation, specially the youth; hence, their proper and just determination is an imperative need. This is
in accordance with the well-entrenched principle that rules of procedure are not inflexible tools
designed to hinder or delay, but to facilitate and promote the administration of justice. Their strict and
rigid application, which would result in technicalities that tend to frustrate, rather than promote
substantial justice, must always be eschewed. (Emphasis supplied)

In view of the seriousness, novelty and weight as precedents, not only to the public, but also to the
bench and bar, the issues raised must be resolved for the guidance of all. After all, the RH Law
drastically affects the constitutional provisions on the right to life and health, the freedom of religion
and expression and other constitutional rights. Mindful of all these and the fact that the issues of
contraception and reproductive health have already caused deep division among a broad spectrum
of society, the Court entertains no doubt that the petitions raise issues of transcendental importance
warranting immediate court adjudication. More importantly, considering that it is the right to life of the
mother and the unborn which is primarily at issue, the Court need not wait for a life to be taken away
before taking action.

The Court cannot, and should not, exercise judicial restraint at this time when rights enshrined in the
Constitution are being imperilled to be violated. To do so, when the life of either the mother or her
child is at stake, would lead to irreparable consequences.

Declaratory Relief

The respondents also assail the petitions because they are essentially petitions for declaratory relief
over which the Court has no original jurisdiction.  Suffice it to state that most of the petitions are
120

praying for injunctive reliefs and so the Court would just consider them as petitions for prohibition
under Rule 65, over which it has original jurisdiction. Where the case has far-reaching implications
and prays for injunctive reliefs, the Court may consider them as petitions for prohibition under Rule
65.121

One Subject-One Title


The petitioners also question the constitutionality of the RH Law, claiming that it violates Section
26(1 ), Article VI of the Constitution,  prescribing the one subject-one title rule. According to them,
122

being one for reproductive health with responsible parenthood, the assailed legislation violates the
constitutional standards of due process by concealing its true intent - to act as a population control
measure. 123

To belittle the challenge, the respondents insist that the RH Law is not a birth or population control
measure,  and that the concepts of "responsible parenthood" and "reproductive health" are both
124

interrelated as they are inseparable.125

Despite efforts to push the RH Law as a reproductive health law, the Court sees it as principally a
population control measure. The corpus of the RH Law is geared towards the reduction of the
country's population. While it claims to save lives and keep our women and children healthy, it also
promotes pregnancy-preventing products. As stated earlier, the RH Law emphasizes the need to
provide Filipinos, especially the poor and the marginalized, with access to information on the full
range of modem family planning products and methods. These family planning methods, natural or
modem, however, are clearly geared towards the prevention of pregnancy.

For said reason, the manifest underlying objective of the RH Law is to reduce the number of births in
the country.

It cannot be denied that the measure also seeks to provide pre-natal and post-natal care as well. A
large portion of the law, however, covers the dissemination of information and provisions on access
to medically-safe, non-abortifacient, effective, legal, affordable, and quality reproductive health care
services, methods, devices, and supplies, which are all intended to prevent pregnancy.

The Court, thus, agrees with the petitioners' contention that the whole idea of contraception
pervades the entire RH Law. It is, in fact, the central idea of the RH Law.  Indeed, remove the
126

provisions that refer to contraception or are related to it and the RH Law loses its very
foundation.  As earlier explained, "the other positive provisions such as skilled birth attendance,
127

maternal care including pre-and post-natal services, prevention and management of reproductive
tract infections including HIV/AIDS are already provided for in the Magna Carta for Women." 128

Be that as it may, the RH Law does not violate the one subject/one bill rule. In Benjamin E.
Cawaling, Jr. v. The Commission on Elections and Rep. Francis Joseph G Escudero, it was written:

It is well-settled that the "one title-one subject" rule does not require the Congress to employ in the
title of the enactment language of such precision as to mirror, fully index or catalogue all the
contents and the minute details therein. The rule is sufficiently complied with if the title is
comprehensive enough as to include the general object which the statute seeks to effect, and where,
as here, the persons interested are informed of the nature, scope and consequences of the
proposed law and its operation. Moreover, this Court has invariably adopted a liberal rather than
technical construction of the rule "so as not to cripple or impede legislation." [Emphases supplied]

In this case, a textual analysis of the various provisions of the law shows that both "reproductive
health" and "responsible parenthood" are interrelated and germane to the overriding objective to
control the population growth. As expressed in the first paragraph of Section 2 of the RH Law:

SEC. 2. Declaration of Policy. - The State recognizes and guarantees the human rights of all
persons including their right to equality and nondiscrimination of these rights, the right to sustainable
human development, the right to health which includes reproductive health, the right to education
and information, and the right to choose and make decisions for themselves in accordance with their
religious convictions, ethics, cultural beliefs, and the demands of responsible parenthood.

The one subject/one title rule expresses the principle that the title of a law must not be "so uncertain
that the average person reading it would not be informed of the purpose of the enactment or put on
inquiry as to its contents, or which is misleading, either in referring to or indicating one subject where
another or different one is really embraced in the act, or in omitting any expression or indication of
the real subject or scope of the act."129

Considering the close intimacy between "reproductive health" and "responsible parenthood" which
bears to the attainment of the goal of achieving "sustainable human development" as stated under
its terms, the Court finds no reason to believe that Congress intentionally sought to deceive the
public as to the contents of the assailed legislation.

II - SUBSTANTIVE ISSUES:

1-The Right to Life


Position of the Petitioners

The petitioners assail the RH Law because it violates the right to life and health of the unborn child
under Section 12, Article II of the Constitution. The assailed legislation allowing access to
abortifacients/abortives effectively sanctions abortion.130

According to the petitioners, despite its express terms prohibiting abortion, Section 4(a) of the RH
Law considers contraceptives that prevent the fertilized ovum to reach and be implanted in the
mother's womb as an abortifacient; thus, sanctioning contraceptives that take effect after fertilization
and prior to implantation, contrary to the intent of the Framers of the Constitution to afford protection
to the fertilized ovum which already has life.

They argue that even if Section 9 of the RH Law allows only "non-abortifacient" hormonal
contraceptives, intrauterine devices, injectables and other safe, legal, non-abortifacient and effective
family planning products and supplies, medical research shows that contraceptives use results in
abortion as they operate to kill the fertilized ovum which already has life.
131

As it opposes the initiation of life, which is a fundamental human good, the petitioners assert that the
State sanction of contraceptive use contravenes natural law and is an affront to the dignity of man. 132

Finally, it is contended that since Section 9 of the RH Law requires the Food and Drug
Administration (FDA) to certify that the product or supply is not to be used as an abortifacient, the
assailed legislation effectively confirms that abortifacients are not prohibited. Also considering that
the FDA is not the agency that will actually supervise or administer the use of these products and
supplies to prospective patients, there is no way it can truthfully make a certification that it shall not
be used for abortifacient purposes. 133

Position of the Respondents

For their part, the defenders of the RH Law point out that the intent of the Framers of the
Constitution was simply the prohibition of abortion. They contend that the RH Law does not violate
the Constitution since the said law emphasizes that only "non-abortifacient" reproductive health care
services, methods, devices products and supplies shall be made accessible to the public. 134
According to the OSG, Congress has made a legislative determination that contraceptives are not
abortifacients by enacting the RH Law. As the RH Law was enacted with due consideration to
various studies and consultations with the World Health Organization (WHO) and other experts in
the medical field, it is asserted that the Court afford deference and respect to such a determination
and pass judgment only when a particular drug or device is later on determined as an abortive. 135

For his part, respondent Lagman argues that the constitutional protection of one's right to life is not
violated considering that various studies of the WHO show that life begins from the implantation of
the fertilized ovum. Consequently, he argues that the RH Law is constitutional since the law
specifically provides that only contraceptives that do not prevent the implantation of the fertilized
ovum are allowed. 136

The Court's Position

It is a universally accepted principle that every human being enjoys the right to life. 137

Even if not formally established, the right to life, being grounded on natural law, is inherent and,
therefore, not a creation of, or dependent upon a particular law, custom, or belief. It precedes and
transcends any authority or the laws of men.

In this jurisdiction, the right to life is given more than ample protection. Section 1, Article III of the
Constitution provides:

Section 1. No person shall be deprived of life, liberty, or property without due process of law, nor
shall any person be denied the equal protection of the laws.

As expounded earlier, the use of contraceptives and family planning methods in the Philippines is
not of recent vintage. From the enactment of R.A. No. 4729, entitled "An Act To Regulate The Sale,
Dispensation, and/or Distribution of Contraceptive Drugs and Devices "on June 18, 1966, prescribing
rules on contraceptive drugs and devices which prevent fertilization,  to the promotion of male
138

vasectomy and tubal ligation,  and the ratification of numerous international agreements, the
139

country has long recognized the need to promote population control through the use of
contraceptives in order to achieve long-term economic development. Through the years, however,
the use of contraceptives and other family planning methods evolved from being a component of
demographic management, to one centered on the promotion of public health, particularly,
reproductive health. 140

This has resulted in the enactment of various measures promoting women's rights and health and
the overall promotion of the family's well-being. Thus, aside from R.A. No. 4729, R.A. No. 6365 or
"The Population Act of the Philippines" and R.A. No. 9710, otherwise known as the "The Magna
Carta of Women" were legislated. Notwithstanding this paradigm shift, the Philippine national
population program has always been grounded two cornerstone principles: "principle of no-abortion"
and the "principle of non-coercion."  As will be discussed later, these principles are not merely
141

grounded on administrative policy, but rather, originates from the constitutional protection expressly
provided to afford protection to life and guarantee religious freedom.

When Life Begins*

Majority of the Members of the Court are of the position that the question of when life begins is a
scientific and medical issue that should not be decided, at this stage, without proper hearing and
evidence. During the deliberation, however, it was agreed upon that the individual members of the
Court could express their own views on this matter.

In this regard, the ponente, is of the strong view that life begins at fertilization.

In answering the question of when life begins, focus should be made on the particular phrase of
Section 12 which reads:

Section 12. The State recognizes the sanctity of family life and shall protect and strengthen the
family as a basic autonomous social institution. It shall equally protect the life of the mother and the
life of the unborn from conception. The natural and primary right and duty of parents in the rearing of
the youth for civic efficiency and the development of moral character shall receive the support of the
Government.

Textually, the Constitution affords protection to the unborn from conception. This is undisputable
because before conception, there is no unborn to speak of. For said reason, it is no surprise that the
Constitution is mute as to any proscription prior to conception or when life begins. The problem has
arisen because, amazingly, there are quarters who have conveniently disregarded the scientific fact
that conception is reckoned from fertilization. They are waving the view that life begins at
implantation. Hence, the issue of when life begins.

In a nutshell, those opposing the RH Law contend that conception is synonymous with "fertilization"
of the female ovum by the male sperm.  On the other side of the spectrum are those who assert that
142

conception refers to the "implantation" of the fertilized ovum in the uterus. 143

Plain and Legal Meaning

It is a canon in statutory construction that the words of the Constitution should be interpreted in their
plain and ordinary meaning. As held in the recent case of Chavez v. Judicial Bar Council: 144

One of the primary and basic rules in statutory construction is that where the words of a statute are
clear, plain, and free from ambiguity, it must be given its literal meaning and applied without
attempted interpretation. It is a well-settled principle of constitutional construction that the language
employed in the Constitution must be given their ordinary meaning except where technical terms are
employed. As much as possible, the words of the Constitution should be understood in the sense
they have in common use. What it says according to the text of the provision to be construed
compels acceptance and negates the power of the courts to alter it, based on the postulate that the
framers and the people mean what they say. Verba legis non est recedendum - from the words of a
statute there should be no departure.

The raison d' etre for the rule is essentially two-fold: First, because it is assumed that the words in
which constitutional provisions are couched express the objective sought to be attained; and second,
because the Constitution is not primarily a lawyer's document but essentially that of the people, in
whose consciousness it should ever be present as an important condition for the rule of law to
prevail.

In conformity with the above principle, the traditional meaning of the word "conception" which, as
described and defined by all reliable and reputable sources, means that life begins at fertilization.
Webster's Third New International Dictionary describes it as the act of becoming pregnant, formation
of a viable zygote; the fertilization that results in a new entity capable of developing into a being like
its parents.
145

Black's Law Dictionary gives legal meaning to the term "conception" as the fecundation of the female
ovum by the male spermatozoon resulting in human life capable of survival and maturation under
normal conditions. 146

Even in jurisprudence, an unborn child has already a legal personality. In Continental Steel
Manufacturing Corporation v. Hon. Accredited Voluntary Arbitrator Allan S. Montano,  it was written:
147

Life is not synonymous with civil personality. One need not acquire civil personality first before
he/she could die. Even a child inside the womb already has life. No less than the Constitution
recognizes the life of the unborn from conception, that the State must protect equally with the life of
the mother. If the unborn already has life, then the cessation thereof even prior to the child being
delivered, qualifies as death. [Emphases in the original]

In Gonzales v. Carhart,  Justice Anthony Kennedy, writing for the US Supreme Court, said that the
148

State "has respect for human life at all stages in the pregnancy" and "a legitimate and substantial
interest in preserving and promoting fetal life." Invariably, in the decision, the fetus was referred to,
or cited, as a baby or a child.149

Intent of the Framers

Records of the Constitutional Convention also shed light on the intention of the Framers regarding
the term "conception" used in Section 12, Article II of the Constitution. From their deliberations, it
clearly refers to the moment of "fertilization." The records reflect the following:

Rev. Rigos: In Section 9, page 3, there is a sentence which reads:

"The State shall equally protect the life of the mother and the life of the unborn from the moment of
conception."

When is the moment of conception?

xxx

Mr. Villegas: As I explained in the sponsorship speech, it is when the ovum is fertilized by the sperm
that there is human life. x x x.150

xxx

As to why conception is reckoned from fertilization and, as such, the beginning of human life, it was
explained:

Mr. Villegas: I propose to review this issue in a biological manner. The first question that needs to be
answered is: Is the fertilized ovum alive? Biologically categorically says yes, the fertilized ovum is
alive. First of all, like all living organisms, it takes in nutrients which it processes by itself. It begins
doing this upon fertilization. Secondly, as it takes in these nutrients, it grows from within. Thirdly, it
multiplies itself at a geometric rate in the continuous process of cell division. All these processes are
vital signs of life. Therefore, there is no question that biologically the fertilized ovum has life.
The second question: Is it human? Genetics gives an equally categorical "yes." At the moment of
conception, the nuclei of the ovum and the sperm rupture. As this happens 23 chromosomes from
the ovum combine with 23 chromosomes of the sperm to form a total of 46 chromosomes. A
chromosome count of 46 is found only - and I repeat, only in human cells. Therefore, the fertilized
ovum is human.

Since these questions have been answered affirmatively, we must conclude that if the fertilized
ovum is both alive and human, then, as night follows day, it must be human life. Its nature is
human. 151

Why the Constitution used the phrase "from the moment of conception" and not "from the moment of
fertilization" was not because of doubt when human life begins, but rather, because:

Mr. Tingson: x x x x the phrase from the moment of conception" was described by us here before
with the scientific phrase "fertilized ovum" may be beyond the comprehension of some people; we
want to use the simpler phrase "from the moment of conception." 152

Thus, in order to ensure that the fertilized ovum is given ample protection under the Constitution, it
was discussed:

Rev. Rigos: Yes, we think that the word "unborn" is sufficient for the purpose of writing a
Constitution, without specifying "from the moment of conception."

Mr. Davide: I would not subscribe to that particular view because according to the Commissioner's
own admission, he would leave it to Congress to define when life begins. So, Congress can define
life to begin from six months after fertilization; and that would really be very, very, dangerous. It is
now determined by science that life begins from the moment of conception. There can be no doubt
about it. So we should not give any doubt to Congress, too. 153

Upon further inquiry, it was asked:

Mr. Gascon: Mr. Presiding Officer, I would like to ask a question on that point. Actually, that is one of
the questions I was going to raise during the period of interpellations but it has been expressed
already. The provision, as proposed right now states:

The State shall equally protect the life of the mother and the life of the unborn from the moment of
conception.

When it speaks of "from the moment of conception," does this mean when the egg meets the
sperm?

Mr. Villegas: Yes, the ovum is fertilized by the sperm.

Mr. Gascon: Therefore that does not leave to Congress the right to determine whether certain
contraceptives that we know today are abortifacient or not because it is a fact that some of the so-
called contraceptives deter the rooting of the ovum in the uterus. If fertilization has already occurred,
the next process is for the fertilized ovum to travel towards the uterus and to take root. What
happens with some contraceptives is that they stop the opportunity for the fertilized ovum to reach
the uterus. Therefore, if we take the provision as it is proposed, these so called contraceptives
should be banned.
Mr. Villegas: Yes, if that physical fact is established, then that is what is called abortifacient and,
therefore, would be unconstitutional and should be banned under this provision.

Mr. Gascon: Yes. So my point is that I do not think it is up to Congress to state whether or not these
certain contraceptives are abortifacient. Scientifically and based on the provision as it is now
proposed, they are already considered abortifacient. 154

From the deliberations above-quoted, it is apparent that the Framers of the Constitution emphasized
that the State shall provide equal protection to both the mother and the unborn child from the earliest
opportunity of life, that is, upon fertilization or upon the union of the male sperm and the female
ovum. It is also apparent is that the Framers of the Constitution intended that to prohibit Congress
from enacting measures that would allow it determine when life begins.

Equally apparent, however, is that the Framers of the Constitution did not intend to ban all
contraceptives for being unconstitutional. In fact, Commissioner Bernardo Villegas, spearheading the
need to have a constitutional provision on the right to life, recognized that the determination of
whether a contraceptive device is an abortifacient is a question of fact which should be left to the
courts to decide on based on established evidence. 155

From the discussions above, contraceptives that kill or destroy the fertilized ovum should be deemed
an abortive and thus prohibited. Conversely, contraceptives that actually prevent the union of the
male sperm and the female ovum, and those that similarly take action prior to fertilization should be
deemed non-abortive, and thus, constitutionally permissible.

As emphasized by the Framers of the Constitution:

x x x           x x x          x x x

Mr. Gascon: xx xx. As I mentioned in my speech on the US bases, I am pro-life, to the point that I
would like not only to protect the life of the unborn, but also the lives of the millions of people in the
world by fighting for a nuclear-free world. I would just like to be assured of the legal and pragmatic
implications of the term "protection of the life of the unborn from the moment of conception." I raised
some of these implications this afternoon when I interjected in the interpellation of Commissioner
Regalado. I would like to ask that question again for a categorical answer.

I mentioned that if we institutionalize the term "the life of the unborn from the moment of conception"
we are also actually saying "no," not "maybe," to certain contraceptives which are already being
encouraged at this point in time. Is that the sense of the committee or does it disagree with me?

Mr. Azcuna: No, Mr. Presiding Officer, because contraceptives would be preventive. There is no
unborn yet. That is yet unshaped.

Mr. Gascon: Yes, Mr. Presiding Officer, but I was speaking more about some contraceptives, such
as the intra-uterine device which actually stops the egg which has already been fertilized from taking
route to the uterus. So if we say "from the moment of conception," what really occurs is that some of
these contraceptives will have to be unconstitutionalized.

Mr. Azcuna: Yes, to the extent that it is after the fertilization.

Mr. Gascon: Thank you, Mr. Presiding Officer. 156


The fact that not all contraceptives are prohibited by the 1987 Constitution is even admitted by
petitioners during the oral arguments. There it was conceded that tubal ligation, vasectomy, even
condoms are not classified as abortifacients. 157

Atty. Noche:

Before the union of the eggs, egg and the sperm, there is no life yet.

Justice Bersamin:

There is no life.

Atty. Noche:

So, there is no life to be protected.

Justice Bersamin:

To be protected.

Atty. Noche:

Under Section 12, yes.

Justice Bersamin:

So you have no objection to condoms?

Atty. Noche:

Not under Section 12, Article II.

Justice Bersamin:

Even if there is already information that condoms sometimes have porosity?

Atty. Noche:

Well, yes, Your Honor, there are scientific findings to that effect, Your Honor, but I am discussing
here Section 12, Article II, Your Honor, yes.

Justice Bersamin:

Alright.

Atty. Noche:

And it's not, I have to admit it's not an abortifacient, Your Honor. 158
Medical Meaning

That conception begins at fertilization is not bereft of medical foundation. Mosby s Medical, Nursing,
and Allied Health Dictionary defines conception as "the beginning of pregnancy usually taken to be
the instant a spermatozoon enters an ovum and forms a viable zygote." 159

It describes fertilization as "the union of male and female gametes to form a zygote from which the
embryo develops." 160

The Textbook of Obstetrics (Physiological & Pathological Obstetrics),  used by medical schools in
161

the Philippines, also concludes that human life (human person) begins at the moment of fertilization
with the union of the egg and the sperm resulting in the formation of a new individual, with a unique
genetic composition that dictates all developmental stages that ensue.

Similarly, recent medical research on the matter also reveals that: "Human development begins after
the union of male and female gametes or germ cells during a process known as fertilization
(conception). Fertilization is a sequence of events that begins with the contact of a sperm
(spermatozoon) with a secondary oocyte (ovum) and ends with the fusion of their pronuclei (the
haploid nuclei of the sperm and ovum) and the mingling of their chromosomes to form a new cell.
This fertilized ovum, known as a zygote, is a large diploid cell that is the beginning, or primordium, of
a human being." 162

The authors of Human Embryology & Teratology  mirror the same position. They wrote: "Although
163

life is a continuous process, fertilization is a critical landmark because, under ordinary


circumstances, a new, genetically distinct human organism is thereby formed.... The combination of
23 chromosomes present in each pronucleus results in 46 chromosomes in the zygote. Thus the
diploid number is restored and the embryonic genome is formed. The embryo now exists as a
genetic unity."

In support of the RH Bill, The Philippine Medical Association came out with a "Paper on the
Reproductive Health Bill (Responsible Parenthood Bill)" and therein concluded that:

CONCLUSION

The PMA throws its full weight in supporting the RH Bill at the same time that PMA maintains its
strong position that fertilization is sacred because it is at this stage that conception, and thus human
life, begins. Human lives are sacred from the moment of conception, and that destroying those new
lives is never licit, no matter what the purported good outcome would be. In terms of biology and
human embryology, a human being begins immediately at fertilization and after that, there is no point
along the continuous line of human embryogenesis where only a "potential" human being can be
posited. Any philosophical, legal, or political conclusion cannot escape this objective scientific fact.

The scientific evidence supports the conclusion that a zygote is a human organism and that the life
of a new human being commences at a scientifically well defined "moment of conception." This
conclusion is objective, consistent with the factual evidence, and independent of any specific ethical,
moral, political, or religious view of human life or of human embryos. 164

Conclusion: The Moment of Conception is Reckoned from


Fertilization
In all, whether it be taken from a plain meaning, or understood under medical parlance, and more
importantly, following the intention of the Framers of the Constitution, the undeniable conclusion is
that a zygote is a human organism and that the life of a new human being commences at a
scientifically well-defined moment of conception, that is, upon fertilization.

For the above reasons, the Court cannot subscribe to the theory advocated by Hon. Lagman that life
begins at implantation.  According to him, "fertilization and conception are two distinct and
165

successive stages in the reproductive process. They are not identical and synonymous."  Citing a 166

letter of the WHO, he wrote that "medical authorities confirm that the implantation of the fertilized
ovum is the commencement of conception and it is only after implantation that pregnancy can be
medically detected." 167

This theory of implantation as the beginning of life is devoid of any legal or scientific mooring. It does
not pertain to the beginning of life but to the viability of the fetus. The fertilized ovum/zygote is not an
inanimate object - it is a living human being complete with DNA and 46 chromosomes.  Implantation
168

has been conceptualized only for convenience by those who had population control in mind. To
adopt it would constitute textual infidelity not only to the RH Law but also to the Constitution.

Not surprisingly, even the OSG does not support this position.

If such theory would be accepted, it would unnervingly legitimize the utilization of any drug or device
that would prevent the implantation of the fetus at the uterine wall. It would be provocative and
further aggravate religious-based divisiveness.

It would legally permit what the Constitution proscribes - abortion and abortifacients.

The RH Law and Abortion

The clear and unequivocal intent of the Framers of the 1987 Constitution in protecting the life of the
unborn from conception was to prevent the Legislature from enacting a measure legalizing abortion.
It was so clear that even the Court cannot interpret it otherwise. This intent of the Framers was
captured in the record of the proceedings of the 1986 Constitutional Commission. Commissioner
Bernardo Villegas, the principal proponent of the protection of the unborn from conception,
explained:

The intention .. .is to make sure that there would be no pro-abortion laws ever passed by Congress
or any pro-abortion decision passed by the Supreme Court. 169

A reading of the RH Law would show that it is in line with this intent and actually proscribes abortion.
While the Court has opted not to make any determination, at this stage, when life begins, it finds that
the RH Law itself clearly mandates that protection be afforded from the moment of fertilization. As
pointed out by Justice Carpio, the RH Law is replete with provisions that embody the policy of the
law to protect to the fertilized ovum and that it should be afforded safe travel to the uterus for
implantation.170

Moreover, the RH Law recognizes that abortion is a crime under Article 256 of the Revised Penal
Code, which penalizes the destruction or expulsion of the fertilized ovum. Thus:

1] xx x.
Section 4. Definition of Terms. - For the purpose of this Act, the following terms shall be defined as
follows:

xxx.

(q) Reproductive health care refers to the access to a full range of methods, facilities, services and
supplies that contribute to reproductive health and well-being by addressing reproductive health-
related problems. It also includes sexual health, the purpose of which is the enhancement of life and
personal relations. The elements of reproductive health care include the following:

xxx.

(3) Proscription of abortion and management of abortion complications;

xxx.

2] xx x.

Section 4. x x x.

(s) Reproductive health rights refers to the rights of individuals and couples, to decide freely and
responsibly whether or not to have children; the number, spacing and timing of their children; to
make other decisions concerning reproduction, free of discrimination, coercion and violence; to have
the information and means to do so; and to attain the highest standard of sexual health and
reproductive health: Provided, however, That reproductive health rights do not include abortion, and
access to abortifacients.

3] xx x.

SEC. 29. Repealing Clause. - Except for prevailing laws against abortion, any law, presidential
decree or issuance, executive order, letter of instruction, administrative order, rule or regulation
contrary to or is inconsistent with the provisions of this Act including Republic Act No. 7392,
otherwise known as the Midwifery Act, is hereby repealed, modified or amended accordingly.

The RH Law and Abortifacients

In carrying out its declared policy, the RH Law is consistent in prohibiting abortifacients. To be clear,
Section 4(a) of the RH Law defines an abortifacient as:

Section 4. Definition of Terms - x x x x

(a) Abortifacient refers to any drug or device that induces abortion or the destruction of a fetus inside
the mother's womb or the prevention of the fertilized ovum to reach and be implanted in the mother's
womb upon determination of the FDA.

As stated above, the RH Law mandates that protection must be afforded from the moment of
fertilization. By using the word " or," the RH Law prohibits not only drugs or devices that prevent
implantation, but also those that induce abortion and those that induce the destruction of a fetus
inside the mother's womb. Thus, an abortifacient is any drug or device that either:

(a) Induces abortion; or


(b) Induces the destruction of a fetus inside the mother's womb; or

(c) Prevents the fertilized ovum to reach and be implanted in the mother's womb, upon
determination of the FDA.

Contrary to the assertions made by the petitioners, the Court finds that the RH Law, consistent with
the Constitution, recognizes that the fertilized ovum already has life and that the State has a
bounden duty to protect it. The conclusion becomes clear because the RH Law, first, prohibits any
drug or device that induces abortion (first kind), which, as discussed exhaustively above, refers to
that which induces the killing or the destruction of the fertilized ovum, and, second, prohibits any
drug or device the fertilized ovum to reach and be implanted in the mother's womb (third kind).

By expressly declaring that any drug or device that prevents the fertilized ovum to reach and be
implanted in the mother's womb is an abortifacient (third kind), the RH Law does not intend to mean
at all that life only begins only at implantation, as Hon. Lagman suggests. It also does not declare
either that protection will only be given upon implantation, as the petitioners likewise suggest.
Rather, it recognizes that: one, there is a need to protect the fertilized ovum which already has life,
and two, the fertilized ovum must be protected the moment it becomes existent - all the way until it
reaches and implants in the mother's womb. After all, if life is only recognized and afforded
protection from the moment the fertilized ovum implants - there is nothing to prevent any drug or
device from killing or destroying the fertilized ovum prior to implantation.

From the foregoing, the Court finds that inasmuch as it affords protection to the fertilized ovum, the
RH Law does not sanction abortion. To repeat, it is the Court's position that life begins at fertilization,
not at implantation. When a fertilized ovum is implanted in the uterine wall , its viability is sustained
but that instance of implantation is not the point of beginning of life. It started earlier. And as defined
by the RH Law, any drug or device that induces abortion, that is, which kills or destroys the fertilized
ovum or prevents the fertilized ovum to reach and be implanted in the mother's womb, is an
abortifacient.

Proviso Under Section 9 of the RH Law

This notwithstanding, the Court finds that the proviso under Section 9 of the law that "any product or
supply included or to be included in the EDL must have a certification from the FDA that said product
and supply is made available on the condition that it is not to be used as an abortifacient" as empty
as it is absurd. The FDA, with all its expertise, cannot fully attest that a drug or device will not all be
used as an abortifacient, since the agency cannot be present in every instance when the
contraceptive product or supply will be used. 171

Pursuant to its declared policy of providing access only to safe, legal and non-abortifacient
contraceptives, however, the Court finds that the proviso of Section 9, as worded, should bend to the
legislative intent and mean that "any product or supply included or to be included in the EDL must
have a certification from the FDA that said product and supply is made available on the condition
that it cannot be used as abortifacient." Such a construction is consistent with the proviso under the
second paragraph of the same section that provides:

Provided, further, That the foregoing offices shall not purchase or acquire by any means emergency
contraceptive pills, postcoital pills, abortifacients that will be used for such purpose and their other
forms or equivalent.

Abortifacients under the RH-IRR


At this juncture, the Court agrees with ALFI that the authors of the RH-IRR gravely abused their
office when they redefined the meaning of abortifacient. The RH Law defines "abortifacient" as
follows:

SEC. 4. Definition of Terms. - For the purpose of this Act, the following terms shall be defined as
follows:

(a) Abortifacient refers to any drug or device that induces abortion or the destruction of a fetus inside
the mother's womb or the prevention of the fertilized ovum to reach and be implanted in the mother's
womb upon determination of the FDA.

Section 3.0l (a) of the IRR, however, redefines "abortifacient" as:

Section 3.01 For purposes of these Rules, the terms shall be defined as follows:

a) Abortifacient refers to any drug or device that primarily induces abortion or the destruction of a
fetus inside the mother's womb or the prevention of the fertilized ovum to reach and be implanted in
the mother's womb upon determination of the Food and Drug Administration (FDA). [Emphasis
supplied]

Again in Section 3.0lG) of the RH-IRR, "contraceptive," is redefined, viz:

j) Contraceptive refers to any safe, legal, effective and scientifically proven modern family planning
method, device, or health product, whether natural or artificial, that prevents pregnancy but does not
primarily destroy a fertilized ovum or prevent a fertilized ovum from being implanted in the mother's
womb in doses of its approved indication as determined by the Food and Drug Administration (FDA).

The above-mentioned section of the RH-IRR allows "contraceptives" and recognizes as


"abortifacient" only those that primarily induce abortion or the destruction of a fetus inside the
mother's womb or the prevention of the fertilized ovum to reach and be implanted in the mother's
womb. 172

This cannot be done.

In this regard, the observations of Justice Brion and Justice Del Castillo are well taken. As they
pointed out, with the insertion of the word "primarily," Section 3.0l(a) and G) of the RH-IRR  must be
173

struck down for being ultra vires.

Evidently, with the addition of the word "primarily," in Section 3.0l(a) and G) of the RH-IRR is indeed
ultra vires. It contravenes Section 4(a) of the RH Law and should, therefore, be declared invalid.
There is danger that the insertion of the qualifier "primarily" will pave the way for the approval of
contraceptives which may harm or destroy the life of the unborn from conception/fertilization in
violation of Article II, Section 12 of the Constitution. With such qualification in the RH-IRR, it appears
to insinuate that a contraceptive will only be considered as an "abortifacient" if its sole known effect
is abortion or, as pertinent here, the prevention of the implantation of the fertilized ovum.

For the same reason, this definition of "contraceptive" would permit the approval of contraceptives
which are actually abortifacients because of their fail-safe mechanism. 174

Also, as discussed earlier, Section 9 calls for the certification by the FDA that these contraceptives
cannot act as abortive. With this, together with the definition of an abortifacient under Section 4 (a) of
the RH Law and its declared policy against abortion, the undeniable conclusion is that
contraceptives to be included in the PNDFS and the EDL will not only be those contraceptives that
do not have the primary action of causing abortion or the destruction of a fetus inside the mother's
womb or the prevention of the fertilized ovum to reach and be implanted in the mother's womb, but
also those that do not have the secondary action of acting the same way.

Indeed, consistent with the constitutional policy prohibiting abortion, and in line with the principle that
laws should be construed in a manner that its constitutionality is sustained, the RH Law and its
implementing rules must be consistent with each other in prohibiting abortion. Thus, the word "
primarily" in Section 3.0l(a) and G) of the RH-IRR should be declared void. To uphold the validity of
Section 3.0l(a) and G) of the RH-IRR and prohibit only those contraceptives that have the primary
effect of being an abortive would effectively "open the floodgates to the approval of contraceptives
which may harm or destroy the life of the unborn from conception/fertilization in violation of Article II,
Section 12 of the Constitution." 175

To repeat and emphasize, in all cases, the "principle of no abortion" embodied in the constitutional
protection of life must be upheld.

2-The Right to Health

The petitioners claim that the RH Law violates the right to health because it requires the inclusion of
hormonal contraceptives, intrauterine devices, injectables and family products and supplies in the
National Drug Formulary and the inclusion of the same in the regular purchase of essential
medicines and supplies of all national hospitals. Citing various studies on the matter, the petitioners
176

posit that the risk of developing breast and cervical cancer is greatly increased in women who use
oral contraceptives as compared to women who never use them. They point out that the risk is
decreased when the use of contraceptives is discontinued. Further, it is contended that the use of
combined oral contraceptive pills is associated with a threefold increased risk of venous
thromboembolism, a twofold increased risk of ischematic stroke, and an indeterminate effect on risk
of myocardial infarction.  Given the definition of "reproductive health" and "sexual health" under
177

Sections 4(p)  and (w)  of the RH Law, the petitioners assert that the assailed legislation only seeks
178 179

to ensure that women have pleasurable and satisfying sex lives. 180

The OSG, however, points out that Section 15, Article II of the Constitution is not self-executory, it
being a mere statement of the administration's principle and policy. Even if it were self-executory, the
OSG posits that medical authorities refute the claim that contraceptive pose a danger to the health of
women. 181

The Court's Position

A component to the right to life is the constitutional right to health. In this regard, the Constitution is
replete with provisions protecting and promoting the right to health. Section 15, Article II of the
Constitution provides:

Section 15. The State shall protect and promote the right to health of the people and instill health
consciousness among them.

A portion of Article XIII also specifically provides for the States' duty to provide for the health of the
people, viz:

HEALTH
Section 11. The State shall adopt an integrated and comprehensive approach to health development
which shall endeavor to make essential goods, health and other social services available to all the
people at affordable cost. There shall be priority for the needs of the underprivileged, sick, elderly,
disabled, women, and children. The State shall endeavor to provide free medical care to paupers.

Section 12. The State shall establish and maintain an effective food and drug regulatory system and
undertake appropriate health, manpower development, and research, responsive to the country's
health needs and problems.

Section 13. The State shall establish a special agency for disabled person for their rehabilitation,
self-development, and self-reliance, and their integration into the mainstream of society.

Finally, Section 9, Article XVI provides:

Section 9. The State shall protect consumers from trade malpractices and from substandard or
hazardous products.

Contrary to the respondent's notion, however, these provisions are self-executing. Unless the
provisions clearly express the contrary, the provisions of the Constitution should be considered self-
executory. There is no need for legislation to implement these self-executing provisions.  In Manila
182

Prince Hotel v. GSIS,  it was stated:


183

x x x Hence, unless it is expressly provided that a legislative act is necessary to enforce a


constitutional mandate, the presumption now is that all provisions of the constitution are self-
executing. If the constitutional provisions are treated as requiring legislation instead of self-
executing, the legislature would have the power to ignore and practically nullify the mandate of the
fundamental law. This can be cataclysmic. That is why the prevailing view is, as it has always been,
that –

... in case of doubt, the Constitution should be considered self-executing rather than non-self-
executing. . . . Unless the contrary is clearly intended, the provisions of the Constitution should be
considered self-executing, as a contrary rule would give the legislature discretion to determine when,
or whether, they shall be effective. These provisions would be subordinated to the will of the
lawmaking body, which could make them entirely meaningless by simply refusing to pass the
needed implementing statute. (Emphases supplied)

This notwithstanding, it bears mentioning that the petitioners, particularly ALFI, do not question
contraception and contraceptives per se.  In fact, ALFI prays that the status quo - under R.A. No.
184

5921 and R.A. No. 4729, the sale and distribution of contraceptives are not prohibited when they are
dispensed by a prescription of a duly licensed by a physician - be maintained. 185

The legislative intent in the enactment of the RH Law in this regard is to leave intact the provisions of
R.A. No. 4729. There is no intention at all to do away with it. It is still a good law and its requirements
are still in to be complied with. Thus, the Court agrees with the observation of respondent Lagman
that the effectivity of the RH Law will not lead to the unmitigated proliferation of contraceptives since
the sale, distribution and dispensation of contraceptive drugs and devices will still require the
prescription of a licensed physician. With R.A. No. 4729 in place, there exists adequate safeguards
to ensure the public that only contraceptives that are safe are made available to the public. As aptly
explained by respondent Lagman:
D. Contraceptives cannot be
dispensed and used without
prescription

108. As an added protection to voluntary users of contraceptives, the same cannot be dispensed
and used without prescription.

109. Republic Act No. 4729 or "An Act to Regulate the Sale, Dispensation, and/ or Distribution of
Contraceptive Drugs and Devices" and Republic Act No. 5921 or "An Act Regulating the Practice of
Pharmacy and Setting Standards of Pharmaceutical Education in the Philippines and for Other
Purposes" are not repealed by the RH Law and the provisions of said Acts are not inconsistent with
the RH Law.

110. Consequently, the sale, distribution and dispensation of contraceptive drugs and devices are
particularly governed by RA No. 4729 which provides in full:

"Section 1. It shall be unlawful for any person, partnership, or corporation, to sell, dispense or
otherwise distribute whether for or without consideration, any contraceptive drug or device, unless
such sale, dispensation or distribution is by a duly licensed drug store or pharmaceutical company
and with the prescription of a qualified medical practitioner.

"Sec. 2 . For the purpose of this Act:

"(a) "Contraceptive drug" is any medicine, drug, chemical, or portion which is used
exclusively for the purpose of preventing fertilization of the female ovum: and

"(b) "Contraceptive device" is any instrument, device, material, or agent introduced into the
female reproductive system for the primary purpose of preventing conception.

"Sec. 3 Any person, partnership, or corporation, violating the provisions of this Act shall be punished
with a fine of not more than five hundred pesos or an imprisonment of not less than six months or
more than one year or both in the discretion of the Court.

"This Act shall take effect upon its approval.

"Approved: June 18, 1966"

111. Of the same import, but in a general manner, Section 25 of RA No. 5921 provides:

"Section 25. Sale of medicine, pharmaceuticals, drugs and devices. No medicine, pharmaceutical, or
drug of whatever nature and kind or device shall be compounded, dispensed, sold or resold, or
otherwise be made available to the consuming public except through a prescription drugstore or
hospital pharmacy, duly established in accordance with the provisions of this Act.

112. With all of the foregoing safeguards, as provided for in the RH Law and other relevant statutes,
the pretension of the petitioners that the RH Law will lead to the unmitigated proliferation of
contraceptives, whether harmful or not, is completely unwarranted and baseless.  [Emphases in the
186

Original. Underlining supplied.]

In Re: Section 10 of the RH Law:


The foregoing safeguards should be read in connection with Section 10 of the RH Law which
provides:

SEC. 10. Procurement and Distribution of Family Planning Supplies. - The DOH shall procure,
distribute to LGUs and monitor the usage of family planning supplies for the whole country. The
DOH shall coordinate with all appropriate local government bodies to plan and implement this
procurement and distribution program. The supply and budget allotments shall be based on, among
others, the current levels and projections of the following:

(a) Number of women of reproductive age and couples who want to space or limit their
children;

(b) Contraceptive prevalence rate, by type of method used; and

(c) Cost of family planning supplies.

Provided, That LGUs may implement its own procurement, distribution and monitoring program
consistent with the overall provisions of this Act and the guidelines of the DOH.

Thus, in the distribution by the DOH of contraceptive drugs and devices, it must consider the
provisions of R.A. No. 4729, which is still in effect, and ensure that the contraceptives that it will
procure shall be from a duly licensed drug store or pharmaceutical company and that the actual
dispensation of these contraceptive drugs and devices will done following a prescription of a
qualified medical practitioner. The distribution of contraceptive drugs and devices must not be
indiscriminately done. The public health must be protected by all possible means. As pointed out by
Justice De Castro, a heavy responsibility and burden are assumed by the government in supplying
contraceptive drugs and devices, for it may be held accountable for any injury, illness or loss of life
resulting from or incidental to their use.
187

At any rate, it bears pointing out that not a single contraceptive has yet been submitted to the FDA
pursuant to the RH Law. It behooves the Court to await its determination which drugs or devices are
declared by the FDA as safe, it being the agency tasked to ensure that food and medicines available
to the public are safe for public consumption. Consequently, the Court finds that, at this point, the
attack on the RH Law on this ground is premature. Indeed, the various kinds of contraceptives must
first be measured up to the constitutional yardstick as expounded herein, to be determined as the
case presents itself.

At this point, the Court is of the strong view that Congress cannot legislate that hormonal
contraceptives and intra-uterine devices are safe and non-abortifacient. The first sentence of Section
9 that ordains their inclusion by the National Drug Formulary in the EDL by using the mandatory
"shall" is to be construed as operative only after they have been tested, evaluated, and approved by
the FDA. The FDA, not Congress, has the expertise to determine whether a particular hormonal
contraceptive or intrauterine device is safe and non-abortifacient. The provision of the third sentence
concerning the requirements for the inclusion or removal of a particular family planning supply from
the EDL supports this construction.

Stated differently, the provision in Section 9 covering the inclusion of hormonal contraceptives, intra-
uterine devices, injectables, and other safe, legal, non-abortifacient and effective family planning
products and supplies by the National Drug Formulary in the EDL is not mandatory. There must first
be a determination by the FDA that they are in fact safe, legal, non-abortifacient and effective family
planning products and supplies. There can be no predetermination by Congress that the gamut of
contraceptives are "safe, legal, non-abortifacient and effective" without the proper scientific
examination.

3 -Freedom of Religion
and the Right to Free Speech

Position of the Petitioners:

1. On Contraception

While contraceptives and procedures like vasectomy and tubal ligation are not covered by the
constitutional proscription, there are those who, because of their religious education and
background, sincerely believe that contraceptives, whether abortifacient or not, are evil. Some of
these are medical practitioners who essentially claim that their beliefs prohibit not only the use of
contraceptives but also the willing participation and cooperation in all things dealing with
contraceptive use. Petitioner PAX explained that "contraception is gravely opposed to marital
chastity, it is contrary to the good of the transmission of life, and to the reciprocal self-giving of the
spouses; it harms true love and denies the sovereign rule of God in the transmission of Human
life."
188

The petitioners question the State-sponsored procurement of contraceptives, arguing that the
expenditure of their taxes on contraceptives violates the guarantee of religious freedom since
contraceptives contravene their religious beliefs. 189

2. On Religious Accommodation and


The Duty to Refer

Petitioners Imbong and Luat note that while the RH Law attempts to address religious sentiments by
making provisions for a conscientious objector, the constitutional guarantee is nonetheless violated
because the law also imposes upon the conscientious objector the duty to refer the patient seeking
reproductive health services to another medical practitioner who would be able to provide for the
patient's needs. For the petitioners, this amounts to requiring the conscientious objector to cooperate
with the very thing he refuses to do without violating his/her religious beliefs.190

They further argue that even if the conscientious objector's duty to refer is recognized, the
recognition is unduly limited, because although it allows a conscientious objector in Section 23 (a)(3)
the option to refer a patient seeking reproductive health services and information - no escape is
afforded the conscientious objector in Section 23 (a)(l) and (2), i.e. against a patient seeking
reproductive health procedures. They claim that the right of other individuals to conscientiously
object, such as: a) those working in public health facilities referred to in Section 7; b) public officers
involved in the implementation of the law referred to in Section 23(b ); and c) teachers in public
schools referred to in Section 14 of the RH Law, are also not recognize. 191

Petitioner Echavez and the other medical practitioners meanwhile, contend that the requirement to
refer the matter to another health care service provider is still considered a compulsion on those
objecting healthcare service providers. They add that compelling them to do the act against their will
violates the Doctrine of Benevolent Neutrality. Sections 9, 14 and 1 7 of the law are too secular that
they tend to disregard the religion of Filipinos. Authorizing the use of contraceptives with abortive
effects, mandatory sex education, mandatory pro-bono reproductive health services to indigents
encroach upon the religious freedom of those upon whom they are required. 192
Petitioner CFC also argues that the requirement for a conscientious objector to refer the person
seeking reproductive health care services to another provider infringes on one's freedom of religion
as it forces the objector to become an unwilling participant in the commission of a serious sin under
Catholic teachings. While the right to act on one's belief may be regulated by the State, the acts
prohibited by the RH Law are passive acts which produce neither harm nor injury to the public. 193

Petitioner CFC adds that the RH Law does not show compelling state interest to justify regulation of
religious freedom because it mentions no emergency, risk or threat that endangers state interests. It
does not explain how the rights of the people (to equality, non-discrimination of rights, sustainable
human development, health, education, information, choice and to make decisions according to
religious convictions, ethics, cultural beliefs and the demands of responsible parenthood) are being
threatened or are not being met as to justify the impairment of religious freedom. 194

Finally, the petitioners also question Section 15 of the RH Law requiring would-be couples to attend
family planning and responsible parenthood seminars and to obtain a certificate of compliance. They
claim that the provision forces individuals to participate in the implementation of the RH Law even if
it contravenes their religious beliefs.  As the assailed law dangles the threat of penalty of fine and/or
195

imprisonment in case of non-compliance with its provisions, the petitioners claim that the RH Law
forcing them to provide, support and facilitate access and information to contraception against their
beliefs must be struck down as it runs afoul to the constitutional guarantee of religious freedom.

The Respondents' Positions

The respondents, on the other hand, contend that the RH Law does not provide that a specific mode
or type of contraceptives be used, be it natural or artificial. It neither imposes nor sanctions any
religion or belief.  They point out that the RH Law only seeks to serve the public interest by
196

providing accessible, effective and quality reproductive health services to ensure maternal and child
health, in line with the State's duty to bring to reality the social justice health guarantees of the
Constitution,  and that what the law only prohibits are those acts or practices, which deprive others
197

of their right to reproductive health.  They assert that the assailed law only seeks to guarantee
198

informed choice, which is an assurance that no one will be compelled to violate his religion against
his free will. 199

The respondents add that by asserting that only natural family planning should be allowed, the
petitioners are effectively going against the constitutional right to religious freedom, the same right
they invoked to assail the constitutionality of the RH Law.  In other words, by seeking the
200

declaration that the RH Law is unconstitutional, the petitioners are asking that the Court recognize
only the Catholic Church's sanctioned natural family planning methods and impose this on the entire
citizenry.
201

With respect to the duty to refer, the respondents insist that the same does not violate the
constitutional guarantee of religious freedom, it being a carefully balanced compromise between the
interests of the religious objector, on one hand, who is allowed to keep silent but is required to refer
-and that of the citizen who needs access to information and who has the right to expect that the
health care professional in front of her will act professionally. For the respondents, the concession
given by the State under Section 7 and 23(a)(3) is sufficient accommodation to the right to freely
exercise one's religion without unnecessarily infringing on the rights of others.
202

Whatever burden is placed on the petitioner's religious freedom is minimal as the duty to refer is
limited in duration, location and impact. 203
Regarding mandatory family planning seminars under Section 15 , the respondents claim that it is a
reasonable regulation providing an opportunity for would-be couples to have access to information
regarding parenthood, family planning, breastfeeding and infant nutrition. It is argued that those who
object to any information received on account of their attendance in the required seminars are not
compelled to accept information given to them. They are completely free to reject any information
they do not agree with and retain the freedom to decide on matters of family life without intervention
of the State.
204

For their part, respondents De Venecia et al., dispute the notion that natural family planning is the
only method acceptable to Catholics and the Catholic hierarchy. Citing various studies and surveys
on the matter, they highlight the changing stand of the Catholic Church on contraception throughout
the years and note the general acceptance of the benefits of contraceptives by its followers in
planning their families.

The Church and The State

At the outset, it cannot be denied that we all live in a heterogeneous society. It is made up of people
of diverse ethnic, cultural and religious beliefs and backgrounds. History has shown us that our
government, in law and in practice, has allowed these various religious, cultural, social and racial
groups to thrive in a single society together. It has embraced minority groups and is tolerant towards
all - the religious people of different sects and the non-believers. The undisputed fact is that our
people generally believe in a deity, whatever they conceived Him to be, and to whom they call for
guidance and enlightenment in crafting our fundamental law. Thus, the preamble of the present
Constitution reads:

We, the sovereign Filipino people, imploring the aid of Almighty God, in order to build a just and
humane society, and establish a Government that shall embody our ideals and aspirations, promote
the common good, conserve and develop our patrimony, and secure to ourselves and our posterity,
the blessings of independence and democracy under the rule of law and a regime of truth, justice,
freedom, love, equality, and peace, do ordain and promulgate this Constitution.

The Filipino people in "imploring the aid of Almighty God " manifested their spirituality innate in our
nature and consciousness as a people, shaped by tradition and historical experience. As this is
embodied in the preamble, it means that the State recognizes with respect the influence of religion in
so far as it instills into the mind the purest principles of morality.  Moreover, in recognition of the
205

contributions of religion to society, the 1935, 1973 and 1987 constitutions contain benevolent and
accommodating provisions towards religions such as tax exemption of church property, salary of
religious officers in government institutions, and optional religious instructions in public schools.

The Framers, however, felt the need to put up a strong barrier so that the State would not encroach
into the affairs of the church, and vice-versa. The principle of separation of Church and State was,
thus, enshrined in Article II, Section 6 of the 1987 Constitution, viz:

Section 6. The separation of Church and State shall be inviolable.

Verily, the principle of separation of Church and State is based on mutual respect.  Generally, the
1âwphi1

State cannot meddle in the internal affairs of the church, much less question its faith and dogmas or
dictate upon it. It cannot favor one religion and discriminate against another. On the other hand, the
church cannot impose its beliefs and convictions on the State and the rest of the citizenry. It cannot
demand that the nation follow its beliefs, even if it sincerely believes that they are good for the
country.
Consistent with the principle that not any one religion should ever be preferred over another, the
Constitution in the above-cited provision utilizes the term "church" in its generic sense, which refers
to a temple, a mosque, an iglesia, or any other house of God which metaphorically symbolizes a
religious organization. Thus, the "Church" means the religious congregations collectively.

Balancing the benefits that religion affords and the need to provide an ample barrier to protect the
State from the pursuit of its secular objectives, the Constitution lays down the following mandate in
Article III, Section 5 and Article VI, Section 29 (2), of the 1987 Constitution:

Section. 5. No law shall be made respecting an establishment of religion, or prohibiting the free
exercise thereof. The free exercise and enjoyment of religious profession and worship, without
discrimination or preference, shall forever be allowed. No religious test shall be required for the
exercise of civil or political rights.

Section 29.

xxx.

No public money or property shall be appropriated, applied, paid, or employed, directly or indirectly,
for the use, benefit, or support of any sect, church, denomination, sectarian institution, or system of
religion, or of any priest, preacher, minister, other religious teacher, or dignitary as such, except
when such priest, preacher, minister, or dignitary is assigned to the armed forces, or to any penal
institution, or government orphanage or leprosarium.

In short, the constitutional assurance of religious freedom provides two guarantees: the
Establishment Clause and the Free Exercise Clause.

The establishment clause "principally prohibits the State from sponsoring any religion or favoring any
religion as against other religions. It mandates a strict neutrality in affairs among religious
groups."  Essentially, it prohibits the establishment of a state religion and the use of public
206

resources for the support or prohibition of a religion.

On the other hand, the basis of the free exercise clause is the respect for the inviolability of the
human conscience.  Under this part of religious freedom guarantee, the State is prohibited from
207

unduly interfering with the outside manifestations of one's belief and faith.  Explaining the concept of
208

religious freedom, the Court, in Victoriano v. Elizalde Rope Workers Union  wrote:
209

The constitutional provisions not only prohibits legislation for the support of any religious tenets or
the modes of worship of any sect, thus forestalling compulsion by law of the acceptance of any
creed or the practice of any form of worship (U.S. Ballard, 322 U.S. 78, 88 L. ed. 1148, 1153), but
also assures the free exercise of one's chosen form of religion within limits of utmost amplitude. It
has been said that the religion clauses of the Constitution are all designed to protect the broadest
possible liberty of conscience, to allow each man to believe as his conscience directs, to profess his
beliefs, and to live as he believes he ought to live, consistent with the liberty of others and with the
common good. Any legislation whose effect or purpose is to impede the observance of one or all
religions, or to discriminate invidiously between the religions, is invalid, even though the burden may
be characterized as being only indirect. (Sherbert v. Verner, 374 U.S. 398, 10 L.ed.2d 965, 83 S. Ct.
1970) But if the state regulates conduct by enacting, within its power, a general law which has for its
purpose and effect to advance the state's secular goals, the statute is valid despite its indirect
burden on religious observance, unless the state can accomplish its purpose without imposing such
burden. (Braunfeld v. Brown, 366 U.S. 599, 6 Led. 2d. 563, 81 S. Ct. 144; McGowan v. Maryland,
366 U.S. 420, 444-5 and 449).
As expounded in Escritor,

The establishment and free exercise clauses were not designed to serve contradictory purposes.
They have a single goal-to promote freedom of individual religious beliefs and practices. In simplest
terms, the free exercise clause prohibits government from inhibiting religious beliefs with penalties
for religious beliefs and practice, while the establishment clause prohibits government from inhibiting
religious belief with rewards for religious beliefs and practices. In other words, the two religion
clauses were intended to deny government the power to use either the carrot or the stick to influence
individual religious beliefs and practices.
210

Corollary to the guarantee of free exercise of one's religion is the principle that the guarantee of
religious freedom is comprised of two parts: the freedom to believe, and the freedom to act on one's
belief. The first part is absolute. As explained in Gerona v. Secretary of Education: 211

The realm of belief and creed is infinite and limitless bounded only by one's imagination and thought.
So is the freedom of belief, including religious belief, limitless and without bounds. One may believe
in most anything, however strange, bizarre and unreasonable the same may appear to others, even
heretical when weighed in the scales of orthodoxy or doctrinal standards. But between the freedom
of belief and the exercise of said belief, there is quite a stretch of road to travel.
212

The second part however, is limited and subject to the awesome power of the State and can be
enjoyed only with proper regard to the rights of others. It is "subject to regulation where the belief is
translated into external acts that affect the public welfare."
213

Legislative Acts and the

Free Exercise Clause

Thus, in case of conflict between the free exercise clause and the State, the Court adheres to the
doctrine of benevolent neutrality. This has been clearly decided by the Court in Estrada v. Escritor,
(Escritor)  where it was stated "that benevolent neutrality-accommodation, whether mandatory or
214

permissive, is the spirit, intent and framework underlying the Philippine Constitution."  In the same
215

case, it was further explained that"

The benevolent neutrality theory believes that with respect to these governmental actions,
accommodation of religion may be allowed, not to promote the government's favored form of
religion, but to allow individuals and groups to exercise their religion without hindrance. "The purpose
of accommodation is to remove a burden on, or facilitate the exercise of, a person's or institution's
religion."  "What is sought under the theory of accommodation is not a declaration of
216

unconstitutionality of a facially neutral law, but an exemption from its application or its 'burdensome
effect,' whether by the legislature or the courts."
217

In ascertaining the limits of the exercise of religious freedom, the compelling state interest test is
proper. Underlying the compelling state interest test is the notion that free exercise is a fundamental
218

right and that laws burdening it should be subject to strict scrutiny.  In Escritor, it was written:
219

Philippine jurisprudence articulates several tests to determine these limits. Beginning with the first
case on the Free Exercise Clause, American Bible Society, the Court mentioned the "clear and
present danger" test but did not employ it. Nevertheless, this test continued to be cited in
subsequent cases on religious liberty. The Gerona case then pronounced that the test of
permissibility of religious freedom is whether it violates the established institutions of society and
law. The Victoriano case mentioned the "immediate and grave danger" test as well as the doctrine
that a law of general applicability may burden religious exercise provided the law is the least
restrictive means to accomplish the goal of the law. The case also used, albeit inappropriately, the
"compelling state interest" test. After Victoriano , German went back to the Gerona rule. Ebralinag
then employed the "grave and immediate danger" test and overruled the Gerona test. The fairly
recent case of Iglesia ni Cristo went back to the " clear and present danger" test in the maiden case
of A merican Bible Society. Not surprisingly, all the cases which employed the "clear and present
danger" or "grave and immediate danger" test involved, in one form or another, religious speech as
this test is often used in cases on freedom of expression. On the other hand, the Gerona and
German cases set the rule that religious freedom will not prevail over established institutions of
society and law. Gerona, however, which was the authority cited by German has been overruled by
Ebralinag which employed the "grave and immediate danger" test . Victoriano was the only case that
employed the "compelling state interest" test, but as explained previously, the use of the test was
inappropriate to the facts of the case.

The case at bar does not involve speech as in A merican Bible Society, Ebralinag and Iglesia ni
Cristo where the "clear and present danger" and "grave and immediate danger" tests were
appropriate as speech has easily discernible or immediate effects. The Gerona and German
doctrine, aside from having been overruled, is not congruent with the benevolent neutrality
approach, thus not appropriate in this jurisdiction. Similar to Victoriano, the present case involves
purely conduct arising from religious belief. The "compelling state interest" test is proper where
conduct is involved for the whole gamut of human conduct has different effects on the state's
interests: some effects may be immediate and short-term while others delayed and far-reaching. A
test that would protect the interests of the state in preventing a substantive evil, whether immediate
or delayed, is therefore necessary. However, not any interest of the state would suffice to prevail
over the right to religious freedom as this is a fundamental right that enjoys a preferred position in
the hierarchy of rights - "the most inalienable and sacred of all human rights", in the words of
Jefferson. This right is sacred for an invocation of the Free Exercise Clause is an appeal to a higher
sovereignty. The entire constitutional order of limited government is premised upon an
acknowledgment of such higher sovereignty, thus the Filipinos implore the "aid of Almighty God in
order to build a just and humane society and establish a government." As held in Sherbert, only the
gravest abuses, endangering paramount interests can limit this fundamental right. A mere balancing
of interests which balances a right with just a colorable state interest is therefore not appropriate.
Instead, only a compelling interest of the state can prevail over the fundamental right to religious
liberty. The test requires the state to carry a heavy burden, a compelling one, for to do otherwise
would allow the state to batter religion, especially the less powerful ones until they are destroyed. In
determining which shall prevail between the state's interest and religious liberty, reasonableness
shall be the guide. The "compelling state interest" serves the purpose of revering religious liberty
while at the same time affording protection to the paramount interests of the state. This was the test
used in Sherbert which involved conduct, i.e. refusal to work on Saturdays. In the end, the
"compelling state interest" test, by upholding the paramount interests of the state, seeks to protect
the very state, without which, religious liberty will not be preserved. [Emphases in the original.
Underlining supplied.]

The Court's Position

In the case at bench, it is not within the province of the Court to determine whether the use of
contraceptives or one's participation in the support of modem reproductive health measures is moral
from a religious standpoint or whether the same is right or wrong according to one's dogma or belief.
For the Court has declared that matters dealing with "faith, practice, doctrine, form of worship,
ecclesiastical law, custom and rule of a church ... are unquestionably ecclesiastical matters which
are outside the province of the civil courts."  The jurisdiction of the Court extends only to public and
220

secular morality. Whatever pronouncement the Court makes in the case at bench should be
understood only in this realm where it has authority. Stated otherwise, while the Court stands without
authority to rule on ecclesiastical matters, as vanguard of the Constitution, it does have authority to
determine whether the RH Law contravenes the guarantee of religious freedom.

At first blush, it appears that the RH Law recognizes and respects religion and religious beliefs and
convictions. It is replete with assurances the no one can be compelled to violate the tenets of his
religion or defy his religious convictions against his free will. Provisions in the RH Law respecting
religious freedom are the following:

1. The State recognizes and guarantees the human rights of all persons including their right to
equality and nondiscrimination of these rights, the right to sustainable human development, the right
to health which includes reproductive health, the right to education and information, and the right to
choose and make decisions for themselves in accordance with their religious convictions, ethics,
cultural beliefs, and the demands of responsible parenthood. [Section 2, Declaration of Policy]

2 . The State recognizes marriage as an inviolable social institution and the foundation of the family
which in turn is the foundation of the nation. Pursuant thereto, the State shall defend:

(a) The right of spouses to found a family in accordance with their religious convictions and the
demands of responsible parenthood." [Section 2, Declaration of Policy]

3. The State shall promote and provide information and access, without bias, to all methods of family
planning, including effective natural and modern methods which have been proven medically safe,
legal, non-abortifacient, and effective in accordance with scientific and evidence-based medical
research standards such as those registered and approved by the FDA for the poor and
marginalized as identified through the NHTS-PR and other government measures of identifying
marginalization: Provided, That the State shall also provide funding support to promote modern
natural methods of family planning, especially the Billings Ovulation Method, consistent with the
needs of acceptors and their religious convictions. [Section 3(e), Declaration of Policy]

4. The State shall promote programs that: (1) enable individuals and couples to have the number of
children they desire with due consideration to the health, particularly of women, and the resources
available and affordable to them and in accordance with existing laws, public morals and their
religious convictions. [Section 3CDJ

5. The State shall respect individuals' preferences and choice of family planning methods that are in
accordance with their religious convictions and cultural beliefs, taking into consideration the State's
obligations under various human rights instruments. [Section 3(h)]

6. Active participation by nongovernment organizations (NGOs) , women's and people's


organizations, civil society, faith-based organizations, the religious sector and communities is crucial
to ensure that reproductive health and population and development policies, plans, and programs
will address the priority needs of women, the poor, and the marginalized. [Section 3(i)]

7. Responsible parenthood refers to the will and ability of a parent to respond to the needs and
aspirations of the family and children. It is likewise a shared responsibility between parents to
determine and achieve the desired number of children, spacing and timing of their children according
to their own family life aspirations, taking into account psychological preparedness, health status,
sociocultural and economic concerns consistent with their religious convictions. [Section 4(v)]
(Emphases supplied)
While the Constitution prohibits abortion, laws were enacted allowing the use of contraceptives. To
some medical practitioners, however, the whole idea of using contraceptives is an anathema.
Consistent with the principle of benevolent neutrality, their beliefs should be respected.

The Establishment Clause

and Contraceptives

In the same breath that the establishment clause restricts what the government can do with religion,
it also limits what religious sects can or cannot do with the government. They can neither cause the
government to adopt their particular doctrines as policy for everyone, nor can they not cause the
government to restrict other groups. To do so, in simple terms, would cause the State to adhere to a
particular religion and, thus, establishing a state religion.

Consequently, the petitioners are misguided in their supposition that the State cannot enhance its
population control program through the RH Law simply because the promotion of contraceptive use
is contrary to their religious beliefs. Indeed, the State is not precluded to pursue its legitimate secular
objectives without being dictated upon by the policies of any one religion. One cannot refuse to pay
his taxes simply because it will cloud his conscience. The demarcation line between Church and
State demands that one render unto Caesar the things that are Caesar's and unto God the things
that are God's.221

The Free Exercise Clause and the Duty to Refer

While the RH Law, in espousing state policy to promote reproductive health manifestly respects
diverse religious beliefs in line with the Non-Establishment Clause, the same conclusion cannot be
reached with respect to Sections 7, 23 and 24 thereof. The said provisions commonly mandate that
a hospital or a medical practitioner to immediately refer a person seeking health care and services
under the law to another accessible healthcare provider despite their conscientious objections based
on religious or ethical beliefs.

In a situation where the free exercise of religion is allegedly burdened by government legislation or
practice, the compelling state interest test in line with the Court's espousal of the Doctrine of
Benevolent Neutrality in Escritor, finds application. In this case, the conscientious objector's claim to
religious freedom would warrant an exemption from obligations under the RH Law, unless the
government succeeds in demonstrating a more compelling state interest in the accomplishment of
an important secular objective. Necessarily so, the plea of conscientious objectors for exemption
from the RH Law deserves no less than strict scrutiny.

In applying the test, the first inquiry is whether a conscientious objector's right to religious freedom
has been burdened. As in Escritor, there is no doubt that an intense tug-of-war plagues a
conscientious objector. One side coaxes him into obedience to the law and the abandonment of his
religious beliefs, while the other entices him to a clean conscience yet under the pain of penalty. The
scenario is an illustration of the predicament of medical practitioners whose religious beliefs are
incongruent with what the RH Law promotes.

The Court is of the view that the obligation to refer imposed by the RH Law violates the religious
belief and conviction of a conscientious objector. Once the medical practitioner, against his will,
refers a patient seeking information on modem reproductive health products, services, procedures
and methods, his conscience is immediately burdened as he has been compelled to perform an act
against his beliefs. As Commissioner Joaquin A. Bernas (Commissioner Bernas) has written, "at the
basis of the free exercise clause is the respect for the inviolability of the human conscience. 222
Though it has been said that the act of referral is an opt-out clause, it is, however, a false
compromise because it makes pro-life health providers complicit in the performance of an act that
they find morally repugnant or offensive. They cannot, in conscience, do indirectly what they cannot
do directly. One may not be the principal, but he is equally guilty if he abets the offensive act by
indirect participation.

Moreover, the guarantee of religious freedom is necessarily intertwined with the right to free speech,
it being an externalization of one's thought and conscience. This in turn includes the right to be
silent. With the constitutional guarantee of religious freedom follows the protection that should be
afforded to individuals in communicating their beliefs to others as well as the protection for simply
being silent. The Bill of Rights guarantees the liberty of the individual to utter what is in his mind and
the liberty not to utter what is not in his mind.  While the RH Law seeks to provide freedom of choice
223

through informed consent, freedom of choice guarantees the liberty of the religious conscience and
prohibits any degree of compulsion or burden, whether direct or indirect, in the practice of one's
religion.
224

In case of conflict between the religious beliefs and moral convictions of individuals, on one hand,
and the interest of the State, on the other, to provide access and information on reproductive health
products, services, procedures and methods to enable the people to determine the timing, number
and spacing of the birth of their children, the Court is of the strong view that the religious freedom of
health providers, whether public or private, should be accorded primacy. Accordingly, a
conscientious objector should be exempt from compliance with the mandates of the RH Law. If he
would be compelled to act contrary to his religious belief and conviction, it would be violative of "the
principle of non-coercion" enshrined in the constitutional right to free exercise of religion.

Interestingly, on April 24, 2013, Scotland's Inner House of the Court of Session, found in the case of
Doogan and Wood v. NHS Greater Glasgow and Clyde Health Board,  that the midwives claiming to
225

be conscientious objectors under the provisions of Scotland's Abortion Act of 1967, could not be
required to delegate, supervise or support staff on their labor ward who were involved in
abortions.  The Inner House stated "that if 'participation' were defined according to whether the
226

person was taking part 'directly' or ' indirectly' this would actually mean more complexity and
uncertainty." 227

While the said case did not cover the act of referral, the applicable principle was the same - they
could not be forced to assist abortions if it would be against their conscience or will.

Institutional Health Providers

The same holds true with respect to non-maternity specialty hospitals and hospitals owned and
operated by a religious group and health care service providers. Considering that Section 24 of the
RH Law penalizes such institutions should they fail or refuse to comply with their duty to refer under
Section 7 and Section 23(a)(3), the Court deems that it must be struck down for being violative of the
freedom of religion. The same applies to Section 23(a)(l) and (a)(2) in relation to Section 24,
considering that in the dissemination of information regarding programs and services and in the
performance of reproductive health procedures, the religious freedom of health care service
providers should be respected.

In the case of Islamic Da'wah Council of the Philippines, Inc. v. Office of the Executive Secretary  it
228

was stressed:

Freedom of religion was accorded preferred status by the framers of our fundamental law. And this
Court has consistently affirmed this preferred status, well aware that it is "designed to protect the
broadest possible liberty of conscience, to allow each man to believe as his conscience directs, to
profess his beliefs, and to live as he believes he ought to live, consistent with the liberty of others
and with the common good." 10

The Court is not oblivious to the view that penalties provided by law endeavour to ensure
compliance. Without set consequences for either an active violation or mere inaction, a law tends to
be toothless and ineffectual. Nonetheless, when what is bartered for an effective implementation of a
law is a constitutionally-protected right the Court firmly chooses to stamp its disapproval. The
punishment of a healthcare service provider, who fails and/or refuses to refer a patient to another, or
who declines to perform reproductive health procedure on a patient because incompatible religious
beliefs, is a clear inhibition of a constitutional guarantee which the Court cannot allow.

The Implementing Rules and Regulation (RH-IRR)

The last paragraph of Section 5.24 of the RH-IRR reads:

Provided, That skilled health professional such as provincial, city or municipal health officers, chiefs
of hospital, head nurses, supervising midwives, among others, who by virtue of their office are
specifically charged with the duty to implement the provisions of the RPRH Act and these Rules,
cannot be considered as conscientious objectors.

This is discriminatory and violative of the equal protection clause. The conscientious objection
clause should be equally protective of the religious belief of public health officers. There is no
perceptible distinction why they should not be considered exempt from the mandates of the law. The
protection accorded to other conscientious objectors should equally apply to all medical practitioners
without distinction whether they belong to the public or private sector. After all, the freedom to
believe is intrinsic in every individual and the protective robe that guarantees its free exercise is not
taken off even if one acquires employment in the government.

It should be stressed that intellectual liberty occupies a place inferior to none in the hierarchy of
human values. The mind must be free to think what it wills, whether in the secular or religious
sphere, to give expression to its beliefs by oral discourse or through the media and, thus, seek other
candid views in occasions or gatherings or in more permanent aggrupation. Embraced in such
concept then are freedom of religion, freedom of speech, of the press, assembly and petition, and
freedom of association. 229

The discriminatory provision is void not only because no such exception is stated in the RH Law
itself but also because it is violative of the equal protection clause in the Constitution. Quoting
respondent Lagman, if there is any conflict between the RH-IRR and the RH Law, the law must
prevail.

Justice Mendoza:

I'll go to another point. The RH law .. .in your Comment- in-Intervention on page 52, you mentioned
RH Law is replete with provisions in upholding the freedom of religion and respecting religious
convictions. Earlier, you affirmed this with qualifications. Now, you have read, I presumed you have
read the IRR-Implementing Rules and Regulations of the RH Bill?

Congressman Lagman:
Yes, Your Honor, I have read but I have to admit, it's a long IRR and I have not thoroughly dissected
the nuances of the provisions.

Justice Mendoza:

I will read to you one provision. It's Section 5.24. This I cannot find in the RH Law. But in the IRR it
says: " .... skilled health professionals such as provincial, city or municipal health officers, chief of
hospitals, head nurses, supervising midwives, among others, who by virtue of their office are
specifically charged with the duty to implement the provisions of the RPRH Act and these Rules,
cannot be considered as conscientious objectors." Do you agree with this?

Congressman Lagman:

I will have to go over again the provisions, Your Honor.

Justice Mendoza:

In other words, public health officers in contrast to the private practitioners who can be conscientious
objectors, skilled health professionals cannot be considered conscientious objectors. Do you agree
with this? Is this not against the constitutional right to the religious belief?

Congressman Lagman:

Your Honor, if there is any conflict between the IRR and the law, the law must prevail. 230

Compelling State Interest

The foregoing discussion then begets the question on whether the respondents, in defense of the
subject provisions, were able to: 1] demonstrate a more compelling state interest to restrain
conscientious objectors in their choice of services to render; and 2] discharge the burden of proof
that the obligatory character of the law is the least intrusive means to achieve the objectives of the
law.

Unfortunately, a deep scrutiny of the respondents' submissions proved to be in vain. The OSG was
curiously silent in the establishment of a more compelling state interest that would rationalize the
curbing of a conscientious objector's right not to adhere to an action contrary to his religious
convictions. During the oral arguments, the OSG maintained the same silence and evasion. The
Transcripts of the Stenographic Notes disclose the following:

Justice De Castro:

Let's go back to the duty of the conscientious objector to refer. ..

Senior State Solicitor Hilbay:

Yes, Justice.

Justice De Castro:
... which you are discussing awhile ago with Justice Abad. What is the compelling State interest in
imposing this duty to refer to a conscientious objector which refuses to do so because of his religious
belief?

Senior State Solicitor Hilbay:

Ahh, Your Honor, ..

Justice De Castro:

What is the compelling State interest to impose this burden?

Senior State Solicitor Hilbay:

In the first place, Your Honor, I don't believe that the standard is a compelling State interest, this is
an ordinary health legislation involving professionals. This is not a free speech matter or a pure free
exercise matter. This is a regulation by the State of the relationship between medical doctors and
their patients.
231

Resultantly, the Court finds no compelling state interest which would limit the free exercise clause of
the conscientious objectors, however few in number. Only the prevention of an immediate and grave
danger to the security and welfare of the community can justify the infringement of religious freedom.
If the government fails to show the seriousness and immediacy of the threat, State intrusion is
constitutionally unacceptable. 232

Freedom of religion means more than just the freedom to believe. It also means the freedom to act
or not to act according to what one believes. And this freedom is violated when one is compelled to
act against one's belief or is prevented from acting according to one's belief.233

Apparently, in these cases, there is no immediate danger to the life or health of an individual in the
perceived scenario of the subject provisions. After all, a couple who plans the timing, number and
spacing of the birth of their children refers to a future event that is contingent on whether or not the
mother decides to adopt or use the information, product, method or supply given to her or whether
she even decides to become pregnant at all. On the other hand, the burden placed upon those who
object to contraceptive use is immediate and occurs the moment a patient seeks consultation on
reproductive health matters.

Moreover, granting that a compelling interest exists to justify the infringement of the conscientious
objector's religious freedom, the respondents have failed to demonstrate "the gravest abuses,
endangering paramount interests" which could limit or override a person's fundamental right to
religious freedom. Also, the respondents have not presented any government effort exerted to show
that the means it takes to achieve its legitimate state objective is the least intrusive means.  Other
234

than the assertion that the act of referring would only be momentary, considering that the act of
referral by a conscientious objector is the very action being contested as violative of religious
freedom, it behooves the respondents to demonstrate that no other means can be undertaken by the
State to achieve its objective without violating the rights of the conscientious objector. The health
concerns of women may still be addressed by other practitioners who may perform reproductive
health-related procedures with open willingness and motivation. Suffice it to say, a person who is
forced to perform an act in utter reluctance deserves the protection of the Court as the last vanguard
of constitutional freedoms.
At any rate, there are other secular steps already taken by the Legislature to ensure that the right to
health is protected. Considering other legislations as they stand now, R.A . No. 4 729 or the
Contraceptive Act, R.A. No. 6365 or "The Population Act of the Philippines" and R.A. No. 9710,
otherwise known as "The Magna Carta of Women," amply cater to the needs of women in relation to
health services and programs. The pertinent provision of Magna Carta on comprehensive health
services and programs for women, in fact, reads:

Section 17. Women's Right to Health. - (a) Comprehensive Health Services. - The State shall, at all
times, provide for a comprehensive, culture-sensitive, and gender-responsive health services and
programs covering all stages of a woman's life cycle and which addresses the major causes of
women's mortality and morbidity: Provided, That in the provision for comprehensive health services,
due respect shall be accorded to women's religious convictions, the rights of the spouses to found a
family in accordance with their religious convictions, and the demands of responsible parenthood,
and the right of women to protection from hazardous drugs, devices, interventions, and substances.

Access to the following services shall be ensured:

(1) Maternal care to include pre- and post-natal services to address pregnancy and
infant health and nutrition;

(2) Promotion of breastfeeding;

(3) Responsible, ethical, legal, safe, and effective methods of family planning;

(4) Family and State collaboration in youth sexuality education and health services
without prejudice to the primary right and duty of parents to educate their children;

(5) Prevention and management of reproductive tract infections, including sexually


transmitted diseases, HIV, and AIDS;

(6) Prevention and management of reproductive tract cancers like breast and cervical
cancers, and other gynecological conditions and disorders;

(7) Prevention of abortion and management of pregnancy-related complications;

(8) In cases of violence against women and children, women and children victims
and survivors shall be provided with comprehensive health services that include
psychosocial, therapeutic, medical, and legal interventions and assistance towards
healing, recovery, and empowerment;

(9) Prevention and management of infertility and sexual dysfunction pursuant to


ethical norms and medical standards;

(10) Care of the elderly women beyond their child-bearing years; and

(11) Management, treatment, and intervention of mental health problems of women


and girls. In addition, healthy lifestyle activities are encouraged and promoted
through programs and projects as strategies in the prevention of diseases.

(b) Comprehensive Health Information and Education. - The State shall provide women in all sectors
with appropriate, timely, complete, and accurate information and education on all the above-stated
aspects of women's health in government education and training programs, with due regard to the
following:

(1) The natural and primary right and duty of parents in the rearing of the youth and
the development of moral character and the right of children to be brought up in an
atmosphere of morality and rectitude for the enrichment and strengthening of
character;

(2) The formation of a person's sexuality that affirms human dignity; and

(3) Ethical, legal, safe, and effective family planning methods including fertility
awareness.

As an afterthought, Asst. Solicitor General Hilbay eventually replied that the compelling state interest
was "Fifteen maternal deaths per day, hundreds of thousands of unintended pregnancies, lives
changed, x x x."  He, however, failed to substantiate this point by concrete facts and figures from
235

reputable sources.

The undisputed fact, however, is that the World Health Organization reported that the Filipino
maternal mortality rate dropped to 48 percent from 1990 to 2008,   although there was still no RH
236

Law at that time. Despite such revelation, the proponents still insist that such number of maternal
deaths constitute a compelling state interest.

Granting that there are still deficiencies and flaws in the delivery of social healthcare programs for
Filipino women, they could not be solved by a measure that puts an unwarrantable stranglehold on
religious beliefs in exchange for blind conformity.

Exception: Life Threatening Cases

All this notwithstanding, the Court properly recognizes a valid exception set forth in the law. While
generally healthcare service providers cannot be forced to render reproductive health care
procedures if doing it would contravene their religious beliefs, an exception must be made in life-
threatening cases that require the performance of emergency procedures. In these situations, the
right to life of the mother should be given preference, considering that a referral by a medical
practitioner would amount to a denial of service, resulting to unnecessarily placing the life of a
mother in grave danger. Thus, during the oral arguments, Atty. Liban, representing CFC, manifested:
"the forced referral clause that we are objecting on grounds of violation of freedom of religion does
not contemplate an emergency." 237

In a conflict situation between the life of the mother and the life of a child, the doctor is morally
obliged always to try to save both lives. If, however, it is impossible, the resulting death to one
should not be deliberate. Atty. Noche explained:

Principle of Double-Effect. - May we please remind the principal author of the RH Bill in the House of
Representatives of the principle of double-effect wherein intentional harm on the life of either the
mother of the child is never justified to bring about a "good" effect. In a conflict situation between the
life of the child and the life of the mother, the doctor is morally obliged always to try to save both
lives. However, he can act in favor of one (not necessarily the mother) when it is medically
impossible to save both, provided that no direct harm is intended to the other. If the above principles
are observed, the loss of the child's life or the mother's life is not intentional and, therefore,
unavoidable. Hence, the doctor would not be guilty of abortion or murder. The mother is never pitted
against the child because both their lives are equally valuable. 238

Accordingly, if it is necessary to save the life of a mother, procedures endangering the life of the
child may be resorted to even if is against the religious sentiments of the medical practitioner. As
quoted above, whatever burden imposed upon a medical practitioner in this case would have been
more than justified considering the life he would be able to save.

Family Planning Seminars

Anent the requirement imposed under Section 15  as a condition for the issuance of a marriage
239

license, the Court finds the same to be a reasonable exercise of police power by the government. A
cursory reading of the assailed provision bares that the religious freedom of the petitioners is not at
all violated. All the law requires is for would-be spouses to attend a seminar on parenthood, family
planning breastfeeding and infant nutrition. It does not even mandate the type of family planning
methods to be included in the seminar, whether they be natural or artificial. As correctly noted by the
OSG, those who receive any information during their attendance in the required seminars are not
compelled to accept the information given to them, are completely free to reject the information they
find unacceptable, and retain the freedom to decide on matters of family life without the intervention
of the State.

4-The Family and the Right to Privacy

Petitioner CFC assails the RH Law because Section 23(a) (2) (i) thereof violates the provisions of
the Constitution by intruding into marital privacy and autonomy. It argues that it cultivates disunity
and fosters animosity in the family rather than promote its solidarity and total development. 240

The Court cannot but agree.

The 1987 Constitution is replete with provisions strengthening the family as it is the basic social
institution. In fact, one article, Article XV, is devoted entirely to the family.

ARTICLE XV
THE FAMILY

Section 1. The State recognizes the Filipino family as the foundation of the nation. Accordingly, it
shall strengthen its solidarity and actively promote its total development.

Section 2. Marriage, as an inviolable social institution, is the foundation of the family and shall be
protected by the State.

Section 3. The State shall defend:

The right of spouses to found a family in accordance with their religious convictions and the
demands of responsible parenthood;

The right of children to assistance, including proper care and nutrition, and special protection from all
forms of neglect, abuse, cruelty, exploitation and other conditions prejudicial to their development;

The right of the family to a family living wage and income; and
The right of families or family assoc1at1ons to participate in the planning and implementation of
policies and programs that affect them.

In this case, the RH Law, in its not-so-hidden desire to control population growth, contains provisions
which tend to wreck the family as a solid social institution. It bars the husband and/or the father from
participating in the decision making process regarding their common future progeny. It likewise
deprives the parents of their authority over their minor daughter simply because she is already a
parent or had suffered a miscarriage.

The Family and Spousal Consent

Section 23(a) (2) (i) of the RH Law states:

The following acts are prohibited:

(a) Any health care service provider, whether public or private, who shall: ...

(2) refuse to perform legal and medically-safe reproductive health procedures on any person of legal
age on the ground of lack of consent or authorization of the following persons in the following
instances:

(i) Spousal consent in case of married persons: provided, That in case of disagreement, the decision
of the one undergoing the procedures shall prevail. [Emphasis supplied]

The above provision refers to reproductive health procedures like tubal litigation and vasectomy
which, by their very nature, should require mutual consent and decision between the husband and
the wife as they affect issues intimately related to the founding of a family. Section 3, Art. XV of the
Constitution espouses that the State shall defend the "right of the spouses to found a family." One
person cannot found a family. The right, therefore, is shared by both spouses. In the same Section
3, their right "to participate in the planning and implementation of policies and programs that affect
them " is equally recognized.

The RH Law cannot be allowed to infringe upon this mutual decision-making. By giving absolute
authority to the spouse who would undergo a procedure, and barring the other spouse from
participating in the decision would drive a wedge between the husband and wife, possibly result in
bitter animosity, and endanger the marriage and the family, all for the sake of reducing the
population. This would be a marked departure from the policy of the State to protect marriage as an
inviolable social institution.
241

Decision-making involving a reproductive health procedure is a private matter which belongs to the
couple, not just one of them. Any decision they would reach would affect their future as a family
because the size of the family or the number of their children significantly matters. The decision
whether or not to undergo the procedure belongs exclusively to, and shared by, both spouses as
one cohesive unit as they chart their own destiny. It is a constitutionally guaranteed private right.
Unless it prejudices the State, which has not shown any compelling interest, the State should see to
it that they chart their destiny together as one family.

As highlighted by Justice Leonardo-De Castro, Section 19( c) of R.A. No. 9710, otherwise known as
the "Magna Carta for Women," provides that women shall have equal rights in all matters relating to
marriage and family relations, including the joint decision on the number and spacing of their
children. Indeed, responsible parenthood, as Section 3(v) of the RH Law states, is a shared
responsibility between parents. Section 23(a)(2)(i) of the RH Law should not be allowed to betray the
constitutional mandate to protect and strengthen the family by giving to only one spouse the
absolute authority to decide whether to undergo reproductive health procedure. 242

The right to chart their own destiny together falls within the protected zone of marital privacy and
such state intervention would encroach into the zones of spousal privacy guaranteed by the
Constitution. In our jurisdiction, the right to privacy was first recognized in Marje v. Mutuc,  where
243

the Court, speaking through Chief Justice Fernando, held that "the right to privacy as such is
accorded recognition independently of its identification with liberty; in itself, it is fully deserving of
constitutional protection."  Marje adopted the ruling of the US Supreme Court in Griswold v.
244

Connecticut,  where Justice William O. Douglas wrote:


245

We deal with a right of privacy older than the Bill of Rights -older than our political parties, older than
our school system. Marriage is a coming together for better or for worse, hopefully enduring, and
intimate to the degree of being sacred. It is an association that promotes a way of life, not causes; a
harmony in living, not political faiths; a bilateral loyalty, not commercial or social projects. Yet it is an
association for as noble a purpose as any involved in our prior decisions.

Ironically, Griswold invalidated a Connecticut statute which made the use of contraceptives a
criminal offense on the ground of its amounting to an unconstitutional invasion of the right to privacy
of married persons. Nevertheless, it recognized the zone of privacy rightfully enjoyed by couples.
Justice Douglas in Grisworld wrote that "specific guarantees in the Bill of Rights have penumbras,
formed by emanations from those guarantees that help give them life and substance. Various
guarantees create zones of privacy." 246

At any rate, in case of conflict between the couple, the courts will decide.

The Family and Parental Consent

Equally deplorable is the debarment of parental consent in cases where the minor, who will be
undergoing a procedure, is already a parent or has had a miscarriage. Section 7 of the RH law
provides:

SEC. 7. Access to Family Planning. – x x x.

No person shall be denied information and access to family planning services, whether natural or
artificial: Provided, That minors will not be allowed access to modern methods of family planning
without written consent from their parents or guardian/s except when the minor is already a parent or
has had a miscarriage.

There can be no other interpretation of this provision except that when a minor is already a parent or
has had a miscarriage, the parents are excluded from the decision making process of the minor with
regard to family planning. Even if she is not yet emancipated, the parental authority is already cut off
just because there is a need to tame population growth.

It is precisely in such situations when a minor parent needs the comfort, care, advice, and guidance
of her own parents. The State cannot replace her natural mother and father when it comes to
providing her needs and comfort. To say that their consent is no longer relevant is clearly anti-family.
It does not promote unity in the family. It is an affront to the constitutional mandate to protect and
strengthen the family as an inviolable social institution.
More alarmingly, it disregards and disobeys the constitutional mandate that "the natural and primary
right and duty of parents in the rearing of the youth for civic efficiency and the development of moral
character shall receive the support of the Government."  In this regard, Commissioner Bernas
247

wrote:

The 1987 provision has added the adjective "primary" to modify the right of parents. It imports the
assertion that the right of parents is superior to that of the State.  [Emphases supplied]
248

To insist on a rule that interferes with the right of parents to exercise parental control over their
minor-child or the right of the spouses to mutually decide on matters which very well affect the very
purpose of marriage, that is, the establishment of conjugal and family life, would result in the
violation of one's privacy with respect to his family. It would be dismissive of the unique and strongly-
held Filipino tradition of maintaining close family ties and violative of the recognition that the State
affords couples entering into the special contract of marriage to as one unit in forming the foundation
of the family and society.

The State cannot, without a compelling state interest, take over the role of parents in the care and
custody of a minor child, whether or not the latter is already a parent or has had a miscarriage. Only
a compelling state interest can justify a state substitution of their parental authority.

First Exception: Access to Information

Whether with respect to the minor referred to under the exception provided in the second paragraph
of Section 7 or with respect to the consenting spouse under Section 23(a)(2)(i), a distinction must be
made. There must be a differentiation between access to information about family planning services,
on one hand, and access to the reproductive health procedures and modern family planning
methods themselves, on the other. Insofar as access to information is concerned, the Court finds no
constitutional objection to the acquisition of information by the minor referred to under the exception
in the second paragraph of Section 7 that would enable her to take proper care of her own body and
that of her unborn child. After all, Section 12, Article II of the Constitution mandates the State to
protect both the life of the mother as that of the unborn child. Considering that information to enable
a person to make informed decisions is essential in the protection and maintenance of ones' health,
access to such information with respect to reproductive health must be allowed. In this situation, the
fear that parents might be deprived of their parental control is unfounded because they are not
prohibited to exercise parental guidance and control over their minor child and assist her in deciding
whether to accept or reject the information received.

Second Exception: Life Threatening Cases

As in the case of the conscientious objector, an exception must be made in life-threatening cases
that require the performance of emergency procedures. In such cases, the life of the minor who has
already suffered a miscarriage and that of the spouse should not be put at grave risk simply for lack
of consent. It should be emphasized that no person should be denied the appropriate medical care
urgently needed to preserve the primordial right, that is, the right to life.

In this connection, the second sentence of Section 23(a)(2)(ii)  should be struck down. By effectively
249

limiting the requirement of parental consent to "only in elective surgical procedures," it denies the
parents their right of parental authority in cases where what is involved are "non-surgical
procedures." Save for the two exceptions discussed above, and in the case of an abused child as
provided in the first sentence of Section 23(a)(2)(ii), the parents should not be deprived of their
constitutional right of parental authority. To deny them of this right would be an affront to the
constitutional mandate to protect and strengthen the family.
5 - Academic Freedom

It is asserted that Section 14 of the RH Law, in relation to Section 24 thereof, mandating the
teaching of Age-and Development-Appropriate Reproductive Health Education under threat of fine
and/or imprisonment violates the principle of academic freedom . According to the petitioners, these
provisions effectively force educational institutions to teach reproductive health education even if
they believe that the same is not suitable to be taught to their students.  Citing various studies
250

conducted in the United States and statistical data gathered in the country, the petitioners aver that
the prevalence of contraceptives has led to an increase of out-of-wedlock births; divorce and
breakdown of families; the acceptance of abortion and euthanasia; the "feminization of poverty"; the
aging of society; and promotion of promiscuity among the youth. 251

At this point, suffice it to state that any attack on the validity of Section 14 of the RH Law is
premature because the Department of Education, Culture and Sports has yet to formulate a
curriculum on age-appropriate reproductive health education. One can only speculate on the
content, manner and medium of instruction that will be used to educate the adolescents and whether
they will contradict the religious beliefs of the petitioners and validate their apprehensions. Thus,
considering the premature nature of this particular issue, the Court declines to rule on its
constitutionality or validity.

At any rate, Section 12, Article II of the 1987 Constitution provides that the natural and primary right
and duty of parents in the rearing of the youth for civic efficiency and development of moral
character shall receive the support of the Government. Like the 1973 Constitution and the 1935
Constitution, the 1987 Constitution affirms the State recognition of the invaluable role of parents in
preparing the youth to become productive members of society. Notably, it places more importance
on the role of parents in the development of their children by recognizing that said role shall be
"primary," that is, that the right of parents in upbringing the youth is superior to that of the State. 252

It is also the inherent right of the State to act as parens patriae to aid parents in the moral
development of the youth. Indeed, the Constitution makes mention of the importance of developing
the youth and their important role in nation building.  Considering that Section 14 provides not only
253

for the age-appropriate-reproductive health education, but also for values formation; the
development of knowledge and skills in self-protection against discrimination; sexual abuse and
violence against women and children and other forms of gender based violence and teen pregnancy;
physical, social and emotional changes in adolescents; women's rights and children's rights;
responsible teenage behavior; gender and development; and responsible parenthood, and that Rule
10, Section 11.01 of the RH-IRR and Section 4(t) of the RH Law itself provides for the teaching of
responsible teenage behavior, gender sensitivity and physical and emotional changes among
adolescents - the Court finds that the legal mandate provided under the assailed provision
supplements, rather than supplants, the rights and duties of the parents in the moral development of
their children.

Furthermore, as Section 14 also mandates that the mandatory reproductive health education
program shall be developed in conjunction with parent-teacher-community associations, school
officials and other interest groups, it could very well be said that it will be in line with the religious
beliefs of the petitioners. By imposing such a condition, it becomes apparent that the petitioners'
contention that Section 14 violates Article XV, Section 3(1) of the Constitution is without merit.254

While the Court notes the possibility that educators might raise their objection to their participation in
the reproductive health education program provided under Section 14 of the RH Law on the ground
that the same violates their religious beliefs, the Court reserves its judgment should an actual case
be filed before it.
6 - Due Process

The petitioners contend that the RH Law suffers from vagueness and, thus violates the due process
clause of the Constitution. According to them, Section 23 (a)(l) mentions a "private health service
provider" among those who may be held punishable but does not define who is a "private health
care service provider." They argue that confusion further results since Section 7 only makes
reference to a "private health care institution."

The petitioners also point out that Section 7 of the assailed legislation exempts hospitals operated by
religious groups from rendering reproductive health service and modern family planning methods. It
is unclear, however, if these institutions are also exempt from giving reproductive health information
under Section 23(a)(l), or from rendering reproductive health procedures under Section 23(a)(2).

Finally, it is averred that the RH Law punishes the withholding, restricting and providing of incorrect
information, but at the same time fails to define "incorrect information."

The arguments fail to persuade.

A statute or act suffers from the defect of vagueness when it lacks comprehensible standards that
men of common intelligence must necessarily guess its meaning and differ as to its application. It is
repugnant to the Constitution in two respects: (1) it violates due process for failure to accord
persons, especially the parties targeted by it, fair notice of the conduct to avoid; and (2) it leaves law
enforcers unbridled discretion in carrying out its provisions and becomes an arbitrary flexing of the
Government muscle.  Moreover, in determining whether the words used in a statute are vague,
255

words must not only be taken in accordance with their plain meaning alone, but also in relation to
other parts of the statute. It is a rule that every part of the statute must be interpreted with reference
to the context, that is, every part of it must be construed together with the other parts and kept
subservient to the general intent of the whole enactment. 256

As correctly noted by the OSG, in determining the definition of "private health care service provider,"
reference must be made to Section 4(n) of the RH Law which defines a "public health service
provider," viz:

(n) Public health care service provider refers to: (1) public health care institution, which is duly
licensed and accredited and devoted primarily to the maintenance and operation of facilities for
health promotion, disease prevention, diagnosis, treatment and care of individuals suffering from
illness, disease, injury, disability or deformity, or in need of obstetrical or other medical and nursing
care; (2) public health care professional, who is a doctor of medicine, a nurse or a midvvife; (3)
public health worker engaged in the delivery of health care services; or (4) barangay health worker
who has undergone training programs under any accredited government and NGO and who
voluntarily renders primarily health care services in the community after having been accredited to
function as such by the local health board in accordance with the guidelines promulgated by the
Department of Health (DOH) .

Further, the use of the term "private health care institution" in Section 7 of the law, instead of "private
health care service provider," should not be a cause of confusion for the obvious reason that they
are used synonymously.

The Court need not belabor the issue of whether the right to be exempt from being obligated to
render reproductive health service and modem family planning methods, includes exemption from
being obligated to give reproductive health information and to render reproductive health
procedures. Clearly, subject to the qualifications and exemptions earlier discussed, the right to be
exempt from being obligated to render reproductive health service and modem family planning
methods, necessarily includes exemption from being obligated to give reproductive health
information and to render reproductive health procedures. The terms "service" and "methods" are
broad enough to include the providing of information and the rendering of medical procedures.

The same can be said with respect to the contention that the RH Law punishes health care service
providers who intentionally withhold, restrict and provide incorrect information regarding reproductive
health programs and services. For ready reference, the assailed provision is hereby quoted as
follows:

SEC. 23. Prohibited Acts. - The following acts are prohibited:

(a) Any health care service provider, whether public or private, who shall:

(1) Knowingly withhold information or restrict the dissemination thereof, and/ or intentionally provide
incorrect information regarding programs and services on reproductive health including the right to
informed choice and access to a full range of legal, medically-safe, non-abortifacient and effective
family planning methods;

From its plain meaning, the word "incorrect" here denotes failing to agree with a copy or model or
with established rules; inaccurate, faulty; failing to agree with the requirements of duty, morality or
propriety; and failing to coincide with the truth.   On the other hand, the word "knowingly" means
257

with awareness or deliberateness that is intentional.  Used together in relation to Section 23(a)(l),
258

they connote a sense of malice and ill motive to mislead or misrepresent the public as to the nature
and effect of programs and services on reproductive health. Public health and safety demand that
health care service providers give their honest and correct medical information in accordance with
what is acceptable in medical practice. While health care service providers are not barred from
expressing their own personal opinions regarding the programs and services on reproductive health,
their right must be tempered with the need to provide public health and safety. The public deserves
no less.

7-Egual Protection

The petitioners also claim that the RH Law violates the equal protection clause under the
Constitution as it discriminates against the poor because it makes them the primary target of the
government program that promotes contraceptive use . They argue that, rather than promoting
reproductive health among the poor, the RH Law introduces contraceptives that would effectively
reduce the number of the poor. Their bases are the various provisions in the RH Law dealing with
the poor, especially those mentioned in the guiding principles  and definition of terms  of the law.
259 260

They add that the exclusion of private educational institutions from the mandatory reproductive
health education program imposed by the RH Law renders it unconstitutional.

In Biraogo v. Philippine Truth Commission,  the Court had the occasion to expound on the concept
261

of equal protection. Thus:

One of the basic principles on which this government was founded is that of the equality of right
which is embodied in Section 1, Article III of the 1987 Constitution. The equal protection of the laws
is embraced in the concept of due process, as every unfair discrimination offends the requirements
of justice and fair play. It has been embodied in a separate clause, however, to provide for a more
specific guaranty against any form of undue favoritism or hostility from the government. Arbitrariness
in general may be challenged on the basis of the due process clause. But if the particular act
assailed partakes of an unwarranted partiality or prejudice, the sharper weapon to cut it down is the
equal protection clause.

"According to a long line of decisions, equal protection simply requires that all persons or things
similarly situated should be treated alike, both as to rights conferred and responsibilities imposed." It
"requires public bodies and inst itutions to treat similarly situated individuals in a similar manner."
"The purpose of the equal protection clause is to secure every person within a state's jurisdiction
against intentional and arbitrary discrimination, whether occasioned by the express terms of a statue
or by its improper execution through the state's duly constituted authorities." "In other words, the
concept of equal justice under the law requires the state to govern impartially, and it may not draw
distinctions between individuals solely on differences that are irrelevant to a legitimate governmental
objective."

The equal protection clause is aimed at all official state actions, not just those of the legislature. Its
inhibitions cover all the departments of the government including the political and executive
departments, and extend to all actions of a state denying equal protection of the laws, through
whatever agency or whatever guise is taken.

It, however, does not require the universal application of the laws to all persons or things without
distinction. What it simply requires is equality among equals as determined according to a valid
classification. Indeed, the equal protection clause permits classification. Such classification,
however, to be valid must pass the test of reasonableness. The test has four requisites: (1) The
classification rests on substantial distinctions; (2) It is germane to the purpose of the law; (3) It is not
limited to existing conditions only; and (4) It applies equally to all members of the same class.
"Superficial differences do not make for a valid classification."

For a classification to meet the requirements of constitutionality, it must include or embrace all
persons who naturally belong to the class. "The classification will be regarded as invalid if all the
members of the class are not similarly treated, both as to rights conferred and obligations imposed. It
is not necessary that the classification be made with absolute symmetry, in the sense that the
members of the class should possess the same characteristics in equal degree. Substantial
similarity will suffice; and as long as this is achieved, all those covered by the classification are to be
treated equally. The mere fact that an individual belonging to a class differs from the other members,
as long as that class is substantially distinguishable from all others, does not justify the non-
application of the law to him."

The classification must not be based on existing circumstances only, or so constituted as to preclude
addition to the number included in the class. It must be of such a nature as to embrace all those who
may thereafter be in similar circumstances and conditions. It must not leave out or "underinclude"
those that should otherwise fall into a certain classification. [Emphases supplied; citations excluded]

To provide that the poor are to be given priority in the government's reproductive health care
program is not a violation of the equal protection clause. In fact, it is pursuant to Section 11, Article
XIII of the Constitution which recognizes the distinct necessity to address the needs of the
underprivileged by providing that they be given priority in addressing the health development of the
people. Thus:

Section 11. The State shall adopt an integrated and comprehensive approach to health development
which shall endeavor to make essential goods, health and other social services available to all the
people at affordable cost. There shall be priority for the needs of the underprivileged, sick, elderly,
disabled, women, and children. The State shall endeavor to provide free medical care to paupers.
It should be noted that Section 7 of the RH Law prioritizes poor and marginalized couples who are
suffering from fertility issues and desire to have children. There is, therefore, no merit to the
contention that the RH Law only seeks to target the poor to reduce their number. While the RH Law
admits the use of contraceptives, it does not, as elucidated above, sanction abortion. As Section 3(1)
explains, the "promotion and/or stabilization of the population growth rate is incidental to the
advancement of reproductive health."

Moreover, the RH Law does not prescribe the number of children a couple may have and does not
impose conditions upon couples who intend to have children. While the petitioners surmise that the
assailed law seeks to charge couples with the duty to have children only if they would raise them in a
truly humane way, a deeper look into its provisions shows that what the law seeks to do is to simply
provide priority to the poor in the implementation of government programs to promote basic
reproductive health care.

With respect to the exclusion of private educational institutions from the mandatory reproductive
health education program under Section 14, suffice it to state that the mere fact that the children of
those who are less fortunate attend public educational institutions does not amount to substantial
distinction sufficient to annul the assailed provision. On the other hand, substantial distinction rests
between public educational institutions and private educational institutions, particularly because
there is a need to recognize the academic freedom of private educational institutions especially with
respect to religious instruction and to consider their sensitivity towards the teaching of reproductive
health education.

8-Involuntary Servitude

The petitioners also aver that the RH Law is constitutionally infirm as it violates the constitutional
prohibition against involuntary servitude. They posit that Section 17 of the assailed legislation
requiring private and non-government health care service providers to render forty-eight (48) hours
of pro bono reproductive health services, actually amounts to involuntary servitude because it
requires medical practitioners to perform acts against their will.
262

The OSG counters that the rendition of pro bono services envisioned in Section 17 can hardly be
considered as forced labor analogous to slavery, as reproductive health care service providers have
the discretion as to the manner and time of giving pro bono services. Moreover, the OSG points out
that the imposition is within the powers of the government, the accreditation of medical practitioners
with PhilHealth being a privilege and not a right.

The point of the OSG is well-taken.

It should first be mentioned that the practice of medicine is undeniably imbued with public interest
that it is both a power and a duty of the State to control and regulate it in order to protect and
promote the public welfare. Like the legal profession, the practice of medicine is not a right but a
privileged burdened with conditions as it directly involves the very lives of the people. A fortiori, this
power includes the power of Congress  to prescribe the qualifications for the practice of professions
263

or trades which affect the public welfare, the public health, the public morals, and the public safety;
and to regulate or control such professions or trades, even to the point of revoking such right
altogether.264

Moreover, as some petitioners put it, the notion of involuntary servitude connotes the presence of
force, threats, intimidation or other similar means of coercion and compulsion.  A reading of the
265

assailed provision, however, reveals that it only encourages private and non- government
reproductive healthcare service providers to render pro bono service. Other than non-accreditation
with PhilHealth, no penalty is imposed should they choose to do otherwise. Private and non-
government reproductive healthcare service providers also enjoy the liberty to choose which kind of
health service they wish to provide, when, where and how to provide it or whether to provide it all.
Clearly, therefore, no compulsion, force or threat is made upon them to render pro bono service
against their will. While the rendering of such service was made a prerequisite to accreditation with
PhilHealth, the Court does not consider the same to be an unreasonable burden, but rather, a
necessary incentive imposed by Congress in the furtherance of a perceived legitimate state interest.

Consistent with what the Court had earlier discussed, however, it should be emphasized that
conscientious objectors are exempt from this provision as long as their religious beliefs and
convictions do not allow them to render reproductive health service, pro bona or otherwise.

9-Delegation of Authority to the FDA

The petitioners likewise question the delegation by Congress to the FDA of the power to determine
whether or not a supply or product is to be included in the Essential Drugs List (EDL).266

The Court finds nothing wrong with the delegation. The FDA does not only have the power but also
the competency to evaluate, register and cover health services and methods. It is the only
government entity empowered to render such services and highly proficient to do so. It should be
understood that health services and methods fall under the gamut of terms that are associated with
what is ordinarily understood as "health products."

In this connection, Section 4 of R.A. No. 3 720, as amended by R.A. No. 9711 reads:

SEC. 4. To carry out the provisions of this Act, there is hereby created an office to be called the
Food and Drug Administration (FDA) in the Department of Health (DOH). Said Administration shall
be under the Office of the Secretary and shall have the following functions, powers and duties:

"(a) To administer the effective implementation of this Act and of the rules and regulations
issued pursuant to the same;

"(b) To assume primary jurisdiction in the collection of samples of health products;

"(c) To analyze and inspect health products in connection with the implementation of this Act;

"(d) To establish analytical data to serve as basis for the preparation of health products
standards, and to recommend standards of identity, purity, safety, efficacy, quality and fill of
container;

"(e) To issue certificates of compliance with technical requirements to serve as basis for the
issuance of appropriate authorization and spot-check for compliance with regulations
regarding operation of manufacturers, importers, exporters, distributors, wholesalers, drug
outlets, and other establishments and facilities of health products, as determined by the FDA;

"x x x

"(h) To conduct appropriate tests on all applicable health products prior to the issuance of
appropriate authorizations to ensure safety, efficacy, purity, and quality;
"(i) To require all manufacturers, traders, distributors, importers, exporters, wholesalers,
retailers, consumers, and non-consumer users of health products to report to the FDA any
incident that reasonably indicates that said product has caused or contributed to the death,
serious illness or serious injury to a consumer, a patient, or any person;

"(j) To issue cease and desist orders motu propio or upon verified complaint for health
products, whether or not registered with the FDA Provided, That for registered health
products, the cease and desist order is valid for thirty (30) days and may be extended for
sixty ( 60) days only after due process has been observed;

"(k) After due process, to order the ban, recall, and/or withdrawal of any health product found
to have caused death, serious illness or serious injury to a consumer or patient, or is found to
be imminently injurious, unsafe, dangerous, or grossly deceptive, and to require all
concerned to implement the risk management plan which is a requirement for the issuance
of the appropriate authorization;

x x x.

As can be gleaned from the above, the functions, powers and duties of the FDA are specific to
enable the agency to carry out the mandates of the law. Being the country's premiere and sole
agency that ensures the safety of food and medicines available to the public, the FDA was equipped
with the necessary powers and functions to make it effective. Pursuant to the principle of necessary
implication, the mandate by Congress to the FDA to ensure public health and safety by permitting
only food and medicines that are safe includes "service" and "methods." From the declared policy of
the RH Law, it is clear that Congress intended that the public be given only those medicines that are
proven medically safe, legal, non-abortifacient, and effective in accordance with scientific and
evidence-based medical research standards. The philosophy behind the permitted delegation was
explained in Echagaray v. Secretary of Justice,  as follows:
267

The reason is the increasing complexity of the task of the government and the growing inability of
the legislature to cope directly with the many problems demanding its attention. The growth of
society has ramified its activities and created peculiar and sophisticated problems that the legislature
cannot be expected reasonably to comprehend. Specialization even in legislation has become
necessary. To many of the problems attendant upon present day undertakings, the legislature may
not have the competence, let alone the interest and the time, to provide the required direct and
efficacious, not to say specific solutions.

10- Autonomy of Local Governments and the Autonomous Region

of Muslim Mindanao (ARMM)

As for the autonomy of local governments, the petitioners claim that the RH Law infringes upon the
powers devolved to local government units (LGUs) under Section 17 of the Local Government Code.
Said Section 17 vested upon the LGUs the duties and functions pertaining to the delivery of basic
services and facilities, as follows:

SECTION 17. Basic Services and Facilities. –

(a) Local government units shall endeavor to be self-reliant and shall continue exercising the
powers and discharging the duties and functions currently vested upon them. They shall also
discharge the functions and responsibilities of national agencies and offices devolved to
them pursuant to this Code. Local government units shall likewise exercise such other
powers and discharge such other functions and responsibilities as are necessary,
appropriate, or incidental to efficient and effective provision of the basic services and
facilities enumerated herein.

(b) Such basic services and facilities include, but are not limited to, x x x.

While the aforementioned provision charges the LGUs to take on the functions and
responsibilities that have already been devolved upon them from the national agencies on
the aspect of providing for basic services and facilities in their respective jurisdictions,
paragraph (c) of the same provision provides a categorical exception of cases involving
nationally-funded projects, facilities, programs and services.  Thus:
268

(c) Notwithstanding the provisions of subsection (b) hereof, public works and infrastructure
projects and other facilities, programs and services funded by the National Government
under the annual General Appropriations Act, other special laws, pertinent executive orders,
and those wholly or partially funded from foreign sources, are not covered under this
Section, except in those cases where the local government unit concerned is duly
designated as the implementing agency for such projects, facilities, programs and services.
[Emphases supplied]

The essence of this express reservation of power by the national government is that, unless an LGU
is particularly designated as the implementing agency, it has no power over a program for which
funding has been provided by the national government under the annual general appropriations act,
even if the program involves the delivery of basic services within the jurisdiction of the LGU.  A 269

complete relinquishment of central government powers on the matter of providing basic facilities and
services cannot be implied as the Local Government Code itself weighs against it. 270

In this case, a reading of the RH Law clearly shows that whether it pertains to the establishment of
health care facilities,  the hiring of skilled health professionals,  or the training of barangay health
271 272

workers,  it will be the national government that will provide for the funding of its implementation.
273

Local autonomy is not absolute. The national government still has the say when it comes to national
priority programs which the local government is called upon to implement like the RH Law.

Moreover, from the use of the word "endeavor," the LG Us are merely encouraged to provide these
services. There is nothing in the wording of the law which can be construed as making the
availability of these services mandatory for the LGUs. For said reason, it cannot be said that the RH
Law amounts to an undue encroachment by the national government upon the autonomy enjoyed by
the local governments.

The ARMM

The fact that the RH Law does not intrude in the autonomy of local governments can be equally
applied to the ARMM. The RH Law does not infringe upon its autonomy. Moreover, Article III,
Sections 6, 10 and 11 of R.A. No. 9054, or the organic act of the ARMM, alluded to by petitioner
Tillah to justify the exemption of the operation of the RH Law in the autonomous region, refer to the
policy statements for the guidance of the regional government. These provisions relied upon by the
petitioners simply delineate the powers that may be exercised by the regional government, which
can, in no manner, be characterized as an abdication by the State of its power to enact legislation
that would benefit the general welfare. After all, despite the veritable autonomy granted the ARMM,
the Constitution and the supporting jurisprudence, as they now stand, reject the notion of imperium
et imperio in the relationship between the national and the regional governments.  Except for the
274
express and implied limitations imposed on it by the Constitution, Congress cannot be restricted to
exercise its inherent and plenary power to legislate on all subjects which extends to all matters of
general concern or common interest. 275

11 - Natural Law

With respect to the argument that the RH Law violates natural law,  suffice it to say that the Court
276

does not duly recognize it as a legal basis for upholding or invalidating a law. Our only guidepost is
the Constitution. While every law enacted by man emanated from what is perceived as natural law,
the Court is not obliged to see if a statute, executive issuance or ordinance is in conformity to it. To
begin with, it is not enacted by an acceptable legitimate body. Moreover, natural laws are mere
thoughts and notions on inherent rights espoused by theorists, philosophers and theologists. The
jurists of the philosophical school are interested in the law as an abstraction, rather than in the actual
law of the past or present.  Unless, a natural right has been transformed into a written law, it cannot
277

serve as a basis to strike down a law. In Republic v. Sandiganbayan,  the very case cited by the
278

petitioners, it was explained that the Court is not duty-bound to examine every law or action and
whether it conforms with both the Constitution and natural law. Rather, natural law is to be used
sparingly only in the most peculiar of circumstances involving rights inherent to man where no law is
applicable.279

At any rate, as earlier expounded, the RH Law does not sanction the taking away of life. It does not
allow abortion in any shape or form. It only seeks to enhance the population control program of the
government by providing information and making non-abortifacient contraceptives more readily
available to the public, especially to the poor.

Facts and Fallacies

and the Wisdom of the Law

In general, the Court does not find the RH Law as unconstitutional insofar as it seeks to provide
access to medically-safe, non-abortifacient, effective, legal, affordable, and quality reproductive
healthcare services, methods, devices, and supplies. As earlier pointed out, however, the religious
freedom of some sectors of society cannot be trampled upon in pursuit of what the law hopes to
achieve. After all, the Constitutional safeguard to religious freedom is a recognition that man stands
accountable to an authority higher than the State.

In conformity with the principle of separation of Church and State, one religious group cannot be
allowed to impose its beliefs on the rest of the society. Philippine modem society leaves enough
room for diversity and pluralism. As such, everyone should be tolerant and open-minded so that
peace and harmony may continue to reign as we exist alongside each other.

As healthful as the intention of the RH Law may be, the idea does not escape the Court that what it
seeks to address is the problem of rising poverty and unemployment in the country. Let it be said
that the cause of these perennial issues is not the large population but the unequal distribution of
wealth. Even if population growth is controlled, poverty will remain as long as the country's wealth
remains in the hands of the very few.

At any rate, population control may not be beneficial for the country in the long run. The European
and Asian countries, which embarked on such a program generations ago , are now burdened with
ageing populations. The number of their young workers is dwindling with adverse effects on their
economy. These young workers represent a significant human capital which could have helped them
invigorate, innovate and fuel their economy. These countries are now trying to reverse their
programs, but they are still struggling. For one, Singapore, even with incentives, is failing.

And in this country, the economy is being propped up by remittances from our Overseas Filipino
Workers. This is because we have an ample supply of young able-bodied workers. What would
happen if the country would be weighed down by an ageing population and the fewer younger
generation would not be able to support them? This would be the situation when our total fertility rate
would go down below the replacement level of two (2) children per woman. 280

Indeed, at the present, the country has a population problem, but the State should not use coercive
measures (like the penal provisions of the RH Law against conscientious objectors) to solve it.
Nonetheless, the policy of the Court is non-interference in the wisdom of a law.

x x x. But this Court cannot go beyond what the legislature has laid down. Its duty is to say what the
law is as enacted by the lawmaking body. That is not the same as saying what the law should be or
what is the correct rule in a given set of circumstances. It is not the province of the judiciary to look
into the wisdom of the law nor to question the policies adopted by the legislative branch. Nor is it the
business of this Tribunal to remedy every unjust situation that may arise from the application of a
particular law. It is for the legislature to enact remedial legislation if that would be necessary in the
premises. But as always, with apt judicial caution and cold neutrality, the Court must carry out the
delicate function of interpreting the law, guided by the Constitution and existing legislation and
mindful of settled jurisprudence. The Court's function is therefore limited, and accordingly, must
confine itself to the judicial task of saying what the law is, as enacted by the lawmaking body.281

Be that as it may, it bears reiterating that the RH Law is a mere compilation and enhancement of the
prior existing contraceptive and reproductive health laws, but with coercive measures. Even if the
Court decrees the RH Law as entirely unconstitutional, there will still be the Population Act (R.A. No.
6365), the Contraceptive Act (R.A. No. 4729) and the reproductive health for women or The Magna
Carta of Women (R.A. No. 9710), sans the coercive provisions of the assailed legislation. All the
same, the principle of "no-abortion" and "non-coercion" in the adoption of any family planning
method should be maintained.

WHEREFORE, the petitions are PARTIALLY GRANTED. Accordingly, the Court declares R.A. No.
10354 as NOT UNCONSTITUTIONAL except with respect to the following provisions which are
declared UNCONSTITUTIONAL:

1) Section 7 and the corresponding provision in the RH-IRR insofar as they: a) require
private health facilities and non-maternity specialty hospitals and hospitals owned and
operated by a religious group to refer patients, not in an emergency or life-threatening case,
as defined under Republic Act No. 8344, to another health facility which is conveniently
accessible; and b) allow minor-parents or minors who have suffered a miscarriage access to
modem methods of family planning without written consent from their parents or guardian/s;

2) Section 23(a)(l) and the corresponding provision in the RH-IRR, particularly Section 5 .24
thereof, insofar as they punish any healthcare service provider who fails and or refuses to
disseminate information regarding programs and services on reproductive health regardless
of his or her religious beliefs.

3) Section 23(a)(2)(i) and the corresponding provision in the RH-IRR insofar as they allow a
married individual, not in an emergency or life-threatening case, as defined under Republic
Act No. 8344, to undergo reproductive health procedures without the consent of the spouse;
4) Section 23(a)(2)(ii) and the corresponding provision in the RH-IRR insofar as they limit the
requirement of parental consent only to elective surgical procedures.

5) Section 23(a)(3) and the corresponding provision in the RH-IRR, particularly Section 5.24
thereof, insofar as they punish any healthcare service provider who fails and/or refuses to
refer a patient not in an emergency or life-threatening case, as defined under Republic Act
No. 8344, to another health care service provider within the same facility or one which is
conveniently accessible regardless of his or her religious beliefs;

6) Section 23(b) and the corresponding provision in the RH-IRR, particularly Section 5 .24
thereof, insofar as they punish any public officer who refuses to support reproductive health
programs or shall do any act that hinders the full implementation of a reproductive health
program, regardless of his or her religious beliefs;

7) Section 17 and the corresponding prov1s10n in the RH-IRR regarding the rendering of pro
bona reproductive health service in so far as they affect the conscientious objector in
securing PhilHealth accreditation; and

8) Section 3.0l(a) and Section 3.01 G) of the RH-IRR, which added the qualifier "primarily" in
defining abortifacients and contraceptives, as they are ultra vires and, therefore, null and
void for contravening Section 4(a) of the RH Law and violating Section 12, Article II of the
Constitution.

The Status Quo Ante Order issued by the Court on March 19, 2013 as extended by its Order, dated
July 16, 2013 , is hereby LIFTED, insofar as the provisions of R.A. No. 10354 which have been
herein declared as constitutional.

SO ORDERED.

January 12, 2016

G.R. No. 212426

RENE A.V. SAGUISAG, WIGBERTO E. TAÑADA, FRANCISCO "DODONG" NEMENZO, JR., SR.
MARY JOHN MANANZAN, PACIFICO A. AGABIN, ESTEBAN "STEVE" SALONGA, H. HARRY L.
ROQUE, JR., EVALYN G. URSUA, EDRE U. OLALIA, DR. CAROL PAGADUAN-ARAULLO, DR.
ROLAND SIMBULAN, AND TEDDY CASIÑO, Petitioners, 
vs.
EXECUTIVE SECRETARY PAQUITO N. OCHOA, JR., DEPARTMENT OF NATIONAL DEFENSE
SECRETARY VOLTAIRE GAZMIN, DEPARTMENT OF FOREIGN AFFAIRS SECRETARY
ALBERT DEL ROSARIO, JR., DEPARTMENT OF BUDGET AND MANAGEMENT SECRETARY
FLORENCIO ABAD, AND ARMED FORCES OF THE PHILIPPINES CHIEF OF STAFF GENERAL
EMMANUEL T. BAUTISTA, Respondents.

x-----------------------x

G.R. No. 212444


BAGONG ALYANSANG MAKABAYAN (BAYAN), REPRESENTED BY ITS SECRETARY
GENERAL RENATO M. REYES, JR., BAYAN MUNA PARTY-LIST REPRESENTATIVES NERI J.
COLMENARES AND CARLOS ZARATE, GABRIELA WOMEN'S PARTY-LIST
REPRESENTATIVES LUZ ILAGAN AND EMERENCIANA DE JESUS, ACT TEACHERS PARTY-
LIST REPRESENTATIVE ANTONIO L. TINIO, ANAKPAWIS PARTY-LIST REPRESENTATIVE
FERNANDO HICAP, KABATAAN PARTY-LIST REPRESENTATIVE TERRY RIDON,
MAKABAYANG KOALISYON NG MAMAMAYAN (MAKABAYAN), REPRESENTED BY
SATURNINO OCAMPO AND LIZA MAZA, BIENVENIDO LUMBERA, JOEL C. LAMANGAN,
RAFAEL MARIANO, SALVADOR FRANCE, ROGELIO M. SOLUTA, AND CLEMENTE G.
BAUTISTA, Petitioners, 
vs.
DEPARTMENT OF NATIONAL DEFENSE (DND) SECRETARY VOLTAIRE GAZMIN,
DEPARTMENT OF FOREIGN AFFAIRS SECRETARY ALBERT DEL ROSARIO, EXECUTIVE
SECRETARY PAQUITO N. OCHOA, JR., ARMED FORCES OF THE PHILIPPINES CHIEF OF
STAFF GENERAL EMMANUEL T. BAUTISTA, DEFENSE UNDERSECRETARY PIO LORENZO
BATINO, AMBASSADOR LOURDES YPARRAGUIRRE, AMBASSADOR J. EDUARDO MALAYA,
DEPARTMENT OF JUSTICE UNDERSECRETARY FRANCISCO BARAAN III, AND DND
ASSISTANT SECRETARY FOR STRATEGIC ASSESSMENTS RAYMUND JOSE QUILOP AS
CHAIRPERSON AND MEMBERS, RESPECTIVELY, OF THE NEGOTIATING PANEL FOR THE
PHILIPPINES ON EDCA, Respondents.

x-----------------------x

KILUSANG MAYO UNO, REPRESENTED BY ITS CHAIRPERSON, ELMER LABOG,


CONFEDERATION FOR UNITY, RECOGNITION AND ADVANCEMENT OF GOVERNMENT
EMPLOYEES (COURAGE), REPRESENTED BY ITS NATIONAL PRESIDENT FERDINAND
GAITE, NATIONAL FEDERATION OF LABOR UNIONS-KILUSANG MAYO UNO,
REPRESENTED BY ITS NATIONAL PRESIDENT JOSELITO USTAREZ, NENITA GONZAGA,
VIOLETA ESPIRITU, VIRGINIA FLORES, AND ARMANDO TEODORO, JR., Petitioners-in-
Intervention, 
RENE A.Q. SAGUISAG, JR., Petitioner-in-Intervention.

DECISION

SERENO, J.:

The petitions  before this Court question the constitutionality of the Enhanced Defense Cooperation
1

Agreement (EDCA) between the Republic of the Philippines and the United States of America (U.S.).
Petitioners allege that respondents committed grave abuse of discretion amounting to lack or excess
of jurisdiction when they entered into EDCA with the U.S.,  claiming that the instrument violated
2

multiple constitutional provisions.  In reply, respondents argue that petitioners lack standing to bring
3

the suit. To support the legality of their actions, respondents invoke the 1987 Constitution, treaties,
and judicial precedents.4

A proper analysis of the issues requires this Court to lay down at the outset the basic parameters of
the constitutional powers and roles of the President and the Senate in respect of the above issues. A
more detailed discussion of these powers and roles will be made in the latter portions.

I. BROAD CONSTITUTIONAL CONTEXT OF THE POWERS OF THE PRESIDENT: DEFENSE,


FOREIGN RELATIONS, AND EDCA

A. The Prime Duty of the State and the Consolidation of Executive Power in the President
Mataimtim kong pinanunumpaan (o pinatotohanan) na tutuparin ko nang buong katapatan at sigasig
ang aking mga tungkulin bilang Pangulo (o Pangalawang Pangulo o Nanunungkulang Pangulo) ng
Pilipinas, pangangalagaan at ipagtatanggol ang kanyang Konstitusyon, ipatutupad ang mga batas
nito, magiging makatarungan sa bawat tao, at itatalaga ang aking sarili sa paglilingkod sa Bansa.
Kasihan nawa aka ng Diyos.

- Panunumpa sa Katungkulan ng Pangulo ng Pilipinas ayon sa Saligang Batas 5

The 1987 Constitution has "vested the executive power in the President of the Republic of the
Philippines."  While the vastness of the executive power that has been consolidated in the person of
6

the President cannot be expressed fully in one provision, the Constitution has stated the prime duty
of the government, of which the President is the head:

The prime duty of the Government is to serve and protect the people. The Government may call
upon the people to defend the State and, in the fulfillment thereof, all citizens may be required, under
conditions provided by law, to render personal military or civil service.  (Emphases supplied)
7

B. The duty to protect the territory and the citizens of the Philippines, the power to call upon
the people to defend the State, and the President as Commander-in-Chief

The duty to protect the State and its people must be carried out earnestly and effectively throughout
the whole territory of the Philippines in accordance with the constitutional provision on national
territory. Hence, the President of the Philippines, as the sole repository of executive power, is the
guardian of the Philippine archipelago, including all the islands and waters embraced therein and all
other territories over which it has sovereignty or jurisdiction. These territories consist of its terrestrial,
fluvial, and aerial domains; including its territorial sea, the seabed, the subsoil, the insular shelves,
and other submarine areas; and the waters around, between, and connecting the islands of the
archipelago, regardless of their breadth and dimensions. 8

To carry out this important duty, the President is equipped with authority over the Armed Forces of
the Philippines (AFP),  which is the protector of the people and the state. The AFP's role is to secure
9

the sovereignty of the State and the integrity of the national territory.  In addition, the Executive is
10

constitutionally empowered to maintain peace and order; protect life, liberty, and property; and
promote the general welfare. 11

In recognition of these powers, Congress has specified that the President must oversee, ensure, and
reinforce our defensive capabilities against external and internal threats  and, in the same vein,
12

ensure that the country is adequately prepared for all national and local emergencies arising from
natural and man-made disasters. 13

To be sure, this power is limited by the Constitution itself. To illustrate, the President may call out the
AFP to prevent or suppress instances of lawless violence, invasion or rebellion,  but not suspend the
14

privilege of the writ of habeas corpus for a period exceeding 60 days, or place the Philippines or any
part thereof under martial law exceeding that same span. In the exercise of these powers, the
President is also duty-bound to submit a report to Congress, in person or in writing, within 48 hours
from the proclamation of martial law or the suspension of the privilege of the writ of habeas corpus;
and Congress may in turn revoke the proclamation or suspension. The same provision provides for
the Supreme Court's review of the factual basis for the proclamation or suspension, as well as the
promulgation of the decision within 30 days from filing.

C. The power and duty to conduct foreign relations


The President also carries the mandate of being the sole organ in the conduct of foreign
relations.  Since every state has the capacity to interact with and engage in relations with other
15

sovereign states,  it is but logical that every state must vest in an agent the authority to represent its
16

interests to those other sovereign states.

The conduct of foreign relations is full of complexities and consequences, sometimes with life and
death significance to the nation especially in times of war. It can only be entrusted to that department
of government which can act on the basis of the best available information and can decide with
decisiveness. x x x It is also the President who possesses the most comprehensive and the most
confidential information about foreign countries for our diplomatic and consular officials regularly
brief him on meaningful events all over the world. He has also unlimited access to ultra-sensitive
military intelligence data. In fine, the presidential role in foreign affairs is dominant and the President
is traditionally accorded a wider degree of discretion in the conduct of foreign affairs. The regularity,
nay, validity of his actions are adjudged under less stringent standards, lest their judicial repudiation
lead to breach of an international obligation, rupture of state relations, forfeiture of confidence,
national embarrassment and a plethora of other problems with equally undesirable consequences. 17

The role of the President in foreign affairs is qualified by the Constitution in that the Chief Executive
must give paramount importance to the sovereignty of the nation, the integrity of its territory, its
interest, and the right of the sovereign Filipino people to self-determination.  In specific provisions,
18

the President's power is also limited, or at least shared, as in Section 2 of Article II on the conduct of
war; Sections 20 and 21 of Article VII on foreign loans, treaties, and international agreements;
Sections 4(2) and 5(2)(a) of Article VIII on the judicial review of executive acts; Sections 4 and 25 of
Article XVIII on treaties and international agreements entered into prior to the Constitution and on
the presence of foreign military troops, bases, or facilities.

D. The relationship between the two major presidential functions and the role of the Senate

Clearly, the power to defend the State and to act as its representative in the international sphere
inheres in the person of the President. This power, however, does not crystallize into absolute
discretion to craft whatever instrument the Chief Executive so desires. As previously mentioned, the
Senate has a role in ensuring that treaties or international agreements the President enters into, as
contemplated in Section 21 of Article VII of the Constitution, obtain the approval of two-thirds of its
members.

Previously, treaties under the 1973 Constitution required ratification by a majority of the Batasang
Pambansa, except in instances wherein the President "may enter into international treaties or
19

agreements as the national welfare and interest may require."  This left a large margin of discretion
20

that the President could use to bypass the Legislature altogether. This was a departure from the
1935 Constitution, which explicitly gave the President the power to enter into treaties only with the
concurrence of two-thirds of all the Members of the Senate.  The 1987 Constitution returned the
21

Senate's power  and, with it, the legislative's traditional role in foreign affairs.
22 23

The responsibility of the President when it comes to treaties and international agreements under the
present Constitution is therefore shared with the Senate. This shared role, petitioners claim, is
bypassed by EDCA.

II. HISTORICAL ANTECEDENTS OF EDCA

A. U.S. takeover of Spanish colonization and its military bases, and the transition to
Philippine independence
The presence of the U.S. military forces in the country can be traced to their pivotal victory in the
1898 Battle of Manila Bay during the Spanish-American War.  Spain relinquished its sovereignty
24

over the Philippine Islands in favor of the U.S. upon its formal surrender a few months later.  By 25

1899, the Americans had consolidated a military administration in the archipelago. 26

When it became clear that the American forces intended to impose colonial control over the
Philippine Islands, General Emilio Aguinaldo immediately led the Filipinos into an all-out war against
the U.S.  The Filipinos were ultimately defeated in the Philippine-American War, which lasted until
27

1902 and led to the downfall of the first Philippine Republic.  The Americans henceforth began to
28

strengthen their foothold in the country.  They took over and expanded the former Spanish Naval
29

Base in Subic Bay, Zambales, and put up a cavalry post called Fort Stotsenberg in Pampanga, now
known as Clark Air Base. 30

When talks of the eventual independence of the Philippine Islands gained ground, the U.S.
manifested the desire to maintain military bases and armed forces in the country.  The U.S. 31

Congress later enacted the Hare-Hawes-Cutting Act of 1933, which required that the proposed
constitution of an independent Philippines recognize the right of the U.S. to maintain the latter's
armed forces and military bases.  The Philippine Legislature rejected that law, as it also gave the
32

U.S. the power to unilaterally designate any part of Philippine territory as a permanent military or
naval base of the U.S. within two years from complete independence. 33

The U.S. Legislature subsequently crafted another law called the Tydings-McDuffie Act or the
Philippine Independence Act of 1934. Compared to the old Hare-Hawes-Cutting Act, the new law
provided for the surrender to the Commonwealth Government of "all military and other reservations"
of the U.S. government in the Philippines, except "naval reservations and refueling
stations."  Furthermore, the law authorized the U.S. President to enter into negotiations for the
34

adjustment and settlement of all questions relating to naval reservations and fueling stations within
two years after the Philippines would have gained independence.  Under the Tydings-McDuffie Act,
35

the U.S. President would proclaim the American withdrawal and surrender of sovereignty over the
islands 10 years after the inauguration of the new government in the Philippines.  This law 36

eventually led to the promulgation of the 1935 Philippine Constitution.

The original plan to surrender the military bases changed.  At the height of the Second World War,
37

the Philippine and the U.S. Legislatures each passed resolutions authorizing their respective
Presidents to negotiate the matter of retaining military bases in the country after the planned
withdrawal of the U.S.  Subsequently, in 1946, the countries entered into the Treaty of General
38

Relations, in which the U.S. relinquished all control and sovereignty over the Philippine
Islands, except the areas that would be covered by the American military bases in the country.  This 39

treaty eventually led to the creation of the post-colonial legal regime on which would hinge the
continued presence of U.S. military forces until 1991: the Military Bases Agreement (MBA) of 1947,
the Military Assistance Agreement of 1947, and the Mutual Defense Treaty (MDT) of 1951. 40

B. Former legal regime on the presence of U.S. armed forces in the territory of an
independent Philippines (1946-1991)

Soon after the Philippines was granted independence, the two countries entered into their first
military arrangement pursuant to the Treaty of General Relations - the 1947 MBA.  The Senate41

concurred on the premise of "mutuality of security interest,"  which provided for the presence and
42

operation of 23 U.S. military bases in the Philippines for 99 years or until the year 2046.  The treaty
43

also obliged the Philippines to negotiate with the U.S. to allow the latter to expand the existing bases
or to acquire new ones as military necessity might require. 44
A number of significant amendments to the 1947 MBA were made.  With respect to its duration, the
45

parties entered into the Ramos-Rusk Agreement of 1966, which reduced the term of the treaty from
99 years to a total of 44 years or until 1991.  Concerning the number of U.S. military bases in the
46

country, the Bohlen-Serrano Memorandum of Agreement provided for the return to the Philippines of
17 U.S. military bases covering a total area of 117,075 hectares.  Twelve years later, the U.S.
47

returned Sangley Point in Cavite City through an exchange of notes.  Then, through the Romulo- 48

Murphy Exchange of Notes of 1979, the parties agreed to the recognition of Philippine sovereignty
over Clark and Subic Bases and the reduction of the areas that could be used by the U.S.
military.  The agreement also provided for the mandatory review of the treaty every five years.  In
49 50

1983, the parties revised the 1947 MBA through the Romualdez-Armacost Agreement.  The revision 51

pertained to the operational use of the military bases by the U.S. government within the context of
Philippine sovereignty,  including the need for prior consultation with the Philippine government on
52

the former' s use of the bases for military combat operations or the establishment of long-range
missiles. 53

Pursuant to the legislative authorization granted under Republic Act No. 9,  the President also 54

entered into the 1947 Military Assistance Agreement  with the U.S. This executive agreement
55

established the conditions under which U.S. military assistance would be granted to the
Philippines,  particularly the provision of military arms, ammunitions, supplies, equipment, vessels,
56

services, and training for the latter's defense forces.  An exchange of notes in 1953 made it clear
57

that the agreement would remain in force until terminated by any of the parties. 58

To further strengthen their defense and security relationship,  the Philippines and the U.S. next
59

entered into the MDT in 1951. Concurred in by both the Philippine  and the U.S.  Senates, the treaty
60 61

has two main features: first, it allowed for mutual assistance in maintaining and developing their
individual and collective capacities to resist an armed attack;  and second, it provided for their
62

mutual self-defense in the event of an armed attack against the territory of either party.  The treaty 63

was premised on their recognition that an armed attack on either of them would equally be a threat
to the security of the other.
64

C. Current legal regime on the presence of U.S. armed forces in the country

In view of the impending expiration of the 1947 MBA in 1991, the Philippines and the U.S. negotiated
for a possible renewal of their defense and security relationship.  Termed as the Treaty of 65

Friendship, Cooperation and Security, the countries sought to recast their military ties by providing a
new framework for their defense cooperation and the use of Philippine installations.  One of the 66

proposed provisions included an arrangement in which U.S. forces would be granted the use of
certain installations within the Philippine naval base in Subic.  On 16 September 1991, the Senate
67

rejected the proposed treaty. 68

The consequent expiration of the 1947 MBA and the resulting paucity of any formal agreement
dealing with the treatment of U.S. personnel in the Philippines led to the suspension in 1995 of large-
scale joint military exercises. In the meantime, the respective governments of the two countries
69

agreed  to hold joint exercises at a substantially reduced level.  The military arrangements between
70 71

them were revived in 1999 when they concluded the first Visiting Forces Agreement (VFA). 72

As a "reaffirm[ation] [of the] obligations under the MDT,"  the VFA has laid down the regulatory
73

mechanism for the treatment of U.S. military and civilian personnel visiting the country.  It contains 74

provisions on the entry and departure of U.S. personnel; the purpose, extent, and limitations of their
activities; criminal and disciplinary jurisdiction; the waiver of certain claims; the importation and
exportation of equipment, materials, supplies, and other pieces of property owned by the U.S.
government; and the movement of U.S. military vehicles, vessels, and aircraft into and within the
country.  The Philippines and the U.S. also entered into a second counterpart agreement (VFA II),
75

which in turn regulated the treatment of Philippine military and civilian personnel visiting the
U.S.  The Philippine Senate concurred in the first VFA on 27 May 1999.
76 77

Beginning in January 2002, U.S. military and civilian personnel started arriving in Mindanao to take
part in joint military exercises with their Filipino counterparts.  Called Balikatan, these exercises
78

involved trainings aimed at simulating joint military maneuvers pursuant to the MDT. 79

In the same year, the Philippines and the U.S. entered into the Mutual Logistics Support Agreement
to "further the interoperability, readiness, and effectiveness of their respective military forces"  in
80

accordance with the MDT, the Military Assistance Agreement of 1953, and the VFA.  The new 81

agreement outlined the basic terms, conditions, and procedures for facilitating the reciprocal
provision of logistics support, supplies, and services between the military forces of the two
countries.  The phrase "logistics support and services" includes billeting, operations support,
82

construction and use of temporary structures, and storage services during an approved activity
under the existing military arrangements.  Already extended twice, the agreement will last until
83

2017. 84

D. The Enhanced Defense Cooperation Agreement

EDCA authorizes the U.S. military forces to have access to and conduct activities within certain
"Agreed Locations" in the country. It was not transmitted to the Senate on the executive's
understanding that to do so was no longer necessary.  Accordingly, in June 2014, the Department of
85

Foreign Affairs (DFA) and the U.S. Embassy exchanged diplomatic notes confirming the completion
of all necessary internal requirements for the agreement to enter into force in the two countries.
86

According to the Philippine government, the conclusion of EDCA was the result of intensive and
comprehensive negotiations in the course of almost two years.  After eight rounds of negotiations,
87

the Secretary of National Defense and the U.S. Ambassador to the Philippines signed the
agreement on 28 April 2014.  President Benigno S. Aquino III ratified EDCA on 6 June 2014.  The
88 89

OSG clarified during the oral arguments  that the Philippine and the U.S. governments had yet to
90

agree formally on the specific sites of the Agreed Locations mentioned in the agreement.

Two petitions for certiorari were thereafter filed before us assailing the constitutionality of EDCA.
They primarily argue that it should have been in the form of a treaty concurred in by the Senate, not
an executive agreement.

On 10 November 2015, months after the oral arguments were concluded and the parties ordered to
file their respective memoranda, the Senators adopted Senate Resolution No. (SR) 105.  The 91

resolution expresses the "strong sense"  of the Senators that for EDCA to become valid and
92

effective, it must first be transmitted to the Senate for deliberation and concurrence.

III. ISSUES

Petitioners mainly seek a declaration that the Executive Department committed grave abuse of
discretion in entering into EDCA in the form of an executive agreement. For this reason, we cull the
issues before us:

A. Whether the essential requisites for judicial review are present


B. Whether the President may enter into an executive agreement on foreign military
bases, troops, or facilities

C. Whether the provisions under EDCA are consistent with the Constitution, as well
as with existing laws and treaties

IV. DISCUSSION

A. Whether the essential requisites for judicial review have been satisfied

Petitioners are hailing this Court's power of judicial review in order to strike down EDCA for violating
the Constitution. They stress that our fundamental law is explicit in prohibiting the presence of
foreign military forces in the country, except under a treaty concurred in by the Senate. Before this
Court may begin to analyze the constitutionality or validity of an official act of a coequal branch of
government, however, petitioners must show that they have satisfied all the essential requisites for
judicial review.
93

Distinguished from the general notion of judicial power, the power of judicial review specially refers
to both the authority and the duty of this Court to determine whether a branch or an instrumentality of
government has acted beyond the scope of the latter's constitutional powers.  As articulated in
94

Section 1, Article VIII of the Constitution, the power of judicial review involves the power to resolve
cases in which the questions concern the constitutionality or validity of any treaty, international or
executive agreement, law, presidential decree, proclamation, order, instruction, ordinance, or
regulation.  In Angara v. Electoral Commission, this Court exhaustively discussed this "moderating
95

power" as part of the system of checks and balances under the Constitution. In our fundamental law,
the role of the Court is to determine whether a branch of government has adhered to the specific
restrictions and limitations of the latter's power:
96

The separation of powers is a fundamental principle in our system of government. It obtains not
through express provision but by actual division in our Constitution. Each department of the
government has exclusive cognizance of matters within its jurisdiction, and is supreme
within its own sphere. But it does not follow from the fact that the three powers are to be kept
separate and distinct that the Constitution intended them to be absolutely unrestrained and
independent of each other. The Constitution has provided for an elaborate system of checks
and balances to secure coordination in the workings of the various departments of the government.
x x x. And the judiciary in turn, with the Supreme Court as the final arbiter, effectively checks the
other departments in the exercise of its power to determine the law, and hence to declare
executive and legislative acts void if violative of the Constitution.

xxxx

As any human production, our Constitution is of course lacking perfection and perfectibility, but as
much as it was within the power of our people, acting through their delegates to so provide, that
instrument which is the expression of their sovereignty however limited, has established a
republican government intended to operate and function as a harmonious whole, under a
system of checks and balances, and subject to specific limitations and restrictions provided
in the said instrument. The Constitution sets forth in no uncertain language the restrictions
and limitations upon governmental powers and agencies. If these restrictions and limitations
are transcended it would be inconceivable if the Constitution had not provided for a
mechanism by which to direct the course of government along constitutional channels, for
then the distribution of powers would be mere verbiage, the bill of rights mere expressions of
sentiment, and the principles of good government mere political apothegms. Certainly, the
limitations and restrictions embodied in our Constitution are real as they should be in any living
constitution. x x x. In our case, this moderating power is granted, if not expressly, by clear implication
from section 2 of article VIII of [the 1935] Constitution.

The Constitution is a definition of the powers of government. Who is to determine the nature, scope
and extent of such powers? The Constitution itself has provided for the instrumentality of the
judiciary as the rational way. And when the judiciary mediates to allocate constitutional
boundaries, it does not assert any superiority over the other departments; it does not in
reality nullify or invalidate an act of the legislature, but only asserts the solemn and sacred
obligation assigned to it by the Constitution to determine conflicting claims of authority
under the Constitution and to establish for the parties in an actual controversy the rights
which that instrument secures and guarantees to them. This is in truth all that is involved in what
is termed "judicial supremacy" which properly is the power of judicial review under the
Constitution. x x x x. (Emphases supplied)

The power of judicial review has since been strengthened in the 1987 Constitution. The scope of that
power has been extended to the determination of whether in matters traditionally considered to be
within the sphere of appreciation of another branch of government, an exercise of discretion has
been attended with grave abuse.  The expansion of this power has made the political question
97

doctrine "no longer the insurmountable obstacle to the exercise of judicial power or the impenetrable
shield that protects executive and legislative actions from judicial inquiry or review."
98

This moderating power, however, must be exercised carefully and only if it cannot be completely
avoided. We stress that our Constitution is so incisively designed that it identifies the spheres of
expertise within which the different branches of government shall function and the questions of
policy that they shall resolve.  Since the power of judicial review involves the delicate exercise of
99

examining the validity or constitutionality of an act of a coequal branch of government, this Court
must continually exercise restraint to avoid the risk of supplanting the wisdom of the constitutionally
appointed actor with that of its own.100

Even as we are left with no recourse but to bare our power to check an act of a coequal branch of
government - in this case the executive - we must abide by the stringent requirements for the
exercise of that power under the Constitution. Demetria v. Alba  and Francisco v. House of
101

Representatives  cite the "pillars" of the limitations on the power of judicial review as enunciated in
102

the concurring opinion of U.S. Supreme Court Justice Brandeis in Ashwander v. Tennessee Valley
Authority.  Francisco  redressed these "pillars" under the following categories:
103 104

1. That there be absolute necessity of deciding a case

2. That rules of constitutional law shall be formulated only as required by the facts of the
case

3. That judgment may not be sustained on some other ground

4. That there be actual injury sustained by the party by reason of the operation of the
statute

5. That the parties are not in estoppel

6. That the Court upholds the presumption of constitutionality


(Emphases supplied)

These are the specific safeguards laid down by the Court when it exercises its power of judicial
review.  Guided by these pillars, it may invoke the power only when the following four stringent
105

requirements are satisfied: (a) there is an actual case or controversy; (b) petitioners possess locus
standi; (c) the question of constitutionality is raised at the earliest opportunity; and (d) the issue of
constitutionality is the lis mota of the case.  Of these four, the first two conditions will be the focus of
106

our discussion.

1. Petitioners have shown the presence of an actual case or controversy.

The OSG maintains  that there is no actual case or controversy that exists, since the Senators have
107

not been deprived of the opportunity to invoke the privileges of the institution they are representing.
It contends that the nonparticipation of the Senators in the present petitions only confirms that even
they believe that EDCA is a binding executive agreement that does not require their concurrence.

It must be emphasized that the Senate has already expressed its position through SR
105.  Through the Resolution, the Senate has taken a position contrary to that of the OSG. As the
108

body tasked to participate in foreign affairs by ratifying treaties, its belief that EDCA infringes upon its
constitutional role indicates that an actual controversy - albeit brought to the Court by non-Senators,
exists.

Moreover, we cannot consider the sheer abstention of the Senators from the present proceedings as
basis for finding that there is no actual case or controversy before us. We point out that the focus of
this requirement is the ripeness for adjudication of the matter at hand, as opposed to its being
merely conjectural or anticipatory.  The case must involve a definite and concrete issue involving
109

real parties with conflicting legal rights and legal claims admitting of specific relief through a decree
conclusive in nature.  It should not equate with a mere request for an opinion or advice on what the
110

law would be upon an abstract, hypothetical, or contingent state of facts.  As explained in Angara v.
111

Electoral Commission: 112

[The] power of judicial review is limited to actual cases and controversies to be exercised after
full opportunity of argument by the parties, and limited further to the constitutional question
raised or the very lis mota presented. Any attempt at abstraction could only lead to dialectics
and barren legal questions and to sterile conclusions of wisdom, justice or expediency of
legislation. More than that, courts accord the presumption of constitutionality to legislative
enactments, not only because the legislature is presumed to abide by the Constitution but also
because the judiciary in the determination of actual cases and controversies must reflect the
wisdom and justice of the people as expressed through their representatives in the executive
and legislative departments of the government. (Emphases supplied)

We find that the matter before us involves an actual case or controversy that is already ripe for
adjudication. The Executive Department has already sent an official confirmation to the U.S.
Embassy that "all internal requirements of the Philippines x x x have already been complied
with."  By this exchange of diplomatic notes, the Executive Department effectively performed the
113

last act required under Article XII(l) of EDCA before the agreement entered into force. Section 25,
Article XVIII of the Constitution, is clear that the presence of foreign military forces in the country
shall only be allowed by virtue of a treaty concurred in by the Senate. Hence, the performance of an
official act by the Executive Department that led to the entry into force of an executive agreement
was sufficient to satisfy the actual case or controversy requirement.
2. While petitioners Saguisag et. al., do not have legal standing, they nonetheless raise
issues involving matters of transcendental importance.

The question of locus standi or legal standing focuses on the determination of whether those
assailing the governmental act have the right of appearance to bring the matter to the court for
adjudication.  They must show that they have a personal and substantial interest in the case, such
114

that they have sustained or are in immediate danger of sustaining, some direct injury as a
consequence of the enforcement of the challenged governmental act.  Here, "interest" in the
115

question involved must be material - an interest that is in issue and will be affected by the official act
- as distinguished from being merely incidental or general.  Clearly, it would be insufficient to show
116

that the law or any governmental act is invalid, and that petitioners stand to suffer in some indefinite
way.  They must show that they have a particular interest in bringing the suit, and that they have
117

been or are about to be denied some right or privilege to which they are lawfully entitled, or that they
are about to be subjected to some burden or penalty by reason of the act complained of.  The 118

reason why those who challenge the validity of a law or an international agreement are required to
allege the existence of a personal stake in the outcome of the controversy is "to assure the concrete
adverseness which sharpens the presentation of issues upon which the court so largely depends for
illumination of difficult constitutional questions."
119

The present petitions cannot qualify as citizens', taxpayers', or legislators' suits; the Senate as a
body has the requisite standing, but considering that it has not formally filed a pleading to join the
suit, as it merely conveyed to the Supreme Court its sense that EDCA needs the Senate's
concurrence to be valid, petitioners continue to suffer from lack of standing.

In assailing the constitutionality of a governmental act, petitioners suing as citizens may dodge the
requirement of having to establish a direct and personal interest if they show that the act affects a
public right.  In arguing that they have legal standing, they claim  that the case they have filed is a
120 121

concerned citizen's suit. But aside from general statements that the petitions involve the protection of
a public right, and that their constitutional rights as citizens would be violated, they fail to make any
specific assertion of a particular public right that would be violated by the enforcement of EDCA. For
their failure to do so, the present petitions cannot be considered by the Court as citizens'
suits that would justify a disregard of the aforementioned requirements.

In claiming that they have legal standing as taxpayers, petitioners  aver that the implementation of
122

EDCA would result in the unlawful use of public funds. They emphasize that Article X(1) refers to an
appropriation of funds; and that the agreement entails a waiver of the payment of taxes, fees, and
rentals. During the oral arguments, however, they admitted that the government had not yet
appropriated or actually disbursed public funds for the purpose of implementing the
agreement.  The OSG, on the other hand, maintains that petitioners cannot sue as
123

taxpayers. Respondent explains that EDCA is neither meant to be a tax measure, nor is it directed
124

at the disbursement of public funds.

A taxpayer's suit concerns a case in which the official act complained of directly involves the illegal
disbursement of public funds derived from taxation.  Here, those challenging the act must
125

specifically show that they have sufficient interest in preventing the illegal expenditure of public
money, and that they will sustain a direct injury as a result of the enforcement of the assailed
act.  Applying that principle to this case, they must establish that EDCA involves the exercise by
126

Congress of its taxing or spending powers. 127

We agree with the OSG that the petitions cannot qualify as taxpayers' suits. We emphasize that a
taxpayers' suit contemplates a situation in which there is already an appropriation or a disbursement
of public funds.  A reading of Article X(l) of EDCA would show that there has been neither an
128

appropriation nor an authorization of disbursement of funds. The cited provision reads:

All obligations under this Agreement are subject to the availability of appropriated


funds authorized for these purposes. (Emphases supplied)

This provision means that if the implementation of EDCA would require the disbursement of public
funds, the money must come from appropriated funds that are specifically authorized for this
purpose. Under the agreement, before there can even be a disbursement of public funds, there must
first be a legislative action. Until and unless the Legislature appropriates funds for EDCA, or
unless petitioners can pinpoint a specific item in the current budget that allows expenditure
under the agreement, we cannot at this time rule that there is in fact an appropriation or a
disbursement of funds that would justify the filing of a taxpayers' suit.

Petitioners Bayan et al. also claim  that their co-petitioners who are party-list representatives have
129

the standing to challenge the act of the Executive Department, especially if it impairs the
constitutional prerogatives, powers, and privileges of their office. While they admit that there is no
incumbent Senator who has taken part in the present petition, they nonetheless assert that they also
stand to sustain a derivative but substantial injury as legislators. They argue that under the
Constitution, legislative power is vested in both the Senate and the House of Representatives;
consequently, it is the entire Legislative Department that has a voice in determining whether or not
the presence of foreign military should be allowed. They maintain that as members of the
Legislature, they have the requisite personality to bring a suit, especially when a constitutional issue
is raised.

The OSG counters  that petitioners do not have any legal standing to file the suits concerning the
130

lack of Senate concurrence in EDCA. Respondent emphasizes that the power to concur in treaties
and international agreements is an "institutional prerogative" granted by the Constitution to the
Senate. Accordingly, the OSG argues that in case of an allegation of impairment of that power, the
injured party would be the Senate as an institution or any of its incumbent members, as it is the
Senate's constitutional function that is allegedly being violated.

The legal standing of an institution of the Legislature or of any of its Members has already been
recognized by this Court in a number of cases.  What is in question here is the alleged impairment
131

of the constitutional duties and powers granted to, or the impermissible intrusion upon the domain of,
the Legislature or an institution thereof.  In the case of suits initiated by the legislators themselves,
132

this Court has recognized their standing to question the validity of any official action that they claim
infringes the prerogatives, powers, and privileges vested by the Constitution in their office.  As aptly
133

explained by Justice Perfecto in Mabanag v. Lopez Vito: 134

Being members of Congress, they are even duty bound to see that the latter act within the
bounds of the Constitution which, as representatives of the people, they should uphold, unless
they are to commit a flagrant betrayal of public trust. They are representatives of the sovereign
people and it is their sacred duty to see to it that the fundamental law embodying the will of
the sovereign people is not trampled upon. (Emphases supplied)

We emphasize that in a legislators' suit, those Members of Congress who are challenging the official
act have standing only to the extent that the alleged violation impinges on their right to participate in
the exercise of the powers of the institution of which they are members.  Legislators have the
135

standing "to maintain inviolate the prerogatives, powers, and privileges vested by the Constitution
in their office and are allowed to sue to question the validity of any official action, which they claim
infringes their prerogatives as legislators."  As legislators, they must clearly show that there was a
136

direct injury to their persons or the institution to which they belong.


137

As correctly argued by respondent, the power to concur in a treaty or an international agreement is


an institutional prerogative granted by the Constitution to the Senate, not to the entire Legislature.
In Pimentel v. Office of the Executive Secretary, this Court did not recognize the standing of one of
the petitioners therein who was a member of the House of Representatives. The petition in that case
sought to compel the transmission to the Senate for concurrence of the signed text of the Statute of
the International Criminal Court. Since that petition invoked the power of the Senate to grant or
withhold its concurrence in a treaty entered into by the Executive Department, only then incumbent
Senator Pimentel was allowed to assert that authority of the Senate of which he was a member.

Therefore, none of the initial petitioners in the present controversy has the standing to
maintain the suits as legislators.

Nevertheless, this Court finds that there is basis for it to review the act of the Executive for the
following reasons.

In any case, petitioners raise issues involving matters of transcendental importance.

Petitioners  argue that the Court may set aside procedural technicalities, as the present petition
138

tackles issues that are of transcendental importance. They point out that the matter before us is
about the proper exercise of the Executive Department's power to enter into international
agreements in relation to that of the Senate to concur in those agreements. They also assert that
EDCA would cause grave injustice, as well as irreparable violation of the Constitution and of the
Filipino people's rights.

The OSG, on the other hand, insists  that petitioners cannot raise the mere fact that the present
139

petitions involve matters of transcendental importance in order to cure their inability to comply with
the constitutional requirement of standing. Respondent bewails the overuse of "transcendental
importance" as an exception to the traditional requirements of constitutional litigation. It stresses that
one of the purposes of these requirements is to protect the Supreme Court from unnecessary
litigation of constitutional questions.

In a number of cases,  this Court has indeed taken a liberal stance towards the requirement of legal
140

standing, especially when paramount interest is involved. Indeed, when those who challenge the
official act are able to craft an issue of transcendental significance to the people, the Court may
exercise its sound discretion and take cognizance of the suit. It may do so in spite of the inability of
the petitioners to show that they have been personally injured by the operation of a law or any other
government act.

While this Court has yet to thoroughly delineate the outer limits of this doctrine, we emphasize that
not every other case, however strong public interest may be, can qualify as an issue of
transcendental importance. Before it can be impelled to brush aside the essential requisites for
exercising its power of judicial review, it must at the very least consider a number of factors: (1) the
character of the funds or other assets involved in the case; (2) the presence of a clear case of
disregard of a constitutional or statutory prohibition by the public respondent agency or
instrumentality of the government; and (3) the lack of any other party that has a more direct and
specific interest in raising the present questions.141

An exhaustive evaluation of the memoranda of the parties, together with the oral arguments, shows
that petitioners have presented serious constitutional issues that provide ample justification for the
Court to set aside the rule on standing. The transcendental importance of the issues presented here
is rooted in the Constitution itself. Section 25, Article XVIII thereof, cannot be any clearer: there is a
much stricter mechanism required before foreign military troops, facilities, or bases may be allowed
in the country. The DFA has already confirmed to the U.S. Embassy that "all internal requirements of
the Philippines x x x have already been complied with."  It behooves the Court in this instance to
142

take a liberal stance towards the rule on standing and to determine forthwith whether there was
grave abuse of discretion on the part of the Executive Department.

We therefore rule that this case is a proper subject for judicial review.

B. Whether the President may enter into an executive agreement on foreign military
bases, troops, or facilities

C. Whether the provisions under EDCA are consistent with the Constitution, as well
as with existing laws and treaties

Issues B and C shall be discussed together infra.

1. The role of the President as the executor of the law includes the duty to defend the State,
for which purpose he may use that power in the conduct of foreign relations

Historically, the Philippines has mirrored the division of powers in the U.S. government. When the
Philippine government was still an agency of the Congress of the U.S., it was as an agent entrusted
with powers categorized as executive, legislative, and judicial, and divided among these three great
branches.  By this division, the law implied that the divided powers cannot be exercised except by
143

the department given the power. 144

This divide continued throughout the different versions of the Philippine Constitution and specifically
vested the supreme executive power in the Governor-General of the Philippines,  a position 145

inherited by the President of the Philippines when the country attained independence. One of the
principal functions of the supreme executive is the responsibility for the faithful execution of the laws
as embodied by the oath of office.  The oath of the President prescribed by the 1987 Constitution
146

reads thus:

I do solemnly swear (or affirm) that I will faithfully and conscientiously fulfill my duties as
President (or Vice-President or Acting President) of the Philippines, preserve and defend its
Constitution, execute its laws, do justice to every man, and consecrate myself to the service of the
Nation. So help me God. (In case of affirmation, last sentence will be omitted.)  (Emphases147

supplied)

This Court has interpreted the faithful execution clause as an obligation imposed on the President,
and not a separate grant of power.  Section 1 7, Article VII of the Constitution, expresses this duty in
148

no uncertain terms and includes it in the provision regarding the President's power of control over
the executive department, viz:

The President shall have control of all the executive departments, bureaus, and offices. He shall
ensure that the laws be faithfully executed.

The equivalent provisions in the next preceding Constitution did not explicitly require this oath from
the President. In the 1973 Constitution, for instance, the provision simply gives the President control
over the ministries.  A similar language, not in the form of the President's oath, was present in the
149
1935 Constitution, particularly in the enumeration of executive functions.  By 1987, executive power
150

was codified not only in the Constitution, but also in the Administrative Code: 151

SECTION 1. Power of Control. - The President shall have control of all the executive departments,
bureaus, and offices. He shall ensure that the laws be faithfully executed. (Emphasis supplied)

Hence, the duty to faithfully execute the laws of the land is inherent in executive power and is
intimately related to the other executive functions. These functions include the faithful execution of
the law in autonomous regions;  the right to prosecute crimes;  the implementation of transportation
152 153

projects;  the duty to ensure compliance with treaties, executive agreements and executive
154

orders;  the authority to deport undesirable aliens;  the conferment of national awards under the
155 156

President's jurisdiction;  and the overall administration and control of the executive department.
157 158

These obligations are as broad as they sound, for a President cannot function with crippled hands,
but must be capable of securing the rule of law within all territories of the Philippine Islands and be
empowered to do so within constitutional limits. Congress cannot, for instance, limit or take over the
President's power to adopt implementing rules and regulations for a law it has enacted. 159

More important, this mandate is self-executory by virtue of its being inherently executive in
nature.  As Justice Antonio T. Carpio previously wrote,
160 161

[i]f the rules are issued by the President in implementation or execution of self-executory
constitutional powers vested in the President, the rule-making power of the President is not a
delegated legislative power. The most important self-executory constitutional power of the President
is the President's constitutional duty and mandate to "ensure that the laws be faithfully executed."
The rule is that the President can execute the law without any delegation of power from the
legislature.

The import of this characteristic is that the manner of the President's execution of the law,
even if not expressly granted by the law, is justified by necessity and limited only by law,
since the President must "take necessary and proper steps to carry into execution the
law."  Justice George Malcolm states this principle in a grand manner:
162 163

The executive should be clothed with sufficient power to administer efficiently the affairs of state. He
should have complete control of the instrumentalities through whom his responsibility is discharged.
It is still true, as said by Hamilton, that "A feeble executive implies a feeble execution of the
government. A feeble execution is but another phrase for a bad execution; and a government ill
executed, whatever it may be in theory, must be in practice a bad government." The mistakes of
State governments need not be repeated here.

xxxx

Every other consideration to one side, this remains certain - The Congress of the United States
clearly intended that the Governor-General's power should be commensurate with his responsibility.
The Congress never intended that the Governor-General should be saddled with the responsibility of
administering the government and of executing the laws but shorn of the power to do so. The
interests of the Philippines will be best served by strict adherence to the basic principles of
constitutional government.

In light of this constitutional duty, it is the President's prerogative to do whatever is legal and
necessary for Philippine defense interests. It is no coincidence that the constitutional provision on
the faithful execution clause was followed by that on the President's commander-in-chief
powers,  which are specifically granted during extraordinary events of lawless violence, invasion, or
164

rebellion. And this duty of defending the country is unceasing, even in times when there is no state
of lawlesss violence, invasion, or rebellion. At such times, the President has full powers to ensure
the faithful execution of the laws.

It would therefore be remiss for the President and repugnant to the faithful-execution clause of the
Constitution to do nothing when the call of the moment requires increasing the military's defensive
capabilities, which could include forging alliances with states that hold a common interest with the
Philippines or bringing an international suit against an offending state.

The context drawn in the analysis above has been termed by Justice Arturo D. Brion's Dissenting
Opinion as the beginning of a "patent misconception."  His dissent argues that this approach taken
165

in analyzing the President's role as executor of the laws is preceded by the duty to preserve and
defend the Constitution, which was allegedly overlooked. 166

In arguing against the approach, however, the dissent grossly failed to appreciate the nuances of the
analysis, if read holistically and in context. The concept that the President cannot function with
crippled hands and therefore can disregard the need for Senate concurrence in treaties  was never
167

expressed or implied. Rather, the appropriate reading of the preceding analysis shows that the point
being elucidated is the reality that the President's duty to execute the laws and protect the
Philippines is inextricably interwoven with his foreign affairs powers, such that he must resolve
issues imbued with both concerns to the full extent of his powers, subject only to the limits supplied
by law. In other words, apart from an expressly mandated limit, or an implied limit by virtue of
incompatibility, the manner of execution by the President must be given utmost deference. This
approach is not different from that taken by the Court in situations with fairly similar contexts.

Thus, the analysis portrayed by the dissent does not give the President authority to bypass
constitutional safeguards and limits. In fact, it specifies what these limitations are, how these
limitations are triggered, how these limitations function, and what can be done within the sphere of
constitutional duties and limitations of the President.

Justice Brion's dissent likewise misinterprets the analysis proffered when it claims that the foreign
relations power of the President should not be interpreted in isolation.  The analysis itself
168

demonstrates how the foreign affairs function, while mostly the President's, is shared in several
instances, namely in Section 2 of Article II on the conduct of war; Sections 20 and 21 of Article VII on
foreign loans, treaties, and international agreements; Sections 4(2) and 5(2)(a) of Article VIII on the
judicial review of executive acts; Sections 4 and 25 of Article XVIII on treaties and international
agreements entered into prior to the Constitution and on the presence of foreign military troops,
bases, or facilities.

In fact, the analysis devotes a whole subheading to the relationship between the two major
presidential functions and the role of the Senate in it.

This approach of giving utmost deference to presidential initiatives in respect of foreign affairs is not
novel to the Court. The President's act of treating EDCA as an executive agreement is not the
principal power being analyzed as the Dissenting Opinion seems to suggest. Rather, the preliminary
analysis is in reference to the expansive power of foreign affairs. We have long treated this power as
something the Courts must not unduly restrict. As we stated recently in Vinuya v. Romulo:

To be sure, not all cases implicating foreign relations present political questions, and courts certainly
possess the authority to construe or invalidate treaties and executive agreements. However, the
question whether the Philippine government should espouse claims of its nationals against a foreign
government is a foreign relations matter, the authority for which is demonstrably committed by our
Constitution not to the courts but to the political branches. In this case, the Executive Department
has already decided that it is to the best interest of the country to waive all claims of its nationals for
reparations against Japan in the Treaty of Peace of 1951. The wisdom of such decision is not for the
courts to question. Neither could petitioners herein assail the said determination by the Executive
Department via the instant petition for certiorari.

In the seminal case of US v. Curtiss-Wright Export Corp., the US Supreme Court held that "[t]he
President is the sole organ of the nation in its external relations, and its sole representative with
foreign relations."

It is quite apparent that if, in the maintenance of our international relations,


embarrassment - perhaps serious embarrassment - is to be avoided and success for
our aims achieved, congressional legislation which is to be made effective through
negotiation and inquiry within the international field must often accord to the
President a degree of discretion and freedom from statutory restriction which
would not be admissible where domestic affairs alone involved. Moreover, he,
not Congress, has the better opportunity of knowing the conditions which prevail in
foreign countries, and especially is this true in time of war. He has his confidential
sources of information. He has his agents in the form of diplomatic, consular and
other officials ....

This ruling has been incorporated in our jurisprudence through  Bavan v. Executive
Secretary  and Pimentel v. Executive Secretary; its overreaching principle was, perhaps, best
articulated in (now Chief) Justice Puno's dissent in Secretary of Justice v. Lantion:

. . . The conduct of foreign relations is full of complexities and consequences,


sometimes with life and death significance to the nation especially in times of war. It
can only be entrusted to that department of government which can act on the basis
of the best available information and can decide with decisiveness .... It is also the
President who possesses the most comprehensive and the most confidential
information about foreign countries for our diplomatic and consular officials regularly
brief him on meaningful events all over the world. He has also unlimited access to
ultra-sensitive military intelligence data. In fine, the presidential role in foreign
affairs is dominant and the President is traditionally accorded a wider degree
of discretion in the conduct of foreign affairs. The regularity, nay, validity of his
actions are adjudged under less stringent standards, lest their judicial
repudiation lead to breach of an international obligation, rupture of state
relations, forfeiture of confidence, national embarrassment and a plethora of
other problems with equally undesirable consequences.  (Emphases supplied)
169

Understandably, this Court must view the instant case with the same perspective and understanding,
knowing full well the constitutional and legal repercussions of any judicial overreach.

2. The plain meaning of the Constitution prohibits the entry of foreign military bases, troops
or facilities, except by way of a treaty concurred in by the Senate - a clear limitation on the
President's dual role as defender of the State and as sole authority in foreign relations.

Despite the President's roles as defender of the State and sole authority in foreign relations, the
1987 Constitution expressly limits his ability in instances when it involves the entry of foreign military
bases, troops or facilities. The initial limitation is found in Section 21 of the provisions on the
Executive Department: "No treaty or international agreement shall be valid and effective unless
concurred in by at least two-thirds of all the Members of the Senate." The specific limitation is given
by Section 25 of the Transitory Provisions, the full text of which reads as follows:

SECTION 25. After the expiration in 1991 of the Agreement between the Republic of the Philippines
and the United States of America concerning Military Bases, foreign military bases, troops, or
facilities shall not be allowed in the Philippines except under a treaty duly concurred in by the Senate
and, when the Congress so requires, ratified by a majority of the votes cast by the people in a
national referendum held for that purpose, and recognized as a treaty by the other contracting State.

It is quite plain that the Transitory Provisions of the 1987 Constitution intended to add to the basic
requirements of a treaty under Section 21 of Article VII. This means that both provisions must be
read as additional limitations to the President's overarching executive function in matters of defense
and foreign relations.

3. The President, however, may enter into an executive agreement on foreign military bases,
troops, or facilities, if (a) it is not the instrument that allows the presence of foreign military
bases, troops, or facilities; or (b) it merely aims to implement an existing law or treaty.

Again we refer to Section 25, Article XVIII of the Constitution:

SECTION 25. After the expiration in 1991 of the Agreement between the Republic of the Philippines
and the United States of America concerning Military Bases, foreign military bases, troops, or
facilities shall not be allowed in the Philippines except under a treaty duly concurred in by
the Senate and, when the Congress so requires, ratified by a majority of the votes cast by the
people in a national referendum held for that purpose, and recognized as a treaty by the other
contracting State. (Emphases supplied)

In view of this provision, petitioners argue  that EDCA must be in the form of a "treaty" duly
170

concurred in by the Senate. They stress that the Constitution is unambigous in mandating the
transmission to the Senate of all international agreements concluded after the expiration of the MBA
in 1991 - agreements that concern the presence of foreign military bases, troops, or facilities in the
country. Accordingly, petitioners maintain that the Executive Department is not given the choice to
conclude agreements like EDCA in the form of an executive agreement.

This is also the view of the Senate, which, through a majority vote of 15 of its members - with 1
against and 2 abstaining - says in SR 105  that EDCA must be submitted to the Senate in the form
171

of a treaty for concurrence by at least two-thirds of all its members.

The Senate cites two constitutional provisions (Article VI, Section 21 and Article XVIII, Section 25) to
support its position. Compared with the lone constitutional provision that the Office of the Solicitor
General (OSG) cites, which is Article XVIII, Section 4(2), which includes the constitutionality of
"executive agreement(s)" among the cases subject to the Supreme Court's power of judicial review,
the Constitution clearly requires submission of EDCA to the Senate. Two specific provisions versus
one general provision means that the specific provisions prevail. The term "executive agreement" is
"a term wandering alone in the Constitution, bereft of provenance and an unidentified constitutional
mystery."

The author of SR 105, Senator Miriam Defensor Santiago, upon interpellation even added that the
MDT, which the Executive claims to be partly implemented through EDCA, is already obsolete.
There are two insurmountable obstacles to this Court's agreement with SR 105, as well as with the
comment on interpellation made by Senator Santiago.

First, the concept of "executive agreement" is so well-entrenched in this Court's pronouncements on


the powers of the President. When the Court validated the concept of "executive agreement," it did
so with full knowledge of the Senate's role in concurring in treaties. It was aware of the
problematique of distinguishing when an international agreement needed Senate concurrence for
validity, and when it did not; and the Court continued to validate the existence of "executive
agreements" even after the 1987 Constitution.  This follows a long line of similar decisions
172

upholding the power of the President to enter into an executive agreement. 173

Second, the MDT has not been rendered obsolescent, considering that as late as 2009,  this Court
174

continued to recognize its validity.

Third, to this Court, a plain textual reading of Article XIII, Section 25, inevitably leads to the
conclusion that it applies only to a proposed agreement between our government and a foreign
government, whereby military bases, troops, or facilities of such foreign government would be
"allowed" or would "gain entry" Philippine territory.

Note that the provision "shall not be allowed" is a negative injunction. This wording signifies that the
President is not authorized by law to allow foreign military bases, troops, or facilities to enter the
Philippines, except under a treaty concurred in by the Senate. Hence, the constitutionally restricted
authority pertains to the entry of the bases, troops, or facilities, and not to the activities to be done
after entry.

Under the principles of constitutional construction, of paramount consideration is the plain meaning
of the language expressed in the Constitution, or the verba legis rule.  It is presumed that the
175

provisions have been carefully crafted in order to express the objective it seeks to attain.  It is
176

incumbent upon the Court to refrain from going beyond the plain meaning of the words used in the
Constitution. It is presumed that the framers and the people meant what they said when they said it,
and that this understanding was reflected in the Constitution and understood by the people in the
way it was meant to be understood when the fundamental law was ordained and promulgated.  As 177

this Court has often said:

We look to the language of the document itself in our search for its meaning. We do not of course
stop there, but that is where we begin. It is to be assumed that the words in which constitutional
provisions are couched express the objective sought to be attained. They are to be given their
ordinary meaning except where technical terms are employed in which case the significance
thus attached to them prevails. As the Constitution is not primarily a lawyer's document, it being
essential for the rule of law to obtain that it should ever be present in the people's consciousness, its
language as much as possible should be understood in the sense they have in common use.
What it says according to the text of the provision to be construed compels acceptance and negates
the power of the courts to alter it, based on the postulate that the framers and the people mean
what they say. Thus, these are the cases where the need for construction is reduced to a
minimum. (Emphases supplied)
178

It is only in those instances in which the constitutional provision is unclear, ambiguous, or silent that
further construction must be done to elicit its meaning.  In Ang Bagong Bayani-OFW v. Commission
179

on Elections,  we reiterated this guiding principle:


180
it [is] safer to construe the Constitution from what appears upon its face. The proper
interpretation therefore depends more on how it was understood by the people adopting it than
in the framers' understanding thereof. (Emphases supplied)

The effect of this statement is surprisingly profound, for, if taken literally, the phrase "shall not be
allowed in the Philippines" plainly refers to the entry of bases, troops, or facilities in the country.
The Oxford English Dictionary defines the word "allow" as a transitive verb that means "to permit,
enable"; "to give consent to the occurrence of or relax restraint on (an action, event, or activity)"; "to
consent to the presence or attendance of (a person)"; and, when with an adverbial of place, "to
permit (a person or animal) to go, come, or be in, out, near, etc."  Black's Law Dictionary defines
181

the term as one that means "[t]o grant, approve, or permit." 182

The verb "allow" is followed by the word "in," which is a preposition used to indicate "place or
position in space or anything having material extension: Within the limits or bounds of, within (any
place or thing)."  That something is the Philippines, which is the noun that follows.
183

It is evident that the constitutional restriction refers solely to the initial entry of the foreign military
bases, troops, or facilities. Once entry is authorized, the subsequent acts are thereafter subject only
to the limitations provided by the rest of the Constitution and Philippine law, and not to the Section
25 requirement of validity through a treaty.

The VFA has already allowed the entry of troops in the Philippines. This Court stated in Lim v.
Executive Secretary:

After studied reflection, it appeared farfetched that the ambiguity surrounding the meaning of the
word "activities" arose from accident. In our view, it was deliberately made that way to give both
parties a certain leeway in negotiation. In this manner, visiting US forces may sojourn in
Philippine territory for purposes other than military. As conceived, the joint exercises may
include training on new techniques of patrol and surveillance to protect the nation's marine
resources, sea search-and-rescue operations to assist vessels in distress, disaster relief operations,
civic action projects such as the building of school houses, medical and humanitarian missions, and
the like.

Under these auspices, the VFA gives legitimacy to the current Balikatan exercises. It is only logical
to assume that "Balikatan 02-1," a "mutual anti- terrorism advising, assisting and training exercise,"
falls under the umbrella of sanctioned or allowable activities in the context of the agreement. Both
the history and intent of the Mutual Defense Treaty and the VFA support the conclusion that combat-
related activities -as opposed to combat itself-such as the one subject of the instant petition, are
indeed authorized.  (Emphasis supplied)
184

Moreover, the Court indicated that the Constitution continues to govern the conduct of foreign
military troops in the Philippines,  readily implying the legality of their initial entry into the country.
185

The OSG emphasizes that EDCA can be in the form of an executive agreement, since it merely
involves "adjustments in detail" in the implementation of the MDT and the VFA.  It points out that
186

there are existing treaties between the Philippines and the U.S. that have already been concurred in
by the Philippine Senate and have thereby met the requirements of the Constitution under Section
25. Because of the status of these prior agreements, respondent emphasizes that EDCA need not
be transmitted to the Senate.

The aforecited Dissenting Opinion of Justice Brion disagrees with the ponencia's application


of verba legis construction to the words of Article XVIII, Section 25.  It claims that the provision is
187
"neither plain, nor that simple."  To buttress its disagreement, the dissent states that the provision
188

refers to a historical incident, which is the expiration of the 1947 MBA.  Accordingly, this position
189

requires questioning the circumstances that led to the historical event, and the meaning of the terms
under Article XVIII, Section 25.

This objection is quite strange. The construction technique of verba legis is not inapplicable just
because a provision has a specific historical context. In fact, every provision of the Constitution has
a specific historical context. The purpose of constitutional and statutory construction is to set tiers of
interpretation to guide the Court as to how a particular provision functions. Verba legis is of
paramount consideration, but it is not the only consideration. As this Court has often said:

We look to the language of the document itself in our search for its meaning. We do not of course
stop there, but that is where we begin. It is to be assumed that the words in which constitutional
provisions are couched express the objective sought to be attained. They are to be given their
ordinary meaning except where technical terms are employed in which case the significance
thus attached to them prevails. As the Constitution is not primarily a lawyer's document, it being
essential for the rule of law to obtain that it should ever be present in the people's consciousness, its
language as much as possible should be understood in the sense they have in common use.
What it says according to the text of the provision to be construed compels acceptance and negates
the power of the courts to alter it, based on the postulate that the framers and the people mean
what they say. Thus, these are the cases where the need for construction is reduced to a
minimum. (Emphases supplied)
190

As applied, verba legis aids in construing the ordinary meaning of terms. In this case, the phrase
being construed is "shall not be allowed in the Philippines" and not the preceding one referring to
"the expiration in 1991 of the Agreement between the Republic of the Philippines and the United
States of America concerning Military Bases, foreign military bases, troops, or facilities." It is explicit
in the wording of the provision itself that any interpretation goes beyond the text itself and into the
discussion of the framers, the context of the Constitutional Commission's time of drafting, and the
history of the 1947 MBA. Without reference to these factors, a reader would not understand those
terms. However, for the phrase "shall not be allowed in the Philippines," there is no need for such
reference. The law is clear. No less than the Senate understood this when it ratified the VFA.

4. The President may generally enter into executive agreements subject to limitations defined
by the Constitution and may be in furtherance of a treaty already concurred in by the Senate.

We discuss in this section why the President can enter into executive agreements.

It would be helpful to put into context the contested language found in Article XVIII, Section 25. Its
more exacting requirement was introduced because of the previous experience of the country when
its representatives felt compelled to consent to the old MBA.  They felt constrained to agree to the
191

MBA in fulfilment of one of the major conditions for the country to gain independence from the
U.S.  As a result of that experience, a second layer of consent for agreements that allow military
192

bases, troops and facilities in the country is now articulated in Article XVIII of our present
Constitution.

This second layer of consent, however, cannot be interpreted in such a way that we completely
ignore the intent of our constitutional framers when they provided for that additional layer, nor the
vigorous statements of this Court that affirm the continued existence of that class of international
agreements called "executive agreements."
The power of the President to enter into binding executive agreements without Senate concurrence
is already well-established in this jurisdiction.  That power has been alluded to in our present and
193

past Constitutions,  in various statutes,  in Supreme Court decisions,  and during the deliberations
194 195 196

of the Constitutional Commission.  They cover a wide array of subjects with varying scopes and
197

purposes,  including those that involve the presence of foreign military forces in the country.
198 199

As the sole organ of our foreign relations  and the constitutionally assigned chief architect of our
200

foreign policy, the President is vested with the exclusive power to conduct and manage the
201

country's interface with other states and governments. Being the principal representative of the
Philippines, the Chief Executive speaks and listens for the nation; initiates, maintains, and develops
diplomatic relations with other states and governments; negotiates and enters into international
agreements; promotes trade, investments, tourism and other economic relations; and settles
international disputes with other states. 202

As previously discussed, this constitutional mandate emanates from the inherent power of the
President to enter into agreements with other states, including the prerogative to
conclude binding executive agreements that do not require further Senate concurrence. The
existence of this presidential power  is so well-entrenched that Section 5(2)(a), Article VIII of the
203

Constitution, even provides for a check on its exercise. As expressed below, executive agreements
are among those official governmental acts that can be the subject of this Court's power of judicial
review:

(2) Review, revise, reverse, modify, or affirm on appeal or certiorari, as the law or the
Rules of Court may provide, final judgments and orders of lower courts in:

(a) All cases in which the constitutionality or


validity of any treaty, international or executive agreement, law, presidential
decree, proclamation, order, instruction, ordinance, or regulation is in question.
(Emphases supplied)

In Commissioner of Customs v. Eastern Sea Trading, executive agreements are defined as


"international agreements embodying adjustments of detail carrying out well-established national
policies and traditions and those involving arrangements of a more or less temporary
nature."  In Bayan Muna v. Romulo, this Court further clarified that executive agreements can cover
204

a wide array of subjects that have various scopes and purposes.  They are no longer limited to the
205

traditional subjects that are usually covered by executive agreements as identified in Eastern Sea
Trading. The Court thoroughly discussed this matter in the following manner:

The categorization of subject matters that may be covered by international


agreementsmentioned in Eastern Sea Trading is not cast in stone. x x x.

As may be noted, almost half a century has elapsed since the Court rendered its decision
in Eastern Sea Trading. Since then, the conduct of foreign affairs has become more complex
and the domain of international law wider, as to include such subjects as human rights, the
environment, and the sea. In fact, in the US alone, the executive agreements executed by its
President from 1980 to 2000 covered subjects such as defense, trade, scientific cooperation,
aviation, atomic energy, environmental cooperation, peace corps, arms limitation, and
nuclear safety, among others. Surely, the enumeration in Eastern Sea Trading cannot
circumscribe the option of each state on the matter of which the international agreement format
would be convenient to serve its best interest. As Francis Sayre said in his work referred to
earlier:
. . . It would be useless to undertake to discuss here the large variety of executive agreements
as such concluded from time to time. Hundreds of executive agreements, other than those
entered into under the trade-agreement act, have been negotiated with foreign governments. . . .
They cover such subjects as the inspection of vessels, navigation dues, income tax on shipping
profits, the admission of civil air craft, custom matters and commercial relations generally,
international claims, postal matters, the registration of trademarks and copyrights, etc .... (Emphases
Supplied)

One of the distinguishing features of executive agreements is that their validity and effectivity are not
affected by a lack of Senate concurrence.  This distinctive feature was recognized as early as
206

in Eastern Sea Trading (1961), viz:

Treaties are formal documents which require ratification with the approval of two-thirds of the


Senate. Executive agreements become binding through executive action without the need of a
vote by the Senate or by Congress.

xxxx

[T]he right of the Executive to enter into binding agreements without the necessity of


subsequent Congressional approval has been confirmed by long usage. From the earliest days
of our history we have entered into executive agreements covering such subjects as commercial and
consular relations, most-favored-nation rights, patent rights, trademark and copyright protection,
postal and navigation arrangements and the settlement of claims. The validity of these has never
been seriously questioned by our courts. (Emphases Supplied)

That notion was carried over to the present Constitution. In fact, the framers specifically deliberated
on whether the general term "international agreement" included executive agreements, and whether
it was necessary to include an express proviso that would exclude executive agreements from the
requirement of Senate concurrence. After noted constitutionalist Fr. Joaquin Bernas quoted the
Court's ruling in Eastern Sea Trading, the Constitutional Commission members ultimately decided
that the term "international agreements" as contemplated in Section 21, Article VII, does not include
executive agreements, and that a proviso is no longer needed. Their discussion is reproduced
below: 207

MS. AQUINO: Madam President, first I would like a clarification from the Committee. We have
retained the words "international agreement" which I think is the correct judgment on the matter
because an international agreement is different from a treaty. A treaty is a contract between parties
which is in the nature of international agreement and also a municipal law in the sense that the
people are bound. So there is a conceptual difference. However, I would like to be clarified if the
international agreements include executive agreements.

MR. CONCEPCION: That depends upon the parties. All parties to these international negotiations
stipulate the conditions which are necessary for the agreement or whatever it may be to become
valid or effective as regards the parties.

MS. AQUINO: Would that depend on the parties or would that depend on the nature of the executive
agreement? According to common usage, there are two types of executive agreement: one is
purely proceeding from an executive act which affects external relations independent of the
legislative and the other is an executive act in pursuance of legislative authorization. The first
kind might take the form of just conventions or exchanges of notes or protocol while the other,
which would be pursuant to the legislative authorization, may be in the nature of commercial
agreements.
MR. CONCEPCION: Executive agreements are generally made to implement a treaty already
enforced or to determine the details for the implementation of the treaty. We are speaking of
executive agreements, not international agreements.

MS. AQUINO: I am in full agreement with that, except that it does not cover the first kind of executive
agreement which is just protocol or an exchange of notes and this would be in the nature of
reinforcement of claims of a citizen against a country, for example.

MR. CONCEPCION: The Commissioner is free to require ratification for validity insofar as the
Philippines is concerned.

MS. AQUINO: It is my humble submission that we should provide, unless the Committee explains


to us otherwise, an explicit proviso which would except executive agreements from
the requirement of concurrence of two-thirds of the Members of the Senate. Unless I am
enlightened by the Committee I propose that tentatively, the sentence should read. "No treaty or
international agreement EXCEPT EXECUTIVE AGREEMENTS shall be valid and effective."

FR. BERNAS: I wonder if a quotation from the Supreme Court decision [in Eastern Sea
Trading] might help clarify this:

The right of the executive to enter into binding agreements without the necessity of
subsequent Congressional approval has been confirmed by long usage. From the earliest days
of our history, we have entered into executive agreements covering such subjects as commercial
and consular relations, most favored nation rights, patent rights, trademark and copyright protection,
postal and navigation arrangements and the settlement of claims. The validity of this has never been
seriously questioned by our Courts.

Agreements with respect to the registration of trademarks have been concluded by the executive of
various countries under the Act of Congress of March 3, 1881 (21 Stat. 502) . . . International
agreements involving political issues or changes of national policy and those involving
international agreements of a permanent character usually take the form of treaties. But
international agreements embodying adjustments of detail, carrying out well established
national policies and traditions and those involving arrangements of a more or less temporary
nature usually take the form of executive agreements.

MR. ROMULO: Is the Commissioner, therefore, excluding the executive agreements?

FR. BERNAS: What we are referring to, therefore, when we say international agreements which
need concurrence by at least two-thirds are those which are permanent in nature.

MS. AQUINO: And it may include commercial agreements which are executive agreements
essentially but which are proceeding from the authorization of Congress. If that is our understanding,
then I am willing to withdraw that amendment.

FR. BERNAS: If it is with prior authorization of Congress, then it does not need subsequent
concurrence by Congress.

MS. AQUINO: In that case, I am withdrawing my amendment.

MR. TINGSON: Madam President.


THE PRESIDENT: Is Commissioner Aquino satisfied?

MS. AQUINO: Yes. There is already an agreement among us on the definition of "executive
agreements" and that would make unnecessary any explicit proviso on the matter.

xxx

MR. GUINGONA: I am not clear as to the meaning of "executive agreements" because I heard that
these executive agreements must rely on treaties. In other words, there must first be treaties.

MR. CONCEPCION: No, I was speaking about the common use, as executive agreements being the
implementation of treaties, details of which do not affect the sovereignty of the State.

MR. GUINGONA: But what about the matter of permanence, Madam President? Would 99 years be
considered permanent? What would be the measure of permanency? I do not conceive of a treaty
that is going to be forever, so there must be some kind of a time limit.

MR. CONCEPCION: I suppose the Commissioner's question is whether this type of agreement
should be included in a provision of the Constitution requiring the concurrence of Congress.

MR. GUINGONA: It depends on the concept of the executive agreement of which I am not clear. If
the executive agreement partakes of the nature of a treaty, then it should also be included.

MR. CONCEPCION: Whether it partakes or not of the nature of a treaty, it is within the power of the
Constitutional Commission to require that.

MR. GUINGONA: Yes. That is why I am trying to clarify whether the words "international
agreements" would include executive agreements.

MR. CONCEPCION: No, not necessarily; generally no.

xxx

MR. ROMULO: I wish to be recognized first. I have only one question. Do we take it, therefore,
that as far as the Committee is concerned, the term "international agreements" does not
include the term "executive agreements" as read by the Commissioner in that text?

FR. BERNAS: Yes. (Emphases Supplied)

The inapplicability to executive agreements of the requirements under Section 21 was again
recognized in Bayan v. Zamora and in Bayan Muna v. Romulo. These cases, both decided under
the aegis of the present Constitution, quoted Eastern Sea Trading in reiterating that executive
agreements are valid and binding even without the concurrence of the Senate.

Executive agreements may dispense with the requirement of Senate concurrence because of the
legal mandate with which they are concluded. As culled from the afore-quoted deliberations of the
Constitutional Commission, past Supreme Court Decisions, and works of noted scholars,  executive
208

agreements merely involve arrangements on the implementation of existing policies, rules, laws, or


agreements. They are concluded (1) to adjust the details of a treaty;  (2) pursuant to or upon
209

confirmation by an act of the Legislature;  or (3) in the exercise of the President's independent
210
powers under the Constitution.  The raison
211
d'etre of executive agreements hinges
on prior constitutional or legislative authorizations.

The special nature of an executive agreement is not just a domestic variation in international
agreements. International practice has accepted the use of various forms and designations of
international agreements, ranging from the traditional notion of a treaty - which connotes a formal,
solemn instrument - to engagements concluded in modem, simplified forms that no longer
necessitate ratification.  An international agreement may take different forms: treaty, act, protocol,
212

agreement, concordat, compromis d'arbitrage, convention, covenant, declaration, exchange of


notes, statute, pact, charter, agreed minute, memorandum of agreement, modus vivendi, or some
other form.  Consequently, under international law, the distinction between a treaty and an
213

international agreement or even an executive agreement is irrelevant for purposes of determining


international rights and obligations.

However, this principle does not mean that the domestic law distinguishing treaties, international
agreements, and executive agreements is relegated to a mere variation in form, or that the
constitutional requirement of Senate concurrence is demoted to an optional constitutional directive.
There remain two very important features that distinguish treaties from executive agreements and
translate them into terms of art in the domestic setting.

First, executive agreements must remain traceable to an express or implied authorization under the
Constitution, statutes, or treaties. The absence of these precedents puts the validity and effectivity of
executive agreements under serious question for the main function of the Executive is to enforce the
Constitution and the laws enacted by the Legislature, not to defeat or interfere in the performance of
these rules.  In turn, executive agreements cannot create new international obligations that are not
214

expressly allowed or reasonably implied in the law they purport to implement.

Second, treaties are, by their very nature, considered superior to executive agreements. Treaties
are products of the acts of the Executive and the Senate  unlike executive agreements, which are
215

solely executive actions. Because of legislative participation through the Senate, a treaty is
216

regarded as being on the same level as a statute.  If there is an irreconcilable conflict, a later law or
217

treaty takes precedence over one that is prior.  An executive agreement is treated differently.
218

Executive agreements that are inconsistent with either a law or a treaty are considered
ineffective.  Both types of international agreement are nevertheless subject to the supremacy of the
219

Constitution. 220

This rule does not imply, though, that the President is given carte blanche to exercise this discretion.
Although the Chief Executive wields the exclusive authority to conduct our foreign relations, this
power must still be exercised within the context and the parameters set by the Constitution, as well
as by existing domestic and international laws. There are constitutional provisions that restrict or limit
the President's prerogative in concluding international agreements, such as those that involve the
following:

a. The policy of freedom from nuclear weapons within Philippine territory 221

b. The fixing of tariff rates, import and export quotas, tonnage and wharfage dues, and other
duties or imposts, which must be pursuant to the authority granted by Congress 222

c. The grant of any tax exemption, which must be pursuant to a law concurred in by a
majority of all the Members of Congress 223
d. The contracting or guaranteeing, on behalf of the Philippines, of foreign loans that must be
previously concurred in by the Monetary Board 224

e. The authorization of the presence of foreign military bases, troops, or facilities in the
country must be in the form of a treaty duly concurred in by the Senate. 225

f. For agreements that do not fall under paragraph 5, the concurrence of the Senate is
required, should the form of the government chosen be a treaty.

5. The President had the choice to enter into EDCA by way of an executive agreement or a
treaty.

No court can tell the President to desist from choosing an executive agreement over a treaty to
embody an international agreement, unless the case falls squarely within Article VIII, Section 25.

As can be gleaned from the debates among the members of the Constitutional Commission, they
were aware that legally binding international agreements were being entered into by countries in
forms other than a treaty. At the same time, it is clear that they were also keen to preserve the
concept of "executive agreements" and the right of the President to enter into such agreements.

What we can glean from the discussions of the Constitutional Commissioners is that they
understood the following realities:

1. Treaties, international agreements, and executive agreements are all constitutional


manifestations of the conduct of foreign affairs with their distinct legal characteristics.

a. Treaties are formal contracts between the Philippines and other States-parties,
which are in the nature of international agreements, and also of municipal laws in the
sense of their binding nature. 226

b. International agreements are similar instruments, the provisions of which may


require the ratification of a designated number of parties thereto. These agreements
involving political issues or changes in national policy, as well as those involving
international agreements of a permanent character, usually take the form of treaties.
They may also include commercial agreements, which are executive agreements
essentially, but which proceed from previous authorization by Congress, thus
dispensing with the requirement of concurrence by the Senate. 227

c. Executive agreements are generally intended to implement a treaty already


enforced or to determine the details of the implementation thereof that do not affect
the sovereignty of the State. 228

2. Treaties and international agreements that cannot be mere executive agreements must,
by constitutional decree, be concurred in by at least two-thirds of the Senate.

3. However, an agreement - the subject of which is the entry of foreign military troops, bases,
or facilities - is particularly restricted. The requirements are that it be in the form of a treaty
concurred in by the Senate; that when Congress so requires, it be ratified by a majority of the
votes cast by the people in a national referendum held for that purpose; and that it be
recognized as a treaty by the other contracting State.
4. Thus, executive agreements can continue to exist as a species of international
agreements.

That is why our Court has ruled the way it has in several cases.

In Bayan Muna v. Romulo, we ruled that the President acted within the scope of her constitutional
authority and discretion when she chose to enter into the RP-U.S. Non-Surrender Agreement in the
form of an executive agreement, instead of a treaty, and in ratifying the agreement without Senate
concurrence. The Court en banc discussed this intrinsic presidential prerogative as follows:

Petitioner parlays the notion that the Agreement is of dubious validity, partaking as it does of the
nature of a treaty; hence, it must be duly concurred in by the Senate. x x x x. Pressing its point,
petitioner submits that the subject of the Agreement does not fall under any of the subject-categories
that xx x may be covered by an executive agreement, such as commercial/consular relations, most-
favored nation rights, patent rights, trademark and copyright protection, postal and navigation
arrangements and settlement of claims.

The categorization of subject matters that may be covered by international agreements mentioned
in Eastern Sea Trading is not cast in stone. There are no hard and fast rules on the propriety of
entering, on a given subject, into a treaty or an executive agreement as an instrument of
international relations. The primary consideration in the choice of the form of agreement is
the parties' intent and desire to craft an international agreement in the form they so wish to
further their respective interests. Verily, the matter of form takes a back seat when it comes to
effectiveness and binding effect of the enforcement of a treaty or an executive agreement, as the
parties in either international agreement each labor under the pacta sunt servanda principle.

xxxx

But over and above the foregoing considerations is the fact that - save for the situation and matters
contemplated in Sec. 25, Art. XVIII of the Constitution - when a treaty is required, the Constitution
does not classify any subject, like that involving political issues, to be in the form of, and
ratified as, a treaty. What the Constitution merely prescribes is that treaties need the concurrence
of the Senate by a vote defined therein to complete the ratification process.

xxxx

x x x. As the President wields vast powers and influence, her conduct in the external affairs of the
nation is, as Bayan would put it, "executive altogether." The right of the President to enter into or
ratify binding executive agreements has been confirmed by long practice.

In thus agreeing to conclude the Agreement thru E/N BF0-028-03, then President Gloria
Macapagal-Arroyo, represented by the Secretary of Foreign Affairs, acted within the scope of the
authority and discretion vested in her by the Constitution. At the end of the day, the President -
by ratifying, thru her deputies, the non-surrender agreement - did nothing more than
discharge a constitutional duty and exercise a prerogative that pertains to her
office. (Emphases supplied)

Indeed, in the field of external affairs, the President must be given a larger measure of authority and
wider discretion, subject only to the least amount of checks and restrictions under the
Constitution.  The rationale behind this power and discretion was recognized by the Court in Vinuya
229

v. Executive Secretary, cited earlier.


230
Section 9 of Executive Order No. 459, or the Guidelines in the Negotiation of International
Agreements and its Ratification, thus, correctly reflected the inherent powers of the President when it
stated that the DFA "shall determine whether an agreement is an executive agreement or a treaty."

Accordingly, in the exercise of its power of judicial review, the Court does not look into whether an
international agreement should be in the form of a treaty or an executive agreement, save in cases
in which the Constitution or a statute requires otherwise. Rather, in view of the vast constitutional
powers and prerogatives granted to the President in the field of foreign affairs, the task of the Court
is to determine whether the international agreement is consistent with the applicable limitations.

6. Executive agreements may cover the matter of foreign military forces if it merely involves
detail adjustments.

The practice of resorting to executive agreements in adjusting the details of a law or a treaty that
already deals with the presence of foreign military forces is not at all unusual in this jurisdiction. In
fact, the Court has already implicitly acknowledged this practice in Lim v. Executive Secretary.  In231

that case, the Court was asked to scrutinize the constitutionality of the Terms of Reference of
the Balikatan 02-1 joint military exercises, which sought to implement the VFA. Concluded in the
form of an executive agreement, the Terms of Reference detailed the coverage of the term
"activities" mentioned in the treaty and settled the matters pertaining to the construction of temporary
structures for the U.S. troops during the activities; the duration and location of the exercises; the
number of participants; and the extent of and limitations on the activities of the U.S. forces. The
Court upheld the Terms of Reference as being consistent with the VFA. It no longer took issue with
the fact that the Balikatan Terms of Reference was not in the form of a treaty concurred in by the
Senate, even if it dealt with the regulation of the activities of foreign military forces on Philippine
territory.

In Nicolas v. Romulo,  the Court again impliedly affirmed the use of an executive agreement in an
232

attempt to adjust the details of a provision of the VFA. The Philippines and the U.S. entered into the
Romulo-Kenney Agreement, which undertook to clarify the detention of a U.S. Armed Forces
member, whose case was pending appeal after his conviction by a trial court for the crime of rape. In
testing the validity of the latter agreement, the Court precisely alluded to one of the inherent
limitations of an executive agreement: it cannot go beyond the terms of the treaty it purports to
implement. It was eventually ruled that the Romulo-Kenney Agreement was "not in accord" with the
VFA, since the former was squarely inconsistent with a provision in the treaty requiring that the
detention be "by Philippine authorities." Consequently, the Court ordered the Secretary of Foreign
Affairs to comply with the VFA and "forthwith negotiate with the United States representatives for the
appropriate agreement on detention facilities under Philippine authorities as provided in Art. V, Sec.
10 of the VFA. "233

Culling from the foregoing discussions, we reiterate the following pronouncements to guide us in
resolving the present controversy:

1. Section 25, Article XVIII of the Constitution, contains stringent requirements that must be
fulfilled by the international agreement allowing the presence of foreign military bases,
troops, or facilities in the Philippines: (a) the agreement must be in the form of a treaty, and
(b) it must be duly concurred in by the Senate.

2. If the agreement is not covered by the above situation, then the President may choose the
form of the agreement (i.e., either an executive agreement or a treaty), provided that the
agreement dealing with foreign military bases, troops, or facilities is not the principal
agreement that first allows their entry or presence in the Philippines.
3. The executive agreement must not go beyond the parameters, limitations, and standards
set by the law and/or treaty that the former purports to implement; and must not unduly
expand the international obligation expressly mentioned or necessarily implied in the law or
treaty.

4. The executive agreement must be consistent with the Constitution, as well as with existing
laws and treaties.

In light of the President's choice to enter into EDCA in the form of an executive agreement,
respondents carry the burden of proving that it is a mere implementation of existing laws and treaties
concurred in by the Senate. EDCA must thus be carefully dissected to ascertain if it remains within
the legal parameters of a valid executive agreement.

7. EDCA is consistent with the content, purpose, and framework of the MDT and the VFA

The starting point of our analysis is the rule that "an executive agreement xx x may not be used to
amend a treaty."  In Lim v. Executive Secretary and in Nicolas v. Romulo, the Court approached
234

the question of the validity of executive agreements by comparing them with the general framework
and the specific provisions of the treaties they seek to implement.

In Lim, the Terms of Reference of the joint military exercises was scrutinized by studying "the
framework of the treaty antecedents to which the Philippines bound itself,"  i.e., the MDT and the
235

VFA. The Court proceeded to examine the extent of the term "activities" as contemplated in Articles
1  and II  of the VFA. It later on found that the term "activities" was deliberately left undefined and
236 237

ambiguous in order to permit "a wide scope of undertakings subject only to the approval of the
Philippine government"  and thereby allow the parties "a certain leeway in negotiation."  The Court
238 239

eventually ruled that the Terms of Reference fell within the sanctioned or allowable activities,
especially in the context of the VFA and the MDT.

The Court applied the same approach to Nicolas v. Romulo. It studied the provisions of the VFA on
custody and detention to ascertain the validity of the Romulo-Kenney Agreement.  It eventually240

found that the two international agreements were not in accord, since the Romulo-Kenney
Agreement had stipulated that U.S. military personnel shall be detained at the U.S. Embassy
Compound and guarded by U.S. military personnel, instead of by Philippine authorities. According to
the Court, the parties "recognized the difference between custody during the trial and detention after
conviction."  Pursuant to Article V(6) of the VFA, the custody of a U.S. military personnel resides
241

with U.S. military authorities during trial. Once there is a finding of guilt, Article V(l0) requires that the
confinement or detention be "by Philippine authorities."

Justice Marvic M.V.F. Leonen's Dissenting Opinion posits that EDCA "substantially modifies or
amends the VFA" and follows with an enumeration of the differences between EDCA and the VFA.
242

While these arguments will be rebutted more fully further on, an initial answer can already be given
to each of the concerns raised by his dissent.

The first difference emphasized is that EDCA does not only regulate visits as the VFA does, but
allows temporary stationing on a rotational basis of U.S. military personnel and their contractors in
physical locations with permanent facilities and pre-positioned military materiel.

This argument does not take into account that these permanent facilities, while built by U.S. forces,
are to be owned by the Philippines once constructed.  Even the VFA allowed construction for the
243

benefit of U.S. forces during their temporary visits.


The second difference stated by the dissent is that EDCA allows the prepositioning of military
materiel, which can include various types of warships, fighter planes, bombers, and vessels, as well
as land and amphibious vehicles and their corresponding ammunition. 244

However, the VFA clearly allows the same kind of equipment, vehicles, vessels, and aircraft to be
brought into the country. Articles VII and VIII of the VFA contemplates that U.S. equipment,
materials, supplies, and other property are imported into or acquired in the Philippines by or on
behalf of the U.S. Armed Forces; as are vehicles, vessels, and aircraft operated by or for U.S. forces
in connection with activities under the VFA. These provisions likewise provide for the waiver of the
specific duties, taxes, charges, and fees that correspond to these equipment.

The third difference adverted to by the Justice Leonen's dissent is that the VFA contemplates the
entry of troops for training exercises, whereas EDCA allows the use of territory for launching military
and paramilitary operations conducted in other states.  The dissent of Justice Teresita J. Leonardo-
245

De Castro also notes that VFA was intended for non-combat activides only, whereas the entry and
activities of U.S. forces into Agreed Locations were borne of military necessity or had a martial
character, and were therefore not contemplated by the VFA. 246

This Court's jurisprudence however established in no uncertain terms that combat-related activities,
as opposed to actual combat, were allowed under the MDT and VFA, viz:

Both the history and intent of the Mutual Defense Treaty and the VFA support the conclusion that
combat-related activities as opposed to combat itself such as the one subject of the instant petition,
are indeed authorized. 247

Hence, even if EDCA was borne of military necessity, it cannot be said to have strayed from the
intent of the VFA since EDCA's combat-related components are allowed under the treaty.

Moreover, both the VFA and EDCA are silent on what these activities actually are. Both the VFA and
EDCA deal with the presence of U.S. forces within the Philippines, but make no mention of being
platforms for activity beyond Philippine territory. While it may be that, as applied, military operations
under either the VFA or EDCA would be carried out in the future the scope of judicial review does
not cover potential breaches of discretion but only actual occurrences or blatantly illegal provisions.
Hence, we cannot invalidate EDCA on the basis of the potentially abusive use of its provisions.

The fourth difference is that EDCA supposedly introduces a new concept not contemplated in the
VFA or the MDT: Agreed Locations, Contractors, Pre-positioning, and Operational Control. 248

As previously mentioned, these points shall be addressed fully and individually in the latter analysis
of EDCA's provisions. However, it must already be clarified that the terms and details used by an
implementing agreement need not be found in the mother treaty. They must be sourced from the
authority derived from the treaty, but are not necessarily expressed word-for-word in the mother
treaty. This concern shall be further elucidated in this Decision.

The fifth difference highlighted by the Dissenting Opinion is that the VFA does not have provisions
that may be construed as a restriction on or modification of obligations found in existing statues,
including the jurisdiction of courts, local autonomy, and taxation. Implied in this argument is that
EDCA contains such restrictions or modifications. 249

This last argument cannot be accepted in view of the clear provisions of EDCA. Both the VFA and
EDCA ensure Philippine jurisdiction in all instances contemplated by both agreements, with the
exception of those outlined by the VFA in Articles III-VI. In the VFA, taxes are clearly waived
whereas in EDCA, taxes are assumed by the government as will be discussed later on. This fact
does not, therefore, produce a diminution of jurisdiction on the part of the Philippines, but rather a
recognition of sovereignty and the rights that attend it, some of which may be waived as in the cases
under Articles III-VI of the VFA.

Taking off from these concerns, the provisions of EDCA must be compared with those of the MDT
and the VFA, which are the two treaties from which EDCA allegedly draws its validity.

"Authorized presence" under the VFA versus "authorized activities" under EDCA: (1) U.S.
personnel and (2) U.S. contractors

The OSG argues  that EDCA merely details existing policies under the MDT and the VFA. It
250

explains that EDCA articulates the principle of defensive preparation embodied in Article II of the


MDT; and seeks to enhance the defensive, strategic, and technological capabilities of both parties
pursuant to the objective of the treaty to strengthen those capabilities to prevent or resist a possible
armed attack. Respondent also points out that EDCA simply implements Article I of the VFA, which
already allows the entry of U.S. troops and personnel into the country. Respondent stresses this
Court's recognition in Lim v. Executive Secretary that U.S. troops and personnel are authorized to
conduct activities that promote the goal of maintaining and developing their defense capability.

Petitioners contest  the assertion that the provisions of EDCA merely implement the MDT.
251

According to them, the treaty does not specifically authorize the entry of U.S. troops in the country in
order to maintain and develop the individual and collective capacities of both the Philippines and the
U.S. to resist an armed attack. They emphasize that the treaty was concluded at a time when there
was as yet no specific constitutional prohibition on the presence of foreign military forces in the
country.

Petitioners also challenge the argument that EDCA simply implements the VFA. They assert that the
agreement covers only short-term or temporary visits of U.S. troops "from time to time" for the
specific purpose of combined military exercises with their Filipino counterparts. They stress that, in
contrast, U.S. troops are allowed under EDCA to perform activities beyond combined military
exercises, such as those enumerated in Articles 111(1) and IV(4) thereof. Furthermore, there is
some degree of permanence in the presence of U.S. troops in the country, since the effectivity of
EDCA is continuous until terminated. They proceed to argue that while troops have a "rotational"
presence, this scheme in fact fosters their permanent presence.

a. Admission of U.S. military and civilian personnel into Philippine territory is already allowed under
the VFA

We shall first deal with the recognition under EDCA of the presence in the country of three distinct
classes of individuals who will be conducting different types of activities within the Agreed Locations:
(1) U.S. military personnel; (2) U.S. civilian personnel; and (3) U.S. contractors. The agreement
refers to them as follows:

"United States personnel" means United States military and civilian personnel temporarily in


the territory of the Philippines in connection with activities approved by the Philippines, as those
terms are defined in the VFA. 252

"United States forces" means the entity comprising United States personnel and all property,


equipment, and materiel of the United States Armed Forces present in the territory of the
Philippines.
253
"United States contractors" means companies and firms, and their employees, under contract
or subcontract to or on behalf of the United States Department of Defense. United States
contractors are not included as part of the definition of United States personnel in this Agreement,
including within the context of the VFA. 254

United States forces may contract for any materiel, supplies, equipment, and


services (including construction) to be furnished or undertaken in the territory of the Philippines
without restriction as to choice of contractor, supplier, or person who provides such materiel,
supplies, equipment, or services. Such contracts shall be solicited, awarded, and administered in
accordance with the laws and regulations of the United States.  (Emphases Supplied)
255

A thorough evaluation of how EDCA is phrased clarities that the agreement does not deal with
the entry into the country of U.S. personnel and contractors per se. While Articles I(l)(b)  and 256

II(4)  speak of "the right to access and use" the Agreed Locations, their wordings indicate the
257

presumption that these groups have already been allowed entry into Philippine territory, for which,
unlike the VFA, EDCA has no specific provision. Instead, Article II of the latter simply alludes to the
VFA in describing U.S. personnel, a term defined under Article I of the treaty as follows:

As used in this Agreement, "United States personnel" means United States military and civilian
personnel temporarily in the Philippines in connection with activities approved by the Philippine
Government. Within this definition:

1. The term "military personnel" refers to military members of the United States


Army, Navy, Marine Corps, Air Force, and Coast Guard.

2. The term "civilian personnel" refers to individuals who are neither nationals of


nor ordinarily resident in the Philippines and who are employed by the United
States armed forces or who are accompanying the United States armed forces,
such as employees of the American Red Cross and the United Services
Organization. 258

Article II of EDCA must then be read with Article III of the VFA, which provides for the entry
accommodations to be accorded to U.S. military and civilian personnel:

1. The Government of the Philippines shall facilitate the admission of United States


personnel and their departure from the Philippines in connection with activities covered by
this agreement.

2. United States military personnel shall be exempt from passport and visa regulations
upon enteringand departing the Philippines.

3. The following documents only, which shall be required in respect of United States military
personnel who enter the Philippines; xx xx.

4. United States civilian personnel shall be exempt from visa requirements but shall


present, upon demand, valid passports upon entry and departure of the Philippines.
(Emphases Supplied)

By virtue of Articles I and III of the VFA, the Philippines already allows U.S. military and civilian
personnel to be "temporarily in the Philippines," so long as their presence is "in connection with
activities approved by the Philippine Government." The Philippines, through Article III, even
guarantees that it shall facilitate the admission of U.S. personnel into the country and grant
exemptions from passport and visa regulations. The VFA does not even limit their temporary
presence to specific locations.

Based on the above provisions, the admission and presence of U.S. military and civilian
personnel in Philippine territory are already allowed under the VFA, the treaty supposedly
being implemented by EDCA. What EDCA has effectively done, in fact, is merely provide the
mechanism to identify the locations in which U.S. personnel may perform allowed activities pursuant
to the VFA. As the implementing agreement, it regulates and limits the presence of U.S. personnel in
the country.

b. EDCA does not provide the legal basis for admission of U.S. contractors into Philippine territory;
their entry must be sourced from extraneous Philippine statutes and regulations for the admission of
alien employees or business persons.

Of the three aforementioned classes of individuals who will be conducting certain activities within the
Agreed Locations, we note that only U.S. contractors are not explicitly mentioned in the VFA. This
does not mean, though, that the recognition of their presence under EDCA is ipso facto an
amendment of the treaty, and that there must be Senate concurrence before they are allowed to
enter the country.

Nowhere in EDCA are U.S. contractors guaranteed immediate admission into the Philippines.
Articles III and IV, in fact, merely grant them the right of access to, and the authority to conduct
certain activities within the Agreed Locations. Since Article II(3) of EDCA specifically leaves out U.S.
contractors from the coverage of the VFA, they shall not be granted the same entry
accommodations and privileges as those enjoyed by U.S. military and civilian personnel under the
VFA.

Consequently, it is neither mandatory nor obligatory on the part of the Philippines to admit U.S.
contractors into the country.  We emphasize that the admission of aliens into Philippine territory is
259

"a matter of pure permission and simple tolerance which creates no obligation on the part of the
government to permit them to stay."  Unlike U.S. personnel who are accorded entry
260

accommodations, U.S. contractors are subject to Philippine immigration laws. The latter must
261

comply with our visa and passport regulations  and prove that they are not subject to exclusion
262

under any provision of Philippine immigration laws.  The President may also deny them entry
263

pursuant to his absolute and unqualified power to prohibit or prevent the admission of aliens whose
presence in the country would be inimical to public interest. 264

In the same vein, the President may exercise the plenary power to expel or deport U.S.
contractors  as may be necessitated by national security, public safety, public health, public morals,
265

and national interest.  They may also be deported if they are found to be illegal or undesirable
266

aliens pursuant to the Philippine Immigration Act  and the Data Privacy Act.  In contrast, Article
267 268

111(5) of the VFA requires a request for removal from the Philippine government before a member
of the U.S. personnel may be "dispos[ed] xx x outside of the Philippines."

c. Authorized activities of U.S. military and civilian personnel within Philippine territory are in
furtherance of the MDT and the VFA

We begin our analysis by quoting the relevant sections of the MDT and the VFA that pertain to the
activities in which U.S. military and civilian personnel may engage:

MUTUAL DEFENSE TREATY


Article II

In order more effectively to achieve the objective of this Treaty, the Parties separately and jointly
byself-help and mutual aid will maintain and develop their individual and collective capacity to
resist armed attack.

Article III

The Parties, through their Foreign Ministers or their deputies, will consult together from time to
time regarding the implementation of this Treaty and whenever in the opinion of either of them the
territorial integrity, political independence or security of either of the Parties is threatened by external
armed attack in the Pacific.

VISITING FORCES AGREEMENT

Preamble

xxx

Reaffirming their obligations under the Mutual Defense Treaty of August 30, 1951;

Noting that from time to time elements of the United States armed forces may visit the Republic of
the Philippines;

Considering that cooperation between the United States and the Republic of the


Philippines promotes their common security interests;

xxx

Article I - Definitions

As used in this Agreement, "United States personnel" means United States military and civilian
personnel temporarily in the Philippines in connection with activities approved by the Philippine
Government. Within this definition: xx x

Article II - Respect for Law

It is the duty of United States personnel to respect the laws of the Republic of the
Philippines and to abstain from any activity inconsistent with the spirit of this agreement,
and, in particular, from any political activity in the Philippines. The Government of the United States
shall take all measures within its authority to ensure that this is done.

Article VII - Importation and Exportation

1. United States Government equipment, materials, supplies, and other property imported into
or acquired in the Philippines by or on behalf of the United States armed forces in connection with
activities to which this agreement applies, shall be free of all Philippine duties, taxes and other
similar charges. Title to such property shall remain with the United States, which may remove such
property from the Philippines at any time, free from export duties, taxes, and other similar charges. x
x x.
Article VIII - Movement of Vessels and Aircraft

1. Aircraft operated by or for the United States armed forces may enter the Philippines upon
approval of the Government of the Philippines in accordance with procedures stipulated
in implementing arrangements.

2. Vessels operated by or for the United States armed forces may enter the Philippines upon
approval of the Government of the Philippines. The movement of vessels shall be in
accordance with international custom and practice governing such vessels, and such agreed
implementing arrangements as necessary. x x x (Emphases Supplied)

Manifest in these provisions is the abundance of references to the creation of further "implementing
arrangements" including the identification of "activities [to be] approved by the Philippine
Government." To determine the parameters of these implementing arrangements and activities, we
referred to the content, purpose, and framework of the MDT and the VFA.

By its very language, the MDT contemplates a situation in which both countries shall engage
in joint activities, so that they can maintain and develop their defense capabilities. The wording itself
evidently invites a reasonable construction that the joint activities shall involve joint military trainings,
maneuvers, and exercises. Both the interpretation  and the subsequent practice  of the parties
269 270

show that the MDT independently allows joint military exercises in the country. Lim v. Executive
Secretary  and Nicolas v. Romulo  recognized that Balikatan exercises, which are activities that
271 272

seek to enhance and develop the strategic and technological capabilities of the parties to resist an
armed attack, "fall squarely under the provisions of the RP-US MDT."  In Lim, the Court especially
273

noted that the Philippines and the U.S. continued to conduct joint military exercises even after the
expiration of the MBA and even before the conclusion of the VFA.  These activities presumably
274

related to the Status of Forces Agreement, in which the parties agreed on the status to be accorded
to U.S. military and civilian personnel while conducting activities in the Philippines in relation to the
MDT. 275

Further, it can be logically inferred from Article V of the MDT that these joint activities may be
conducted on Philippine or on U.S. soil. The article expressly provides that the term armed
attack includes "an armed attack on the metropolitan territory of either of the Parties, or on
the island territories under its jurisdiction in the Pacific or on its armed forces, public vessels
or aircraft in the Pacific." Surely, in maintaining and developing our defense capabilities, an
assessment or training will need to be performed, separately and jointly by self-help and mutual aid,
in the territories of the contracting parties. It is reasonable to conclude that the assessment of
defense capabilities would entail understanding the terrain, wind flow patterns, and other
environmental factors unique to the Philippines.

It would also be reasonable to conclude that a simulation of how to respond to attacks in vulnerable
areas would be part of the training of the parties to maintain and develop their capacity to resist an
actual armed attack and to test and validate the defense plan of the Philippines. It is likewise
reasonable to imagine that part of the training would involve an analysis of the effect of the weapons
that may be used and how to be prepared for the eventuality. This Court recognizes that all of this
may require training in the area where an armed attack might be directed at the Philippine territory.

The provisions of the MDT must then be read in conjunction with those of the VFA.

Article I of the VFA indicates that the presence of U.S. military and civilian personnel in the
Philippines is "in connection with activities approved by the Philippine Government." While the treaty
does not expressly enumerate or detail the nature of activities of U.S. troops in the country, its
Preamble makes explicit references to the reaffirmation of the obligations of both countries under the
MDT. These obligations include the strengthening of international and regional security in the Pacific
area and the promotion of common security interests.

The Court has already settled in Lim v. Executive Secretary that the phrase "activities approved by
the Philippine Government" under Article I of the VFA was intended to be ambiguous in order to
afford the parties flexibility to adjust the details of the purpose of the visit of U.S. personnel.  In
276

ruling that the Terms of Reference for the Balikatan Exercises in 2002 fell within the context of the
treaty, this Court explained:

After studied reflection, it appeared farfetched that the ambiguity surrounding the meaning of


the word "activities" arose from accident. In our view, it was deliberately made that way to give
both parties a certain leeway in negotiation. In this manner, visiting US forces may sojourn in
Philippine territory for purposes other than military. As conceived, the joint exercises may
include training on new techniques of patrol and surveillance to protect the nation's marine
resources, sea search-and-rescue operations to assist vessels in distress, disaster relief operations,
civic action projects such as the building of school houses, medical and humanitarian missions, and
the like.

Under these auspices, the VFA gives legitimacy to the current Balikatan exercises. It is only logical
to assume that "Balikatan 02-1," a "mutual anti-terrorism advising, assisting and training
exercise," falls under the umbrella of sanctioned or allowable activities in the context of the
agreement. Both the history and intent of the Mutual Defense Treaty and the VFA support the
conclusion that combat-related activities - as opposed to combat itself- such as the one subject of
the instant petition, are indeed authorized. (Emphases Supplied)

The joint report of the Senate committees on foreign relations and on national defense and security
further explains the wide range and variety of activities contemplated in the VFA, and how these
activities shall be identified:
277

These joint exercises envisioned in the VFA are not limited to combat-related activities; they


have a wide range and variety. They include exercises that will reinforce the AFP's ability
to acquire new techniques of patrol and surveillance to protect the country's maritime
resources; sea-search and rescue operations to assist ships in distress; and disaster-relief
operations to aid the civilian victims of natural calamities, such as earthquakes, typhoons and tidal
waves.

xxxx

Joint activities under the VFA will include combat maneuvers; training in aircraft maintenance and
equipment repair; civic-action projects; and consultations and meetings of the Philippine-U.S. Mutual
Defense Board. It is at the level of the Mutual Defense Board-which is headed jointly by the Chief
of Staff of the AFP and the Commander in Chief of the U.S. Pacific Command-that the VFA
exercises are planned. Final approval of any activity involving U.S. forces is,
however, invariably given by the Philippine Government.

xxxx

Siazon clarified that it is not the VFA by itself that determines what activities will be
conductedbetween the armed forces of the U.S. and the Philippines. The VFA regulates and
provides the legal framework for the presence, conduct and legal status of U.S.
personnel while they are in the country for visits, joint exercises and other related activities.
(Emphases Supplied)

What can be gleaned from the provisions of the VFA, the joint report of the Senate
committees on foreign relations and on national defense and security, and the ruling of this
Court in Lim is that the "activities" referred to in the treaty are meant to be specified and
identified infurther agreements. EDCA is one such agreement.

EDCA seeks to be an instrument that enumerates the Philippine-approved activities of U.S.


personnel referred to in the VFA. EDCA allows U.S. military and civilian personnel to perform
"activities approved by the Philippines, as those terms are defined in the VFA"  and clarifies that
278

these activities include those conducted within the Agreed Locations:

1. Security cooperation exercises; joint and combined training activities; humanitarian assistance
and disaster relief activities; and such other activities as may be agreed upon by the Parties 279

2. Training; transit; support and related activities; refueling of aircraft; bunkering of vessels;
temporary maintenance of vehicles, vessels, and aircraft; temporary accommodation of personnel;
communications; prepositioning of equipment, supplies, and materiel; deployment of forces and
materiel; and such other activities as the Parties may agree 280

3. Exercise of operational control over the Agreed Locations for construction activities and other
types of activity, including alterations and improvements thereof 281

4. Exercise of all rights and authorities within the Agreed Locations that are necessary for their
operational control or defense, including the adoption of apfropriate measures to protect U.S. forces
and contractors 282

5. Use of water, electricity, and other public utilities 283

6. Operation of their own telecommunication systems, including the utilization of such means and
services as are required to ensure the full ability to operate telecommunication systems, as well as
the use of the necessary radio spectrum allocated for this purpose 284

According to Article I of EDCA, one of the purposes of these activities is to maintain and develop,
jointly and by mutual aid, the individual and collective capacities of both countries to resist an armed
attack. It further states that the activities are in furtherance of the MDT and within the context of the
VFA.

We note that these planned activities are very similar to those under the Terms of
Reference  mentioned in Lim. Both EDCA and the Terms of Reference authorize the U.S. to
285

perform the following: (a) participate in training exercises; (b) retain command over their forces; (c)
establish temporary structures in the country; (d) share in the use of their respective resources,
equipment and other assets; and (e) exercise their right to self-defense. We quote the relevant
portion of the Terms and Conditions as follows: 286

I. POLICY LEVEL

xxxx
No permanent US basing and support facilities shall be established. Temporary structures such as
those for troop billeting, classroom instruction and messing may be set up for use by RP and
US Forces during the Exercise.

The Exercise shall be implemented jointly by RP and US Exercise Co-Directors under the authority
of the Chief of Staff, AFP. In no instance will US Forces operate independently during field training
exercises (FTX). AFP and US Unit Commanders will retain command over their respective
forces under the overall authority of the Exercise Co-Directors. RP and US participants shall
comply with operational instructions of the AFP during the FTX.

The exercise shall be conducted and completed within a period of not more than six months, with the
projected participation of 660 US personnel and 3,800 RP Forces. The Chief of Staff, AFP shall
direct the Exercise Co-Directors to wind up and terminate the Exercise and other activities within the
six month Exercise period.

The Exercise is a mutual counter-terrorism advising, assisting and training Exercise relative to


Philippine efforts against the ASG, and will be conducted on the Island of Basilan. Further advising,
assisting and training exercises shall be conducted in Malagutay and the Zamboanga area. Related
activities in Cebu will be for support of the Exercise.

xx xx.

US exercise participants shall not engage in combat, without prejudice to their right of self-
defense.

These terms of Reference are for purposes of this Exercise only and do not create additional legal
obligations between the US Government and the Republic of the Philippines.

II. EXERCISE LEVEL

1. TRAINING

a. The Exercise shall involve the conduct of mutual military assisting, advising
and trainingof RP and US Forces with the primary objective of enhancing the
operational capabilities of both forces to combat terrorism.

b. At no time shall US Forces operate independently within RP territory.

c. Flight plans of all aircraft involved in the exercise will comply with the local air
traffic regulations.

2. ADMINISTRATION & LOGISTICS

xxxx

a. RP and US participating forces may share, in accordance with their respective laws and
regulations, in the use of their resources, equipment and other assets. They will use their
respective logistics channels. x x x. (Emphases Supplied)

After a thorough examination of the content, purpose, and framework of the MDT and the VFA, we
find that EDCA has remained within the parameters set in these two treaties. Just like the Terms of
Reference mentioned in Lim, mere adjustments in detail to implement the MDT and the VFA can be
in the form of executive agreements.

Petitioners assert  that the duration of the activities mentioned in EDCA is no longer consistent with
287

the temporary nature of the visits as contemplated in the VFA. They point out that Article XII(4) of
EDCA has an initial term of 10 years, a term automatically renewed unless the Philippines or the
U.S. terminates the agreement. According to petitioners, such length of time already has a badge of
permanency.

In connection with this, Justice Teresita J. Leonardo-De Castro likewise argues in her Concurring
and Dissenting Opinion that the VFA contemplated mere temporary visits from U.S. forces, whereas
EDCA allows an unlimited period for U.S. forces to stay in the Philippines.288

However, the provisions of EDCA directly contradict this argument by limiting itself to 10 years of
effectivity. Although this term is automatically renewed, the process for terminating the agreement is
unilateral and the right to do so automatically accrues at the end of the 10 year period. Clearly, this
method does not create a permanent obligation.

Drawing on the reasoning in Lim, we also believe that it could not have been by chance that the VFA
does not include a maximum time limit with respect to the presence of U.S. personnel in the country.
We construe this lack of specificity as a deliberate effort on the part of the Philippine and the U.S.
governments to leave out this aspect and reserve it for the "adjustment in detail" stage of the
implementation of the treaty. We interpret the subsequent, unconditional concurrence of the Senate
in the entire text of the VFA as an implicit grant to the President of a margin of appreciation in
determining the duration of the "temporary" presence of U.S. personnel in the country.

Justice Brion's dissent argues that the presence of U.S. forces under EDCA is "more permanent" in
nature. However, this argument has not taken root by virtue of a simple glance at its provisions on
289

the effectivity period. EDCA does not grant permanent bases, but rather temporary rotational access
to facilities for efficiency. As Professor Aileen S.P. Baviera notes:

The new EDCA would grant American troops, ships and planes rotational access to facilities of the
Armed Forces of the Philippines – but not permanent bases which are prohibited under the
Philippine Constitution - with the result of reducing response time should an external threat from a
common adversary crystallize. 290

EDCA is far from being permanent in nature compared to the practice of states as shown in other
defense cooperation agreements. For example, Article XIV(l) of the U.S.-Romania defense
agreement provides the following:

This Agreement is concluded for an indefinite period and shall enter into force in accordance with
the internal laws of each Party x x x. (emphasis supplied)

Likewise, Article 36(2) of the US-Poland Status of Forces Agreement reads:

This Agreement has been concluded for an indefinite period of time. It may be terminated by
written notification by either Party and in that event it terminates 2 years after the receipt of the
notification. (emphasis supplied)

Section VIII of US.-Denmark Mutual Support Agreement similarly provides:


8.1 This Agreement, which consists of a Preamble, SECTIONs I-VIII, and Annexes A and B, shall
become effective on the date of the last signature affixed below and shall remain in force until
terminated by the Parties, provided that it may be terminated by either Party upon 180 days written
notice of its intention to do so to the other Party. (emphasis supplied)

On the other hand, Article XXI(3) of the US.-Australia Force Posture Agreement provides a longer
initial term:

3. This Agreement shall have an initial term of 25 years and thereafter shall continue in force,
but may be terminated by either Party at any time upon one year's written notice to the other Party
through diplomatic channels. (emphasis supplied)

The phrasing in EDCA is similar to that in the U.S.-Australia treaty but with a term less than half of
that is provided in the latter agreement. This means that EDCA merely follows the practice of other
states in not specifying a non-extendible maximum term. This practice, however, does not
automatically grant a badge of permanency to its terms. Article XII(4) of EDCA provides very clearly,
in fact, that its effectivity is for an initial term of 10 years, which is far shorter than the terms of
effectivity between the U.S. and other states. It is simply illogical to conclude that the initial,
extendible term of 10 years somehow gives EDCA provisions a permanent character.

The reasoning behind this interpretation is rooted in the constitutional role of the President who, as
Commander-in-Chief of our armed forces, is the principal strategist of the nation and, as such, duty-
bound to defend our national sovereignty and territorial integrity;  who, as chief architect of our
291

foreign relations, is the head policymaker tasked to assess, ensure, and protect our national security
and interests;  who holds the most comprehensive and most confidential information about foreign
292

countries  that may affect how we conduct our external affairs; and who has unrestricted access to
293

highly classified military intelligence data  that may threaten the life of the nation. Thus, if after a
294

geopolitical prognosis of situations affecting the country, a belief is engendered that a much longer
period of military training is needed, the President must be given ample discretion to adopt
necessary measures including the flexibility to set an extended timetable.

Due to the sensitivity and often strict confidentiality of these concerns, we acknowledge that the
President may not always be able to candidly and openly discuss the complete situation being faced
by the nation. The Chief Executive's hands must not be unduly tied, especially if the situation calls
for crafting programs and setting timelines for approved activities. These activities may be necessary
for maintaining and developing our capacity to resist an armed attack, ensuring our national
sovereignty and territorial integrity, and securing our national interests. If the Senate decides that the
President is in the best position to define in operational terms the meaning of temporary in relation to
the visits, considered individually or in their totality, the Court must respect that policy decision. If the
Senate feels that there is no need to set a time limit to these visits, neither should we.

Evidently, the fact that the VFA does not provide specificity in regard to the extent of the "temporary"
nature of the visits of U.S. personnel does not suggest that the duration to which the President may
agree is unlimited. Instead, the boundaries of the meaning of the term temporary in Article I of the
treaty must be measured depending on the purpose of each visit or activity.  That purpose must be
295

analyzed on a case-by-case basis depending on the factual circumstances surrounding the


conclusion of the implementing agreement. While the validity of the President's actions will be
judged under less stringent standards, the power of this Court to determine whether there was grave
abuse of discretion remains unimpaired.
d. Authorized activities performed by US. contractors within Philippine territory - who were
legitimately permitted to enter the country independent of EDCA - are subject to relevant Philippine
statutes and regulations and must be consistent with the MDT and the VFA

Petitioners also raise  concerns about the U.S. government's purported practice of hiring private
296

security contractors in other countries. They claim that these contractors - one of which has already
been operating in Mindanao since 2004 - have been implicated in incidents or scandals in other
parts of the globe involving rendition, torture and other human rights violations. They also assert that
these contractors employ paramilitary forces in other countries where they are operating.

Under Articles III and IV of EDCA, U.S. contractors are authorized to perform only the following
activities:

1. Training; transit; support and related activities; refueling of aircraft; bunkering of vessels;
temporary maintenance of vehicles, vessels, and aircraft; temporary accommodation of
personnel; communications; prepositioning of equipment, supplies, and materiel; deployment
of forces and materiel; and such other activities as the Parties may agree 297

2. Prepositioning and storage of defense equipment, supplies, and materiel, including


delivery, management, inspection, use, maintenance, and removal of such equipment,
supplies and materiel 298

3. Carrying out of matters in accordance with, and to the extent permissible under, U.S. laws,
regulations, and policies 299

EDCA requires that all activities within Philippine territory be in accordance with Philippine law. This
means that certain privileges denied to aliens are likewise denied to foreign military contractors.
Relevantly, providing security and carrying, owning, and possessing firearms  are illegal for foreign
300 301

civilians.

The laws in place already address issues regarding the regulation of contractors. In the 2015
Foreign Investment Negative list,  the Executive Department has already identified corporations that
302

have equity restrictions in Philippine jurisdiction. Of note is No. 5 on the list - private security
agencies that cannot have any foreign equity by virtue of Section 4 of Republic Act No. 5487;  and 303

No. 15, which regulates contracts for the construction of defense-related structures based on
Commonwealth Act No. 541.

Hence, any other entity brought into the Philippines by virtue of EDCA must subscribe to corporate
and civil requirements imposed by the law, depending on the entity's corporate structure and the
nature of its business.

That Philippine laws extraneous to EDCA shall govern the regulation of the activities of U.S.
contractors has been clear even to some of the present members of the Senate.

For instance, in 2012, a U.S. Navy contractor, the Glenn Marine, was accused of spilling fuel in the
waters off Manila Bay.  The Senate Committee on Foreign Relations and the Senate Committee on
304

Environment and Natural Resources chairperson claimed environmental and procedural violations
by the contractor.  The U.S. Navy investigated the contractor and promised stricter guidelines to be
305

imposed upon its contractors.  The statement attributed to Commander Ron Steiner of the public
306

affairs office of the U.S. Navy's 7th Fleet - that U.S. Navy contractors are bound by Philippine laws -
is of particular relevance. The statement acknowledges not just the presence of the contractors, but
also the U.S. position that these contractors are bound by the local laws of their host state. This
stance was echoed by other U.S. Navy representatives. 307

This incident simply shows that the Senate was well aware of the presence of U.S. contractors for
the purpose of fulfilling the terms of the VFA. That they are bound by Philippine law is clear to all,
even to the U.S.

As applied to EDCA, even when U.S. contractors are granted access to the Agreed Locations, all
their activities must be consistent with Philippine laws and regulations and pursuant to the MDT and
the VFA.

While we recognize the concerns of petitioners, they do not give the Court enough justification to
strike down EDCA. In Lim v. Executive Secretary, we have already explained that we cannot take
judicial notice of claims aired in news reports, "not because of any issue as to their truth, accuracy,
or impartiality, but for the simple reason that facts must be established in accordance with the rules
of evidence."  What is more, we cannot move one step ahead and speculate that the alleged illegal
308

activities of these contractors in other countries would take place in the Philippines with certainty. As
can be seen from the above discussion, making sure that U.S. contractors comply with Philippine
laws is a function of law enforcement. EDCA does not stand in the way of law enforcement.

Nevertheless, we emphasize that U.S. contractors are explicitly excluded from the coverage of the
VFA. As visiting aliens, their entry, presence, and activities are subject to all laws and treaties
applicable within the Philippine territory. They may be refused entry or expelled from the country if
they engage in illegal or undesirable activities. There is nothing that prevents them from being
detained in the country or being subject to the jurisdiction of our courts. Our penal laws,  labor
309

laws,  and immigrations laws  apply to them and therefore limit their activities here. Until and
310 311

unless there is another law or treaty that specifically deals with their entry and activities, their
presence in the country is subject to unqualified Philippine jurisdiction.

EDCA does not allow the presence of U.S.-owned or -controlled military facilities and bases
in the Philippines

Petitioners Saguisag et al. claim that EDCA permits the establishment of U.S. military bases through
the "euphemistically" termed "Agreed Locations. "  Alluding to the definition of this term in Article
312

II(4) of EDCA, they point out that these locations are actually military bases, as the definition refers
to facilities and areas to which U.S. military forces have access for a variety of purposes. Petitioners
claim that there are several badges of exclusivity in the use of the Agreed Locations by U.S.
forces. First, Article V(2) of EDCA alludes to a "return" of these areas once they are no longer
needed by U.S. forces, indicating that there would be some transfer of use. Second, Article IV(4)
ofEDCA talks about American forces' unimpeded access to the Agreed Locations for all matters
relating to the prepositioning and storage of U.S. military equipment, supplies, and materiel. Third,
Article VII of EDCA authorizes U.S. forces to use public utilities and to operate their own
telecommunications system.

a. Preliminary point on badges of exclusivity

As a preliminary observation, petitioners have cherry-picked provisions of EDCA by presenting so-


called "badges of exclusivity," despite the presence of contrary provisions within the text of the
agreement itself.

First, they clarify the word "return" in Article V(2) of EDCA. However, the use of the word "return" is
within the context of a lengthy provision. The provision as a whole reads as follows:
The United States shall return to the Philippines any Agreed Locations, or any portion thereof,
including non-relocatable structures and assemblies constructed, modified, or improved by the
United States, once no longer required by United States forces for activities under this Agreement.
The Parties or the Designated Authorities shall consult regarding the terms of return of any Agreed
Locations, including possible compensation for improvements or construction.

The context of use is "required by United States forces for activities under this Agreement."
Therefore, the return of an Agreed Location would be within the parameters of an activity that the
Mutual Defense Board (MDB) and the Security Engagement Board (SEB) would authorize. Thus,
possession by the U.S. prior to its return of the Agreed Location would be based on the authority
given to it by a joint body co-chaired by the "AFP Chief of Staff and Commander, U.S. PACOM with
representatives from the Philippines' Department of National Defense and Department of Foreign
Affairs sitting as members."  The terms shall be negotiated by both the Philippines and the U.S., or
313

through their Designated Authorities. This provision, seen as a whole, contradicts petitioners'
interpretation of the return as a "badge of exclusivity." In fact, it shows the cooperation and
partnership aspect of EDCA in full bloom.

Second, the term "unimpeded access" must likewise be viewed from a contextual perspective.
Article IV(4) states that U.S. forces and U.S. contractors shall have "unimpeded access to Agreed
Locations for all matters relating to the prepositioning and storage of defense equipment, supplies,
and materiel, including delivery, management, inspection, use, maintenance, and removal of such
equipment, supplies and materiel."

At the beginning of Article IV, EDCA states that the Philippines gives the U.S. the authority to bring
in these equipment, supplies, and materiel through the MDB and SEB security mechanism. These
items are owned by the U.S.,  are exclusively for the use of the U.S.  and, after going through the
314 315

joint consent mechanisms of the MDB and the SEB, are within the control of the U.S.  More 316

importantly, before these items are considered prepositioned, they must have gone through the
process of prior authorization by the MDB and the SEB and given proper notification to the AFP. 317

Therefore, this "unimpeded access" to the Agreed Locations is a necessary adjunct to the
ownership, use, and control of the U.S. over its own equipment, supplies, and materiel and must
have first been allowed by the joint mechanisms in play between the two states since the time of the
MDT and the VFA. It is not the use of the Agreed Locations that is exclusive per se; it is mere
access to items in order to exercise the rights of ownership granted by virtue of the Philippine Civil
Code. 318

As for the view that EDCA authorizes U.S. forces to use public utilities and to operate their own
telecommunications system, it will be met and answered in part D, infra.

Petitioners also point out  that EDCA is strongly reminiscent of and in fact bears a one-to-one
319

correspondence with the provisions of the 1947 MBA. They assert that both agreements (a) allow
similar activities within the area; (b) provide for the same "species of ownership" over facilities; and
(c) grant operational control over the entire area. Finally, they argue  that EDCA is in fact an
320

implementation of the new defense policy of the U.S. According to them, this policy was not what
was originally intended either by the MDT or by the VFA.

On these points, the Court is not persuaded.

The similar activities cited by petitioners  simply show that under the MBA, the U.S. had the right to
321

construct, operate, maintain, utilize, occupy, garrison, and control the bases. The so-called parallel
provisions of EDCA allow only operational control over the Agreed Locations specifically for
construction activities. They do not allow the overarching power to operate, maintain, utilize, occupy,
garrison, and control a base with full discretion. EDCA in fact limits the rights of the U.S. in respect of
every activity, including construction, by giving the MDB and the SEB the power to determine the
details of all activities such as, but not limited to, operation, maintenance, utility, occupancy,
garrisoning, and control. 322

The "species of ownership" on the other hand, is distinguished by the nature of the property. For
immovable property constructed or developed by the U.S., EDCA expresses that ownership will
automatically be vested to the Philippines.  On the other hand, for movable properties brought into
323

the Philippines by the U.S., EDCA provides that ownership is retained by the latter. In contrast, the
MBA dictates that the U.S. retains ownership over immovable and movable properties.

To our mind, both EDCA and the MBA simply incorporate what is already the law of the land in the
Philippines. The Civil Code's provisions on ownership, as applied, grant the owner of a movable
property full rights over that property, even if located in another person's property.324

The parallelism, however, ends when the situation involves facilities that can be considered
immovable. Under the MBA, the U.S. retains ownership if it paid for the facility.  Under EDCA, an
325

immovable is owned by the Philippines, even if built completely on the back of U.S. funding.  This is
326

consistent with the constitutional prohibition on foreign land ownership. 327

Despite the apparent similarity, the ownership of property is but a part of a larger whole that must be
considered before the constitutional restriction is violated. Thus, petitioners' points on operational
control will be given more attention in the discussion below. The arguments on policy are, however,
outside the scope of judicial review and will not be discussed

Moreover, a direct comparison of the MBA and EDCA will result in several important distinctions that
would allay suspicion that EDCA is but a disguised version of the MBA.

b. There are substantial matters that the US. cannot do under EDCA, but which it was authorized to
do under the 1947 MBA

The Philippine experience with U.S. military bases under the 1947 MBA is simply not possible under
EDCA for a number of important reasons.

First, in the 1947 MBA, the U.S. retained all rights of jurisdiction in and over Philippine territory
occupied by American bases. In contrast, the U.S. under EDCA does not enjoy any such right over
any part of the Philippines in which its forces or equipment may be found. Below is a comparative
table between the old treaty and EDCA:

1947 MBA/ 1946 Treaty of General EDCA


Relations

1947 MBA, Art. I(1): EDCA, preamble:

The Government of the Republic of Affirming that the Parties share an


the Philippines (hereinafter referred to as the understanding for the United States not to
Philippines) grants to the Government of establish a permanent military presence or
the United States of America (hereinafter base in the territory of the Philippines;
referred to as the United States) the right to
retain the use of the bases in the
Philippines listed in Annex A attached hereto. x x x x

1947 MBA, Art. XVII(2): Recognizing that all United States access to


and use of facilities and areas will be at the
All buildings and structures which invitation of the Philippines and with full
are erected by the United States in the respect for the Philippine Constitution and
bases shall be the property of the United Philippine laws;
States and may be removed by it before the
expiration of this Agreement or the earlier x x x x
relinquishment of the base on which the
structures are situated. There shall be no EDCA, Art. II(4):
obligation on the part of the Philippines or of
the United States to rebuild or repair any "Agreed Locations" means facilities and
destruction or damage inflicted from any areas that are provided by the Government
cause whatsoever on any of the said buildings of the Philippines through the AFP and that
or structures owned or used by the United United States forces, United States
States in the bases. x x x x. contractors, and others as mutually agreed,
shall have the right to access and use
1946 Treaty of Gen. Relations, Art. I: pursuant to this Agreement. Such Agreed
Locations may be listed in an annex to be
The United States of America agrees to appended to this Agreement, and may be
withdraw and surrender, and does hereby further described in implementing
withdraw and surrender, all rights of arrangements.
possession, supervision, jurisdiction,
control or sovereignty existing and EDCA, Art. V:
exercised by the United States of America in
and over the territory and the people of the 1. The Philippines shall retain ownership of
Philippine Islands, except the use of such and title to Agreed Locations.
bases, necessary appurtenances to such
bases, and the rights incident thereto, as
xxxx
the United States of America, by agreement
with the Republic of the Philippines may deem
necessary to retain for the mutual protection of 4. All buildings, non-relocatable structures,
the Republic of the Philippines and of the and assemblies affixed to the land in the
United States of America. x x x. Agreed Locations, including ones altered or
improved by United States forces, remain
the property of the Philippines.Permanent
buildings constructed by United States forces
become the property of the Philippines, once
constructed, but shall be used by United
States forces until no longer required by
United States forces.

Second, in the bases agreement, the U.S. and the Philippines were visibly not on equal footing
when it came to deciding whether to expand or to increase the number of bases, as the Philippines
may be compelled to negotiate with the U.S. the moment the latter requested an expansion of the
existing bases or to acquire additional bases. In EDCA, U.S. access is purely at the invitation of the
Philippines.
1947 MBA/ 1946 Treaty of General EDCA
Relations

1947 MBA, Art.I(3): EDCA, preamble:

The Philippines agree to enter into Recognizing that all United States access to


negotiations with the United States at the and use of facilities and areas will be at the
latter's request, to permit the United invitation of the Philippines and with full
States to expand such bases, to exchange respect for the Philippine Constitution and
such bases for other bases, to acquire Philippine laws;
additional bases, or relinquish rights to
bases, as any of such exigencies may be xxxx
required by military necessity.
EDCA. Art. II(4):
1946 Treaty of Gen. Relations, Art. I:
"Agreed Locations" means facilities and
The United States of America agrees to areas that are provided by the Government
withdraw and surrender, and does hereby of the Philippines through the AFP and that
withdraw and surrender, all rights of United States forces, United States
possession, supervision, jurisdiction, contractors, and others as mutually agreed,
control or sovereignty existing and shall have the right to access and use
exercised by the United States of America in pursuant to this Agreement. Such Agreed
and over the territory and the people of the Locations may be listed in an annex to be
Philippine Islands, except the use of such appended to this Agreement, and may be
bases, necessary appurtenances to such further described in implementing
bases, and the rights incident thereto, as the arrangements.
United States of America, by agreement
with the Republic of the Philippines may
deem necessary to retain for the mutual
protection of the Republic of the Philippines
and of the United States of America. x x x.

Third, in EDCA, the Philippines is- guaranteed access over the entire area of the Agreed Locations.
On the other hand, given that the U.S. had complete control over its military bases under the 1947
MBA, the treaty did not provide for any express recognition of the right of access of Philippine
authorities. Without that provision and in light of the retention of U.S. sovereignty over the old military
bases, the U.S. could effectively prevent Philippine authorities from entering those bases.

1947 MBA EDCA

No equivalent provision. EDCA, Art. III(5):

The Philippine Designated Authority and its


authorized representative shall have access
to the entire area of the Agreed Locations.
Such access shall be provided promptly
consistent with operational safety and security
requirements in accordance with agreed
procedures developed by the Parties.
Fourth, in the bases agreement, the U.S. retained the right, power, and authority over the
establishment, use, operation, defense, and control of military bases, including the limits of territorial
waters and air space adjacent to or in the vicinity of those bases. The only standard used in
determining the extent of its control was military necessity. On the other hand, there is no such grant
of power or authority under EDCA. It merely allows the U.S. to exercise operational control over the
construction of Philippine-owned structures and facilities:

1947 MBA EDCA

1947 MBA, Art.I(2): EDCA, Art. III(4):

The Philippines agrees to permit the United The Philippines hereby grants to the United
States, upon notice to the Philippines, to use States, through bilateral security
such of those bases listed in Annex B as the mechanisms, such as the MDB and
United States determines to be required by SEB, operational control of Agreed
military necessity. Locations for construction
activities and authority to undertake such
1947 MBA, Art. III(1): activities on, and make alterations and
improvements to, Agreed Locations. United
It is mutually agreed that the United States forces shall consult on issues
Statesshall have the rights, power and regarding such construction, alterations,
authority within the bases which and improvements based on the Parties'
are necessary for the establishment, use, shared intent that the technical requirements
operation and defense thereof or and construction standards of any such
appropriate for the control thereof and all projects undertaken by or on behalf of United
the rights, power and authority within the States forces should be consistent with the
limits of territorial waters and air space requirements and standards of both Parties.
adjacent to, or in the vicinity of, the bases
which are necessary to provide access to
them, or appropriate for their control.

Fifth, the U.S. under the bases agreement was given the authority to use Philippine territory for
additional staging areas, bombing and gunnery ranges. No such right is given under EDCA, as seen
below:

1947 MBA EDCA

1947 MBA, Art. VI: EDCA, Art. III(1):

The United States shall, subject to previous With consideration of the views of


agreement with the Philippines, have the right the Parties,
to use land and coastal sea areas of the Philippines hereby authorizes and
appropriate size and location for periodic agrees that United States forces, United
maneuvers, for additional staging areas, States contractors, and vehicles, vessels, and
bombing and gunnery ranges, and for such aircraft operated by or for United States forces
intermediate airfields as may be required for may conduct the following activities with
safe and efficient air operations. Operations in respect to Agreed Locations: training; transit;
such areas shall be carried on with due regard support and related activities; refueling of
and safeguards for the public safety. aircraft; bunkering of vessels; temporary
maintenance of vehicles, vessels, and aircraft;
temporary accommodation of personnel;
1947 MBA, Art.I(2): communications; prepositioning of equipment,
supplies, and materiel; deploying forces and
The Philippines agrees to permit the United materiel; and such other activities as the
States, upon notice to the Philippines, to use Parties may agree.
such of those bases listed in Annex B as the
United States determines to be required by
military necessity.

Sixth, under the MBA, the U.S. was given the right, power, and authority to control and prohibit the
movement and operation of all types of vehicles within the vicinity of the bases. The U.S. does not
have any right, power, or authority to do so under EDCA.

1947 MBA EDCA

1947 MBA, Art. 111(2)(c) No equivalent provision.

Such rights, power and authority shall


include, inter alia, the right, power and
authority: x x x x to control (including the
right to prohibit) in so far as may be required
for the efficient operation and safety of the
bases, and within the limits of military
necessity, anchorages, moorings, landings,
takeoffs, movements and operation of
ships and water-borne craft, aircraft and
other vehicles on water, in the air or on
land comprising

Seventh, under EDCA, the U.S. is merely given temporary access to land and facilities (including
roads, ports, and airfields). On the other hand, the old treaty gave the U.S. the right to improve and
deepen the harbors, channels, entrances, and anchorages; and to construct or maintain necessary
roads and bridges that would afford it access to its military bases.

1947 MBA EDCA

1947 MBA, Art. III(2)(b): EDCA, Art. III(2):

Such rights, power and authority shall When requested, the Designated Authority of
include, inter alia, the right, power and the Philippines shall assist in facilitating
authority: x x x x to improve and deepen transit or temporary access by United
the harbors, channels, entrances and States forces to public land and facilities
anchorages, and to construct or maintain (including roads, ports, and airfields), including
necessary roadsand bridges affording access those owned or controlled by local
to the bases. governments, and to other land and facilities
(including roads, ports, and airfields).

Eighth, in the 1947 MBA, the U.S. was granted the automatic right to use any and all public utilities,
services and facilities, airfields, ports, harbors, roads, highways, railroads, bridges, viaducts, canals,
lakes, rivers, and streams in the Philippines in the same manner that Philippine military forces
enjoyed that right. No such arrangement appears in EDCA. In fact, it merely extends to U.S. forces
temporary access to public land and facilities when requested:

1947 MBA EDCA

1947 MBA, Art. VII: EDCA, Art. III(2):

It is mutually agreed that the United States When requested, the Designated Authority of
may employ and use for United States the Philippines shall assist in facilitating
military forces any and all public utilities, transit or temporary access by United
other services and facilities, airfields, ports, States forces to public land and facilities
harbors, roads, highways, railroads, bridges, (including roads, ports, and airfields), including
viaducts, canals, lakes, rivers and streams in those owned or controlled by local
the Philippines under conditions no less governments, and to other land and facilities
favorable than those that may (including roads, ports, and airfields).
be applicablefrom time to time to the military
forces of the Philippines.

Ninth, under EDCA, the U.S. no longer has the right, power, and authority to construct, install,
maintain, and employ any type of facility, weapon, substance, device, vessel or vehicle, or system
unlike in the old treaty. EDCA merely grants the U.S., through bilateral security mechanisms, the
authority to undertake construction, alteration, or improvements on the Philippine-owned Agreed
Locations.

1947 MBA EDCA

1947 MBA, Art. III(2)(e): EDCA, Art. III(4):

Such rights, power and authority shall The Philippines hereby grants to the United
include, inter alia, the right, power and States, through bilateral security
authority: x x x x to construct, install, mechanisms, such as the MDB and SEB,
maintain, and employ on any base any operational control of Agreed Locations for
type of facilities, weapons, substance, construction activities and authority to
device, vessel or vehicle on or under the undertake such activities on, and make
ground, in the air or on or under the water that alterations and improvements to, Agreed
may be requisite or appropriate, including Locations. United States forces shall consult
meteorological systems, aerial and water on issues regarding such construction,
navigation lights, radio and radar apparatus alterations, and improvements based on the
and electronic devices, of any desired power, Parties' shared intent that the technical
type of emission and frequency. requirements and construction standards of
any such projects undertaken by or on behalf
of United States forces should be consistent
with the requirements and standards of both
Parties.

Tenth, EDCA does not allow the U.S. to acquire, by condemnation or expropriation proceedings,
real property belonging to any private person. The old military bases agreement gave this right to the
U.S. as seen below:

1947 MBA EDCA


1947 MBA, Art. XXII(l): No equivalent provision.

Whenever it is necessary to acquire by

condemnation or expropriation
proceedings real property belonging to any
private persons, associations or corporations
located in bases named in Annex A and
Annex B in order to carry out the purposes of
this Agreement, the Philippines will institute
and prosecute such condemnation or
expropriation proceedings in accordance with
the laws of the Philippines. The United States
agrees to reimburse the Philippines for all the
reasonable expenses, damages and costs
therebv incurred, including the value of the
property as determined by the Court. In
addition, subject to the mutual agreement of
the two Governments, the United States will
reimburse the Philippines for the reasonable
costs of transportation and removal of any
occupants displaced or ejected by reason of
the condemnation or expropriation.

Eleventh, EDCA does not allow the U.S. to unilaterally bring into the country non-Philippine
nationals who are under its employ, together with their families, in connection with the construction,
maintenance, or operation of the bases. EDCA strictly adheres to the limits under the VFA.

1947 MBA EDCA

1947 MBA, Art. XI(l): EDCA, Art. II:

It is mutually agreed that the United States 1. "United States personnel" means United


shall have the right to bring into the States military and civilian
Philippines members of the United States personneltemporarily in the territory of the
military forces and the United States Philippines in connection with activities
nationals employed by or under a contract approved by the Philippines, as those terms
with the United States together with their are defined in the VFA.
families, and technical personnel of other
nationalities (not being persons excluded by x xx x
the laws of the Philippines) in connection with
the construction, maintenance, or operation of 3. "United States contractors" means
the bases. The United States shall make companies and firms, and their employees,
suitable arrangements so that such persons under contract or subcontract to or on behalf
may be readily identified and their status of the United States Department of Defense.
established when necessary by the Philippine United States contractors are not includedas
authorities. Such persons, other than part of the definition of United States
members of the United States armed forces in personnel in this Agreement, including
uniform, shall present their travel documents within the context of the VFA.
to the appropriate Philippine authorities for
visas, it being understood that no objection
will be made to their travel to the  
Philippines as non-immigrants.

Twelfth, EDCA does not allow the U.S. to exercise jurisdiction over any offense committed by any
person within the Agreed Locations, unlike in the former military bases:

1947 MBA EDCA

1947 MBA, Art. XIII(l)(a): No equivalent provision.

The Philippines consents that the United

States shall have the right to exercise


jurisdiction over the following offenses:
(a) Any offense committed by any
person within any base except where the
offender and offended parties are both
Philippine citizens (not members of the armed
forces of the United States on active duty) or
the offense is against the security of the
Philippines.

Thirteenth, EDCA does not allow the U.S. to operate military post exchange (PX) facilities, which is
free of customs duties and taxes, unlike what the expired MBA expressly allowed. Parenthetically,
the PX store has become the cultural icon of U.S. military presence in the country.

1947 MBA EDCA

1947 MBA, Art. XVIII(l): No equivalent provision.

It is mutually agreed that the United States

shall have the right to establish on bases,


free of all licenses; fees; sales, excise or
other taxes, or imposts; Government
agencies, including concessions, such
as sales commissaries and post
exchanges; messes and social clubs, for the
exclusive use of the United States military
forces and authorized civilian personnel
and their families. The merchandise or
services sold or dispensed by such agencies
shall be free of all taxes, duties and
inspection by the Philippine
authorities. Administrative measures shall be
taken by the appropriate authorities of the
United States to prevent the resale of goods
which are sold under the provisions of this
Article to persons not entitled to buy goods at
such agencies and, generally, to prevent
abuse of the privileges granted under this
Article. There shall be cooperation between
such authorities and the Philippines to this
end.

In sum, EDCA is a far cry from a basing agreement as was understood by the people at the time that
the 1987 Constitution was adopted.

Nevertheless, a comprehensive review of what the Constitution means by "foreign military bases"
and "facilities" is required before EDCA can be deemed to have passed judicial scrutiny.

c. The meaning of military facilities and bases

An appreciation of what a military base is, as understood by the Filipino people in 1987, would be
vital in determining whether EDCA breached the constitutional restriction.

Prior to the drafting of the 1987 Constitution, the last definition of "military base" was provided under
Presidential Decree No. (PD) 1227.  Unlawful entry into a military base is punishable under the
328

decree as supported by Article 281 of the Revised Penal Code, which itself prohibits the act of
trespass.

Section 2 of the law defines the term in this manner: "'[M]ilitary base' as used in this decree means
any military, air, naval, or coast guard reservation, base, fort, camp, arsenal, yard, station, or
installation in the Philippines."

Commissioner Tadeo, in presenting his objections to U.S. presence in the Philippines before the
1986 Constitutional Commission, listed the areas that he considered as military bases:

1,000 hectares Camp O'Donnel

20,000 hectares Crow Valley Weapon's Range

55,000 hectares Clark Air Base

150 hectares Wallace Air Station

400 hectares John Hay Air Station

15,000 hectares Subic Naval Base

1,000 hectares San Miguel Naval Communication

750 hectares Radio Transmitter in Capas, Tarlac

900 hectares Radio Bigot Annex at Bamban, Tarlac 329


The Bases Conversion and Development Act of 1992 described its coverage in its Declaration of
Policies:

Sec. 2. Declaration of Policies. - It is hereby declared the policy of the Government to accelerate the
sound and balanced conversion into alternative productive uses of the Clark and Subic military
reservations and their extensions (John Hay Station, Wallace Air Station, O'Donnell Transmitter
Station, San Miguel Naval Communications Station and Capas Relay Station), to raise funds by the
sale of portions of Metro Manila military camps, and to apply said funds as provided herein for the
development and conversion to productive civilian use of the lands covered under the 194 7 Military
Bases Agreement between the Philippines and the United States of America, as amended. 330

The result of the debates and subsequent voting is Section 25, Article XVIII of the Constitution,
which specifically restricts, among others, foreign military facilities or bases. At the time of its crafting
of the Constitution, the 1986 Constitutional Commission had a clear idea of what exactly it was
restricting. While the term "facilities and bases" was left undefined, its point of reference was clearly
those areas covered by the 1947 MBA as amended.

Notably, nearly 30 years have passed since then, and the ever-evolving world of military technology
and geopolitics has surpassed the understanding of the Philippine people in 1986. The last direct
military action of the U.S. in the region was the use of Subic base as the staging ground for Desert
Shield and Desert Storm during the Gulf War. In 1991, the Philippine Senate rejected the successor
331

treaty of the 1947 MBA that would have allowed the continuation of U.S. bases in the Philippines.

Henceforth, any proposed entry of U.S. forces into the Philippines had to evolve likewise, taking into
consideration the subsisting agreements between both parties, the rejection of the 1991 proposal,
and a concrete understanding of what was constitutionally restricted. This trend birthed the VFA
which, as discussed, has already been upheld by this Court.

The latest agreement is EDCA, which proposes a novel concept termed "Agreed Locations."

By definition, Agreed Locations are

facilities and areas that are provided by the Government of the Philippines through the AFP and that
United States forces, United States contractors, and others as mutually agreed, shall have the right
to access and use pursuant to this Agreement. Such Agreed Locations may be listed in an annex to
be appended to this Agreement, and may be further described in implementing arrangements. 332

Preliminarily, respondent already claims that the proviso that the Philippines shall retain ownership
of and title to the Agreed Locations means that EDCA is "consistent with Article II of the VFA which
recognizes Philippine sovereignty and jurisdiction over locations within Philippine territory. 333

By this interpretation, respondent acknowledges that the contention of petitioners springs from an
understanding that the Agreed Locations merely circumvent the constitutional restrictions. Framed
differently, the bone of contention is whether the Agreed Locations are, from a legal perspective,
foreign military facilities or bases. This legal framework triggers Section 25, Article XVIII, and makes
Senate concurrence a sine qua non.

Article III of EDCA provides for Agreed Locations, in which the U.S. is authorized by the Philippines
to "conduct the following activities: "training; transit; support and related activities; refueling of
aircraft; bunkering of vessels; temporary maintenance of vehicles, vessels and aircraft; temporary
accommodation of personnel; communications; prepositioning of equipment, supplies and materiel;
deploying forces and materiel; and such other activities as the Parties may agree."

This creation of EDCA must then be tested against a proper interpretation of the Section 25
restriction.

d. Reasons for the constitutional requirements and legal standards for constitutionally compatible
military bases and facilities

Section 25 does not define what is meant by a "foreign military facility or base." While it specifically
alludes to U.S. military facilities and bases that existed during the framing of the Constitution, the
provision was clearly meant to apply to those bases existing at the time and to any future facility or
base. The basis for the restriction must first be deduced from the spirit of the law, in order to set a
standard for the application of its text, given the particular historical events preceding the agreement.

Once more, we must look to the 1986 Constitutional Commissioners to glean, from their collective
wisdom, the intent of Section 25. Their speeches are rich with history and wisdom and present a
clear picture of what they considered in the crafting the provision.

SPEECH OF COMMISSIONER REGALADO 334

xxxx

We have been regaled here by those who favor the adoption of the anti-bases provisions with what
purports to be an objective presentation of the historical background of the military bases in the
Philippines. Care appears, however, to have been taken to underscore the inequity in their
inception as well as their implementation, as to seriously reflect on the supposed objectivity of
the report. Pronouncements of military and civilian officials shortly after World War II are quoted in
support of the proposition on neutrality; regrettably, the implication is that the same remains valid
today, as if the world and international activity stood still for the last 40 years.

We have been given inspired lectures on the effect of the presence of the military bases on
our sovereignty - whether in its legal or political sense is not clear - and the theory that any
country with foreign bases in its territory cannot claim to be fully sovereign or completely
independent. I was not aware that the concepts of sovereignty and independence have now
assumed the totality principle, such that a willing assumption of some delimitations in the exercise of
some aspects thereof would put that State in a lower bracket of nationhood.

xxxx

We have been receiving a continuous influx of materials on the pros and cons on the advisability of
having military bases within our shores. Most of us who, only about three months ago, were just
mulling the prospects of these varying contentions are now expected, like armchair generals, to
decide not only on the geopolitical aspects and contingent implications of the military bases but also
on their political, social, economic and cultural impact on our national life. We are asked to answer a
plethora of questions, such as: 1) whether the bases are magnets of nuclear attack or are deterrents
to such attack; 2) whether an alliance or mutual defense treaty is a derogation of our national
sovereignty; 3) whether criticism of us by Russia, Vietnam and North Korea is outweighed by the
support for us of the ASEAN countries, the United States, South Korea, Taiwan, Australia and New
Zealand; and 4) whether the social, moral and legal problems spawned by the military bases and
their operations can be compensated by the economic benefits outlined in papers which have been
furnished recently to all of us. 335

xxxx

Of course, one side of persuasion has submitted categorical, unequivocal and forceful assertions of
their positions. They are entitled to the luxury of the absolutes. We are urged now to adopt the
proposed declaration as a "golden," "unique" and "last" opportunity for Filipinos to assert
their sovereign rights. Unfortunately, I have never been enchanted by superlatives, much less for
the applause of the moment or the ovation of the hour. Nor do I look forward to any glorious summer
after a winter of political discontent. Hence, if I may join Commissioner Laurel, I also invoke a caveat
not only against the tyranny of labels but also the tyranny of slogans.
336

xxxx

SPEECH OF COMMISSIONER SUAREZ 337

MR. SUAREZ: Thank you, Madam President.

I am quite satisfied that the crucial issues involved in the resolution of the problem of the removal of
foreign bases from the Philippines have been adequately treated by previous speakers. Let me,
therefore, just recapitulate the arguments adduced in favor of a foreign bases-free Philippines:

1. That every nation should be free to shape its own destiny without outside
interference;

2. That no lasting peace and no true sovereignty would ever be achieved so long as there
are foreign military forces in our country;

3. That the presence of foreign military bases deprives us of the very substance of


national sovereigntyand this is a constant source of national embarrassment and an insult
to our national dignity and selfrespect as a nation;

4. That these foreign military bases unnecessarily expose our country to devastating


nuclear attacks;

5. That these foreign military bases create social problems and are designed to perpetuate
the strangle-hold of United States interests in our national economy and development;

6. That the extraterritorial rights enjoyed by these foreign bases operate to deprive our


country of jurisdiction over civil and criminal offenses committed within our own national
territory and against Filipinos;

7. That the bases agreements are colonial impositions and dictations upon our helpless
country; and

8. That on the legal viewpoint and in the ultimate analysis, all the bases agreements are null
and void ab initio, especially because they did not count the sovereign consent and will of
the Filipino people.
338

xxxx
In the real sense, Madam President, if we in the Commission could accommodate the provisions I
have cited, what is our objection to include in our Constitution a matter as priceless as the nationalist
values we cherish? A matter of the gravest concern for the safety and survival of this
nation indeed deserves a place in our Constitution.

xxxx

x x x Why should we bargain away our dignity and our self-respect as a nation and the future of
generations to come with thirty pieces of silver? 339

SPEECH OF COMMISSIONER BENNAGEN 340

xxxx

The underlying principle of military bases and nuclear weapons wherever they are found and
whoever owns them is that those are for killing people or for terrorizing humanity. This objective
by itself at any point in history is morally repugnant. This alone is reason enough for us to
constitutionalize the ban on foreign military bases and on nuclear weapons. 341

SPEECH OF COMMISSIONER BACANI 342

xxxx

x x x Hence, the remedy to prostitution does not seem to be primarily to remove the


bases because even if the bases are removed, the girls mired in poverty will look for their clientele
elsewhere. The remedy to the problem of prostitution lies primarily elsewhere - in an alert and
concerned citizenry, a healthy economy and a sound education in values. 343

SPEECH OF COMMISSIONER JAMIR 344

xxxx

One of the reasons advanced against the maintenance of foreign military bases here is that
they impair portions of our sovereignty. While I agree that our country's sovereignty should not
be impaired, I also hold the view that there are times when it is necessary to do so according to the
imperatives of national interest. There are precedents to this effect. Thus, during World War II,
England leased its bases in the West Indies and in Bermuda for 99 years to the United States for its
use as naval and air bases. It was done in consideration of 50 overaged destroyers which the United
States gave to England for its use in the Battle of the Atlantic.

A few years ago, England gave the Island of Diego Garcia to the United States for the latter's use as
a naval base in the Indian Ocean. About the same time, the United States obtained bases in Spain,
Egypt and Israel. In doing so, these countries, in effect, contributed to the launching of a preventive
defense posture against possible trouble in the Middle East and in the Indian Ocean for their own
protection.345

SPEECH OF COMMISSIONER TINGSON 346

xxxx
In the case of the Philippines and the other Southeast Asian nations, the presence of American
troops in the country is a projection of America's security interest. Enrile said that nonetheless, they
also serve, although in an incidental and secondary way, the security interest of the Republic of the
Philippines and the region. Yes, of course, Mr. Enrile also echoes the sentiments of most of us in
this Commission, namely: It is ideal for us as an independent and sovereign nation to
ultimately abrogate the RP-US military treaty and, at the right time, build our own air and
naval might. 347

xxxx

Allow me to say in summation that I am for the retention of American military bases in the
Philippines provided that such an extension from one period to another shall be concluded
upon concurrence of the parties, and such extension shall be based on justice, the historical
amity of the people of the Philippines and the United States and their common defense
interest.348

SPEECH OF COMMISSIONER ALONTO 349

xxxx

Madam President, sometime ago after this Commission started with this task of framing a
constitution, I read a statement of President Aquino to the effect that she is for the removal of the
U.S. military bases in this country but that the removal of the U.S. military bases should not be done
just to give way to other foreign bases. Today, there are two world superpowers, both vying to
control any and all countries which have importance to their strategy for world domination. The
Philippines is one such country.

Madam President, I submit that I am one of those ready to completely remove any vestiges of
the days of enslavement, but not prepared to erase them if to do so would merely leave a vacuum
to be occupied by a far worse type. 350

SPEECH OF COMMISSIONER GASCON 351

xxxx

Let us consider the situation of peace in our world today. Consider our brethren in the Middle East,
in Indo-China, Central America, in South Africa - there has been escalation of war in some of these
areas because of foreign intervention which views these conflicts through the narrow prism of the
East-West conflict. The United States bases have been used as springboards for intervention
in some of these conflicts. We should not allow ourselves to be party to the warlike mentality
of these foreign interventionists. We must always be on the side of peace – this means that we
should not always rely on military solution.
352

xxxx

x x x The United States bases, therefore, are springboards for intervention in our own internal
affairs and in the affairs of other nations in this region.

xxxx
Thus, I firmly believe that a self-respecting nation should safeguard its fundamental freedoms which
should logically be declared in black and white in our fundamental law of the land - the
Constitution. Let us express our desire for national sovereignty so we may be able to achieve
national self-determination. Let us express our desire for neutrality so that we may be able to
follow active nonaligned independent foreign policies. Let us express our desire for peace and a
nuclear-free zone so we may be able to pursue a healthy and tranquil existence, to have peace that
is autonomous and not imposed.  353

xxxx

SPEECH OF COMMISSIONER TADEO 354

Para sa magbubukid, ano ha ang kahulugan ng U.S. military bases? Para sa magbubukid, ang


kahulugan nito ay pagkaalipin. Para sa magbubukid, ang pananatili ng U.S. military
bases ay tinik sa dibdib ng sambayanang Pilipinong patuloy na nakabaon. Para sa
sambayanang magbubukid, ang ibig sabihin ng U.S. military bases ay batong pabigat na patuloy
na pinapasan ng sambayanang Pilipino. Para sa sambayanang magbubukid, ang pananatili
ng U.S. military bases ay isang nagdudumilat na katotohanan ng patuloy na paggahasa ng
imperyalistang Estados Unidos sa ating Inang Bayan - economically, politically and
culturally. Para sa sambayanang magbubukid ang U.S. military bases ay kasingkahulugan
ng nuclear weapon - ang kahulugan ay magneto ng isang nuclear war. Para sa sambayanang
magbubukid, ang kahulugan ng U.S. military bases ay isang salot. 355

SPEECH OF COMMISSIONER QUESADA 356

xxxx

The drift in the voting on issues related to freeing ourselves from the instruments of domination
and subservience has clearly been defined these past weeks.

xxxx

So for the record, Mr. Presiding Officer, I would like to declare my support for the committee's
position to enshrine in the Constitution a fundamental principle forbidding foreign military bases,
troops or facilities in any part of the Philippine territory as a clear and concrete manifestation of
our inherent right to national self-determination, independence and sovereignty.

Mr. Presiding Officer, I would like to relate now these attributes of genuine nationhood to the social
cost of allowing foreign countries to maintain military bases in our country. Previous speakers have
dwelt on this subject, either to highlight its importance in relation to the other issues or to gloss over
its significance and !llake this a part of future negotiations.
357

xxxx

Mr. Presiding Officer, I feel that banning foreign military bases is one of the solutions and is the
response of the Filipino people against this condition and other conditions that have already been
clearly and emphatically discussed in past deliberations. The deletion, therefore, of Section 3 in the
Constitution we are drafting will have the following implications:

First, the failure of the Constitutional Commission to decisively respond to the continuing violation
of our territorial integrity via the military bases agreement which permits the retention of U.S.
facilities within the Philippine soil over which our authorities have no exclusive jurisdiction
contrary to the accepted definition of the exercise of sovereignty.

Second, consent by this forum, this Constitutional Commission, to an exception in the application
of a provision in the Bill of Rights that we have just drafted regarding equal application of the laws
of the land to all inhabitants, permanent or otherwise, within its territorial boundaries.

Third, the continued exercise by the United States of extraterritoriality despite the


condemnations of such practice by the world community of nations in the light of overwhelming
international approval of eradicating all vestiges of colonialism.
358

xxxx

Sixth, the deification of a new concept called pragmatic sovereignty, in the hope that such can be
wielded to force the United States government to concede to better terms and conditions concerning
the military bases agreement, including the transfer of complete control to the Philippine
government of the U.S. facilities, while in the meantime we have to suffer all existing indignities
and disrespect towards our rights as a sovereign nation.

xxxx

Eighth, the utter failure of this forum to view the issue of foreign military bases as essentially
a question of sovereignty which does not require in-depth studies or analyses and which this
forum has, as a constituent assembly drafting a constitution, the expertise and capacity to decide on
except that it lacks the political will that brought it to existence and now engages in an elaborate
scheme of buck-passing.

xxxx

Without any doubt we can establish a new social order in our country, if we reclaim, restore, uphold
and defend our national sovereignty. National sovereignty is what the military bases issue is all
about. It is only the sovereign people exercising their national sovereignty who can design an
independent course and take full control of their national destiny. 359

SPEECH OF COMMISSIONER P ADILLA 360

xxxx

Mr. Presiding Officer, in advocating the majority committee report, specifically Sections 3 and 4 on
neutrality, nuclear and bases-free country, some views stress sovereignty of the Republic and
even invoke survival of the Filipino nation and people. 361

REBUTTAL OF COMMISSIONER NOLLEDO 362

xxxx

The anachronistic and ephemeral arguments against the provisions of the committee report to
dismantle the American bases after 1991 only show the urgent need to free our country from the
entangling alliance with any power bloc. 363

xxxx
xx x Mr. Presiding Officer, it is not necessary for us to possess expertise to know that the so-called
RP-US Bases Agreement will expire in 1991, that it infringes on our sovereignty and jurisdiction
as well as national dignity and honor, that it goes against the UN policy of disarmament and that it
constitutes unjust intervention in our internal affairs.  (Emphases Supplied)
364

The Constitutional Commission eventually agreed to allow foreign military bases, troops, or facilities,
subject to the provisions of Section 25. It is thus important to read its discussions carefully. From
these discussions, we can deduce three legal standards that were articulated by the Constitutional
Commission Members. These are characteristics of any agreement that the country, and by
extension this Court, must ensure are observed. We can thereby determine whether a military base
or facility in the Philippines, which houses or is accessed by foreign military troops, is foreign or
remains a Philippine military base or facility. The legal standards we find applicable are:
independence from foreign control, sovereignty and applicable law, and national security and
territorial integrity.

i. First standard: independence from foreign control

Very clearly, much of the opposition to the U.S. bases at the time of the Constitution's drafting was
aimed at asserting Philippine independence from the U.S., as well as control over our country's
territory and military.

Under the Civil Code, there are several aspects of control exercised over property.

Property is classified as private or public.  It is public if "intended for public use, such as roads,
365

canals, rivers, torrents, ports and bridges constructed by the State, banks, shores, roadsteads, and
others of similar character[,]" or "[t]hose which belong to the State, without being for public use, and
are intended for some public service or for the development of the national wealth. " 366

Quite clearly, the Agreed Locations are contained within a property for public use, be it within a
government military camp or property that belongs to the Philippines. 1avvphi1

Once ownership is established, then the rights of ownership flow freely. Article 428 of the Civil Code
provides that "[t]he owner has the right to enjoy and dispose of a thing, without other limitations than
those established by law." Moreover, the owner "has also a right of action against the holder and
possessor of the thing in order to recover it."

Philippine civil law therefore accords very strong rights to the owner of property, even against those
who hold the property. Possession, after all, merely raises a disputable presumption of ownership,
which can be contested through normal judicial processes. 367

In this case, EDCA explicitly provides that ownership of the Agreed Locations remains with the
Philippine govemment.  What U.S. personnel have a right to, pending mutual agreement, is access
368

to and use of these locations.369

The right of the owner of the property to allow access and use is consistent with the Civil Code,
since the owner may dispose of the property in whatever way deemed fit, subject to the limits of the
law. So long as the right of ownership itself is not transferred, then whatever rights are transmitted
by agreement does not completely divest the owner of the rights over the property, but may only limit
them in accordance with law.
Hence, even control over the property is something that an owner may transmit freely. This act does
not translate into the full transfer of ownership, but only of certain rights. In Roman Catholic
Apostolic Administrator of Davao, Inc. v. Land Registration Commission, we stated that the
constitutional proscription on property ownership is not violated despite the foreign national's control
over the property. 370

EDCA, in respect of its provisions on Agreed Locations, is essentially a contract of use and access.
Under its pertinent provisions, it is the Designated Authority of the Philippines that shall, when
requested, assist in facilitating transit or access to public land and facilities.  The activities carried
371

out within these locations are subject to agreement as authorized by the Philippine
govemment.  Granting the U.S. operational control over these locations is likewise subject to EDCA'
372

s security mechanisms, which are bilateral procedures involving Philippine consent and
cooperation.  Finally, the Philippine Designated Authority or a duly designated representative is
373

given access to the Agreed Locations. 374

To our mind, these provisions do not raise the spectre of U.S. control, which was so feared by the
Constitutional Commission. In fact, they seem to have been the product of deliberate negotiation
from the point of view of the Philippine government, which balanced constitutional restrictions on
foreign military bases and facilities against the security needs of the country. In the 1947 MBA, the
U.S. forces had "the right, power and authority x x x to construct (including dredging and filling),
operate, maintain, utilize, occupy, garrison and control the bases."  No similarly explicit provision is
375

present in EDCA.

Nevertheless, the threshold for allowing the presence of foreign military facilities and bases has
been raised by the present Constitution. Section 25 is explicit that foreign military bases, troops, or
facilities shall not be allowed in the Philippines, except under a treaty duly concurred in by the
Senate. Merely stating that the Philippines would retain ownership would do violence to the
constitutional requirement if the Agreed Locations were simply to become a less obvious
manifestation of the U.S. bases that were rejected in 1991.

When debates took place over the military provisions of the Constitution, the committee rejected a
specific provision proposed by Commissioner Sarmiento. The discussion illuminates and provides
context to the 1986 Constitutional Commission's vision of control and independence from the U.S.,
to wit:

MR. SARMIENTO: Madam President, my proposed amendment reads as follows: "THE STATE
SHALL ESTABLISH AND MAINTAIN AN INDEPENDENT AND SELF-RELIANT ARMED FORCES
OF THE PHILIPPINES." Allow me to briefly explain, Madam President. The Armed Forces of the
Philippines is a vital component of Philippine society depending upon its training, orientation and
support. It will either be the people's protector or a staunch supporter of a usurper or tyrant, local and
foreign interest. The Armed Forces of the Philippines' past and recent experience shows it has
never been independent and self-reliant. Facts, data and statistics will show that it has been
substantially dependent upon a foreign power. In March 1968, Congressman Barbero, himself a
member of the Armed Forces of the Philippines, revealed top secret documents showing what he
described as U.S. dictation over the affairs of the Armed Forces of the Philippines. He showed that
under existing arrangements, the United States unilaterally determines not only the types and
quantity of arms and equipments that our armed forces would have, but also the time when
these items are to be made available to us. It is clear, as he pointed out, that the composition,
capability and schedule of development of the Armed Forces of the Philippines is under the
effective control of the U.S. government.  (Emphases supplied)
376
Commissioner Sarmiento proposed a motherhood statement in the 1987 Constitution that would
assert "independent" and "self-reliant" armed forces. This proposal was rejected by the
committee, however. As Commissioner De Castro asserted, the involvement of the Philippine
military with the U.S. did not, by itself, rob the Philippines of its real independence. He made
reference to the context of the times: that the limited resources of the Philippines and the current
insurgency at that time necessitated a strong military relationship with the U.S. He said that the U.S.
would not in any way control the Philippine military despite this relationship and the fact that the
former would furnish military hardware or extend military assistance and training to our military.
Rather, he claimed that the proposal was in compliance with the treaties between the two states.

MR. DE CASTRO: If the Commissioner will take note of my speech on U.S. military bases on 12
September 1986, I spoke on the selfreliance policy of the armed forces. However, due to very limited
resources, the only thing we could do is manufacture small arms ammunition. We cannot blame the
armed forces. We have to blame the whole Republic of the Philippines for failure to provide the
necessary funds to make the Philippine Armed Forces self-reliant. Indeed that is a beautiful dream.
And I would like it that way. But as of this time, fighting an insurgency case, a rebellion in our country
- insurgency - and with very limited funds and very limited number of men, it will be quite impossible
for the Philippines to appropriate the necessary funds therefor. However, if we say that the U.S.
government is furnishing us the military hardware, it is not control of our armed forces or of
our government. It is in compliance with the Mutual Defense Treaty. It is under the military
assistance program that it becomes the responsibility of the United States to furnish us the
necessary hardware in connection with the military bases agreement. Please be informed that there
are three (3) treaties connected with the military bases agreement; namely: the RP-US Military
Bases Agreement, the Mutual Defense Treaty and the Military Assistance Program.

My dear Commissioner, when we enter into a treaty and we are furnished the military
hardware pursuant to that treaty, it is not in control of our armed forces nor control of our
government. True indeed, we have military officers trained in the U.S. armed forces school. This is
part of our Military Assistance Program, but it does not mean that the minds of our military officers
are for the U.S. government, no. I am one of those who took four courses in the United States
schools, but I assure you, my mind is for the Filipino people. Also, while we are sending military
officers to train or to study in U.S. military schools, we are also sending our officers to study in other
military schools such as in Australia, England and in Paris. So, it does not mean that when we send
military officers to United States schools or to other military schools, we will be under the control of
that country. We also have foreign officers in our schools, we in the Command and General Staff
College in Fort Bonifacio and in our National Defense College, also in Fort Bonifacio.  (Emphases
377

supplied)

This logic was accepted in Tañada v. Angara, in which the Court ruled that independence does not
mean the absence of foreign participation:

Furthermore, the constitutional policy of a "self-reliant and independent national economy" does not
necessarily rule out the entry of foreign investments, goods and services. It contemplates
neither "economic seclusion" nor "mendicancy in the international community." As explained by
Constitutional Commissioner Bernardo Villegas, sponsor of this constitutional policy:

Economic self reliance is a primary objective of a developing country that is keenly aware of
overdependence on external assistance for even its most basic needs. It does not mean autarky or
economic seclusion; rather, it means avoiding mendicancy in the international
community. Independence refers to the freedom from undue foreign control of the national
economy, especially in such strategic industries as in the development of natural resources and
public utilities.  (Emphases supplied)
378
The heart of the constitutional restriction on foreign military facilities and bases is therefore the
assertion of independence from the U.S. and other foreign powers, as independence is exhibited by
the degree of foreign control exerted over these areas.  The essence of that independence is self-
1âwphi1

governance and self-control.  Independence itself is "[t]he state or condition of being free from
379

dependence, subjection, or control. " 380

Petitioners assert that EDCA provides the U.S. extensive control and authority over Philippine
facilities and locations, such that the agreement effectively violates Section 25 of the 1987
Constitution.381

Under Article VI(3) of EDCA, U.S. forces are authorized to act as necessary for "operational control
and defense." The term "operational control" has led petitioners to regard U.S. control over the
Agreed Locations as unqualified and, therefore, total.  Petitioners contend that the word "their"
382

refers to the subject "Agreed Locations."

This argument misreads the text, which is quoted below:

United States forces are authorized to exercise all rights and authorities within Agreed Locations that
are necessary for their operational control or defense, including taking appropriate measure to
protect United States forces and United States contractors. The United States should coordinate
such measures with appropriate authorities of the Philippines.

A basic textual construction would show that the word "their," as understood above, is a possessive
pronoun for the subject "they," a third-person personal pronoun in plural form. Thus, "their" cannot
be used for a non-personal subject such as "Agreed Locations." The simple grammatical conclusion
is that "their" refers to the previous third-person plural noun, which is "United States forces." This
conclusion is in line with the definition of operational control.

a. U.S. operational control as the exercise of authority over U.S. personnel, and not over the Agreed
Locations

Operational control, as cited by both petitioner and respondents, is a military term referring to

[t]he authority to perform those functions of command over subordinate forces involving organizing
and employing commands and forces, assigning tasks, designating objective, and giving
authoritative direction necessary to accomplish the mission. 383

At times, though, operational control can mean something slightly different. In JUSMAG Philippines
v. National Labor Relations Commission, the Memorandum of Agreement between the AFP and
JUSMAG Philippines defined the term as follows: 384

The term "Operational Control" includes, but is not limited to, all personnel administrative actions,
such as: hiring recommendations; firing recommendations; position classification; discipline;
nomination and approval of incentive awards; and payroll computation.

Clearly, traditional standards define "operational control" as personnel control. Philippine law, for
instance, deems operational control as one exercised by police officers and civilian authorities over
their subordinates and is distinct from the administrative control that they also exercise over police
subordinates.  Similarly, a municipal mayor exercises operational control over the police within the
385

municipal government,  just as city mayor possesses the same power over the police within the city
386

government. 387
Thus, the legal concept of operational control involves authority over personnel in a commander-
subordinate relationship and does not include control over the Agreed Locations in this particular
case. Though not necessarily stated in EDCA provisions, this interpretation is readily implied by the
reference to the taking of "appropriate measures to protect United States forces and United States
contractors."

It is but logical, even necessary, for the U.S. to have operational control over its own forces, in much
the same way that the Philippines exercises operational control over its own units.

For actual operations, EDCA is clear that any activity must be planned and pre-approved by the
MDB-SEB.  This provision evinces the partnership aspect of EDCA, such that both stakeholders
388

have a say on how its provisions should be put into effect.

b. Operational control vis-à-vis effective command and control

Petitioners assert that beyond the concept of operational control over personnel, qualifying access to
the Agreed Locations by the Philippine Designated Authority with the phrase "consistent with
operational safety and security requirements in accordance with agreed procedures developed by
the Parties" leads to the conclusion that the U.S. exercises effective control over the Agreed
Locations.  They claim that if the Philippines exercises possession of and control over a given area,
389

its representative should not have to be authorized by a special provision. 390

For these reasons, petitioners argue that the "operational control" in EDCA is the "effective
command and control" in the 1947 MBA.  In their Memorandum, they distinguish effective command
391

and control from operational control in U.S. parlance.  Citing the Doctrine for the Armed Forces of
392

the United States, Joint Publication 1, "command and control (C2)" is defined as "the exercise of
authority and direction by a properly designated commander over assigned and attached forces in
the accomplishment of the mission x x x."  Operational control, on the other hand, refers to "[t]hose
393

functions of command over assigned forces involving the composition of subordinate forces, the
assignment of tasks, the designation of objectives, the overall control of assigned resources, and the
full authoritative direction necessary to accomplish the mission."394

Two things demonstrate the errors in petitioners' line of argument.

Firstly, the phrase "consistent with operational safety and security requirements in accordance with
agreed procedures developed by the Parties" does not add any qualification beyond that which is
already imposed by existing treaties. To recall, EDCA is based upon prior treaties, namely the VFA
and the MDT.  Treaties are in themselves contracts from which rights and obligations may be
395

claimed or waived.  In this particular case, the Philippines has already agreed to abide by the
396

security mechanisms that have long been in place between the U.S. and the Philippines based on
the implementation of their treaty relations. 397

Secondly, the full document cited by petitioners contradicts the equation of "operational control" with
"effective command and control," since it defines the terms quite differently, viz:
398

Command and control encompasses the exercise of authority, responsibility, and direction by a
commander over assigned and attached forces to accomplish the mission. Command at all levels is
the art of motivating and directing people and organizations into action to accomplish missions.
Control is inherent in command. To control is to manage and direct forces and functions consistent
with a commander's command authority. Control of forces and functions helps commanders and
staffs compute requirements, allocate means, and integrate efforts. Mission command is the
preferred method of exercising C2. A complete discussion of tenets, organization, and processes for
effective C2 is provided in Section B, "Command and Control of Joint Forces," of Chapter V "Joint
Command and Control."

Operational control is defined thus: 399

OPCON is able to be delegated from a lesser authority than COCOM. It is the authority to perform
those functions of command over subordinate forces involving organizing and employing commands
and forces, assigning tasks, designating objectives, and giving authoritative direction over all
aspects of military operations and joint training necessary to accomplish the mission. It should be
delegated to and exercised by the commanders of subordinate organizations; normally, this authority
is exercised through subordinate JFCs, Service, and/or functional component commanders. OPCON
provides authority to organize and employ commands and forces as the commander considers
necessary to accomplish assigned missions. It does not include authoritative direction for logistics or
matters of administration, discipline, internal organization, or unit training. These elements of
COCOM must be specifically delegated by the CCDR. OPCON does include the authority to
delineate functional responsibilities and operational areas of subordinate JFCs.

Operational control is therefore the delegable aspect of combatant command, while command and
control is the overall power and responsibility exercised by the commander with reference to a
mission. Operational control is a narrower power and must be given, while command and control is
plenary and vested in a commander. Operational control does not include the planning,
programming, budgeting, and execution process input; the assignment of subordinate commanders;
the building of relationships with Department of Defense agencies; or the directive authority for
logistics, whereas these factors are included in the concept of command and control. 400

This distinction, found in the same document cited by petitioners, destroys the very foundation of the
arguments they have built: that EDCA is the same as the MBA.

c. Limited operational control over the Agreed Locations only for construction activitites

As petitioners assert, EDCA indeed contains a specific provision that gives to the U.S. operational
control within the Agreed Locations during construction activities.  This exercise of operational
401

control is premised upon the approval by the MDB and the SEB of the construction activity through
consultation and mutual agreement on the requirements and standards of the construction,
alteration, or improvement. 402

Despite this grant of operational control to the U.S., it must be emphasized that the grant is only for
construction activities. The narrow and limited instance wherein the U.S. is given operational control
within an Agreed Location cannot be equated with foreign military control, which is so abhorred by
the Constitution.

The clear import of the provision is that in the absence of construction activities, operational control
over the Agreed Location is vested in the Philippine authorities. This meaning is implicit in the
specific grant of operational control only during construction activities. The principle of constitutional
construction, "expressio unius est exclusio alterius," means the failure to mention the thing becomes
the ground for inferring that it was deliberately excluded. Following this construction, since EDCA
403

mentions the existence of U.S. operational control over the Agreed Locations for construction
activities, then it is quite logical to conclude that it is not exercised over other activities.

Limited control does not violate the Constitution. The fear of the commissioners was total control, to
the point that the foreign military forces might dictate the terms of their acts within the
Philippines.  More important, limited control does not mean an abdication or derogation of Philippine
404
sovereignty and legal jurisdiction over the Agreed Locations. It is more akin to the extension of
diplomatic courtesies and rights to diplomatic agents,  which is a waiver of control on a limited scale
405

and subject to the terms of the treaty.

This point leads us to the second standard envisioned by the framers of the Constitution: that the
Philippines must retain sovereignty and jurisdiction over its territory.

ii. Second standard: Philippine sovereignty and applicable law

EDCA states in its Preamble the "understanding for the United States not to establish a permanent
military presence or base in the territory of the Philippines." Further on, it likewise states the
recognition that "all United States access to and use of facilities and areas will be at the invitation of
the Philippines and with full respect for the Philippine Constitution and Philippine laws."

The sensitivity of EDCA provisions to the laws of the Philippines must be seen in light of Philippine
sovereignty and jurisdiction over the Agreed Locations.

Sovereignty is the possession of sovereign power,  while jurisdiction is the conferment by law of
406

power and authority to apply the law.  Article I of the 1987 Constitution states:
407

The national territory comprises the Philippine archipelago, with all the islands and waters embraced
therein, and all other territories over which the Philippines has sovereignty or jurisdiction,
consisting of its terrestrial, fluvial, and aerial domains, including its territorial sea, the seabed, the
subsoil, the insular shelves, and other submarine areas. The waters around, between, and
connecting the islands of the archipelago, regardless of their breadth and dimensions, form part of
the internal waters of the Philippines. (Emphasis supplied)

From the text of EDCA itself, Agreed Locations are territories of the Philippines that the U.S. forces
are allowed to access and use.  By withholding ownership of these areas and retaining unrestricted
408

access to them, the government asserts sovereignty over its territory. That sovereignty exists so
long as the Filipino people exist. 409

Significantly, the Philippines retains primary responsibility for security with respect to the Agreed
Locations. Hence, Philippine law remains in force therein, and it cannot be said that jurisdiction has
410

been transferred to the U.S. Even the previously discussed necessary measures for operational
control and defense over U.S. forces must be coordinated with Philippine authorities. 411

Jurisprudence bears out the fact that even under the former legal regime of the MBA, Philippine laws
continue to be in force within the bases.  The difference between then and now is that EDCA retains
412

the primary jurisdiction of the Philippines over the security of the Agreed Locations, an important
provision that gives it actual control over those locations. Previously, it was the provost marshal of
the U.S. who kept the peace and enforced Philippine law in the bases. In this instance, Philippine
forces act as peace officers, in stark contrast to the 1947 MBA provisions on jurisdiction. 413

iii. Third standard: must respect national security and territorial integrity

The last standard this Court must set is that the EDCA provisions on the Agreed Locations must not
impair or threaten the national security and territorial integrity of the Philippines.

This Court acknowledged in Bayan v. Zamora that the evolution of technology has essentially
rendered the prior notion of permanent military bases obsolete.
Moreover, military bases established within the territory of another state is no longer viable because
of the alternatives offered by new means and weapons of warfare such as nuclear weapons, guided
missiles as well as huge sea vessels that can stay afloat in the sea even for months and years
without returning to their home country. These military warships are actually used as substitutes for
a land-home base not only of military aircraft but also of military personnel and facilities. Besides,
vessels are mobile as compared to a land-based military headquarters. 414

The VFA serves as the basis for the entry of U.S. troops in a limited scope. It does not allow, for
instance, the re-establishment of the Subic military base or the Clark Air Field as U.S. military
reservations. In this context, therefore, this Court has interpreted the restrictions on foreign bases,
troops, or facilities as three independent restrictions. In accord with this interpretation, each
restriction must have its own qualification.

Petitioners quote from the website http://en.wikipedia.org to define what a military base is.  While 415

the source is not authoritative, petitioners make the point that the Agreed Locations, by granting
access and use to U.S. forces and contractors, are U.S. bases under a different name.  More 416

important, they claim that the Agreed Locations invite instances of attack on the Philippines from
enemies of the U.S. 417

We believe that the raised fear of an attack on the Philippines is not in the realm of law, but of
politics and policy. At the very least, we can say that under international law, EDCA does not provide
a legal basis for a justified attack on the Philippines.

In the first place, international law disallows any attack on the Agreed Locations simply because of
the presence of U.S. personnel. Article 2(4) of the United Nations Charter states that "All Members
shall refrain in their international relations from the threat or use of force against the territorial
integrity or political independence of any state, or in any other manner inconsistent with the
Purposes of the United Nations."  Any unlawful attack on the Philippines breaches the treaty, and
418

triggers Article 51 of the same charter, which guarantees the inherent right of individual or collective
self-defence.

Moreover, even if the lawfulness of the attack were not in question, international humanitarian law
standards prevent participants in an armed conflict from targeting non-participants. International
humanitarian law, which is the branch of international law applicable to armed conflict, expressly
limits allowable military conduct exhibited by forces of a participant in an armed conflict.  Under this
419

legal regime, participants to an armed conflict are held to specific standards of conduct that require
them to distinguish between combatants and non-combatants,  as embodied by the Geneva
420

Conventions and their Additional Protocols. 421

Corollary to this point, Professor John Woodcliffe, professor of international law at the University of
Leicester, noted that there is no legal consensus for what constitutes a base, as opposed to other
terms such as "facilities" or "installation."  In strategic literature, "base" is defined as an installation
422

"over which the user State has a right to exclusive control in an extraterritorial sense."  Since this
423

definition would exclude most foreign military installations, a more important distinction must be
made.

For Woodcliffe, a type of installation excluded from the definition of "base" is one that does not fulfill
a combat role. He cites an example of the use of the territory of a state for training purposes, such
as to obtain experience in local geography and climactic conditions or to carry out joint
exercises.  Another example given is an advanced communications technology installation for
424

purposes of information gathering and communication.  Unsurprisingly, he deems these non-


425
combat uses as borderline situations that would be excluded from the functional understanding of
military bases and installations.426

By virtue of this ambiguity, the laws of war dictate that the status of a building or person is presumed
to be protected, unless proven otherwise.  Moreover, the principle of distinction requires combatants
427

in an armed conflict to distinguish between lawful targets  and protected targets.  In an actual
428 429

armed conflict between the U.S. and a third state, the Agreed Locations cannot be considered U.S.
territory, since ownership of territory even in times of armed conflict does not change. 430

Hence, any armed attack by forces of a third state against an Agreed Location can only be legitimate
under international humanitarian law if it is against a bona fide U.S. military base, facility, or
installation that directly contributes to the military effort of the U.S. Moreover, the third state's forces
must take all measures to ensure that they have complied with the principle of distinction (between
combatants and non-combatants).

There is, then, ample legal protection for the Philippines under international law that would ensure its
territorial integrity and national security in the event an Agreed Location is subjected to attack. As
EDCA stands, it does not create the situation so feared by petitioners - one in which the Philippines,
while not participating in an armed conflict, would be legitimately targeted by an enemy of the U.S. 431

In the second place, this is a policy question about the wisdom of allowing the presence of U.S.
personnel within our territory and is therefore outside the scope of judicial review.

Evidently, the concept of giving foreign troops access to "agreed" locations, areas, or facilities within
the military base of another sovereign state is nothing new on the international plane. In fact, this
arrangement has been used as the framework for several defense cooperation agreements, such as
in the following:

1. 2006 U.S.-Bulgaria Defense Cooperation Agreement 432

2. 2009 U.S.-Colombia Defense Cooperation Agreement 433

3. 2009 U.S.-Poland Status of Forces Agreement 434

4. 2014 U.S.-Australia Force Posture Agreement 435

5. 2014 U.S.-Afghanistan Security and Defense Cooperation Agreement 436

In all of these arrangements, the host state grants U.S. forces access to their military bases.  That 437

access is without rental or similar costs to the U.S.  Further, U.S. forces are allowed to undertake
438

construction activities in, and make alterations and improvements to, the agreed locations, facilities,
or areas.  As in EDCA, the host states retain ownership and jurisdiction over the said bases.
439 440

In fact, some of the host states in these agreements give specific military-related rights to the U.S.
For example, under Article IV(l) of the US.-Bulgaria Defense Cooperation Agreement, "the United
States forces x x x are authorized access to and may use agreed facilities and areas x x x for
staging and deploying of forces and materiel, with the purpose of conducting x x x contingency
operations and other missions, including those undertaken in the framework of the North Atlantic
Treaty." In some of these agreements, host countries allow U.S. forces to construct facilities for the
latter’s exclusive use.441
Troop billeting, including construction of temporary structures, is nothing new. In Lim v. Executive
Secretary, the Court already upheld the Terms of Reference of Balikatan 02-1, which authorized
U.S. forces to set up "[t]emporary structures such as those for troop billeting, classroom instruction
and messing x x x during the Exercise." Similar provisions are also in the Mutual Logistics Support
Agreement of 2002 and 2007, which are essentially executive agreements that implement the VFA,
the MDT, and the 1953 Military Assistance Agreement. These executive agreements similarly tackle
the "reciprocal provision of logistic support, supplies, and services,"  which include "[b ]illeting, x x x
442

operations support (and construction and use of temporary structures incident to operations
support), training services, x x x storage services, x x x during an approved activity."  These logistic
443

supplies, support, and services include temporary use of "nonlethal items of military equipment
which are not designated as significant military equipment on the U.S. Munitions List, during an
approved activity."  The first Mutual Logistics Support Agreement has lapsed, while the second one
444

has been extended until 2017 without any formal objection before this Court from the Senate or any
of its members.

The provisions in EDCA dealing with Agreed Locations are analogous to those in the
aforementioned executive agreements. Instead of authorizing the building of temporary structures as
previous agreements have done, EDCA authorizes the U.S. to build permanent structures or alter or
improve existing ones for, and to be owned by, the Philippines.  EDCA is clear that the Philippines
445

retains ownership of altered or improved facilities and newly constructed permanent or non-
relocatable structures.  Under EDCA, U.S. forces will also be allowed to use facilities and areas for
446

"training; x x x; support and related activities; x x x; temporary accommodation of personnel;


communications" and agreed activities. 447

Concerns on national security problems that arise from foreign military equipment being present in
the Philippines must likewise be contextualized. Most significantly, the VFA already authorizes the
presence of U.S. military equipment in the country. Article VII of the VFA already authorizes the
U.S. to import into or acquire in the Philippines "equipment, materials, supplies, and other property"
that will be used "in connection with activities" contemplated therein. The same section also
recognizes that "[t]itle to such property shall remain" with the US and that they have the discretion to
"remove such property from the Philippines at any time."

There is nothing novel, either, in the EDCA provision on the prepositioning and storing of "defense
equipment, supplies, and materiel,"  since these are sanctioned in the VFA. In fact, the two
448

countries have already entered into various implementing agreements in the past that are
comparable to the present one. The Balikatan 02-1 Terms of Reference mentioned in Lim v.
Executive Secretary specifically recognizes that Philippine and U.S. forces "may share x x x in the
use of their resources, equipment and other assets." Both the 2002 and 2007 Mutual Logistics
Support Agreements speak of the provision of support and services, including the "construction and
use of temporary structures incident to operations support" and "storage services" during approved
activities.  These logistic supplies, support, and services include the "temporary use of x x x
449

nonlethal items of military equipment which are not designated as significant military equipment on
the U.S. Munitions List, during an approved activity." Those activities include "combined exercises
450

and training, operations and other deployments" and "cooperative efforts, such as humanitarian
assistance, disaster relief and rescue operations, and maritime anti-pollution operations" within or
outside Philippine territory.  Under EDCA, the equipment, supplies, and materiel that will be
451

prepositioned at Agreed Locations include "humanitarian assistance and disaster relief equipment,
supplies, and materiel. "  Nuclear weapons are specifically excluded from the materiel that will be
452

prepositioned.

Therefore, there is no basis to invalidate EDCA on fears that it increases the threat to our national
security. If anything, EDCA increases the likelihood that, in an event requiring a defensive response,
the Philippines will be prepared alongside the U.S. to defend its islands and insure its territorial
integrity pursuant to a relationship built on the MDT and VFA.

8. Others issues and concerns raised

A point was raised during the oral arguments that the language of the MDT only refers to mutual
help and defense in the Pacific area.  We believe that any discussion of the activities to be
453

undertaken under EDCA vis-a-vis the defense of areas beyond the Pacific is premature. We note
that a proper petition on that issue must be filed before we rule thereon. We also note that none of
the petitions or memoranda has attempted to discuss this issue, except only to theorize that the U.S.
will not come to our aid in the event of an attack outside of the Pacific. This is a matter of policy and
is beyond the scope of this judicial review.

In reference to the issue on telecommunications, suffice it to say that the initial impression of the
facility adverted to does appear to be one of those that require a public franchise by way of
congressional action under Section 11, Article XII of the Constitution. As respondents submit,
however, the system referred to in the agreement does not provide telecommunications services to
the public for compensation.  It is clear from Article VIl(2) of EDCA that the telecommunication
454

system is solely for the use of the U.S. and not the public in general, and that this system will not
interfere with that which local operators use. Consequently, a public franchise is no longer
necessary.

Additionally, the charge that EDCA allows nuclear weapons within Philippine territory is entirely
speculative. It is noteworthy that the agreement in fact specifies that the prepositioned materiel shall
not include nuclear weapons. Petitioners argue that only prepositioned nuclear weapons are
455

prohibited by EDCA; and that, therefore, the U.S. would insidiously bring nuclear weapons to
Philippine territory.  The general prohibition on nuclear weapons, whether prepositioned or not, is
456

already expressed in the 1987 Constitution.  It would be unnecessary or superfluous to include all
457

prohibitions already in the Constitution or in the law through a document like EDCA.

Finally, petitioners allege that EDCA creates a tax exemption, which under the law must originate
from Congress. This allegation ignores jurisprudence on the government's assumption of tax liability.
EDCA simply states that the taxes on the use of water, electricity, and public utilities are for the
account of the Philippine Government.  This provision creates a situation in which a contracting
458

party assumes the tax liability of the other.  In National Power Corporation v. Province of
459

Quezon, we distinguished between enforceable and unenforceable stipulations on the assumption of


tax liability. Afterwards, we concluded that an enforceable assumption of tax liability requires the
party assuming the liability to have actual interest in the property taxed.  This rule applies to EDCA,
460

since the Philippine Government stands to benefit not only from the structures to be built thereon or
improved, but also from the joint training with U.S. forces, disaster preparation, and the preferential
use of Philippine suppliers.  Hence, the provision on the assumption of tax liability does not
461

constitute a tax exemption as petitioners have posited.

Additional issues were raised by petitioners, all relating principally to provisions already sufficiently
addressed above. This Court takes this occasion to emphasize that the agreement has been
construed herein as to absolutely disauthorize the violation of the Constitution or any applicable
statute. On the contrary, the applicability of Philippine law is explicit in EDCA.

EPILOGUE

The fear that EDCA is a reincarnation of the U.S. bases so zealously protested by noted
personalities in Philippine history arises not so much from xenophobia, but from a genuine desire for
self-determination, nationalism, and above all a commitment to ensure the independence of the
Philippine Republic from any foreign domination.

Mere fears, however, cannot curtail the exercise by the President of the Philippines of his
Constitutional prerogatives in respect of foreign affairs. They cannot cripple him when he deems that
additional security measures are made necessary by the times. As it stands, the Philippines through
the Department of Foreign Affairs has filed several diplomatic protests against the actions of the
People's Republic of China in the West Philippine Sea;  initiated arbitration against that country
462

under the United Nations Convention on the Law of the Sea;  is in the process of negotiations with
463

the Moro Islamic Liberation Front for peace in Southern Philippines,  which is the subject of a
464

current case before this Court; and faces increasing incidents of kidnappings of Filipinos and
foreigners allegedly by the Abu Sayyaf or the New People's Army.  The Philippine military is
465

conducting reforms that seek to ensure the security and safety of the nation in the years to
come.  In the future, the Philippines must navigate a world in which armed forces fight with
466

increasing sophistication in both strategy and technology, while employing asymmetric warfare and
remote weapons.

Additionally, our country is fighting a most terrifying enemy: the backlash of Mother Nature. The
Philippines is one of the countries most directly affected and damaged by climate change. It is no
coincidence that the record-setting tropical cyclone Yolanda (internationally named Haiyan), one of
the most devastating forces of nature the world has ever seen hit the Philippines on 8 November
2013 and killed at least 6,000 people.  This necessitated a massive rehabilitation project.  In the
467 468

aftermath, the U.S. military was among the first to extend help and support to the Philippines.

That calamity brought out the best in the Filipinos as thousands upon thousands volunteered their
help, their wealth, and their prayers to those affected. It also brought to the fore the value of having
friends in the international community.

In order to keep the peace in its archipelago in this region of the world, and to sustain itself at the
same time against the destructive forces of nature, the Philippines will need friends. Who they are,
and what form the friendships will take, are for the President to decide. The only restriction is what
the Constitution itself expressly prohibits. It appears that this overarching concern for balancing
constitutional requirements against the dictates of necessity was what led to EDCA.

As it is, EDCA is not constitutionally infirm. As an executive agreement, it remains consistent with
existing laws and treaties that it purports to implement.

WHEREFORE, we hereby DISMISS the petitions.

SO ORDERED.

G.R. No. 169815             August 13, 2008

BUREAU OF FISHERIES AND AQUATIC RESOURCES (BFAR) EMPLOYEES UNION, REGIONAL


OFFICE NO. VII, CEBU CITY, petitioner, 
vs.
COMMISSION ON AUDIT, respondent.
DECISION

PUNO, C.J.:

On appeal are the Decision1 dated April 8, 2005 of respondent Commission on Audit (COA) in LAO-N-
2005-119 upholding the disallowance by the COA Legal and Adjudication Office (COA-LAO), Regional
Office No. VII, Cebu City of the P10,000.00 Food Basket Allowance granted by BFAR to each of its
employees in 1999, and COA Resolution2 dated August 5, 2005, denying petitioner’s motion for
reconsideration of said Decision.

First, the facts:

On October 18, 1999, petitioner Bureau of Fisheries and Aquatic Resources (BFAR) Employees Union,
Regional Office No. VII, Cebu City issued Resolution No. 01, series of 1999 requesting the BFAR Central
Office for a Food Basket Allowance. It justified its request on the high cost of living, i.e., "the increase in
prices of petroleum products which catapulted the cost of food commodities, has greatly affected the
economic conditions and living standard of the government employees of BFAR Region VII and could
hardly sustain its need to cope up with the four (4) basic needs, i.e., food, shelter, clothing and
education."3 It also relied on the Employees Suggestions and Incentive Awards System (ESIAS),
pursuant to Book V of Executive Order No. 292, or the Administrative Code of 1987, and approved by the
Civil Service Commission on December 3, 1996. The ESIAS "includes the granting of incentives that will
help employees overcome present economic difficulties, boost their morale, and further commitment and
dedication to public service."4 Regional Director Corazon M. Corrales of BFAR Region VII indorsed the
Resolution, and Malcolm I. Sarmiento, Jr., Director of BFAR recommended its approval. Honorable Cesar
M. Drilon, Jr., Undersecretary for Fisheries and Livestock of the Department of Agriculture, approved the
request for Authority to Grant a Gift Check or the Food Basket Allowance at the rate of P10,000.00 each
to the 130 employees of BFAR Region VII, or in the total amount of P1,322,682.00.5 On the strength of
the approval, Regional Director Corrales released the allowance to the BFAR employees.

On post audit, the Commission on Audit – Legal and Adjudication Office (COA-LAO) Regional Office No.
VII, Cebu City disallowed the grant of Food Basket Allowance under Notice of Disallowance No. 2003-
022-101 (1999) dated September 19, 2003. It ruled that the allowance had no legal basis and that it
violated: a) Sec. 15(d) of the General Appropriations Act of 1999, prohibiting the payment of honoraria,
allowances, or other forms of compensation to any government official or employee, except those
specifically authorized by law; b) par. 4.5 of Budget Circular No. 16 dated November 28, 1998, prohibiting
the grant of food, rice, gift checks, or any other form of incentives/allowances, except those authorized via
Administrative Order by the Office of the President; and c) Sec. 12 of Republic Act (R.A.) No. 6758, or the
Salary Standardization Law of 1989, which includes all allowances in the standardized salary rates,
subject to certain exceptions.

On February 26, 2004, BFAR Regional Office No. VII, through Regional Director Corrales, moved for
reconsideration and prayed for the lifting of the disallowance. It argued that the grant of Food Basket
Allowance would enhance the welfare and productivity of the employees. Further, it contended that the
approval by the Honorable Drilon, Undersecretary for Fisheries and Livestock, of the said benefit was the
law itself which vested the specific authority for its release. The Commission on Audit – Legal and
Adjudication Office (COA-LAO) Regional Office No. VII, Cebu City denied the motion.

Petitioner appealed to the Commission on Audit – Legal and Adjudication Office (COA-LAO) National,
Quezon City. The appeal was denied in a Decision dated April 8, 2005. Petitioner’s motion for
reconsideration was likewise denied in a Resolution dated August 5, 2005.

Hence, this appeal.

Petitioner cites the following grounds for its appeal:


1. The disallowance in question is unconstitutional as it contravenes the fundamental principle of
the State enshrined under Sections 9 and 10, Article II of the 1987 Constitution, which provide as
follows:

SEC. 9. The State shall promote a just and dynamic social order that will ensure the
prosperity and independence of the nation and free the people from poverty through
policies that provide adequate social services, promote full employment, a rising standard
of living, and an improved quality of life for all.

SEC. 10. The State shall promote social justice in all phases of national development.6

2. The Undersecretary for Fisheries and Livestock is an extension of the Secretary of Agriculture
who is an alter-ego of the President. His approval was tantamount to the authority from the Office
of the President, as contemplated in DBM Budget Circular No. 16, dated November 28, 1998.7

3. The grant of the Food Basket Allowance is in conformity with Sec. 12 of the Salary
Standardization Law.8

We deny the petition.

First, we rule on the issue of constitutionality. Petitioner invokes the provisions of the 1987 Constitution on
social justice to warrant the grant of the Food Basket Allowance. Time and again, we have ruled that the
social justice provisions of the Constitution are not self-executing principles ready for enforcement
through the courts. They are merely statements of principles and policies. To give them effect, legislative
enactment is required. As we held in Kilosbayan, Incorporated v. Morato,9 the principles and state
policies enumerated in Article II and some sections of Article XII are "not self-executing provisions, the
disregard of which can give rise to a cause of action in the courts. They do not embody judicially
enforceable constitutional rights but guidelines for legislation."10

Second, petitioner contends that the approval of the Department of Agriculture (DA) Undersecretary for
Fisheries and Livestock of the Food Basket Allowance is the law which authorizes its release. It is crystal
clear that the DA Undersecretary has no authority to grant any allowance to the employees of BFAR.
Section 4.5 of Budget Circular No. 16 dated November 28, 1998 states:

All agencies are hereby prohibited from granting any food, rice, gift checks, or any other form of
incentives/allowances except those authorized via Administrative Order by the Office of the
President.

In the instant case, no Administrative Order has been issued by the Office of the President to exempt
BFAR from the express prohibition against the grant of any food, rice, gift checks, or any other form of
incentive/allowance to its employees.

Petitioner argues that the grant of the Food Basket Allowance does not violate Sec. 12 of R.A. No. 6758
or the Salary Standardization Law. This law was passed to standardize salary rates among government
personnel and do away with multiple allowances and other incentive packages and the resulting
differences in compensation among them.11 Sec. 12 of the law provides:

Consolidation of Allowances and Compensation. — All allowances, except for representation and
transportation allowances; clothing and laundry allowances; subsistence allowance of marine
officers and crew on board government vessels and hospital personnel; hazard pay; allowances
of foreign service personnel stationed abroad; and such other additional compensation not
otherwise specified herein as may be determined by the DBM [Department of Budget and
Management], shall be deemed included in the standardized salary rates herein prescribed. Such
other additional compensation, whether in cash or in kind, being received by incumbents only as
of July 1, 1989 not integrated into the standardized salary rates shall continue to be authorized.

Existing additional compensation of any national government official or employee paid from local
funds of a local government unit shall be absorbed into the basic salary of said official or
employee and shall be paid by the National Government.

Under Sec. 12, as quoted, all kinds of allowances are integrated in the standardized salary rates. The
exceptions are:

1. representation and transportation allowance (RATA);

2. clothing and laundry allowance;

3. subsistence allowance of marine officers and crew on board government vessels;

4. subsistence allowance of hospital personnel;

5. hazard pay;

6. allowances of foreign service personnel stationed abroad; and

7. such other additional compensation not otherwise specified herein as may be determined by
the DBM.

Petitioner contends that the Food Basket Allowance falls under the 7th category above, that of "other
additional compensation not otherwise specified herein as may be determined by the DBM."

The Court has had the occasion to interpret Sec. 12 of R.A. No. 6758. In National Tobacco
Administration v. Commission on Audit,12 we held that under the first sentence of Section 12, the
benefits excluded from the standardized salary rates are the "allowances" or those which are usually
granted to officials and employees of the government to defray or reimburse the expenses incurred in the
performance of their official functions. These are the RATA, clothing and laundry allowance, subsistence
allowance of marine officers and crew on board government vessels and hospital personnel, hazard pay,
and others, as enumerated in the first sentence of Section 12. We further ruled that the phrase "and such
other additional compensation not otherwise specified herein as may be determined by the DBM" is a
catch-all proviso for benefits in the nature of allowances similar to those enumerated. In Philippine Ports
Authority v. Commission on Audit,13 we explained that if these allowances were consolidated with the
standardized salary rates, then government officials or employees would be compelled to spend their
personal funds in attending to their duties.

In the instant case, the Food Basket Allowance is definitely not in the nature of an allowance to reimburse
expenses incurred by officials and employees of the government in the performance of their official
functions. It is not payment in consideration of the fulfillment of official duty. It is a form of financial
assistance to all officials and employees of BFAR. Petitioner itself stated that the Food Basket Allowance
has the purpose of alleviating the economic condition of BFAR employees.

Next, petitioner relies on National Compensation Circular No. 59 dated September 30, 1989, issued by
the DBM, which is the "List of Allowances/Additional Compensation of Government Officials and
Employees which shall be Deemed Integrated into the Basic Salary." The list enumerates the following
allowances/additional compensation which shall be incorporated in the basic salary, hence, may no
longer be granted to government employees:
1. Cost of Living Allowance (COLA);

2. Inflation connected allowance;

3. Living Allowance;

4. Emergency Allowance;

5. Additional Compensation of Public Health Nurses assigned to public health nursing;

6. Additional Compensation of Rural Health Physicians;

7. Additional Compensation of Nurses in Malacañang Clinic;

8. Nurses Allowance in the Air Transportation Office;

9. Assignment Allowance of School Superintendents;

10. Post allowance of Postal Service Office employees;

11. Honoraria/allowances which are regularly given except the following:

a. those for teaching overload;

b. in lieu of overtime pay;

c. for employees on detail with task forces/special projects;

d. researchers, experts and specialists who are acknowledged authorities in their field of
specialization;

e. lecturers and resource persons;

f. Municipal Treasurers deputized by the Bureau of Internal Revenue to collect and remit
internal revenue collections; and

g. Executive positions in State Universities and Colleges filled by designation from among
their faculty members.

12. Subsistence Allowance of employees except those authorized under EO [Executive Order]
No. 346 and uniformed personnel of the Armed Forces of the Philippines and Integrated National
Police;

13. Laundry Allowance of employees except those hospital/sanitaria personnel who attend
directly to patients and who by the nature of their duties are required to wear uniforms, prison
guards and uniformed personnel of the Armed Forces of the Philippines and Integrated National
Police; and

14. Incentive allowance/fee/pay except those authorized under the General Appropriations Act
and Section 33 of P.D. No. 807.
Petitioner invokes the rule of statutory construction that "what is not included is excluded." Inclusio unius
est exclusio alterius. Petitioner claims that the Food Basket Allowance is distinct and separate from the
specific allowances/additional compensation listed in the circular.

Again, we reject petitioner’s contention. The Food Basket Allowance falls under the 14 th category, that of
incentive allowance/fee/pay. Petitioner itself justified the Food Basket Allowance as an incentive to the
employees to encourage them to be more productive and efficient.14 Under National Compensation
Circular No. 59, exceptions to the incentive allowance/fee/pay category are those authorized under the
General Appropriations Act (GAA) and Section 33 of Presidential Decree (P.D.) No. 807. Sec. 15(d) of the
GAA for Fiscal Year 1999 or R.A. No. 8745 clearly prohibits the payment of honoraria, allowances or
other forms of compensation to any government official or employee, except those specifically authorized
by law. There is no law authorizing the grant of the subject Food Basket Allowance. Further, Sec. 33 of
P.D. No. 807 or the Civil Service Decree of the Philippines does not exempt the Food Basket Allowance
from the general rule. Sec. 33 states:

Section 33. Employee Suggestions and Incentive Award System. There shall be established a


government-wide employee suggestions and incentive awards system which shall be
administered under such rules, regulations, and standards as may be promulgated by the
Commission.

In accordance with rules, regulations, and standards promulgated by the Commission, the
President or the head of each department or agency is authorized to incur whatever necessary
expenses involved in the honorary recognition of subordinate officers and employees of the
government who by their suggestions, inventions, superior accomplishment, and other personal
efforts contribute to the efficiency, economy, or other improvement of government operations, or
who perform such other extraordinary acts or services in the public interest in connection with, or
in relation to, their official employment.

We are not convinced that the Food Basket Allowance falls under the incentive award system
contemplated above. The decree speaks of suggestions, inventions, superior accomplishments, and other
personal efforts contributed by an employee to the efficiency, economy, or other improvement of
government operations, or other extraordinary acts or services performed by an employee in the public
interest in connection with, or in relation to, his official employment. In the instant case, the Food Basket
Allowance was granted to all BFAR employees, without distinction. It was not granted due to any
extraordinary contribution or exceptional accomplishment by an employee. The Food Basket Allowance
was primarily an economic monetary assistance to the employees.

Lastly, we note, as the Office of the Solicitor General, on behalf of respondent did, that petitioner failed to
exhaust its administrative remedies. It stopped seeking remedies at the level of respondent’s Legal and
Adjudication Office. It failed to appeal the latter’s adverse decision to the Commission on Audit proper.
The consequence for failure to exhaust administrative remedies is clear: the disallowance, as ruled by the
Commission on Audit – Legal and Adjudication Office Regional Office No. VII, Cebu City and upheld by
the Commission on Audit – Legal and Adjudication Office National, Quezon City, became final and
executory. Sections 48 and 51 of Presidential Decree No. 1445, or the Government Auditing Code of the
Philippines provide:

Section 48. Appeal from decision of auditors. – Any person aggrieved by the decision of an
auditor of any government agency in the settlement of an account or claim may, within six months
from receipt of a copy of the decision, appeal in writing to the Commission.

Section 51. Finality of decisions of the Commission or any auditor. – A decision of the
Commission or of any auditor upon any matter within its or his jurisdiction, if not appealed as
herein provided, shall be final and executory.
IN VIEW WHEREOF, the petition is DENIED. The Decision and Resolution of the Commission on Audit –
Legal and Adjudication Office dated April 8, 2005 and August 5, 2005, respectively, in LAO-N-2005-119,
are AFFIRMED.

SO ORDERED.

G.R. No. 179267               June 25, 2013

JESUS C. GARCIA, Petitioner, 
vs.
THE HONORABLE RAY ALAN T. DRILON, Presiding Judge, Regional Trial Court-Branch 41,
Bacolod City, and ROSALIE JAYPE-GARCIA, for herself and in behalf of minor children,
namely: JO-ANN, JOSEPH EDUARD, JESSE ANTHONE, all surnamed GARCIA, Respondents.

DECISION

PERLAS-BERNABE, J.:

Hailed as the bastion of Christianity in Asia, the Philippines boasts of 86.8 million Filipinos- or 93
percent of a total population of 93.3 million – adhering to the teachings of Jesus Christ. 1 Yet, the
admonition for husbands to love their wives as their own bodies just as Christ loved the church and
gave himself up for her2 failed to prevent, or even to curb, the pervasiveness of violence against
Filipino women. The National Commission on the Role of Filipino Women (NCRFW) reported that,
for the years 2000-2003, "female violence comprised more than 90o/o of all forms of abuse and
violence and more than 90% of these reported cases were committed by the women's intimate
partners such as their husbands and live-in partners."3

Thus, on March 8, 2004, after nine (9) years of spirited advocacy by women's groups, Congress
enacted Republic Act (R.A.) No. 9262, entitled "An Act Defining Violence Against Women and Their
Children, Providing for Protective Measures for Victims, Prescribing Penalties Therefor, and for
Other Purposes." It took effect on March 27, 2004.4

R.A. 9262 is a landmark legislation that defines and criminalizes acts of violence against women and
their children (VAWC) perpetrated by women's intimate partners, i.e, husband; former husband; or
any person who has or had a sexual or dating relationship, or with whom the woman has a common
child.5 The law provides for protection orders from the barangay and the courts to prevent the
commission of further acts of VAWC; and outlines the duties and responsibilities of barangay
officials, law enforcers, prosecutors and court personnel, social workers, health care providers, and
other local government officials in responding to complaints of VAWC or requests for assistance.

A husband is now before the Court assailing the constitutionality of R.A. 9262 as being violative of
the equal protection and due process clauses, and an undue delegation of judicial power to
barangay officials.

The Factual Antecedents

On March 23, 2006, Rosalie Jaype-Garcia (private respondent) filed, for herself and in behalf of her
minor children, a verified petition6 (Civil Case No. 06-797) before the Regional Trial Court (RTC) of
Bacolod City for the issuance of a Temporary Protection Order (TPO) against her husband, Jesus C.
Garcia (petitioner), pursuant to R.A. 9262. She claimed to be a victim of physical abuse; emotional,
psychological, and economic violence as a result of marital infidelity on the part of petitioner, with
threats of deprivation of custody of her children and of financial support.7

Private respondent's claims

Private respondent married petitioner in 2002 when she was 34 years old and the former was eleven
years her senior. They have three (3) children, namely: Jo-Ann J. Garcia, 17 years old, who is the
natural child of petitioner but whom private respondent adopted; Jessie Anthone J. Garcia, 6 years
old; and Joseph Eduard J. Garcia, 3 years old.8

Private respondent described herself as a dutiful and faithful wife, whose life revolved around her
husband. On the other hand, petitioner, who is of Filipino-Chinese descent, is dominant, controlling,
and demands absolute obedience from his wife and children. He forbade private respondent to pray,
and deliberately isolated her from her friends. When she took up law, and even when she was
already working part time at a law office, petitioner trivialized her ambitions and prevailed upon her
to just stay at home. He was often jealous of the fact that his attractive wife still catches the eye of
some men, at one point threatening that he would have any man eyeing her killed.9

Things turned for the worse when petitioner took up an affair with a bank manager of Robinson's
Bank, Bacolod City, who is the godmother of one of their sons. Petitioner admitted to the affair when
private respondent confronted him about it in 2004. He even boasted to the household help about
his sexual relations with said bank manager. Petitioner told private respondent, though, that he was
just using the woman because of their accounts with the bank.10

Petitioner's infidelity spawned a series of fights that left private respondent physically and
emotionally wounded. In one of their quarrels, petitioner grabbed private respondent on both arms
and shook her with such force that caused bruises and hematoma. At another time, petitioner hit
private respondent forcefully on the lips that caused some bleeding. Petitioner sometimes turned his
ire on their daughter, Jo-Ann, who had seen the text messages he sent to his paramour and whom
he blamed for squealing on him. He beat Jo-Ann on the chest and slapped her many times. When
private respondent decided to leave petitioner, Jo-Ann begged her mother to stay for fear that if the
latter leaves, petitioner would beat her up. Even the small boys are aware of private respondent's
sufferings. Their 6-year-old son said that when he grows up, he would beat up his father because of
his cruelty to private respondent.11

All the emotional and psychological turmoil drove private respondent to the brink of despair. On
December 17, 2005, while at home, she attempted suicide by cutting her wrist. She was found by
her son bleeding on the floor. Petitioner simply fled the house instead of taking her to the hospital.
Private respondent was hospitalized for about seven (7) days in which time petitioner never bothered
to visit, nor apologized or showed pity on her. Since then, private respondent has been undergoing
therapy almost every week and is taking anti-depressant medications.12

When private respondent informed the management of Robinson's Bank that she intends to file
charges against the bank manager, petitioner got angry with her for jeopardizing the manager's job.
He then packed his things and told private respondent that he was leaving her for good. He even
told private respondent's mother, who lives with them in the family home, that private respondent
should just accept his extramarital affair since he is not cohabiting with his paramour and has not
sired a child with her.13
Private respondent is determined to separate from petitioner but she is afraid that he would take her
children from her and deprive her of financial support. Petitioner had previously warned her that if
she goes on a legal battle with him, she would not get a single centavo.14

Petitioner controls the family businesses involving mostly the construction of deep wells. He is the
President of three corporations – 326 Realty Holdings, Inc., Negros Rotadrill Corporation, and J-Bros
Trading Corporation – of which he and private respondent are both stockholders. In contrast to the
absolute control of petitioner over said corporations, private respondent merely draws a monthly
salary of ₱20,000.00 from one corporation only, the Negros Rotadrill Corporation. Household
expenses amounting to not less than ₱200,000.00 a month are paid for by private respondent
through the use of credit cards, which, in turn, are paid by the same corporation together with the
bills for utilities.15

On the other hand, petitioner receives a monthly salary of ₱60,000.00 from Negros Rotadrill
Corporation, and enjoys unlimited cash advances and other benefits in hundreds of thousands of
pesos from the corporations.16 After private respondent confronted him about the affair, petitioner
forbade her to hold office at JBTC Building, Mandalagan, where all the businesses of the
corporations are conducted, thereby depriving her of access to full information about said
businesses. Until the filing of the petition a quo, petitioner has not given private respondent an
accounting of the businesses the value of which she had helped raise to millions of pesos.17

Action of the RTC of Bacolod City

Finding reasonable ground to believe that an imminent danger of violence against the private
respondent and her children exists or is about to recur, the RTC issued a TPO 18 on March 24, 2006
effective for thirty (30) days, which is quoted hereunder:

Respondent (petitioner herein), Jesus Chua Garcia, is hereby:

a) Ordered to remove all his personal belongings from the conjugal dwelling or family home
within 24 hours from receipt of the Temporary Restraining Order and if he refuses, ordering
that he be removed by police officers from the conjugal dwelling; this order is enforceable
notwithstanding that the house is under the name of 236 Realty Holdings Inc. (Republic Act
No. 9262 states "regardless of ownership"), this is to allow the Petitioner (private respondent
herein) to enter the conjugal dwelling without any danger from the Respondent.

After the Respondent leaves or is removed from the conjugal dwelling, or anytime the
Petitioner decides to return to the conjugal dwelling to remove things, the Petitioner shall be
assisted by police officers when re-entering the family home.

The Chief of Police shall also give the Petitioner police assistance on Sunday, 26 March
2006 because of the danger that the Respondent will attempt to take her children from her
when he arrives from Manila and finds out about this suit.

b) To stay away from the petitioner and her children, mother and all her household help and
driver from a distance of 1,000 meters, and shall not enter the gate of the subdivision where
the Petitioner may be temporarily residing.

c) Not to harass, annoy, telephone, contact or otherwise communicate with the Petitioner,
directly or indirectly, or through other persons, or contact directly or indirectly her children,
mother and household help, nor send gifts, cards, flowers, letters and the like. Visitation
rights to the children may be subject of a modified TPO in the future.

d) To surrender all his firearms including a .9MM caliber firearm and a Walther PPK and
ordering the Philippine National Police Firearms and Explosives Unit and the Provincial
Director of the PNP to cancel all the Respondent's firearm licenses. He should also be
ordered to surrender any unlicensed firearms in his possession or control.

e) To pay full financial support for the Petitioner and the children, including rental of a house
for them, and educational and medical expenses.

f) Not to dissipate the conjugal business.

g) To render an accounting of all advances, benefits, bonuses and other cash he received
from all the corporations from 1 January 2006 up to 31 March 2006, which himself and as
President of the corporations and his Comptroller, must submit to the Court not later than 2
April 2006. Thereafter, an accounting of all these funds shall be reported to the court by the
Comptroller, copy furnished to the Petitioner, every 15 days of the month, under pain of
Indirect Contempt of Court.

h) To ensure compliance especially with the order granting support pendente lite, and
considering the financial resources of the Respondent and his threat that if the Petitioner
sues she will not get a single centavo, the Respondent is ordered to put up a BOND TO
KEEP THE PEACE in the amount of FIVE MILLION PESOS, in two sufficient sureties.

On April 24, 2006, upon motion 19 of private respondent, the trial court issued an amended
TPO,20 effective for thirty (30) days, which included the following additional provisions:

i) The petitioners (private respondents herein) are given the continued use of the Nissan
Patrol and the Starex Van which they are using in Negros Occidental.

j) The petitioners are given the continued use and occupation of the house in Parañaque, the
continued use of the Starex van in Metro Manila, whenever they go to Manila.

k) Respondent is ordered to immediately post a bond to keep the peace, in two sufficient
sureties.

l) To give monthly support to the petitioner provisionally fixed in the sum of One Hundred
Fifty Thousand Pesos (Php 150,000.00) per month plus rental expenses of Fifty Thousand
Pesos (Php 50,000.00) per month until the matter of support could be finally resolved.

Two days later, or on April 26, 2006, petitioner filed an Opposition to the Urgent Ex-Parte Motion for
Renewal of the TPO21 seeking the denial of the renewal of the TPO on the grounds that it did not (1)
comply with the three-day notice rule, and (2) contain a notice of hearing. He further asked that the
TPO be modified by (1) removing one vehicle used by private respondent and returning the same to
its rightful owner, the J-Bros Trading Corporation, and (2) cancelling or reducing the amount of the
bond from ₱5,000,000.00 to a more manageable level at ₱100,000.00.

Subsequently, on May 23, 2006, petitioner moved22 for the modification of the TPO to allow him
visitation rights to his children.
On May 24, 2006, the TPO was renewed and extended yet again, but subject only to the following
modifications prayed for by private respondent:

a) That respondent (petitioner herein) return the clothes and other personal belongings of
Rosalie and her children to Judge Jesus Ramos, co-counsel for Petitioner, within 24 hours
from receipt of the Temporary Protection Order by his counsel, otherwise be declared in
Indirect Contempt of Court;

b) Respondent shall make an accounting or list of furniture and equipment in the conjugal
house in Pitimini St., Capitolville Subdivision, Bacolod City within 24 hours from receipt of the
Temporary Protection Order by his counsel;

c) Ordering the Chief of the Women's Desk of the Bacolod City Police Headquarters to
remove Respondent from the conjugal dwelling within eight (8) hours from receipt of the
Temporary Protection Order by his counsel, and that he cannot return until 48 hours after the
petitioners have left, so that the petitioner Rosalie and her representatives can remove
things from the conjugal home and make an inventory of the household furniture, equipment
and other things in the conjugal home, which shall be submitted to the Court.

d) Deliver full financial support of Php200,000.00 and Php50,000.00 for rental and
Php25,000.00 for clothes of the three petitioners (sic) children within 24 hours from receipt of
the Temporary Protection Order by his counsel, otherwise be declared in indirect contempt of
Court;

e) That respondent surrender his two firearms and all unlicensed firearms to the Clerk of
Court within 24 hours from receipt of the Temporary Protection Order by his counsel;

f) That respondent shall pay petitioner educational expenses of the children upon
presentation of proof of payment of such expenses.23

Claiming that petitioner continued to deprive them of financial support; failed to faithfully comply with
the TPO; and committed new acts of harassment against her and their children, private respondent
filed another application24 for the issuance of a TPO ex parte. She alleged inter

alia that petitioner contrived a replevin suit against himself by J-Bros Trading, Inc., of which the latter
was purportedly no longer president, with the end in view of recovering the Nissan Patrol and Starex
Van used by private respondent and the children. A writ of replevin was served upon private
respondent by a group of six or seven policemen with long firearms that scared the two small boys,
Jessie Anthone and Joseph Eduard.25

While Joseph Eduard, then three years old, was driven to school, two men allegedly attempted to
kidnap him, which incident traumatized the boy resulting in his refusal to go back to school. On
another occasion, petitioner allegedly grabbed their daughter, Jo-Ann, by the arm and threatened
her.26 The incident was reported to the police, and Jo-Ann subsequently filed a criminal complaint
against her father for violation of R.A. 7610, also known as the "Special Protection of Children
Against Child Abuse, Exploitation and Discrimination Act."

Aside from the replevin suit, petitioner's lawyers initiated the filing by the housemaids working at the
conjugal home of a complaint for kidnapping and illegal detention against private respondent. This
came about after private respondent, armed with a TPO, went to said home to get her and her
children's belongings. Finding some of her things inside a housemaid's (Sheryl Jamola) bag in the
maids' room, private respondent filed a case for qualified theft against Jamola.27

On August 23, 2006, the RTC issued a TPO,28 effective for thirty (30) days, which reads as follows:

Respondent (petitioner herein), Jesus Chua Garcia, is hereby:

1) Prohibited from threatening to commit or committing, personally or through another, acts


of violence against the offended party;

2) Prohibited from harassing, annoying, telephoning, contacting or otherwise communicating


in any form with the offended party, either directly or indirectly;

3) Required to stay away, personally or through his friends, relatives, employees or agents,
from all the Petitioners Rosalie J. Garcia and her children, Rosalie J. Garcia's three brothers,
her mother Primitiva Jaype, cook Novelita Caranzo, driver Romeo Hontiveros,
laundrywoman Mercedita Bornales, security guard Darwin Gayona and the petitioner's other
household helpers from a distance of 1,000 meters, and shall not enter the gate of the
subdivision where the Petitioners are temporarily residing, as well as from the schools of the
three children; Furthermore, that respondent shall not contact the schools of the children
directly or indirectly in any manner including, ostensibly to pay for their tuition or other fees
directly, otherwise he will have access to the children through the schools and the TPO will
be rendered nugatory;

4) Directed to surrender all his firearms including .9MM caliber firearm and a Walther PPK to
the Court;

5) Directed to deliver in full financial support of Php200,000.00 a month and Php50,000.00


for rental for the period from August 6 to September 6, 2006; and support in arrears from
March 2006 to August 2006 the total amount of Php1,312,000.00;

6) Directed to deliver educational expenses for 2006-2007 the amount of Php75,000.00 and
Php25,000.00;

7) Directed to allow the continued use of a Nissan Patrol with Plate No. FEW 508 and a
Starex van with Plate No. FFD 991 and should the respondent fail to deliver said vehicles,
respondent is ordered to provide the petitioner another vehicle which is the one taken by J
Bros Tading;

8) Ordered not to dissipate, encumber, alienate, sell, lease or otherwise dispose of the
conjugal assets, or those real properties in the name of Jesus Chua Garcia only and those in
which the conjugal partnership of gains of the Petitioner Rosalie J. Garcia and respondent
have an interest in, especially the conjugal home located in No. 14, Pitimini St., Capitolville
Subdivision, Bacolod City, and other properties which are conjugal assets or those in which
the conjugal partnership of gains of Petitioner Rosalie J. Garcia and the respondent have an
interest in and listed in Annexes "I," "I-1," and "I-2," including properties covered by TCT
Nos. T-186325 and T-168814;

9) Ordered that the Register of Deeds of Bacolod City and E.B. Magalona shall be served a
copy of this TEMPORARY PROTECTION ORDER and are ordered not to allow the transfer,
sale, encumbrance or disposition of these above-cited properties to any person, entity or
corporation without the personal presence of petitioner Rosalie J. Garcia, who shall affix her
signature in the presence of the Register of Deeds, due to the fear of petitioner Rosalie that
her signature will be forged in order to effect the encumbrance or sale of these properties to
defraud her or the conjugal partnership of gains.

In its Order29 dated September 26, 2006, the trial court extended the aforequoted TPO for another
ten (10) days, and gave petitioner a period of five (5) days within which to show cause why the TPO
should not be renewed, extended, or modified. Upon petitioner's manifestation,30 however, that he
has not received a copy of private respondent's motion to modify/renew the TPO, the trial court
directed in its Order31 dated October 6, 2006 that petitioner be furnished a copy of said motion.
Nonetheless, an Order32 dated a day earlier, October 5, had already been issued renewing the TPO
dated August 23, 2006. The pertinent portion is quoted hereunder:

xxxx

x x x it appearing further that the hearing could not yet be finally terminated, the Temporary
Protection Order issued on August 23, 2006 is hereby renewed and extended for thirty (30) days and
continuously extended and renewed for thirty (30) days, after each expiration, until further orders,
and subject to such modifications as may be ordered by the court.

After having received a copy of the foregoing Order, petitioner no longer submitted the required
comment to private respondent's motion for renewal of the TPO arguing that it would only be an
"exercise in futility."33

Proceedings before the CA

During the pendency of Civil Case No. 06-797, petitioner filed before the Court of Appeals (CA) a
petition34 for prohibition (CA-G.R. CEB-SP. No. 01698), with prayer for injunction and temporary
restraining order, challenging (1) the constitutionality of R.A. 9262 for being violative of the due
process and the equal protection clauses, and (2) the validity of the modified TPO issued in the civil
case for being "an unwanted product of an invalid law."

On May 26, 2006, the appellate court issued a 60-day Temporary Restraining Order36 (TRO) against
the enforcement of the TPO, the amended TPOs and other orders pursuant thereto.

Subsequently, however, on January 24, 2007, the appellate court dismissed36 the petition for failure
of petitioner to raise the constitutional issue in his pleadings before the trial court in the civil case,
which is clothed with jurisdiction to resolve the same. Secondly, the challenge to the validity

of R.A. 9262 through a petition for prohibition seeking to annul the protection orders issued by the
trial court constituted a collateral attack on said law.

His motion for reconsideration of the foregoing Decision having been denied in the
Resolution37 dated August 14, 2007, petitioner is now before us alleging that –

The Issues

I.

THE COURT OF APPEALS ERRED IN DISMISSING THE PETITION ON THE THEORY THAT THE
ISSUE OF CONSTITUTIONALITY WAS NOT RAISED AT THE EARLIEST OPPORTUNITY AND
THAT, THE PETITION CONSTITUTES A COLLATERAL ATTACK ON THE VALIDITY OF THE
LAW.

II.

THE COURT OF APPEALS COMMITTED SERIOUS ERROR IN FAILING TO CONCLUDE THAT


R.A. 9262 IS DISCRIMINATORY, UNJUST, AND VIOLATIVE OF THE EQUAL PROTECTION
CLAUSE.

III.

THE COURT OF APPEALS COMMITTED GRAVE MISTAKE IN NOT FINDING THAT R.A. 9262
RUNS COUNTER TO THE DUE PROCESS CLAUSE OF THE CONSTITUTION.

IV.

THE COURT OF APPEALS ERRED IN NOT FINDING THAT THE LAW DOES VIOLENCE TO THE
POLICY OF THE STATE TO PROTECT THE FAMILY AS A BASIC SOCIAL INSTITUTION.

V.

THE COURT OF APPEALS SERIOUSLY ERRED IN NOT DECLARING R.A. No. 9262 AS INVALID
AND UNCONSTITUTIONAL BECAUSE IT ALLOWS AN UNDUE DELEGATION OF JUDICIAL
POWER TO THE BARANGAY OFFICIALS.38

The Ruling of the Court

Before delving into the arguments propounded by petitioner against the constitutionality of R.A.
9262, we shall first tackle the propriety of the dismissal by the appellate court of the petition for
prohibition (CA-G.R. CEB-SP. No. 01698) filed by petitioner.

As a general rule, the question of constitutionality must be raised at the earliest opportunity so that if
not raised in the pleadings, ordinarily it may not be raised in the trial, and if not raised in the trial
court, it will not be considered on appeal.39 Courts will not anticipate a question of constitutional law
in advance of the necessity of deciding it.40

In defending his failure to attack the constitutionality of R.A. 9262 before the RTC of Bacolod City,
petitioner argues that the Family Court has limited authority and jurisdiction that is "inadequate to
tackle the complex issue of constitutionality."41

We disagree.

Family Courts have authority and jurisdiction to consider the constitutionality of a statute.

At the outset, it must be stressed that Family Courts are special courts, of the same level as
Regional Trial Courts. Under R.A. 8369, otherwise known as the "Family Courts Act of 1997," family
courts have exclusive original jurisdiction to hear and decide cases of domestic violence against
women and children.42 In accordance with said law, the Supreme Court designated from among the
branches of the Regional Trial Courts at least one Family Court in each of several key cities
identified.43 To achieve harmony with the first mentioned law, Section 7 of R.A. 9262 now provides
that Regional Trial Courts designated as Family Courts shall have original and exclusive jurisdiction
over cases of VAWC defined under the latter law, viz:

SEC. 7. Venue. – The Regional Trial Court designated as a Family Court shall have original and
exclusive jurisdiction over cases of violence against women and their children under this law. In the
absence of such court in the place where the offense was committed, the case shall be filed in the
Regional Trial Court where the crime or any of its elements was committed at the option of the
complainant. (Emphasis supplied)

Inspite of its designation as a family court, the RTC of Bacolod City remains possessed of authority
as a court of general original jurisdiction to pass upon all kinds of cases whether civil, criminal,
special proceedings, land registration, guardianship, naturalization, admiralty or insolvency. 44 It is
settled that RTCs have jurisdiction to resolve the constitutionality of a statute, 45 "this authority being
embraced in the general definition of the judicial power to determine what are the valid and binding
laws by the criterion of their conformity to the fundamental law."46The Constitution vests the power of
judicial review or the power to declare the constitutionality or validity of a law, treaty, international or
executive agreement, presidential decree, order, instruction, ordinance, or regulation not only in this
Court, but in all RTCs.47 We said in J.M. Tuason and Co., Inc. v. CA48 that, "plainly the Constitution
contemplates that the inferior courts should have jurisdiction in cases involving constitutionality of
any treaty or law, for it speaks of appellate review of final judgments of inferior courts in cases where
such constitutionality happens to be in issue." Section 5, Article VIII of the 1987 Constitution reads in
part as follows:

SEC. 5. The Supreme Court shall have the following powers:

xxx

2. Review, revise, reverse, modify, or affirm on appeal or certiorari, as the law or the Rules of Court
may provide, final judgments and orders of lower courts in:

a. All cases in which the constitutionality or validity of any treaty, international or executive
agreement, law, presidential decree, proclamation, order, instruction, ordinance, or regulation is in
question.

xxxx

Thus, contrary to the posturing of petitioner, the issue of constitutionality of R.A. 9262 could have
been raised at the earliest opportunity in his Opposition to the petition for protection order before the
RTC of Bacolod City, which had jurisdiction to determine the same, subject to the review of this
Court.

Section 20 of A.M. No. 04-10-11-SC, the Rule on Violence Against Women and Their Children, lays
down a new kind of procedure requiring the respondent to file an opposition to the petition and not
an answer.49 Thus:

SEC. 20. Opposition to petition. – (a) The respondent may file an opposition to the petition which he
himself shall verify. It must be accompanied by the affidavits of witnesses and shall show cause why
a temporary or permanent protection order should not be issued.
(b) Respondent shall not include in the opposition any counterclaim, cross-claim or third-party
complaint, but any cause of action which could be the subject thereof may be litigated in a separate
civil action. (Emphasis supplied)

We cannot subscribe to the theory espoused by petitioner that, since a counterclaim, cross-claim
and third-party complaint are to be excluded from the opposition, the issue of constitutionality cannot
likewise be raised therein. A counterclaim is defined as any claim for money or other relief which a
defending party may have against an opposing party.50 A cross-claim, on the other hand, is any claim
by one party against a co-party arising out of the transaction or occurrence that is the subject matter
either of the original action or of a counterclaim therein. 51Finally, a third-party complaint is a claim
that a defending party may, with leave of court, file against a person not a party to the action for
contribution, indemnity, subrogation or any other relief, in respect of his opponent's claim. 52As
pointed out by Justice Teresita J. Leonardo-De Castro, the unconstitutionality of a statute is not a
cause of action that could be the subject of a counterclaim, cross-claim or a third-party complaint.
Therefore, it is not prohibited from being raised in the opposition in view of the familiar maxim
expressio unius est exclusio alterius.

Moreover, it cannot be denied that this issue affects the resolution of the case a quo because the
right of private respondent to a protection order is founded solely on the very statute the validity of
which is being attacked53 by petitioner who has sustained, or will sustain, direct injury as a result of
its enforcement. The alleged unconstitutionality of R.A. 9262 is, for all intents and purposes, a valid
cause for the non-issuance of a protection order.

That the proceedings in Civil Case No. 06-797 are summary in nature should not have deterred
petitioner from raising the same in his Opposition. The question relative to the constitutionality of a
statute is one of law which does not need to be supported by evidence. 54 Be that as it may, Section
25 of A.M. No. 04-10-11-SC nonetheless allows the conduct of a hearing to determine legal issues,
among others, viz:

SEC. 25. Order for further hearing. - In case the court determines the need for further hearing, it may
issue an order containing the following:

(a) Facts undisputed and admitted;

(b) Factual and legal issues to be resolved;

(c) Evidence, including objects and documents that have been marked and will be
presented;

(d) Names of witnesses who will be ordered to present their direct testimonies in the form of
affidavits; and

(e) Schedule of the presentation of evidence by both parties which shall be done in one day,
to the extent possible, within the 30-day period of the effectivity of the temporary protection
order issued. (Emphasis supplied)

To obviate potential dangers that may arise concomitant to the conduct of a hearing when
necessary, Section 26 (b) of A.M. No. 04-10-11-SC provides that if a temporary protection order
issued is due to expire, the trial court may extend or renew the said order for a period of thirty (30)
days each time until final judgment is rendered. It may likewise modify the extended or renewed
temporary protection order as may be necessary to meet the needs of the parties. With the private
respondent given ample protection, petitioner could proceed to litigate the constitutional issues,
without necessarily running afoul of the very purpose for the adoption of the rules on summary
procedure.

In view of all the foregoing, the appellate court correctly dismissed the petition for prohibition with
prayer for injunction and temporary restraining order (CA-G.R. CEB - SP. No. 01698). Petitioner may
have proceeded upon an honest belief that if he finds succor in a superior court, he could be granted
an injunctive relief. However, Section 22(j) of A.M. No. 04-10-11-SC expressly disallows the filing of
a petition for certiorari, mandamus or prohibition against any interlocutory order issued by the trial
court. Hence, the 60-day TRO issued by the appellate court in this case against the enforcement of
the TPO, the amended TPOs and other orders pursuant thereto was improper, and it effectively
hindered the case from taking its normal course in an expeditious and summary manner.

As the rules stand, a review of the case by appeal or certiorari before judgment is prohibited.
Moreover, if the appeal of a judgment granting permanent protection shall not stay its
enforcement,55 with more reason that a TPO, which is valid only for thirty (30) days at a time, 56 should
not be enjoined.

The mere fact that a statute is alleged to be unconstitutional or invalid, does not of itself entitle a
litigant to have the same enjoined. 57 In Younger v. Harris, Jr.,58 the Supreme Court of the United
States declared, thus:

Federal injunctions against state criminal statutes, either in their entirety or with respect to their
separate and distinct prohibitions, are not to be granted as a matter of course, even if such statutes
are unconstitutional. No citizen or member of the community is immune from prosecution, in good
faith, for his alleged criminal acts. The imminence of such a prosecution even though alleged to be
unauthorized and, hence, unlawful is not alone ground for relief in equity which exerts its
extraordinary powers only to prevent irreparable injury to the plaintiff who seeks its aid. (Citations
omitted)

The sole objective of injunctions is to preserve the status quo until the trial court hears fully the
merits of the case. It bears stressing, however, that protection orders are granted ex parte so as to
protect women and their children from acts of violence. To issue an injunction against such orders
will defeat the very purpose of the law against VAWC.

Notwithstanding all these procedural flaws, we shall not shirk from our obligation to determine novel
issues, or issues of first impression, with far-reaching implications. We have, time and again,
discharged our solemn duty as final arbiter of constitutional issues, and with more reason now, in
view of private respondent's plea in her Comment 59 to the instant Petition that we should put the
challenge to the constitutionality of R.A. 9262 to rest. And so we shall.

Intent of Congress in enacting R.A. 9262.

Petitioner claims that since R.A. 9262 is intended to prevent and criminalize spousal and child
abuse, which could very well be committed by either the husband or the wife, gender alone is not
enough basis to deprive the husband/father of the remedies under the law.60

A perusal of the deliberations of Congress on Senate Bill No. 2723, 61 which became R.A. 9262,
reveals that while the sponsor, Senator Luisa Pimentel-Ejercito (better known as Senator Loi
Estrada), had originally proposed what she called a "synthesized measure" 62 – an amalgamation of
two measures, namely, the "Anti-Domestic Violence Act" and the "Anti-Abuse of Women in Intimate
Relationships Act"63 – providing protection to "all family members, leaving no one in isolation" but at
the same time giving special attention to women as the "usual victims" of violence and
abuse,64 nonetheless, it was eventually agreed that men be denied protection under the same
measure. We quote pertinent portions of the deliberations:

Wednesday, December 10, 2003

Senator Pangilinan. I just wanted to place this on record, Mr. President. Some women's groups have
expressed concerns and relayed these concerns to me that if we are to include domestic violence
apart from against women as well as other members of the household, including children or the
husband, they fear that this would weaken the efforts to address domestic violence of which the
main victims or the bulk of the victims really are the wives, the spouses or the female partners in a
relationship. We would like to place that on record. How does the good Senator respond to this kind
of observation?

Senator Estrada. Yes, Mr. President, there is this group of women who call themselves "WIIR"
Women in Intimate Relationship. They do not want to include men in this domestic violence. But
plenty of men are also being abused by women. I am playing safe so I placed here members of the
family, prescribing penalties therefor and providing protective measures for victims. This includes the
men, children, live-in, common-law wives, and those related with the family.65

xxx

Wednesday, January 14, 2004

xxxx

The President Pro Tempore. x x x

Also, may the Chair remind the group that there was the discussion whether to limit this to women
and not to families which was the issue of the AWIR group. The understanding that I have is that we
would be having a broader scope rather than just women, if I remember correctly, Madam sponsor.

Senator Estrada. Yes, Mr. President.

As a matter of fact, that was brought up by Senator Pangilinan during the interpellation period.

I think Senator Sotto has something to say to that.

Senator Legarda. Mr. President, the reason I am in support of the measure. Do not get me wrong.
However, I believe that there is a need to protect women's rights especially in the domestic
environment.

As I said earlier, there are nameless, countless, voiceless women who have not had the opportunity
to file a case against their spouses, their live-in partners after years, if not decade, of battery and
abuse. If we broaden the scope to include even the men, assuming they can at all be abused by the
women or their spouses, then it would not equalize the already difficult situation for women, Mr.
President.

I think that the sponsor, based on our earlier conversations, concurs with this position. I am sure that
the men in this Chamber who love their women in their lives so dearly will agree with this
representation. Whether we like it or not, it is an unequal world. Whether we like it or not, no matter
how empowered the women are, we are not given equal opportunities especially in the domestic
environment where the macho Filipino man would always feel that he is stronger, more superior to
the Filipino woman.

xxxx

The President Pro Tempore. What does the sponsor say?

Senator Estrada. Mr. President, before accepting this, the committee came up with this bill because
the family members have been included in this proposed measure since the other members of the
family other than women are also possible victims of violence. While women are most likely the
intended victims, one reason incidentally why the measure focuses on women, the fact remains that
in some relatively few cases, men also stand to be victimized and that children are almost always
the helpless victims of violence. I am worried that there may not be enough protection extended to
other family members particularly children who are excluded. Although Republic Act No. 7610, for
instance, more or less, addresses the special needs of abused children. The same law is
inadequate. Protection orders for one are not available in said law.

I am aware that some groups are apprehensive about granting the same protection to men, fearing
that they may use this law to justify their abusive behavior against women. However, we should also
recognize that there are established procedures and standards in our courts which give credence to
evidentiary support and cannot just arbitrarily and whimsically entertain baseless complaints.

Mr. President, this measure is intended to harmonize family relations and to protect the family as the
basic social institution. Though I recognize the unequal power relations between men and women in
our society, I believe we have an obligation to uphold inherent rights and dignity of both husband
and wife and their immediate family members, particularly children.

While I prefer to focus mainly on women, I was compelled to include other family members as a
critical input arrived at after a series of consultations/meetings with various NGOs, experts, sports
groups and other affected sectors, Mr. President.

Senator Sotto. Mr. President.

The President Pro Tempore. Yes, with the permission of the other senators.

Senator Sotto. Yes, with the permission of the two ladies on the Floor.

The President Pro Tempore. Yes, Sen. Vicente C. Sotto III is recognized.

Senator Sotto. I presume that the effect of the proposed amendment of Senator Legarda would be
removing the "men and children" in this particular bill and focus specifically on women alone. That
will be the net effect of that proposed amendment. Hearing the rationale mentioned by the
distinguished sponsor, Sen. Luisa "Loi" Ejercito Estrada, I am not sure now whether she is inclined
to accept the proposed amendment of Senator Legarda.

I am willing to wait whether she is accepting this or not because if she is going to accept this, I will
propose an amendment to the amendment rather than object to the amendment, Mr. President.

xxxx
Senator Estrada. The amendment is accepted, Mr. President.

The President Pro Tempore. Is there any objection?

xxxx

Senator Sotto. x x x May I propose an amendment to the amendment.

The President Pro Tempore. Before we act on the amendment?

Senator Sotto. Yes, Mr. President.

The President Pro Tempore. Yes, please proceed.

Senator Sotto. Mr. President, I am inclined to believe the rationale used by the distinguished
proponent of the amendment. As a matter of fact, I tend to agree. Kung may maaabuso, mas
malamang iyong babae kaysa sa lalake. At saka iyong mga lalake, puwede na talagang magulpi
iyan. Okey lang iyan. But I cannot agree that we remove the children from this particular measure.

So, if I may propose an amendment –

The President Pro Tempore. To the amendment.

Senator Sotto. – more than the women, the children are very much abused. As a matter of fact, it is
not limited to minors. The abuse is not limited to seven, six, 5-year-old children. I have seen 14, 15-
year-old children being abused by their fathers, even by their mothers. And it breaks my heart to find
out about these things.

Because of the inadequate existing law on abuse of children, this particular measure will update that.
It will enhance and hopefully prevent the abuse of children and not only women.

SOTTO-LEGARDA AMENDMENTS

Therefore, may I propose an amendment that, yes, we remove the aspect of the men in the bill but
not the children.

Senator Legarda. I agree, Mr. President, with the Minority Leader.

The President Pro Tempore. Effectively then, it will be women AND CHILDREN.

Senator Sotto. Yes, Mr. President.

Senator Estrada. It is accepted, Mr. President.

The President Pro Tempore. Is there any objection? [Silence] There being none, the amendment, as
amended, is approved.66

It is settled that courts are not concerned with the wisdom, justice, policy, or expediency of a
statute.67 Hence, we dare not venture into the real motivations and wisdom of the members of
Congress in limiting the protection against violence and abuse under R.A. 9262 to women and
children only. No proper challenge on said grounds may be entertained in this proceeding. Congress
has made its choice and it is not our prerogative to supplant this judgment. The choice may be
perceived as erroneous but even then, the remedy against it is to seek its amendment or repeal by
the legislative. By the principle of separation of powers, it is the legislative that determines the
necessity, adequacy, wisdom and expediency of any law.68 We only step in when there is a violation
of the Constitution. However, none was sufficiently shown in this case.

R.A. 9262 does not violate the guaranty of equal protection of the laws.

Equal protection simply requires that all persons or things similarly situated should be treated alike,
both as to rights conferred and responsibilities imposed. The oft-repeated disquisition in the early
case of Victoriano v. Elizalde Rope Workers' Union69 is instructive:

The guaranty of equal protection of the laws is not a guaranty of equality in the application of the
laws upon all citizens of the state. It is not, therefore, a requirement, in order to avoid the
constitutional prohibition against inequality, that every man, woman and child should be affected
alike by a statute. Equality of operation of statutes does not mean indiscriminate operation on
persons merely as such, but on persons according to the circumstances surrounding them. It
guarantees equality, not identity of rights. The Constitution does not require that things which are
different in fact be treated in law as though they were the same. The equal protection clause does
not forbid discrimination as to things that are different. It does not prohibit legislation which is limited
either in the object to which it is directed or by the territory within which it is to operate.

The equal protection of the laws clause of the Constitution allows classification. Classification in law,
as in the other departments of knowledge or practice, is the grouping of things in speculation or
practice because they agree with one another in certain particulars. A law is not invalid because of
simple inequality. The very idea of classification is that of inequality, so that it goes without saying
that the mere fact of inequality in no manner determines the matter of constitutionality. All that is
required of a valid classification is that it be reasonable, which means that the classification should
be based on substantial distinctions which make for real differences; that it must be germane to the
purpose of the law; that it must not be limited to existing conditions only; and that it must apply
equally to each member of the class. This Court has held that the standard is satisfied if the
classification or distinction is based on a reasonable foundation or rational basis and is not palpably
arbitrary. (Emphasis supplied)

Measured against the foregoing jurisprudential yardstick, we find that R.A. 9262 is based on a valid
classification as shall hereinafter be discussed and, as such, did not violate the equal protection
clause by favoring women over men as victims of violence and abuse to whom the State extends its
protection.

I. R.A. 9262 rests on substantial distinctions.

The unequal power relationship between women and men; the fact that women are more likely than
men to be victims of violence; and the widespread gender bias and prejudice against women all
make for real differences justifying the classification under the law. As Justice McIntyre succinctly
states, "the accommodation of differences ... is the essence of true equality."70

A. Unequal power relationship between men and women

According to the Philippine Commission on Women (the National Machinery for Gender Equality and
Women's Empowerment), violence against women (VAW) is deemed to be closely linked with the
unequal power relationship between women and men otherwise known as "gender-based violence".
Societal norms and traditions dictate people to think men are the leaders, pursuers, providers, and
take on dominant roles in society while women are nurturers, men's companions and supporters,
and take on subordinate roles in society. This perception leads to men gaining more power over
women. With power comes the need to control to retain that power. And VAW is a form of men's
expression of controlling women to retain power.71

The United Nations, which has long recognized VAW as a human rights issue, passed its Resolution
48/104 on the Declaration on Elimination of Violence Against Women on December 20, 1993 stating
that "violence against women is a manifestation of historically unequal power relations between men
and women, which have led to domination over and discrimination against women by men and to the
prevention of the full advancement of women, and that violence against women is one of the crucial
social mechanisms by which women are forced into subordinate positions, compared with men."72

Then Chief Justice Reynato S. Puno traced the historical and social context of gender-based
violence and developments in advocacies to eradicate VAW, in his remarks delivered during the
Joint Launching of R.A. 9262 and its Implementing Rules last October 27, 2004, the pertinent
portions of which are quoted hereunder:

History reveals that most societies sanctioned the use of violence against women. The patriarch of a
family was accorded the right to use force on members of the family under his control. I quote the
early studies:

Traditions subordinating women have a long history rooted in patriarchy – the institutional rule of
men. Women were seen in virtually all societies to be naturally inferior both physically and
intellectually. In ancient Western societies, women whether slave, concubine or wife, were under the
authority of men. In law, they were treated as property.

The Roman concept of patria potestas allowed the husband to beat, or even kill, his wife if she
endangered his property right over her. Judaism, Christianity and other religions oriented towards
the patriarchal family strengthened the male dominated structure of society.

English feudal law reinforced the tradition of male control over women. Even the eminent Blackstone
has been quoted in his commentaries as saying husband and wife were one and that one was the
husband. However, in the late 1500s and through the entire 1600s, English common law began to
limit the right of husbands to chastise their wives. Thus, common law developed the rule of thumb,
which allowed husbands to beat their wives with a rod or stick no thicker than their thumb.

In the later part of the 19th century, legal recognition of these rights to chastise wives or inflict
corporeal punishment ceased. Even then, the preservation of the family was given more importance
than preventing violence to women.

The metamorphosis of the law on violence in the United States followed that of the English common
law. In 1871, the Supreme Court of Alabama became the first appellate court to strike down the
common law right of a husband to beat his wife:

The privilege, ancient though it may be, to beat one's wife with a stick, to pull her hair, choke her,
spit in her face or kick her about the floor, or to inflict upon her like indignities, is not now
acknowledged by our law... In person, the wife is entitled to the same protection of the law that the
husband can invoke for himself.
As time marched on, the women's advocacy movement became more organized. The temperance
leagues initiated it. These leagues had a simple focus. They considered the evils of alcoholism as
the root cause of wife abuse. Hence, they demonstrated and picketed saloons, bars and their
husbands' other watering holes. Soon, however, their crusade was joined by suffragette movements,
expanding the liberation movement's agenda. They fought for women's right to vote, to own
property, and more. Since then, the feminist movement was on the roll.

The feminist movement exposed the private invisibility of the domestic violence to the public gaze.
They succeeded in transforming the issue into an important public concern. No less than the United
States Supreme Court, in 1992 case Planned Parenthood v. Casey, noted:

In an average 12-month period in this country, approximately two million women are the victims of
severe assaults by their male partners. In a 1985 survey, women reported that nearly one of every
eight husbands had assaulted their wives during the past year. The [American Medical Association]
views these figures as "marked underestimates," because the nature of these incidents discourages
women from reporting them, and because surveys typically exclude the very poor, those who do not
speak English well, and women who are homeless or in institutions or hospitals when the survey is
conducted. According to the AMA, "researchers on family violence agree that the true incidence of
partner violence is probably double the above estimates; or four million severely assaulted women
per year."

Studies on prevalence suggest that from one-fifth to one-third of all women will be physically
assaulted by a partner or ex-partner during their lifetime... Thus on an average day in the United
States, nearly 11,000 women are severely assaulted by their male partners. Many of these incidents
involve sexual assault... In families where wife beating takes place, moreover, child abuse is often
present as well.

Other studies fill in the rest of this troubling picture. Physical violence is only the most visible form of
abuse. Psychological abuse, particularly forced social and economic isolation of women, is also
common.

Many victims of domestic violence remain with their abusers, perhaps because they perceive no
superior alternative...Many abused women who find temporary refuge in shelters return to their
husbands, in large part because they have no other source of income... Returning to one's abuser
can be dangerous. Recent Federal Bureau of Investigation statistics disclose that 8.8 percent of all
homicide victims in the United States are killed by their spouses...Thirty percent of female homicide
victims are killed by their male partners.

Finally in 1994, the United States Congress enacted the Violence Against Women Act.

In the International front, the women's struggle for equality was no less successful. The United
States Charter and the Universal Declaration of Human Rights affirmed the equality of all human
beings. In 1979, the UN General Assembly adopted the landmark Convention on the Elimination of
all Forms of Discrimination Against Women (CEDAW). In 1993, the UN General Assembly also
adopted the Declaration on the Elimination of Violence Against Women. World conferences on the
role and rights of women have been regularly held in Mexico City, Copenhagen, Nairobi and Beijing.
The UN itself established a Commission on the Status of Women.

The Philippines has been in cadence with the half – and full – steps of all these women's
movements. No less than Section 14, Article II of our 1987 Constitution mandates the State to
recognize the role of women in nation building and to ensure the fundamental equality before the law
of women and men. Our Senate has ratified the CEDAW as well as the Convention on the Rights of
the Child and its two protocols. To cap it all, Congress, on March 8, 2004, enacted Rep. Act No.
9262, entitled "An Act Defining Violence Against Women and Their Children, Providing for Protective
Measures for Victims, Prescribing Penalties therefor and for other Purposes." (Citations omitted)

B. Women are the "usual" and "most likely"

victims of violence.

At the time of the presentation of Senate Bill No. 2723, official statistics on violence against women
and children show that –

x x x physical injuries had the highest number of cases at 5,058 in 2002 representing 55.63% of total
cases reported (9,903). And for the first semester of 2003, there were 2,381 reported cases out of
4,354 cases which represent 54.31%. xxx (T)he total number of women in especially difficult
circumstances served by the Department of Social Welfare and Development (DSWD) for the year
2002, there are 1,417 physically abused/maltreated cases out of the total of 5,608 cases. xxx
(T)here are 1,091 DSWD cases out of a total number of 3,471 cases for the first semester of 2003.
Female violence comprised more than 90% of all forms of abuse and violence and more than 90% of
these reported cases were committed by the women's intimate partners such as their husbands and
live-in partners.73

Recently, the Philippine Commission on Women presented comparative statistics on violence


against women across an eight-year period from 2004 to August of 2011 with violations under R.A.
9262 ranking first among the different VAW categories since its implementation in 2004,74 thus:

Table 1. Annual Comparative Statistics on Violence Against Women, 2004 - 2011*

Reported 200 200 200 200 200 200


2010 2011
Cases 4 5 6 7 8 9

Rape 997 927 659 837 811 770 1,042 832

Incestuous
38 46 26 22 28 27 19 23
Rape

Attempted
194 148 185 147 204 167 268 201
Rape

Acts of
Lasciviousn 580 536 382 358 445 485 745 625
ess

Physical 3,55 2,33 1,89 1,50 1,30 1,49


2,018 1,588
Injuries 3 5 2 5 7 8
Sexual
53 37 38 46 18 54 83 63
Harassment

1,26 2,38 3,59 5,28


RA 9262 218 924 9,974 9,021
9 7 9 5

Threats 319 223 199 182 220 208 374 213

Seduction 62 19 29 30 19 19 25 15

Concubinag
121 102 93 109 109 99 158 128
e

RA 9208 17 11 16 24 34 152 190 62

Abduction
/Kidnapping 16 34 23 28 18 25 22
29

Unjust
90 50 59 59 83 703 183 155
Vexation

6,27 5,37 4,88 5,72 6,90 9,48 15,10 12,94


Total
1 4 1 9 5 5 4 8

*2011 report covers only from January to August

Source: Philippine National Police – Women and Children Protection Center (WCPC)

On the other hand, no reliable estimates may be obtained on domestic abuse and violence against
men in the Philippines because incidents thereof are relatively low and, perhaps, because many
men will not even attempt to report the situation. In the United Kingdom, 32% of women who had
ever experienced domestic violence did so four or five (or more) times, compared with 11% of the
smaller number of men who had ever experienced domestic violence; and women constituted 89%
of all those who had experienced 4 or more incidents of domestic violence. 75Statistics in Canada
show that spousal violence by a woman against a man is less likely to cause injury than the other
way around (18 percent versus 44 percent). Men, who experience violence from their spouses are
much less likely to live in fear of violence at the hands of their spouses, and much less likely to
experience sexual assault. In fact, many cases of physical violence by a woman against a spouse
are in self-defense or the result of many years of physical or emotional abuse.76

While there are, indeed, relatively few cases of violence and abuse perpetrated against men in the
Philippines, the same cannot render R.A. 9262 invalid.
In a 1960 case involving the violation of a city ordinance requiring drivers of animal-drawn vehicles
to pick up, gather and deposit in receptacles the manure emitted or discharged by their vehicle-
drawing animals in any public highways, streets, plazas, parks or alleys, said ordinance was
challenged as violative of the guaranty of equal protection of laws as its application is limited to
owners and drivers of vehicle-drawing animals and not to those animals, although not utilized, but
similarly pass through the same streets.

The ordinance was upheld as a valid classification for the reason that, while there may be non-
vehicle-drawing animals that also traverse the city roads, "but their number must be negligible and
their appearance therein merely occasional, compared to the rig-drawing ones, as not to constitute a
menace to the health of the community."77 The mere fact that the legislative classification may result
in actual inequality is not violative of the right to equal protection, for every classification of persons
or things for regulation by law produces inequality in some degree, but the law is not thereby
rendered invalid.78

C. Gender bias and prejudices

From the initial report to the police through prosecution, trial, and sentencing, crimes against women
are often treated differently and less seriously than other crimes. This was argued by then United
States Senator Joseph R. Biden, Jr., now Vice President, chief sponsor of the Violence Against
Women Act (VAWA), in defending the civil rights remedy as a valid exercise of the U.S. Congress'
authority under the Commerce and Equal Protection Clauses. He stressed that the widespread
gender bias in the U.S. has institutionalized historic prejudices against victims of rape or domestic
violence, subjecting them to "double victimization" – first at the hands of the offender and then of the
legal system.79

Our own Senator Loi Estrada lamented in her Sponsorship Speech for Senate Bill No. 2723 that
"(w)henever violence occurs in the family, the police treat it as a private matter and advise the
parties to settle the conflict themselves. Once the complainant brings the case to the prosecutor, the
latter is hesitant to file the complaint for fear that it might later be withdrawn. This lack of response or
reluctance to be involved by the police and prosecution reinforces the escalating, recurring and often
serious nature of domestic violence."80

Sadly, our own courts, as well, have exhibited prejudices and biases against our women.

In a recent case resolved on March 9, 2011, we fined RTC Judge Venancio J. Amila for Conduct
Unbecoming of a Judge. He used derogatory and irreverent language in reference to the
complainant in a petition for TPO and PPO under R.A. 9262, calling her as "only a live-in partner"
and presenting her as an "opportunist" and a "mistress" in an "illegitimate relationship." Judge Amila
even called her a "prostitute," and accused her of being motivated by "insatiable greed" and of
absconding with the contested property.81 Such remarks betrayed Judge Amila's prejudices and lack
of gender sensitivity.

The enactment of R.A. 9262 aims to address the discrimination brought about by biases and
prejudices against women. As emphasized by the CEDAW Committee on the Elimination of
Discrimination against Women, addressing or correcting discrimination through specific measures
focused on women does not discriminate against men.82Petitioner's contention,83 therefore, that R.A.
9262 is discriminatory and that it is an "anti-male," "husband-bashing," and "hate-men" law deserves
scant consideration. As a State Party to the CEDAW, the Philippines bound itself to take all
appropriate measures "to modify the social and cultural patterns of conduct of men and women, with
a view to achieving the elimination of prejudices and customary and all other practices which are
based on the idea of the inferiority or the superiority of either of the sexes or on stereotyped roles for
men and women."84 Justice Puno correctly pointed out that "(t)he paradigm shift changing the
character of domestic violence from a private affair to a public offense will require the development
of a distinct mindset on the part of the police, the prosecution and the judges."85

II. The classification is germane to the purpose of the law.

The distinction between men and women is germane to the purpose of R.A. 9262, which is to
address violence committed against women and children, spelled out in its Declaration of Policy, as
follows:

SEC. 2. Declaration of Policy. – It is hereby declared that the State values the dignity of women and
children and guarantees full respect for human rights. The State also recognizes the need to protect
the family and its members particularly women and children, from violence and threats to their
personal safety and security.

Towards this end, the State shall exert efforts to address violence committed against women and
children in keeping with the fundamental freedoms guaranteed under the Constitution and the
provisions of the Universal Declaration of Human Rights, the Convention on the Elimination of All
Forms of Discrimination Against Women, Convention on the Rights of the Child and other
international human rights instruments of which the Philippines is a party.

In 1979, the U.N. General Assembly adopted the CEDAW, which the Philippines ratified on August
5, 1981. Subsequently, the Optional Protocol to the CEDAW was also ratified by the Philippines on
October 6, 2003.86 This Convention mandates that State parties shall accord to women equality with
men before the law87 and shall take all appropriate measures to eliminate discrimination against
women in all matters relating to marriage and family relations on the basis of equality of men and
women.88 The Philippines likewise ratified the Convention on the Rights of the Child and its two
protocols.89 It is, thus, bound by said Conventions and their respective protocols.

III. The classification is not limited to existing

conditions only, and apply equally to all members

Moreover, the application of R.A. 9262 is not limited to the existing conditions when it was
promulgated, but to future conditions as well, for as long as the safety and security of women and
their children are threatened by violence and abuse.

R.A. 9262 applies equally to all women and children who suffer violence and abuse. Section 3
thereof defines VAWC as:

x x x any act or a series of acts committed by any person against a woman who is his wife, former
wife, or against a woman with whom the person has or had a sexual or dating relationship, or with
whom he has a common child, or against her child whether legitimate or illegitimate, within or without
the family abode, which result in or is likely to result in physical, sexual, psychological harm or
suffering, or economic abuse including threats of such acts, battery, assault, coercion, harassment
or arbitrary deprivation of liberty. It includes, but is not limited to, the following acts:

A. "Physical Violence" refers to acts that include bodily or physical harm;

B. "Sexual violence" refers to an act which is sexual in nature, committed against a woman or her
child. It includes, but is not limited to:
a) rape, sexual harassment, acts of lasciviousness, treating a woman or her child as
a sex object, making demeaning and sexually suggestive remarks, physically
attacking the sexual parts of the victim's body, forcing her/him to watch obscene
publications and indecent shows or forcing the woman or her child to do indecent
acts and/or make films thereof, forcing the wife and mistress/lover to live in the
conjugal home or sleep together in the same room with the abuser;

b) acts causing or attempting to cause the victim to engage in any sexual activity by
force, threat of force, physical or other harm or threat of physical or other harm or
coercion;

c) Prostituting the woman or child.

C. "Psychological violence" refers to acts or omissions causing or likely to cause mental or emotional
suffering of the victim such as but not limited to intimidation, harassment, stalking, damage to
property, public ridicule or humiliation, repeated verbal abuse and marital infidelity. It includes
causing or allowing the victim to witness the physical, sexual or psychological abuse of a member of
the family to which the victim belongs, or to witness pornography in any form or to witness abusive
injury to pets or to unlawful or unwanted deprivation of the right to custody and/or visitation of
common children.

D. "Economic abuse" refers to acts that make or attempt to make a woman financially dependent
which includes, but is not limited to the following:

1. withdrawal of financial support or preventing the victim from engaging in any


legitimate profession, occupation, business or activity, except in cases wherein the
other spouse/partner objects on valid, serious and moral grounds as defined in
Article 73 of the Family Code;

2. deprivation or threat of deprivation of financial resources and the right to the use
and enjoyment of the conjugal, community or property owned in common;

3. destroying household property;

4. controlling the victims' own money or properties or solely controlling the conjugal
money or properties.

It should be stressed that the acts enumerated in the aforequoted provision are attributable to
research that has exposed the dimensions and dynamics of battery. The acts described here are
also found in the U.N. Declaration on the Elimination of Violence Against Women. 90 Hence, the
argument advanced by petitioner that the definition of what constitutes abuse removes the difference
between violent action and simple marital tiffs is tenuous.

There is nothing in the definition of VAWC that is vague and ambiguous that will confuse petitioner in
his defense. The acts enumerated above are easily understood and provide adequate contrast
between the innocent and the prohibited acts. They are worded with sufficient definiteness that
persons of ordinary intelligence can understand what conduct is prohibited, and need not guess at
its meaning nor differ in its application. 91 Yet, petitioner insists92that phrases like "depriving or
threatening to deprive the woman or her child of a legal right," "solely controlling the conjugal or
common money or properties," "marital infidelity," and "causing mental or emotional anguish" are so
vague that they make every quarrel a case of spousal abuse. However, we have stressed that the
"vagueness" doctrine merely requires a reasonable degree of certainty for the statute to be upheld –
not absolute precision or mathematical exactitude, as petitioner seems to suggest. Flexibility, rather
than meticulous specificity, is permissible as long as the metes and bounds of the statute are clearly
delineated. An act will not be held invalid merely because it might have been more explicit in its
wordings or detailed in its provisions.93

There is likewise no merit to the contention that R.A. 9262 singles out the husband or father as the
culprit. As defined above, VAWC may likewise be committed "against a woman with whom the
person has or had a sexual or dating relationship." Clearly, the use of the gender-neutral word
"person" who has or had a sexual or dating relationship with the woman encompasses even lesbian
relationships. Moreover, while the law provides that the offender be related or connected to the
victim by marriage, former marriage, or a sexual or dating relationship, it does not preclude the
application of the principle of conspiracy under the Revised Penal Code (RPC). Thus, in the case of
Go-Tan v. Spouses Tan,94 the parents-in-law of Sharica Mari L. Go-Tan, the victim, were held to be
proper respondents in the case filed by the latter upon the allegation that they and their son (Go-
Tan's husband) had community of design and purpose in tormenting her by giving her insufficient
financial support; harassing and pressuring her to be ejected from the family home; and in
repeatedly abusing her verbally, emotionally, mentally and physically.

R.A. 9262 is not violative of the


due process clause of the Constitution.

Petitioner bewails the disregard of R.A. 9262, specifically in the issuance of POs, of all protections
afforded by the due process clause of the Constitution. Says he: "On the basis of unsubstantiated
allegations, and practically no opportunity to respond, the husband is stripped of family, property,
guns, money, children, job, future employment and reputation, all in a matter of seconds, without an
inkling of what happened."95

A protection order is an order issued to prevent further acts of violence against women and their
children, their family or household members, and to grant other necessary reliefs. Its purpose is to
safeguard the offended parties from further harm, minimize any disruption in their daily life and
facilitate the opportunity and ability to regain control of their life.96

"The scope of reliefs in protection orders is broadened to ensure that the victim or offended party is
afforded all the remedies necessary to curtail access by a perpetrator to the victim. This serves to
safeguard the victim from greater risk of violence; to accord the victim and any designated family or
household member safety in the family residence, and to prevent the perpetrator from committing
acts that jeopardize the employment and support of the victim. It also enables the court to award
temporary custody of minor children to protect the children from violence, to prevent their abduction
by the perpetrator and to ensure their financial support."97

The rules require that petitions for protection order be in writing, signed and verified by the
petitioner98 thereby undertaking full responsibility, criminal or civil, for every allegation therein. Since
"time is of the essence in cases of VAWC if further violence is to be prevented," 99 the court is
authorized to issue ex parte a TPO after raffle but before notice and hearing when the life, limb or
property of the victim is in jeopardy and there is reasonable ground to believe that the order is
necessary to protect the victim from the immediate and imminent danger of VAWC or to prevent
such violence, which is about to recur.100

There need not be any fear that the judge may have no rational basis to issue an ex parte order. The
victim is required not only to verify the allegations in the petition, but also to attach her witnesses'
affidavits to the petition.101
The grant of a TPO ex parte cannot, therefore, be challenged as violative of the right to due process.
Just like a writ of preliminary attachment which is issued without notice and hearing because the
time in which the hearing will take could be enough to enable the defendant to abscond or dispose of
his property,102 in the same way, the victim of VAWC may already have suffered harrowing
experiences in the hands of her tormentor, and possibly even death, if notice and hearing were
required before such acts could be prevented. It is a constitutional commonplace that the ordinary
requirements of procedural due process must yield to the necessities of protecting vital public
interests,103among which is protection of women and children from violence and threats to their
personal safety and security.

It should be pointed out that when the TPO is issued ex parte, the court shall likewise order that
notice be immediately given to the respondent directing him to file an opposition within five (5) days
from service. Moreover, the court shall order that notice, copies of the petition and TPO be served
immediately on the respondent by the court sheriffs. The TPOs are initially effective for thirty (30)
days from service on the respondent.104

Where no TPO is issued ex parte, the court will nonetheless order the immediate issuance and
service of the notice upon the respondent requiring him to file an opposition to the petition within five
(5) days from service. The date of the preliminary conference and hearing on the merits shall
likewise be indicated on the notice.105

The opposition to the petition which the respondent himself shall verify, must be accompanied by the
affidavits of witnesses and shall show cause why a temporary or permanent protection order should
not be issued.106

It is clear from the foregoing rules that the respondent of a petition for protection order should be
apprised of the charges imputed to him and afforded an opportunity to present his side. Thus, the
fear of petitioner of being "stripped of family, property, guns, money, children, job, future employment
and reputation, all in a matter of seconds, without an inkling of what happened" is a mere product of
an overactive imagination. The essence of due process is to be found in the reasonable opportunity
to be heard and submit any evidence one may have in support of one's defense. "To be heard" does
not only mean verbal arguments in court; one may be heard also through pleadings. Where
opportunity to be heard, either through oral arguments or pleadings, is accorded, there is no denial
of procedural due process.107

It should be recalled that petitioner filed on April 26, 2006 an Opposition to the Urgent Ex-Parte
Motion for Renewal of the TPO that was granted only two days earlier on April 24, 2006. Likewise,
on May 23, 2006, petitioner filed a motion for the modification of the TPO to allow him visitation
rights to his children. Still, the trial court in its Order dated September 26, 2006, gave him five days
(5) within which to show cause why the TPO should not be renewed or extended. Yet, he chose not
to file the required comment arguing that it would just be an "exercise in futility," conveniently
forgetting that the renewal of the questioned TPO was only for a limited period (30 days) each time,
and that he could prevent the continued renewal of said order if he can show sufficient cause
therefor. Having failed to do so, petitioner may not now be heard to complain that he was denied due
process of law.

Petitioner next laments that the removal and exclusion of the respondent in the VAWC case from the
residence of the victim, regardless of ownership of the residence, is virtually a "blank check" issued
to the wife to claim any property as her conjugal home.108

The wording of the pertinent rule, however, does not by any stretch of the imagination suggest that
this is so. It states:
SEC. 11. Reliefs available to the offended party. -- The protection order shall include any, some or
all of the following reliefs:

xxxx

(c) Removing and excluding the respondent from the residence of the offended party, regardless of
ownership of the residence, either temporarily for the purpose of protecting the offended party, or
permanently where no property rights are violated. If the respondent must remove personal effects
from the residence, the court shall direct a law enforcement agent to accompany the respondent to
the residence, remain there until the respondent has gathered his things and escort him from the
residence;

xxxx

Indubitably, petitioner may be removed and excluded from private respondent's residence,
regardless of ownership, only temporarily for the purpose of protecting the latter. Such removal and
exclusion may be permanent only where no property rights are violated. How then can the private
respondent just claim any property and appropriate it for herself, as petitioner seems to suggest?

The non-referral of a VAWC case


to a mediator is justified.

Petitioner argues that "by criminalizing run-of-the-mill arguments, instead of encouraging mediation
and counseling, the law has done violence to the avowed policy of the State to "protect and
strengthen the family as a basic autonomous social institution."109

Under Section 23(c) of A.M. No. 04-10-11-SC, the court shall not refer the case or any issue thereof
to a mediator. The reason behind this provision is well-explained by the Commentary on Section 311
of the Model Code on Domestic and Family Violence as follows:110

This section prohibits a court from ordering or referring parties to mediation in a proceeding for an
order for protection. Mediation is a process by which parties in equivalent bargaining positions
voluntarily reach consensual agreement about the issue at hand. Violence, however, is not a subject
for compromise. A process which involves parties mediating the issue of violence implies that the
victim is somehow at fault. In addition, mediation of issues in a proceeding for an order of protection
is problematic because the petitioner is frequently unable to participate equally with the person
against whom the protection order has been sought. (Emphasis supplied)

There is no undue delegation of


judicial power to barangay officials.

Petitioner contends that protection orders involve the exercise of judicial power which, under the
Constitution, is placed upon the "Supreme Court and such other lower courts as may be established
by law" and, thus, protests the delegation of power to barangay officials to issue protection
orders.111 The pertinent provision reads, as follows:

SEC. 14. Barangay Protection Orders (BPOs); Who May Issue and How. – Barangay Protection
Orders (BPOs) refer to the protection order issued by the Punong Barangay ordering the perpetrator
to desist from committing acts under Section 5 (a) and (b) of this Act.  A Punong Barangay who
1âwphi1

receives applications for a BPO shall issue the protection order to the applicant on the date of filing
after ex parte determination of the basis of the application. If the Punong Barangay is unavailable to
act on the application for a BPO, the application shall be acted upon by any available Barangay
Kagawad. If the BPO is issued by a Barangay Kagawad, the order must be accompanied by an
attestation by the Barangay Kagawad that the Punong Barangay was unavailable at the time of the
issuance of the BPO. BPOs shall be effective for fifteen (15) days. Immediately after the issuance of
an ex parte BPO, the Punong Barangay or Barangay Kagawad shall personally serve a copy of the
same on the respondent, or direct any barangay official to effect its personal service.

The parties may be accompanied by a non-lawyer advocate in any proceeding before the Punong
Barangay.

Judicial power includes the duty of the courts of justice to settle actual controversies involving rights
which are legally demandable and enforceable, and to determine whether or not there has been a
grave abuse of discretion amounting to lack or excess of jurisdiction on the part of any branch or
instrumentality of the Government.112 On the other hand, executive power "is generally defined as the
power to enforce and administer the laws. It is the power of carrying the laws into practical operation
and enforcing their due observance."113

As clearly delimited by the aforequoted provision, the BPO issued by the Punong Barangay or, in his
unavailability, by any available Barangay Kagawad, merely orders the perpetrator to desist from (a)
causing physical harm to the woman or her child; and (2) threatening to cause the woman or her
child physical harm. Such function of the Punong Barangay is, thus, purely executive in nature, in
pursuance of his duty under the Local Government Code to "enforce all laws and ordinances," and
to "maintain public order in the barangay."114

We have held that "(t)he mere fact that an officer is required by law to inquire into the existence of
certain facts and to apply the law thereto in order to determine what his official conduct shall be and
the fact that these acts may affect private rights do not constitute an exercise of judicial powers."115

In the same manner as the public prosecutor ascertains through a preliminary inquiry or proceeding
"whether there is reasonable ground to believe that an offense has been committed and the accused
is probably guilty thereof," the Punong Barangay must determine reasonable ground to believe that
an imminent danger of violence against the woman and her children exists or is about to recur that
would necessitate the issuance of a BPO. The preliminary investigation conducted by the prosecutor
is, concededly, an executive, not a judicial, function. The same holds true with the issuance of a
BPO.

We need not even belabor the issue raised by petitioner that since barangay officials and other law
enforcement agencies are required to extend assistance to victims of violence and abuse, it would
be very unlikely that they would remain objective and impartial, and that the chances of acquittal are
nil. As already stated, assistance by barangay officials and other law enforcement agencies is
consistent with their duty to enforce the law and to maintain peace and order.

Conclusion

Before a statute or its provisions duly challenged are voided, an unequivocal breach of, or a clear
conflict with the Constitution, not merely a doubtful or argumentative one, must be demonstrated in
such a manner as to leave no doubt in the mind of the Court. In other words, the grounds for nullity
must be beyond reasonable doubt.116 In the instant case, however, no concrete evidence and
convincing arguments were presented by petitioner to warrant a declaration of the unconstitutionality
of R.A. 9262, which is an act of Congress and signed into law by the highest officer of the co-equal
executive department. As we said in Estrada v. Sandiganbayan, 117 courts must assume that the
legislature is ever conscious of the borders and edges of its plenary powers, and passed laws with
full knowledge of the facts and for the purpose of promoting what is right and advancing the welfare
of the majority.

We reiterate here Justice Puno's observation that "the history of the women's movement against
domestic violence shows that one of its most difficult struggles was the fight against the violence of
law itself. If we keep that in mind, law will not again be a hindrance to the struggle of women for
equality but will be its fulfillment."118 Accordingly, the constitutionality of R.A. 9262 is, as it should be,
sustained.

WHEREFORE, the instant petition for review on certiorari is hereby DENIED for lack of merit.

SO ORDERED.

G.R. Nos. 180849 and 187143               November 16, 2011

PHILIPPINE NATIONAL BANK, Petitioner, 


vs.
DAN PADAO, Respondent.

DECISION

MENDOZA, J.:

These are two consolidated petitions for review on certiorari under Rule 45 of the Rules of Court.

In G.R. No. 180849, petitioner Philippine National Bank (PNB) seeks the reversal of the December
14, 2006 Decision1 and October 2, 2007 Resolution2 of the Court of Appeals (CA) in CA-G.R. SP No.
76584, which upheld the ruling of the National Labor Relations Commission, Cagayan de Oro City
(NLRC) in its October 30, 2002 Resolution, 3 reversing the June 21, 2001 Decision4 of the Executive
Labor Arbiter (ELA) which found the dismissal of respondent Dan Padao (Padao) valid.

In G.R. No. 187143, PNB seeks the reversal of the December 9, 2008 Decision5 and February 24,
2009 Resolution6of the CA in CA-G.R. SP No. 00945, which allowed the execution of the October
30, 2002 NLRC Resolution.

THE FACTS

A. G.R. No. 180849

On August 21, 1981, Padao was hired by PNB as a clerk at its Dipolog City Branch. He was later
designated as a credit investigator in an acting capacity on November 9, 1993. On March 23, 1995,
he was appointed regular Credit Investigator III, and was ultimately promoted to the position of Loan
and Credit Officer IV.

Sometime in 1994, PNB became embroiled in a scandal involving "behest loans." A certain Sih Wat
Kai complained to the Provincial Office of the Commission on Audit (COA) of Zamboanga del Norte
that anomalous loans were being granted by its officers: Assistant Vice President (AVP) and Branch
Manager Aurelio De Guzman (AVP de Guzman), Assistant Department Manager and Cashier Olson
Sala (Sala), and Loans and Senior Credit Investigator Primitivo Virtudazo (Virtudazo).

The questionable loans were reportedly being extended to select bank clients, among them Joseph
Liong, Danilo Dangcalan, Jacinto Salac, Catherine Opulentisima, and Virgie Pango. The exposé
triggered the conduct of separate investigations by the COA and PNB’s Internal Audit Department
(IAD) from January to August 1995. Both investigations confirmed that the collateral provided in
numerous loan accommodations were grossly over-appraised. The credit standing of the loan
applicants was also fabricated, allowing them to obtain larger loan portfolios from PNB. These
borrowers eventually defaulted on the payment of their loans, causing PNB to suffer millions in
losses.

In August 1995, Credit Investigators Rolando Palomares (Palomares) and Cayo Dagpin (Dagpin)
were administratively charged with Dishonesty, Grave Misconduct, Gross Neglect of Duty, Conduct
Prejudicial to the Best Interest of the Service, and violation of Republic Act (R.A.) No. 3019 (Anti-
Graft and Corrupt Practices Act), in connection with an anomalous loan granted to the spouses,
Jaime and Allyn Lim (the Lims). These charges, however, were later ordered dropped by PNB, citing
its findings that Dagpin and Palomares signed the Inspection and Appraisal Report (IAR) and the
Credit Inspection Report (CIR) in support of the Lims’ loan application in good faith, and upon the
instruction of their superior officers. PNB also considered using Dagpin and Palomares as
prosecution witnesses against AVP de Guzman, Loan Division Chief Melindo Bidad (Bidad) and
Sala.

The following month, September 1995, administrative charges for Grave Misconduct, Gross Neglect
of Duty and Gross Violations of Bank Rules and Regulations and criminal cases for violation of R.A.
No. 3019 were filed against AVP de Guzman, Sala, Virtudazo, and Bidad. Consequently, they were
all dismissed from the service by PNB in November 1996. Later, Virtudazo was ordered reinstated.

On June 14, 1996, Padao and Division Chief Wilma Velasco (Velasco) were similarly
administratively charged with Dishonesty, Grave Misconduct, Gross Neglect of Duty, Conduct
Prejudicial to the Best Interest of the Service, and violation of R.A. No. 3019.

The case against Padao was grounded on his having allegedly presented a deceptively positive
status of the business, credit standing/rating and financial capability of loan applicants Reynaldo and
Luzvilla Baluma and eleven (11) others. It was later found that either said borrowers’ businesses
were inadequate to meet their loan obligations, or that the projects they sought to be financed did
not exist.

Padao was also accused of having over-appraised the collateral of the spouses Gardito and Alma
Ajero, the spouses Ibaba, and Rolly Pango.

On January 10, 1997, after due investigation, PNB found Padao guilty of gross and habitual neglect
of duty and ordered him dismissed from the bank. Padao appealed to the bank’s Board of Directors.
On January 20, 1997, Velasco was also held guilty of the offenses charged against her, and was
similarly meted the penalty of dismissal. Her motion for reconsideration, however, was later granted
by the bank, and she was reinstated.

On October 11, 1999, after almost three (3) years of inaction on the part of the Board, Padao
instituted a complaint7against PNB and its then AVP, Napoleon Matienzo (Matienzo), with the Labor
Arbitration Branch of the NLRC Regional Arbitration Branch (RAB) No. IX in Zamboanga City for 1]
Reinstatement; 2] Backwages; 3] Illegal Dismissal; and 4] Treachery/Bad Faith and Palpable
Discrimination in the Treatment of Employees with administrative cases. The case was docketed as
RAB 09-04-00098-01.

In a Decision dated June 21, 2001, the ELA found Padao’s dismissal valid. Despite the finding of
legality, the ELA still awarded separation pay of one-half (1/2) month’s pay for every year of service,
citing PLDT v. NLRC & Abucay. 8The ELA held that in view of the peculiar conditions attendant to
Padao’s dismissal, there being no clear conclusive showing of moral turpitude, Padao should not be
left without any remedy.

Padao appealed to the NLRC, which, in its Resolution 9 dated October 30, 2002, reversed and set
aside the ELA Decision and declared Padao’s dismissal to be illegal. He was thereby ordered
reinstated to his previous position without loss of seniority rights and PNB was ordered to pay him
full backwages and attorney’s fees equivalent to ten percent (10%) of the total monetary award.

PNB’s Motion for Reconsideration 10 was denied by the NLRC in its Resolution 11 dated December 27,
2002.

Aggrieved, PNB filed a petition for certiorari12 with the CA but it was dismissed in a Decision13 dated
December 14, 2006. PNB moved for reconsideration 14 but the motion was denied in the CA
Resolution15 dated October 2, 2007.

B. G.R. No. 187143

During the pendency of G.R. No. 180849 before the Court, the NLRC issued an entry of judgment on
September 22, 2003, certifying that on February 28, 2003, its October 30, 2002 Resolution had
become final and executory.16

On December 5, 2003, Padao filed a Motion for Execution of the NLRC Resolution dated October
30, 2002. This was granted by the ELA on April 22, 2004.

On May 4, 2004, PNB and AVP Matienzo sought reconsideration of the ELA’s Order based on the
following grounds: (1) the October 30, 2003 Resolution was inexistent and, thus, could not become
final and executory; and (2) Padao’s motion for execution was granted without hearing.

Acting thereon, the ELA denied PNB’s motion for reconsideration on the ground that motions for
reconsideration of an order are prohibited under Section 19, Rule V of the NLRC Rules of
Procedure.

Thus, Padao filed his Motion to Admit Computation17 dated July 14, 2004. In its Comment,18 PNB
alleged that the computation was grossly exaggerated and without basis, and prayed for a period of
thirty (30) days within which to submit its counter-computation since the same would come from its
head office in Pasay City.

On September 22, 2004, the ELA issued the Order19 granting Padao’s Motion to Admit Computation.
The order cited PNB’s failure to submit its counter-computation within the two extended periods
(totaling forty days), which the ELA construed as a waiver to submit the same. Thus, the ELA
ordered the issuance of a writ of execution for the payment of backwages due to Padao in the
amount of ₱ 2,589,236.21.

In a motion20 dated September 29, 2004, PNB sought reconsideration of the order with an attached
counter-computation. The ELA denied the same in its Order 21 dated October 20, 2004 on the ground
that the motions for reconsideration of orders and decisions of the Labor Arbiter are prohibited under
Section 19, Rule V of the NLRC Rules of Procedure. The ELA further stated that PNB had been
given more than ample opportunity to submit its own computation in this case, and the belatedly
submitted counter-computation of claims could not be considered. Thus, a writ of execution22 was
issued on October 21, 2004.

On November 11, 2004 and January 19, 2005, PNB filed its Motion to Quash Writ of Execution and
its Motion to Dissolve Alias Writ of Execution, respectively. Both were denied by the ELA in an
Order23 dated February 8, 2005.

On February 18, 2005, PNB filed a Notice of Appeal with Memorandum on Appeal 24 with the NLRC.
On September 20, 2005, however, the NLRC issued a Resolution25 dismissing the bank’s appeal.
PNB’s Motion for Reconsideration26 was also denied in the December 21, 2005 Resolution.27

Thus, on March 7, 2006, PNB filed a Petition for Certiorari 28 with the CA, assailing the findings of
ELA Plagata and the NLRC.

In a Decision29 dated December 9, 2008, the CA dismissed the petition, and later denied PNB’s
motion for reconsideration on February 24, 2009.

ISSUES

In G.R. No. 180849, PNB presents the following Assignment of Errors:30

A. THE COURT OF APPEALS ERRED IN NOT CONSIDERING THAT THE POSITION OF


A CREDIT INVESTIGATOR IS ONE IMBUED WITH [THE] TRUST AND CONFIDENCE OF
THE EMPLOYER.

B. THE COURT OF APPEALS ERRED IN TREATING THE ACT OF FALSIFYING THE


CREDIT AND APPRAISAL REPORTS AND THAT OF MERELY AFFIXING ONE’S
SIGNATURE IN A FALSE REPORT PREPARED BY ANOTHER AS ONE AND THE SAME
DEGREE OF MISCONDUCT WHICH WARRANTS THE SAME PENALTY.

In G.R. No. 187143, PNB presents the following Assignment of Errors:31

THE LABOR COURTS AND THE APPELLATE COURT ERRED WHEN THEY INVARIABLY
IGNORED PNB’S COUNTER-COMPUTATION AND MERELY RELIED ON RESPONDENT DAN
PADAO’S SELF-SERVING COMPUTATION OF HIS MONEY AWARD.

THE LABOR COURTS AND THE APPELLATE COURT ERRED WHEN THEY ACCEPTED THE
COMPUTATION OF RESPONDENT PADAO WITHOUT REQUIRING PROOF TO SUPPORT THE
SAME.

In G.R. No. 180849, PNB argues that the position of a credit investigator is one reposed with trust
and confidence, such that its holder may be validly dismissed based on loss of trust and confidence.
In disciplining employees, the employer has the right to exercise discretion in determining the
individual liability of each erring employee and in imposing a penalty commensurate with the degree
of participation of each. PNB further contends that the findings of the CA are not in accordance with
the evidence on record, thus, necessitating a review of the facts of the present case by this Court.32
On the other hand, Padao counters that local bank policies implemented by the highest-ranking
branch officials such as the assistant vice-president/branch manager, assistant manager/cashier,
chief of the loans division and legal counsel, are presumed to be sanctioned and approved by the
bank, and a subordinate employee should not be faulted for his reliance thereon. He argues that a
person who acts in obedience to an order issued by a superior for some lawful purpose cannot be
held liable. PNB is bound by the acts of its senior officers and he, like his fellow credit investigators,
having acted in good faith in affixing his signature on the reports based on the instruction, order and
directive of senior local bank officials, should not be held liable.33

Padao also claims that PNB cruelly betrayed him by charging and dismissing him after using him as
a prosecution witness to secure the conviction of the senior bank officials, that he was never part of
the conspiracy, and that he did not derive any benefit from the scheme.34

The Court’s Ruling

In the 1987 Constitution, provisions on social justice and the protection of labor underscore the
importance and economic significance of labor. Article II, Section 18 characterizes labor as a
"primary social economic force," and as such, the State is bound to "protect the rights of workers and
promote their welfare." Moreover, workers are "entitled to security of tenure, humane conditions of
work, and a living wage."35

The Labor Code declares as policy that the State shall afford protection to labor, promote full
employment, ensure equal work opportunities regardless of sex, race or creed, and regulate the
relations between workers and employers. The State shall assure the rights of workers to self-
organization, collective bargaining, security of tenure, and just and humane conditions of work.36

While it is an employer’s basic right to freely select or discharge its employees, if only as a measure
of self-protection against acts inimical to its interest, 37 the law sets the valid grounds for termination
as well as the proper procedure to be followed when terminating the services of an employee.38

Thus, in cases of regular employment, the employer is prohibited from terminating the services of an
employee except for a just or authorized cause.39 Such just causes for which an employer may
terminate an employee are enumerated in Article 282 of the Labor Code:

(a) Serious misconduct or willful disobedience by the employee of the lawful orders of his
employer or representative in connection with his work;

(b) Gross and habitual neglect by the employee of his duties;

(c) Fraud or willful breach by the employee of the trust reposed in him by his employer or
duly authorized representative;

(d) Commission of a crime or offense by the employee against the person of his employer or
any immediate family member of his family or his duly authorized representative; and

(e) Other causes analogous to the foregoing.

Further, due process requires that employers follow the procedure set by the Labor Code:

Art. 277. Miscellaneous provisions.


xxx

b. Subject to the constitutional right of workers to security of tenure and their right to be protected
against dismissal except for a just and authorized cause and without prejudice to the requirement of
notice under Article 283 of this Code, the employer shall furnish the worker whose employment is
sought to be terminated a written notice containing a statement of the causes for termination and
shall afford the latter ample opportunity to be heard and to defend himself with the assistance of his
representative if he so desires in accordance with company rules and regulations promulgated
pursuant to guidelines set by the Department of Labor and Employment. Any decision taken by the
employer shall be without prejudice to the right of the worker to contest the validity or legality of his
dismissal by filing a complaint with the regional branch of the National Labor Relations Commission.
The burden of proving that the termination was for a valid or authorized cause shall rest on the
employer. The Secretary of the Department of Labor and Employment may suspend the effects of
the termination pending resolution of the dispute in the event of a prima facie finding by the
appropriate official of the Department of Labor and Employment before whom such dispute is
pending that the termination may cause a serious labor dispute or is in implementation of a mass
lay-off. (As amended by Section 33, Republic Act No. 6715, March 21, 1989)

xxx

In this case, Padao was dismissed by PNB for gross and habitual neglect of duties under Article 282
(b) of the Labor Code.

Gross negligence connotes want of care in the performance of one’s duties, while habitual neglect
implies repeated failure to perform one’s duties for a period of time, depending on the
circumstances.40 Gross negligence has been defined as the want or absence of or failure to exercise
slight care or diligence, or the entire absence of care. It evinces a thoughtless disregard of
consequences without exerting any effort to avoid them.41

In the case at bench, Padao was accused of having presented a fraudulently positive evaluation of
the business, credit standing/rating and financial capability of Reynaldo and Luzvilla Baluma and
eleven other loan applicants.42Some businesses were eventually found not to exist at all, while in
other transactions, the financial status of the borrowers simply could not support the grant of loans in
the approved amounts.43 Moreover, Padao over-appraised the collateral of spouses Gardito and
Alma Ajero, and that of spouses Ihaba and Rolly Pango.44

The role that a credit investigator plays in the conduct of a bank’s business cannot be overestimated.
The amount of loans to be extended by a bank depends upon the report of the credit investigator on
the collateral being offered. If a loan is not fairly secured, the bank is at the mercy of the borrower
who may just opt to have the collateral foreclosed. If the scheme is repeated a hundredfold, it may
lead to the collapse of the bank. In the case of Sawadjaan v. Court of Appeals,45 the Court stressed
the crucial role that a credit investigator or an appraiser plays. Thus:

Petitioner himself admits that the position of appraiser/inspector is "one of the most serious [and]
sensitive job[s] in the banking operations." He should have been aware that accepting such a
designation, he is obliged to perform the task at hand by the exercise of more than ordinary
prudence. As appraiser/investigator, the petitioner was expected to conduct an ocular inspection of
the properties offered by CAMEC as collaterals and check the copies of the certificates of title
against those on file with the Registry of Deeds. Not only did he fail to conduct these routine checks,
but he also deliberately misrepresented in his appraisal report that after reviewing the documents
and conducting a site inspection, he found the CAMEC loan application to be in order. Despite the
number of pleadings he has filed, he has failed to offer an alternative explanation for his actions.
[Emphasis supplied]

In fact, banks are mandated to exercise more care and prudence in dealing with registered lands:

[B]anks are cautioned to exercise more care and prudence in dealing even with registered lands,
than private individuals, "for their business is one affected with public interest, keeping in trust
money belonging to their depositors, which they should guard against loss by not committing any act
of negligence which amounts to lack of good faith by which they would be denied the protective
mantle of the land registration statute Act 496, extended only to purchasers for value and in good
faith, as well as to mortgagees of the same character and description. It is for this reason that banks
before approving a loan send representatives to the premises of the land offered as collateral and
investigate who are the true owners thereof.46

Padao’s repeated failure to discharge his duties as a credit investigator of the bank amounted to
gross and habitual neglect of duties under Article 282 (b) of the Labor Code. He not only failed to
perform what he was employed to do, but also did so repetitively and habitually, causing millions of
pesos in damage to PNB. Thus, PNB acted within the bounds of the law by meting out the penalty of
dismissal, which it deemed appropriate given the circumstances.

The CA was correct in stating that when the violation of company policy or breach of company rules
and regulations is tolerated by management, it cannot serve as a basis for termination.47 Such ruling,
however, does not apply here. The principle only applies when the breach or violation is one which
neither amounts to nor involves fraud or illegal activities. In such a case, one cannot evade liability or
culpability based on obedience to the corporate chain of command.

Padao cited Llosa-Tan v. Silahis International Hotel,48 where the "violation" of corporate policy was
held not per se fraudulent or illegal. Moreover, the said "violation" was done in compliance with the
apparent lawful orders of the concerned employee’s superiors. Management-sanctioned deviations
in the said case did not amount to fraud or illegal activities. If anything, it merely represented flawed
policy implementation.

In sharp contrast, Padao, in affixing his signature on the fraudulent reports, attested to the
falsehoods contained therein. Moreover, by doing so, he repeatedly failed to perform his duties as a
credit investigator.

Further, even Article 11(6) of the Revised Penal Code requires that any person, who acts in
obedience to an order issued by a superior does so for some lawful purpose in order for such person
not to incur criminal liability. The succeeding article exempts from criminal liability any person who
acts under the compulsion of an irresistible force(Article 12, paragraph 6) or under the impulse of
an uncontrollable fear of an equal or greater injury (Article 12, paragraph 7).

Assuming solely for the sake of argument that these principles apply by analogy, even an extremely
liberal interpretation of these justifying or exempting circumstances will not allow Padao to escape
liability.

Also, had Padao wanted immunity in exchange for his testimony as a prosecution witness, he should
have demanded that there be a written agreement. Without it, his claim is self-serving and
unreliable.
That there is no proof that Padao derived any benefit from the scheme is immaterial.49 What is
crucial is that his gross and habitual negligence caused great damage to his employer. Padao was
aware that there was something irregular about the practices being implemented by his superiors,
but he went along with, became part of, and participated in the scheme.

It does not speak well for a person to apparently blindly follow his superiors, particularly when, with
the exercise of ordinary diligence, one would be able to determine that what he or she was being
ordered to do was highly irregular, if not illegal, and would, and did, work to the great disadvantage
of his or her employer.

PNB, as an employer, has the basic right to freely select and discharge employees (subject to the
Labor Code requirements on substantive and procedural due process), if only as a measure of self-
protection against acts inimical to its interests.50 It has the authority to impose what penalty it deems
sufficient or commensurate to an employee’s offense. Having satisfied the requirements of
procedural and substantive due process, it is thus left to the discretion of the employer to impose
such sanction as it sees befitting based on the circumstances.

Finally, Padao claims that he should be accorded the same treatment as his co-employees. 51 As the
ELA, however, correctly observed:

[A]s pointed out by the respondents, the case of the complainant was different, and his culpability,
much more than his aforementioned co-employees.  In the case of Palomares and Dagpin, they
1âwphi1

were involved in only one case of over-appraisal of collateral in the loan account of the spouses
Jaime Lim and Allyn Tan (Respondents’ Comments, p. 1), but in the case of complainant, his over-
appraisals involved three (3) loan accounts and amounting to ₱ 9,537,759.00 (Ibid.), not to mention
that he also submitted falsified Credit Investigation Reports for the loan accounts of seven (7) other
borrowers of PNB (Ibid., pp. 1-2).

xxx

The number of over-appraisals (3) and falsified credit investigation reports (7) or countersigned by
the complainant indicates habituality, or the propensity to do the same. The best that can be said of
his acts is the lack of moral strength to resist the repeated commission of illegal or prohibited acts in
loan transactions. He thus cannot interpose undue pressure or coercion exerted upon [him] by his
superiors, to absolve himself of liability for his signing or countersigning the aforementioned falsified
reports. It may have been allowable or justifiable for him to give in to one anomalous loan
transaction report, but definitely not for ten (10) loan accounts. It is axiomatic that obedience to one’s
superiors extends only to lawful orders, not to unlawful orders calling for unauthorized, prohibited or
immoral acts to be done.

In the case of Wilma Velasco, PNB did not pursue legal action and even discontinued the
administrative case filed against her because, according to PNB, she appeared to have been the
victim of the misrepresentations and falsifications of the credit investigation and appraisal reports of
the complainant upon which she had to reply in acting on loan applications filed with the PNB and for
which such reports were made. She was not obliged to conduct a separate or personal appraisal of
the properties offered as collaterals, or separate credit investigations of the borrowers of PNB.
These functions pertained to PNB inspectors/credit investigators, like the complainant.
Unfortunately, the latter was derelict in the performance of those duties, if he did not deliberately
misuse or abuse such duties.

As can be seen, therefore, the complainant and Wilma Velasco did not stand on the same footing
relative to their involvement or participation in the anomalous loan transactions earlier mentioned.
Therefore, PNB cannot be faulted for freeing her from liability and punishment, while dismissing the
complainant from service. [Emphases supplied]

Given the above ruling of the Court in G.R. No. 180849, the ruling of the CA in CA-G.R. SP No.
00945, an action stemming from the execution of the decision in said case, must perforce be
reversed.

However, Padao is not entitled to financial assistance. In Toyota Motor Phils. Corp. Workers
Association v. NLRC,52the Court reaffirmed the general rule that separation pay shall be allowed as a
measure of social justice only in those instances where the employee is validly dismissed for
causes other than serious misconduct, willful disobedience, gross and habitual neglect of
duty, fraud or willful breach of trust, commission of a crime against the employer or his
family, or those reflecting on his moral character. These five grounds are just causes for
dismissal as provided in Article 282 of the Labor Code.

In Central Philippine Bandag Retreaders, Inc. v. Diasnes,53 cited in Quiambao v. Manila Electric


Company,54 we discussed the parameters of awarding separation pay to dismissed employees as a
measure of financial assistance:

To reiterate our ruling in Toyota, labor adjudicatory officials and the CA must demur the award of
separation pay based on social justice when an employee’s dismissal is based on serious
misconduct or willful disobedience; gross and habitual neglect of duty; fraud or willfull breach of
trust; or commission of a crime against the person of the employer or his immediate family – grounds
under Art. 282 of the Labor Code that sanction dismissal of employees. They must be judicious and
circumspect in awarding separation pay or financial assistance as the constitutional policy to provide
full protection to labor is not meant to be an instrument to oppress the employers. The commitment
of the Court to the cause of labor should not embarrass us from sustaining the employers when they
are right, as here. In fine, we should be more cautions in awarding financial assistance to the
undeserving and those who are unworthy of the liberality of the law.55 [Emphasis original.
Underscoring supplied]

Clearly, given the Court’s findings, Padao is not entitled to financial assistance. 1avvphi1

WHEREFORE, the petitions in G.R. No. 180849 and G.R. No. 187143 are GRANTED. In G.R. No.
180849, the December 14, 2006 Decision and the October 2, 2007 Resolution of the Court of
Appeals in CA-G.R. SP No. 76584 are REVERSED and SET ASIDE.

In G.R. No. 187143, the December 9, 2008 Decision and the February 24, 2009 Resolution of the
Court of Appeals in CA-G.R. SP No. 00945 are REVERSED and SET ASIDE.

The June 21, 2001 Decision of the Executive Labor Arbiter is hereby ordered REINSTATED, with
the MODIFICATION that the award of financial assistance is DELETED.

SO ORDERED.

JULY 3, 2018

G.R. No. 199802


CONGRESSMAN HERMILANDO I. MANDANAS; MAYOR EFREN B. DIONA; MAYOR
ANTONINO A. AURELIO; KAGA WAD MARIOILAGAN;BARANGAY CHAIR PERLITO MANALO;
BARANGA Y CHAIR MEDEL MEDRANO;BARANGAY KAGA WAD CRIS RAMOS; BARANGA Y
KAGA WAD ELISA D. BALBAGO, and ATTY. JOSE MALVAR VILLEGAS, Petitioners 
vs.
EXECUTIVE SECRETARY PAQUITO N. OCHOA, JR.; SECRETARY CESAR PURISIMA,
Department of Finance; SECRETARY FLORENCIO H. ABAD, Department of Budget and
Management; COMMISSIONER KIM JACINTO-HENARES, Bureau of Internal Revenue; and
NATIONAL TREASURER ROBERTO TAN, Bureau of the Treasury, Respondents

G.R. No. 208488

HONORABLE ENRIQUE T. GARCIA, JR., in his personal and official capacity as


Representative of the 2ndDistrict of the Province of Bataan, Petitioner 
vs.
HONORABLE [PAQUITO) N. OCHOA, JR., Executive Secretary; HONORABLE CESAR V.
PURISIMA, Secretary, Department of Finance; HONORABLE FLORENCIO H. ABAD, Secretary,
Department of Budget and Management; HONORABLE KIM S. JACINTO-HENARES,
Commissioner, Bureau of Internal Revenue; and HONORABLE ROZZANO RUFINO B.
BIAZON, Commissioner, Bureau of Customs, Respondents

DECISION

BERSAMIN, J.:

The petitioners hereby challenge the manner in which the just share in the national taxes of the local
government units (LGUs) has been computed.

Antecedents

One of the key features of the 1987 Constitution is its push towards decentralization of government
and local autonomy. Local autonomy has two facets, the administrative and the fiscal. Fiscal
autonomy means that local governments have the power to create their own sources of revenue in
addition to their equitable share in the national taxes released by the National Government, as well
as the power to allocate their resources in accordance with their own priorities.  Such autonomy is as
1

indispensable to the viability of the policy of decentralization as the other.

Implementing the constitutional mandate for decentralization and local autonomy, Congress enacted
Republic Act No. 7160, otherwise known as the Local Government Code (LGC), in order to
guarantee the fiscal autonomy of the LGUs by specifically providing that:

SECTION 284. Allotment of Internal Revenue Taxes. - Local government units shall have a share in
the national internal revenue taxes based on the collection of the third fiscal year preceding the
current fiscal year as follows:

(a) On the first year of the effectivity of this Code, thirty percent (30%); (b) On the second year,
thirty-five percent (35%); and

(c) On the third year and thereafter, forty percent (40%).


Provided, That in the event that the National Government incurs an unmanageable public sector
deficit, the President of the Philippines is hereby authorized, upon the recommendation of Secretary
of Finance, Secretary of Interior and Local Government, and Secretary of Budget and Management,
and subject to consultation with the presiding officers of both Houses of Congress and the
presidents of the "liga", to make the necessary adjustments in the internal revenue allotment of local
government units but in no case shall the allotment be less than thirty percent (30%) of the collection
of national internal revenue taxes of the third fiscal year preceding the current fiscal year: Provided,
further, That in the first year of the effectivity of this Code, the local government units shall, in
addition to the thirty percent (30%) internal revenue allotment which shall include the cost of
devolved functions for essential public services, be entitled to receive the amount equivalent to the
cost of devolved personal services.

The share of the LGUs, heretofore known as the Internal Revenue Allotment (IRA), has been
regularly released to the LGUs. According to the implementing rules and regulations of the LGC, the
IRA is determined on the basis of the actual collections of the National Internal Revenue Taxes
(NIRTs) as certified by the Bureau of Internal Revenue (BIR). 2

G.R. No. 199802 (Mandanas, et al.) is a special civil action for certiorari, prohibition


and mandamus assailing the manner the General Appropriations Act (GAA) for FY 2012 computed
the IRA for the LGUs.

Mandanas, et al. allege herein that certain collections of NIR Ts by the Bureau of Customs (BOC) -
specifically: excise taxes, value added taxes (VATs) and documentary stamp taxes (DSTs) - have
not been included in the base amounts for the computation of the IRA; that such taxes, albeit
collected by the BOC, should form part of the base from which the IRA should be computed because
they constituted NIRTs; that, consequently, the release of the additional amount of
₱60,750,000,000.00 to the LGUs as their IRA for FY 2012 should be ordered; and that for the same
reason the LGUs should also be released their unpaid IRA for FY 1992 to FY 2011, inclusive,
totaling ₱438,103,906,675.73.

In G.R. No. 208488, Congressman Enrique Garcia, Jr., the lone petitioner, seeks the writ
of mandamus to compel the respondents thereat to compute the just share of the LGUs on the basis
of all national taxes. His petition insists on a literal reading of Section 6, Article X of the 1987
Constitution. He avers that the insertion by Congress of the words internal revenue in the
phrase national taxes found in Section 284 of the LGC caused the diminution of the base for
determining the just share of the LGUs, and should be declared unconstitutional; that, moreover, the
exclusion of certain taxes and accounts pursuant to or in accordance with special laws was similarly
constitutionally untenable; that the VA Ts and excise taxes collected by the BOC should be included
in the computation of the IRA; and that the respondents should compute the IRA on the basis of all
national tax collections, and thereafter distribute any shortfall to the LGUs.

It is noted that named as common respondents were the then incumbent Executive Secretary,
Secretary of Finance, the Secretary of the Department of Budget and Management (DBM), and the
Commissioner of Internal Revenue. In addition, Mandanas, et al. impleaded the National Treasurer,
while Garcia added the Commissioner of Customs.

The cases were consolidated on October 22, 2013.   In the meanwhile, Congressman Garcia, Jr.
3

passed away. Jose Enrique Garcia III, who was subsequently elected to the same congressional
post, was substituted for Congressman Garcia, Jr. as the petitioner in G.R. No. 208488 under the
resolution promulgated on August 23, 2016. 4
In response to the petitions, the several respondents, represented by the Office of the Solicitor
General (OSG), urged the dismissal of the petitions upon procedural and substantive considerations.

Anent the procedural considerations, the OSG argues that the petitions are procedurally defective
because, firstly, mandamus does not lie in order to achieve the reliefs sought because Congress
may not be compelled to appropriate the sums allegedly illegally withheld for to do so will violate the
doctrine of separation of powers; and, secondly, mandamus does not also lie to compel the DBM to
release the amounts to the LGUs because such disbursements will be contrary to the purposes
specified in the GAA; that Garcia has no clear legal right to sustain his suit for mandamus; that the
filing of Garcia's suit violates the doctrine of hierarchy of courts; and that Garcia's petition seeks
declaratory relief but the Court cannot grant such relief in the exercise of its original jurisdiction.

On the substantive considerations, the OSG avers that Article 284 of the LGC is consistent with the
mandate of Section 6, Article X of the 1987 Constitution to the effect that the LGUs shall have a just
share in the national taxes; that the determination of the just share is within the discretion of
Congress; that the limitation under the LGC of the basis for the just share in the NIRTs was within
the powers granted to Congress by the 1987 Constitution; that the LGUs have been receiving
their just share in the national taxes based on the correct base amount; that Congress has the
authority to exclude certain taxes from the base amount in computing the IRA; that there is a
distinction between the VA Ts, excise taxes and DSTs collected by the BIR, on one hand, and the
VA Ts, excise taxes and DSTs collected by the BOC, on the other, thereby warranting their different
treatment; and that Development Budget Coordination Committee (DBCC) Resolution No. 2003-02
dated September 4, 2003 has limited the base amount for the computation of the IRA to the "cash
collections based on the BIR data as reconciled with the Bureau of Treasury;" and that the collection
of such national taxes by the BOC should be excluded.

Issues

The issues for resolution are limited to the following, namely:

I.

Whether or not Mandamus is the proper vehicle to assail the constitutionality of the relevant
provisions of the GAA and the LGC;

II.

Whether or not Section 284 of the LGC is unconstitutional for being repugnant to Section 6, Article X
of the 1987 Constitution;

III.

Whether or not the existing shares given to the LGUs by virtue of the GAA is consistent with the
constitutional mandate to give LGUs a 'just share" to national taxes following Article X, Section 6 of
the 1987 Constitution;

IV.

Whether or not the petitioners are entitled to the reliefs prayed for.
Simply stated, the petitioners raise the novel question of whether or not the exclusion of certain
national taxes from the base amount for the computation of the just share of the LGUs in the national
taxes is constitutional.

Ruling of the Court

The petitions are partly meritorious.

I
Mandamus is an improper remedy

Mandanas, et al. seek the writs of certiorari, prohibition and mandamus, while Garcia prays for the
writ of mandamus. Both groups of petitioners impugn the validity of Section 284 of the LGC.

The remedy of mandamus is defined in Section 3, Rule 65 of the Rules of Court, which provides:

Section 3. Petition for mandamus. - When any tribunal, corporation, board, officer or person
unlawfully neglects the performance of an act which the law specifically enjoins as a duty resulting
from an office, trust, or station, or unlawfully excludes another from the use and enjoyment of a right
or office to which such other is entitled, and there is no other plain, speedy and adequate remedy in
the ordinary course of law, the person aggrieved thereby may file a verified petition in the proper
court, alleging the facts with certainty and praying that judgment be rendered commanding the
respondent, immediately or at some other time to be specified by the court, to do the act required to
be done to protect the rights of the petitioner, and to pay the damages sustained by the petitioner by
reason of the wrongful acts of the respondent.

The petition shall also contain a sworn certification of non-forum shopping as provided in the third
paragraph of section 3, Rule 46.

For the writ of mandamus to issue, the petitioner must show that the act sought to be performed or
compelled is ministerial on the part of the respondent. An act is ministerial when it does not require
the exercise of judgment and the act is performed pursuant to a legal mandate. The burden of proof
is on the mandamus petitioner to show that he is entitled to the performance of a legal right, and that
the respondent has a corresponding duty to perform the act. The writ of mandamus may not issue to
compel an official to do anything that is not his duty to do, or that is his duty not to do, or to obtain for
the petitioner anything to which he is not entitled by law.  5

Considering that its determination of what constitutes the just share of the LGUs in the national taxes
under the 1987 Constitution is an entirely discretionary power, Congress cannot be compelled by
writ of mandamus to act either way. The discretion of Congress thereon, being exclusive, is not
subject to external direction; otherwise, the delicate balance underlying our system of government
may be unduly disturbed. This conclusion should at once then demand the dismissal of the Garcia
petition in G.R. No. 208488, but we do not dismiss it. Garcia has attributed the non-release of some
portions of their IRA balances to an alleged congressional indiscretion - the diminution of the base
amount for computing the LGU's just share. He has asserted that Congress altered the constitutional
base not only by limiting the base to the NIRTs instead of including therein all national taxes, but
also by excluding some national taxes and revenues that only benefitted a few LGUs to the
detriment of the rest of the LGUs.

Garcia's petition, while dubbed as a petition for mandamus, is also a petition for certiorari because it


alleges that Congress thereby committed grave abuse of discretion amounting to lack or excess of
jurisdiction. It is worth reminding that the actual nature of every action is determined by the
allegations in the body of the pleading or the complaint itself, not by the nomenclature used to
designate the same.   Moreover, neither should the prayer for relief be controlling; hence, the courts
6

may still grant the proper relief as the facts alleged in the pleadings and the evidence introduced
may warrant even without a prayer for specific remedy. 7

In this regard, Garcia's allegation of the unconstitutionality of the insertion by Congress of the
words internal revenue in the phrase national taxes justifies treating his petition as one
for certiorari. It becomes our duty, then, to assume jurisdiction over his petition. In Araullo v. Aquino
III,  the Court has emphatically opined that the Court's certiorari jurisdiction under the expanded
8

judicial power as stated in the second paragraph of Section 1, Article VIII of the Constitution can be
asserted:

xxxx to set right and undo any act of grave abuse of discretion amounting to lack or excess of
jurisdiction by any branch or instrumentality of the Government, the Court is not at all precluded from
making the inquiry provided the challenge was properly brought by interested or affected parties.
The Court has been thereby entrusted expressly or by necessary implication with both the duty and
the obligation of determining, in appropriate cases, the validity of any assailed legislative or
executive action. This entrustment is consistent with the republican system of checks and balances.  9

Further, observing that one of the reliefs being sought by Garcia is identical to the main relief sought
by Mandanas, et al., the Court should rightly dwell on the substantive arguments posited by Garcia
to the extent that they are relevant to the ultimate resolution of these consolidated suits.

II.
Municipal corporations and their relationship with Congress

The correct resolution and fair disposition of the issues interposed for our consideration require a
review of the basic principles underlying our system of local governments, and of the extent of the
autonomy granted to the LGUs by the 1987 Constitution.

Municipal corporations are now commonly known as local governments. They are the bodies politic
established by law partly as agencies of the State to assist in the civil governance of the country.
Their chief purpose has been to regulate and administer the local and internal affairs of the cities,
municipalities or districts. They are legal institutions formed by charters from the sovereign power,
whereby the populations within communities living within prescribed areas have formed themselves
into bodies politic and corporate, and assumed their corporate names with the right of continuous
succession and for the purposes and with the authority of subordinate self-government and
improvement and the local administration of the affairs of the State. 10

Municipal corporations, being the mere creatures of the State, are subject to the will of Congress,
their creator. Their continued existence and the grant of their powers are dependent on the
discretion of Congress. On this matter, Judge John F. Dillon of the State of Iowa in the United States
of America enunciated in Merriam v. Moody's Executors  the rule of statutory construction that came
11

to be oft-mentioned as Dillon's Rule, to wit:

[A] municipal corporation possesses and can exercise the following powers and no others: First,
those granted in express words; second, those necessarily implied or necessarily incident to the
powers expressly granted; third, those absolutely essential to the declared objects and purposes of
the corporation-not simply convenient but indispensible; fourth, any fair doubt as to the existence of
a power is resolved by the courts against the corporation-against the existence of the powers.  12
The formulation of Dillon's Rule has since undergone slight modifications. Judge Dillon himself
introduced some of the modifications through his post-Merriam writings with the objective of
alleviating the original formulation's harshness. The word fairly was added to the second proviso; the
word absolutely was deleted from the third proviso; and the words reasonable and substantial were
added to the fourth proviso, thusly:

x x x second, those necessarily or fairly implied in or incident to the powers expressly granted; third,
those essential to x x x. Any fair, reasonable, doubt. 
13

The modified Dillon's Rule has been followed in this jurisdiction, and has remained despite both the
1973 Constitution and the 1987 Constitution mandating autonomy for local governments. This has
been made evident in several rulings of the Court, one of which was that handed down in Magtajas
v. Pryce Properties Corporation, lnc.: 
14

In light of all the above considerations, we see no way of arriving at the conclusion urged on us by
the petitioners that the ordinances in question are valid. On the contrary, we find that the ordinances
violate P.D. 1869, which has the character and force of a statute, as well as the public policy
expressed in the decree allowing the playing of certain games of chance despite the prohibition of
gambling in general.

The rationale of the requirement that the ordinances should not contravene a statute is
obvious. Municipal governments are only agents of the national government. Local councils
exercise only delegated legislative powers conferred on them by Congress as the national
lawmaking body. The delegate cannot be superior to the principal or exercise powers higher
than those of the latter. It is a heresy to suggest that the local government units can undo the
acts of Congress, from which they have derived their power in the first place, and negate by
mere ordinance the mandate of the statute.

Municipal corporations owe their origin to, and derive their powers and rights wholly from the
legislature. It breathes into them the breath of life, without which they cannot exist. As it
creates, so it may destroy. As it may destroy, it may abridge and control. Unless there is
some constitutional limitation on the right, the legislature might, by a single act, and if we
can suppose it capable of so great a folly and so great a wrong, sweep from existence all of
the municipal corporations in the State, and the corporation could not prevent it. We know of
no limitation on the right so far as to the corporation themselves are concerned. They are, so
to phrase it, the mere tenants at will of the legislature.

This basic relationship between the national legislature and the local government units has
not been enfeebled by the new provisions in the Constitution strengthening the policy of
local autonomy. Without meaning to detract from that policy, we here confirm that Congress
retains control of the local government units although in significantly reduced degree now
than under our previous Constitutions. The power to create still includes the power to
destroy. The power to grant still includes the power to withhold or recall.

True, there are certain notable innovations in the Constitution, like the direct conferment on
the local government units of the power to tax, which cannot now be withdrawn by mere
statute. By and large, however, the national legislature is still the principal of the local
government units, which cannot defy its will or modify or violate it. [Bold underscoring supplied
for emphasis]
Also, in the earlier ruling in Ganzon v. Court of Appeals,   the Court has pointed out that the 1987
15

Constitution, in mandating autonomy for the LGUs, did not intend to deprive Congress of its authority
and prerogatives over the LGUs.

Nonetheless, the LGC has tempered the application of Dillon's Rule in the Philippines by providing a
norm of interpretation in favor of the LGUs in its Section 5(a), to wit:

xxxx

(a) Any provision on a power of a local government unit shall be liberally interpreted in its favor, and
in case of doubt, any question thereon shall be resolved in favor of devolution of powers and of the
local government unit. Any fair and reasonable doubt as to the existence of the power shall be
interpreted in favor of the local government unit concerned; [Bold underscoring supplied for
emphasis]

xxxx

III.
The extent of local autonomy in the Philippines

Regardless, there remains no question that Congress possesses and wields plenary power to
control and direct the destiny of the LGUs, subject only to the Constitution itself, for Congress, just
like any branch of the Government, should bow down to the majesty of the Constitution, which is
always supreme.

The 1987 Constitution limits Congress' control over the LGUs by ordaining in Section 25 of its Article
II that: "The State shall ensure the autonomy of local governments." The autonomy of the LGUs as
thereby ensured does not contemplate the fragmentation of the Philippines into a collection of mini-
states,   or the creation of imperium in imperio.   The grant of autonomy simply means that
16 17

Congress will allow the LGUs to perform certain functions and exercise certain powers in order not
for them to be overly dependent on the National Government subject to the limitations that the 1987
Constitution or Congress may impose.   Local autonomy recognizes the wholeness of the Philippine
18

society in its ethnolinguistic, cultural, and even religious diversities.


19

The constitutional mandate to ensure local autonomy refers to decentralization.  In its broad or
20

general sense, decentralization has two forms in the Philippine setting, namely: the decentralization
of power and the decentralization of administration. The decentralization of power involves the
abdication of political power in favor of the autonomous LGUs as to grant them the freedom to chart
their own destinies and to shape their futures with minimum intervention from the central
government. This amounts to self-immolation because the autonomous LGUs thereby become
accountable not to the central authorities but to their constituencies. On the other hand, the
decentralization of administration occurs when the central government delegates administrative
powers to the LGUs as the means of broadening the base of governmental powers and of making
the LGUs more responsive and accountable in the process, and thereby ensure their fullest
development as self-reliant communities and more effective partners in the pursuit of the goals of
national development and social progress. This form of decentralization further relieves the central
government of the burden of managing local affairs so that it can concentrate on national concerns. 21

Two groups of LGUs enjoy decentralization in distinct ways. The decentralization of power has been
given to the regional units (namely, the Autonomous Region for Muslim Mindanao [ARMM] and the
constitutionally-mandated Cordillera Autonomous Region [CAR]). The other group of
LGUs (i.e., provinces, cities, municipalities and barangays) enjoy the decentralization of
administration.  The distinction can be reasonably understood. The provinces, cities, municipalities
22

and barangays are given decentralized administration to make governance at the local levels more
directly responsive and effective. In turn, the economic, political and social developments of the
smaller political units are expected to propel social and economic growth and development.   In 23

contrast, the regional autonomy of the ARMM and the CAR aims to permit determinate groups with
common traditions and shared social-cultural characteristics to freely develop their ways of life and
heritage, to exercise their rights, and to be in charge of their own affairs through the establishment of
a special governance regime for certain member communities who choose their own authorities from
within themselves, and exercise the jurisdictional authority legally accorded to them to decide their
internal community affairs. 24

It is to be underscored, however, that the decentralization of power in favor of the regional units is
not unlimited but involves only the powers enumerated by Section 20, Article X of the 1987
Constitution and by the acts of Congress. For, with various powers being devolved to the regional
units, the grant and exercise of such powers should always be consistent with and limited by the
1987 Constitution and the national laws.   In other words, the powers are guardedly, not absolutely,
25

abdicated by the National Government.

Illustrative of the limitation is what transpired in Serna v. Commission on Elections,  where the Court
26

struck down Section 19, Article VI of Republic Act No. 9054 (An Act to Strengthen and Expand the
Organic Act for the Autonomous Region in Muslim Mindanao, Amending for the Purpose Republic
Act No. 6734, entitled "An Act Providing for the Autonomous Region in Muslim Mindanao," as
Amended) insofar as the provision granted to the ARMM the power to create provinces and cities,
and consequently declared as void Muslim Mindanao Autonomy Act No. 201 creating the Province
of Shariff Kabunsuan for being contrary to Section 5, Article VI and Section 20, Article X of the 1987
Constitution, as well as Section 3 of the Ordinance appended to the 1987 Constitution. The Court
clarified therein that only Congress could create provinces and cities. This was because the creation
of provinces and cities necessarily entailed the creation of legislative districts, a power that only
Congress could exercise pursuant to Section 5, Article VI of the 1987 Constitution and Section 3 of
the Ordinance appended to the Constitution; as such, the ARMM would be thereby usurping the
power of Congress to create legislative districts and national offices.
27

The 1987 Constitution has surely encouraged decentralization by mandating that a system of
decentralization be instituted through the LGC in order to enable a more responsive and accountable
local government structure.  It has also delegated the power to tax to the LGUs by authorizing them
28

to create their own sources of income that would make them self-reliant.  It further ensures that each
29

and every LGU will have a just share in national taxes as well in the development of the national
wealth.30

The LGC has further delineated in its Section 3 the different operative principles of decentralization
to be adhered to consistently with the constitutional policy on local autonomy, viz.:

Sec. 3. Operative Principles of Decentralization-

The formulation and implementation of policies and measures on local autonomy shall be guided by
the following operative principles:

(a) There shall be an effective allocation among the different local government units of their
respective powers, functions, responsibilities, and resources;
(b) There shall be established in every local government unit an accountable, efficient, and
dynamic organizational structure and operating mechanism that will meet the priority needs
and service requirements of its communities;

(c) Subject to civil service law, rules and regulations, local officials and employees paid
wholly or mainly from local funds shall be appointed or removed, according to merit and
fitness, by the appropriate appointing authority;

(d) The vesting of duty, responsibility, and accountability in local government units shall be
accompanied with provision for reasonably adequate resources to discharge their powers
and effectively carry out their functions: hence, they shall have the power to create and
broaden their own sources of revenue and the right to a just share in national taxes and an
equitable share in the proceeds of the utilization and development of the national wealth
within their respective areas;

(e) Provinces with respect to component cities and municipalities, and cities and
municipalities with respect to component barangays, shall ensure that the acts of their
component units are within the scope of their prescribed powers and functions;

(f) Local government units may group themselves, consolidate or coordinate their efforts,
services, and resources commonly beneficial to them;

(g) The capabilities of local government units, especially the municipalities and barangays,
shall be enhanced by providing them with opportunities to participate actively in the
implementation of national programs and projects;

(h) There shall be a continuing mechanism to enhance local autonomy not only by legislative
enabling acts but also by administrative and organizational reforms;

(i) Local government units shall share with the national government the responsibility in the
management and maintenance of ecological balance within their territorial jurisdiction,
subject to the provisions of this Code and national policies;

(j) Effective mechanisms for ensuring the accountability of local government units to their
respective constituents shall be strengthened in order to upgrade continually the quality of
local leadership;

(k) The realization of local autonomy shall be facilitated through improved coordination of
national government policies and programs an extension of adequate technical and material
assistance to less developed and deserving local government units;

(l) The participation of the private sector in local governance, particularly in the delivery of
basic services, shall be encouraged to ensure the viability of local autonomy as an
alternative strategy for sustainable development; and

(m) The national government shall ensure that decentralization contributes to the continuing
improvement of the performance of local government units and the quality of community life.

Based on the foregoing delineation, decentralization can be considered as the decision by the
central government to empower its subordinates, whether geographically or functionally constituted,
to exercise authority in certain areas. It involves decision-making by subnational units, and is
typically a delegated power, whereby a larger government chooses to delegate authority to more
local governments.  It is also a process, being the set of policies, electoral or constitutional reforms
31

that transfer responsibilities, resources or authority from the higher to the lower levels of
government.  It is often viewed as a shift of authority towards local governments and away from the
32

central government, with total government authority over society and economy imagined as fixed. 33

As a system of transferring authority and power from the National Government to the LGUs,
decentralization in the Philippines may be categorized into four, namely: (1) political decentralization
or devolution; (2) administrative decentralization or deconcentration; (3) fiscal decentralization; and
(4) policy or decision-making decentralization.

Political decentralization or devolution occurs when there is a transfer of powers, responsibilities,


and resources from the central government to the LOU s for the performance of certain functions. It
is a more liberal form of decentralization because there is an actual transfer of powers and
responsibilities. It aims to grant greater autonomy to the LGUs in cognizance of their right to self-
government, to make them self-reliant, and to improve their administrative and technical
capabilities.  It is an act by which the National Government confers power and authority upon the
34

various LGUs to perform specific functions and responsibilities.  It encompasses reforms to open
35

sub-national representation and policies to "devolve political authority or electoral capacities to sub-
national actors. "  Section 16 to Section 19 of the LGC characterize political decentralization in the
36

LGC as different LGUs empowered to address the different needs of their constituents. In contrast,
devolution in favor of the regional units is more expansive because they are given the authority to
regulate a wider array of subjects, including personal, family and property relations.

Administrative decentralization or deconcentration involves the transfer of functions or the delegation


of authority and responsibility from the national office to the regional and local offices.   Consistent
37

with this concept, the LGC has created the Local School Boards,  the Local Health Boards  and the
38 39

Local Development Councils,  and has transferred some of the authority from the agencies of the
40

National Government, like the Department of Education and the Department of Health, to such
bodies to better cope up with the needs of particular localities.

Fiscal decentralization means that the LGUs have the power to create their own sources of revenue
in addition to their just share in the national taxes released by the National Government. It includes
the power to allocate their resources in accordance with their own priorities. It thus extends to the
preparation of their budgets, so that the local officials have to work within the constraints of their
budgets. The budgets are not formulated at the national level and imposed on local governments,
without regard as to whether or not they are relevant to local needs and resources. Hence, the
necessity of a balancing of viewpoints and the harmonization of proposals from both local and
national officials, who in any case are partners in the attainment of national goals, is recognized and
addressed. 41

Fiscal decentralization emanates from a specific constitutional mandate that is expressed in several
provisions of Article X (Local Government) of the 1987 Constitution, specifically: Section 5;  Section
42

6;  and Section 7.
43 44

The constitutional authority extended to each and every LGU to create its own sources of income
and revenue has been formalized from Section 128 to Section 133 of the LGC. To implement the
LGUs' entitlement to the just share in the national taxes, Congress has enacted Section 284 to
Section 288 of the LGC. Congress has further enacted Section 289 to Section 294 of the LGC to
define the share of the LGUs in the national wealth. Indeed, the requirement for the automatic
release to the LGUs of their just share in the national taxes is but the consequence of the
constitutional mandate for fiscal decentralization. 
45
For sure, fiscal decentralization does not signify the absolute freedom of the LGUs to create their
own sources of revenue and to spend their revenues unrestrictedly or upon their individual whims
and caprices. Congress has subjected the LGUs' power to tax to the guidelines set in Section 130 of
the LGC and to the limitations stated in Section 133 of the LGC. The concept of local fiscal
autonomy does not exclude any manner of intervention by the National Government in the form of
supervision if only to ensure that the local programs, fiscal and otherwise, are consistent with the
national goals.46

Lastly, policy- or decision-making decentralization exists if at least one sub-national tier of


government has exclusive authority to make decisions on at least one policy issue. 47

In fine, certain limitations are and can be imposed by Congress in all the forms of decentralization,
for local autonomy, whether as to power or as to administration, is not absolute. The LGUs remain to
be the tenants of the will of Congress subject to the guarantees that the Constitution itself imposes.

IV.
Section 284 of the LGC deviates from the plain language
of Section 6 of Article X of the 1987 Constitution

Section 6, Article X the 1987 Constitution textually commands the allocation to the LGUs of a just
share in the national taxes, viz.:

Section 6. Local government units shall have a just share, as determined by law, in the national
taxes which shall be automatically released to them.

Section 6, when parsed, embodies three mandates, namely: (1) the LGUs shall have a just share in
the national taxes; (2) the just share shall be determined by law; and (3) the just share shall
be automatically released to the LGUs. 48

Congress has sought to carry out the second mandate of Section 6 by enacting Section 284, Title
III (Shares of Local Government Units in the Proceeds of National Taxes), of the LGC, which is
again quoted for ready reference:

Section 284. Allotment of Internal Revenue Taxes. - Local government units shall have a share in
the national internal revenue taxes based on the collection of the third fiscal year preceding the
current fiscal year as follows:

(a) On the first year of the effectivity of this Code, thirty percent (30%);

(b) On the second year, thirty-five percent (35%); and

(c) On the third year and thereafter, forty percent (40%).

Provided, That in the event that the national government incurs an unmanageable public sector
deficit, the President of the Philippines is hereby authorized, upon the recommendation of Secretary
of Finance, Secretary of Interior and Local Government and Secretary of Budget and Management,
and subject to consultation with the presiding officers of both Houses of Congress and the
presidents of the "liga", to make the necessary adjustments in the internal. revenue allotment of local
government units but in no case shall the allotment be less than thirty percent (30%) of the collection
of national internal revenue taxes of the third fiscal year preceding the current fiscal year: Provided,
further, That in the first year of the effectivity of this Code, the local government units shall, in
addition to the thirty percent (30%) internal revenue allotment which shall include the cost of
devolved functions for essential public services, be entitled to receive the amount equivalent to the
cost of devolved personal services.

There is no issue as to what constitutes the LGUs' just share expressed in percentages of the


national taxes (i.e.,30%, 35% and 40% stipulated in subparagraphs (a), (b), and (c) of Section 284 ).
Yet, Section 6, supra, mentions national taxes as the source of the just share of the LGUs while
Section 284 ordains that the share of the LG Us be taken from national internal revenue
taxes instead.

Has not Congress thereby infringed the constitutional provision?

Garcia contends that Congress has exceeded its constitutional boundary by limiting to the NIRTs the
base from which to compute the just share of the LGUs.

We agree with Garcia's contention.

Although the power of Congress to make laws is plenary in nature, congressional lawmaking
remains subject to the limitations stated in the 1987 Constitution.  The phrase national internal
49

revenue taxes engrafted in Section 284 is undoubtedly more restrictive than the term national
taxes written in Section 6. As such, Congress has actually departed from the letter of the 1987
Constitution stating that national taxes should be the base from which the just share of the LGU
comes. Such departure is impermissible. Verba legis non est recedendum (from the words of a
statute there should be no departure).   Equally impermissible is that Congress has also thereby
50

curtailed the guarantee of fiscal autonomy in favor of the LGUs under the 1987 Constitution.

Taxes are the enforced proportional contributions exacted by the State from persons and properties
pursuant to its sovereignty in order to support the Gove1nment and to defray all the public needs.
Every tax has three elements, namely: (a) it is an enforced proportional contribution from persons
and properties; (b) it is imposed by the State by virtue of its sovereignty; and (c) it is levied for the
support of the Government.  Taxes are classified into national and local. National taxes are those
51

levied by the National Government, while local taxes are those levied by the LGUs. 52

What the phrase national internal revenue taxes as used in Section 284 included are all the taxes
enumerated in Section 21 of the National Internal Revenue Code (NIRC), as amended by R.A. No.
8424, viz.:

Section 21. Sources of Revenue. - The following taxes, fees and charges are deemed to be national
internal revenue taxes:

(a) Income tax;

(b) Estate and donor's taxes;

(c) Value-added tax;

(d) Other percentage taxes;

(e) Excise taxes;

(f) Documentary stan1p taxes; and


(g) Such other taxes as arc or hereafter may be imposed and collected by the Bureau of
Internal Revenue.

In view of the foregoing enumeration of what are the national internal revenue taxes, Section 284
has effectively deprived the LGUs from deriving their just share from other national taxes, like the
customs duties.

Strictly speaking, customs duties are also taxes because they are exactions whose proceeds
become public funds. According to Garcia v. Executive Secretary,  customs duties is the
53

nomenclature given to taxes imposed on the importation and exportation of commodities and
merchandise to or from a foreign country. Although customs duties have either or both the
generation of revenue and the regulation of economic or social activity as their moving purposes, it is
often difficult to say which of the two is the principal objective in a particular instance, for, verily,
customs duties, much like internal revenue taxes, are rarely designed to achieve only one policy
objective.  We further note that Section 102(00) of R.A. No. 10863 (Customs Modernization and
54

Tariff Act) expressly includes all fees and charges imposed under the Act under the blanket term
of taxes.

It is clear from the foregoing clarification that the exclusion of other national taxes like customs
duties from the base for determining the just share of the LG Us contravened the express
constitutional edict in Section 6, Article X the 1987 Constitution.

Still, the OSG posits that Congress can manipulate, by law, the base of the allocation of the just
share in the national taxes of the LGUs.

The position of the OSG cannot be sustained. Although it has the primary discretion to determine
and fix the just share of the LGUs in the national taxes (e.g., Section 284 of the LGC), Congress
cannot disobey the express mandate of Section 6, Article X of the 1987 Constitution for the just
share of the LGUs to be derived from the national taxes. The phrase as determined by law in
Section 6 follows and qualifies the phrase just share, and cannot be construed as qualifying the
succeeding phrase in the national taxes. The intent of the people in respect of Section 6 is really that
the base for reckoning the just share of the LGUs should includes all national taxes. To read Section
6 differently as requiring that the just share of LGUs in the national taxes shall be determined by
law is tantamount to the unauthorized revision of the 1987 Constitution.

V.
Congress can validly exclude taxes that will constitute the base amount for
the computation of the IRA only if a Constitutional provision allows such exclusion

Garcia submits that even assuming that the present version of Section 284 of the LGC is
constitutionally valid, the implementation thereof has been erroneous because Section 284 does not
authorize any exclusion or deduction from the collections of the NIRTs for purposes of the
computation of the allocations to the LGUs. He further submits that the exclusion of certain NIRTs
diminishes the fiscal autonomy granted to the LGUs. He claims that the following NIRTs have been
illegally excluded from the base for determining the fair share of the LGUs in the IRA, to wit:

(1) NIRTs collected by the cities and provinces and divided exclusively among the LGUs of
the Autonomous Region for Muslim Mindanao (ARMM), the regional government and the
central government, pursuant to Section 15  in relation to Section 9,  Article IX of R.A. No.
55 56

9054 (An Act to Strengthen and Expand the Organic Act for the Autonomous Region in
Muslim Mindanao, amending for the purpose Republic Act No. 6734, entitled An Act
providing for an Organic Act for the Autonomous Region in Muslim Mindanao);
(2) The shares in the excise taxes on mineral products of the different LG Us, as provided in
Section 287 of the NIRC  in relation to Section 290 of the LGC;
57 58

(3) The shares of the relevant LGUs in the franchise taxes paid by Manila Jockey Club,
Inc.  and Philippine Racing Club, Inc.;
59 60

(4) The shares of various municipalities in VAT collections under R.A. No. 7643 (An Act to
Empower the Commissioner of Internal Revenue to Require the Payment of the Value
Added Tax Every Month and to Allow Local Government Units to Share in VAT Revenue,
Amending for this Purpose Certain Sections of the National Internal Revenue Code) as
embodied in Section 283 of the NIRC; 61

(5) The shares of relevant LGUs in the proceeds of the sale and conversion of former military
bases in accordance with R.A. No. 7227 (Bases Conversion and Development Act of 1992); 62

(6) The shares of different LGUs in the excise taxes imposed on locally manufactured
Virginia tobacco products as provided in Section 3 of R.A. No. 7171 (An Act to Promote the
Development of the Farmers in the Virginia Tobacco Producing Provinces), and as now
provided in Section 289 of the NIRC; 63

(7) The shares of different LGUs in the incremental revenues from Burley and native tobacco
products under Section 8 of R.A. No. 8240 (An Act Amending Sections 138, 140 and 142 of
the National Internal Revenue Code as Amended and for Other Purposes) and as now
provided in Section 288 of the NIRC;  and
64

(8) The share of the Commission of Audit (COA) in the NIRTs as provided in Section 24p) of
P.D. No. 1445 (Government Auditing Code of the Philippines)   in relation to Section 284 of
65

the NIRC. 66

Garcia insists that the foregoing taxes and revenues should have been included by Congress and,
by extension, the BIR in the base for computing the IRA on the strength of the cited provisions; that
the LGC did not authorize such exclusion; and that the continued exclusion has undermined the
fiscal autonomy guaranteed by the 1987 Constitution.

The insistence of Garcia is valid to an extent.

An examination of the above-enumerated laws confirms that the following have been excluded from
the base for reckoning the just share of the LGUs as required by Section 6, Article X of the 1987
Constitution, namely:

(a) The share of the affected LGUs in the proceeds of the sale and conversion of former military
bases in accordance with R.A. No. 7227;

(b) The share of the different LGUs in the excise taxes imposed on locally manufactured Virginia
tobacco products as provided for in Section 3, R.A. No. 7171, and as now provided in Section 289 of
the NIRC;

(c) The share of the different LGU s in incremental revenues from Burley and native tobacco
products under Section 8 of R.A. No. 8240, and as now provided for in Section 288 of the NIRC;
(d) The share of the COA in the NIRTs as provided in Section 24(3) of P.D. No. 1445  in relation to
67

Section 284 of the NIRC;

(e) The shares of the different LGUs in the excise taxes on mineral products, as provided in Section
287 of the NIRC in relation to Section 290 of the LGC;

(f) The NIRTs collected by the cities and provinces and divided exclusively among the LGUs of the
ARMM, the regional government and the central government, pursuant to Section 15  in relation to
68

Section 9,  Article IX of R. A. No. 9054; and


69

(g) The shares of the relevant LG Us in the franchise taxes paid by Manila Jockey Club, Inc., and the
Philippine Racing Club, Inc.

Anent the share of the affected LG Us in the proceeds of the sale and conversion of the former
military bases pursuant to R.A. No. 7227, the exclusion is warranted for the reason that such
proceeds do not come from a tax, fee or exaction imposed on the sale and conversion.

As to the share of the affected LGUs in the excise taxes imposed on locally manufactured Virginia
tobacco products under R.A. No. 7171 (now Section 289 of the NIRC); the share of the affected
LGUs in incremental revenues from Burley and native tobacco products under Section 8, R.A. No.
8240 (now Section 288 of the NIRC); the share of the COA in the NIRTs pursuant to Section 24(3) of
P.D. No. 1445 in relation to Section 284 of the NIRC; and the share of the host LGUs in the
franchise taxes paid by the Manila Jockey Club, Inc., and Philippine Racing Club, Inc., under Section
6 of R.A. No. 6631 and Section 8 of R:A. No. 6632, respectively, the exclusion is also justified.
Although such shares involved national taxes as defined under the NIRC, Congress had the
authority to exclude them by virtue of their being taxes imposed for special purposes. A reading of
Section 288 and Section 289 of the NIRC and Section 24(3) of P.D. No. 1445 in relation to Section
284 of the NIRC reveals that all such taxes are levied and collected for a special purpose.   The70

same is true for the franchise taxes paid under Section 6 of R.A. No. 6631 and Section 8 of R.A. No.
6632, inasmuch as certain percentages of the franchise taxes go to different beneficiaries. The
exclusion conforms to Section 29(3), Article VI of the 1987 Constitution, which states:

Section 29. x x x

xxxx

(3) All money collected on any tax levied for a special purpose shall be treated as a special
fund and paid out for such purpose only. If the purpose for which a special fund was created has
been fulfilled or abandoned, the balance, if any, shall be transferred to the general funds of the
Government. [Bold emphasis supplied]

The exclusion of the share of the different LGUs in the excise taxes imposed on mineral products
pursuant to Section 287 of the NIRC in relation to Section 290 of the LGC is premised on a different
constitutional provision. Section 7, Article X of the 1987 Constitution allows affected LGUs to have
an equitable share in the proceeds of the utilization of the nation's national wealth "within their
respective areas," to wit:

Section 7. Local governments shall be entitled to an equitable share in the proceeds of the utilization
and development of the national wealth within their respective areas, in the manner provided by law,
including sharing the same with the inhabitants by way of direct benefits.
This constitutional provision is implemented by Section 287 of the NIRC and Section 290 of the LGC
thusly:

SEC. 287. Shares of Local Government Units in the Proceeds from the Development and Utilization
of the National Wealth. - Local Government units shall have an equitable share in the proceeds
derived from the utilization and development of the national wealth, within their respective areas,
including sharing the same with the inhabitants by way of direct benefits.

(A) Amount of Share of Local Government Units. - Local government units shall, in addition to the
internal revenue allotment, have a share of forty percent (40'Yo) of the gross collection derived by
the national government from the preceding fiscal year from excise taxes on mineral products,
royalties, and such other taxes, fees or charges, including related surcharges, interests or fines, and
from its share in any co-production, joint venture or production sharing agreement in the utilization
and development of the national wealth within their territorial jurisdiction.

(B) Share of the Local Governments from Any Government Agency or Government-owned or -
Controlled Corporation. - Local Government Units shall have a share, based on the preceding fiscal
year, from the proceeds derived by any government agency or government-owned or controlled
corporation engaged in the utilization and development of the national wealth based on the following
formula, whichever will produce a higher share for the local government unit:

(1) One percent (l %) of the gross sales or receipts of the preceding calendar year, or

(2) Forty percent (40%) of the excise taxes on mineral products, royalties, and such other taxes, fees
or charges, including related surcharges, interests or fines the government agency or government-
owned or -controlled corporations would have paid if it were not otherwise exempt. [Bold emphasis
supplied]

SEC. 290. Amount of Share of Local Government Units. - Local government units shall, in addition to
the internal revenue allotment, have a share of forty percent ( 40%) of the gross collection derived by
the national government from the preceding fiscal year from mining taxes, royalties, forestry and
fishery charges, and such other taxes, fees, or charges, including related surcharges, interests, or
fines, and from its share in any co-production, joint venture or production sharing agreement in the
utilization and development of the national wealth within their territorial jurisdiction. [Bold emphasis
supplied]

Lastly, the NIRTs collected by the provinces and cities within the ARMM whose portions are
distributed to the ARMM's provincial, city and regional governments are also properly excluded for
such taxes are intended to truly enable a sustainable and feasible autonomous region as guaranteed
by the 1987 Constitution. The mandate under Section 15 to Section 21, Article X of the 1987
Constitution is to allow the separate development of peoples with distinctive cultures and traditions in
the autonomous areas.  The grant of autonomy to the autonomous regions includes the right of self-
71

determination-which in turn ensures the right of the peoples residing therein to the necessary level of
autonomy that will guarantee the support of their own cultural identities, the establishment of
priorities by their respective communities' internal decision-making processes and the management
of collective matters by themselves.  As such, the NIRTs collected by the provinces and cities within
72

the ARMM will ensure local autonomy and their very existence with a continuous supply of funding
sourced from their very own areas. The ARMM will become self-reliant and dynamic consistent with
the dictates of the 1987 Constitution.
The shares of the municipalities in the VATs collected pursuant to R.A. No. 7643 should be included
in determining the base for computing the just share because such VATs are national taxes, and
nothing can validly justify their exclusion.

In recapitulation, the national taxes to be included in the base for computing the just share the LGUs
shall henceforth be, but shall not be limited to, the following:

1. The NIRTs enumerated in Section 21 of the NIRC, as amended, to be inclusive of the VA Ts,
excise taxes, and DSTs collected by the BIR and the BOC, and their deputized agents;

2. Tariff and customs duties collected by the BOC;

3. 50% of the VATs collected in the ARMM, and 30% of all other national taxes collected in the
ARMM; the remaining 50% of the VA Ts and 70% of the collections of the other national taxes in the
ARMM shall be the exclusive share of the ARMM pursuant to Section 9 and Section 15 of R.A. No.
9054;

4. 60% of the national taxes collected from the exploitation and development of the national wealth;
the remaining 40% will exclusively accrue to the host LGUs pursuant to Section 290 of the LGC;

5. 85% of the excise taxes collected from locally manufactured Virginia and other tobacco products;
the remaining 15% shall accrue to the special purpose funds pursuant created in R.A. No. 7171 and
R.A. No. 7227;

6. The entire 50% of the national taxes collected under Section 106, Section 108 and Section 116 of
the NIRC in excess of the increase in collections for the immediately preceding year; and

7. 5% of the franchise taxes in favor of the national government paid by franchise holders in
accordance with Section 6 of R.A. No. 6631 and Section 8 of R.A. No. 6632.

VI.
Entitlement to the reliefs sought

The petitioners' prayer for the payment of the arrears of the LGUs' just share on the theory that the
computation of the base amount had been unconstitutional all along cannot be granted.

It is true that with our declaration today that the IRA is not in accordance with the constitutional
determination of the just share of the LGUs in the national taxes, logic demands that the LGUs
should receive the difference between the just share they should have received had the LGC
properly reckoned such just share from all national taxes, on the one hand, and the share -
represented by the IRA- the LGUs have actually received since the effectivity of the IRA under the
LGC, on the other. This puts the National Government in arrears as to the just share of the LGUs. A
legislative or executive act declared void for being unconstitutional cannot give rise to any right or
obligation. 
73

Yet, the Court has conceded in Arau/lo v. Aquino III that:


74

x x x the generality of the rule makes us ponder whether rigidly applying the rule may at times
be impracticable or wasteful. Should we not recognize the need to except from the rigid
application of the rule the instances in which the void law or executive act produced an
almost irreversible result?
The need is answered by the doctrine of operative fact. The doctrine, definitely not a novel one, has
been exhaustively explained in De Agbayani v. Philippine National Bank:

The decision now on appeal reflects the orthodox view that an unconstitutional act, for that matter an
executive order or a municipal ordinance likewise suffering from that infirmity, cannot be the source
of any legal rights or duties. Nor can it justify any official act taken under it. Its repugnancy to the
fundamental law once judicially declared results in its being to all intents and purposes a mere scrap
of paper. As the new Civil Code puts it: 'When the courts declare a law to be inconsistent with the
Constitution, the former shall be void and the latter shall govern.' Administrative or executive acts,
orders and regulations shall be valid only when they are not contrary to the laws of the Constitution.
It is understandable why it should be so, the Constitution being supreme and paramount. Any
legislative or executive act contrary to its terms cannot survive.

Such a view has support in logic and possesses the merit of simplicity. It may not however
be sufficiently realistic. It does not admit of doubt that prior to the declaration of nullity such
challenged legislative or executive act must have been in force and had to be complied with.
This is so as until after the judiciary, in an appropriate case, declares its invalidity, it is
entitled to obedience and respect. Parties may have acted under it and may have changed
their positions. What could be more fitting than that in a subsequent litigation regard be had
to what has been done while such legislative or executive act was in operation and presumed
to be valid in all respects. It is now accepted as a doctrine that prior to its being nullified, its
existence as a fact must be reckoned with. This is merely to reflect awareness that precisely
because the judiciary is the governmental organ which has the final say on whether or not a
legislative or executive measure is valid, a period of time may have elapsed before it can
exercise the power of judicial review that may lead to a declaration of nullity. It would be to
deprive the law of its quality of fairness and justice then, if there be no recognition of what
had transpired prior to such adjudication.

In the language of an American Supreme Court decision: ‘The actual existence of a statute, prior to
such a determination [of unconstitutionality], is an operative fact and may have consequences which
cannot justly be ignored. The past cannot always be erased by a new judicial declaration. The effect
of the subsequent ruling as to invalidity may have to be considered in various aspects, with respect
to particular relations, individual and corporate, and particular conduct, private and official.'

The doctrine of operative fact recognizes the existence of the law or executive act prior to the
determination of its unconstitutionality as an operative fact that produced consequences that cannot
always be erased, ignored or disregarded. In short, it nullifies the void law or executive act but
sustains its effects. It provides an exception to the general rule that a void or unconstitutional law
produces no effect.  But its use must be subjected to great scrutiny and circumspection, and it
75

cannot be invoked to validate an unconstitutional law or executive act, but is resorted to only as a
matter of equity and fair play.   It applies only to cases where extraordinary circumstances exist, and
76

only when the extraordinary circumstances have met the stringent conditions that will permit its
application.

Conformably with the foregoing pronouncements in Araullo v. Aquino III, the effect of our declaration
through this decision of the unconstitutionality of Section 284 of the LGC and its related laws as far
as they limited the source of the just share of the LGUs to the NIRTs is prospective. It cannot be
otherwise.

VII.
Automatic release of the LGUs' just share in the National Taxes
Section 6, Article X of the 1987 Constitution commands that the just share of the LGUs in national
taxes shall be automatically released to them. The term automatic connotes something mechanical,
spontaneous and perfunctory; and, in the context of this case, the LGUs are not required to perform
any act or thing in order to receive their just share in the national taxes.
77

Before anything, we must highlight that the 1987 Constitution includes several provisions that
actually deal with and authorize the automatic release of funds by the National Government.

To begin with, Section 3 of Article VIII favors the Judiciary with the automatic and regular release of
its appropriations:

Section 3. The Judiciary shall enjoy fiscal autonomy. Appropriations for the Judiciary may not be
reduced by the legislature below the amount appropriated for the previous year and, after approval,
shall be automatically and regularly released.

Then there is Section 5 of Article IX(A), which contains the common provision in favor of the
Constitutional Commissions:

Section 5. The Commission shall enjoy fiscal autonomy. Their approved annual appropriations shall
be automatically and regularly released.

Section 14 of Article XI extends to the Office of the Ombudsman a similar privilege:

Section 14. The Office of the Ombudsman shall enjoy fiscal autonomy. Its approved annual
appropriations shall be automatically and regularly released.

Section 17(4) of Article XIII replicates the privilege in favour of the Commission on Human Rights:

Section 17(4) The approved annual appropriations of the Commission shall be automatically and
regularly released.

The foregoing constitutional provisions share two aspects. The first relates to the grant of fiscal
autonomy, and the second concerns the automatic release of funds.   The common denominator of
78

the provisions is that the automatic release of the appropriated amounts is predicated on the
approval of the annual appropriations of the offices or agencies concerned.

Directly contrasting with the foregoing provisions is Section 6, Article X of the 1987 Constitution
because the latter provision forthrightly ordains that the "(l)ocal government units shall have a just
share, as determined by law, in the national taxes which shall be automatically released to
them." Section 6 does not mention of appropriation as a condition for the automatic release of the
just share to the LGUs. This is because Congress not only already determined the just
share through the LGC's fixing the percentage of the collections of the NIRTs to constitute such fair
share subject to the power of the President to adjust the same in order to manage public sector
deficits subject to limitations on the adjustments, but also explicitly authorized such just share to
be "automatically released" to the LGUs in the proportions and regularity set under Section 285  of79

the LGC without need of annual appropriation. To operationalize the automatic release without need
of appropriation, Section 286 of the LGC clearly provides that the automatic release of the just
share directly to the provincial, city, municipal or barangay treasurer, as the case may be, shall
be "without need of any further action," viz.:
Section 286. Automatic Release of Shares. - (a) The share of each local government unit shall
be released, without need of any further action; directly to the provincial, city, municipal or
barangay treasurer, as the case may be, on a quarterly basis within five (5) days after the end
of each quarter, and which shall not be subject to any lien or holdback that may be imposed
by the National Government for whatever purpose. x x x (Bold emphasis supplied)

The 1987 Constitution is forthright and unequivocal in ordering that the just share of the LGUs in the
national taxes shall be automatically released to them. With Congress having established the just
share through the LGC, it seems to be beyond debate that the inclusion of the just share of the
LGUs in the annual GAAs is unnecessary, if not superfluous. Hence, the just share of the LGUs in
the national taxes shall be released to them without need of yearly appropriation.

1. DECLARES the phrase "internal revenue" appearing in Section 284 of Republic Act No.
7160 (Local Government Code) UNCONSTITUTIONAL, and DELETES the phrase from Section
284.

Section 284, as hereby modified, shall henceforth read as follows:

Section 284. Allotment of Taxes. - Local government units shall have a share in the national taxes
based on the collection of the third fiscal year preceding the current fiscal year as follows:

(a) On the first year of the effectivity of this Code, thirty percent (30%);

(b) On the second year, thirty-five percent (35%); and

(c) On the third year and thereafter, forty percent (40%).

Provided, That in the event that the national government incurs an unmanageable public sector
deficit, the President of the Philippines is hereby authorized, upon the recommendation of Secretary
of Finance, Secretary of Interior and Local Government and Secretary of Budget and Management,
and subject to consultation with the presiding officers of both Houses of Congress and the
presidents of the "liga", to make the necessary adjustments in the allotment of local government
units but in no case shall the allotment be less than thirty percent (30%) of the collection of national
taxes of the third fiscal year preceding the current fiscal year; Provided, further, That in the first year
of the effectivity of this Code, the local government units shall, in addition to the thirty percent (30%)
allotment which shall include the cost of devolved functions for essential public services, be entitled
to receive the amount equivalent to the cost of devolved personal services.

The phrase "internal revenue" is likewise hereby DELETED from the related sections of Republic Act
No. 7160 (Local Government Code), specifically Section 285, Section 287, and Section 290, which
provisions shall henceforth read as follows:

Section 285. Allocation to Local Government Units. - The share of local government units in the
allotment shall be collected in the following manner:

(a) Provinces - Twenty-three percent (23%);

(b) Cities - Twenty-three percent (23%);

(c) Municipalities - Thirty-four percent (34%); and


(d) Barangays - Twenty percent (20%)

Provided, however, That the share of each province, city, and municipality shall be determined on
the basis of the following formula:

(a) Population -- Fifty percent (50%);

(b) Land Area-· Twenty-five percent (25%); and

(c) Equal sharing--Twenty-five percent (25%)

Provided, further. That the share of each barangay with a population of not less than one hundred
(100) inhabitants shall not be less than Eighty thousand (₱80,000.00) per annum chargeable against
the twenty percent (20%) share of the barangay from the allotment, and the balance to be allocated
on the basis of the following formula:

(a) On the first year of the effoctivity of this Code:

(1) Population - Forty percent (40%); and

(2) Equal sharing - Sixty percent (50%)

(b) On the second year:

(1) Population - Fifty percent (50%); and

(2) Equal sharing - Fifty percent (50%)

(c) On the third year and thereafter.

(1) Population - Sixty percent (60%); and

(2) Equal sharing - Forty percent (40%).

Provided, finally, That the financial requirements of barangays created by local government units
after the effectivity of this Code shall be the responsibility of the local government unit concerned.

xxxx

Sectfon 287. Local Development Projects. - Each local government unit shall appropriate in its
annual budget no less than twenty percent (20%) of its annual allotment for development projects.
Copies of the development plans of local government units shall be furnished the Department of
Interior and Local Government.

xxxx

Section 290. Amount of Share of Local Government Units. - Local government units shall, in
addition to the allotment, have a share of forty percent (40%) of the gross collection derived by the
national government from the preceding fiscal year from mining taxes, royalties, forestry and fishery
charges, and such other taxes, fees, or charges, including related surcharges, interests, or fines,
and from its share in any co-production, joint venture or production sharing agreement in the
utilization and development of the national wealth within their territorial jurisdiction.

Article 378, Article 379, Article 380, Article 382, Article 409, Article 461, and related provisions of the
Implementing Rules and Regulations of R.A. No. 7160 are hereby MODIFIED to reflect the deletion
of the phrase "internal revenue" as directed herein.

Henceforth, any mention of "Internal Revenue Allotment" or "IRA" in Republic Act No. 7160 (Local
Government Code) and its Implementing Rules and Regulations shall be understood as pertaining to
the allotment of the Local Government Units derived from the national taxes;

2. ORDERS the SECRETARY OF THE DEPARTMENT OF FINANCE; the SECRETARY OF THE


DEPARTMENT OF BUDGET AND MANAGEMENT; the COMMISSIONER OF INTERNAL
REVENUE; the COMMISSIONER OF CUSTOMS; and the NATIONAL TREASURER to
include ALL COLLECTIONS OF NATIONAL TAXES in the computation of the base of the just
share of the Local Government Units according to the ratio provided in the now-modified Section 284
of Republic Act No. 7160 (Local Government Code) except those accruing to special purpose funds
and special allotments for the utilization and development of the national wealth.

For this purpose, the collections of national taxes for inclusion in the base of the just share the Local
Government Units shall include, but shall not be limited to, the following:

(a) The national internal revenue taxes enumerated in Section 21 of the National Internal
Revenue Code, as amended, collected by the Bureau of Internal Revenue and the Bureau of
Customs;

(b) Tariff and customs duties collected by the Bureau of Customs;

(c) 50% of the value-added taxes collected in the Autonomous Region in Muslim Mindanao,
and 30% of all other national tax collected in the Autonomous Region in Muslim Mindanao.

The remaining 50% of the collections of value-added taxes and 70% of the collections of the
other national taxes in the Autonomous Region in Muslim Mindanao shall be the exclusive
share of the Autonomous Region in Muslim Mindanao pursuant to Section 9 and Section 15
of Republic Act No. 9054.

(d) 60% of the national taxes collected from the exploitation and development of the national
wealth.

The remaining 401% of the national taxes collected from the exploitation and development of
the national wealth shall exclusively accrue to the host Local Government Units pursuant to
Section 290 of Republic Act No. 7160 (Local Government Code);

(e) 85% of the excise taxes collected from locally manufactured Virginia and other tobacco
products.

The remaining 15% shall accrue to the special purpose funds created by Republic Act No.
7171 and Republic Act No. 7227;

(f) The entire 50% of the national taxes collected under Sections 106, 108 and 116 of the
NIRC as provided under Section 283 of the NIRC; and
(g) 5% of the 25% franchise taxes given to the National Government under Section 6 of
Republic Act No. 6631 and Section 8 of Republic Act No. 6632.

3. DECLARES that:

(a) The apportionment of the 25% of the franchise taxes collected from the Manila Jockey
Club and Philippine Racing Club, Inc. - that is, five percent (5%) to the National Government;
five percent (5%) to the host municipality or city; seven percent (7%) to the Philippine Charity
Sweepstakes Office; six percent (6%) to the Anti-Tuberculosis Society; and two percent (2%)
to the White Cross pursuant to Section 6 of Republic Act No. 6631 and Section 8 of Republic
Act No. 6632 - is VALID;

(b) Section 8 and Section 12 of Republic Act No. 7227


are VALID; and, ACCORDINGLY, the proceeds from the sale of the former military bases
converted to alienable lands thereunder are EXCLUDED from the computation of the
national tax allocations of the Local Government Units; and

(c) Section 24(3) of Presidential Decree No. 1445, in relation to Section 284 of the National
Internal Revenue Code, apportioning one-half of one percent (1/2of1%) of national tax
collections as the auditing fee of the Commission on Audit is VALID;

4. DIRECTS the Bureau of Internal Revenue and the Bureau of Customs and their deputized
collecting agents to certify all national tax collections, pursuant to Article 3 78 of the Implementing
Rules and Regulations of R.A. No. 7160;

5. DISMISSES the claims of the Local Government Units for the settlement by the National
Government of arrears in the just share on the ground that this decision shall have PROSPECTIVE
APPLICATION; and

6. COMMANDS the AUTOMATIC RELEASE WITHOUT NEED OF FURTHER ACTION of the just


shares of the Local Government Units in the national taxes, through their respective provincial, city,
municipal, or barangay treasurers, as the case may be, on a quarterly basis but not beyond five (5)
days from the end of each quarter, as directed in Section 6, Article X of the 1987 Constitution and
Section 286 of Republic Act No. 7160 (Local Government Code), and operationalized by Article 383
of the Implementing Rules and Regulations of RA 7160.

Let a copy of this decision be furnished to the President of the Republic of the Philippines, the
President of the Senate, and the Speaker of the House of Representatives for their information and
guidance.

SO ORDERED.

G.R. No. 161872             April 13, 2004

REV. ELLY CHAVEZ PAMATONG, ESQUIRE, petitioner, 


vs.
COMMISSION ON ELECTIONS, respondent.

RESOLUTION
TINGA, J.:

Petitioner Rev. Elly Velez Pamatong filed his Certificate of Candidacy for President on December 17,
2003. Respondent Commission on Elections (COMELEC) refused to give due course to
petitioner’s Certificate of Candidacy in its Resolution No. 6558 dated January 17, 2004. The
decision, however, was not unanimous since Commissioners Luzviminda G. Tancangco and Mehol
K. Sadain voted to include petitioner as they believed he had parties or movements to back up his
candidacy.

On January 15, 2004, petitioner moved for reconsideration of Resolution No.


6558. Petitioner’s Motion for Reconsideration was docketed as SPP (MP) No. 04-001. The
COMELEC, acting on petitioner’s Motion for Reconsideration and on similar motions filed by other
aspirants for national elective positions, denied the same under the aegis of Omnibus Resolution
No. 6604 dated February 11, 2004. The COMELEC declared petitioner and thirty-five (35) others
nuisance candidates who could not wage a nationwide campaign and/or are not nominated by a
political party or are not supported by a registered political party with a national constituency.
Commissioner Sadain maintained his vote for petitioner. By then, Commissioner Tancangco had
retired.

In this Petition For Writ of Certiorari, petitioner seeks to reverse the resolutions which were allegedly
rendered in violation of his right to "equal access to opportunities for public service" under Section
26, Article II of the 1987

Constitution,1 by limiting the number of qualified candidates only to those who can afford to wage a
nationwide campaign and/or are nominated by political parties. In so doing, petitioner argues that the
COMELEC indirectly amended the constitutional provisions on the electoral process and limited the
power of the sovereign people to choose their leaders. The COMELEC supposedly erred in
disqualifying him since he is the most qualified among all the presidential candidates, i.e., he
possesses all the constitutional and legal qualifications for the office of the president, he is capable
of waging a national campaign since he has numerous national organizations under his leadership,
he also has the capacity to wage an international campaign since he has practiced law in other
countries, and he has a platform of government. Petitioner likewise attacks the validity of the form for
the Certificate of Candidacy prepared by the COMELEC. Petitioner claims that the form does not
provide clear and reasonable guidelines for determining the qualifications of candidates since it does
not ask for the candidate’s bio-data and his program of government.

First, the constitutional and legal dimensions involved.

Implicit in the petitioner’s invocation of the constitutional provision ensuring "equal access to
opportunities for public office" is the claim that there is a constitutional right to run for or hold public
office and, particularly in his case, to seek the presidency. There is none. What is recognized is
merely a privilege subject to limitations imposed by law. Section 26, Article II of the Constitution
neither bestows such a right nor elevates the privilege to the level of an enforceable right. There is
nothing in the plain language of the provision which suggests such a thrust or justifies an
interpretation of the sort.

The "equal access" provision is a subsumed part of Article II of the Constitution, entitled "Declaration
of Principles and State Policies." The provisions under the Article are generally considered not self-
executing,2 and there is no plausible reason for according a different treatment to the "equal access"
provision. Like the rest of the policies enumerated in Article II, the provision does not contain any
judicially enforceable constitutional right but merely specifies a guideline for legislative or executive
action.3 The disregard of the provision does not give rise to any cause of action before the courts.4
An inquiry into the intent of the framers5 produces the same determination that the provision is not
self-executory. The original wording of the present Section 26, Article II had read, "The State shall
broaden opportunities to public office and prohibit public dynasties." 6 Commissioner (now Chief
Justice) Hilario Davide, Jr. successfully brought forth an amendment that changed the word
"broaden" to the phrase "ensure equal access," and the substitution of the word "office" to "service."
He explained his proposal in this wise:

I changed the word "broaden" to "ENSURE EQUAL ACCESS TO" because what is important
would be equal access to the opportunity. If you broaden, it would necessarily mean that
the government would be mandated to create as many offices as are possible to
accommodate as many people as are also possible. That is the meaning of broadening
opportunities to public service. So, in order that we should not mandate the State to
make the government the number one employer and to limit offices only to what may
be necessary and expedient yet offering equal opportunities to access to it, I change
the word "broaden."7 (emphasis supplied)

Obviously, the provision is not intended to compel the State to enact positive measures that would
accommodate as many people as possible into public office. The approval of the "Davide
amendment" indicates the design of the framers to cast the provision as simply enunciatory of a
desired policy objective and not reflective of the imposition of a clear State burden.

Moreover, the provision as written leaves much to be desired if it is to be regarded as the source of
positive rights. It is difficult to interpret the clause as operative in the absence of legislation since its
effective means and reach are not properly defined. Broadly written, the myriad of claims that can be
subsumed under this rubric appear to be entirely open-ended.8 Words and phrases such as "equal
access," "opportunities," and "public service" are susceptible to countless interpretations owing to
their inherent impreciseness. Certainly, it was not the intention of the framers to inflict on the people
an operative but amorphous foundation from which innately unenforceable rights may be sourced.

As earlier noted, the privilege of equal access to opportunities to public office may be subjected to
limitations. Some valid limitations specifically on the privilege to seek elective office are found in the
provisions9 of the Omnibus Election Code on "Nuisance Candidates" and COMELEC Resolution No.
645210 dated December 10, 2002 outlining the instances wherein the COMELEC may motu
proprio refuse to give due course to or cancel a Certificate of Candidacy.

As long as the limitations apply to everybody equally without discrimination, however, the equal
access clause is not violated. Equality is not sacrificed as long as the burdens engendered by the
limitations are meant to be borne by any one who is minded to file a certificate of candidacy. In the
case at bar, there is no showing that any person is exempt from the limitations or the burdens which
they create.

Significantly, petitioner does not challenge the constitutionality or validity of Section 69 of the
Omnibus Election Code and COMELEC Resolution No. 6452 dated 10 December 2003. Thus, their
presumed validity stands and has to be accorded due weight.

Clearly, therefore, petitioner’s reliance on the equal access clause in Section 26, Article II of the
Constitution is misplaced.

The rationale behind the prohibition against nuisance candidates and the disqualification of
candidates who have not evinced a bona fide intention to run for office is easy to divine. The State
has a compelling interest to ensure that its electoral exercises are rational, objective, and orderly.
Towards this end, the State takes into account the practical considerations in conducting elections.
Inevitably, the greater the number of candidates, the greater the opportunities for logistical
confusion, not to mention the increased allocation of time and resources in preparation for the
election. These practical difficulties should, of course, never exempt the State from the conduct of a
mandated electoral exercise. At the same time, remedial actions should be available to alleviate
these logistical hardships, whenever necessary and proper. Ultimately, a disorderly election is not
merely a textbook example of inefficiency, but a rot that erodes faith in our democratic institutions.
As the United States Supreme Court held:

[T]here is surely an important state interest in requiring some preliminary showing of a


significant modicum of support before printing the name of a political organization and its
candidates on the ballot – the interest, if no other, in avoiding confusion, deception and even
frustration of the democratic [process].11

The COMELEC itself recognized these practical considerations when it promulgated Resolution No.
6558 on 17 January 2004, adopting the study Memorandum of its Law Department dated 11
January 2004. As observed in the COMELEC’s Comment:

There is a need to limit the number of candidates especially in the case of candidates for
national positions because the election process becomes a mockery even if those who
cannot clearly wage a national campaign are allowed to run. Their names would have to be
printed in the Certified List of Candidates, Voters Information Sheet and the Official Ballots.
These would entail additional costs to the government. For the official ballots in automated
counting and canvassing of votes, an additional page would amount to more or less FOUR
HUNDRED FIFTY MILLION PESOS (₱450,000,000.00).

xxx[I]t serves no practical purpose to allow those candidates to continue if they cannot wage
a decent campaign enough to project the prospect of winning, no matter how slim.12

The preparation of ballots is but one aspect that would be affected by allowance of "nuisance
candidates" to run in the elections. Our election laws provide various entitlements for candidates for
public office, such as watchers in every polling place, 13 watchers in the board of canvassers,14 or
even the receipt of electoral contributions. 15Moreover, there are election rules and regulations the
formulations of which are dependent on the number of candidates in a given election.

Given these considerations, the ignominious nature of a nuisance candidacy becomes even more
galling. The organization of an election with bona fide candidates standing is onerous enough. To
add into the mix candidates with no serious intentions or capabilities to run a viable campaign would
actually impair the electoral process. This is not to mention the candidacies which are palpably
ridiculous so as to constitute a one-note joke. The poll body would be bogged by irrelevant minutiae
covering every step of the electoral process, most probably posed at the instance of these nuisance
candidates. It would be a senseless sacrifice on the part of the State.

Owing to the superior interest in ensuring a credible and orderly election, the State could exclude
nuisance candidates and need not indulge in, as the song goes, "their trips to the moon on
gossamer wings."

The Omnibus Election Code and COMELEC Resolution No. 6452 are cognizant of the compelling
State interest to ensure orderly and credible elections by excising impediments thereto, such as
nuisance candidacies that distract and detract from the larger purpose. The COMELEC is mandated
by the Constitution with the administration of elections16 and endowed with considerable latitude in
adopting means and methods that will ensure the promotion of free, orderly and honest
elections.17 Moreover, the Constitution guarantees that only bona fide candidates for public office
shall be free from any form of harassment and discrimination. 18 The determination of bona
fidecandidates is governed by the statutes, and the concept, to our mind is, satisfactorily defined in
the Omnibus Election Code.

Now, the needed factual premises.

However valid the law and the COMELEC issuance involved are, their proper application in the case
of the petitioner cannot be tested and reviewed by this Court on the basis of what is now before it.
The assailed resolutions of the COMELEC do not direct the Court to the evidence which it
considered in determining that petitioner was a nuisance candidate. This precludes the Court from
reviewing at this instance whether the COMELEC committed grave abuse of discretion in
disqualifying petitioner, since such a review would necessarily take into account the matters which
the COMELEC considered in arriving at its decisions.

Petitioner has submitted to this Court mere photocopies of various documents purportedly evincing
his credentials as an eligible candidate for the presidency. Yet this Court, not being a trier of facts,
can not properly pass upon the reproductions as evidence at this level. Neither the COMELEC nor
the Solicitor General appended any document to their respective Comments.

The question of whether a candidate is a nuisance candidate or not is both legal and factual. The
basis of the factual determination is not before this Court. Thus, the remand of this case for the
reception of further evidence is in order.

A word of caution is in order. What is at stake is petitioner’s aspiration and offer to serve in the
government. It deserves not a cursory treatment but a hearing which conforms to the requirements
of due process.

As to petitioner’s attacks on the validity of the form for the certificate of candidacy, suffice it to say
that the form strictly complies with Section 74 of the Omnibus Election Code. This provision
specifically enumerates what a certificate of candidacy should contain, with the required information
tending to show that the candidate possesses the minimum qualifications for the position aspired for
as established by the Constitution and other election laws.

IN VIEW OF THE FOREGOING, COMELEC Case No. SPP (MP) No. 04-001 is hereby remanded to
the COMELEC for the reception of further evidence, to determine the question on whether petitioner
Elly Velez Lao Pamatong is a nuisance candidate as contemplated in Section 69 of the Omnibus
Election Code.

The COMELEC is directed to hold and complete the reception of evidence and report its findings to
this Court with deliberate dispatch.

SO ORDERED.

G.R. No. 175210, February 01, 2016

MARIO JOSE E. SERENO, EXECUTIVE DIRECTOR OF THE ASSOCIATION OF PETROCHEMICAL


MANUFACTURERS OF THE PHILIPPINES, INC. (APMP), Petitioner, v. COMMITTEE ON TRADE AND
RELATED MATTERS (CTRM) OF THE NATIONAL ECONOMIC AND DEVELOPMENT AUTHORITY
(NEDA), COMPOSED OF THE DIRECTOR-GENERAL OF THE NEDA SECRETARIAT, THE EXECUTIVE
SECRETARY, THE SECRETARIES OF TRADE AND INDUSTRY, FINANCE, FOREIGN AFFAIRS,
AGRICULTURE, ENVIRONMENT AND NATURAL RESOURCES, BUDGET AND MANAGEMENT,
TRANSPORTATION AND COMMUNICATION, LABOR AND EMPLOYMENT, AGRARIAN REFORM, THE
GOVERNOR OF THE BANGKO SENTRAL NG PILIPINAS AND THE CHAIRMAN OF THE TARIFF
COMMISSION, AND BRENDA R. MENDOZA IN HER CAPACITY AS DIRECTOR OF THE TRADE,
INDUSTRY & UTILITIES STAFF, Respondents.

DECISION

BERSAMIN, J.:

The constitutional guarantee to information does not open every door to any and all information, but is
rather confined to matters of public concern. It is subject to such limitations as may be provided by law. The
State's policy of full public disclosure is restricted to transactions involving public interest, and is tempered
by reasonable conditions prescribed by law.

The Case

The petitioner appeals the decision rendered on October 16, 2006 by the Regional Trial Court (RTC), Branch
268, in Pasig City1 dismissing the petition for mandamus he had filed in his capacity as a citizen and as a
stakeholder in the Philippine petrochemical industry to compel respondent Committee on Tariff and Related
Matters (CTRM) to provide him a copy of the minutes of its May 23, 2005 meeting; as well as to provide
copies of all official records, documents, papers and government research data used as basis for the
issuance of Executive Order No. 486.2

Antecedents

On May 23, 2005, the CTRM, an office under the National Economic Development Authority (NEDA), held a
meeting in which it resolved to recommend to President Gloria Macapagal-Arroyo the lifting of the
suspension of the tariff reduction schedule on petrochemicals and certain plastic products, thereby reducing
the Common Effective Preferential Tariff (CEPT) rates on products covered by Executive Order (E.O.) No.
161 from 7% or 10% to 5% starting July 2005.3

On June 9, 2005, Wilfredo A. Paras (Paras), then the Chairman of the Association of Petrochemical
Manufacturers of the Philippines (APMP), the main industry association in the petrochemical sector, wrote to
the CTRM Secretariat, through its Director Brenda Mendoza (Director Mendoza), to request a copy of the
minutes of the meeting held on May 23, 2005.

Director Mendoza denied


chanRoblesvirtualLawlibrary
the request through her letter of June 20, 2005, 4 to wit:
With reference to your request for a copy of the minutes and resolution of the Committee on Tariff and
Related Matters (CTRM) meeting held on 23 May 2005, our Legal Staff advised that we cannot provide the
minutes of the meeting detailing the position and views of different CTRM member agencies. We may,
however, provide you
chanRoblesvirtualLawlibrary
with the action taken of the CTRM as follows:
"The CTRM agreed to reduce the CEPT rates on petrochemical resins and plastic products covered under EO
161 from 7% / 10% to 5% starting July 2005, and to revert the CEPT rates on these products to EO 161
levels once the proposed naphtha cracker plant is in commercial operation."
The CTRM has yet to confirm the minutes including the action taken during the said meeting since it has not
met after 23 May 2005.
The CTRM, again through Director Mendoza, sent a second letter dated August 31, 2005 as a response to
the series of
chanRoblesvirtualLawlibrary
letter-requests from the APMP, stating:
The CTRM during its meeting on 14 July 2005 noted that Section 3, Rule IV of the Implementing Rules and
Regulations of Republic Act 6713 or the Code of Conduct and Ethical Standards for Public Officials and
Employees provides that every department, office or agency shall provide official information, records or
documents to any requesting public (sic). However, the section also provides exceptions to the rules, such
as if '...(c) such information, record or document south (sic) falls within the concepts of established
privileged or recognized exceptions as may be provided by law or settled policy or jurisprudence...' The
acknowledged limitations to information access under Section 3 (c) include diplomatic correspondence,
closed-door Cabinet meetings and executive sessions of either House of Congress, as well as internal
deliberations of the Supreme Court (Chavez vs. Presidential Commission on Good Government, 299 SCRA
744)
The CTRM is of the view that the limitation pertaining to closed-door cabinet meetings under Section 3 (c) of
the IRR applies to the minutes of the meeting requested by APMP. In view thereof, the CTRM is constrained
[not] to provide the said minutes to the APMP.5ChanRoblesVirtualawlibrary

The APMP sent another letter-request dated October 27, 2005 to the CTRM through Director Mendoza
reminding about the legal implications of the refusal to furnish copies of the minutes as in violation of the
petitioner's Constitutional right of access to information on matters of public concern. However, the CTRM
continued to refuse access to the documents sought by the APMP. 6

The attitude of the CTRM prompted the petitioner and the APMP to bring the petition for mandamus in the
RTC to compel the CTRM to provide the copy of the minutes and to grant access to the minutes. The case
was docketed as SCA No. 2903.

The APMP, through Paras and Concepcion I. Tanglao, respectively its Chairman and President at the time,
sent letters dated December 12, 20057 and January 10, 20068 to the Office of the President (OP), stating the
reasons why the recommendation of the CTRM should be rejected, but the OP did not respond to the letters.

Thereafter, the petitioner filed an Urgent Motion for the Issuance of a Writ of Preliminary Mandatory
Injunction dated January 3, 2006, to which the respondent filed its Opposition dated January 26, 2006
and Motion to Dismiss dated February 16, 2006.9

Meanwhile, President Arroyo signed Executive Order No. 486, 10 dated January 12, 2006, to lift the
suspension of the tariff reduction on petrochemical resins and other plastic products under the ASEAN Free
Trade Area - Common Effective Preferential Tariff (AFTA-CEPT) Scheme. The relevant portions of E.O. No.
486
chanRoblesvirtualLawlibrary
read:
WHEREAS, Executive Order 234 dated 27 April 2000, which implemented the 2000-2003 Philippine
schedule of tariff reduction of products transferred from the Temporary Exclusion List and the Sensitive List
to the Inclusion List of the accelerated CEPT Scheme for the AFTA, provided that the CEPT rates on
petrochemicals and certain plastic products will be reduced to 5% on 01 January 2003;

WHEREAS, Executive Order 161 issued on 9 January 2003 provides for the suspension of the application of
the tariff reduction schedule on petrochemicals and certain products in 2003 and 2004 only;

WHEREAS, the government recognizes the need to provide an enabling environment for the naphtha
cracker plant to attain international competitiveness;

WHEREAS, the NEDA Board approved the lifting of the suspension of the aforesaid tariff reduction schedule
on petrochemicals and certain plastic products and the reversion of the CEPT rates on these products to EO
161 (s.2003) levels once the naphtha cracker plant is in commercial operation;

NOW, THEREFORE, I, GLORIA MACAPAGAL-ARROYO, President of the Republic of the Philippines,


pursuant to the powers vested in me under Section 402 of the Tariff and Customs Code of 1978 (Presidential
Decree No. 1464), as amended, do hereby order:

SECTION 1. The articles specifically listed in Annex "A" (Articles Granted Concession under the CEPT
Scheme for the AFT A) hereof, as classified under Section 104 of the Tariff and Customs Code of 1978, as
amended shall be subject to the ASEAN CEPT rates in accordance with the schedule indicated in Column 4
of Annex "A". The ASEAN CEPT rates so indicated shall be accorded to imports coming from ASEAN Member
States applying CEPT concession to the same product pursuant to Article 4 of the CEPT Agreement and its
Interpretative Notes.
In its order of May 9, 2006, the RTC denied the Urgent Motion for the Issuance of a Writ of Preliminary
Mandatory Injunction but directed the parties to file their respective memorandums after noting that the
controversy involved a pure question of law.11

Subsequently, the RTC rendered its assailed decision on October 16, 200612 dismissing the petition
for mandamus for lack of merit. It relied on the relevant portions of Section 3 of Rule IV of the
Implementing Rules and Regulations of R.A. No. 6713 (Code of Conduct and Ethical Standards for Public
Officials and
chanRoblesvirtualLawlibrary
Employees), to wit:
Sec 3. Every department, office or agency shall provide official information, records and documents to any
requesting public except if:
x x x x

(c) the information, record or document sought falls within the concepts of established privilege or
recognized exceptions as may be provided by law or settled policy or jurisprudence;

(d) such information, record or document comprises drafts or decisions, orders, rulings, policies,
memoranda, etc.
and relevant portions
chanRoblesvirtualLawlibrary
of Section 7 (c) of the same law, viz.:
Section 7. Prohibited Acts and Transactions. - In addition to acts and omissions of public officials and
employees now prescribed in the Constitution and existing laws, the following shall constitute prohibited acts
and transactions of any public official and employee and are hereby declared unlawful:

x x x x

(c) Disclosure and/or misuse of confidential information - Public officials and employees shall not use or
divulge confidential or classified information officially known to them by reason of their office and not made
available to the public either:

x x x x

(2) To the prejudice of public interest.13 ChanRoblesVirtualawlibrary

The RTC declared that the "CTRM is an advisory body composed of various department heads or secretaries
and is classified as cabinet meetings and inter-agency communications;" 14 and that the record of the
communications of such body "falls under the category of privileged information because of the sensitive
subject matter which could seriously affect public interest." 15

Hence, this appeal directly to the Court on questions of law.16

Issues

The
chanRoblesvirtualLawlibrary
petitioner submits the following issues for resolution, namely:

I. Are meetings of the CTRM and the minutes thereof exempt from the Constitutional right of
access to information?

II. Assuming arguendo that the minutes of CTRM meetings are privileged or confidential, is


such privilege or confidentiality absolute?

III. Can privilege or confidentiality be invoked to evade public accountability, or worse, to cover
up incompetence and malice?17

In short, the issue is whether or not the CTRM may be compelled by mandamus to furnish the petitioner
with a copy of the minutes of the May 23, 2005 meeting based on the constitutional right to information on
matters of public concern and the State's policy of full public disclosure. The request for information was
motivated by his desire to understand the basis for the CTRM's recommendation that allegedly caused
tremendous losses to the petrochemical industry through the issuance of E.O. No. 486.

In seeking the nullification of the assailed decision of the RTC, and the consequent release of the minutes
and the disclosure of all official records, documents, papers and government research data used as the basis
for the issuance of E.O. No. 486, the petitioner invokes the following provisions of the 1987 Constitution and
R.A. No. 6713, thusly:

Section 28
chanRoblesvirtualLawlibrary
of Article II of the 1987 Constitution:
Section 28. Subject to reasonable conditions prescribed by law, the State adopts and implements a policy of
full public disclosure of all its transactions involving public interest.
Section 7 of
chanRoblesvirtualLawlibrary
Article III of the 1987 Constitution:
Section 7. The right of the people to information on matters of public concern shall be recognized. Access to
official records, and to documents, and papers pertaining to official acts, transactions, or decisions, as well
as to government research data used as basis for policy development, shall be afforded the citizen, subject
to such limitations as may be provided by law.
Section 1
chanRoblesvirtualLawlibrary
of Article XI of the 1987 Constitution:
Section 1. Public office is a public trust. Public officers and employees must at all times be accountable to
the people, serve them with utmost responsibility, integrity, loyalty, and efficiency, act with patriotism and
justice, and lead modest lives.
Section 5
chanRoblesvirtualLawlibrary
of R.A. No. 6713:
Section 5. Duties of Public Officials and Employees. - In the performance of their duties, all public officials
and employees
chanRoblesvirtualLawlibrary
are under obligation to:
x x x x

(e) Make documents accessible to the public. - All public documents must be made accessible to, and readily
available for inspection by, the public within reasonable working hours.
Ruling of the Court

The dismissal of the petition for mandamus by the RTC is affirmed.

The constitutional guarantee of the right to information on matters of public concern enunciated in Section 7
of Article III of the 1987 Constitution complements the State's policy of full public disclosure in all
transactions involving public interest expressed in Section 28 of Article II of the 1987 Constitution. These
provisions are aimed at ensuring transparency in policy-making as well as in the operations of the
Government, and at safeguarding the exercise by the people of the freedom of expression. In a democratic
society like ours, the free exchange of information is necessary, and can be possible only if the people are
provided the proper information on matters that affect them. But the people's right to information is not
absolute. According to Legaspi v. Civil Service Commission, 18 the constitutional guarantee to information
"does not open every door to any and all information."19 It is limited to matters of public concern, and is
subject to such limitations as may be provided by law.20 Likewise, the State's policy of full public disclosure
is restricted to transactions involving public interest, and is further subject to reasonable conditions
prescribed by law.21

Two requisites must concur before the right to information may be compelled by writ of mandamus. Firstly,
the information sought must be in relation to matters of public concern or public interest. And, secondly, it
must not be exempt by law from the operation of the constitutional guarantee.

As to the first requisite, there is no rigid test in determining whether or not a particular information is of
public concern or public interest.22 Both terms cover a wide-range of issues that the public may want to be
familiar with either because the issues have a direct effect on them or because the issues "naturally arouse
the interest of an ordinary citizen."23 As such, whether or not the information sought is of public interest or
public concern is left to the proper determination of the courts on a case to case basis.

In his capacity as a citizen and as the Executive Director of the APMP, the petitioner has sought to obtain
official information dealing with the policy recommendation of the CTRM with respect to the reduction of
tariffs on petrochemical resins and plastic products. He has asserted that the recommendation, which would
be effected through E.O. No. 486, not only brought significant losses to the petrochemical industry that
undermined the industry's long-term viability and survival, but also conflicted with official government
pronouncements, policy directives, and enactments designed to support and develop an integrated
petrochemical industry. He has claimed that the implementation of E.O. No. 486 effectively deprived the
industry of tariff support and market share, thereby jeopardizing large investments without due process of
law.24

The Philippine petrochemical industry centers on the manufacture of plastic and other related materials, and
provides essential input requirements for the agricultural and industrial sectors of the country. Thus, the
position of the petrochemical industry as an essential contributor to the overall growth of our country's
economy easily makes the information sought a matter of public concern or interest.

The second requisite is that the information requested must not be excluded by law from the constitutional
guarantee. In that regard, the Court has already declared that the constitutional guarantee of the people's
right to information does not cover national security matters and intelligence information, trade secrets and
banking transactions and criminal matters. 25 Equally excluded from coverage of the constitutional guarantee
are diplomatic correspondence, closed-door Cabinet meeting and executive sessions of either house of
Congress, as well as the internal deliberations of the Supreme Court.26 In Chavez v. Public Estates
Authority,27 the Court has ruled that the right to information does not extend to matters acknowledged as
"privileged information under the separation of powers," which include "Presidential conversations,
correspondences, or discussions during closed-door Cabinet meetings."28Likewise exempted from the right to
information are "information on military and diplomatic secrets, information affecting national security, and
information on investigations of crimes by law enforcement agencies before the prosecution of the
accused."29

The respondents claim exemption on the ground that the May 23, 2005 meeting was classified as a closed-
door Cabinet meeting by virtue of the committee's composition and the nature of its mandate dealing with
matters of foreign affairs, trade and policy-making. They assert that the information withheld was within the
scope of the exemption from disclosure because the CTRM meetings were directly related to the exercise of
the sovereign prerogative of the President as the Head of State in the conduct of foreign affairs and the
regulation of trade, as provided in Section 3 (a) of Rule IV of the Rules Implementing R.A. No. 6713.30

The authority of the CTRM as the advisory body of the President and the NEDA is set forth in E.O. No. 230,
series of 1987 (Reorganization Act of the National Economic and Development Authority), to wit:
chanRoblesvirtualLawlibrary

SECTION 6. National Economic and Development Authority Inter-agency Committees. - To assist the NEDA
Board in the performance ol its functions, there are hereby created the following committees which shall
nereafter be under the direct control of the NEDA Board and shall submit all their recommendations to the
President for approval on matters involving their respective concerns. The Chairman of these committees
shall be designated by the President. The NEDA Board shall likewise determine where the technical staff of
the said committees shall be based.

xxxx

(e) Committee on Tariff and Related Matters (TRM) - The TRM to be composed of the Director-General of the
National Economic and Development Authority Secretariat, the Executive Secretary, the Secretaries of Trade
and Industry, Foreign Affairs, Agriculture, Environment and Natural Resources and of Budget and
Management, the Governor of the Central Bank and the Chairman of the Tariff Commission shall have the
following
chanRoblesvirtualLawlibrary
functions:
(i) Advise the President and the NEDA Board on tariff and related matters, and on the effects on the country
of various international developments;

(ii) Coordinate agency positions and recommend national positions for international economic negotiations;

(iii) Recommend to the President a continuous rationalization program for the country's tariff structure.
(underlining supplied)
The respondents are correct. It is always necessary, given the highly important and complex powers to fix
tariff rates vested in the President,31 that the recommendations submitted for the President's consideration
be well-thought out and well-deliberated. The Court has expressly recognized in Chavez v. Public Estates
Authority32 that "a frank exchange of exploratory ideas and assessments, free from the glare of publicity and
pressure by interested parties, is essential to protect the independence of decision-making of those tasked
to exercise Presidential, Legislative and Judicial power." In Almonte v. Vasquez33the Court has stressed the
need for confidentiality and privacy, stating thusly: "A President and those who assist him must be free to
explore alternatives in the process of shaping policies and making decisions and to do so in a way many
would be unwilling to express except privately." 34 Without doubt, therefore, ensuring and promoting the free
exchange of ideas among the members of the committee tasked to give tariff recommendations to the
President were truly imperative.

Every claim of exemption, being a limitation on a right constitutionally granted to the people, is liberally
construed in favor of disclosure and strictly against the claim of confidentiality. However, the claim of
privilege as a cause for exemption from the obligation to disclose information must be clearly asserted by
specifying the grounds for the exemption.35 In case of denial of access to the information, it is the
government agency concerned that has the burden of showing that the information sought to be obtained is
not a matter of public concern, or that the same is exempted from the coverage of the constitutional
guarantee.36 We reiterate, therefore, that the burden has been well discharged herein.

The respondents further assert that the information sought fell within the concept of established privilege
provided by jurisprudence under Section 3 (c) of Rule IV of the Rules Implementing R.A. No. 6713, the May
23, 2005 meeting being regarded as a closed-door Cabinet meeting. 37 The petitioner, disagreeing, posits
that R.A. No. 6713, by itself, neither provides exceptions to the constitutional right to information nor
specifies limitations on the State policy of full public disclosure; that the Implementing Rules and
Regulations went beyond the scope of R.A. No. 6713 in providing exceptions not covered by the law; that
the alleged closed-door Cabinet meeting exception, so as to fall within the ambit of Section 3(c) of the Rules
Implementing R.A. No. 6713, was not established under settled policy or jurisprudence; that the reliance on
the rulings in Chavez v. PCGG and Chavez v. PEA-Amari that declared the closed-door Cabinet meeting as
an exception to the right to information was misplaced considering that the exception was not squarely in
issue in those cases; that the pronouncement could only be regarded as obiter dicta; that the closed-door
Cabinet meeting exception, assuming though not admitting the same to have been established by law or
settled jurisprudence, could not be automatically applied to all the CTRM meetings because the CTRM was
different from the Cabinet inasmuch as two of its members, namely, the Governor of the Bangko Sentral ng
Pilipinas and the Chairman of the Tariff Commission, were not members of the President's Cabinet; and that
the deliberations of the CTRM as a body merely akin to the Cabinet could not be given the privilege and
confidentiality not expressly provided for by law or jurisprudence, most especially considering that only by
legislative enactment could the constitutional guarantee to the right to information be restricted.

We cannot side with the petitioner.

In Senate of the Philippines v. Ermita,38 we have said that executive privilege is properly invoked in relation
to specific categories of information, not to categories of persons. As such, the fact that some members of
the committee were not part of the President's Cabinet was of no moment. What should determine whether
or not information was within the ambit of the exception from the people's right to access to information
was not the composition of the body, but the nature of the information sought to be accessed. A different
holding would only result to the unwanted situation wherein any concerned citizen, like the petitioner,
invoking the right to information on a matter of public concern and the State's policy of full public disclosure,
could demand information from any government agency under all conditions whenever he felt aggrieved by
the decision or recommendation of the latter.

In case of conflict, there is a need to strike a balance between the right of the people and the interest of the
Government to be protected. Here, the need to ensure the protection of the privilege of non-disclosure is
necessary to allow the free exchange of ideas among Government officials as well as to guarantee the well-
considered recommendation free from interference of the inquisitive public. chanrobleslaw

WHEREFORE, the Court DENIES the petition for review on certiorari; and AFFIRMS the decision of the
Regional Trial Court in Special Civil Action No. 2903, without pronouncement on costs of suit.

SO ORDERED. cralawlawlibrary

ART. IV CITIZENSHIP

G.R. No. 128195           October 3, 2001

ELIZABETH LEE and PACITA YU LEE, HON. JUDGE JOSE D. ALOVERA,* Presiding Judge,
Regional Trial Court, Branch 17, Roxas City, THE REGISTER OF DEEDS OF ROXAS
CITY, petitioners, 
vs.
REPUBLIC OF THE PHILIPPINES, represented by THE DIRECTOR OF LANDS AND THE
ADMINISTRATOR, LAND REGISTRATION AUTHORITY and THE HON. COURT OF
APPEALS,* respondents.

PARDO, J.:
The case under consideration is a petition for review on certiorari of the decision1 of the court of
appeals nullifying that of the Regional Trial Court, Roxas City, in Reconstitution case No. R-
1928,2 pertaining to lot 398, Capiz Cadastre, covered by Original Certificate of Title No. 3389.

Sometime in March 1936, Carmen, Francisco, Jr., Ramon, Lourdes, Mercedes, Concepcion,
Mariano, Jose, Loreto, Manuel, Rizal, and Jimmy, alll surnamed Dinglasan sold to Lee Liong, A
Chinese citizen, a parcel of land with an approximate area of 1,631 square meters, designed as lot
398 and covered by Original Certificate of Title No. 3389, situated at the corner of Roxas Avenue
and Pavia Street, Roxas City.3

However, in 1948, the former owners filed with the Court of First Instance, Capiz an action against
the heirs of Lee Liong for annulment of sale and recovery of land.4 The plaintiffs assailed the validity
of the sale because of the constitutional prohibition against aliens acquiring ownership of private
agriculture land, including residential, commercial or industrial land. Rebuffed in the trial court and
the Court of Appeals, plaintiffs appealed to the Supreme Court. On June 27, 1956, the Supreme
Court ruled thus:

"granting the sale to be null and void and can not give title to the vendee, it does not
necessarily follow therefrom that title remained in the vendor, who had also violated the
constitutional prohibition, or that he (vendor) has the right to recover the title of which he has
divested himself by his in ignoring the prohibition. In such contingency another principle of
law sets in to bar the equally guilty vendor from recovering the title which he had voluntarily
conveyed for a consideration, that of pari delicto."5

On July 1, 1968, the same former owners Rafael A. Dinglasan, together with Francisco, Carmen,
Ramon, Lourdes, Mercedes, Concepcion, Mariano, Jose, Loreto, Rizal, Jimmy, and Jesse Dinglasan
filed with the Court of First Instance, Capiz an action for recovery of the same parcel of land. 6 Citing
the case of the Philippines Banking Corporation v. Lui She,7 they submitted that the sale to Lee Liong
was null and void for being violative of the Constitution. On September 23, 1968, the heirs of Lee
Liong filed with the trial court a motion to dismiss the case on the ground of res judicata.8 On October
10, 1968, and November 9, 1968, the trial court denied the motion.9 The heirs of Lee Liong elevated
the case to the Supreme Court by petition for certiorari. On April 22, 1977, the Supreme Court
annulled the orders of the trial court and directed it to dismiss the case, holding that the suit was
barred by res judicata.10

On September 7, 1993, Elizabeth Manuel-Lee and Pacita Yu Lee filed with the regional Trial Court,
Roxas City a petition for reconstitution of title of Lot No. 398 of the Capiz Cadastre, formerly covered
by Original Certificate of title No. 3389 of the register of Deeds of Roxas City. 11 Petitioners alleged
that they were the windows of the deceased Lee Bing Hoo and Lee Bun Ting , who were the heirs of
Lee Liong, The owner of the lot. Lee Liong died intestate in February 1944. On June 30, 1947, Lee
Liong's widow, Ang Chia, and his two sons, Lee Bun Ting and Lee Bing Ho, executed an extra
judicial settlement of the state of Lee Liong, adjudicating to themselves the subject parcel of
land.12 Petitioner Elizabeth Lee acquired her share in lot No. 398 through an extra-judicial settlement
and donation executed in her favor by her deceased husband Lee Bong Hoo. Petitioner Pacita Yu
Lee acquired her share in the same lot by succession from her deceased husband Lee Bun Ting, as
evidenced by a deed of extra-judicial settlement.13

Previously, on December 9, 1948, the Register of Deeds, Capiz Salvador Villaluz, issued a
certification that a transfer certificate of title over the property was issued in the name of Lee
Liong.14 However, the records of the Register of Deeds, Roxas City were burned during the war.
Thus, as heretofore stated, on September 7, 1968, petitioners filed a petition for reconstitution of
title.
1âwphi1.nêt
On June 10, 1994, the Regional Trial Court, Roxas City, Branch 17, ordered the reconstitution of the
lost or destroyed certificate of title in the name of Lee Liong on the basis of an approved plan and
technical description.15The dispositive portion of the trial Court's decision reads thus:

"WHEREFORE, in reiteration the Register of Deeds for the City of Roxas is ordered to
reconstitute the lost or destroyed certificate of title in the name Lee Liong, deceased, of
Roxas City, with all the conditions stated in paragraph 2 of this decision. This decision shall
become final after the lapse of thirty (30) days from receipt by the Register of Deeds and by
the Commissioner of LRA of a notice of such judgement without any appeal having been
filed by any of such officials.

"SO ORDERED.

"Given at Roxas City, Philippines,

"June 10, 1994.

"JOSE O. ALOVERA
"Judge"16

On August 18, 1994, the Clerk of Court, Regional Trial Court, Roxas City, Branch 17 issued an Entry
of Judgement.17

On January 25, 1995, the Solicitor General filed with the Court of Appeals a petition for annulment of
judgement in Reconstitution Case No. 1928, alleging that the Regional Trial Court, Roxas City had
no jurisdiction over the case.18The Solicitor General contended that the petitioners were not the
proper parties in the reconstitution of title, since their predecessor-in-interest Lee Liong did not
acquire title to the lot because he was a Chinese citizen and was constitutionally not qualified to own
the subject land.

On April 30, 1996, the Court of Appeals promulgated its decision declaring the judgement of
reconstitution void.19

On May 24, 1996, Elizabeth Manuel-Lee and Pacita Yu Lee filed with the Court of Appeals a motion
for reconsideration of the decision.  20 On February 18, 19976, the Court of appeals denied the
motion.21

Hence this petition.22

Petitioners submitted that the Solicitor General was estopped from seeking annulment of the
judgement of reconstitution after failing to object during the reconstitution proceedings before the
trial court, despite due notice. Petitioners alleged that the Solicitor General merely acted on the
request of private and politically powerful individuals who wished to capitalize on the prime location
of the subject land.

Petitioners emphasized that the ownership of the land had been settled in two previous cases of the
Supreme Court, where the Court ruled in favor of their predecessor-in-interest, Lee Liong.
Petitioners also pointed out that they acquired ownership of the land through actual possession of
the lot and their consistent payment of taxes over the land for more than sixty years.
On the other hand, the Solicitor General submitted that the decision in the reconstitution case was
void; otherwise, it would amount to circumventing the constitutional proscription against aliens
acquiring ownership of private or public agricultural lands.

We grant the petition.

The reconstitution of a certificate of title denotes restoration in the original form and condition of a
lost or destroyed instrument attesting the title of a person to a piece of land23. The purpose of the
reconstruction of title is to have, after observing the procedures prescribed by law, the title
reproduced in exactly the same way it has been when the loss or destruction occurred.24

In this case, petitioners sought a reconstitution of title in the name of Lee Liong, alleging that the
transfer of certificate of title issued to him was lost or destroyed during World War II. All the
documents recorded and issued by the Register of Deed, Capiz, which include the transfer
certificate of title issued in the name of Lee Liong, were all destroyed during the war. The fact that
the original of the transfer certificate of title was not in the files of the Office of the Register of Deeds
did not imply that a transfer certificate of title had not been issued.25 In the trial court proceeding,
petitioners presented evidence proving the sale of the land from the Dinglasans to Lee Liong and the
latter's subsequent possessions of the of the property in the concept of owner. Thus, the trial court
after examining all the evidence before it, ordered the reconstruction of title in the name of Lee
Liong.

However, there is a question as to whether Lee Liong as the qualification to own land in the
Philippines.

The sale of the land in question was consummated sometime in March 1936, during the effectivity of
the 1935 Constitution. Under the 1935 Constitution,26 aliens could not acquire private agricultural
lands, save in cases of hereditary succession. 27 Thus, Lee Liong, a chinese citizen, was disqualified
to acquire the land in question.28

The fact that the Court did not annul the sale of the land to an alien did not validate the transaction,
for it was still contrary to the constitutional proscription against aliens acquiring lands of the public or
private domain. However, the proper party to assail the illegality of the transaction was not the
parties to the transaction.29 "In sales of real estate to aliens incapable of holding title thereto by virtue
of the provisions of the Constitution both the vendor and the vendee are deemed to have committed
the constitutional violation and being thus in pari delicto the courts will not afford protection to either
party."30 The proper party to assail the sale is the Solicitor General. This was what was done in this
case when the Solicitor General initiated an action for annulment of judgment of reconstitution of
title. While it took the Republic more than sixty years to assert itself, it is not barred from initiating
such action. Prescription never against the State.31

Although ownership of the land cannot revert to the original sellers, because of the doctrine of pari
delicto, the Solicitor General may initiate an action for reversion or escheat of the land to the State,
subject to other defenses, as hereafter set forth.32

In this case, subsequent circumstances militate against escheat proceedings because the land is
now in the hands of Filipinos. The original vendee, Lee Liong, has since died and the land has been
inherited by his heirs and subsequently their heirs, petitioners herein. Petitioners are Filipino citizens,
a fact the Solicitor General does not dispute.

The constitutional proscription on alien ownership of lands of the public or private domain was
intended to protect lands from falling in the hands of non-Filipinos. In this case, however, there
would be no more public policy violated since the land is in the hands of Filipinos qualified to acquire
and own such land. "If land is invalidly transferred to an alien who subsequently becomes a citizen or
transfers it to a citizen, the flaw in the original transaction is considered cured and the title of the
transferee is rendered valid.33 Thus, the subsequent transfer of the property to qualified Filipinos may
no longer be impugned on the basis of the invalidity of the initial transfer. 34 The objective of the
constitutional provision to keep our lands in Filipino hands has been achieved.

Incidentally, it must be mentioned that reconstitution of the original certificate of title must be based
on an owner's duplicate, secondary evidence thereof, or other valid sources of the title be
reconstituted.35 In this case, reconstitution was based on the plan and technical description approved
by the Land Registration Authority.36 This renders the order of reconstitution void for lack of factual
support.37 A judgment with absolute nothing to support it is void.38

As earlier mentioned, a reconstitution of the title is the reissuance of a new certificate of title lost or
destroyed in its original form and condition.39 It does not pass upon the ownership of the land
covered by the lost or destroyed title.40 Any change in the ownership of the property must be the
subject of a separate suit.41 Thus, although petitioners are in possession of the land, a separate
proceedings is necessary to thresh out the issue of ownership of the land.

WHEREFORE, the Court REVERSES and SETS ASIDE the decision of the Court of Appeals in CA-


G. R. SP No. 36274. In lieu thereof, the Court sets aside the order of reconstitution of title in
Reconstitution Case No. R-1928, Regional Trial Court, Roxas City, and dismisses the petition,
without prejudice. 1âwphi1.nêt

No Cost.

SO ORDERED.

G.R. No. 187567               February 15, 2012

THE REPUBLIC OF THE PHILIPPINES, Petitioner, 


vs.
NORA FE SAGUN, Respondent.

DECISION

VILLARAMA, JR., J.:

Before us is a petition for review on certiorari filed by the Solicitor General on behalf of the Republic
of the Philippines, seeking the reversal of the April 3, 2009 Decision of the Regional Trial Court

(RTC), Branch 3, of Baguio City in Spcl. Pro. Case No. 17-R. The RTC granted the petition filed by 2 

respondent Nora Fe Sagun entitled "In re: Judicial Declaration of Election of Filipino Citizenship,
Nora Fe Sagun v. The Local Civil Registrar of Baguio City."

The facts follow:

Respondent is the legitimate child of Albert S. Chan, a Chinese national, and Marta Borromeo, a
Filipino citizen. She was born on August 8, 1959 in Baguio City and did not elect Philippine

citizenship upon reaching the age of majority. In 1992, at the age of 33 and after getting married to
Alex Sagun, she executed an Oath of Allegiance to the Republic of the Philippines. Said document

was notarized by Atty. Cristeta Leung on December 17, 1992, but was not recorded and registered
with the Local Civil Registrar of Baguio City.

Sometime in September 2005, respondent applied for a Philippine passport. Her application was
denied due to the citizenship of her father and there being no annotation on her birth certificate that
she has elected Philippine citizenship. Consequently, she sought a judicial declaration of her
election of Philippine citizenship and prayed that the Local Civil Registrar of Baguio City be ordered
to annotate the same on her birth certificate.

In her petition, respondent averred that she was raised as a Filipino, speaks Ilocano and Tagalog
fluently and attended local schools in Baguio City, including Holy Family Academy and the Saint
Louis University. Respondent claimed that despite her part-Chinese ancestry, she always thought of
herself as a Filipino. She is a registered voter of Precinct No. 0419A of Barangay Manuel A. Roxas
in Baguio City and had voted in local and national elections as shown in the Voter
Certification issued by Atty. Maribelle Uminga of the Commission on Elections of Baguio City.

She asserted that by virtue of her positive acts, she has effectively elected Philippine citizenship and
such fact should be annotated on her record of birth so as to entitle her to the issuance of a
Philippine passport.

On August 7, 2007, the Office of the Solicitor General (OSG) entered its appearance as counsel for
the Republic of the Philippines and authorized the City Prosecutor of Baguio City to appear in the
above mentioned case. However, no comment was filed by the City Prosecutor.

After conducting a hearing, the trial court rendered the assailed Decision on April 3, 2009 granting
the petition and declaring respondent a Filipino citizen. The fallo of the decision reads:

WHEREFORE, the instant petition is hereby GRANTED. Petitioner Nora Fe Sagun y Chan is hereby
DECLARED [a] FILIPINO CITIZEN, having chosen or elected Filipino citizenship.

Upon payment of the required fees, the Local Civil Registrar of Baguio City is hereby directed to
annotate [on] her birth certificate, this judicial declaration of Filipino citizenship of said petitioner.

IT IS SO ORDERED. 7

Contending that the lower court erred in so ruling, petitioner, through the OSG, directly filed the
instant recourse viaa petition for review on certiorari before us. Petitioner raises the following issues:

Whether or not an action or proceeding for judicial declaration of Philippine citizenship is


procedurally and jurisdictionally permissible; and,

II

Whether or not an election of Philippine citizenship, made twelve (12) years after reaching
the age of majority, is considered to have been made "within a reasonable time" as
interpreted by jurisprudence. 8
Petitioner argues that respondent’s petition before the RTC was improper on two counts: for one,
law and jurisprudence clearly contemplate no judicial action or proceeding for the declaration of
Philippine citizenship; and for another, the pleaded registration of the oath of allegiance with the
local civil registry and its annotation on respondent’s birth certificate are the ministerial duties of the
registrar; hence, they require no court order. Petitioner asserts that respondent’s petition before the
trial court seeking a judicial declaration of her election of Philippine citizenship undeniably entails a
determination and consequent declaration of her status as a Filipino citizen which is not allowed
under our legal system. Petitioner also argues that if respondent’s intention in filing the petition is
ultimately to have her oath of allegiance registered with the local civil registry and annotated on her
birth certificate, then she does not have to resort to court proceedings.

Petitioner further argues that even assuming that respondent’s action is sanctioned, the trial court
erred in finding respondent as having duly elected Philippine citizenship since her purported election
was not in accordance with the procedure prescribed by law and was not made within a "reasonable
time." Petitioner points out that while respondent executed an oath of allegiance before a notary
public, there was no affidavit of her election of Philippine citizenship. Additionally, her oath of
allegiance which was not registered with the nearest local civil registry was executed when she was
already 33 years old or 12 years after she reached the age of majority. Accordingly, it was made
beyond the period allowed by law.

In her Comment, respondent avers that notwithstanding her failure to formally elect Filipino

citizenship upon reaching the age of majority, she has in fact effectively elected Filipino citizenship
by her performance of positive acts, among which is the exercise of the right of suffrage. She claims
that she had voted and participated in all local and national elections from the time she was of legal
age. She also insists that she is a Filipino citizen despite the fact that her "election" of Philippine
citizenship was delayed and unregistered.

In reply, petitioner argues that the special circumstances invoked by respondent, like her continuous
10 

and uninterrupted stay in the Philippines, her having been educated in schools in the country, her
choice of staying here despite the naturalization of her parents as American citizens, and her being a
registered voter, cannot confer on her Philippine citizenship as the law specifically provides the
requirements for acquisition of Philippine citizenship by election.

Essentially, the issues for our resolution are: (1) whether respondent’s petition for declaration of
election of Philippine citizenship is sanctioned by the Rules of Court and jurisprudence; (2) whether
respondent has effectively elected Philippine citizenship in accordance with the procedure
prescribed by law.

The petition is meritorious.

At the outset, it is necessary to stress that a direct recourse to this Court from the decisions, final
resolutions and orders of the RTC may be taken where only questions of law are raised or involved.
There is a question of law when the doubt or difference arises as to what the law is on a certain state
of facts, which does not call for an examination of the probative value of the evidence presented by
the parties-litigants. On the other hand, there is a question of fact when the doubt or controversy
arises as to the truth or falsity of the alleged facts. Simply put, when there is no dispute as to fact,
the question of whether the conclusion drawn therefrom is correct or not, is a question of law. 11

In the present case, petitioner assails the propriety of the decision of the trial court declaring
respondent a Filipino citizen after finding that respondent was able to substantiate her election of
Filipino citizenship. Petitioner contends that respondent’s petition for judicial declaration of election
of Philippine citizenship is procedurally and jurisdictionally impermissible. Verily, petitioner has
raised questions of law as the resolution of these issues rest solely on what the law provides given
the attendant circumstances.

In granting the petition, the trial court stated:

This Court believes that petitioner was able to fully substantiate her petition regarding her election of
Filipino citizenship, and the Local Civil Registrar of Baguio City should be ordered to annotate in her
birth certificate her election of Filipino citizenship. This Court adds that the petitioner’s election of
Filipino citizenship should be welcomed by this country and people because the petitioner has the
choice to elect citizenship of powerful countries like the United States of America and China,
however, petitioner has chosen Filipino citizenship because she grew up in this country, and has
learned to love the Philippines. Her choice of electing Filipino citizenship is, in fact, a testimony that
many of our people still wish to live in the Philippines, and are very proud of our country.

WHEREFORE, the instant petition is hereby GRANTED. Petitioner Nora Fe Sagun y Chan is hereby
DECLARED as FILIPINO CITIZEN, having chosen or elected Filipino citizenship. 12

For sure, this Court has consistently ruled that there is no proceeding established by law, or the
Rules for the judicial declaration of the citizenship of an individual. There is no specific legislation
13 

authorizing the institution of a judicial proceeding to declare that a given person is part of our
citizenry. This was our ruling in Yung Uan Chu v. Republic citing the early case of Tan v. Republic
14  15

of the Philippines, where we clearly stated:


16 

Under our laws, there can be no action or proceeding for the judicial declaration of the citizenship of
an individual. Courts of justice exist for settlement of justiciable controversies, which imply a given
right, legally demandable and enforceable, an act or omission violative of said right, and a remedy,
granted or sanctioned by law, for said breach of right. As an incident only of the adjudication of the
rights of the parties to a controversy, the court may pass upon, and make a pronouncement relative
to their status. Otherwise, such a pronouncement is beyond judicial power. x x x

Clearly, it was erroneous for the trial court to make a specific declaration of respondent’s Filipino
citizenship as such pronouncement was not within the court’s competence.

As to the propriety of respondent’s petition seeking a judicial declaration of election of Philippine


citizenship, it is imperative that we determine whether respondent is required under the law to make
an election and if so, whether she has complied with the procedural requirements in the election of
Philippine citizenship.

When respondent was born on August 8, 1959, the governing charter was the 1935 Constitution,
which declares as citizens of the Philippines those whose mothers are citizens of the Philippines and
elect Philippine citizenship upon reaching the age of majority. Sec. 1, Art. IV of the 1935
Constitution reads:

Section 1. The following are citizens of the Philippines:

xxxx

(4) Those whose mothers are citizens of the Philippines and, upon reaching the age of majority,
elect Philippine citizenship.
Under Article IV, Section 1(4) of the 1935 Constitution, the citizenship of a legitimate child born of a
Filipino mother and an alien father followed the citizenship of the father, unless, upon reaching the
age of majority, the child elected Philippine citizenship. The right to elect Philippine citizenship was
recognized in the 1973 Constitution when it provided that "[t]hose who elect Philippine citizenship
pursuant to the provisions of the Constitution of nineteen hundred and thirty-five" are citizens of the
Philippines. Likewise, this recognition by the 1973 Constitution was carried over to the 1987
17 

Constitution which states that "[t]hose born before January 17, 1973 of Filipino mothers, who elect
Philippine citizenship upon reaching the age of majority" are Philippine citizens. It should be noted,
18 

however, that the 1973 and 1987 Constitutional provisions on the election of Philippine citizenship
should not be understood as having a curative effect on any irregularity in the acquisition of
citizenship for those covered by the 1935 Constitution. If the citizenship of a person was subject to
challenge under the old charter, it remains subject to challenge under the new charter even if the
judicial challenge had not been commenced before the effectivity of the new Constitution. 19

Being a legitimate child, respondent’s citizenship followed that of her father who is Chinese, unless
upon reaching the age of majority, she elects Philippine citizenship. It is a settled rule that only
legitimate children follow the citizenship of the father and that illegitimate children are under the
parental authority of the mother and follow her nationality. An illegitimate child of Filipina need not
20 

perform any act to confer upon him all the rights and privileges attached to citizens of the
Philippines; he automatically becomes a citizen himself. But in the case of respondent, for her to be
21 

considered a Filipino citizen, she must have validly elected Philippine citizenship upon reaching the
age of majority.

Commonwealth Act (C.A.) No. 625, enacted pursuant to Section 1(4), Article IV of the 1935
22 

Constitution, prescribes the procedure that should be followed in order to make a valid election of
Philippine citizenship, to wit:

Section 1. The option to elect Philippine citizenship in accordance with subsection (4), [S]ection 1,
Article IV, of the Constitution shall be expressed in a statement to be signed and sworn to by the
party concerned before any officer authorized to administer oaths, and shall be filed with the nearest
civil registry. The said party shall accompany the aforesaid statement with the oath of allegiance to
the Constitution and the Government of the Philippines.

Based on the foregoing, the statutory formalities of electing Philippine citizenship are: (1) a
statement of election under oath; (2) an oath of allegiance to the Constitution and Government of the
Philippines; and (3) registration of the statement of election and of the oath with the nearest civil
registry. 23

Furthermore, no election of Philippine citizenship shall be accepted for registration under C.A. No.
625 unless the party exercising the right of election has complied with the requirements of the Alien
Registration Act of 1950. In other words, he should first be required to register as an
alien. Pertinently, the person electing Philippine citizenship is required to file a petition with the
24 

Commission of Immigration and Deportation (now Bureau of Immigration) for the cancellation of his
alien certificate of registration based on his aforesaid election of Philippine citizenship and said
Office will initially decide, based on the evidence presented the validity or invalidity of said
election. Afterwards, the same is elevated to the Ministry (now Department) of Justice for final
25 

determination and review. 26 


1âwphi1

It should be stressed that there is no specific statutory or procedural rule which authorizes the direct
filing of a petition for declaration of election of Philippine citizenship before the courts. The special
proceeding provided under Section 2, Rule 108 of the Rules of Court on Cancellation or Correction
of Entries in the Civil Registry, merely allows any interested party to file an action for cancellation or
correction of entry in the civil registry, i.e., election, loss and recovery of citizenship, which is not the
relief prayed for by the respondent.

Be that as it may, even if we set aside this procedural infirmity, still the trial court’s conclusion that
respondent duly elected Philippine citizenship is erroneous since the records undisputably show that
respondent failed to comply with the legal requirements for a valid election. Specifically, respondent
had not executed a sworn statement of her election of Philippine citizenship. The only documentary
evidence submitted by respondent in support of her claim of alleged election was her oath of
allegiance, executed 12 years after she reached the age of majority, which was unregistered. As
aptly pointed out by the petitioner, even assuming arguendo that respondent’s oath of allegiance
suffices, its execution was not within a reasonable time after respondent attained the age of majority
and was not registered with the nearest civil registry as required under Section 1 of C.A. No. 625.
The phrase "reasonable time" has been interpreted to mean that the election should be made
generally within three (3) years from reaching the age of majority. Moreover, there was no
27 

satisfactory explanation proffered by respondent for the delay and the failure to register with the
nearest local civil registry.

Based on the foregoing circumstances, respondent clearly failed to comply with the procedural
requirements for a valid and effective election of Philippine citizenship. Respondent cannot assert
that the exercise of suffrage and the participation in election exercises constitutes a positive act of
election of Philippine citizenship since the law specifically lays down the requirements for acquisition
of citizenship by election. The mere exercise of suffrage, continuous and uninterrupted stay in the
Philippines, and other similar acts showing exercise of Philippine citizenship cannot take the place of
election of Philippine citizenship. Hence, respondent cannot now be allowed to seek the intervention
of the court to confer upon her Philippine citizenship when clearly she has failed to validly elect
Philippine citizenship. As we held in Ching, the prescribed procedure in electing Philippine
28 

citizenship is certainly not a tedious and painstaking process. All that is required of the elector is to
execute an affidavit of election of Philippine citizenship and, thereafter, file the same with the nearest
civil registry. Having failed to comply with the foregoing requirements, respondent’s petition before
the trial court must be denied.

WHEREFORE, the petition is GRANTED. The Decision dated April 3, 2009 of the Regional Trial
Court, Branch 3 of Baguio City in Spcl. Pro. Case No. 17-R is REVERSED and SET ASIDE. The
petition for judicial declaration of election of Philippine citizenship filed by respondent Nora Fe Sagun
is hereby DISMISSED for lack of merit.

No costs.

SO ORDERED.

EN BANC

G.R. No. L-1663             March 31, 1948

FLORETINA VILLAHERMOSA, petitioner-appellant, 
vs.
THE COMMISSIONER OF IMMIGRATION, respondent-appellee.
Victoriano V. Valle for appellant.
First Assistant Solicitor General Roberto A. Gianzon and Solicitor Francisco Carreon for appellee.

BENGZON, J.:

This is an appeal from the order of Honorable Sotero Rodas, Judge of the Manila Court of First
Instance, denying the writ of habeas corpus requested by Florentina Villahermosa on behalf of her
son Delfin Co, who is under detention by the immigration authorities for purposes of deportation.

In the night of March 24, 1947, a party of sixty-nine Chinese landed clandestinely on the shores of
Sto. Domingo, Ilocos Sur, in an attempt to evade our immigration laws. Leading them was Delfin Co,
a young man, 18 years old, born in Paniqui, Tarlac, of a Chinese father named Co Sut, alias Yu Kui,
and Florentina Villahermosa his wife. Co Suy died in July 1940, and inn February 1946, Delfin left
the Philippines for China on board S/S Cushman as a Chinese repatriate, in company with his
relative Co Chi Pe. However, due to financial difficulties in China, he took steps to return; but having
met a Chines (Co Soon Tiong), who informed him of a plan to smuggle their compatriots into this
country, he agreed to lead the party to Ilocos Sur where his mother had relatives who could render
valuable assistance. The voyage was undertaken; but unfortunately, the immigrants were discovered
and apprehended immediately after arrival, and on the 27th day of March, Delfin Co was examined
by the Commissioner of Immigration. Formal investigation of the case began on April 10, 1947. Four
days later, the corresponding board recommended that said Delfin Co be deported to China as a
Chinese citizen. The Commissioner of Immigration agreed with the board, and acting on this
recommendation, rendered a decision ordering the deportation of Delfin Co.

It appears that on April 29, 1947, Florentina Villahermosa, after knowing the apprehension of her son
Delfin, filed in the civil registry of Tarlac under Commonwealth Act No. 63 an oath of allegiance for
the purpose of resuming her Philippine citizenship which she had lost upon her marriage to Co Suy.
On the strength of such reacquisition of Philippine citizenship by Florentina, it was contended before
the immigration authorities that Delfin, being a minor, followed the citizenship of his mother, and was
a national not subject to deportation. These contentions were overruled. They were repeated before
the court of first instance in this habeas corpus proceeding and were likewise rejected. Appellant
stresses he same defense.

There are two reasons why Delfin Co must be returned to China. First, he is not now a Filipino
citizen; and second, granting that he is, at the time he entered this country from China he was a
Chinese subject to deportation, and any subsequent change in his status can not erase the taint of
his unlawful, surreptitious entry.

Section 1 of Article IV of the Constitution enumerates those who are citizens of the Philippines, as
follows:

(1) Those who are citizens of the Philippine Islands at the time of the adoption of the
Constitution.

(2) Those born in the Philippine Islands of foreign parents who, before adoption of this
Constitution, had been elected to public office in the Philippine Islands.

(3) Those whose fathers are citizens of the Philippines.

(4) Those whose mothers are citizens of the Philippines and upon reaching the age of
majority, elect Philippine citizenship.
(5) Those who are naturalized in accordance with law.

Delfin Co's claim to citizenship can only be predicated, if at all, on paragraph 4 of the above section.
But, being a minor he has not had the opportunity to elect Philippine citizenship, and therefore he is
as yet an alien, his father being a Chinese.

We have heretofore held1 that, after the Constitution, mere birth in the Philippines of a Chinese
father and Filipino mother does not ipso facto confer Philippine citizenship and that jus
sanguinis instead off jus soli is the predominating factor on questions of citizenship, thereby
rendering obsolete the decision in Roa vs. Collector of Customs, 23 Phil., and U.S. vs. Lim Bin, 36
Phil., and similar cases on which petitioner's counsel relies.

Nevertheless, it is contended that Florentina Villahermosa being a Filipina, Delfin Co, should likewise
be a Filipino. Commonwealth Act No. 63 does not provide that upon repatriation of a Filipina her
children acquire Philippine citizenship. It would be illogical to consider Delfin as repatriated like his
mother, because he never was a Filipino citizen and could not have reacquired such citizenship.

While his Chinese father lived, Delfin was not a Filipino. His mother was not a Filipina; she was
Chinese. After the death of such father, Villahermosa continued to be a Chinese, until she
reacquired her Filipino citizenship in April, 1947. After that reacquisition Delfin could claim that his
mother was a Filipina within the meaning of paragraph 4, section 1 of Article IV of the Constitution;
but, according to that same Organic Act, he had to elect Philippine citizenship upon attaining his
majority. Until he becomes of age and makes the election, he is the Chinese citizen that he was at
the time of his father's demise2.

It does not help petitioner's case to assert that as a mother she has a right to retain custody of her
minor son and to keep him here. Where such son has violated the immigration laws and rendered
himself liable to deportation no rule or principle should frustrate the Government's action by the
interposition of the mother's right to custody. This consideration becomes stronger where, as in this
case, the re-assumption of Philippine citizenship by Villahermosa has all the earmarks of an attempt
to impede the banishment of Delfin Co, who by the way, besides being guilty of violating our laws,
has not shown any signs of eagerness to adopt our ways of life.

This petition is moreover to be denied on the strength of precedents heretofore established, because
Delfin was a Chinese when he arrived here; and any posterior change of status can not affect the
legality of his detention for purposes of deportation.

In Juan Co vs. Rafferty, 14 Phil., 235, a Chinaman claimed the right to enter the Islands, and being
refused by the customs officials, gave bond that he would present himself for deportation if the claim
were disallowed. While under bond, he was adopted as a son by another Chinaman domiciled
herein, in legal form. Held: he is subject to deportation, because such adoption had no effect upon
his right to enter or remain in the Islands. This Court said that the status of an immigrant and his
right to stay here is to be determined as of the time of his entry (U.S. vs. Ju-Toy, 198 U.S., 253, 263)
and that he could not do afterwards anything to render valid what was originally an illegal entry.

A Chinese person, not a merchant at the time he applies to enter the Islands, will not be
permitted to remain here upon the theory that he became a merchant during the time he was
waiting for the decision of the proper authorities, (Tan Guam Sien vs. Collector of Customs,
31 Phil., 56.) (See also, U.S. vs. Chan Sam, 17 Phil., 448)

We declare that Delfin Co is not now a Filipino. We also declare that he having entered this country
surreptitiously is subject to deportation.
The decision of the lower court denying his petition for habeas corpus is affirmed. With costs.

Paras, Pablo, Briones, and Padilla, JJ., concur.

Separate Opinions

HILADO, J., concurring:

I concur in the foregoing decision. Besides, I will only point out that petitioner, by the very purpose
for which she filed the oath of allegiance mentioned therein, made herself unworthy and disqualified
to be repatriated under Commonwealth Act No. 63.

Section 4 of said Act provides that repatriation shall be effected by merely taking the necessary oath
of allegiance to the Commonwealth of the Philippines (now Republic of the Philippines) and
registration in the proper civil registry. Allegiance requires the person pledging it, among other
things, to respect and obey the laws of the country to which the pledge is made. But here the person
taking the oath of allegiance did so for the express purpose of legalizing, so to say, a most serious
violation of the immigration laws of the Philippines by her son. An oath of allegiance taken for that
end is, an affront to the sovereign, besides the criminal responsibilities it entails.

PERFECTO, J., dissenting:

The majority decision fails to abide by one of the elemental rules of law, enunciated by human
wisdom.

That rule is stated in article 18 of the Civil Code as follows:

Children, while they remain under parental authority, have the nationality of their parents.

That rule is reaffirmed by the Naturalization Law, No. 2927, as amended by Act No. 3448. It provides
that children under 20 years of age and residing in the Philippines shall become citizens upon
naturalization of their parents.

The rule is founded on human nature. Because minor children depend on their parents for their
sustenance, support and protection, it stands to reason that they should follow the nationality of said
parents. They have to live under the same roof with their parents and as near enough to them to
enjoy parental care and protection. Minor children have to follow their parents wherever the latter, by
political, moral, mental and economic exigencies, have to establish their abode.

To accept the majority's position is to justify its inevitable consequences, one of them being the
possibility of a fratricidal battle, should the nation of one happen to be at war with that of the other.
One shudders at the mere thought that parents, as soldiers of one belligerent nation, should fire in
murderous battle against their own children fighting in the enemy trenches, while the children aim
their guns at the very authors of their lives.
There is unanimity of opinion that petitioner Florentino Villahermosa is a FIlipino citizen. There is no
question that she was born of Filipino parents in Lapog, Ilocos Sur, in March 1905. She is living in
Paniqui, Tarlac, the province of Ambassador Romulo. Since her birth she has resided in the
Philippines. She never went to China. She is a widow. She is the mother of Delfin Co, a minor of 18
years. She is the mother of another minor named Benjamin Co, who is living with her. There should
not be any question that under express statutory provisions, Delfin Co follows the nationality of his
mother. His mother is a Filipino citizen. Delfin Co is a Filipino citizen.

When on July 8, 1940, her Chinese husband died, Florentina Villahermosa must have felt that she
regained her FIlipino citizenship. She was ignorant of the provisions of Commonwealth Act No. 63,
so she failed to file her oath of allegiance required by it. Because her son came into trouble, she
happened to learn about the legal requirement on March 25, 1947, and took the oath which was filed
on the 29th of the same month with the civil registrar of Paniqui, Tarlac.

That the purpose of the said oath of allegiance is, by her repatriation, to keep her son at her side and
within the folds of this country, appears to have provoked some indignation, as if petitioner has
committed a crime or, at least, a reprehensible act. There is absolutely no ground for taking such an
attitude. Petitioner had only exercised a right expressly granted to her by law. The statutory provision
does not deal with motives or purposes. It is as impersonal as the constitutional provisions
guaranteeing fundamental rights without taking into consideration the purposes and motives for the
exercise of said rights.

That the petitioner had exercised a right expressly granted to her by law for the benefit of her son or
for the purpose of protecting him against an action harmful to him, is only logical. There is nothing
objectionable in her taking advantage of the law to give tangible expression to her maternal love,
which is, without any doubt, universally considered the most sublime feeling nature has infused in
human hearts. The feeling is so elemental that it is not unknown even to the lowest phila of the
animal kingdom. That even the fiercest wild animals are not devoid of such feeling is a wonder that
cannot fail to move he most indifferent person. Many perceive in that fact the operation of an infinite
intelligence taking care of all living things.

That petitioner had only obeyed the mandates of nature, that she yielded to an unconquerable
feeling, the one most praised my moralists, defied by spiritual and religious leaders, the subject of
glowing eulogium in eloquent prose and inspired poetry, whenever and wherever literature has
flourished, instead of causing criticism, should only merit panegyric and be acclaimed, she having
followed the noblest impulses of her nature.

Since his birth on May 31, 1928, Delfin Co has been a resident of the Philippines until February 2,
1946, when, probably yielding to the youthful lust for adventure, without the consent or knowledge of
his mother, he stealthily went to China. Having returned on March 29, 1947, to the Philippines, his
place of residence, it is only natural that he should want to remain here and that his mother should
exert all efforts so that he should not go away again. By the repatriation of Florentino Villahermosa,
Delfin Co became ipso facto a Filipino citizen. As a resident of the Philippines and as a FIlipino
citizen, he is entitled to stay.

The unfortunate fact that a character by the name of Co Soon Tiong was able to persuade him to
smuggle a bunch of Chinese into this country, by landing them in Lapog, Ilocos Sur, in consideration
of a free passage to the Philippines, is no reason to deprive him of the right to remain in the country
of which he is a resident and a citizen.

There are indications that he is entitled to more pity than blame, by his failure to resist the wiles of a
scheming person, who took undue advantage of his immaturity. His anxiousness to return to his
country and be at his mothers side must have been too strong for him to refuse a free passage, a
mere pittance when, as amply publicized, to secure entrance of Chinese immigrants, middlemen or
procurators earn thousands of pesos per person.

Did Delfin Co commit any crime or offense punishable by law? If he did, let him be prosecuted and
sentenced through due process of law, and if deportation is the punishment provided by law by
competent courts of justice, let the judgment be rendered and enforced. But it is admitted on all
sides that there is no law punishing the act of Delfin of rendering help to the smuggling of a bunch of
Chinese in question. If he did not commit any crime or offense, only a subverted sense of justice
may justify punishing him with deportation.

We vote, with the revocation of the appealed order of the lower court, to declare null and void the
order of the Commission on Immigration deporting Delfin Co to Amoy, China.

TUASON, J., dissenting:

With regret I am constrained to disagree with the views of the majority. I shall briefly state the
reasons for my dissent..

1. Article 18 of the Civil Code is explicit in its provision that "Children, while they remain under
parental authority, have the nationality of their parents." Delfin Co has become, in my opinion, a
Filipino citizen by reason of his mother's reacquisition of Philippine citizenship after her husband's
death. I see no difference, and no valid reason for differentiating, between a legitimate child of a
Filipino mother by a deceased foreign father and a Filipino mother's illegitimate child. The latter
under the rules of international law as well as the Civil Code takes the citizenship of its mother.

The intention of the framers of the Constitution to withhold Philippine citizenship from the child of a
Filipino mother and an alien father until the child reaches the age of majority, does not create an
exception to the general rule. It is my humble and considered opinion that the deferment of
conferring Filipino citizenship on such a child extends only to those cases in which both parents are
alive and retain their foreign nationality, or where the father having died, the mother has not chosen
to regain her original citizenship.

It is not good law which prevents minor child of a citizen of the country, a child to whom by law and
by nature she owes protection, from joining its parent. I do not believe that the Constitutional
Convention could ever have contemplated such an inadmissible and irrational situation.

I do not share the apprehension of some members of the Court that if a child like Delfin Co should
follow the citizenship of her mother his citizenship would be at the mercy of being changed as often
as its mother changes her citizenship by marriage or otherwise. If that should happen, there is
nothing wrong or ridiculous about it. On the contrary, I think it is more in accordance with natural law.
That is what happens in the case of an illegitimate child of a Filipino mother marrying a foreigner or
obtaining another citizenship; and there is in this connection no perceptible difference between an
illegitimate child and a legitimate child whose father is dead. If a mother can and wants to change
her citizenship daily, certainly it is natural rather than queer that her minor child, which depends
upon her for care and support, should not be left stranded.
2. The decision says: "This petition is moreover to be denied on the strength of precedence
heretofore established, because Delfin was a Chinese when he arrived here; and any posterior
change of status can not affect the legality of the detention for the purposes of deportation." I do not
think that this doctrine is applicable to the present case. The principle established by the decisions
cited on this point is that an immigrant can not take advantage of his unlawful entry to acquire the
conditions imposed by the immigration laws. In the language of this Court (U.S. vs. Chan Sam, 17
Phil., 448-456), "to say to him (immigrant) that if by any means he can gain an unlawful entry in the
Islands he will be relieved of the consequences flowing from his unlawful act if at any time after he
gains his unlawful entrance he changes his status and assumes the occupation of one of the
privileged classes, would be to set a premium on the unlawful but successful evasion by Chinese
laborers of the laws prohibiting their entrance into the Islands." And in Tan Guan Sien vs. Collector
of Customs, 31 Phil., 56, the Court had the same idea when it said, "The law does not contemplate
that Chinese persons may, by one method or another, gain an entrance into the territory of the
United States without the 'section six certificate', and after such entrance become such a merchant,
and then as such, insist upon his right to remain." In these two cases, and in the case of Juan Co
vs. Rafferty, 14 Phil., 235, in which the immigrant was adopted by a resident while the immigrant's
right to enter was under investigation, the changes in the immigrant status were effected by him or
with his intervention and could not have been accomplished in his absence.

In the case at hand the conversion of the immigrant to Philippine citizenship was entirely
independent of his will and of his presence in the Philippines. The bond that binds the petitioner and
her child existed before the latter entered the Philippine territory and not from the date of her
repatriation only. It is the legal and absolute right of the immigrant's mother to reclaim her Philippine
citizenship regardless of any mental reservation, her motives or her attitude toward her country. The
legality of her reacquisition of Philippine citizenship is nowhere challenged. Assuming then that
Delfin Co's nationality follows that of his mother, as we believe it does, has Co forfeited his right to
be with her as a result of his entering the Philippines unlawfully? I know of no law which sanctions
such punishment for an immigrant's fault. If, on the other hand, the theory is that the immigrant must
first be purged of his sin by deportation after which he may be allowed to come back and settle here,
the Court would be adopting an empty ceremony that would lead to no useful purpose nor enhance
the prestige of the administration of law.

G.R. No. 153883             January 13, 2004

REPUBLIC OF THE PHILIPPINES, petitioner, 


vs.
CHULE Y. LIM, respondent.

DECISION

YNARES-SANTIAGO, J.:

This petition for review on certiorari under Rule 45 of the Rules of Court stemmed from a petition for
correction of entries under Rule 108 of the Rules of Court filed by respondent Chule Y. Lim with the
Regional Trial Court of Lanao del Norte, Branch 4, docketed as Sp. Proc. No. 4933.

In her petition, respondent claimed that she was born on October 29, 1954 in Buru-an, Iligan City.
Her birth was registered in Kauswagan, Lanao del Norte but the Municipal Civil Registrar of
Kauswagan transferred her record of birth to Iligan City. She alleged that both her Kauswagan and
Iligan City records of birth have four erroneous entries, and prays that they be corrected.
The trial court then issued an Order,1 which reads:

WHEREFORE, finding the petition to be sufficient in form and substance, let the hearing of
this case be set on December 27, 1999 before this Court, Hall of Justice, Rosario Heights,
Tubod, Iligan City at 8:30 o’clock in the afternoon at which date, place and time any
interested person may appear and show cause why the petition should not be granted.

Let this order be published in a newspaper of general circulation in the City of Iligan and the
Province of Lanao del Norte once a week for three (3) consecutive weeks at the expense of
the petitioner.

Furnish copies of this order the Office of the Solicitor General at 134 Amorsolo St., Legaspi
Vill., Makati City and the Office of the Local Civil Registrar of Iligan City at Quezon Ave.,
Pala-o, Iligan City.

SO ORDERED.

During the hearing, respondent testified thus:

First, she claims that her surname "Yu" was misspelled as "Yo". She has been using "Yu" in
all her school records and in her marriage certificate. 2 She presented a clearance from the
National Bureau of Investigation (NBI)3 to further show the consistency in her use of the
surname "Yu".

Second, she claims that her father’s name in her birth record was written as "Yo Diu To (Co
Tian)" when it should have been "Yu Dio To (Co Tian)."

Third, her nationality was entered as Chinese when it should have been Filipino considering that her
father and mother never got married. Only her deceased father was Chinese, while her mother is
Filipina. She claims that her being a registered voter attests to the fact that she is a Filipino citizen.

Finally, it was erroneously indicated in her birth certificate that she was a legitimate child when she
should have been described as illegitimate considering that her parents were never married.

Placida Anto, respondent’s mother, testified that she is a Filipino citizen as her parents were both
Filipinos from Camiguin. She added that she and her daughter’s father were never married because
the latter had a prior subsisting marriage contracted in China.

In this connection, respondent presented a certification attested by officials of the local civil registries
of Iligan City and Kauswagan, Lanao del Norte that there is no record of marriage between Placida
Anto and Yu Dio To from 1948 to the present.

The Republic, through the City Prosecutor of Iligan City, did not present any evidence although it
actively participated in the proceedings by attending hearings and cross-examining respondent and
her witnesses.

On February 22, 2000, the trial court granted respondent’s petition and rendered judgment as
follows:
WHEREFORE, the foregoing premises considered, to set the records of the petitioner
straight and in their proper perspective, the petition is granted and the Civil Registrar of Iligan
City is directed to make the following corrections in the birth records of the petitioner, to wit:

1. Her family name from "YO" to "YU";

2. Her father’s name from "YO DIU TO (CO TIAN)" to "YU DIOTO (CO TIAN)";

3. Her status from "legitimate" to "illegitimate" by changing "YES" to "NO" in answer


to the question "LEGITIMATE?"; and,

4. Her citizenship from "Chinese" to "Filipino".

SO ORDERED.4

The Republic of the Philippines appealed the decision to the Court of Appeals which affirmed the
trial court’s decision.5

Hence, this petition on the following assigned errors:

THE COURT OF APPEALS ERRED IN ORDERING THE CORRECTION OF THE


CITIZENSHIP OF RESPONDENT CHULE Y. LIM FROM "CHINESE" TO "FILIPINO"
DESPITE THE FACT THAT RESPONDENT NEVER DEMONSTRATED ANY
COMPLIANCE WITH THE LEGAL REQUIREMENTS FOR ELECTION OF CITIZENSHIP.

II

THE COURT OF APPEALS ERRED IN ALLOWING RESPONDENT TO CONTINUE USING


HER FATHER’S SURNAME DESPITE ITS FINDING THAT RESPONDENT IS AN
ILLEGITIMATE CHILD.6

To digress, it is just as well that the Republic did not cite as error respondent’s recourse to Rule 108
of the Rules of Court to effect what indisputably are substantial corrections and changes in entries in
the civil register. To clarify, Rule 108 of the Revised Rules of Court provides the procedure for
cancellation or correction of entries in the civil registry. The proceedings under said rule may either
be summary or adversary in nature. If the correction sought to be made in the civil register is clerical,
then the procedure to be adopted is summary. If the rectification affects the civil status, citizenship or
nationality of a party, it is deemed substantial, and the procedure to be adopted is adversary. This is
our ruling in Republic v. Valencia7 where we held that even substantial errors in a civil registry may
be corrected and the true facts established under Rule 108 provided the parties aggrieved by the
error avail themselves of the appropriate adversary proceeding. An appropriate adversary suit or
proceeding is one where the trial court has conducted proceedings where all relevant facts have
been fully and properly developed, where opposing counsel have been given opportunity to
demolish the opposite party’s case, and where the evidence has been thoroughly weighed and
considered.8

As likewise observed by the Court of Appeals, we take it that the Republic’s failure to cite this error
amounts to a recognition that this case properly falls under Rule 108 of the Revised Rules of Court
considering that the proceeding can be appropriately classified as adversarial.
Instead, in its first assignment of error, the Republic avers that respondent did not comply with the
constitutional requirement of electing Filipino citizenship when she reached the age of majority. It
cites Article IV, Section 1(3) of the 1935 Constitution, which provides that the citizenship of a
legitimate child born of a Filipino mother and an alien father followed the citizenship of the father,
unless, upon reaching the age of majority, the child elected Philippine citizenship. 9 Likewise, the
Republic invokes the provision in Section 1 of Commonwealth Act No. 625, that legitimate children
born of Filipino mothers may elect Philippine citizenship by expressing such intention "in a statement
to be signed and sworn to by the party concerned before any officer authorized to administer oaths,
and shall be filed with the nearest civil registry. The said party shall accompany the aforesaid
statement with the oath of allegiance to the Constitution and the Government of the Philippines."10

Plainly, the above constitutional and statutory requirements of electing Filipino citizenship apply only
to legitimatechildren. These do not apply in the case of respondent who was concededly an
illegitimate child, considering that her Chinese father and Filipino mother were never married. As
such, she was not required to comply with said constitutional and statutory requirements to become
a Filipino citizen. By being an illegitimate child of a Filipino mother, respondent automatically
became a Filipino upon birth. Stated differently, she is a Filipino since birth without having to elect
Filipino citizenship when she reached the age of majority.

In Ching, Re: Application for Admission to the Bar,11 citing In re Florencio Mallare,12 we held:

Esteban Mallare, natural child of Ana Mallare, a Filipina, is therefore himself a Filipino, and
no other act would be necessary to confer on him all the rights and privileges attached to
Philippine citizenship (U.S. vs. Ong Tianse, 29 Phil. 332; Santos Co vs. Government of the
Philippine Islands, 42 Phil. 543; Serra vs. Republic, L-4223, May 12, 1952; Sy Quimsuan vs.
Republic, L-4693, Feb. 16, 1953; Pitallano vs. Republic, L-5111, June 28, 1954). Neither
could any act be taken on the erroneous belief that he is a non-Filipino divest him of the
citizenship privileges to which he is rightfully entitled.13

This notwithstanding, the records show that respondent elected Filipino citizenship when she
reached the age of majority. She registered as a voter in Misamis Oriental when she was 18 years
old.14 The exercise of the right of suffrage and the participation in election exercises constitute a
positive act of election of Philippine citizenship.15

In its second assignment of error, the Republic assails the Court of Appeals’ decision in allowing
respondent to use her father’s surname despite its finding that she is illegitimate.

The Republic’s submission is misleading. The Court of Appeals did not allow respondent to use her
father’s surname. What it did allow was the correction of her father’s misspelled surname which she
has been using ever since she can remember. In this regard, respondent does not need a court
pronouncement for her to use her father’s surname.

We agree with the Court of Appeals when it held:

Firstly, Petitioner-appellee is now 47 years old. To bar her at this time from using her father’s
surname which she has used for four decades without any known objection from anybody,
would only sow confusion. Concededly, one of the reasons allowed for changing one’s name
or surname is to avoid confusion.

Secondly, under Sec. 1 of Commonwealth Act No. 142, the law regulating the use of aliases,
a person is allowed to use a name "by which he has been known since childhood."
Thirdly, the Supreme Court has already addressed the same issue. In Pabellar v. Rep. of the
Phils.,16 we held:

Section 1 of Commonwealth Act No. 142, which regulates the use of aliases, allows a person
to use a name "by which he has been known since childhood" (Lim Hok Albano v. Republic,
104 Phil. 795; People v. Uy Jui Pio, 102 Phil. 679; Republic v. Tañada, infra). Even legitimate
children cannot enjoin the illegitimate children of their father from using his surname (De
Valencia v. Rodriguez, 84 Phil. 222).17

While judicial authority is required for a change of name or surname, 18 there is no such requirement
for the continued use of a surname which a person has already been using since childhood.19

The doctrine that disallows such change of name as would give the false impression of family
relationship remains valid but only to the extent that the proposed change of name would in great
probability cause prejudice or future mischief to the family whose surname it is that is involved or to
the community in general.20 In this case, the Republic has not shown that the Yu family in China
would probably be prejudiced or be the object of future mischief. In respondent’s case, the change in
the surname that she has been using for 40 years would even avoid confusion to her community in
general.

WHEREFORE, in view of the foregoing, the instant petition for review is DENIED. The decision of
the Court of Appeals in CA-G.R. CV No. 68893 dated May 29, 2002, is AFFIRMED. Accordingly, the
Civil Registrar of Iligan City is DIRECTED to make the following corrections in the birth record of
respondent Chule Y. Lim, to wit:

1. Her family name from "YO" to "YU";

2. Her father’s name from "YO DIU TO (CO TIAN)" to "YU DIOTO (CO TIAN)";

3. Her status from "legitimate" to "illegitimate" by changing "YES" to "NO" in answer to the
question "LEGITIMATE?"; and,

4. Her citizenship from "Chinese" to "Filipino".

SO ORDERED.

G.R. No. 161434             March 3, 2004

MARIA JEANETTE C. TECSON and FELIX B. DESIDERIO, JR., petitioners, 


vs.
The COMMISSION ON ELECTIONS, RONALD ALLAN KELLY POE (a.k.a. FERNANDO POE,
JR.) and VICTORINO X. FORNIER, respondents.

x-----------------------------x

G.R. No. 161634             March 3, 2004

ZOILO ANTONIO VELEZ, petitioner, 


vs.
RONALD ALLAN KELLEY POE, a.k.a. FERNANDO POE, JR., respondent.
x-----------------------------x

G. R. No. 161824             March 3, 2004

VICTORINO X. FORNIER, petitioner, 
vs.
HON. COMMISSION ON ELECTIONS and RONALD ALLAN KELLEY POE, ALSO KNOWN AS
FERNANDO POE JR., respondents.

DECISION

VITUG, J.:

Citizenship is a treasured right conferred on those whom the state believes are deserving of
the privilege. It is a "precious heritage, as well as an inestimable acquisition," 1 that cannot be
taken lightly by anyone - either by those who enjoy it or by those who dispute it.

Before the Court are three consolidated cases, all of which raise a single question of profound
importance to the nation. The issue of citizenship is brought up to challenge the qualifications of a
presidential candidate to hold the highest office of the land. Our people are waiting for the judgment
of the Court with bated breath. Is Fernando Poe, Jr., the hero of silver screen, and now one of the
main contenders for the presidency, a natural-born Filipino or is he not?

The moment of introspection takes us face to face with Spanish and American colonial roots and
reminds us of the rich heritage of civil law and common law traditions, the fusion resulting in a hybrid
of laws and jurisprudence that could be no less than distinctly Filipino.

Antecedent Case Settings

On 31 December 2003, respondent Ronald Allan Kelly Poe, also known as Fernando Poe, Jr.
(hereinafter "FPJ"), filed his certificate of candidacy for the position of President of the Republic of
the Philippines under the Koalisyon ng Nagkakaisang Pilipino (KNP) Party, in the forthcoming
national elections. In his certificate of candidacy, FPJ, representing himself to be a natural-born
citizen of the Philippines, stated his name to be "Fernando Jr.," or "Ronald Allan" Poe, his date of
birth to be 20 August 1939 and his place of birth to be Manila.

Victorino X. Fornier, petitioner in G.R. No. 161824, entitled "Victorino X. Fornier, Petitioner, versus
Hon. Commission on Elections and Ronald Allan Kelley Poe, also known as Fernando Poe, Jr.,
Respondents," initiated, on 09 January 2004, a petition docketed SPA No. 04-003 before the
Commission on Elections ("COMELEC") to disqualify FPJ and to deny due course or to cancel his
certificate of candidacy upon the thesis that FPJ made a material misrepresentation in his certificate
of candidacy by claiming to be a natural-born Filipino citizen when in truth, according to Fornier, his
parents were foreigners; his mother, Bessie Kelley Poe, was an American, and his father, Allan Poe,
was a Spanish national, being the son of Lorenzo Pou, a Spanish subject. Granting, petitioner
asseverated, that Allan F. Poe was a Filipino citizen, he could not have transmitted his Filipino
citizenship to FPJ, the latter being an illegitimate child of an alien mother. Petitioner based the
allegation of the illegitimate birth of respondent on two assertions - first, Allan F. Poe contracted a
prior marriage to a certain Paulita Gomez before his marriage to Bessie Kelley and, second, even if
no such prior marriage had existed, Allan F. Poe, married Bessie Kelly only a year after the birth of
respondent.
In the hearing before the Third Division of the COMELEC on 19 January 2004, petitioner, in support
of his claim, presented several documentary exhibits - 1) a copy of the certificate of birth of FPJ, 2) a
certified photocopy of an affidavit executed in Spanish by Paulita Poe y Gomez attesting to her
having filed a case for bigamy and concubinage against the father of respondent, Allan F. Poe, after
discovering his bigamous relationship with Bessie Kelley, 3) an English translation of the affidavit
aforesaid, 4) a certified photocopy of the certificate of birth of Allan F. Poe, 5) a certification issued
by the Director of the Records Management and Archives Office, attesting to the fact that there was
no record in the National Archives that a Lorenzo Poe or Lorenzo Pou resided or entered the
Philippines before 1907, and 6) a certification from the Officer-In-Charge of the Archives Division of
the National Archives to the effect that no available information could be found in the files of the
National Archives regarding the birth of Allan F. Poe.

On his part, respondent, presented twenty-two documentary pieces of evidence, the more significant
ones being - a) a certification issued by Estrella M. Domingo of the Archives Division of the National
Archives that there appeared to be no available information regarding the birth of Allan F. Poe in the
registry of births for San Carlos, Pangasinan, b) a certification issued by the Officer-In-Charge of the
Archives Division of the National Archives that no available information about the marriage of Allan
F. Poe and Paulita Gomez could be found, c) a certificate of birth of Ronald Allan Poe, d) Original
Certificate of Title No. P-2247 of the Registry of Deeds for the Province of Pangasinan, in the name
of Lorenzo Pou, e) copies of Tax Declaration No. 20844, No. 20643, No. 23477 and No. 23478 in
the name of Lorenzo Pou, f) a copy of the certificate of death of Lorenzo Pou, g) a copy of the
purported marriage contract between Fernando Pou and Bessie Kelley, and h) a certification issued
by the City Civil Registrar of San Carlos City, Pangasinan, stating that the records of birth in the said
office during the period of from 1900 until May 1946 were totally destroyed during World War II.

On 23 January 2004, the COMELEC dismissed SPA No. 04-003 for lack of merit. Three days later,
or on 26 January 2004, Fornier filed his motion for reconsideration. The motion was denied on 06
February 2004 by the COMELEC en banc. On 10 February 2004, petitioner assailed the decision of
the COMELEC before this Court conformably with Rule 64, in relation to Rule 65, of the Revised
Rules of Civil Procedure. The petition, docketed G. R. No. 161824, likewise prayed for a temporary
restraining order, a writ of preliminary injunction or any other resolution that would stay the finality
and/or execution of the COMELEC resolutions.

The other petitions, later consolidated with G. R. No. 161824, would include G. R. No. 161434,
entitled "Maria Jeanette C. Tecson, and Felix B. Desiderio, Jr., vs. The Commission on Elections,
Ronald Allan Kelley Poe (a.k.a. ‘Fernando Poe, Jr.’), and Victorino X. Fornier," and the other,
docketed G. R. No. 161634, entitled "Zoilo Antonio G. Velez, vs. Ronald Allan Kelley Poe, a.k.a.
Fernando Poe, Jr.," both challenging the jurisdiction of the COMELEC and asserting that, under
Article VII, Section 4, paragraph 7, of the 1987 Constitution, only the Supreme Court had original
and exclusive jurisdiction to resolve the basic issue on the case.

Jurisdiction of the Court

In G. R. No. 161824

In seeking the disqualification of the candidacy of FPJ and to have the COMELEC deny due course
to or cancel FPJ’s certificate of candidacy for alleged misrepresentation of a material fact (i.e., that
FPJ was a natural-born citizen) before the COMELEC, petitioner Fornier invoked Section 78 of the
Omnibus Election Code –

"Section 78. Petition to deny due course to or cancel a certificate of candidacy. --- A verified
petition seeking to deny due course or to cancel a certificate of candidacy may be filed by
any person exclusively on the ground that any material representation contained therein as
required under Section 74 hereof is false" –

in consonance with the general powers of COMELEC expressed in Section 52 of the Omnibus
Election Code -

"Section 52. Powers and functions of the Commission on Elections. In addition to the powers
and functions conferred upon it by the Constitution, the Commission shall have exclusive
charge of the enforcement and administration of all laws relative to the conduct of elections
for the purpose of ensuring free, orderly and honest elections" -

and in relation to Article 69 of the Omnibus Election Code which would authorize "any
interested party" to file a verified petition to deny or cancel the certificate of candidacy of any
nuisance candidate.

Decisions of the COMELEC on disqualification cases may be reviewed by the Supreme Court per
Rule 642 in an action for certiorari under Rule 653 of the Revised Rules of Civil Procedure. Section 7,
Article IX, of the 1987 Constitution also reads –

"Each Commission shall decide by a majority vote of all its Members any case or matter
brought before it within sixty days from the date of its submission for decision or resolution. A
case or matter is deemed submitted for decision or resolution upon the filing of the last
pleading, brief, or memorandum, required by the rules of the Commission or by the
Commission itself. Unless otherwise provided by this Constitution or by law, any decision,
order, or ruling of each Commission may be brought to the Supreme Court on certiorari by
the aggrieved party within thirty days from receipt of a copy thereof."

Additionally, Section 1, Article VIII, of the same Constitution provides that judicial power is vested in
one Supreme Court and in such lower courts as may be established by law which power "includes
the duty of the courts of justice to settle actual controversies involving rights which are legally
demandable and enforceable, and to determine whether or not there has been a grave abuse of
discretion amounting to lack or excess of jurisdiction on the part of any branch or instrumentality of
the Government."

It is sufficiently clear that the petition brought up in G. R. No. 161824 was aptly elevated to, and
could well be taken cognizance of by, this Court. A contrary view could be a gross denial to our
people of their fundamental right to be fully informed, and to make a proper choice, on who could or
should be elected to occupy the highest government post in the land.

In G. R. No. 161434 and G. R. No. 161634

Petitioners Tecson, et al., in G. R. No. 161434, and Velez, in G. R. No. 161634, invoke the
provisions of Article VII, Section 4, paragraph 7, of the 1987 Constitution in assailing the jurisdiction
of the COMELEC when it took cognizance of SPA No. 04-003 and in urging the Supreme Court to
instead take on the petitions they directly instituted before it. The Constitutional provision cited
reads:

"The Supreme Court, sitting en banc, shall be the sole judge of all contests relating to the
election, returns, and qualifications of the President or Vice-President, and may promulgate
its rules for the purpose."
The provision is an innovation of the 1987 Constitution. The omission in the 1935 and the 1973
Constitution to designate any tribunal to be the sole judge of presidential and vice-presidential
contests, has constrained this Court to declare, in Lopez vs. Roxas,4 as "not (being) justiciable"
controversies or disputes involving contests on the elections, returns and qualifications of the
President or Vice-President. The constitutional lapse prompted Congress, on 21 June 1957, to enact
Republic Act No. 1793, "An Act Constituting an Independent Presidential Electoral Tribunal to Try,
Hear and Decide Protests Contesting the Election of the President-Elect and the Vice-President-
Elect of the Philippines and Providing for the Manner of Hearing the Same." Republic Act 1793
designated the Chief Justice and the Associate Justices of the Supreme Court to be the members of
the tribunal. Although the subsequent adoption of the parliamentary form of government under the
1973 Constitution might have implicitly affected Republic Act No. 1793, the statutory set-up,
nonetheless, would now be deemed revived under the present Section 4, paragraph 7, of the 1987
Constitution.

Ordinary usage would characterize a "contest" in reference to a post-election scenario. Election


contests consist of either an election protest or a quo warranto which, although two distinct
remedies, would have one objective in view, i.e., to dislodge the winning candidate from office. A
perusal of the phraseology in Rule 12, Rule 13, and Rule 14 of the "Rules of the Presidential
Electoral Tribunal," promulgated by the Supreme Court en banc on 18 April 1992, would support this
premise -

"Rule 12. Jurisdiction. - The Tribunal shall be the sole judge of all contests relating to the
election, returns, and qualifications of the President or Vice-President of the Philippines.

"Rule 13. How Initiated. - An election contest is initiated by the filing of an election protest or
a petition for quo warranto against the President or Vice-President. An election protest shall
not include a petition for quo warranto. A petition for quo warranto shall not include an
election protest.

"Rule 14. Election Protest. - Only the registered candidate for President or for Vice-President
of the Philippines who received the second or third highest number of votes may contest the
election of the President or the Vice-President, as the case may be, by filing a verified
petition with the Clerk of the Presidential Electoral Tribunal within thirty (30) days after the
proclamation of the winner."

The rules categorically speak of the jurisdiction of the tribunal over contests relating to the election,
returns and qualifications of the "President" or "Vice-President", of the Philippines, and not of
"candidates" for President or Vice-President. A quo warranto proceeding is generally defined as
being an action against a person who usurps, intrudes into, or unlawfully holds or exercises a public
office.5 In such context, the election contest can only contemplate a post-election scenario. In Rule
14, only a registered candidate who would have received either the second or third highest number
of votes could file an election protest. This rule again presupposes a post-election scenario.

It is fair to conclude that the jurisdiction of the Supreme Court, defined by Section 4, paragraph 7, of
the 1987 Constitution, would not include cases directly brought before it, questioning the
qualifications of a candidate for the presidency or vice-presidency before the elections are held.

Accordingly, G. R. No. 161434, entitled "Maria Jeanette C. Tecson, et al., vs. Commission on
Elections et al.," and G. R. No. 161634, entitled "Zoilo Antonio Velez vs. Ronald Allan Kelley Poe
a.k.a. Fernando Poe, Jr." would have to be dismissed for want of jurisdiction.

The Citizenship Issue


Now, to the basic issue; it should be helpful to first give a brief historical background on the concept
of citizenship.

Perhaps, the earliest understanding of citizenship was that given by Aristotle, who, sometime in 384
to 322 B.C., described the "citizen" to refer to a man who shared in the administration of justice and
in the holding of an office.6Aristotle saw its significance if only to determine the constituency of the
"State," which he described as being composed of such persons who would be adequate in number
to achieve a self-sufficient existence.7 The concept grew to include one who would both govern and
be governed, for which qualifications like autonomy, judgment and loyalty could be expected.
Citizenship was seen to deal with rights and entitlements, on the one hand, and with concomitant
obligations, on the other.8 In its ideal setting, a citizen was active in public life and fundamentally
willing to submit his private interests to the general interest of society.

The concept of citizenship had undergone changes over the centuries. In the 18th century, the
concept was limited, by and large, to civil citizenship, which established the rights necessary for
individual freedom, such as rights to property, personal liberty and justice.9 Its meaning expanded
during the 19th century to include political citizenship, which encompassed the right to participate in
the exercise of political power. 10 The 20th century saw the next stage of the development of social
citizenship, which laid emphasis on the right of the citizen to economic well-being and social
security.11 The idea of citizenship has gained expression in the modern welfare state as it so
developed in Western Europe. An ongoing and final stage of development, in keeping with the
rapidly shrinking global village, might well be the internationalization of citizenship.12

The Local Setting - from Spanish Times to the Present

There was no such term as "Philippine citizens" during the Spanish regime but "subjects of Spain" or
"Spanish subjects."13 In church records, the natives were called 'indios', denoting a low regard for the
inhabitants of the archipelago. Spanish laws on citizenship became highly codified during the 19th
century but their sheer number made it difficult to point to one comprehensive law. Not all of these
citizenship laws of Spain however, were made to apply to the Philippine Islands except for those
explicitly extended by Royal Decrees.14

Spanish laws on citizenship were traced back to the Novisima Recopilacion, promulgated in Spain
on 16 July 1805 but as to whether the law was extended to the Philippines remained to be the
subject of differing views among experts;15 however, three royal decrees were undisputably made
applicable to Spaniards in the Philippines - the Order de la Regencia of 14 August 1841,16 the Royal
Decree of 23 August 1868 specifically defining the political status of children born in the Philippine
Islands,17 and finally, the Ley Extranjera de Ultramar of 04 July 1870, which was expressly made
applicable to the Philippines by the Royal Decree of 13 July 1870.18

The Spanish Constitution of 1876 was never extended to the Philippine Islands because of the
express mandate of its Article 89, according to which the provisions of the Ultramar among which
this country was included, would be governed by special laws.19

It was only the Civil Code of Spain, made effective in this jurisdiction on 18 December 1889, which
came out with the first categorical enumeration of who were Spanish citizens. -

"(a) Persons born in Spanish territory,

"(b) Children of a Spanish father or mother, even if they were born outside of Spain,
"(c) Foreigners who have obtained naturalization papers,

"(d) Those who, without such papers, may have become domiciled inhabitants of any town of
the Monarchy."20

The year 1898 was another turning point in Philippine history. Already in the state of decline as a
superpower, Spain was forced to so cede her sole colony in the East to an upcoming world power,
the United States. An accepted principle of international law dictated that a change in sovereignty,
while resulting in an abrogation of all political laws then in force, would have no effect on civil laws,
which would remain virtually intact.

The Treaty of Paris was entered into on 10 December 1898 between Spain and the United
States.21 Under Article IX of the treaty, the civil rights and political status of the native inhabitants of
the territories ceded to the United States would be determined by its Congress -

"Spanish subjects, natives of the Peninsula, residing in the territory over which Spain by the
present treaty relinquishes or cedes her sovereignty may remain in such territory or may
remove therefrom, retaining in either event all their rights of property, including the right to
sell or dispose of such property or of its proceeds; and they shall also have the right to carry
on their industry, commerce, and professions, being subject in respect thereof to such laws
as are applicable to foreigners. In case they remain in the territory they may preserve their
allegiance to the Crown of Spain by making, before a court of record, within a year from the
date of the exchange of ratifications of this treaty, a declaration of their decision to preserve
such allegiance; in default of which declaration they shall be held to have renounced it and to
have adopted the nationality of the territory in which they reside.

Thus –

"The civil rights and political status of the native inhabitants of the territories hereby ceded to
the United States shall be determined by the Congress."22

Upon the ratification of the treaty, and pending legislation by the United States Congress on the
subject, the native inhabitants of the Philippines ceased to be Spanish subjects. Although they did
not become American citizens, they, however, also ceased to be "aliens" under American laws and
were thus issued passports describing them to be citizens of the Philippines entitled to the protection
of the United States.

The term "citizens of the Philippine Islands" appeared for the first time in the Philippine Bill of 1902,
also commonly referred to as the Philippine Organic Act of 1902, the first comprehensive legislation
of the Congress of the United States on the Philippines -

".... that all inhabitants of the Philippine Islands continuing to reside therein, who were
Spanish subjects on the 11th day of April, 1891, and then resided in said Islands, and their
children born subsequent thereto, shall be deemed and held to be citizens of the Philippine
Islands and as such entitled to the protection of the United States, except such as shall have
elected to preserve their allegiance to the Crown of Spain in accordance with the provisions
of the treaty of peace between the United States and Spain, signed at Paris, December tenth
eighteen hundred and ninety eight."23

Under the organic act, a "citizen of the Philippines" was one who was an inhabitant of the
Philippines, and a Spanish subject on the 11th day of April 1899. The term "inhabitant" was taken to
include 1) a native-born inhabitant, 2) an inhabitant who was a native of Peninsular Spain, and 3) an
inhabitant who obtained Spanish papers on or before 11 April 1899.24

Controversy arose on to the status of children born in the Philippines from 11 April 1899 to 01 July
1902, during which period no citizenship law was extant in the Philippines. Weight was given to the
view, articulated in jurisprudential writing at the time, that the common law principle of jus soli,
otherwise also known as the principle of territoriality, operative in the United States and England,
governed those born in the Philippine Archipelago within that period.25 More about this later.

In 23 March 1912, the Congress of the United States made the following amendment to the
Philippine Bill of 1902 -

"Provided, That the Philippine Legislature is hereby authorized to provide by law for the
acquisition of Philippine citizenship by those natives of the Philippine Islands who do not
come within the foregoing provisions, the natives of other insular possession of the United
States, and such other persons residing in the Philippine Islands who would become citizens
of the United States, under the laws of the United States, if residing therein."26

With the adoption of the Philippine Bill of 1902, the concept of "Philippine citizens" had for the first
time crystallized. The word "Filipino" was used by William H. Taft, the first Civil Governor General in
the Philippines when he initially made mention of it in his slogan, "The Philippines for the Filipinos."
In 1916, the Philippine Autonomy Act, also known as the Jones Law restated virtually the provisions
of the Philippine Bill of 1902, as so amended by the Act of Congress in 1912 -

"That all inhabitants of the Philippine Islands who were Spanish subjects on the eleventh day
of April, eighteen hundred and ninety-nine, and then resided in said Islands, and their
children born subsequently thereto, shall be deemed and held to be citizens of the Philippine
Islands, except such as shall have elected to preserve their allegiance to the Crown of Spain
in accordance with the provisions of the treaty of peace between the United States and
Spain, signed at Paris December tenth, eighteen hundred and ninety-eight and except such
others as have since become citizens of some other country; Provided, That the Philippine
Legislature, herein provided for, is hereby authorized to provide for the acquisition of
Philippine citizenship by those natives of the Philippine Islands who do not come within the
foregoing provisions, the natives of the insular possessions of the United States, and such
other persons residing in the Philippine Islands who are citizens of the United States, or who
could become citizens of the United States under the laws of the United States, if residing
therein."

Under the Jones Law, a native-born inhabitant of the Philippines was deemed to be a citizen of the
Philippines as of 11 April 1899 if he was 1) a subject of Spain on 11 April 1899, 2) residing in the
Philippines on said date, and, 3) since that date, not a citizen of some other country.

While there was, at one brief time, divergent views on whether or not jus soli was a mode of
acquiring citizenship, the 1935 Constitution brought to an end to any such link with common law, by
adopting, once and for all, jus sanguinis or blood relationship as being the basis of Filipino
citizenship -

"Section 1, Article III, 1935 Constitution. The following are citizens of the Philippines -

"(1) Those who are citizens of the Philippine Islands at the time of the adoption of this
Constitution
"(2) Those born in the Philippines Islands of foreign parents who, before the adoption of this
Constitution, had been elected to public office in the Philippine Islands.

"(3) Those whose fathers are citizens of the Philippines.

"(4) Those whose mothers are citizens of the Philippines and upon reaching the age of
majority, elect Philippine citizenship.

"(5) Those who are naturalized in accordance with law."

Subsection (4), Article III, of the 1935 Constitution, taken together with existing civil law provisions at
the time, which provided that women would automatically lose their Filipino citizenship and acquire
that of their foreign husbands, resulted in discriminatory situations that effectively incapacitated the
women from transmitting their Filipino citizenship to their legitimate children and required illegitimate
children of Filipino mothers to still elect Filipino citizenship upon reaching the age of majority.
Seeking to correct this anomaly, as well as fully cognizant of the newly found status of Filipino
women as equals to men, the framers of the 1973 Constitution crafted the provisions of the new
Constitution on citizenship to reflect such concerns -

"Section 1, Article III, 1973 Constitution - The following are citizens of the Philippines:

"(1) Those who are citizens of the Philippines at the time of the adoption of this Constitution.

"(2) Those whose fathers or mothers are citizens of the Philippines.

"(3) Those who elect Philippine citizenship pursuant to the provisions of the Constitution of
nineteen hundred and thirty-five.

"(4) Those who are naturalized in accordance with law."

For good measure, Section 2 of the same article also further provided that –

"A female citizen of the Philippines who marries an alien retains her Philippine citizenship,
unless by her act or omission she is deemed, under the law to have renounced her
citizenship."

The 1987 Constitution generally adopted the provisions of the 1973 Constitution, except for
subsection (3) thereof that aimed to correct the irregular situation generated by the questionable
proviso in the 1935 Constitution.

Section I, Article IV, 1987 Constitution now provides:

"The following are citizens of the Philippines:

"(1) Those who are citizens of the Philippines at the time of the adoption of this
Constitution.

"(2) Those whose fathers or mothers are citizens of the Philippines.


"(3) Those born before January 17, 1973 of Filipino mothers, who elect Philippine
citizenship upon reaching the age of majority; and

"(4) Those who are naturalized in accordance with law."

The Case Of FPJ

Section 2, Article VII, of the 1987 Constitution expresses:

"No person may be elected President unless he is a natural-born citizen of the Philippines, a
registered voter, able to read and write, at least forty years of age on the day of the election,
and a resident of the Philippines for at least ten years immediately preceding such election."

The term "natural-born citizens," is defined to include "those who are citizens of the Philippines from
birth without having to perform any act to acquire or perfect their Philippine citizenship."27

The date, month and year of birth of FPJ appeared to be 20 August 1939 during the regime of the
1935 Constitution. Through its history, four modes of acquiring citizenship - naturalization, jus soli,
res judicata and jus sanguinis28 – had been in vogue. Only two, i.e., jus soli and jus sanguinis, could
qualify a person to being a "natural-born" citizen of the Philippines. Jus soli, per Roa vs. Collector of
Customs29 (1912), did not last long. With the adoption of the 1935 Constitution and the reversal of
Roa in Tan Chong vs. Secretary of Labor 30 (1947), jus sanguinis or blood relationship would now
become the primary basis of citizenship by birth.

Documentary evidence adduced by petitioner would tend to indicate that the earliest established
direct ascendant of FPJ was his paternal grandfather Lorenzo Pou, married to Marta Reyes, the
father of Allan F. Poe. While the record of birth of Lorenzo Pou had not been presented in evidence,
his death certificate, however, identified him to be a Filipino, a resident of San Carlos, Pangasinan,
and 84 years old at the time of his death on 11 September 1954. The certificate of birth of the father
of FPJ, Allan F. Poe, showed that he was born on 17 May 1915 to an Español father, Lorenzo Pou,
and a mestiza Español mother, Marta Reyes. Introduced by petitioner was an "uncertified" copy of a
supposed certificate of the alleged marriage of Allan F. Poe and Paulita Gomez on 05 July 1936.
The marriage certificate of Allan F. Poe and Bessie Kelley reflected the date of their marriage to be
on 16 September 1940. In the same certificate, Allan F. Poe was stated to be twenty-five years old,
unmarried, and a Filipino citizen, and Bessie Kelley to be twenty-two years old, unmarried, and an
American citizen. The birth certificate of FPJ, would disclose that he was born on 20 August 1939 to
Allan F. Poe, a Filipino, twenty-four years old, married to Bessie Kelly, an American citizen, twenty-
one years old and married.

Considering the reservations made by the parties on the veracity of some of the entries on the birth
certificate of respondent and the marriage certificate of his parents, the only conclusions that could
be drawn with some degree of certainty from the documents would be that -

1. The parents of FPJ were Allan F. Poe and Bessie Kelley;

2. FPJ was born to them on 20 August 1939;

3. Allan F. Poe and Bessie Kelley were married to each other on 16 September, 1940;

4. The father of Allan F. Poe was Lorenzo Poe; and


5. At the time of his death on 11 September 1954, Lorenzo Poe was 84 years old.

Would the above facts be sufficient or insufficient to establish the fact that FPJ is a natural-born
Filipino citizen? The marriage certificate of Allan F. Poe and Bessie Kelley, the birth certificate of
FPJ, and the death certificate of Lorenzo Pou are documents of public record in the custody of a
public officer. The documents have been submitted in evidence by both contending parties during
the proceedings before the COMELEC.

The birth certificate of FPJ was marked Exhibit "A" for petitioner and Exhibit "3" for respondent. The
marriage certificate of Allan F. Poe to Bessie Kelley was submitted as Exhibit "21" for respondent.
The death certificate of Lorenzo Pou was submitted by respondent as his Exhibit "5." While the last
two documents were submitted in evidence for respondent, the admissibility thereof, particularly in
reference to the facts which they purported to show, i.e., the marriage certificate in relation to the
date of marriage of Allan F. Poe to Bessie Kelley and the death certificate relative to the death of
Lorenzo Pou on 11 September 1954 in San Carlos, Pangasinan, were all admitted by petitioner, who
had utilized those material statements in his argument. All three documents were certified true
copies of the originals.

Section 3, Rule 130, Rules of Court states that -

"Original document must be produced; exceptions. - When the subject of inquiry is the
contents of a document, no evidence shall be admissible other than the original document
itself, except in the following cases:

"x x x           x x x           x x x

"(d) When the original is a public record in the custody of a public office or is recorded in a
public office."

Being public documents, the death certificate of Lorenzo Pou, the marriage certificate of Allan F. Poe
and Bessie Kelly, and the birth certificate of FPJ, constitute prima facie proof of their contents.
Section 44, Rule 130, of the Rules of Court provides:

"Entries in official records. Entries in official records made in the performance of his duty by a
public officer of the Philippines, or by a person in the performance of a duty specially
enjoined by law, are prima facie evidence of the facts therein stated."

The trustworthiness of public documents and the value given to the entries made therein could be
grounded on 1) the sense of official duty in the preparation of the statement made, 2) the penalty
which is usually affixed to a breach of that duty, 3) the routine and disinterested origin of most such
statements, and 4) the publicity of record which makes more likely the prior exposure of such errors
as might have occurred.31

The death certificate of Lorenzo Pou would indicate that he died on 11 September 1954, at the age
of 84 years, in San Carlos, Pangasinan. It could thus be assumed that Lorenzo Pou was born
sometime in the year 1870 when the Philippines was still a colony of Spain. Petitioner would argue
that Lorenzo Pou was not in the Philippines during the crucial period of from 1898 to 1902
considering that there was no existing record about such fact in the Records Management and
Archives Office. Petitioner, however, likewise failed to show that Lorenzo Pou was at any other place
during the same period. In his death certificate, the residence of Lorenzo Pou was stated to be San
Carlos, Pangasinan. In the absence of any evidence to the contrary, it should be sound to conclude,
or at least to presume, that the place of residence of a person at the time of his death was also his
residence before death. It would be extremely doubtful if the Records Management and Archives
Office would have had complete records of all residents of the Philippines from 1898 to 1902.

Proof of Paternity and Filiation

Under Civil Law.

Petitioner submits, in any case, that in establishing filiation (relationship or civil status of the child to
the father [or mother]) or paternity (relationship or civil status of the father to the child) of an
illegitimate child, FPJ evidently being an illegitimate son according to petitioner, the mandatory rules
under civil law must be used.

Under the Civil Code of Spain, which was in force in the Philippines from 08 December 1889 up until
the day prior to 30 August 1950 when the Civil Code of the Philippines took effect, acknowledgment
was required to establish filiation or paternity. Acknowledgment was either judicial (compulsory) or
voluntary. Judicial or compulsory acknowledgment was possible only if done during the lifetime of
the putative parent; voluntary acknowledgment could only be had in a record of birth, a will, or a
public document.32 Complementary to the new code was Act No. 3753 or the Civil Registry Law
expressing in Section 5 thereof, that -

"In case of an illegitimate child, the birth certificate shall be signed and sworn to jointly by the
parents of the infant or only by the mother if the father refuses. In the latter case, it shall not
be permissible to state or reveal in the document the name of the father who refuses to
acknowledge the child, or to give therein any information by which such father could be
identified."

In order that the birth certificate could then be utilized to prove voluntary acknowledgment of filiation
or paternity, the certificate was required to be signed or sworn to by the father. The failure of such
requirement rendered the same useless as being an authoritative document of recognition. 33 In
Mendoza vs. Mella,34 the Court ruled -

"Since Rodolfo was born in 1935, after the registry law was enacted, the question here really
is whether or not his birth certificate (Exhibit 1), which is merely a certified copy of the
registry record, may be relied upon as sufficient proof of his having been voluntarily
recognized. No such reliance, in our judgment, may be placed upon it. While it contains the
names of both parents, there is no showing that they signed the original, let alone swore to
its contents as required in Section 5 of Act No. 3753. For all that might have happened, it
was not even they or either of them who furnished the data to be entered in the civil register.
Petitioners say that in any event the birth certificate is in the nature of a public document
wherein voluntary recognition of a natural child may also be made, according to the same
Article 131. True enough, but in such a case, there must be a clear statement in the
document that the parent recognizes the child as his or her own."

In the birth certificate of respondent FPJ, presented by both parties, nowhere in the document was
the signature of Allan F. Poe found. There being no will apparently executed, or at least shown to
have been executed, by decedent Allan F. Poe, the only other proof of voluntary recognition
remained to be "some other public document." In Pareja vs. Pareja, 35 this Court defined what could
constitute such a document as proof of voluntary acknowledgment:

"Under the Spanish Civil Code there are two classes of public documents, those executed by
private individuals which must be authenticated by notaries, and those issued by competent
public officials by reason of their office. The public document pointed out in Article 131 as
one of the means by which recognition may be made belongs to the first class."

Let us leave it at that for the moment.

The 1950 Civil Code categorized the acknowledgment or recognition of illegitimate children into
voluntary, legal or compulsory. Voluntary recognition was required to be expressedly made in a
record of birth, a will, a statement before a court of record or in any authentic writing. Legal
acknowledgment took place in favor of full blood brothers and sisters of an illegitimate child who was
recognized or judicially declared as natural. Compulsory acknowledgment could be demanded
generally in cases when the child had in his favor any evidence to prove filiation. Unlike an action to
claim legitimacy which would last during the lifetime of the child, and might pass exceptionally to the
heirs of the child, an action to claim acknowledgment, however, could only be brought during the
lifetime of the presumed parent.

Amicus Curiae Ruben F. Balane defined, during the oral argument, "authentic writing," so as to be
an authentic writing for purposes of voluntary recognition, simply as being a genuine or indubitable
writing of the father. The term would include a public instrument (one duly acknowledged before a
notary public or other competent official) or a private writing admitted by the father to be his.

The Family Code has further liberalized the rules; Article 172, Article 173, and Article 175 provide:

"Art. 172. The filiation of legitimate children is established by any of the following:

"(1) The record of birth appearing in the civil register or a final judgment; or

"(2) An admission of legitimate filiation in a public document or a private handwritten


instrument and signed by the parent concerned.

"In the absence of the foregoing evidence, the legitimate filiation shall be proved by:

"(1) The open and continuous possession of the status of a legitimate child; or

"(2) Any other means allowed by the Rules of Court and special laws.

"Art. 173. The action to claim legitimacy may be brought by the child during his or her lifetime
and shall be transmitted to the heirs should the child die during minority or in a state of
insanity. In these cases, the heirs shall have a period of five years within which to institute
the action.

"The action already commenced by the child shall survive notwithstanding the death of either
or both of the parties.

"x x x           x x x           x x x.

"Art. 175. Illegitimate children may establish their illegitimate filiation in the same way and on
the same, evidence as legitimate children.

"The action must be brought within the same period specified in Article 173, except when the
action is based on the second paragraph of Article 172, in which case the action may be
brought during the lifetime of the alleged parent."
The provisions of the Family Code are retroactively applied; Article 256 of the code reads:

"Art. 256. This Code shall have retroactive effect insofar as it does not prejudice or impair
vested or acquired rights in accordance with the Civil Code or other laws."

Thus, in Vda. de Sy-Quia vs. Court of Appeals,36 the Court has ruled:

"We hold that whether Jose was a voluntarily recognized natural child should be decided
under Article 278 of the Civil Code of the Philippines. Article 2260 of that Code provides that
'the voluntary recognition of a natural child shall take place according to this Code, even if
the child was born before the effectivity of this body of laws' or before August 30, 1950.
Hence, Article 278 may be given retroactive effect."

It should be apparent that the growing trend to liberalize the acknowledgment or recognition of
illegitimate children is an attempt to break away from the traditional idea of keeping well apart
legitimate and non-legitimate relationships within the family in favor of the greater interest and
welfare of the child. The provisions are intended to merely govern the private and personal affairs of
the family. There is little, if any, to indicate that the legitimate or illegitimate civil status of the
individual would also affect his political rights or, in general, his relationship to the State. While,
indeed, provisions on "citizenship" could be found in the Civil Code, such provisions must be taken
in the context of private relations, the domain of civil law; particularly -

"Civil Law is that branch of law which has for its double purpose the organization of the
family and the regulation of property. It has thus [been] defined as the mass of precepts
which determine and regulate the relations of assistance, authority and obedience among
members of a family, and those which exist among members of a society for the protection of
private interests."37

In Yañez de Barnuevo vs. Fuster,38 the Court has held:

"In accordance with Article 9 of the Civil Code of Spain, x x x the laws relating to family rights
and duties, or to the status, condition and legal capacity of persons, govern Spaniards
although they reside in a foreign country; that, in consequence, 'all questions of a civil
nature, such as those dealing with the validity or nullity of the matrimonial bond, the domicile
of the husband and wife, their support, as between them, the separation of their properties,
the rules governing property, marital authority, division of conjugal property, the classification
of their property, legal causes for divorce, the extent of the latter, the authority to decree it,
and, in general, the civil effects of marriage and divorce upon the persons and properties of
the spouses, are questions that are governed exclusively by the national law of the husband
and wife."

The relevance of "citizenship" or "nationality" to Civil Law is best exemplified in Article 15 of the Civil
Code, stating that -

"Laws relating to family rights and duties, or to the status, condition and legal capacity of
persons are binding upon citizens of the Philippines, even though living abroad" -

that explains the need to incorporate in the code a reiteration of the Constitutional provisions on
citizenship. Similarly, citizenship is significant in civil relationships found in different parts of the Civil
Code,39 such as on successional rights and family relations.40 In adoption, for instance, an adopted
child would be considered the child of his adoptive parents and accorded the same rights as their
legitimate child but such legal fiction extended only to define his rights under civil law 41 and not his
political status.

Civil law provisions point to an obvious bias against illegitimacy. This discriminatory attitude may be
traced to the Spanish family and property laws, which, while defining proprietary and successional
rights of members of the family, provided distinctions in the rights of legitimate and illegitimate
children. In the monarchial set-up of old Spain, the distribution and inheritance of titles and wealth
were strictly according to bloodlines and the concern to keep these bloodlines uncontaminated by
foreign blood was paramount.

These distinctions between legitimacy and illegitimacy were codified in the Spanish Civil Code, and
the invidious discrimination survived when the Spanish Civil Code became the primary source of our
own Civil Code. Such distinction, however, remains and should remain only in the sphere of civil law
and not unduly impede or impinge on the domain of political law.

The proof of filiation or paternity for purposes of determining his citizenship status should thus be
deemed independent from and not inextricably tied up with that prescribed for civil law purposes.
The Civil Code or Family Code provisions on proof of filiation or paternity, although good law, do not
have preclusive effects on matters alien to personal and family relations. The ordinary rules on
evidence could well and should govern. For instance, the matter about pedigree is not necessarily
precluded from being applicable by the Civil Code or Family Code provisions.

Section 39, Rule 130, of the Rules of Court provides -

"Act or Declaration about pedigree. The act or declaration of a person deceased, or unable
to testify, in respect to the pedigree of another person related to him by birth or marriage,
may be received in evidence where it occurred before the controversy, and the relationship
between the two persons is shown by evidence other than such act or declaration. The word
`pedigree’ includes relationship, family genealogy, birth, marriage, death, the dates when
and the places where these facts occurred, and the names of the relatives. It embraces also
facts of family history intimately connected with pedigree."

For the above rule to apply, it would be necessary that (a) the declarant is already dead or unable to
testify, (b) the pedigree of a person must be at issue, (c) the declarant must be a relative of the
person whose pedigree is in question, (d) declaration must be made before the controversy has
occurred, and (e) the relationship between the declarant and the person whose pedigree is in
question must be shown by evidence other than such act or declaration.

Thus, the duly notarized declaration made by Ruby Kelley Mangahas, sister of Bessie Kelley Poe
submitted as Exhibit 20 before the COMELEC, might be accepted to prove the acts of Allan F. Poe,
recognizing his own paternal relationship with FPJ, i.e, living together with Bessie Kelley and his
children (including respondent FPJ) in one house, and as one family -

"I, Ruby Kelley Mangahas, of legal age and sound mind, presently residing in Stockton,
California, U.S.A., after being sworn in accordance with law do hereby declare that:

"1. I am the sister of the late Bessie Kelley Poe.

"2. Bessie Kelley Poe was the wife of Fernando Poe, Sr.
"3. Fernando and Bessie Poe had a son by the name of Ronald Allan Poe, more popularly
known in the Philippines as `Fernando Poe, Jr.,’ or `FPJ’.

"4. Ronald Allan Poe `FPJ’ was born on August 20, 1939 at St. Luke's Hospital, Magdalena
Street, Manila.

"x x x           x x x           x x x

"7. Fernando Poe Sr., and my sister Bessie, met and became engaged while they were
students at the University of the Philippines in 1936. I was also introduced to Fernando Poe,
Sr., by my sister that same year.

"8. Fernando Poe, Sr., and my sister Bessie had their first child in 1938.

"9. Fernando Poe, Sr., my sister Bessie and their first three children, Elizabeth, Ronald, Allan
and Fernando II, and myself lived together with our mother at our family's house on Dakota
St. (now Jorge Bocobo St.), Malate until the liberation of Manila in 1945, except for some
months between 1943-1944.

"10. Fernando Poe, Sr., and my sister, Bessie, were blessed with four (4) more children after
Ronald Allan Poe.

"x x x           x x x           x x x

"18. I am executing this Declaration to attest to the fact that my nephew, Ronald Allan Poe is
a natural born Filipino, and that he is the legitimate child of Fernando Poe, Sr.

"Done in City of Stockton, California, U.S.A., this 12th day of January 2004.

Ruby Kelley Mangahas Declarant DNA Testing

In case proof of filiation or paternity would be unlikely to satisfactorily establish or would be difficult to
obtain, DNA testing, which examines genetic codes obtained from body cells of the illegitimate child
and any physical residue of the long dead parent could be resorted to. A positive match would clear
up filiation or paternity. In Tijing vs. Court of Appeals,42 this Court has acknowledged the strong weight of
DNA testing -

"Parentage will still be resolved using conventional methods unless we adopt the modern and
scientific ways available. Fortunately, we have now the facility and expertise in using DNA test for
identification and parentage testing. The University of the Philippines Natural Science Research
Institute (UP-NSRI) DNA Analysis Laboratory has now the capability to conduct DNA typing using
short tandem repeat (STR) analysis. The analysis is based on the fact that the DNA of a child/person
has two (2) copies, one copy from the mother and the other from the father. The DNA from the
mother, the alleged father and the child are analyzed to establish parentage. Of course, being a
novel scientific technique, the use of DNA test as evidence is still open to challenge. Eventually, as
the appropriate case comes, courts should not hesitate to rule on the admissibility of DNA evidence.
For it was said, that courts should apply the results of science when competently obtained in aid of
situations presented, since to reject said result is to deny progress."

Petitioner’s Argument For Jurisprudential Conclusiveness


Petitioner would have it that even if Allan F. Poe were a Filipino citizen, he could not have
transmitted his citizenship to respondent FPJ, the latter being an illegitimate child. According to
petitioner, prior to his marriage to Bessie Kelley, Allan F. Poe, on July 5, 1936, contracted marriage
with a certain Paulita Gomez, making his subsequent marriage to Bessie Kelley bigamous and
respondent FPJ an illegitimate child. The veracity of the supposed certificate of marriage between
Allan F. Poe and Paulita Gomez could be most doubtful at best. But the documentary evidence
introduced by no less than respondent himself, consisting of a birth certificate of respondent and a
marriage certificate of his parents showed that FPJ was born on 20 August 1939 to a Filipino father
and an American mother who were married to each other a year later, or on 16 September 1940.
Birth to unmarried parents would make FPJ an illegitimate child. Petitioner contended that as an
illegitimate child, FPJ so followed the citizenship of his mother, Bessie Kelley, an American citizen,
basing his stand on the ruling of this Court in Morano vs. Vivo, 43 citing Chiongbian vs. de Leo44 and
Serra vs. Republic.45

On the above score, the disquisition made by amicus curiae Joaquin G. Bernas, SJ, is most
convincing; he states -

"We must analyze these cases and ask what the lis mota was in each of them. If the
pronouncement of the Court on jus sanguinis was on the lis mota, the pronouncement would
be a decision constituting doctrine under the rule of stare decisis. But if the pronouncement
was irrelevant to the lis mota, the pronouncement would not be a decision but a mere obiter
dictum which did not establish doctrine. I therefore invite the Court to look closely into these
cases.

"First, Morano vs. Vivo. The case was not about an illegitimate child of a Filipino father. It
was about a stepson of a Filipino, a stepson who was the child of a Chinese mother and a
Chinese father. The issue was whether the stepson followed the naturalization of the
stepfather. Nothing about jus sanguinis there. The stepson did not have the blood of the
naturalized stepfather.

"Second, Chiongbian vs. de Leon. This case was not about the illegitimate son of a Filipino
father. It was about a legitimate son of a father who had become Filipino by election to public
office before the 1935 Constitution pursuant to Article IV, Section 1(2) of the 1935
Constitution. No one was illegitimate here.

"Third, Serra vs. Republic. The case was not about the illegitimate son of a Filipino father.
Serra was an illegitimate child of a Chinese father and a Filipino mother. The issue was
whether one who was already a Filipino because of his mother who still needed to be
naturalized. There is nothing there about invidious jus sanguinis.

"Finally, Paa vs. Chan.46 This is a more complicated case. The case was about the
citizenship of Quintin Chan who was the son of Leoncio Chan. Quintin Chan claimed that his
father, Leoncio, was the illegitimate son of a Chinese father and a Filipino mother. Quintin
therefore argued that he got his citizenship from Leoncio, his father. But the Supreme Court
said that there was no valid proof that Leoncio was in fact the son of a Filipina mother. The
Court therefore concluded that Leoncio was not Filipino. If Leoncio was not Filipino, neither
was his son Quintin. Quintin therefore was not only not a natural-born Filipino but was not
even a Filipino.

"The Court should have stopped there. But instead it followed with an obiter dictum. The
Court said obiter that even if Leoncio, Quintin's father, were Filipino, Quintin would not be
Filipino because Quintin was illegitimate. This statement about Quintin, based on a contrary
to fact assumption, was absolutely unnecessary for the case. x x x It was obiter dictum, pure
and simple, simply repeating the obiter dictum in Morano vs. Vivo.

"x x x           x x x           x x x

"Aside from the fact that such a pronouncement would have no textual foundation in the
Constitution, it would also violate the equal protection clause of the Constitution not once but
twice. First, it would make an illegitimate distinction between a legitimate child and an
illegitimate child, and second, it would make an illegitimate distinction between the
illegitimate child of a Filipino father and the illegitimate child of a Filipino mother.

"The doctrine on constitutionally allowable distinctions was established long ago by People
vs. Cayat.47 I would grant that the distinction between legitimate children and illegitimate
children rests on real differences. x x x But real differences alone do not justify invidious
distinction. Real differences may justify distinction for one purpose but not for another
purpose.

"x x x What is the relevance of legitimacy or illegitimacy to elective public service? What
possible state interest can there be for disqualifying an illegitimate child from becoming a
public officer. It was not the fault of the child that his parents had illicit liaison. Why deprive
the child of the fullness of political rights for no fault of his own? To disqualify an illegitimate
child from holding an important public office is to punish him for the indiscretion of his
parents. There is neither justice nor rationality in that. And if there is neither justice nor
rationality in the distinction, then the distinction transgresses the equal protection clause and
must be reprobated."

The other amici curiae, Mr. Justice Vicente Mendoza (a former member of this Court), Professor
Ruben Balane and Dean Martin Magallona, at bottom, have expressed similar views. The thesis of
petitioner, unfortunately hinging solely on pure obiter dicta, should indeed fail.

Where jurisprudence regarded an illegitimate child as taking after the citizenship of its mother, it did
so for the benefit the child. It was to ensure a Filipino nationality for the illegitimate child of an alien
father in line with the assumption that the mother had custody, would exercise parental authority and
had the duty to support her illegitimate child. It was to help the child, not to prejudice or discriminate
against him.

The fact of the matter – perhaps the most significant consideration – is that the 1935 Constitution,
the fundamental law prevailing on the day, month and year of birth of respondent FPJ, can never be
more explicit than it is. Providing neither conditions nor distinctions, the Constitution states that
among the citizens of the Philippines are "those whose fathers are citizens of the Philippines." There
utterly is no cogent justification to prescribe conditions or distinctions where there clearly are none
provided.

In Sum –

(1) The Court, in the exercise of its power of judicial review, possesses jurisdiction over the
petition in G. R. No. 161824, filed under Rule 64, in relation to Rule 65, of the Revised Rules
of Civil Procedure. G.R. No. 161824 assails the resolution of the COMELEC for alleged
grave abuse of discretion in dismissing, for lack of merit, the petition in SPA No. 04-003
which has prayed for the disqualification of respondent FPJ from running for the position of
President in the 10th May 2004 national elections on the contention that FPJ has committed
material representation in his certificate of candidacy by representing himself to be a natural-
born citizen of the Philippines.

(2) The Court must dismiss, for lack of jurisdiction and prematurity, the petitions in G. R. No.
161434 and No. 161634 both having been directly elevated to this Court in the latter’s
capacity as the only tribunal to resolve a presidential and vice-presidential election contest
under the Constitution. Evidently, the primary jurisdiction of the Court can directly be invoked
only after, not before, the elections are held.

(3) In ascertaining, in G.R. No. 161824, whether grave abuse of discretion has been
committed by the COMELEC, it is necessary to take on the matter of whether or not
respondent FPJ is a natural-born citizen, which, in turn, depended on whether or not the
father of respondent, Allan F. Poe, would have himself been a Filipino citizen and, in the
affirmative, whether or not the alleged illegitimacy of respondent prevents him from taking
after the Filipino citizenship of his putative father. Any conclusion on the Filipino citizenship
of Lorenzo Pou could only be drawn from the presumption that having died in 1954 at 84
years old, Lorenzo would have been born sometime in the year 1870, when the Philippines
was under Spanish rule, and that San Carlos, Pangasinan, his place of residence upon his
death in 1954, in the absence of any other evidence, could have well been his place of
residence before death, such that Lorenzo Pou would have benefited from the "en masse
Filipinization" that the Philippine Bill had effected in 1902. That citizenship (of Lorenzo Pou),
if acquired, would thereby extend to his son, Allan F. Poe, father of respondent FPJ. The
1935 Constitution, during which regime respondent FPJ has seen first light, confers
citizenship to all persons whose fathers are Filipino citizens regardless of whether such
children are legitimate or illegitimate.

(4) But while the totality of the evidence may not establish conclusively that respondent FPJ
is a natural-born citizen of the Philippines, the evidence on hand still would preponderate in
his favor enough to hold that he cannot be held guilty of having made a material
misrepresentation in his certificate of candidacy in violation of Section 78, in relation to
Section 74, of the Omnibus Election Code. Petitioner has utterly failed to substantiate his
case before the Court, notwithstanding the ample opportunity given to the parties to present
their position and evidence, and to prove whether or not there has been material
misrepresentation, which, as so ruled in Romualdez-Marcos vs. COMELEC, 48 must not only
be material, but also deliberate and willful.

WHEREFORE, the Court RESOLVES to DISMISS –

1. G. R. No. 161434, entitled "Maria Jeanette C. Tecson and Felix B. Desiderio, Jr.,
Petitioners, versus Commission on Elections, Ronald Allan Kelley Poe (a.k.a. "Fernando
Poe, Jr.,) and Victorino X. Fornier, Respondents," and G. R. No. 161634, entitled "Zoilo
Antonio Velez, Petitioner, versus Ronald Allan Kelley Poe, a.k.a. Fernando Poe, Jr.,
Respondent," for want of jurisdiction.

2. G. R. No. 161824, entitled "Victorino X. Fornier, Petitioner, versus Hon. Commission on


Elections and Ronald Allan Kelley Poe, also known as Fernando Poe, Jr.," for failure to show
grave abuse of discretion on the part of respondent Commission on Elections in dismissing
the petition in SPA No. 04-003.

No Costs.

SO ORDERED.
G.R. No. 137000               August 9, 2000

CIRILO R. VALLES, petitioner, 
vs.
COMMISSION ON ELECTIONS and ROSALIND YBASCO LOPEZ, respondents.

DECISION

PURISIMA, J.:

This is a petition for certiorari under Rule 65, pursuant to Section 2, Rule 64 of the 1997 Rules of
Civil Procedure, assailing Resolutions dated July 17, 1998 and January 15, 1999, respectively, of
the Commission on Elections in SPA No. 98-336, dismissing the petition for disqualification filed by
the herein petitioner, Cirilo R. Valles, against private respondent Rosalind Ybasco Lopez, in the May
1998 elections for governor of Davao Oriental.

Rosalind Ybasco Lopez was born on May 16, 1934 in Napier Terrace, Broome, Western Australia, to
the spouses, Telesforo Ybasco, a Filipino citizen and native of Daet, Camarines Norte, and Theresa
Marquez, an Australian. In 1949, at the age of fifteen, she left Australia and came to settle in the
Philippines.

On June 27, 1952, she was married to Leopoldo Lopez, a Filipino citizen, at the Malate Catholic
Church in Manila. Since then, she has continuously participated in the electoral process not only as
a voter but as a candidate, as well. She served as Provincial Board Member of the Sangguniang
Panlalawigan of Davao Oriental. In 1992, she ran for and was elected governor of Davao Oriental.
Her election was contested by her opponent, Gil Taojo, Jr., in a petition for quo warranto, docketed
as EPC No. 92-54, alleging as ground therefor her alleged Australian citizenship. However, finding
no sufficient proof that respondent had renounced her Philippine citizenship, the Commission on
Elections en banc dismissed the petition, ratiocinating thus:

"A cursory reading of the records of this case vis-a-vis the impugned resolution shows that
respondent was able to produce documentary proofs of the Filipino citizenship of her late father...
and consequently, prove her own citizenship and filiation by virtue of the Principle of Jus Sanguinis,
the perorations of the petitioner to the contrary notwithstanding.

On the other hand, except for the three (3) alleged important documents . . . no other evidence
substantial in nature surfaced to confirm the allegations of petitioner that respondent is an Australian
citizen and not a Filipino. Express renunciation of citizenship as a mode of losing citizenship under
Commonwealth Act No. 63 is an equivocal and deliberate act with full awareness of its significance
and consequence. The evidence adduced by petitioner are inadequate, nay meager, to prove that
respondent contemplated renunciation of her Filipino citizenship". 1

In the 1995 local elections, respondent Rosalind Ybasco Lopez ran for re-election as governor of
Davao Oriental. Her opponent, Francisco Rabat, filed a petition for disqualification, docketed as SPA
No. 95-066 before the COMELEC, First Division, contesting her Filipino citizenship but the said
petition was likewise dismissed by the COMELEC, reiterating substantially its decision in EPC 92-54.
The citizenship of private respondent was once again raised as an issue when she ran for re-
election as governor of Davao Oriental in the May 11, 1998 elections. Her candidacy was questioned
by the herein petitioner, Cirilo Valles, in SPA No. 98-336.

On July 17, 1998, the COMELEC’s First Division came out with a Resolution dismissing the petition,
and disposing as follows:

"Assuming arguendo that res judicata does not apply and We are to dispose the instant case on the
merits trying it de novo, the above table definitely shows that petitioner herein has presented no new
evidence to disturb the Resolution of this Commission in SPA No. 95-066. The present petition
merely restates the same matters and incidents already passed upon by this Commission not just in
1995 Resolution but likewise in the Resolution of EPC No. 92-54. Not having put forth any new
evidence and matter substantial in nature, persuasive in character or sufficiently provocative to
compel reversal of such Resolutions, the dismissal of the present petition follows as a matter of
course.

x x x           x x x          x x x

"WHEREFORE, premises considered and there being no new matters and issues tendered, We find
no convincing reason or impressive explanation to disturb and reverse the Resolutions promulgated
by this Commission in EPC 92-54 and SPA. 95-066. This Commission RESOLVES as it hereby
RESOLVES to DISMISS the present petition.

SO ORDERED." 2

Petitioner interposed a motion for reconsideration of the aforesaid Resolution but to no avail. The
same was denied by the COMELEC in its en banc Resolution of January 15, 1999.

Undaunted, petitioner found his way to this Court via the present petition; questioning the citizenship
of private respondent Rosalind Ybasco Lopez.

The Commission on Elections ruled that private respondent Rosalind Ybasco Lopez is a Filipino
citizen and therefore, qualified to run for a public office because (1) her father, Telesforo Ybasco, is
a Filipino citizen, and by virtue of the principle of jus sanguinis she was a Filipino citizen under the
1987 Philippine Constitution; (2) she was married to a Filipino, thereby making her also a Filipino
citizen ipso jure under Section 4 of Commonwealth Act 473; (3) and that, she renounced her
Australian citizenship on January 15, 1992 before the Department of Immigration and Ethnic Affairs
of Australia and her Australian passport was accordingly cancelled as certified to by the Australian
Embassy in Manila; and (4) furthermore, there are the COMELEC Resolutions in EPC No. 92-54 and
SPA Case No. 95-066, declaring her a Filipino citizen duly qualified to run for the elective position of
Davao Oriental governor.

Petitioner, on the other hand, maintains that the private respondent is an Australian citizen, placing
reliance on the admitted facts that:

a) In 1988, private respondent registered herself with the Bureau of Immigration as an


Australian national and was issued Alien Certificate of Registration No. 404695 dated
September 19, 1988;

b) On even date, she applied for the issuance of an Immigrant Certificate of Residence
(ICR), and
c) She was issued Australian Passport No. H700888 on March 3, 1988.

Petitioner theorizes that under the aforestated facts and circumstances, the private respondent had
renounced her Filipino citizenship. He contends that in her application for alien certificate of
registration and immigrant certificate of residence, private respondent expressly declared under oath
that she was a citizen or subject of Australia; and said declaration forfeited her Philippine citizenship,
and operated to disqualify her to run for elective office.

As regards the COMELEC’s finding that private respondent had renounced her Australian citizenship
on January 15, 1992 before the Department of Immigration and Ethnic Affairs of Australia and had
her Australian passport cancelled on February 11, 1992, as certified to by the Australian Embassy
here in Manila, petitioner argues that the said acts did not automatically restore the status of private
respondent as a Filipino citizen. According to petitioner, for the private respondent to reacquire
Philippine citizenship she must comply with the mandatory requirements for repatriation under
Republic Act 8171; and the election of private respondent to public office did not mean the
restoration of her Filipino citizenship since the private respondent was not legally repatriated.
Coupled with her alleged renunciation of Australian citizenship, private respondent has effectively
become a stateless person and as such, is disqualified to run for a public office in the Philippines;
petitioner concluded.

Petitioner theorizes further that the Commission on Elections erred in applying the principle of res
judicata to the case under consideration; citing the ruling in Moy Ya Lim Yao vs. Commissioner of
Immigration, that:

"xxx Everytime the citizenship of a person is material or indispensable in a judicial or administrative


case, whatever the corresponding court or administrative authority decides therein as to such
citizenship is generally not considered as res adjudicata, hence it has to be threshed out again and
again as the occasion may demand. xxx"

The petition is unmeritorious.

The Philippine law on citizenship adheres to the principle of jus sanguinis. Thereunder, a child
follows the nationality or citizenship of the parents regardless of the place of his/her birth, as
opposed to the doctrine of jus soli which determines nationality or citizenship on the basis of place of
birth.

Private respondent Rosalind Ybasco Lopez was born on May 16, 1934 in Napier Terrace, Broome,
Western Australia, to the spouses, Telesforo Ybasco, a Filipino citizen and native of Daet,
Camarines Norte, and Theresa Marquez, an Australian. Historically, this was a year before the 1935
Constitution took into effect and at that time, what served as the Constitution of the Philippines were
the principal organic acts by which the United States governed the country. These were the
Philippine Bill of July 1, 1902 and the Philippine Autonomy Act of August 29, 1916, also known as
the Jones Law.

Among others, these laws defined who were deemed to be citizens of the Philippine islands. The
Philippine Bill of 1902 defined Philippine citizens as:

SEC. 4 xxx all inhabitants of the Philippine Islands continuing to reside therein who were Spanish
subjects on the eleventh day of April, eighteen hundred and ninety-nine, and then resided in the
Philippine Islands, and their children born subsequent thereto, shall be deemed and held to be
citizens of the Philippine Islands and as such entitled to the protection of the United States, except
such as shall have elected to preserve their allegiance to the Crown of Spain in accordance with the
provisions of the treaty of peace between the United States and Spain signed at Paris December
tenth, eighteen hundred and ninety-eight. (underscoring ours)

The Jones Law, on the other hand, provides:

SEC. 2 That all inhabitants of the Philippine Islands who were Spanish subjects on the eleventh day
of April, eighteen hundred and ninety-nine, and then resided in said Islands, and their children born
subsequent thereto, shall be deemed and held to be citizens of the Philippine Islands, except such
as shall have elected to preserve their allegiance to the Crown of Spain in accordance with the
provisions of the treaty of peace between the United States and Spain, signed at Paris December
tenth, eighteen hundred and ninety-eight, and except such others as have since become citizens of
some other country: Provided, That the Philippine Legislature, herein provided for, is hereby
authorized to provide by law for the acquisition of Philippine citizenship by those natives of the
Philippine Islands who cannot come within the foregoing provisions, the natives of the insular
possessions of the United States, and such other persons residing in the Philippine Islands who are
citizens of the United States, or who could become citizens of the United States under the laws of
the United States if residing therein. (underscoring ours)

Under both organic acts, all inhabitants of the Philippines who were Spanish subjects on April 11,
1899 and resided therein including their children are deemed to be Philippine citizens. Private
respondent’s father, Telesforo Ybasco, was born on January 5, 1879 in Daet, Camarines Norte, a
fact duly evidenced by a certified true copy of an entry in the Registry of Births. Thus, under the
Philippine Bill of 1902 and the Jones Law, Telesforo Ybasco was deemed to be a Philippine citizen.
By virtue of the same laws, which were the laws in force at the time of her birth, Telesforo’s
daughter, herein private respondent Rosalind Ybasco Lopez, is likewise a citizen of the Philippines.

The signing into law of the 1935 Philippine Constitution has established the principle of jus
sanguinis as basis for the acquisition of Philippine citizenship, to wit:

(1) Those who are citizens of the Philippine Islands at the time of the adoption of this
Constitution.

(2) Those born in the Philippine Islands of foreign parents who, before the adoption of this
Constitution had been elected to public office in the Philippine Islands.

(3) Those whose fathers are citizens of the Philippines.

(4) Those whose mothers are citizens of the Philippines and, upon reaching the age of
majority, elect Philippine citizenship.

(5) Those who are naturalized in accordance with law.

So also, the principle of jus sanguinis, which confers citizenship by virtue of blood relationship, was
subsequently retained under the 1973 and 1987 Constitutions. Thus, the herein private respondent,
4  5 

Rosalind Ybasco Lopez, is a Filipino citizen, having been born to a Filipino father. The fact of her
being born in Australia is not tantamount to her losing her Philippine citizenship. If Australia follows
the principle of jus soli, then at most, private respondent can also claim Australian citizenship
resulting to her possession of dual citizenship.

Petitioner also contends that even on the assumption that the private respondent is a Filipino citizen,
she has nonetheless renounced her Philippine citizenship. To buttress this contention, petitioner
cited private respondent’s application for an Alien Certificate of Registration (ACR) and Immigrant
Certificate of Residence (ICR), on September 19, 1988, and the issuance to her of an Australian
passport on March 3, 1988.

Under Commonwealth Act No. 63, a Filipino citizen may lose his citizenship:

(1) By naturalization in a foreign country;

(2) By express renunciation of citizenship;

(3) By subscribing to an oath of allegiance to support the constitution or laws of a foreign


country upon attaining twenty-one years of age or more;

(4) By accepting commission in the military, naval or air service of a foreign country;

(5) By cancellation of the certificate of naturalization;

(6) By having been declared by competent authority, a deserter of the Philippine armed
forces in time of war, unless subsequently, a plenary pardon or amnesty has been granted:
and

(7) In case of a woman, upon her marriage, to a foreigner if, by virtue of the laws in force in
her husband’s country, she acquires his nationality.

In order that citizenship may be lost by renunciation, such renunciation must be express. Petitioner’s
contention that the application of private respondent for an alien certificate of registration, and her
Australian passport, is bereft of merit. This issue was put to rest in the case of Aznar vs.
COMELEC and in the more recent case of Mercado vs. Manzano and COMELEC.
6  7

In the case of Aznar, the Court ruled that the mere fact that respondent Osmena was a holder of a
certificate stating that he is an American did not mean that he is no longer a Filipino, and that an
application for an alien certificate of registration was not tantamount to renunciation of his Philippine
citizenship.

And, in Mercado vs. Manzano and COMELEC, it was held that the fact that respondent Manzano
was registered as an American citizen in the Bureau of Immigration and Deportation and was
holding an American passport on April 22, 1997, only a year before he filed a certificate of candidacy
for vice-mayor of Makati, were just assertions of his American nationality before the termination of
his American citizenship.

Thus, the mere fact that private respondent Rosalind Ybasco Lopez was a holder of an Australian
passport and had an alien certificate of registration are not acts constituting an effective renunciation
of citizenship and do not militate against her claim of Filipino citizenship. For renunciation to
effectively result in the loss of citizenship, the same must be express. As held by this court in the

aforecited case of Aznar, an application for an alien certificate of registration does not amount to an
express renunciation or repudiation of one’s citizenship. The application of the herein private
respondent for an alien certificate of registration, and her holding of an Australian passport, as in the
case of Mercado vs. Manzano, were mere acts of assertion of her Australian citizenship before she
effectively renounced the same. Thus, at the most, private respondent had dual citizenship - she
was an Australian and a Filipino, as well.
Moreover, under Commonwealth Act 63, the fact that a child of Filipino parent/s was born in another
country has not been included as a ground for losing one’s Philippine citizenship. Since private
respondent did not lose or renounce her Philippine citizenship, petitioner’s claim that respondent
must go through the process of repatriation does not hold water.

Petitioner also maintains that even on the assumption that the private respondent had dual
citizenship, still, she is disqualified to run for governor of Davao Oriental; citing Section 40 of
Republic Act 7160 otherwise known as the Local Government Code of 1991, which states:

"SEC. 40. Disqualifications. The following persons are disqualified from running for any elective local
position:

x x x           x x x          x x x

(d) Those with dual citizenship;

x x x           x x x          x x x

Again, petitioner’s contention is untenable.

In the aforecited case of Mercado vs. Manzano, the Court clarified "dual citizenship" as used in the
Local Government Code and reconciled the same with Article IV, Section 5 of the 1987 Constitution
on dual allegiance. Recognizing situations in which a Filipino citizen may, without performing any
9

act, and as an involuntary consequence of the conflicting laws of different countries, be also a citizen
of another state, the Court explained that dual citizenship as a disqualification must refer to citizens
with dual allegiance. The Court succinctly pronounced:

"xxx the phrase ‘dual citizenship’ in R.A. No. 7160, xxx 40 (d) and in R.A. No. 7854, xxx 20 must be
understood as referring to ‘dual allegiance’. Consequently, persons with mere dual citizenship do not
fall under this disqualification."

Thus, the fact that the private respondent had dual citizenship did not automatically disqualify her
from running for a public office. Furthermore, it was ruled that for candidates with dual citizenship, it
is enough that they elect Philippine citizenship upon the filing of their certificate of candidacy, to
terminate their status as persons with dual citizenship. The filing of a certificate of candidacy sufficed
10

to renounce foreign citizenship, effectively removing any disqualification as a dual citizen. This is so
11 

because in the certificate of candidacy, one declares that he/she is a Filipino citizen and that he/she
will support and defend the Constitution of the Philippines and will maintain true faith and allegiance
thereto. Such declaration, which is under oath, operates as an effective renunciation of foreign
citizenship. Therefore, when the herein private respondent filed her certificate of candidacy in 1992,
such fact alone terminated her Australian citizenship.

Then, too, it is significant to note that on January 15 1992, private respondent executed a
Declaration of Renunciation of Australian Citizenship, duly registered in the Department of
Immigration and Ethnic Affairs of Australia on May 12, 1992. And, as a result, on February 11, 1992,
the Australian passport of private respondent was cancelled, as certified to by Second Secretary
Richard F. Munro of the Embassy of Australia in Manila. As aptly appreciated by the COMELEC, the
aforesaid acts were enough to settle the issue of the alleged dual citizenship of Rosalind Ybasco
Lopez. Since her renunciation was effective, petitioner’s claim that private respondent must go
through the whole process of repatriation holds no water.
Petitioner maintains further that when citizenship is raised as an issue in judicial or administrative
proceedings, the resolution or decision thereon is generally not considered res judicata in any
subsequent proceeding challenging the same; citing the case of Moy Ya Lim Yao vs. Commissioner
of Immigration. He insists that the same issue of citizenship may be threshed out anew.
12 

Petitioner is correct insofar as the general rule is concerned, i.e. the principle of res


judicata generally does not apply in cases hinging on the issue of citizenship. However, in the case
of Burca vs. Republic, an exception to this general rule was recognized. The Court ruled in that case
13 

that in order that the doctrine of res judicata may be applied in cases of citizenship, the following
must be present:

1) a person’s citizenship be raised as a material issue in a controversy where said person is


a party;

2) the Solicitor General or his authorized representative took active part in the resolution
thereof, and

3) the finding on citizenship is affirmed by this Court.

Although the general rule was set forth in the case of Moy Ya Lim Yao, the case did not foreclose
the weight of prior rulings on citizenship. It elucidated that reliance may somehow be placed on
these antecedent official findings, though not really binding, to make the effort easier or
simpler. Indeed, there appears sufficient basis to rely on the prior rulings of the Commission on
14 

Elections in SPA. No. 95-066 and EPC 92-54 which resolved the issue of citizenship in favor of the
herein private respondent. The evidence adduced by petitioner is substantially the same evidence
presented in these two prior cases. Petitioner failed to show any new evidence or supervening event
to warrant a reversal of such prior resolutions. However, the procedural issue notwithstanding,
considered on the merits, the petition cannot prosper.

WHEREFORE, the petition is hereby DISMISSED and the COMELEC Resolutions, dated July 17,
1998 and January 15, 1999, respectively, in SPA No. 98-336 AFFIRMED.

Private respondent Rosalind Ybasco Lopez is hereby adjudged qualified to run for governor of
Davao Oriental. No pronouncement as to costs.

SO ORDERED.

BAR MATTER No. 914 October 1, 1999

RE: APPLICATION FOR ADMISSION TO THE PHILIPPINE BAR,

vs.

VICENTE D. CHING, applicant.

RESOLUTION

 
KAPUNAN, J.:

Can a legitimate child born under the 1935 Constitution of a Filipino mother and an alien father
validly elect Philippine citizenship fourteen (14) years after he has reached the age of majority? This
is the question sought to be resolved in the present case involving the application for admission to
the Philippine Bar of Vicente D. Ching.

The facts of this case are as follows:

Vicente D. Ching, the legitimate son of the spouses Tat Ching, a Chinese citizen, and Prescila A.
Dulay, a Filipino, was born in Francia West, Tubao, La Union on 11 April 1964. Since his birth, Ching
has resided in the Philippines.

On 17 July 1998, Ching, after having completed a Bachelor of Laws course at the St. Louis
University in Baguio City, filed an application to take the 1998 Bar Examinations. In a Resolution of
this Court, dated 1 September 1998, he was allowed to take the Bar Examinations, subject to the
condition that he must submit to the Court proof of his Philippine citizenship.

In compliance with the above resolution, Ching submitted on 18 November 1998, the following
documents:

1. Certification, dated 9 June 1986, issued by the Board of Accountancy of the


Professional Regulations Commission showing that Ching is a certified public
accountant;

2. Voter Certification, dated 14 June 1997, issued by Elizabeth B. Cerezo, Election


Officer of the Commission on Elections (COMELEC) in Tubao La Union showing that
Ching is a registered voter of the said place; and

3. Certification, dated 12 October 1998, also issued by Elizabeth B. Cerezo, showing


that Ching was elected as a member of the Sangguniang Bayan of Tubao, La Union
during the 12 May 1992 synchronized elections.

On 5 April 1999, the results of the 1998 Bar Examinations were released and Ching was one of the
successful Bar examinees. The oath-taking of the successful Bar examinees was scheduled on 5
May 1999. However, because of the questionable status of Ching's citizenship, he was not allowed
to take his oath. Pursuant to the resolution of this Court, dated 20 April 1999, he was required to
submit further proof of his citizenship. In the same resolution, the Office of the Solicitor General
(OSG) was required to file a comment on Ching's petition for admission to the bar and on the
documents evidencing his Philippine citizenship.

The OSG filed its comment on 8 July 1999, stating that Ching, being the "legitimate child of a
Chinese father and a Filipino mother born under the 1935 Constitution was a Chinese citizen and
continued to be so, unless upon reaching the age of majority he elected Philippine citizenship" 1 in
strict compliance with the provisions of Commonwealth Act No. 625 entitled "An Act Providing for the
Manner in which the Option to Elect Philippine Citizenship shall be Declared by a Person Whose
Mother is a Filipino Citizen." The OSG adds that "(w)hat he acquired at best was only an inchoate
Philippine citizenship which he could perfect by election upon reaching the age of majority." 2 In this
regard, the OSG clarifies that "two (2) conditions must concur in order that the election of Philippine
citizenship may be effective, namely: (a) the mother of the person making the election must be a
citizen of the Philippines; and (b) said election must be made upon reaching the age of
majority." 3 The OSG then explains the meaning of the phrase "upon reaching the age of majority:"

The clause "upon reaching the age of majority" has been construed to mean a
reasonable time after reaching the age of majority which had been interpreted by the
Secretary of Justice to be three (3) years (VELAYO, supra at p. 51 citing Op., Sec. of
Justice No. 70, s. 1940, Feb. 27, 1940). Said period may be extended under certain
circumstances, as when a (sic) person concerned has always considered himself a
Filipino (ibid., citing Op. Nos. 355 and 422, s. 1955; 3, 12, 46, 86 and 97, s. 1953).
But in Cuenco, it was held that an election done after over seven (7) years was not
made within a reasonable time.

In conclusion, the OSG points out that Ching has not formally elected Philippine citizenship and, if
ever he does, it would already be beyond the "reasonable time" allowed by present jurisprudence.
However, due to the peculiar circumstances surrounding Ching's case, the OSG recommends the
relaxation of the standing rule on the construction of the phrase "reasonable period" and the
allowance of Ching to elect Philippine citizenship in accordance with C.A. No. 625 prior to taking his
oath as a member of the Philippine Bar.

On 27 July 1999, Ching filed a Manifestation, attaching therewith his Affidavit of Election of
Philippine Citizenship and his Oath of Allegiance, both dated 15 July 1999. In his Manifestation,
Ching states:

1. I have always considered myself as a Filipino;

2. I was registered as a Filipino and consistently declared myself as one in my school


records and other official documents;

3. I am practicing a profession (Certified Public Accountant) reserved for Filipino


citizens;

4. I participated in electoral process[es] since the time I was eligible to vote;

5. I had served the people of Tubao, La Union as a member of the Sangguniang


Bayan from 1992 to 1995;

6. I elected Philippine citizenship on July 15, 1999 in accordance with


Commonwealth Act No. 625;

7. My election was expressed in a statement signed and sworn to by me before a


notary public;

8. I accompanied my election of Philippine citizenship with the oath of allegiance to


the Constitution and the Government of the Philippines;

9. I filed my election of Philippine citizenship and my oath of allegiance to (sic) the


Civil Registrar of Tubao La Union, and

10. I paid the amount of TEN PESOS (Ps. 10.00) as filing fees.
Since Ching has already elected Philippine citizenship on 15 July 1999, the question raised is
whether he has elected Philippine citizenship within a "reasonable time." In the affirmative, whether
his citizenship by election retroacted to the time he took the bar examination.

When Ching was born in 1964, the governing charter was the 1935 Constitution. Under Article IV,
Section 1(3) of the 1935 Constitution, the citizenship of a legitimate child born of a Filipino mother
and an alien father followed the citizenship of the father, unless, upon reaching the age of majority,
the child elected Philippine citizenship. 4 This right to elect Philippine citizenship was recognized in
the 1973 Constitution when it provided that "(t)hose who elect Philippine citizenship pursuant to the
provisions of the Constitution of nineteen hundred and thirty-five" are citizens of the
Philippines. 5 Likewise, this recognition by the 1973 Constitution was carried over to the 1987
Constitution which states that "(t)hose born before January 17, 1973 of Filipino mothers, who elect
Philippine citizenship upon reaching the age of majority" are Philippine citizens. 6 It should be noted,
however, that the 1973 and 1987 Constitutional provisions on the election of Philippine citizenship
should not be understood as having a curative effect on any irregularity in the acquisition of
citizenship for those covered by the 1935 Constitution. 7 If the citizenship of a person was subject to
challenge under the old charter, it remains subject to challenge under the new charter even if the
judicial challenge had not been commenced before the effectivity of the new Constitution. 8

C.A. No. 625 which was enacted pursuant to Section 1(3), Article IV of the 1935 Constitution,
prescribes the procedure that should be followed in order to make a valid election of Philippine
citizenship. Under Section 1 thereof, legitimate children born of Filipino mothers may elect Philippine
citizenship by expressing such intention "in a statement to be signed and sworn to by the party
concerned before any officer authorized to administer oaths, and shall be filed with the nearest civil
registry. The said party shall accompany the aforesaid statement with the oath of allegiance to the
Constitution and the Government of the Philippines."

However, the 1935 Constitution and C.A. No. 625 did not prescribe a time period within which the
election of Philippine citizenship should be made. The 1935 Charter only provides that the election
should be made "upon reaching the age of majority." The age of majority then commenced upon
reaching twenty-one (21) years. 9 In the opinions of the Secretary of Justice on cases involving the
validity of election of Philippine citizenship, this dilemma was resolved by basing the time period on
the decisions of this Court prior to the effectivity of the 1935 Constitution. In these decisions, the
proper period for electing Philippine citizenship was, in turn, based on the pronouncements of the
Department of State of the United States Government to the effect that the election should be made
within a "reasonable time" after attaining the age of majority. 10 The phrase "reasonable time" has
been interpreted to mean that the election should be made within three (3) years from reaching the
age of
majority. 11 However, we held in Cuenco vs. Secretary of Justice, 12 that the three (3) year period is
not an inflexible rule. We said:

It is true that this clause has been construed to mean a reasonable period after
reaching the age of majority, and that the Secretary of Justice has ruled that three (3)
years is the reasonable time to elect Philippine citizenship under the constitutional
provision adverted to above, which period may be extended under certain
circumstances, as when the person concerned has always considered himself a
Filipino. 13

However, we cautioned in Cuenco that the extension of the option to elect Philippine citizenship is
not indefinite:
Regardless of the foregoing, petitioner was born on February 16, 1923. He became
of age on February 16, 1944. His election of citizenship was made on May 15, 1951,
when he was over twenty-eight (28) years of age, or over seven (7) years after he
had reached the age of majority. It is clear that said election has not been made
"upon reaching the age of majority." 14

In the present case, Ching, having been born on 11 April 1964, was already thirty-five (35) years old
when he complied with the requirements of C.A. No. 625 on 15 June 1999, or over fourteen (14)
years after he had reached the age of majority. Based on the interpretation of the phrase "upon
reaching the age of majority," Ching's election was clearly beyond, by any reasonable yardstick, the
allowable period within which to exercise the privilege. It should be stated, in this connection, that
the special circumstances invoked by Ching, i.e., his continuous and uninterrupted stay in the
Philippines and his being a certified public accountant, a registered voter and a former elected public
official, cannot vest in him Philippine citizenship as the law specifically lays down the requirements
for acquisition of Philippine citizenship by election.

Definitely, the so-called special circumstances cannot constitute what Ching erroneously labels as
informal election of citizenship. Ching cannot find a refuge in the case of In re: Florencio
Mallare, 15 the pertinent portion of which reads:

And even assuming arguendo that Ana Mallare were (sic) legally married to an alien,
Esteban's exercise of the right of suffrage when he came of age, constitutes a
positive act of election of Philippine citizenship. It has been established that Esteban
Mallare was a registered voter as of April 14, 1928, and that as early as 1925 (when
he was about 22 years old), Esteban was already participating in the elections and
campaigning for certain candidate[s]. These acts are sufficient to show his
preference for Philippine citizenship. 16

Ching's reliance on Mallare is misplaced. The facts and circumstances obtaining therein are very
different from those in the present case, thus, negating its applicability. First, Esteban Mallare was
born before the effectivity of the 1935 Constitution and the enactment of C.A. No. 625. Hence, the
requirements and procedures prescribed under the 1935 Constitution and C.A. No. 625 for electing
Philippine citizenship would not be applicable to him. Second, the ruling in Mallare was an obiter
since, as correctly pointed out by the OSG, it was not necessary for Esteban Mallare to elect
Philippine citizenship because he was already a Filipino, he being a natural child of a Filipino
mother. In this regard, the Court stated:

Esteban Mallare, natural child of Ana Mallare, a Filipina, is therefore himself a


Filipino, and no other act would be necessary to confer on him all the rights and
privileges attached to Philippine citizenship (U.S. vs. Ong Tianse, 29 Phil. 332;
Santos Co vs. Government of the Philippine Islands, 42 Phil. 543, Serra vs. Republic,
L-4223, May 12, 1952, Sy Quimsuan vs. Republic, L-4693, Feb. 16, 1953; Pitallano
vs. Republic, L-5111, June 28, 1954). Neither could any act be taken on the
erroneous belief that he is a non-filipino divest him of the citizenship privileges to
which he is rightfully entitled. 17

The ruling in Mallare was reiterated and further elaborated in Co vs. Electoral Tribunal of the House
of Representatives, 18 where we held:

We have jurisprudence that defines "election" as both a formal and an informal


process.
In the case of In re: Florencio Mallare (59 SCRA 45 [1974]), the Court held that the
exercise of the right of suffrage and the participation in election exercises constitute a
positive act of election of Philippine citizenship. In the exact pronouncement of the
Court, we held:

Esteban's exercise of the right of suffrage when he came of age


constitutes a positive act of Philippine citizenship. (p. 52: emphasis
supplied)

The private respondent did more than merely exercise his right of suffrage. He has established his
life here in the Philippines.

For those in the peculiar situation of the respondent who cannot be excepted to have
elected Philippine citizenship as they were already citizens, we apply the In Re
Mallare rule.

xxx xxx xxx

The filing of sworn statement or formal declaration is a requirement for those who
still have to elect citizenship. For those already Filipinos when the time to elect came
up, there are acts of deliberate choice which cannot be less binding. Entering a
profession open only to Filipinos, serving in public office where citizenship is a
qualification, voting during election time, running for public office, and other
categorical acts of similar nature are themselves formal manifestations for these
persons.

An election of Philippine citizenship presupposes that the person electing is an alien.


Or his status is doubtful because he is a national of two countries. There is no doubt
in this case about Mr. Ong's being a Filipino when he turned twenty-one (21).

We repeat that any election of Philippine citizenship on the part of the private
respondent would not only have been superfluous but it would also have resulted in
an absurdity. How can a Filipino citizen elect Philippine citizenship? 19

The Court, like the OSG, is sympathetic with the plight of Ching. However, even if we consider the
special circumstances in the life of Ching like his having lived in the Philippines all his life and his
consistent belief that he is a Filipino, controlling statutes and jurisprudence constrain us to disagree
with the recommendation of the OSG. Consequently, we hold that Ching failed to validly elect
Philippine citizenship. The span of fourteen (14) years that lapsed from the time he reached the age
of majority until he finally expressed his intention to elect Philippine citizenship is clearly way beyond
the contemplation of the requirement of electing "upon reaching the age of majority." Moreover,
Ching has offered no reason why he delayed his election of Philippine citizenship. The prescribed
procedure in electing Philippine citizenship is certainly not a tedious and painstaking process. All that
is required of the elector is to execute an affidavit of election of Philippine citizenship and, thereafter,
file the same with the nearest civil registry. Ching's unreasonable and unexplained delay in making
his election cannot be simply glossed over.

Philippine citizenship can never be treated like a commodity that can be claimed when needed and
suppressed when convenient. 20 One who is privileged to elect Philippine citizenship has only an
inchoate right to such citizenship. As such, he should avail of the right with fervor, enthusiasm and
promptitude. Sadly, in this case, Ching slept on his opportunity to elect Philippine citizenship and, as
a result. this golden privilege slipped away from his grasp.
IN VIEW OF THE FOREGOING, the Court Resolves to DENY Vicente D. Ching's application for
admission to the Philippine Bar.

SO ORDERED.

Bengson v HRET G.R. No 142840, May 7, 2001


Bengson v House of Representatives Electoral Tribunal 
G.R. No 142840, May 7, 2001

Facts: The citizenship of Teodoro Cruz, a member of the HOR, is being questioned on the ground that he
is not a natural-born citizen of the Philippines.

Cruz was born in the Philippines in 1960, the time when the acquisition of citizenship rule was still jus soli.
However, he enlisted to the US Marine Corps and he was naturalized as US citizen in connection
therewith. He reacquired Philippine citizenship through repatriation under RA 2630 and ran for and was
elected as a representative. When his nationality was questioned by petitioner, the HRET decided that
Cruz was a natural born citizen of the Philippines.

Issue: WON Cruz is a natural born citizen of the Philippines.

Held: YES. Natural-born citizens "are those citizens of the Philippines from birth without having to perform
any act to acquire or perfect his Philippine citezenship." On the other hand, naturalized citizens are those
who have become Filipino citizens through naturalization, generally under Commonwealth Act No. 473,
otherwise known as the Revised Naturalization Law, which repealed the former Naturalization Law (Act
No. 2927), and by Republic Act No. 530.11 To be naturalized, an applicant has to prove that he
possesses all the qualifications12 and none of the disqualification.

Filipino citizens who have lost their citizenship may however reacquire the same in the manner provided
by law. Commonwealth Act. No. (C.A. No. 63), enumerates the three modes by which Philippine
citizenship may be reacquired by a former citizen: (1) by naturalization, (2) by repatriation, and (3) by
direct act of Congress.

Naturalization is mode for both acquisition and reacquisition of Philippine citizenship. As a mode of initially
acquiring Philippine citizenship, naturalization is governed by Commonwealth Act No. 473, as amended.
On the other hand, naturalization as a mode for reacquiring Philippine citizenship is governed by
Commonwealth Act No. 63.16 Under this law, a former Filipino citizen who wishes to reacquire Philippine
citizenship must possess certain qualifications and none of the disqualification mentioned in Section 4 of
C.A. 473.

Repatriation, on the other hand, may be had under various statutes by those who lost their citizenship due
to: (1) desertion of the armed forces; services in the armed forces of the allied forces in World War II; (3)
service in the Armed Forces of the United States at any other time, (4) marriage of a Filipino woman to an
alien; and (5) political economic necessity.

As distinguished from the lengthy process of naturalization, repatriation simply consists of the taking of an
oath of allegiance to the Republic of the Philippine and registering said oath in the Local Civil Registry of
the place where the person concerned resides or last resided.

Moreover, repatriation results in the recovery of the original nationality. This means that a naturalized
Filipino who lost his citizenship will be restored to his prior status as a naturalized Filipino citizen. On the
other hand, if he was originally a natural-born citizen before he lost his Philippine citizenship, he will be
restored to his former status as a natural-born Filipino.

In respondent Cruz's case, he lost his Filipino citizenship when he rendered service in the Armed Forces
of the United States. However, he subsequently reacquired Philippine citizenship under R.A. No. 2630.

Having thus taken the required oath of allegiance to the Republic and having registered the same in the
Civil Registry of Magantarem, Pangasinan in accordance with the aforecited provision, respondent Cruz is
deemed to have recovered his original status as a natural-born citizen, a status which he acquired at birth
as the son of a Filipino father. It bears stressing that the act of repatriation allows him to recover, or return
to, his original status before he lost his Philippine citizenship

G.R. Nos. 92191-92             July 30, 1991

ANTONIO Y. CO, petitioner, 
vs.
ELECTORAL TRIBUNAL OF THE HOUSE OF REPRESENTATIVES AND JOSE ONG,
JR., respondents.

G.R. Nos. 92202-03             July 30, 1991

SIXTO T. BALANQUIT, JR., petitioner, 


vs.
ELECTORAL TRIBUNAL OF THE HOUSE OF REPRESENTATIVES AND JOSE ONG,
JR., respondents.

Hechanova & Associates for petitioner Co.


Brillantes, Nachura, Navarro and Arcilla Law Offices for respondent Ong, Jr.

GUTIERREZ, JR., J.:

The petitioners come to this Court asking for the setting aside and reversal of a decision of the
House of Representatives Electoral Tribunal (HRET).

The HRET declared that respondent Jose Ong, Jr. is a natural born Filipino citizen and a resident of
Laoang, Northern Samar for voting purposes. The sole issue before us is whether or not, in making
that determination, the HRET acted with grave abuse of discretion.

On May 11, 1987, the congressional election for the second district of Northern Samar was held.

Among the candidates who vied for the position of representative in the second legislative district of
Northern Samar are the petitioners, Sixto Balinquit and Antonio Co and the private respondent, Jose
Ong, Jr.

Respondent Ong was proclaimed the duly elected representative of the second district of Northern
Samar.
The petitioners filed election protests against the private respondent premised on the following
grounds:

1) Jose Ong, Jr. is not a natural born citizen of the Philippines; and

2) Jose Ong, Jr. is not a resident of the second district of Northern Samar.

The HRET in its decision dated November 6, 1989, found for the private respondent.

A motion for reconsideration was filed by the petitioners on November 12, 1989. This was, however,
denied by the HRET in its resolution dated February 22, 1989.

Hence, these petitions for certiorari.

We treat the comments as answers and decide the issues raised in the petitions.

ON THE ISSUE OF JURISDICTION

The first question which arises refers to our jurisdiction.

The Constitution explicitly provides that the House of Representatives Electoral Tribunal (HRET) and
the Senate Electoral Tribunal (SET) shall be the sole judges of all contests relating to the election,
returns, and qualifications of their respective members. (See Article VI, Section 17, Constitution)

The authority conferred upon the Electoral Tribunal is full, clear and complete. The use of the
word sole emphasizes the exclusivity of the jurisdiction of these Tribunals.

The Supreme Court in the case of Lazatin v. HRET (168 SCRA 391 [1988]) stated that under the
1987 Constitution, the jurisdiction of the Electoral Tribunal is original and exclusive, viz:

The use of the word "sole" emphasizes the exclusive character of the jurisdiction conferred
(Angara v. Electoral Commission, supra at p. 162). The exercise of power by the Electoral
Commission under the 1935 Constitution has been described as "intended to be as complete
and unimpaired as if it had originally remained in the legislature." (id., at p. 175) Earlier this
grant of power to the legislature was characterized by Justice Malcolm as "full, clear and
complete; (Veloso v. Board of Canvassers of Leyte and Samar, 39 Phil. 886 [1919]) Under
the amended 1935 Constitution, the power was unqualifiedly reposed upon the Electoral
Tribunal and it remained as full, clear and complete as that previously granted the
Legislature and the Electoral Commission, (Lachica v. Yap, 25 SCRA 140 [1968]) The same
may be said with regard to the jurisdiction of the Electoral Tribunal under the 1987
Constitution. (p. 401)

The Court continued further, ". . . so long as the Constitution grants the HRET the power to be the
sole judge of all contests relating to election, returns and qualifications of members of the House of
Representatives, any final action taken by the HRET on a matter within its jurisdiction shall, as a
rule, not be reviewed by this Court . . . the power granted to the Electoral Tribunal is full, clear and
complete and excludes the exercise of any authority on the part of this Court that would in any wise
restrict it or curtail it or even affect the same." (pp. 403-404)

When may the Court inquire into acts of the Electoral Tribunals under our constitutional grants of
power?
In the later case of Robles v. HRET (181 SCRA 780 [1990]) the Supreme Court stated that the
judgments of the Tribunal are beyond judicial interference save only "in the exercise of this Court's
so-called extraordinary jurisdiction, . . . upon a determination that the Tribunal's decision or
resolution was rendered without or in excess of its jurisdiction, or with grave abuse of discretion or
paraphrasing Morrero, upon a clear showing of such arbitrary and improvident use by the Tribunal of
its power as constitutes a denial of due process of law, or upon a demonstration of a very clear
unmitigated ERROR, manifestly constituting such GRAVE ABUSE OF DISCRETION that there has
to be a remedy for such abuse." (at pp. 785-786)

In the leading case of Morrero v. Bocar (66 Phil. 429 [1938]) the Court ruled that the power of the
Electoral Commission "is beyond judicial interference except, in any event, upon a clear showing of
such arbitrary and improvident use of power as will constitute a denial of due process." The Court
does not venture into the perilous area of trying to correct perceived errors of independent branches
of the Government, It comes in only when it has to vindicate a denial of due process or correct an
abuse of discretion so grave or glaring that no less than the Constitution calls for remedial action.

The Supreme Court under the 1987 Constitution, has been given an expanded jurisdiction, so to
speak, to review the decisions of the other branches and agencies of the government to determine
whether or not they have acted within the bounds of the Constitution. (See Article VIII, Section 1,
Constitution)

Yet, in the exercise thereof, the Court is to merely check whether or not the governmental branch or
agency has gone beyond the Constitutional limits of its jurisdiction, not that it erred or has a different
view. In the absence of a showing that the HRET has committed grave abuse of discretion
amounting to lack of jurisdiction, there is no occasion for the Court to exercise its corrective power; it
will not decide a matter which by its nature is for the HRET alone to decide. (See Marcos v.
Manglapus, 177 SCRA 668 [1989]) It has no power to look into what it thinks is apparent error.

As constitutional creations invested with necessary power, the Electoral Tribunals, although not
powers in the tripartite scheme of the government, are, in the exercise of their functions independent
organs — independent of Congress and the Supreme Court. The power granted to HRET by the
Constitution is intended to be as complete and unimpaired as if it had remained originally in the
legislature. (Angara v. Electoral Commission, 63 Phil. 139 [1936])

In passing upon petitions, the Court with its traditional and careful regard for the balance of powers,
must permit this exclusive privilege of the Tribunals to remain where the Sovereign authority has
place it. (See Veloso v. Boards of Canvassers of Leyte and Samar, 39 Phil. 886 [1919])

It has been argued that under Article VI, Section 17 of the present Constitution, the situation may
exist as it exists today where there is an unhealthy one-sided political composition of the two
Electoral Tribunals. There is nothing in the Constitution, however, that makes the HRET because of
its composition any less independent from the Court or its constitutional functions any less exclusive.
The degree of judicial intervention should not be made to depend on how many legislative members
of the HRET belong to this party or that party. The test remains the same-manifest grave abuse of
discretion.

In the case at bar, the Court finds no improvident use of power, no denial of due process on the part
of the HRET which will necessitate the exercise of the power of judicial review by the Supreme
Court.

ON THE ISSUE OF CITIZENSHIP


The records show that in the year 1895, the private respondent's grandfather, Ong Te, arrived in the
Philippines from China. Ong Te established his residence in the municipality of Laoang, Samar on
land which he bought from the fruits of hard work.

As a resident of Laoang, Ong Te was able to obtain a certificate of residence from the then Spanish
colonial administration.

The father of the private respondent, Jose Ong Chuan was born in China in 1905. He was brought
by Ong Te to Samar in the year 1915.

Jose Ong Chuan spent his childhood in the province of Samar. In Laoang, he was able to establish
an enduring relationship with his neighbors, resulting in his easy assimilation into the community.

As Jose Ong Chuan grew older in the rural and seaside community of Laoang, he absorbed Filipino
cultural values and practices. He was baptized into Christianity. As the years passed, Jose Ong
Chuan met a natural born-Filipino, Agripina Lao. The two fell in love and, thereafter, got married in
1932 according to Catholic faith and practice.

The couple bore eight children, one of whom is the private respondent who was born in 1948.

The private respondent's father never emigrated from this country. He decided to put up a hardware
store and shared and survived the vicissitudes of life in Samar.

The business prospered. Expansion became inevitable. As a result, a branch was set-up in Binondo,
Manila. In the meantime, the father of the private respondent, unsure of his legal status and in an
unequivocal affirmation of where he cast his life and family, filed with the Court of First Instance of
Samar an application for naturalization on February 15, 1954.

On April 28, 1955, the CFI of Samar, after trial, declared Jose Ong Chuan a Filipino citizen.

On May 15, 1957, the Court of First Instance of Samar issued an order declaring the decision of
April 28, 1955 as final and executory and that Jose Ong Chuan may already take his Oath of
Allegiance.

Pursuant to said order, Jose Ong Chuan took his Oath of Allegiance; correspondingly, a certificate of
naturalization was issued to him.

At the time Jose Ong Chuan took his oath, the private respondent then a minor of nine years was
finishing his elementary education in the province of Samar. There is nothing in the records to
differentiate him from other Filipinos insofar as the customs and practices of the local populace were
concerned.

Fortunes changed. The house of the family of the private respondent in Laoang, Samar was burned
to the ground.

Undaunted by the catastrophe, the private respondent's family constructed another one in place of
their ruined house. Again, there is no showing other than that Laoang was their abode and home.

After completing his elementary education, the private respondent, in search for better education,
went to Manila in order to acquire his secondary and college education.
In the meantime, another misfortune was suffered by the family in 1975 when a fire gutted their
second house in Laoang, Samar. The respondent's family constructed still another house, this time a
16-door apartment building, two doors of which were reserved for the family.

The private respondent graduated from college, and thereafter took and passed the CPA Board
Examinations.

Since employment opportunities were better in Manila, the respondent looked for work here. He
found a job in the Central Bank of the Philippines as an examiner. Later, however, he worked in the
hardware business of his family in Manila. In 1971, his elder brother, Emil, was elected as a delegate
to the 1971 Constitutional Convention. His status as a natural born citizen was challenged.
Parenthetically, the Convention which in drafting the Constitution removed the unequal treatment
given to derived citizenship on the basis of the mother's citizenship formally and solemnly declared
Emil Ong, respondent's full brother, as a natural born Filipino. The Constitutional Convention had to
be aware of the meaning of natural born citizenship since it was precisely amending the article on
this subject.

The private respondent frequently went home to Laoang, Samar, where he grew up and spent his
childhood days.

In 1984, the private respondent married a Filipina named Desiree Lim.

For the elections of 1984 and 1986, Jose Ong, Jr. registered himself as a voter of Laoang, Samar,
and correspondingly, voted there during those elections.

The private respondent after being engaged for several years in the management of their family
business decided to be of greater service to his province and ran for public office. Hence, when the
opportunity came in 1987, he ran in the elections for representative in the second district of Northern
Samar.

Mr. Ong was overwhelmingly voted by the people of Northern Samar as their representative in
Congress. Even if the total votes of the two petitioners are combined, Ong would still lead the two by
more than 7,000 votes.

The pertinent portions of the Constitution found in Article IV read:

SECTION 1, the following are citizens of the Philippines:

1. Those who are citizens of the Philippines at the time of the adoption of the Constitution;

2. Those whose fathers or mothers are citizens of the Philippines;

3. Those born before January 17, 1973, of Filipino mothers, who elect Philippine citizenship
upon reaching the age of majority; and

4. Those who are naturalized in accordance with law.

SECTION 2, Natural-born Citizens are those who are citizens of the Philippines from birth
without having to perform any act to acquire or perfect their citizenship. Those who elect
Philippine citizenship in accordance with paragraph 3 hereof shall be deemed natural-born
citizens.
The Court interprets Section 1, Paragraph 3 above as applying not only to those who elect Philippine
citizenship after February 2, 1987 but also to those who, having been born of Filipino mothers,
elected citizenship before that date.

The provision in Paragraph 3 was intended to correct an unfair position which discriminates against
Filipino women. There is no ambiguity in the deliberations of the Constitutional Commission, viz:

Mr. Azcuna: With respect to the provision of section 4, would this refer only to those who
elect Philippine citizenship after the effectivity of the 1973 Constitution or would it also cover
those who elected it under the 1973 Constitution?

Fr. Bernas: It would apply to anybody who elected Philippine citizenship by virtue of the
provision of the 1935 Constitution whether the election was done before or after January 17,
1973. (Records of the Constitutional Commission, Vol. 1, p. 228; Emphasis supplied)

x x x           x x x          x x x

Mr. Trenas: The Committee on Citizenship, Bill of Rights, Political Rights and Obligations
and Human Rights has more or less decided to extend the interpretation of who is a natural-
born citizen as provided in section 4 of the 1973 Constitution by adding that persons who
have elected Philippine Citizenship under the 1935 Constitution shall be natural-born? Am I
right Mr. Presiding Officer?

Fr. Bernas: yes.

x x x           x x x          x x x

Mr. Nolledo: And I remember very well that in the Reverend Father Bernas' well written book,
he said that the decision was designed merely to accommodate former delegate Ernesto
Ang and that the definition on natural-born has no retroactive effect. Now it seems that the
Reverend Father Bernas is going against this intention by supporting the amendment?

Fr. Bernas: As the Commissioner can see, there has been an evolution in my thinking.
(Records of the Constitutional Commission, Vol. 1, p. 189)

x x x           x x x          x x x

Mr. Rodrigo: But this provision becomes very important because his election of Philippine
citizenship makes him not only a Filipino citizen but a natural-born Filipino citizen entitling
him to run for Congress. . .

Fr. Bernas: Correct. We are quite aware of that and for that reason we will leave it to the
body to approve that provision of section 4.

Mr. Rodrigo: I think there is a good basis for the provision because it strikes me as unfair that
the Filipino citizen who was born a day before January 17, 1973 cannot be a Filipino citizen
or a natural-born citizen. (Records of the Constitutional Commission, Vol. 1, p. 231)

x x x           x x x          x x x
Mr. Rodrigo: The purpose of that provision is to remedy an inequitable situation.  Between
1avvphi1

1935 and 1973 when we were under the 1935 Constitution, those born of Filipino fathers but
alien mothers were natural-born Filipinos. However, those born of Filipino mothers but alien
fathers would have to elect Philippine citizenship upon reaching the age of majority; and if
they do elect, they become Filipino citizens but not natural-born Filipino citizens. (Records of
the Constitutional Commission, Vol. 1, p. 356)

The foregoing significantly reveals the intent of the framers. To make the provision prospective from
February 3, 1987 is to give a narrow interpretation resulting in an inequitable situation. It must also
be retroactive.

It should be noted that in construing the law, the Courts are not always to be hedged in by the literal
meaning of its language. The spirit and intendment thereof, must prevail over the letter, especially
where adherence to the latter would result in absurdity and injustice. (Casela v. Court of Appeals, 35
SCRA 279 [1970])

A Constitutional provision should be construed so as to give it effective operation and suppress the
mischief at which it is aimed, hence, it is the spirit of the provision which should prevail over the letter
thereof. (Jarrolt v. Mabberly, 103 U.S. 580)

In the words of the Court in the case of J.M. Tuason v. LTA (31 SCRA 413 [1970]:

To that primordial intent, all else is subordinated. Our Constitution, any constitution is not to
be construed narrowly or pedantically for the prescriptions therein contained, to paraphrase
Justice Holmes, are not mathematical formulas having their essence in their form but are
organic living institutions, the significance of which is vital not formal. . . . (p. 427)

The provision in question was enacted to correct the anomalous situation where one born of a
Filipino father and an alien mother was automatically granted the status of a natural-born citizen
while one born of a Filipino mother and an alien father would still have to elect Philippine citizenship.
If one so elected, he was not, under earlier laws, conferred the status of a natural-born.

Under the 1973 Constitution, those born of Filipino fathers and those born of Filipino mothers with an
alien father were placed on equal footing. They were both considered as natural-born citizens.

Hence, the bestowment of the status of "natural-born" cannot be made to depend on the fleeting
accident of time or result in two kinds of citizens made up of essentially the same similarly situated
members.

It is for this reason that the amendments were enacted, that is, in order to remedy this accidental
anomaly, and, therefore, treat equally all those born before the 1973 Constitution and who elected
Philippine citizenship either before or after the effectivity of that Constitution.

The Constitutional provision in question is, therefore curative in nature. The enactment was meant to
correct the inequitable and absurd situation which then prevailed, and thus, render those acts valid
which would have been nil at the time had it not been for the curative provisions. (See Development
Bank of the Philippines v. Court of Appeals, 96 SCRA 342 [1980])

There is no dispute that the respondent's mother was a natural born Filipina at the time of her
marriage. Crucial to this case is the issue of whether or not the respondent elected or chose to be a
Filipino citizen.
Election becomes material because Section 2 of Article IV of the Constitution accords natural born
status to children born of Filipino mothers before January 17, 1973, if they elect citizenship upon
reaching the age of majority.

To expect the respondent to have formally or in writing elected citizenship when he came of age is to
ask for the unnatural and unnecessary. The reason is obvious. He was already a citizen. Not only
was his mother a natural born citizen but his father had been naturalized when the respondent was
only nine (9) years old. He could not have divined when he came of age that in 1973 and 1987 the
Constitution would be amended to require him to have filed a sworn statement in 1969 electing
citizenship inspite of his already having been a citizen since 1957. In 1969, election through a sworn
statement would have been an unusual and unnecessary procedure for one who had been a citizen
since he was nine years old.

We have jurisprudence that defines "election" as both a formal and an informal process.

In the case of In Re: Florencio Mallare (59 SCRA 45 [1974]), the Court held that the exercise of the
right of suffrage and the participation in election exercises constitute a positive act of election of
Philippine citizenship. In the exact pronouncement of the Court, we held:

Esteban's exercise of the right of suffrage when he came of age, constitutes a positive act of
election of Philippine citizenship (p. 52; emphasis supplied)

The private respondent did more than merely exercise his right of suffrage. He has established his
life here in the Philippines.

For those in the peculiar situation of the respondent who cannot be expected to have elected
citizenship as they were already citizens, we apply the In Re Mallare rule.

The respondent was born in an outlying rural town of Samar where there are no alien enclaves and
no racial distinctions. The respondent has lived the life of a Filipino since birth. His father applied for
naturalization when the child was still a small boy. He is a Roman Catholic. He has worked for a
sensitive government agency. His profession requires citizenship for taking the examinations and
getting a license. He has participated in political exercises as a Filipino and has always considered
himself a Filipino citizen. There is nothing in the records to show that he does not embrace
Philippine customs and values, nothing to indicate any tinge of alien-ness no acts to show that this
country is not his natural homeland. The mass of voters of Northern Samar are frilly aware of Mr.
Ong's parentage. They should know him better than any member of this Court will ever know him.
They voted by overwhelming numbers to have him represent them in Congress. Because of his acts
since childhood, they have considered him as a Filipino.

The filing of sworn statement or formal declaration is a requirement for those who still have to elect
citizenship. For those already Filipinos when the time to elect came up, there are acts of deliberate
choice which cannot be less binding. Entering a profession open only to Filipinos, serving in public
office where citizenship is a qualification, voting during election time, running for public office, and
other categorical acts of similar nature are themselves formal manifestations of choice for these
persons.

An election of Philippine citizenship presupposes that the person electing is an alien. Or his status is
doubtful because he is a national of two countries. There is no doubt in this case about Mr. Ong's
being a Filipino when he turned twenty-one (21).
We repeat that any election of Philippine citizenship on the part of the private respondent would not
only have been superfluous but it would also have resulted in an absurdity. How can a Filipino
citizen elect Philippine citizenship?

The respondent HRET has an interesting view as to how Mr. Ong elected citizenship. It observed
that "when protestee was only nine years of age, his father, Jose Ong Chuan became a naturalized
Filipino. Section 15 of the Revised Naturalization Act squarely applies its benefit to him for he was
then a minor residing in this country. Concededly, it was the law itself that had already elected
Philippine citizenship for protestee by declaring him as such." (Emphasis supplied)

The petitioners argue that the respondent's father was not, validly, a naturalized citizen because of
his premature taking of the oath of citizenship.

The Court cannot go into the collateral procedure of stripping Mr. Ong's father of his citizenship after
his death and at this very late date just so we can go after the son.

The petitioners question the citizenship of the father through a collateral approach. This can not be
done. In our jurisdiction, an attack on a person's citizenship may only be done through a direct action
for its nullity. (See Queto v. Catolico, 31 SCRA 52 [1970])

To ask the Court to declare the grant of Philippine citizenship to Jose Ong Chuan as null and void
would run against the principle of due process. Jose Ong Chuan has already been laid to rest. How
can he be given a fair opportunity to defend himself. A dead man cannot speak. To quote the words
of the HRET "Ong Chuan's lips have long been muted to perpetuity by his demise and obviously he
could not use beyond where his mortal remains now lie to defend himself were this matter to be
made a central issue in this case."

The issue before us is not the nullification of the grant of citizenship to Jose Ong Chuan. Our
function is to determine whether or not the HRET committed abuse of authority in the exercise of its
powers. Moreover, the respondent traces his natural born citizenship through his mother, not
through the citizenship of his father. The citizenship of the father is relevant only to determine
whether or not the respondent "chose" to be a Filipino when he came of age. At that time and up to
the present, both mother and father were Filipinos. Respondent Ong could not have elected any
other citizenship unless he first formally renounced Philippine citizenship in favor of a foreign
nationality. Unlike other persons faced with a problem of election, there was no foreign nationality of
his father which he could possibly have chosen.

There is another reason why we cannot declare the HRET as having committed manifest grave
abuse of discretion. The same issue of natural-born citizenship has already been decided by the
Constitutional Convention of 1971 and by the Batasang Pambansa convened by authority of the
Constitution drafted by that Convention. Emil Ong, full blood brother of the respondent, was declared
and accepted as a natural born citizen by both bodies.

Assuming that our opinion is different from that of the Constitutional Convention, the Batasang
Pambansa, and the respondent HRET, such a difference could only be characterized as error. There
would be no basis to call the HRET decision so arbitrary and whimsical as to amount to grave abuse
of discretion.

What was the basis for the Constitutional Convention's declaring Emil Ong a natural born citizen?
Under the Philippine Bill of 1902, inhabitants of the Philippines who were Spanish subjects on the
11th day of April 1899 and then residing in said islands and their children born subsequent thereto
were conferred the status of a Filipino citizen.

Was the grandfather of the private respondent a Spanish subject?

Article 17 of the Civil Code of Spain enumerates those who were considered Spanish Subjects, viz:

ARTICLE 17. The following are Spaniards:

1. Persons born in Spanish territory.

2. Children born of a Spanish father or mother, even though they were born out of Spain.

3. Foreigners who may have obtained naturalization papers.

4. Those without such papers, who may have acquired domicile in any town in the
Monarchy. (Emphasis supplied)

The domicile of a natural person is the place of his habitual residence. This domicile, once
established is considered to continue and will not be deemed lost until a new one is established.
(Article 50, NCC; Article 40, Civil Code of Spain; Zuellig v. Republic, 83 Phil. 768 [1949])

As earlier stated, Ong Te became a permanent resident of Laoang, Samar around 1895.
Correspondingly, a certificate of residence was then issued to him by virtue of his being a resident of
Laoang, Samar. (Report of the Committee on Election Protests and Credentials of the 1971
Constitutional Convention, September 7, 1972, p. 3)

The domicile that Ong Te established in 1895 continued until April 11, 1899; it even went beyond the
turn of the 19th century. It is also in this place were Ong Te set-up his business and acquired his real
property.

As concluded by the Constitutional Convention, Ong Te falls within the meaning of sub-paragraph 4
of Article 17 of the Civil Code of Spain.

Although Ong Te made brief visits to China, he, nevertheless, always returned to the Philippines.
The fact that he died in China, during one of his visits in said country, was of no moment. This will
not change the fact that he already had his domicile fixed in the Philippines and pursuant to the Civil
Code of Spain, he had become a Spanish subject.

If Ong Te became a Spanish subject by virtue of having established his domicile in a town under the
Monarchy of Spain, necessarily, Ong Te was also an inhabitant of the Philippines for an inhabitant
has been defined as one who has actual fixed residence in a place; one who has a domicile in a
place. (Bouvier's Law Dictionary, Vol. II) A priori, there can be no other logical conclusion but to
educe that Ong Te qualified as a Filipino citizen under the provisions of section 4 of the Philippine
Bill of 1902.

The HRET itself found this fact of absolute verity in concluding that the private respondent was a
natural-born Filipino.
The petitioners' sole ground in disputing this fact is that document presented to prove it were not in
compliance with the best the evidence rule. The petitioners allege that the private respondent failed
to present the original of the documentary evidence, testimonial evidence and of the transcript of the
proceedings of the body which the aforesaid resolution of the 1971 Constitutional Convention was
predicated.

On the contrary, the documents presented by the private respondent fall under the exceptions to the
best evidence rule.

It was established in the proceedings before the HRET that the originals of the Committee Report
No. 12, the minutes of the plenary session of 1971 Constitutional Convention held on November 28,
1972 cannot be found.

This was affirmed by Atty. Ricafrente, Assistant Secretary of the 1971 Constitutional Convention; by
Atty. Nolledo, Delegate to the 1971 Constitutional Convention; and by Atty. Antonio Santos, Chief
Librarian of the U.P Law Center, in their respective testimonies given before the HRET to the effect
that there is no governmental agency which is the official custodian of the records of the 1971
Constitutional Convention. (TSN, December 12, 1988, pp. 30-31; TSN, January 17, 1989, pp. 34-35;
TSN, February 1, 1989, p. 44; TSN, February 6, 1989, pp. 28-29)

The execution of the originals was established by Atty. Ricafrente, who as the Assistant Secretary of
the 1971 Constitutional Convention was the proper party to testify to such execution. (TSN,
December 12, 1989, pp. 11-24)

The inability to produce the originals before the HRET was also testified to as aforestated by Atty.
Ricafrente, Atty. Nolledo, and Atty. Santos. In proving the inability to produce, the law does not
require the degree of proof to be of sufficient certainty; it is enough that it be shown that after a bona
fide diligent search, the same cannot be found. (see Government of P.I. v. Martinez, 44 Phil. 817
[1918])

Since the execution of the document and the inability to produce were adequately established, the
contents of the questioned documents can be proven by a copy thereof or by the recollection of
witnesses.

Moreover, to erase all doubts as to the authenticity of the documentary evidence cited in the
Committee Report, the former member of the 1971 Constitutional Convention, Atty. Nolledo, when
he was presented as a witness in the hearing of the protest against the private respondent,
categorically stated that he saw the disputed documents presented during the hearing of the election
protest against the brother of the private respondent. (TSN, February 1, 1989, pp. 8-9)

In his concurring opinion, Mr. Justice Sarmiento, a vice-president of the Constitutional Convention,
states that he was presiding officer of the plenary session which deliberated on the report on the
election protest against Delegate Emil Ong. He cites a long list of names of delegates present.
Among them are Mr. Chief Justice Fernan, and Mr. Justice Davide, Jr. The petitioners could have
presented any one of the long list of delegates to refute Mr. Ong's having been declared a natural-
born citizen. They did not do so. Nor did they demur to the contents of the documents presented by
the private respondent. They merely relied on the procedural objections respecting the admissibility
of the evidence presented.

The Constitutional Convention was the sole judge of the qualifications of Emil Ong to be a member
of that body. The HRET by explicit mandate of the Constitution, is the sole judge of the qualifications
of Jose Ong, Jr. to be a member of Congress. Both bodies deliberated at length on the controversies
over which they were sole judges. Decisions were arrived at only after a full presentation of all
relevant factors which the parties wished to present. Even assuming that we disagree with their
conclusions, we cannot declare their acts as committed with grave abuse of discretion. We have to
keep clear the line between error and grave abuse.

ON THE ISSUE OF RESIDENCE

The petitioners question the residence qualification of respondent Ong.

The petitioners lose sight of the meaning of "residence" under the Constitution. The term "residence"
has been understood as synonymous with domicile not only under the previous Constitutions but
also under the 1987 Constitution.

The deliberations of the Constitutional Commission reveal that the meaning of residence vis-a-
vis the qualifications of a candidate for Congress continues to remain the same as that of domicile,
to wit:

Mr. Nolledo: With respect to Section 5, I remember that in the 1971 Constitutional
Convention, there was an attempt to require residence in the place not less than one year
immediately preceding the day of the elections. So my question is: What is the Committee's
concept of residence of a candidate for the legislature? Is it actual residence or is it the
concept of domicile or constructive residence?

Mr. Davide: Madame President, in so far as the regular members of the National Assembly
are concerned, the proposed section merely provides, among others, and a resident thereof,
that is, in the district, for a period of not less than one year preceding the day of the election.
This was in effect lifted from the 1973 Constitution, the interpretation given to it was domicile.
(Records of the 1987 Constitutional Convention, Vol. 11, July 22, 1986. p. 87)

x x x           x x x          x x x

Mrs. Rosario Braid: The next question is on Section 7, page 2. I think Commissioner Nolledo
has raised the same point that "resident" has been interpreted at times as a matter of
intention rather than actual residence.

Mr. De los Reyes: Domicile.

Ms. Rosario Braid: Yes, So, would the gentlemen consider at the proper time to go back to
actual residence rather than mere intention to reside?

Mr. De los Reyes: But we might encounter some difficulty especially considering that a
provision in the Constitution in the Article on Suffrage says that Filipinos living abroad may
vote as enacted by law. So, we have to stick to the original concept that it should be by
domicile and not physical and actual residence. (Records of the 1987 Constitutional
Commission, Vol. 11, July 22, 1986, p. 110)

The framers of the Constitution adhered to the earlier definition given to the word "residence" which
regarded it as having the same meaning as domicile.

The term "domicile" denotes a fixed permanent residence to which when absent for business or
pleasure, one intends to return. (Ong Huan Tin v. Republic, 19 SCRA 966 [1967]) The absence of a
person from said permanent residence, no matter how long, notwithstanding, it continues to be the
domicile of that person. In other words, domicile is characterized by animus revertendi (Ujano v.
Republic, 17 SCRA 147 [1966])

The domicile of origin of the private respondent, which was the domicile of his parents, is fixed at
Laoang, Samar. Contrary to the petitioners' imputation, Jose Ong, Jr. never abandoned said
domicile; it remained fixed therein even up to the present.

The private respondent, in the proceedings before the HRET sufficiently established that after the
fire that gutted their house in 1961, another one was constructed.

Likewise, after the second fire which again destroyed their house in 1975, a sixteen-door apartment
was built by their family, two doors of which were reserved as their family residence. (TSN, Jose
Ong, Jr., November 18,1988, p. 8)

The petitioners' allegation that since the private respondent owns no property in Laoang, Samar, he
cannot, therefore, be a resident of said place is misplaced.

The properties owned by the Ong Family are in the name of the private respondent's parents. Upon
the demise of his parents, necessarily, the private respondent, pursuant to the laws of succession,
became the co-owner thereof (as a co- heir), notwithstanding the fact that these were still in the
names of his parents.

Even assuming that the private respondent does not own any property in Samar, the Supreme Court
in the case of De los Reyes v. Solidum (61 Phil. 893 [1935]) held that it is not required that a person
should have a house in order to establish his residence and domicile. It is enough that he should live
in the municipality or in a rented house or in that of a friend or relative. (Emphasis supplied)

To require the private respondent to own property in order to be eligible to run for Congress would
be tantamount to a property qualification. The Constitution only requires that the candidate meet the
age, citizenship, voting and residence requirements. Nowhere is it required by the Constitution that
the candidate should also own property in order to be qualified to run. (see Maquera v. Borra, 122
Phil. 412 [1965])

It has also been settled that absence from residence to pursue studies or practice a profession or
registration as a voter other than in the place where one is elected, does not constitute loss of
residence. (Faypon v. Quirino, 96 Phil. 294 [1954])

As previously stated, the private respondent stayed in Manila for the purpose of finishing his studies
and later to practice his profession, There was no intention to abandon the residence in Laoang,
Samar. On the contrary, the periodical journeys made to his home province reveal that he always
had the animus revertendi.

The Philippines is made up not only of a single race; it has, rather, undergone an interracial
evolution. Throughout our history, there has been a continuing influx of Malays, Chinese, Americans,
Japanese, Spaniards and other nationalities. This racial diversity gives strength to our country.

Many great Filipinos have not been whole-blooded nationals, if there is such a person, for there is
none. To mention a few, the great Jose Rizal was part Chinese, the late Chief Justice Claudio
Teehankee was part Chinese, and of course our own President, Corazon Aquino is also part
Chinese. Verily, some Filipinos of whom we are proud were ethnically more Chinese than the private
respondent.

Our citizens no doubt constitute the country's greatest wealth. Citizenship is a special privilege which
one must forever cherish.

However, in order to truly revere this treasure of citizenship, we do not, on the basis of too harsh an
interpretation, have to unreasonably deny it to those who qualify to share in its richness.

Under the overly strict jurisprudence surrounding our antiquated naturalization laws only the very
affluent backed by influential patrons, who were willing to suffer the indignities of a lengthy,
sometimes humiliating, and often corrupt process of clearances by minor bureaucrats and whose
lawyers knew how to overcome so many technical traps of the judicial process were able to acquire
citizenship. It is time for the naturalization law to be revised to enable a more positive, affirmative,
and meaningful examination of an applicant's suitability to be a Filipino. A more humane, more
indubitable and less technical approach to citizenship problems is essential.

WHEREFORE, the petitions are hereby DISMISSED. The questioned decision of the House of
Representatives Electoral Tribunal is AFFIRMED. Respondent Jose Ong, Jr. is declared a natural-
born citizen of the Philippines and a resident of Laoang, Northern Samar.

SO ORDERED.

G.R. No. L-21289 October 4, 1971

MOY YA LIM YAO alias EDILBERTO AGUINALDO LIM and LAU YUEN YEUNG, petitioners-
appellants, 
vs.
THE COMMISSIONER OF IMMIGRATION, respondent-appellee.

Aruego, Mamaril & Associates for petitioners-appellants.

Office of the Solicitor General Arturo A. Alafriz, Assistant Solicitor General Frine' C. Zaballero and
Solicitor Sumilang V. Bernardo for respondent-appellee.

BARREDO, J.:

Appeal from the following decision of the Court of First Instance of Manila in its Civil Case No. 49705
entitled Moy Ya Lim Yao, etc., et al. vs. The Commissioner of Immigration which, brief as it is,
sufficiently depicts the factual setting of and the fundamental issues involved in this case thus:

In the instant case, petitioners seek the issuance of a writ of injunction against the
Commissioner of Immigration, "restraining the latter and/or his authorized
representative from ordering plaintiff Lau Yuen Yeung to leave the Philippines and
causing her arrest and deportation and the confiscation of her bond, upon her failure
to do so."
The prayer for preliminary injunction embodied in the complaint, having been denied,
the case was heard on the merits and the parties submitted their respective
evidence.

The facts of the case, as substantially and correctly stated by the Solicitor General
are these:

On February 8, 1961, Lau Yuen Yeung applied for a passport visa to


enter the Philippines as a non-immigrant. In the interrogation made in
connection with her application for a temporary visitor's visa to enter
the Philippines, she stated that she was a Chinese residing at
Kowloon, Hongkong, and that she desired to take a pleasure trip to
the Philippines to visit her great (grand) uncle Lau Ching Ping for a
period of one month (Exhibits "l," "1-a," and "2"). She was permitted
to come into the Philippines on March 13, 1961, and was permitted to
stay for a period of one month which would expire on April 13, 1961.
On the date of her arrival, Asher Y, Cheng filed a bond in the amount
of P1,000.00 to undertake, among others that said Lau Yuen Yeung
would actually depart from the Philippines on or before the expiration
of her authorized period of stay in this country or within the period as
in his discretion the Commissioner of Immigration or his authorized
representative might properly allow. After repeated extensions,
petitioner Lau Yuen Yeung was allowed to stay in the Philippines up
to February 13, 1962 (Exhibit "4"). On January 25, 1962, she
contracted marriage with Moy Ya Lim Yao alias Edilberto Aguinaldo
Lim an alleged Filipino citizen. Because of the contemplated action of
respondent to confiscate her bond and order her arrest and
immediate deportation, after the expiration of her authorized stay, she
brought this action for injunction with preliminary injunction. At the
hearing which took place one and a half years after her arrival, it was
admitted that petitioner Lau Yuen Yeung could not write either
English or Tagalog. Except for a few words, she could not speak
either English or Tagalog. She could not name any Filipino neighbor,
with a Filipino name except one, Rosa. She did not know the names
of her brothers-in-law, or sisters-in-law.

Under the facts unfolded above, the Court is of the considered opinion, and so holds,
that the instant petition for injunction cannot be sustained for the same reason as set
forth in the Order of this Court, dated March 19, 1962, the pertinent portions of which
read:

First, Section 15 of the Revised Naturalization Law provides:

Effect of the naturalization on wife and children. — Any woman who


is now or may hereafter be married to a citizen of the Philippines, and
who might herself be lawfully naturalized shall be deemed a citizen of
the Philippines.

The above-quoted provision is clear and its import unequivocal and hence it should
be held to mean what it plainly and explicitly expresses in unmistakable terms. The
clause "who might herself be lawfully naturalized" incontestably implies that an alien
woman may be deemed a citizen of the Philippines by virtue of her marriage to a
Filipino citizen only if she possesses all the qualifications and none of the
disqualifications specified in the law, because these are the explicit requisites
provided by law for an alien to be naturalized. (Lee Suan Ay, Alberto Tan and Lee
Chiao vs. Emilio Galang, etc., G. R. No. L-11855). However, from the allegation of
paragraph 3 of the complaint, to wit:

3. That plaintiff Lau Yuen Yeung, Chinese by birth, who might herself
be lawfully naturalized as a Filipino citizen (not being disqualified to
become such by naturalization), is a Filipino citizen by virtue of her
marriage on January 25, 1962 to plaintiff MOY YA LIM YAO alias
EDILBERTO AGUINALDO LIM, under the Naturalization Laws of the
Philippines.

it can be deduced beyond debate that petitioner Lau Yuen Yeung while claiming not
to be disqualified, does not and cannot allege that she possesses all the
qualifications to be naturalized, naturally because, having been admitted as a
temporary visitor only on March 13, 1961, it is obvious at once that she lacks at least,
the requisite length of residence in the Philippines (Revised Naturalization Law, Sec.
2, Case No. 2, Sec. 3, Case No. 3).

Were if the intention of the law that the alien woman, to be deemed a
citizen of the Philippines by virtue of marriage to a Filipino citizen,
need only be not disqualified under the Naturalization Law, it would
have been worded "and who herself is not disqualified to become a
citizen of the Philippines."

Second, Lau Yuen Yeung, a temporary Chinese woman visitor, whose authorized
stay in the Philippines, after repeated extensions thereof, was to expire last February
28, 1962, having married her co-plaintiff only on January 25, 1962, or just a little over
one month before the expiry date of her stay, it is evident that said marriage was
effected merely for convenience to defeat or avoid her then impending compulsory
departure, not to say deportation. This cannot be permitted.

Third, as the Solicitor General has well stated:

5. That petitioner Lau Yuen Yeung, having been admitted as a


temporary alien visitor on the strength of a deliberate and voluntary
representation that she will enter and stay only for a period of one
month and thereby secured a visa, cannot go back on her
representation to stay permanently without first departing from the
Philippines as she had promised. (Chung Tiao Bing, et al. vs.
Commissioner of Immigration, G. R. No. L-9966, September 29,
1956; Ong Se Lun vs. Board of Commissioners, G. R. No. L-6017,
September 16, 1954; Sec. 9, last par., Phil. Immigration Law).

The aforequoted argument of the Solicitor General is well buttressed not only by the
decided cases of the Supreme Court on the point mentioned above, but also on the
very provisions of Section 9, sub-paragraph (g) of the Philippine Immigration Act of
1940 which reads:

An alien who is admitted as a non-immigrant cannot remain in the


Philippines permanently. To obtain permanent admission, a non-
immigrant alien must depart voluntarily to some foreign country and
procure from the appropriate Philippine Consul the proper visa and
thereafter undergo examination by the Officers of the Bureau of
Immigration at a Philippine port of entry for determination of his
admissibility in accordance with the requirements of this Act. (This
paragraph is added by Republic Act 503). (Sec. 9, subparagraph (g)
of the Philippine Immigration Act of 1940).

And fourth, respondent Commissioner of Immigration is charged with the


administration of all laws relating to immigration (Sec. 3, Com. Act No. 613) and in
the performance of his duties in relation to alien immigrants, the law gives the
Commissioner of Immigration a wide discretion, a quasi-judicial function in
determining cases presented to him (Pedro Uy So vs. Commissioner of Immigration
CA-G. R. No. 23336-R, Dec. 15, 1960), so that his decision thereon may not be
disturbed unless he acted with abuse of discretion or in excess of his jurisdiction.

It may also be not amiss to state that wife Lau Yuen Yeung, while she barely and
insufficiently talk in broken Tagalog and English, she admitted that she cannot write
either language.

The only matter of fact not clearly passed upon by His Honor which could have some bearing in the
resolution of this appeal is the allegation in the brief of petitioners-appellants, not denied in the
governments brief, that "in the hearing ..., it was shown thru the testimony of the plaintiff Lau Yuen
Yeung that she does not possess any of the disqualifications for naturalization." Of course, as an
additional somehow relevant factual matter, it is also emphasized by said appellants that during the
hearing in the lower court, held almost ten months after the alleged marriage of petitioners, "Lau
Yuen Yeung was already carrying in her womb for seven months a child by her husband."

Appellants have assigned six errors allegedly committed by the court a quo, thus:

THE LOWER COURT ERRED IN HOLDING THAT THE CLAUSE "WHO MIGHT
HERSELF BE LAWFULLY NATURALIZED" (OF SECTION 15, REVISED
NATURALIZATION LAW) INCONTESTABLY IMPLIES THAT AN ALIEN WOMAN
MAY BE DEEMED A CITIZEN OF THE PHILIPPINES BY VIRTUE OF HER
MARRIAGE TO A FILIPINO CITIZEN, ONLY IF SHE POSSESSES ALL THE
QUALIFICATIONS AND NONE OF THE DISQUALIFICATIONS SPECIFIED IN THE
LAW.

II

THE LOWER COURT ERRED IN HOLDING THAT A WOMAN FOREIGNER WHO


DOES NOT POSSESS ANY OF THE DISQUALIFICATIONS FOR CITIZENSHIP
AND WHO MARRIED A FILIPINO CITIZEN IS STILL CONSIDERED AN ALIEN
EVEN AFTER SUCH MARRIAGE AS TO FALL WITHIN THE REQUIREMENT OF
SECTION 9, SUB-PARAGRAPH (9) OF THE PHILIPPINE IMMIGRATION ACT OF
1940.

III
THE COURT ERRED IN CONCLUDING THAT LAU YUEN YEUNG'S MARRIAGE
TO A FILIPINO CITIZEN WAS ONLY FOR CONVENIENCE, MERELY BECAUSE
THE SAME WAS CELEBRATED JUST OVER A MONTH BEFORE THE EXPIRY
DATE OF HER AUTHORIZED STAY.

IV

THE LOWER COURT ERRED IN FAILING TO FIND THAT THE COMMISSIONER


OF IMMIGRATION ACTED WITH ABUSE OF DISCRETION OR IN EXCESS OF
HIS JURISDICTION WHEN SAID OFFICER THREATENED TO SEND OUT OF THE
COUNTRY PLAINTIFF LAU YUEN YEUNG WITH WARNING THAT HER FAILURE
TO DO SO WOULD MEAN CONFISCATION OF HER BOND, ARREST AND
IMMEDIATE DEPORTATION, IN SPITE OF THE FACT THAT LAU YUEN YEUNG
IS NOW A FILIPINO CITIZEN.

THE LOWER COURT ERRED IN DISMISSING PLAINTIFFS-APPELLANTS'


COMPLAINT AND IN REFUSING TO PERMANENTLY ENJOIN THE
COMMISSIONER FROM ORDERING PLAINTIFF LAU YUEN YEUNG TO LEAVE
THE PHILIPPINES AS A TEMPORARY VISITOR WHICH SHE IS NOT.

VI

THE LOWER COURT ERRED IN REFUSING TO GRANT PLAINTIFFS-


APPELLANTS' MOTION FOR PRELIMINARY INJUNCTION EMBODIED IN THEIR
COMPLAINT, IN AN ORDER DATED MARCH 19, 1962. (PAGES 36-41, RECORD
ON APPEAL) .

We need not discuss these assigned errors separately. In effect, the above decision upheld the two
main grounds of objection of the Solicitor General to the petition in the court below, viz:

That petitioner Lau Yuen Yeung, having been admitted as a temporary alien visitor
on the strength of a deliberate and voluntary representation that she will enter and
stay only for a period of one month and thereby secured a visa, cannot go back on
her representation to stay permanently without first departing from the Philippines as
she had promised. (Chung Tiao Bing, et al. vs. Commissioner of Immigration, G.R.
No. L-9966, September 29, 1956; Ong Se Lun vs. Board of Commissioners, G.R. No.
L-6017, Sept. 16, 1954, Sec. 9, last par. Phil. Immigration Law);

That the mere marriage of a Filipino citizen to an alien does not automatically confer
on the latter Philippine citizenship. The alien wife must possess all the qualifications
required by law to become a Filipino citizen by naturalization and none of the
disqualifications. (Lee Suan Ay, Alberto Tan and Lee Chiao vs. Galang, etc., G. R.
No. L-11855, Dec. 25, 1959)

It is obvious from the nature of these objection that their proper resolution would necessarily cover
all the points raised in appellants' assignments of error, hence, We will base our discussions, more
or less, on said objections.

I
The first objection of the Solicitor General which covers the matters dealt with in appellants' second
and fourth assignments of error does not require any lengthy discussion. As a matter of fact, it seem
evident that the Solicitor General's pose that an alien who has been admitted into the Philippines as
a non-immigrant cannot remain here permanently unless he voluntarily leaves the country first and
goes to a foreign country to secure thereat from the appropriate Philippine consul the proper visa
and thereafter undergo examination by officers of the Bureau of Immigration at a Philippine port of
entry for determination of his admissibility in accordance with the requirements of the Philippine
Immigration Act of 1940, as amended by Republic Act 503, is premised on the assumption that
petitioner Lau Yuen Yeung is not a Filipino citizen. We note the same line of reasoning in the
appealed decision of the court a quo. Accordingly, it is but safe to assume that were the Solicitor
General and His Honor of the view that said petitioner had become ipso facto a Filipina by virtue of
her marriage to her Filipino husband, they would have held her as entitled to assume the status of a
permanent resident without having to depart as required of aliens by Section 9 (g) of the law.

In any event, to set this point at rest, We hereby hold that portion of Section 9 (g) of the Immigration
Act providing:

An alien who is admitted as a non-immigrant cannot remain in the Philippines


permanently. To obtain permanent admission, a non-immigrant alien must depart
voluntarily to some foreign country and procure from the appropriate Philippine
consul the proper visa and thereafter undergo examination by the officers of the
Bureau of Immigration at a Philippine port of entry for determination of his
admissibility in accordance with the requirements of this Act.

does not apply to aliens who after coming into the Philippines as temporary visitors, legitimately
become Filipino citizens or acquire Filipino citizenship. Such change of nationality naturally bestows
upon their the right to stay in the Philippines permanently or not, as they may choose, and if they
elect to reside here, the immigration authorities may neither deport them nor confiscate their bonds.
True it is that this Court has vehemently expressed disapproval of convenient ruses employed by
alien to convert their status from temporary visitors to permanent residents in circumvention of the
procedure prescribed by the legal provision already mentioned, such as in Chiong Tiao Bing vs.
Commissioner of Immigration, 99 Phil. 1020, wherein, thru Mr. Justice J.B.L. Reyes, the Court,
reiterating the ruling in Ong Se Lun vs. Board of Immigration Commissioners, 95 PMI. 785, said:

... It is clear that if an alien gains admission to the Islands on the strength of a
deliberate and voluntary representation that he will enter only for a limited time, and
thereby secures the benefit of a temporary visa, the law will not allow him
subsequently to go back on his representation and stay permanently, without first
departing from the Philippines as he had promised. No officer can relieve him of the
departure requirements of section 9 of the Immigration Act, under the guise of
"change" or "correction", for the law makes no distinctions, and no officer is above
the law. Any other ruling would, as stated in our previous decision, encourage aliens
to enter the Islands on false pretences; every alien so permitted to enter for a limited
time, might then claim a right to permanent admission, however flimsy such claim
should be, and thereby compel our government to spend time, money and effort to
examining and verifying whether or not every such alien really has a right to take up
permanent residence here. In the meanwhile, the alien would be able to prolong his
stay and evade his return to the port whence he came, contrary to what he promised
to do when he entered. The damages inherent in such ruling are self-evident.

On the other hand, however, We cannot see any reason why an alien who has been here as a
temporary visitor but who has in the meanwhile become a Filipino should be required to still leave
the Philippines for a foreign country, only to apply thereat for a re-entry here and undergo the
process of showing that he is entitled to come back, when after all, such right has become
incontestible as a necessary concomitant of his assumption of our nationality by whatever legal
means this has been conferred upon him. Consider for example, precisely the case of the minor
children of an alien who is naturalized. It is indubitable that they become ipso facto citizens of the
Philippines. Could it be the law that before they can be allowed permanent residence, they still have
to be taken abroad so that they may be processed to determine whether or not they have a right to
have permanent residence here? The difficulties and hardships which such a requirement entails
and its seeming unreasonableness argue against such a rather absurd construction. Indeed, as
early as 1957, in Ly Giok Ha vs. Galang, 101 Phil. 459, Mr. Justice Concepcion, our present Chief
Justice, already ruled thus:

... (P)etitioners allege that, upon her marriage to a Filipino, Ly Giok Ha became also
a citizen of the Philippines. Indeed, if this conclusion were correct, it would follow
that, in consequence of her marriage, she had been naturalized as such citizen, and,
hence the decision appealed from would have to be affirmed, for section 40(c) of
Commonwealth Act 613 provides that "in the event of the naturalization as a
Philippine citizen ... of the alien on whose behalf the bond deposit is given, the bond
shall be cancelled or the sum deposited shall be returned to the depositor or his legal
representative." (At. pp. 462-463)

In other words, the applicable statute itself more than implies that the naturalization of an alien visitor
as a Philippine citizen logically produces the effect of conferring upon him ipso facto all the rights of
citizenship including that of being entitled to permanently stay in the Philippines outside the orbit of
authority of the Commissioner of Immigration vis-a-vis aliens, if only because by its very nature and
express provisions, the Immigration Law is a law only for aliens and is inapplicable to citizens of the
Philippines. In the sense thus discussed therefore, appellants' second and fourth assignments of
error are well taken.

II

Precisely, the second objection, of the Solicitor General sustained by the trial judge is that appellant
Lau Yuen Yeung's marriage to appellant Moya Lim Yao alias Edilberto Aguinaldo whose Filipino
citizenship is not denied did not have the effect of making her a Filipino, since it has not been shown
that she "might herself be lawfully naturalized," it appearing clearly in the record that she does not
possess all the qualifications required of applicants for naturalization by the Revised Naturalization
Law, Commonwealth Act 473, even if she has proven that she does not suffer from any of the
disqualifications thereunder. In other words, the Solicitor General implicitly concedes that had it been
established in the proceedings below that appellant Lau Yuen Yeung possesses all the qualifications
required by the law of applicants for naturalization, she would have been recognized by the
respondent as a Filipino citizen in the instant case, without requiring her to submit to the usual
proceedings for naturalization.

To be sure, this position of the Solicitor General is in accord with what used to be the view of this
Court since Lee Suan Ay, et al. v. Emilio Galang, etc., et al., G.R. No. L-11855, promulgated
December 23, 1959, 106 Phil., 706,713,  for it was only in Zita Ngo Burca vs. Republic, G.R. NO. L-
1

24252 which was promulgated on January 30, 1967 (19 SCRA 186), that over the pen of Mr. Justice
Conrado Sanchez, this Court held that for an alien woman who marries a Filipino to be deemed a
Filipina, she has to apply for naturalization in accordance with the procedure prescribed by the
Revised Naturalization Law and prove in said naturalization proceeding not only that she has all the
qualifications and none of the disqualifications provided in the law but also that she has complied
with all the formalities required thereby like any other applicant for naturalization,  albeit said decision
2
is not yet part of our jurisprudence inasmuch as the motion for its reconsideration is still pending
resolution. Appellants are in effect urging Us, however, in their first and second assignments of error,
not only to reconsider Burca but to even reexamine Lee Suan Ay which, as a matter of fact, is the
prevailing rule, having been reiterated in all subsequent decisions up to Go Im Ty. 3

Actually, the first case in which Section 15 of the Naturalization Law, Commonwealth Act 473,
underwent judicial construction was in the first Ly Giok Ha case,  one almost identical to the one at
4

bar. Ly Giok Ha, a woman of Chinese nationality, was a temporary visitor here whose authority to
stay was to expire on March 14, 1956. She filed a bond to guaranty her timely departure. On March
8, 1956, eight days before the expiration of her authority to stay, she married a Filipino by the name
of Restituto Lacasta. On March 9, 1956, her husband notified the Commissioner of Immigration of
said marriage and, contending that his wife had become a Filipina by reason of said marriage,
demanded for the cancellation of her bond, but instead of acceding to such request, the
Commissioner required her to leave, and upon her failure to do so, on March 16, 1956, the
Commissioner confiscated her bond; a suit was filed for the recovery of the bond; the lower court
sustained her contention that she had no obligation to leave, because she had become Filipina by
marriage, hence her bond should be returned. The Commissioner appealed to this Court. In the said
appeal, Mr. Justice Roberto Concepcion, our present Chief Justice, spoke for the Court, thus:

The next and most important question for determination is whether her marriage to a
Filipino justified or, at least, excused the aforesaid failure of Ly Giok Ha to depart
from the Philippines on or before March 14, 1956. In maintaining the affirmative view,
petitioners alleged that, upon her marriage to a Filipino, Ly Giok Ha became, also, a
citizen of the Philippines. Indeed, if this conclusion were correct, it would follow that,
in consequence of her marriage, she had been naturalized as such citizen, and,
hence, the decision appealed from would have to be affirmed, for section 40(c) of
Commonwealth Act No. 613 provides that "in the event of the naturalization as a
Philippine citizen ... of the alien on whose behalf the bond deposit is given, the bond
shall be cancelled or the sum deposited shall be returned to the depositor or his legal
representative." Thus the issue boils down to whether an alien female who marries a
male citizen of the Philippines follows ipso facto his political status.

The pertinent part of section 15 of Commonwealth Act No. 473, upon which
petitioners rely, reads:

Any woman who is now or may hereafter be married to a citizen of


the Philippines, and who might herself be lawfully naturalized shall be
deemed a citizen of the Philippines.

Pursuant thereto, marriage to a male Filipino does not vest Philippine citizenship to
his foreign wife, unless she "herself may be lawfully naturalized." As correctly held in
an opinion of the Secretary of Justice (Op. No. 52, series of 1950),* this limitation of
section 15 excludes, from the benefits of naturalization by marriage, those
disqualified from being naturalized as citizens of the Philippines under section 4 of
said Commonwealth Act No. 473, namely:

(a) Persons opposed to organized government or affiliated with any


association or group of persons who uphold and teach doctrines
opposing all organized governments;
(b) Persons defending or teaching the necessity or propriety of
violence, personal assault, or assassination for the success and
predominance of their ideas;

(c) Polygamists or believers in the practice of polygamy;

(d) Persons convicted of crimes involving moral turpitude;

(e) Persons suffering from mental alienation or incurable contagious


diseases;

(f) Persons who, during the period of their residence in the


Philippines, have not mingled socially with the Filipinos, or who have
not evinced a sincere desire to learn and embrace the customs,
traditions, and ideals of the Filipinos;

(g) Citizens or subjects of nations with whom the ... Philippines are at
war, during the period of such war;

(h) Citizens or subjects of a foreign country other than the United


States, whose laws does not grant Filipinos the right to become
naturalized citizens or subjects thereof.

In the case at bar, there is neither proof nor allegation in the pleadings that Ly Giok
Ha does not fall under any of the classes disqualified by law. Moreover, as the
parties who claim that, despite her failure to depart from the Philippines within the
period specified in the bond in question, there has been no breach thereof,
petitioners have the burden of proving her alleged change of political status, from
alien to citizen. Strictly speaking, petitioners have not made out, therefore a case
against the respondents-appellants.

Considering, however, that neither in the administrative proceedings, nor in the lower
court, had the parties seemingly felt that there was an issue on whether Ly Giok Ha
may "be lawfully naturalized," and this being a case of first impression in our courts,
we are of the opinion that, in the interest of equity and justice, the parties herein
should be given an opportunity to introduce evidence, if they have any, on said issue.
(At pp. 462-464.) .

As may be seen, although not specifically in so many words, no doubt was left in the above decision
as regards the following propositions: .

1. That under Section 15 of Commonwealth Act 473, the Revised Naturalization Law, the marriage
of an alien woman to a Filipino makes her a Filipina, if she "herself might be lawfully naturalized";

2. That this Court declared as correct the opinion of the Secretary of Justice that the limitation of
Section 15 of the Naturalization Law excludes from the benefits of naturalization by marriage, only
those disqualified from being naturalized under Section 4 of the law qouted in the decision;

3. That evidence to the effect that she is not disqualified may be presented in the action to recover
her bond confiscated by the Commissioner of Immigration;
4. That upon proof of such fact, she may be recognized as Filipina; and

5. That in referring to the disqualification enumerated in the law, the Court somehow left the
impression that no inquiry need be made as to qualifications,  specially considering that the decision
5

cited and footnotes several opinions of the Secretary of Justice, the immediate superior of the
Commissioner of Immigration, the most important of which are the following:

Paragraph (a), section 13 of Act No. 2927, as amended, (now section 15,
Commonwealth Act No. 473), provided that "any woman who is now or may hereafter
be married to a citizen of the Philippines, and who might herself be lawfully
naturalized shall be deemed a citizen of the Philippines." A similar provision in the
naturalization law of the United States has been construed as not requiring the
woman to have the qualifications of residence, good character, etc., as in the case of
naturalization by judicial proceedings, but merely that she is of the race of persons
who may be naturalized. (Kelly v. Owen [Dist. Col. 1868] 7 Wall 496, 5F, 11, 12; ex
parte Tryason [D. C. Wash. 1914] 215 F. 449, 27 Op. Atty. Gen. 507). (Op. No. 168,
s. 1940 of Justice Sec. Jose Abad Santos.)

In a previous opinion rendered for your Office, I stated that the clause "who might
herself be lawfully naturalized", should be construed as not requiring the woman to
have the qualifications of residence, good character, etc., as in cases of
naturalization by judicial proceedings, but merely that she is of the race of persons
who may be naturalized. (Op. No. 79, s. 1940)

Inasmuch as the race qualification has been removed by the Revised Naturalization
Law, it results that any woman who married a citizen of the Philippines prior to or
after June 17, 1939, and the marriage not having been dissolved, and on the
assumption that she possesses none of the disqualifications mentioned in Section 4
of Commonwealth Act No. 473, follows the citizenship of her husband. (Op. No. 176,
s. 1940 of Justice Sec. Jose Abad Santos.)

From the foregoing narration of facts, it would seem that the only material point of
inquiry is as to the citizenship of Arce Machura. If he shall be found to be a citizen of
the Philippines, his wife, Mrs. Lily James Machura, shall likewise be deemed a citizen
of the Philippines pursuant to the provision of Section 15, Commonwealth Act No.
473, which reads in part as follows:

Any woman who is now or may hereafter be married to a citizen of


the Philippines, and who might herself be lawfully naturalized shall be
deemed a citizen of the Philippines.

The phrase "who might herself be lawfully naturalized", as contained in the above
provision, means that the woman who is married to a Filipino citizen must not belong
to any of the disqualified classes enumerated in Section 4 of the Naturalization Law
(Ops., Sec. of Jus., No. 28, s. 1950; No. 43, s. 1948, No. 95, s. 1941; Nos. 79 and
168, s. 1940). Under the facts stated in the within papers, Mrs. Machura does not
appear to be among the disqualified classes mentioned in the law.

It having been shown that Arce Machura or Arsenio Guevara was born as an
illegitimate of a Filipino mother, he should be considered as a citizen of the
Philippines in consonance with the well-settled rule that an illegitimate child follows
the citizenship of his only legally recognized parent, the mother (Op., Sec. of Jus.,
Nos. 58, 98 & 281, s. 1948; No. 96, s. 1949). Her husband being a Filipino, Mrs.
Machura must necessarily be deemed as a citizen of the Philippines by marriage
(Sec. 15, Com. Act No. 473.) (Op. No. 52, s. 1950 of Justice Sec. Ricardo
Nepomuceno.)

The logic and authority of these opinions, compelling as they are, must have so appealed to this
Court that five days later, on May 22, 1957, in Ricardo Cua v. The Board of Commissioners, 101
Phil. 521, Mr. Justice J.B.L. Reyes, reiterated the same ruling on the basis of the following facts:

Tjioe Wu Suan, an Indonesian, arrived in Manila on November 1, 1952, but it turned out that her
passport was forged. On December 10, 1953, a warrant was issued for her arrest for purpose of
deportation. Later, on December 20, 1953, she married Ricardo Cua, a Filipino, and because of said
marriage, the Board of Special Inquiry considered her a Filipina. Upon a review of the case,
however, the Board of Immigration Commissioners insisted on continuing with the deportation
proceedings and so, the husband filed prohibition and mandamus proceedings. The lower court
denied the petition. Although this Court affirmed said decision, it held, on the other hand, that:

Granting the validity of marriage, this Court has ruled in the recent case of Ly Giok
Ha v. Galang, supra, p. 459, that the bare fact of a valid marriage to a citizen does
not suffice to confer his citizenship upon the wife. Section 15 of the Naturalization
Law requires that the alien woman who marries a Filipino must show, in addition, that
she "might herself be lawfully naturalized" as a Filipino citizen. As construed in the
decision cited, this last condition requires proof that the woman who married a
Filipino is herself not disqualified under section 4 of the Naturalization Law.

No such evidence appearing on record, the claim of assumption of Filipino


citizenship by Tjioe Wu Suan, upon her marriage to petitioner, is untenable. The
lower court, therefore, committed no error in refusing to interfere with the deportation
proceedings, where she can anyway establish the requisites indispensable for her
acquisition of Filipino citizenship, as well as the alleged validity of her Indonesian
passport. (Ricardo Cua v. The Board of Immigration Commissioners, G. R. No. L-
9997, May 22, 1957, 101 Phil. 521, 523.) [Emphasis supplied] .

For emphasis, it is reiterated that in the above two cases, this Court expressly gave the parties
concerned opportunity to prove the fact that they were not suffering from any of the disqualifications
of the law without the need of undergoing any judicial naturalization proceeding. It may be stated,
therefore, that according to the above decisions, the law in this country, on the matter of the effect of
marriage of an alien woman to a Filipino is that she thereby becomes a Filipina, if it can be proven
that at the time of such marriage, she does not possess any of the disqualifications enumerated in
Section 4 of the Naturalization Law, without the need of submitting to any naturalization proceedings
under said law.

It is to be admitted that both of the above decisions made no reference to qualifications, that is, as to
whether or not they need also to be proved, but, in any event, it is a fact that the Secretary of Justice
understood them to mean that such qualifications need not be possessed nor proven. Then
Secretary of Justice Jesus Barrera, who later became a distinguished member of this Court,  so 6

ruled in opinions rendered by him subsequent to Ly Giok Ha, the most illustrative of which held: .

At the outset it is important to note that an alien woman married to a Filipino citizen
needs only to show that she "might herself be lawfully naturalized" in order to acquire
Philippine citizenship. Compliance with other conditions of the statute, such as those
relating to the qualifications of an applicant for naturalization through judicial
proceedings, is not necessary. (See: Leonard v. Grant, 5 Fed. 11; 27 Ops. Atty. Gen
[U.S.] 507; Ops. Sec. of Justice, No. 776, s. 1940, and No. 111, s. 1953.

This view finds support in the case of Ly Giok Ha et al. v. Galang et al., G.R. No. L-
10760, promulgated May 17, 1957, where the Supreme Court, construing the
abovequoted section of the Naturalization Law, held that "marriage to a male Filipino
does not vest Philippine citizenship to his foreign wife," unless she "herself may be
lawfully naturalized," and that "this limitation of Section 15 excludes, from the
benefits of naturalization by marriage, those disqualified from being naturalized as
citizens of the Philippines under Section 4 of said Commonwealth Act No. 473." In
other words, disqualification for any of the causes enumerated in Section 4 of the Act
is the decisive factor that defeats the right of the foreign wife of a Philippine citizen to
acquire Philippine citizenship.

xxx xxx xxx

Does petitioner, Lim King Bian, belong to any of these groups The Commissioner of
Immigration does not say so but merely predicates his negative action on the ground
that a warrant of deportation for "overstaying" is pending against the petitioner.

We do not believe the position is well taken. Since the grounds for disqualification for
naturalization are expressly enumerated in the law, a warrant of deportation not
based on a finding of unfitness to become naturalized for any of those specified
causes may not be invoked to negate acquisition of Philippine citizenship by a
foreign wife of a Philippine citizen under Section 15 of the Naturalization Law.
(Inclusio unius est exclusio alterius) (Op. No. 12, s. 1958 of Justice Undersec. Jesus
G. Barrera.)

Regarding the steps that should be taken by an alien woman married to a Filipino
citizen in order to acquire Philippine citizenship, the procedure followed in the Bureau
of Immigration is as follows: The alien woman must file a petition for the cancellation
of her alien certificate of registration alleging, among other things, that she is married
to a Filipino citizen and that she is not disqualified from acquiring her husband's
citizenship pursuant to section 4 of Commonwealth Act No. 473, as amended. Upon
the filing of said petition, which should be accompanied or supported by the joint
affidavit of the petitioner and her Filipino husband to the effect that the petitioner
does not belong to any of the groups disqualified by the cited section from becoming
naturalized Filipino citizen (please see attached CEB Form 1), the Bureau of
Immigration conducts an investigation and thereafter promulgates its order or
decision granting or denying the petition. (Op. No. 38, s. 19058 of Justice Sec. Jesus
G. Barrera.)

This view finds support in the case of Ly Giok Ha et al., v. Galang et al. (G.R. No. L-
10760, promulgated May 17, 1957), where the Supreme Court, construing the
above-quoted section in the Revised Naturalization Law, held that "marriage to a
male Filipino does not vest Philippine citizenship to his foreign wife, unless she
herself may be lawfully naturalized," and that "this limitation of Section 15 excludes,
from the benefits of naturalization by marriage, those disqualified from being
naturalized as citizens of the Philippines under Section 4 of said Commonwealth Act
No. 473." In other words, disqualification for any of the causes enumerated in section
4 of the Act is the decisive factor that defeats the right of an alien woman married to
a Filipino citizen to acquire Philippine citizenship. (Op. 57, s. 1958 of Justice Sec.
Jesus G. Barrera.)

The contention is untenable. The doctrine enunciated in the Ly Giok Ha case is not a
new one. In that case, the Supreme Court held that under paragraph I of Section 15
Of Commonwealth Act No. 473, 'marriage to a male Filipino does not vest Philippine
citizenship to his foreign wife unless she "herself may be lawfully naturalized"', and,
quoting several earlier opinions of the Secretary of Justice, namely: No. 52, s. 1950;
No. 168, s. 1940; No. 95, s. 1941; No. 63, s. 1948; No. 28. s. 1950, "this limitation of
section 15 excludes from the benefits of naturalization by marriage, those disqualified
from being naturalized as citizens of the Philippines under section 4 of said
Commonwealth Act No. 473." (Op. 134, s. 1962 of Justice Undersec. Magno S.
Gatmaitan.)

It was not until more than two years later that, in one respect, the above construction of the law was
importantly modified by this Court in Lee Suan Ay, supra, in which the facts were as follows:

Upon expiration of the appellant Lee Suan Ay's authorized period of temporary stay
in the Philippines (25 March 1955), on 26 March 1955 the Commissioner of
Immigration asked the bondsman to present her to the Bureau of Immigration within
24 hours from receipt of notice, otherwise the bond will be confiscated(Annex 1). For
failure of the bondsman to comply with the foregoing order, on 1 April 1955. the
Commissioner of Immigration ordered the cash bond confiscated (Annex E).
Therefore, there was an order issued by the Commissioner of Immigration
confiscating or forfeiting the cash bond. Unlike in forfeiture of bail bonds in criminal
proceedings, where the Court must enter an order forfeiting the bail bond and the
bondsman must be given an opportunity to present his principal or give a satisfactory
reason for his inability to do so, before final judgment may be entered against the
bondsman,(section 15, Rule 110; U.S. v. Bonoan, 22 Phil. 1.) in forfeiture of bonds
posted for the temporary stay of an alien in the Philippines, no court proceeding is
necessary. Once a breach of the terms and conditions of the undertaking in the bond
is committed, the Commissioner of Immigration may, under the terms and conditions
thereof, declare it forfeited in favor of the Government. (In the meanwhile, on April 1,
1955, Lee Suan Ay and Alberto Tan, a Filipino, were joined in marriage by the
Justice of the Peace of Las Piñas, Rizal.)

Mr. Justice Sabino Padilla speaking for a unanimous court which included Justices Concepcion and
Reyes who had penned Ly Giok Ha, and Ricardo Cua, ruled thus:

The fact that Lee Suan Ay (a Chinese) was married to a Filipino citizen does not
relieve the bondsman from his liability on the bond. The marriage took place on 1
April 1955, and the violation of the terms and conditions of the undertaking in the
bond — failure to depart from the Philippines upon expiration of her authorized
period of temporary stay in the Philippines (25 March 1955) and failure to report to
the Commissioner of Immigration within 24 hours from receipt of notice — were
committed before the marriage. Moreover, the marriage of a Filipino citizen to an
alien does not automatically confer Philippine citizenship upon the latter. She must
possess the qualifications required by law to become a Filipino citizen by
naturalization.* There is no showing that the appellant Lee Suan Ay possesses all
the qualifications and none of the disqualifications provided for by law to become a
Filipino citizen by naturalization.
Pertinently to be noted at once in this ruling, which, to be sure, is the one relied upon in the appealed
decision now before Us, is the fact that the footnote of the statement therein that the alien wife "must
possess the qualifications required by law to become a Filipino citizen by naturalization" makes
reference to Section 15, Commonwealth Act 473 and precisely, also to Ly Giok Ha v. Galang, supra.
As will be recalled, on the other hand, in the opinions of the Secretary of Justice explicitly adopted by
the Court in Ly Giok Ha, among them, Opinion No. 176, Series of 1940, above-quoted, it was clearly
held that "(I)n a previous opinion rendered for your Office, I stated that the clause "who might herself
be lawfully naturalized", should be construed as not requiring the woman to have the qualifications of
residence, good character, etc., as in cases of naturalization by judicial proceedings but merely that
she is of the race by persons who may be naturalized. (Op. No. 79, s. 1940)

Since Justice Padilla gave no reason at all for the obviously significant modification of the
construction of the law, it could be said that there was need for clarification of the seemingly new
posture of the Court. The occasion for such clarification should have been in Kua Suy, etc., et al. vs.
The Commissioner of Immigration, G.R. No. L-13790, October 31, 1963, penned by Mr. Justice
J.B.L. Reyes, who had rendered the opinion in Ricardo Cua, supra, which followed that in Ly Giok
Ha, supra, but apparently seeing no immediate relevancy in the case on hand then of the particular
point in issue now, since it was not squarely raised therein similarly as in Lee Suan Ay, hence,
anything said on the said matter would at best be no more than obiter dictum, Justice Reyes limited
himself to holding that "Under Section 15 of the Naturalization Act, the wife is deemed a citizen of
the Philippines only if she "might herself be lawfully naturalized," so that the fact of marriage to a
citizen, by itself alone, does not suffice to confer citizenship, as this Court has previously ruled in Ly
Giok Ha v. Galang, 54 O.G. 356, and in Cua v. Board of Immigration Commissioners, 53 O.G. 8567;
and there is here no evidence of record as to the qualifications or absence of disqualifications of
appellee Kua Suy", without explaining the apparent departure already pointed out from Ly Giok Ha
and Ricardo Cua. Even Justice Makalintal, who wrote a separate concurring and dissenting opinion
merely lumped together Ly Giok Ha, Ricardo Cua and Lee Suan Ay and opined that both
qualifications and non-disqualifications have to be shown without elucidating on what seemed to be
departure from the said first two decisions.

It was only on November 30, 1963 that to Mr. Justice Roberto Regala fell the task of rationalizing the
Court's position. In Lo San Tuang v. Galang, G.R. No. L-18775, November 30, 1963, 9 SCRA 638,
the facts were simply these: Lo San Tuang, a Chinese woman, arrived in the Philippines on July 1,
1960 as a temporary visitor with authority to stay up to June 30, 1961. She married a Filipino on
January 7, 1961, almost six months before the expiry date of her permit, and when she was
requested to leave after her authority to stay had expired, she refused to do so, claiming she had
become a Filipina by marriage, and to bolster her position, she submitted an affidavit stating
explicitly that she does not possess any of the disqualifications enumerated in the Naturalization
Law, Commonwealth Act 473. When the case reached the court, the trial judge held for the
government that in addition to not having any of the disqualifications referred to, there was need that
Lo San Tuang should have also possessed all the qualifications of residence, moral character,
knowledge of a native principal dialect, etc., provided by the law. Recognizing that the issue squarely
to be passed upon was whether or not the possession of all the qualifications were indeed needed to
be shown apart from non-disqualification, Justice Regala held affirmatively for the Court, reasoning
out thus: .

It is to be noted that the petitioner has anchored her claim for citizenship on the basis
of the decision laid down in the case of Leonard v. Grant, 5 Swy. 603, 5 F 11, where
the Circuit Court of Oregon held that it was only necessary that the woman "should
be a person of the class or race permitted to be naturalized by existing laws, and that
in respect of the qualifications arising out of her conduct or opinions, being the wife of
a citizen, she is to be regarded as qualified for citizenship, and therefore considered
a citizen." (In explanation of its conclusion, the Court said: "If, whenever during the
life of the woman or afterwards, the question of her citizenship arises in a legal
proceeding, the party asserting her citizenship by reason of her marriage with a
citizen must not only prove such marriage, but also that the woman then possessed
all the further qualifications necessary to her becoming naturalized under existing
laws, the statute will be practically nugatory, if not a delusion and a share. The proof
of the facts may have existed at the time of the marriage, but years after, when a
controversy arises upon the subject, it may be lost or difficult to find.")

In other words, all that she was required to prove was that she was a free white
woman or a woman of African descent or nativity, in order to be deemed an
American citizen, because, with respect to the rest of the qualifications on residence,
moral character, etc., she was presumed to be qualified.

Like the law in the United States, our former Naturalization Law (Act No. 2927, as
amended by Act No. 3448) specified the classes of persons who alone might
become citizens of the Philippines, even as it provided who were disqualified. Thus,
the pertinent provisions of that law provided:

Section 1. Who may become Philippine citizens — Philippine


citizenship may be acquired by (a) natives of the Philippines who are
not citizens thereof under the Jones Law; (b) natives of the Insular
possessions of the United States; (c) citizens of the United States, or
foreigners who under the laws of the United States may become
citizens of said country if residing therein.

Section 2. Who are disqualified. — The following cannot be


naturalized as Philippine citizens: (a) Persons opposed to organized
government or affiliated with any association or group of persons who
uphold and teach doctrines opposing all organized government; (b)
persons defending or teaching the necessity or propriety of violence,
personal assault or assassination for the success and predominance
of their ideas; (c) polygamists or believers in the practice of
polygamy; (d) persons convicted of crimes involving moral turpitude;
(e) persons suffering from mental alienation or incurable contagious
diseases; (f) citizens or subjects of nations with whom the United
States and the Philippines are at war, during the period of such war.

Section 3. Qualifications. — The persons comprised in subsection (a)


of section one of this Act, in order to be able to acquire Philippine
citizenship, must be not less than twenty-one years of age on the day
of the hearing of their petition.

The persons comprised in subsections (b) and (c) of said section one
shall, in addition to being not less than twenty-one years of age on
the day of the hearing of the petition, have all and each of the
following qualifications:

First. Residence in the Philippine Islands for a continuous period of


not less than five years, except as provided in the next following
section;
Second. To have conducted themselves in a proper and
irreproachable manner during the entire period of their residence in
the Philippine Islands, in their relation with the constituted
government as well as with the community in which they are living;

Third. To hold in the Philippine Islands real estate worth not less than
one thousand pesos, Philippine currency, or have some known trade
or profession; and

Fourth. To speak and write English, Spanish, or some native tongue.

In case the petitioner is a foreign subject, he shall, besides, declare in


writing and under oath his intention of renouncing absolutely and
perpetually all faith and allegiance to the foreign authority, state or
sovereignty of which he was a native, citizen or subject.

Applying the interpretation given by Leonard v. Grant supra, to our law as it then
stood, alien women married to citizens of the Philippines must, in order to be deemed
citizens of the Philippines, be either (1) natives of the Philippines who were not
citizens thereof under the Jones Law, or (2) natives of other Insular possessions of
the United States, or (3) citizens of the United States or foreigners who under the
laws of the United States might become citizens of that country if residing therein.
With respect to the qualifications set forth in Section 3 of the former law, they were
deemed to have the same for all intents and purposes.

But, with the approval of the Revised Naturalization Law (Commonwealth Act No.
473) on June 17, 1939, Congress has since discarded class or racial consideration
from the qualifications of applicants for naturalization (according to its proponent, the
purpose in eliminating this consideration was, first, to remove the features of the
existing naturalization act which discriminated in favor of the Caucasians and against
Asiatics who are our neighbors, and are related to us by racial affinity and, second, to
foster amity with all nations [Sinco, Phil. Political Law 502 — 11 ed.]), even as it
retained in Section 15 the phrase in question. The result is that the phrase "who
might herself be lawfully naturalized" must be understood in the context in which it is
now found, in a setting so different from that in which it was found by the Court
in Leonard v. Grant.

The only logical deduction from the elimination of class or racial consideration is that,
as the Solicitor General points out, the phrase "who might herself be lawfully
naturalized" must now be understood as referring to those who under Section 2 of
the law are qualified to become citizens of the Philippines.

There is simply no support for the view that the phrase "who might herself be lawfully
naturalized" must now be understood as requiring merely that the alien woman must
not belong to the class of disqualified persons under Section 4 of the Revised
Naturalization Law. Such a proposition misreads the ruling laid down in Leonard v.
Grant. A person who is not disqualified is not necessarily qualified to become a
citizen of the Philippines, because the law treats "qualifications" and
"disqualifications" in separate sections. And then it must not be lost sight of that even
under the interpretation given to the former law, it was to be understood that the alien
woman was not disqualified under Section 2 of that law. Leonard v. Grant did not rule
that it was enough if the alien woman does not belong to the class of disqualified
persons in order that she may be deemed to follow the citizenship of her husband:
What that case held was that the phrase "who might herself be lawfully naturalized,
merely means that she belongs to the class or race of persons qualified to become
citizens by naturalization — the assumption being always that she is not otherwise
disqualified.

We therefore hold that under the first paragraph of Section 15 of the Naturalization
Law, an alien woman, who is married to a citizen of the Philippines, acquires the
citizenship of her husband only if she has all the qualifications and none of the
disqualifications provided by law. Since there is no proof in this case that petitioner
has all the qualifications and is not in any way disqualified, her marriage to a Filipino
citizen does not automatically make her a Filipino citizen. Her affidavit to the effect
that she is not in any way disqualified to become a citizen of this country was
correctly disregarded by the trial court, the same being self-serving.

Naturally, almost a month later in Sun Peck Yong v. Commissioner of Immigration, G.R. No. L-
20784, December 27, 1963, 9 SCRA 875, wherein the Secretary of Foreign Affairs reversed a
previous resolution of the preceding administration to allow Sun Peck Yong and her minor son to
await the taking of the oath of Filipino citizenship of her husband two years after the decision
granting him nationalization and required her to leave and this order was contested in court, Justice
Barrera held:

In the case of Lo San Tuang v. Commissioner of Immigration (G.R. No. L-18775,


promulgated November 30, 1963; Kua Suy vs. Commissioner of Immigration, L-
13790, promulgated October 31, 1963), we held that the fact that the husband
became a naturalized citizen does not automatically make the wife a citizen of the
Philippines. It must also be shown that she herself possesses all the qualifications,
and none of the disqualifications, to become a citizen. In this case, there is no
allegation, much less showing, that petitioner-wife is qualified to become a Filipino
citizen herself. Furthermore, the fact that a decision was favorably made on the
naturalization petition of her husband is no assurance that he (the husband) would
become a citizen, as to make a basis for the extension of her temporary stay.

On the same day, in Tong Siok Sy v. Vivo, G.R. No. L-21136, December 27, 1963, 9 SCRA 876,
Justice Barrera reiterated the same ruling and citing particularly Lo San Tuang and Kua Suy, held
that the marriage of Tong Siok Sy to a Filipino on November 12, 1960 at Taichung, Taiwan and her
taking oath of Filipino citizenship before the Philippine Vice-Consul at Taipeh, Taiwan on January 6,
1961 did not make her a Filipino citizen, since she came here only in 1961 and obviously, she had
not had the necessary ten-year residence in the Philippines required by the law.

Such then was the status of the jurisprudential law on the matter under discussion when Justice
Makalintal sought a reexamination thereof in Choy King Tee v. Galang, G.R. No. L-18351, March 26,
1965, 13 SCRA 402. Choy King Tee's husband was granted Philippine citizenship on January 13,
1959 and took the oath on January 31 of the same year. Choy King Tee first came to the Philippines
in 1955 and kept commuting between Manila and Hongkong since then, her last visa before the case
being due to expire on February 14, 1961. On January 27, 1961, her husband asked the
Commissioner of Immigration to cancel her alien certificate of registration, as well as their child's, for
the reason that they were Filipinos, and when the request was denied as to the wife, a mandamus
was sought, which the trial court granted. Discussing anew the issue of the need for qualifications,
Justice Makalintal not only reiterated the arguments of Justice Regala in Lo San Tuang but added
further that the ruling is believed to be in line with the national policy of selective admission to
Philippine citizenship.
7
No wonder, upon this authority, in Austria v. Conchu, G.R. No. L-20716, June 22, 1965, 14 SCRA
336, Justice J.P. Bengzon readily reversed the decision of the lower court granting the writs of
mandamus and prohibition against the Commissioner of Immigration, considering that Austria's wife,
while admitting she did not possess all the qualifications for naturalization, had submitted only an
affidavit that she had none of the disqualifications therefor. So also did Justice Dizon similarly hold
eight days later in Brito v. Commissioner, G.R. No. L-16829, June 30, 1965, 14 SCRA 539.

Then came the second Ly Giok Ha case  wherein Justice J. B. L. Reyes took occasion to expand on
8

the reasoning of Choy King Tee by illustrating with examples "the danger of relying exclusively on
the absence of disqualifications, without taking into account the other affirmative requirements of the
law."
9

Lastly, in Go Im Ty v. Republic, G.R. No. L-17919, decided on July 30, 1966,   Justice Zaldivar held
10

for the Court that an alien woman who is widowed during the dependency of the naturalization
proceedings of her husband, in order that she may be allowed to take the oath as Filipino, must,
aside from proving compliance with the requirements of Republic Act 530, show that she possesses
all the qualifications and does not suffer from any of the disqualifications under the Naturalization
Law, citing in the process the decision to such effect discussed above,  even as he impliedly
11

reversed pro tanto the ruling in Tan Lin v. Republic, G.R. No. L-13786, May 31, 1961, 2 SCRA 383.

Accordingly, in Burca, Justice Sanchez premised his opinion on the assumption that the point now
under discussion is settled law.

In the case now at bar, the Court is again called upon to rule on the same issue. Under Section 15 of
the Naturalization Law, Commonwealth Act 473, providing that:

SEC. 15. Effect of the naturalization on wife and children. — Any woman, who is now
or may hereafter be married to a citizen of the Philippines, and who might herself be
lawfully naturalized shall be deemed a citizen of the Philippines.

Minor children of persons naturalized under this law who have been born in the
Philippines shall be considered citizens thereof.

A foreign-born minor child, if dwelling in the Philippines at the time of the


naturalization of the parent, shall automatically become a Philippine citizen, and a
foreign-born child, who is not in the Philippines at the time the parent is naturalized,
shall be deemed a Philippine citizen only during his minority, unless he begins to
reside permanently in the Philippines when still a minor, in which case, he will
continue to be a Philippine citizen even after becoming of age.

A child born outside of the Philippines after the naturalization of his parent, shall be
considered a Philippine citizen unless within one year after reaching the age of
majority he fails to register himself as a Philippine citizen at the American Consulate
of the country where he resides, and to take the necessary oath of allegiance.

is it necessary, in order that an alien woman who marries a Filipino or who is married to a man who
subsequently becomes a Filipino, may become a Filipino citizen herself, that, aside from not
suffering from any of the disqualifications enumerated in the law, she must also possess all the
qualifications required by said law? if nothing but the unbroken line from Lee Suan Ay to Go Im Ty,
as recounted above, were to be considered, it is obvious that an affirmative answer to the question
would be inevitable, specially, if it is noted that the present case was actually submitted for decision
on January 21, 1964 yet, shortly after Lo San Tuang, Tong Siok Sy and Sun Peck Yong, all supra,
and even before Choy King Tee, supra, were decided. There are other circumstances, however,
which make it desirable, if not necessary, that the Court take up the matter anew. There has been a
substantial change in the membership of the Court since Go Im Ty, and of those who were in the
Court already when Burca was decided, two members, Justice Makalintal and Castro concurred only
in the result, precisely, according to them, because (they wanted to leave the point now under
discussion open in so far as they are concerned.   Truth to tell, the views and arguments discussed
12

at length with copious relevant authorities, in the motion for reconsideration as well as in the
memorandum of the amici curae   in the Burca case cannot just be taken lightly and summarily
13

ignored, since they project in the most forceful manner, not only the legal and logical angles of the
issue, but also the imperative practical aspects thereof in the light of the actual situation of the
thousands of alien wives of Filipinos who have so long, even decades, considered themselves as
Filipinas and have always lived and acted as such, officially or otherwise, relying on the long
standing continuous recognition of their status as such by the administrative authorities in charge of
the matter, as well as by the courts. Under these circumstances, and if only to afford the Court an
opportunity to consider the views of the five justices who took no part in Go Im Ty (including the
writer of this opinion), the Court decided to further reexamine the matter. After all, the ruling first laid
in Lee Suan Ay, and later in Lo San Tuang, Choy King Tee stand the second (1966) Ly Giok Ha, did
not categorically repudiate the opinions of the Secretary of Justice relied upon by the first (1959) Ly
Giok Ha. Besides, some points brought to light during the deliberations in this case would seem to
indicate that the premises of the later cases can still bear further consideration.

Whether We like it or not, it is undeniably factual that the legal provision We are construing, Section
15, aforequoted, of the Naturalization Law has been taken directly, copied and adopted from its
American counterpart. To be more accurate, said provision is nothing less than a reenactment of the
American provision. A brief review of its history proves this beyond per adventure of doubt.

The first Naturalization Law of the Philippines approved by the Philippine Legislature under
American sovereignty was that of March 26, 1920, Act No. 2927. Before then, as a consequence of
the Treaty of Paris, our citizenship laws were found only in the Organic Laws, the Philippine Bill of
1902, the Act of the United States Congress of March 23, 1912 and later the Jones Law of 1916. In
fact, Act No. 2927 was enacted pursuant to express authority granted by the Jones Law. For obvious
reasons, the Philippines gained autonomy on the subjects of citizenship and immigration only after
the effectivity of the Philippine Independence Act. This made it practically impossible for our laws on
said subject to have any perspective or orientation of our own; everything was American.

The Philippine Bill of 1902 provided pertinently: .

SECTION 4. That all inhabitants of the Philippine Islands continuing to reside herein
who were Spanish subjects on the eleventh day of April, eighteen-hundred and
ninety-nine, and then resided in said Islands, and their children born subsequent
thereto, shall be deemed and held to be citizens of the Philippine Islands and as
such entitled to the protection of the United States, except such as shall have elected
to preserve their allegiance to the Crown of Spain in accordance with the provisions
of the treaty of peace between the United States and Spain signed at Paris
December tenth, eighteen hundred and ninety-eight.

This Section 4 of the Philippine Bill of 1902 was amended by Act of Congress of March 23, 1912, by
adding a provision as follows:

Provided, That the Philippine Legislature is hereby authorized to provide by law for
the acquisition of Philippine citizenship by those natives of the Philippine Islands who
do not come within the foregoing provisions, the natives of other insular possessions
of the United States, and such other persons residing in the Philippine Islands who
would become citizens of the United States, under the laws of the United States, if
residing therein.

The Jones Law reenacted these provisions substantially: .

SECTION 2. That all inhabitants of the Philippine Islands who were Spanish subjects
on the eleventh day of April, eighteen hundred and ninety-nine, and then resided in
said islands, and their children born subsequent thereto, shall be deemed and held to
be citizens of the Philippine Islands, except such as shall have elected to preserve
their allegiance to the Crown of Spain in accordance with the provisions of the treaty
of peace between the United States and Spain, signed at Paris December tenth,
eighteen hundred and ninety-eight and except such others as have since become
citizens of some other country: Provided, That the Philippine Legislature, herein
provided for, is hereby authorized to provide by law for the acquisition of Philippine
citizenship by those natives of the Philippine Islands who do not come within the
foregoing provisions, the natives of the insular possessions of the United States, and
such other persons residing in the Philippine Islands who are citizens of the United
States under the laws of the United States if residing therein.

For aught that appears, there was nothing in any of the said organic laws regarding the effect of
marriage to a Filipino upon the nationality of an alien woman, albeit under the Spanish Civil Code
provisions on citizenship, Articles 17 to 27, which were, however, abrogated upon the change of
sovereignty, it was unquestionable that the citizenship of the wife always followed that of the
husband. Not even Act 2927 contained any provision regarding the effect of naturalization of an
alien, upon the citizenship of his alien wife, nor of the marriage of such alien woman with a native
born Filipino or one who had become a Filipino before the marriage, although Section 13 thereof
provided thus: .

SEC. 13. Right of widow and children of petitioners who have died. — In case a
petitioner should die before the final decision has been rendered, his widow and
minor children may continue the proceedings. The decision rendered in the case
shall, so far as the widow and minor children are concerned, produce the same legal
effect as if it had been rendered during the life of the petitioner.

It was not until November 30, 1928, upon the approval of Act 3448, amending Act 2977, that the
following provisions were added to the above Section 13:

SECTION 1. The following new sections are hereby inserted between sections
thirteen and fourteen of Act Numbered Twenty-nine hundred and Twenty-seven:

SEC. 13(a). Any woman who is now or may hereafter be married to a


citizen of the Philippine Islands and who might herself be lawfully
naturalized, shall be deemed a citizen of the Philippine Islands.

SEC. 13(b). Children of persons who have been duly naturalized


under this law, being under the age of twenty-one years at the time of
the naturalization of their parents, shall, if dwelling in the Philippine
Islands, be considered citizens thereof.
SEC. 13(c). Children of persons naturalized under this law who have
been born in the Philippine Islands after the naturalization of their
parents shall be considered citizens thereof.

When Commonwealth Act 473, the current naturalization law, was enacted on June 17, 1939, the
above Section 13 became its Section 15 which has already been quoted earlier in this decision. As
can be seen, Section 13 (a) abovequoted was re-enacted practically word for word in the first
paragraph of this Section 15 except for the change of Philippine Islands to Philippines. And it could
not have been on any other basis than this legislative history of our naturalization law that each and
everyone of the decisions of this Court from the first Ly Giok Ha to Go Im Ty, discussed above, were
rendered.

As stated earlier, in the opinion of Chief Justice Concepcion in the first Ly Giok Ha, it was quite clear
that for an alien woman who marries a Filipino to become herself a Filipino citizen, there is no need
for any naturalization proceeding because she becomes a Filipina ipso facto from the time of such
marriage, provided she does not suffer any of the disqualifications enumerated in Section 4 of
Commonwealth Act 473, with no mention being made of whether or not the qualifications
enumerated in Section 2 thereof need be shown. It was only in Lee Suan Ay in 1959 that the
possession of qualifications were specifically required, but it was not until 1963, in Lo San Tuang,
that Justice Regala reasoned out why the possession of the qualifications provided by the law
should also be shown to be possessed by the alien wife of a Filipino, for her to become a Filipina by
marriage.

As may be recalled, the basic argument advanced by Justice Regala was briefly as follows: That
"like the law in the United States, our Naturalization Law specified the classes of persons who alone
might become citizens, even as it provided who were disqualified," and inasmuch as Commonwealth
Act 473, our Naturalization Law since 1939 did not reenact the section providing who might become
citizens, allegedly in order to remove racial discrimination in favor of Caucasians and against
Asiatics, "the only logical deduction ... is that the phrase "who might herself be lawfully naturalized"
must now be understood as referring to those who under Section 2 of the law are qualified to
become citizens of the Philippines" and "there is simply no support for the view that the phrase "who
might herself be lawfully naturalized" must now be understood as requiring merely that the alien
woman must not belong to the class of disqualified persons under Section 4 of the Revised
Naturalization Law." 14

A similar line of reasoning was followed in Choy King Tee, which for ready reference may be qouted:

The question has been settled by the uniform ruling of this Court in a number of
cases. The alien wife of a Filipino citizen must first prove that she has all the
qualifications required by Section 2 and none of the disqualifications enumerated in
Section 4 of the Naturalization Law before she may be deemed a Philippine citizen
(Lao Chay v. Galang, L-190977, Oct. 30, 1964, citing Lo San Tuang v. Galang, L-
18775, Nov. 30, 1963; Sun Peck Yong v. Commissioner of Immigration, L-20784,
December 27, 1963; Tong Siok Sy v. Vivo, L-21136, December 27, 1963). The writer
of this opinion has submitted the question anew to the court for a possible
reexamination of the said ruling in the light of the interpretation of a similar law in the
United States after which Section 15 of our Naturalization Law was patterned. That
law was section 2 of the Act of February 10, 1855 (Section 1994 of the Revised
Statutes of the U.S.). The local law, Act No. 3448, was passed on November 30,
1928 as an amendment to the former Philippine Naturalization Law, Act No. 2927,
which was approved on March 26, 1920. Under this Naturalization Law, acquisition of
Philippine citizenship was limited to three classes of persons, (a) Natives of the
Philippines who were not citizens thereof; (b) natives of the other insular possessions
of the United States; and (c) citizens of the United States, or foreigners who, under
the laws of the United States, may become citizens of the latter country if residing
therein. The reference in subdivision (c) to foreigners who may become American
Citizens is restrictive in character, for only persons of certain specified races were
qualified thereunder. In other words, in so far as racial restrictions were concerned
there was at the time a similarity between the naturalization laws of the two countries
and hence there was reason to accord here persuasive force to the interpretation
given in the United States to the statutory provision concerning the citizenship of
alien women marrying American citizens.

This Court, however, believes that such reason has ceased to exist since the
enactment of the Revised Naturalization Law, (Commonwealth Act No. 473) on June
17, 1939. The racial restrictions have been eliminated in this Act, but the provision
found in Act No. 3448 has been maintained. It is logical to presume that when
Congress chose to retain the said provision — that to be deemed a Philippine citizen
upon marriage the alien wife must be one "who might herself be lawfully naturalized,"
the reference is no longer to the class or race to which the woman belongs, for class
or race has become immaterial, but to the qualifications and disqualifications for
naturalization as enumerated in Sections 2 and 4 of the statute. Otherwise the
requirement that the woman "might herself be lawfully naturalized" would be
meaningless surplusage, contrary to settled norms of statutory construction.

The rule laid down by this Court in this and in other cases heretofore decided is
believed to be in line with the national policy of selective admission to Philippine
citizenship, which after all is a privilege granted only to those who are found worthy
thereof, and not indiscriminately to anybody at all on the basis alone of marriage to a
man who is a citizen of the Philippines, irrespective of moral character, ideological
beliefs, and identification with Filipino ideals, customs and traditions.

Appellee here having failed to prove that she has all the qualifications for
naturalization, even, indeed, that she has none of the disqualifications, she is not
entitled to recognition as a Philippine citizen.

In the second Ly Giok Ha, the Court further fortified the arguments in favor of the same conclusion
thus:

On cross-examination, she (Ly Giok Ha) failed to establish that: (1) she has been
residing in the Philippines for a continuous period of at least (10) years (p. 27,
t.s.n., id.); (2) she has a lucrative trade, profession, or lawful occupation (p. 13,
t.s.n., id.); and (3) she can speak and write English, or any of the principal Philippine
languages (pp. 12, 13, t.s.n., id.).

While the appellant Immigration Commissioner contends that the words emphasized
indicate that the present Naturalization Law requires that an alien woman who
marries a Filipino husband must possess the qualifications prescribed by section 2 in
addition to not being disqualified under any of the eight ("a" to "h") subheadings of
section 4 of Commonwealth Act No. 473, in order to claim our citizenship by
marriage, both the appellee and the court below (in its second decision) sustain the
view that all that the law demands is that the woman be not disqualified under
section 4.
At the time the present case was remanded to the court of origin (1960) the question
at issue could be regarded as not conclusively settled, there being only the concise
pronouncement in Lee Suan Ay, et al. v. Galang, G. R. No. L-11855, Dec. 23, 1959,
to the effect that:

The marriage of a Filipino citizen to an alien does not automatically


confer Philippine citizenship upon the latter. She must possess the
qualifications required by law to become a Filipino citizen by
naturalization.

Since that time, however, a long line of decisions of this Court has firmly established
the rule that the requirement of section 15 of Commonwealth Act 473 (the
Naturalization Act), that an alien woman married to a citizen should be one who
"might herself be lawfully naturalized," means not only woman free from the
disqualifications enumerated in section 4 of the Act but also one who possesses the
qualifications prescribed by section 2 of Commonwealth Act 473 (San Tuan v.
Galang, L-18775, Nov. 30, 1963; Sun Peck Yong v. Com. of Immigration, L-20784,
Dee. 27, 1963; Tong Siok Sy v. Vivo, L-21136, Dec. 27, 1963; Austria v. Conchu, L-
20716, June 22, 1965; Choy King Tee v. Galang, L-18351, March 26, 1965; Brito v.
Com. of Immigration, L-16829, June 30, 1965).

Reflection will reveal why this must be so. The qualifications prescribed under
section 2 of the Naturalization Act, and the disqualifications enumerated in its section
4 are not mutually exclusive; and if all that were to be required is that the wife of a
Filipino be not disqualified under section 4, the result might well be that citizenship
would be conferred upon persons in violation of the policy of the statute. For
example, section 4 disqualifies only —

(c) Polygamists or believers in the practice of polygamy; and

(d) Persons convicted of crimes involving moral turpitude,

so that a blackmailer, or a maintainer of gambling or bawdy houses, not previously


convicted by a competent court would not be thereby disqualified; still, it is certain
that the law did not intend such person to be admitted as a citizen in view of the
requirement of section 2 that an applicant for citizenship "must be of good moral
character."

Similarly, the citizen's wife might be a convinced believer in racial supremacy, in


government by certain selected classes, in the right to vote exclusively by certain
"herrenvolk", and thus disbelieve in the principles underlying the Philippine
Constitution; yet she would not be disqualified under section 4, as long as she is not
"opposed to organized government," nor affiliated to groups "upholding or teaching
doctrines opposing all organized governments", nor "defending or teaching the
necessity or propriety of violence, personal assault or assassination for the success
or predominance of their ideas." Et sic de caeteris.

The foregoing instances should suffice to illustrate the danger of relying exclusively
on the absence of disqualifications, without taking into account the other affirmative
requirements of the law, which, in the case at bar, the appellee Ly Giok Ha
admittedly does not possess.
As to the argument that the phrase "might herself be lawfully naturalized" was
derived from the U.S. Revised Statutes (section 1994) and should be given the same
territorial and racial significance given to it by American courts, this Court has
rejected the same in Lon San Tuang v. Galang, L-18775, November 30, 1963; and
in Choy King Tee v. Galang, L-18351, March 26, 1965.

It is difficult to minimize the persuasive force of the foregoing rationalizations, but a closer study
thereof cannot bat reveal certain relevant considerations which adversely affect the premises on
which they are predicated, thus rendering the conclusions arrived thereby not entirely unassailable.

1. The main proposition, for instance, that in eliminating Section 1 of Act 2927 providing who are
eligible for Philippine citizenship, the purpose of Commonwealth Act 473, the Revised Naturalization
Law, was to remove the racial requirements for naturalization, thereby opening the door of Filipino
nationality to Asiatics instead of allowing the admission thereto of Caucasians only, suffers from lack
of exact accuracy. It is important to note, to start with, that Commonwealth Act 473 did away with the
whole Section 1 of Act 2927 which reads, thus:

SECTION 1. Who may become Philippine citizens. — Philippine citizenship may be


acquired by: (a) natives of the Philippines who are not citizens thereof under the
Jones Law; (b) natives of the other Insular possessions of the United States; (c)
citizens of the United States, or foreigners who under the laws of the United States
may become citizens of said country if residing therein.

and not only subdivision (c) thereof. Nowhere in this whole provision was there any mention of race
or color of the persons who were then eligible for Philippine citizenship. What is more evident from
said provision is that it reflected the inevitable subordination of our legislation during the pre-
Commonwealth American regime to the understandable stations flowing from our staffs as a territory
of the United States by virtue of the Treaty of Paris. In fact, Section 1 of Act 2927 was precisely
approved pursuant to express authority without which it could not have been done, granted by an
amendment to Section 4 of the Philippine Bill of 1902 introduced by the Act of the United States
Congress of March 23, 1912 and which was reenacted as part of the Jones Law of 1916, the
pertinent provisions of which have already been footed earlier. In truth, therefore, it was because of
the establishment of the Philippine Commonwealth and in the exercise of our legislative autonomy
on citizenship matters under the Philippine Independence Act that Section 1 of Act 2927 was
eliminated,   and not purposely to eliminate any racial discrimination contained in our Naturalization
15

Law. The Philippine Legislature naturally wished to free our Naturalization Law from the impositions
of American legislation. In other words, the fact that such discrimination was removed was one of the
effects rather than the intended purpose of the amendment.

2. Again, the statement in Choy King Tee to the effect that "the reference in subdivision (c) (of
Section 1 of Act 2927) to foreigners who may become American citizens is restrictive in character,
for only persons of certain specified races were qualified thereunder" fails to consider the exact
import of the said subdivision. Explicitly, the thrust of the said subdivision was to confine the grant
under it of Philippine citizenship only to the three classes of persons therein mentioned, the third of
which were citizens of the United States and, corollarily, persons who could be American citizens
under her laws. The words used in the provision do not convey any idea of favoring aliens of any
particular race or color and of excluding others, but more accurately, they refer to all the
disqualifications of foreigners for American citizenship under the laws of the United States. The fact
is that even as of 1906, or long before 1920, when our Act 2927 became a law, the naturalization,
laws of the United States already provided for the following disqualifications in the Act of the
Congress of June 29, 1906:
SEC. 7. That no person who disbelieves in or who is opposed to organized
government, or who is a member of or affiliated with any organization entertaining
and teaching such disbelief in or opposition to organized government, or who
advocates or teaches the duty, necessity, or propriety of the unlawful assaulting or
killing of any officer or officers, either of specific individuals or of officers generally, of
the Government of the United States, or of any other organized government,
because of his or their official character, or who is a polygamist, shall be naturalized
or be made a citizen of the United States.

and all these disqualified persons were, therefore, ineligible for Philippine citizenship under Section 1
of Act 2927 even if they happened to be Caucasians. More importantly, as a matter of fact, said
American law, which was the first "Act to Establish a Bureau of Immigration and Naturalization and
to provide for a Uniform Rule for Naturalization of Aliens throughout the United States" contained no
racial disqualification requirement, except as to Chinese, the Act of May 6, 1882 not being among
the expressly repealed by this law, hence it is clear that when Act 2927 was enacted, subdivision (e)
of its Section 1 could not have had any connotation of racial exclusion necessarily, even if it were
traced back to its origin in the Act of the United States Congress of 1912 already mentioned
above.   Thus, it would seem that the rationalization in the qouted decisions predicated on the theory
16

that the elimination of Section 1 of Act 2927 by Commonwealth Act 473 was purposely for no other
end than the abolition of racial discrimination in our naturalization law has no clear factual basis.  17

3. In view of these considerations, there appears to be no cogent reason why the construction
adopted in the opinions of the Secretary of Justice referred to in the first Ly Giok Ha decision of the
Chief Justice should not prevail. It is beyond dispute that the first paragraph of Section 15 of
Commonwealth Act 473 is a reenactment of Section 13(a) of Act 2927, as amended by Act 3448,
and that the latter is nothing but an exact copy, deliberately made, of Section 1994 of the Raised
Statutes of the United States as it stood before its repeal in 1922.   Before such repeal, the phrase
18

"who might herself be lawfully naturalized" found in said Section 15 had a definite unmistakable
construction uniformly foIlowed in all courts of the United States that had occasion to apply the same
and which, therefore, must be considered, as if it were written in the statute itself. It is almost trite to
say that when our legislators enacted said section, they knew of its unvarying construction in the
United States and that, therefore, in adopting verbatim the American statute, they have in effect
incorporated into the provision, as thus enacted, the construction given to it by the American courts
as well as the Attorney General of the United States and all administrative authorities, charged with
the implementation of the naturalization and immigration laws of that country. (Lo Cham v. Ocampo,
77 Phil., 635 [1946]; Laxamana v. Baltazar, 92 Phil., 32 [1952]; Hartley v. Commissioner, 295 U.S.
216, 79 L. ed. 1399, 55 S Ct. 756 [19353; Helvering v. Winmill, 305 U.S. 79, 83 L ed. 52, 59 S Ct. 45
[1938]; Helvering v. R. J. Reynolds Tobacco Co., 306 U.S. 110, 83 L ed. 536, 59 S Ct. 423 [1939].
[p. 32, Memo of Amicus Curiae]).

A fairly comprehensive summary of the said construction by the American courts and administrative
authorities is contained in United States of America ex rel. Dora Sejnensky v. Robert E. Tod,
Commissioner of Immigration, Appt., 295 Fed. 523, decided November 14, 1922, 26 A. L. R. 1316
as follows:

Section 1994 of the Revised Statutes (Comp. Stat. 3948, 2 Fed. Sta. Anno. 2d ed. p.
117) provides as follows: "Any woman who is now or may hereafter be married to a
citizen of the United States, and who might herself be lawfully naturalized, shall be
deemed a citizen."

Section 1944 of the Revised Stat. is said to originate in the Act of Congress of
February 10, 1855 (10 Stat. at L. 604, chap. 71), which in its second section provided
"that any woman, who might lawfully be naturalized under the existing laws, married,
or who shall be married to a citizen of the United States, shall be deemed and taken
to be a citizen."

And the American Statute of 1855 is substantially a copy of the earlier British Statute
7 & 8 Vict. chap. 66, s 16, 1844, which provided that "any woman married, or who
shall be married, to a natural-born subject or person naturalized, shall be deemed
and taken to be herself naturalized, and have all the rights and privileges of a natural
born subject."

The Act of Congress of September 22, 1922 (42 Stat. at L. 1021, chap. 411, Comp.
Stat. 4358b, Fed. Stat. Anno. Supp. 1922, p. 255), being "An Act Relative to the
Naturalization and Citizenship of Married Women," in 2, provides "that any woman
who marries a citizen of the United States after the passage of this Act, ... shall not
become a citizen of the United States by reason of such marriage ..."

Section 6 of the act also provides "that 1994 of the Revised Statutes ... are
repealed."

Section 6 also provides that `such repeal shall not terminate citizenship acquired or
retained under either of such sections, ..." meaning 2 and 6. So that this Act of
September 22, 1922, has no application to the facts of the present case, as the
marriage of the relator took place prior to its passage. This case, therefore, depends
upon the meaning to be attached to 1994 of the Revised Statutes.

In 1868 the Supreme Court, in Kelly v. Owen, 7 Wall. 496, 498, 19 L. ed. 283, 284,
construed this provision as found in the Act of 1855 as follows: "The term, "who
might lawfully be naturalized under the existing laws," only limits the application of
the law to free white women. The previous Naturalization Act, existing at the time,
only required that the person applying for its benefits should be "a free white person,"
and not an alien enemy."

This construction limited the effect of the statute to those aliens who belonged to the
class or race which might be lawfully naturalized, and did not refer to any of the other
provisions of the naturalization laws as to residence or moral character, or to any of
the provisions of the immigration laws relating to the exclusion or deportation of
aliens.

In 1880, in Leonard v. Grant (C. C.) 5 Fed. 11, District Judge Deady also construed
the Act of 1855, declaring that "any woman who is now or may hereafter be married
to a citizen of the United States, and might herself be lawfully naturalized, shall be
deemed a citizen." He held that "upon the authorities, and the reason, if not the
necessity, of the case," the statute must be construed as in effect declaring that an
alien woman, who is of the class or race that may be lawfully naturalized under the
existing laws, and who marries a citizen of the United States, is such a citizen also,
and it was not necessary that it should appear affirmatively that she possessed the
other qualifications at the time of her marriage to entitle her to naturalization.

In 1882, the Act of 1855 came before Mr. Justice Harlan, sitting in the circuit court,
in United States v. Kellar, 13 Fed. 82. An alien woman, a subject of Prussia came to
the United States and married here a naturalized citizen. Mr. Justice Harlan, with the
concurrence of Judge Treat, held that upon her marriage she became ipso facto a
citizen of the United States as fully as if she had complied with all of the provisions of
the statutes upon the subject of naturalization. He added: "There can be no doubt of
this, in view of the decision of the Supreme Court of the United, States in Kelly v.
Owen, 7 Wall. 496, 19 L. ed. 283." The alien "belonged to the class of persons" who
might be lawfully naturalized.

In 1904, in Hopkins v. Fachant, 65 C. C. A. 1, 130 Fed. 839, an alien woman came to


the United States from France and entered the country contrary to the immigration
laws. The immigration authorities took her into custody at the port of New York, with
the view of deporting her. She applied for her release under a writ of habeas corpus,
and pending the disposition of the matter she married a naturalized American citizen.
The circuit court of appeals for the ninth Circuit held, affirming the court below, that
she was entitled to be discharged from custody. The court declared: "The rule is well
settled that her marriage to a naturalized citizen of the United States entitled her to
be discharged. The status of the wife follows that of her husband, ... and by virtue of
her marriage her husband's domicil became her domicil." .

In 1908, the circuit court for the district of Rhode Island in Re Rustigian, 165. Fed.
980, had before it the application of a husband for his final decree of naturalization. It
appeared that at that time his wife was held by the immigration authorities at New
York on the ground that she was afflicted with a dangerous and contagious disease.
Counsel on both sides agreed that the effect of the husband's naturalization would
be to confer citizenship upon the wife. In view of that contingency District Judge
Brown declined to pass upon the husband's application for naturalization, and
thought it best to wait until it was determined whether the wife's disease was curable.
He placed his failure to act on the express ground that the effect of naturalizing the
husband might naturalize her. At the same time he express his opinion that the
husband's naturalization would not effect her naturalization, as she was not one who
could become lawfully naturalized. "Her own capacity (to become naturalized)," the
court stated "is a prerequisite to her attaining citizenship. If herself lacking in that
capacity, the married status cannot confer it upon her." Nothing, however, was
actually decided in that case, and the views expressed therein are really nothing
more than mere dicta. But, if they can be regarded as something more than that, we
find ourselves, with all due respect for the learned judge, unable to accept them.

In 1909, in United States ex rel. Nicola v. Williams, 173 Fed. 626, District Judge
Learned Hand held that an alien woman, a subject of the Turkish Empire, who
married an American citizen while visiting Turkey, and then came to the United
States, could not be excluded, although she had, at the time of her entry, a disease
which under the immigration laws would have been sufficient ground for her
exclusion, if she bad not had the status of a citizen. The case was brought into this
court on appeal, and in 1911 was affirmed, in 106 C. C. A. 464, 184 Fed. 322. In that
case, however at the time the relators married, they might have been lawfully
naturalized, and we said: "Even if we assume the contention of the district attorney to
be correct that marriage will not make a citizen of a woman who would be excluded
under our immigration laws, it does not affect these relators."

We held that, being citizens, they could not be excluded as aliens; and it was also
said to be inconsistent with the policy of our law that the husband should be a citizen
and the wife an alien. The distinction between that case and the one now before the
court is that, in the former case, the marriage took place before any order of
exclusion had been made, while in this the marriage was celebrated after such an
order was made. But such an order is a mere administrative provision, and has not
the force of a judgment of a court, and works no estoppel. The administrative order is
based on the circumstances that existed at the time the order of exclusion was
made. If the circumstances change prior to the order being carried into effect, it
cannot be executed. For example, if an order of exclusion should be based on the
ground that the alien was at the time afflicted with a contagious disease, and it
should be made satisfactorily to appear, prior to actual deportation, that the alien had
entirely recovered from the disease, we think it plain that the order could not be
carried into effect. So, in this case, if, after the making of the order of exclusion and
while she is permitted temporarily to remain, she in good faith marries an American
citizen, we cannot doubt the validity of her marriage, and that she thereby acquired,
under international law and under 1994 of the Revised Statutes, American
citizenship, and ceased to be an alien. There upon, the immigration authorities lost
their jurisdiction over her, as that jurisdiction applies only to aliens, and not to
citizens.

In 1910, District Judge Dodge, in Ex parte Kaprielian, 188 Fed. 694, sustained the
right of the officials to deport a woman under the following circumstances: She
entered this country in July, 1910, being an alien and having been born in Turkey.
She was taken into custody by the immigration authorities in the following
September, and in October a warrant for her deportation was issued. Pending
hearings as to the validity of that order, she was paroled in the custody of her
counsel. The ground alleged for her deportation was that she was afflicted with a
dangerous and contagious disease at the time of her entry. One of the reasons
assigned to defeat deportation was that the woman had married a citizen of the
United States pending the proceedings for her deportation. Judge Dodge declared
himself unable to believe that a marriage under such circumstances "is capable of
having the effect claimed, in view of the facts shown." He held that it was no part of
the intended policy of 1994 to annul or override the immigration laws, so as to
authorize the admission into the country of the wife of a naturalized alien not
otherwise entitled to enter, and that an alien woman, who is of a class of persons
excluded by law from admission to the United States does not come within the
provisions of that section. The court relied wholly upon the dicta contained in the
Rustigian Case. No other authorities were cited.

In 1914, District Judge Neterer, in Ex parte Grayson, 215 Fed. 449, construed 1994
and held that where, pending proceedings to deport an alien native of France as an
alien prostitute, she was married to a citizen of the United States, she thereby
became a citizen, and was not subject to deportation until her citizenship was
revoked by due process of law. It was his opinion that if, as was contended, her
marriage was conceived in fraud, and was entered into for the purpose of evading
the immigration laws and preventing her deportation, such fact should be established
in a court of competent jurisdiction in an action commenced for the purpose. The
case was appealed and the appeal was dismissed. 134 C. C. A. 666, 219 Fed. 1022.

It is interesting also to observe the construction placed upon the language of the
statute by the Department of Justice. In 1874, Attorney General Williams, 14 Ops.
Atty. Gen. 402, passing upon the Act of February 10, 1855, held that residence within
the United States for the period required by the naturalization laws was riot
necessary in order to constitute an alien woman a citizen, she having married a
citizen of the United States abroad, although she never resided in the United States,
she and her husband having continued to reside abroad after the marriage.
In 1909, a similar construction was given to the Immigration Act of May 5, 1907, in an
opinion rendered by Attorney General Wickersham. It appeared an unmarried
woman, twenty-eight years of age and a native of Belgium, arrived in New York and
went at once to a town in Nebraska, where she continued to reside. About fifteen
months after her arrival she was taken before a United States commissioner by way
of instituting proceedings under the Immigration Act (34 Stat. at L. 898, chap. 1134,
Comp. Stat. 4242, 3 Fed. Stat. Anno. 2d ed. p. 637) for her deportation, on the
ground that she had entered this country for the purpose of prostitution, and had
been found an inmate of a house of prostitution and practicing the same within three
years after landing. It appeared, however, that after she was taken before the United
States commissioner, but prior to her arrest under a warrant by the Department of
Justice, she was lawfully married to a native-born citizen of the United States. The
woman professed at the time of her marriage an intention to abandon her previous
mode of life and to remove with her husband to his home in Pennsylvania. He knew
what her mode of life had been, but professed to believe in her good intentions. The
question was raised as to the right to deport her, the claim being advance that by her
marriage she bad become an American citizen and therefore could not be deported.
The Attorney General ruled against the right to deport her as she had become an
American citizen. He held that the words, "who might herself be lawfully naturalized,"
refer to a class or race who might be lawfully naturalized, and that compliance with
the other conditions of the naturalization laws was not required. 27 Ops. Atty. Gen.
507.

Before concluding this opinion, we may add that it has not escaped our observation
that Congress, in enacting the Immigration Act of 1917, so as to provide, in 19, "that
the marriage to an American citizen of a female of the sexually immoral classes ...
shall not invest such female with United States citizenship if the marriage of such
alien female shall be solemnized after her arrest or after the commission of acts
which make her liable to deportation under this act."

Two conclusions seem irresistibly to follow from the above change in the law:

(1) Congress deemed legislation essential to prevent women of the immoral class
avoiding deportation through the device of marrying an American citizen.

(2) If Congress intended that the marriage of an American citizen with an alien
woman of any other of the excluded classes, either before or after her detention,
should not confer upon her American citizenship, thereby entitling her to enter the
country, its intention would have been expressed, and 19 would not have been
confined solely to women of the immoral class.

Indeed, We have examined all the leading American decisions on the subject and We have found no
warrant for the proposition that the phrase "who might herself be lawfully naturalized" in Section
1994 of the Revised Statutes was meant solely as a racial bar, even if loose statements in some
decisions and other treaties and other writings on the subject would seem to give such impression.
The case of Kelley v. Owen, supra, which appears to be the most cited among the first of the
decisions   simply held:
19

As we construe this Act, it confers the privileges of citizenship upon women married
to citizens of the United States, if they are of the class of persons for whose
naturalization the previous Acts of Congress provide. The terms "married" or "who
shall be married," do not refer in our judgment, to the time when the ceremony of
marriage is celebrated, but to a state of marriage. They mean that, whenever a
woman, who under previous Acts might be naturalized, is in a state of marriage to a
citizen, whether his citizenship existed at the passage of the Act or subsequently, or
before or after the marriage, she becomes, by that fact, a citizen also. His citizenship,
whenever it exists, confers, under the Act, citizenship upon her. The construction
which would restrict the Act to women whose husbands, at the time of marriage, are
citizens, would exclude far the greater number, for whose benefit, as we think, the
Act was intended. Its object, in our opinion, was to allow her citizenship to follow that
of her husband, without the necessity of any application for naturalization on her part;
and, if this was the object, there is no reason for the restriction suggested.

The terms, "who might lawfully be naturalized under the existing laws," only limit the
application of the law to free white women. The previous Naturalization Act, existing
at the time only required that the person applying for its benefits should be "a free
white person," and not an alien enemy. Act of April 14th, 1802, 2 Stat. at L. 153.

A similar construction was given to the Act by the Court of Appeals of New York,
in Burton v. Burton, 40 N. Y. 373; and is the one which gives the widest extension to
its provisions.

Note that write the court did say that "the terms, "who might lawfully be naturalized under existing
laws" only limit the application to free white women"   it hastened to add that "the previous
20

Naturalization Act, existing at the time, ... required that the person applying for its benefits should be
(not only) a "free white person" (but also) ... not an alien enemy." This is simply because under the
Naturalization Law of the United States at the time the case was decided, the disqualification of
enemy aliens had already been removed by the Act of July 30, 1813, as may be seen in the
corresponding footnote hereof anon. In other words, if in the case of Kelly v. Owen only the race
requirement was mentioned, the reason was that there was no other non-racial requirement or no
more alien enemy disqualification at the time; and this is demonstrated by the fact that the court took
care to make it clear that under the previous naturalization law, there was also such requirement in
addition to race. This is impotent, since as stated in re Rustigian, 165 Fed. Rep. 980, "The
expression used by Mr. Justice Field, (in Kelly v. Owen) the terms "who might lawfully be naturalized
under existing laws" only limit the application of the law to free white women, must be interpreted in
the application to the special facts and to the incapacities under the then existing laws," (at p. 982)
meaning that whether or not an alien wife marrying a citizen would be a citizen was dependent, not
only on her race and nothing more necessarily, but on whether or not there were other
disqualifications under the law in force at the time of her marriage or the naturalization of her
husband.

4. As already stated, in Lo San Tuang, Choy King Tee and the second Ly Giok Ha, the Court drew
the evidence that because Section 1 of Act 2927 was eliminated by Commonwealth Act 473, it
follows that in place of the said eliminated section particularly its subdivision (c), being the criterion
of whether or not an alien wife "may be lawfully naturalized," what should be required is not only that
she must not be disqualified under Section 4 but that she must also possess the qualifications
enumerated in Section 2, such as those of age, residence, good moral character, adherence to the
underlying principles of the Philippine Constitution, irreproachable conduct, lucrative employment or
ownership of real estate, capacity to speak and write English or Spanish and one of the principal
local languages, education of children in certain schools, etc., thereby implying that, in effect, sails
Section 2 has been purposely intended to take the place of Section 1 of Act 2927. Upon further
consideration of the proper premises, We have come, to the conclusion that such inference is not
sufficiently justified.
To begin with, nothing extant in the legislative history, which We have already explained above of
the mentioned provisions has been shown or can be shown to indicate that such was the clear intent
of the legislature. Rather, what is definite is that Section 15 is, an exact copy of Section 1994 of the
Revised Statutes of the United States, which, at the time of the approval of Commonwealth Act 473
had already a settled construction by American courts and administrative authorities.

Secondly, as may be gleaned from the summary of pertinent American decisions quoted above,
there can be no doubt that in the construction of the identically worded provision in the Revised
Statutes of the United States, (Section 1994, which was taken, from the Act of February 10, 1855) all
authorities in the United States are unanimously agreed that the qualifications of residence, good
moral character, adherence to the Constitution, etc. are not supposed to be considered, and that the
only eligibility to be taken into account is that of the race or class to which the subject belongs, the
conceptual scope of which, We have just discussed.   In the very case of Leonard v. Grant, supra,
21

discussed by Justice Regala in Lo San Tuang, the explanation for such posture of the American
authorities was made thus:

The phrase, "shall be deemed a citizen" in section 1994 Rev. St., or as it was in the
Act of 1855, supra, "shall be deemed and taken to be a citizen" while it may imply
that the person to whom it relates has not actually become a citizen by ordinary
means or in the usual way, as by the judgment of a competent court, upon a proper
application and proof, yet it does not follow that such person is on that account
practically any the less a citizen. The word "deemed" is the equivalent of
"considered" or "judged"; and, therefore, whatever an act of Congress requires to be
"deemed" or "taken" as true of any person or thing, must, in law, be considered as
having been duly adjudged or established concerning "such person or thing, and
have force and effect accordingly. When, therefore, Congress declares that an alien
woman shall, under certain circumstances, be "deemed' an American citizen, the
effect when the contingency occurs, is equivalent to her being naturalized directly by
an act of Congress, or in the usual mode thereby prescribed.

Unless We disregard now the long settled familiar rule of statutory construction that in a situation like
this wherein our legislature has copied an American statute word for word, it is understood that the
construction already given to such statute before its being copied constitute part of our own law,
there seems to be no reason how We can give a different connotation or meaning to the provision in
question. At least, We have already seen that the views sustaining the contrary conclusion appear to
be based on in accurate factual premises related to the real legislative background of the framing of
our naturalization law in its present form.

Thirdly, the idea of equating the qualifications enumerated in Section 2 of Commonwealth Act 473
with the eligibility requirements of Section 1 of Act 2927 cannot bear close scrutiny from any point of
view. There is no question that Section 2 of Commonwealth Act 473 is more or less substantially the
same as Section 3 of Act 2927. In other words, Section 1 of Act 2927 co-existed already with
practically the same provision as Section 2 of Commonwealth Act 473. If it were true that the phrase
"who may be lawfully naturalized" in Section 13 (a) of Act 2927, as amended by Act 3448, referred to
the so-called racial requirement in Section 1 of the same Act, without regard to the provisions of
Section 3 thereof, how could the elimination of Section 1 have the effect of shifting the reference to
Section 3, when precisely, according to the American jurisprudence, which was prevailing at the time
Commonwealth Act 473 was approved, such qualifications as were embodied in said Section 3,
which had their counterpart in the corresponding American statutes, are not supposed to be taken
into account and that what should be considered only are the requirements similar to those provided
for in said Section 1 together with the disqualifications enumerated in Section 4?
Fourthly, it is difficult to conceive that the phrase "who might be lawfully naturalized" in Section 15
could have been intended to convey a meaning different than that given to it by the American courts
and administrative authorities. As already stated, Act 3448 which contained said phrase and from
which it was taken by Commonwealth Act 473, was enacted in 1928. By that, time, Section 1994 of
the Revised Statutes of the United States was no longer in force because it had been repealed
expressly the Act of September 22, 1922 which did away with the automatic naturalization of alien
wives of American citizens and required, instead, that they submit to regular naturalization
proceedings, albeit under more liberal terms than those of other applicants. In other words, when our
legislature adopted the phrase in question, which, as already demonstrated, had a definite
construction in American law, the Americans had already abandoned said phraseology in favor of a
categorical compulsion for alien wives to be natural judicially. Simple logic would seem to dictate
that, since our lawmakers, at the time of the approval of Act 3448, had two choices, one to adopt the
phraseology of Section 1994 with its settled construction and the other to follow the new posture of
the Americans of requiring judicial naturalization and it appears that they have opted for the first, We
have no alternative but to conclude that our law still follows the old or previous American Law On the
subject. Indeed, when Commonwealth Act 473 was approved in 1939, the Philippine Legislature,
already autonomous then from the American Congress, had a clearer chance to disregard the old
American law and make one of our own, or, at least, follow the trend of the Act of the U.S. Congress
of 1922, but still, our legislators chose to maintain the language of the old law. What then is
significantly important is not that the legislature maintained said phraseology after Section 1 of Act
2927 was eliminated, but that it continued insisting on using it even after the Americans had
amended their law in order to provide for what is now contended to be the construction that should
be given to the phrase in question. Stated differently, had our legislature adopted a phrase from an
American statute before the American courts had given it a construction which was acquiesced to by
those given upon to apply the same, it would be possible for Us to adopt a construction here
different from that of the Americans, but as things stand, the fact is that our legislature borrowed the
phrase when there was already a settled construction thereof, and what is more, it appears that our
legislators even ignored the modification of the American law and persisted in maintaining the old
phraseology. Under these circumstances, it would be in defiance of reason and the principles of
Statutory construction to say that Section 15 has a nationalistic and selective orientation and that it
should be construed independently of the previous American posture because of the difference of
circumstances here and in the United States. It is always safe to say that in the construction of a
statute, We cannot fall on possible judicial fiat or perspective when the demonstrated legislative
point of view seems to indicate otherwise.

5. Viewing the matter from another angle, there is need to emphasize that in reality and in effect, the
so called racial requirements, whether under the American laws or the Philippine laws, have hardly
been considered as qualifications in the same sense as those enumerated in Section 3 of Act 2927
and later in Section 2 of Commonwealth Act 473. More accurately, they have always been
considered as disqualifications, in the sense that those who did not possess them were the ones
who could not "be lawfully naturalized," just as if they were suffering from any of the disqualifications
under Section 2 of Act 2927 and later those under Section 4 of Commonwealth Act 473, which,
incidentally, are practically identical to those in the former law, except those in paragraphs (f) and (h)
of the latter.   Indeed, such is the clear impression anyone will surely get after going over all the
22

American decisions and opinions quoted and/or cited in the latest USCA (1970), Title 8, section
1430, pp. 598-602, and the first decisions of this Court on the matter, Ly Giok Ha (1959) and Ricardo
Cua, citing with approval the opinions of the secretary of Justice.   Such being the case, that is, that
23

the so-called racial requirements were always treated as disqualifications in the same light as the
other disqualifications under the law, why should their elimination not be viewed or understood as a
subtraction from or a lessening of the disqualifications? Why should such elimination have instead
the meaning that what were previously considered as irrelevant qualifications have become
disqualifications, as seems to be the import of the holding in Choy King Tee to the effect that the
retention in Section 15 of Commonwealth Act 473 of the same language of what used to be Section
13 (a) of Act 2927 (as amended by Act 3448), notwithstanding the elimination of Section 1 of the
latter, necessarily indicates that the legislature had in mind making the phrase in question "who may
be lawfully naturalized" refer no longer to any racial disqualification but to the qualification under
Section 2 of Commonwealth Act 473? Otherwise stated, under Act 2927, there were two groups of
persons that could not be naturalized, namely, those falling under Section 1 and those falling under
Section 2, and surely, the elimination of one group, i.e. those belonging to Section 1, could not have
had, by any process of reasoning, the effect of increasing, rather than decreasing, the
disqualifications that used to be before such elimination. We cannot see by what alchemy of logic
such elimination could have convicted qualifications into disqualifications specially in the light of the
fact that, after all, these are disqualifications clearly set out as such in the law distinctly and
separately from qualifications and, as already demonstrated, in American jurisprudence,
qualifications had never been considered to be of any relevance in determining "who might be
lawfully naturalized," as such phrase is used in the statute governing the status of alien wives of
American citizens, and our law on the matter was merely copied verbatim from the American
statutes.

6. In addition to these arguments based on the applicable legal provisions and judicial opinions,
whether here or in the United States, there are practical considerations that militate towards the
same conclusions. As aptly stated in the motion for reconsideration of counsel for petitioner-appellee
dated February 23, 1967, filed in the case of Zita Ngo Burca v. Republic, supra:

Unreasonableness of requiring alien wife to prove "qualifications" —

There is one practical consideration that strongly militates against a construction that
Section 15 of the law requires that an alien wife of a Filipino must affirmatively prove
that she possesses the qualifications prescribed under Section 2, before she may be
deemed a citizen. Such condition, if imposed upon an alien wife, becomes
unreasonably onerous and compliance therewith manifestly difficult. The
unreasonableness of such requirement is shown by the following:

1. One of the qualifications required of an Applicant for naturalization


under Section 2 of the law is that the applicant "must have resided in
the Philippines for a continuous period of not less than ten years." If
this requirement is applied to an alien wife married to a Filipino
citizen, this means that for a period of ten years at least, she cannot
hope to acquire the citizenship of her husband. If the wife happens to
be a citizen of a country whose law declares that upon her marriage
to a foreigner she automatically loses her citizenship and acquires
the citizenship of her husband, this could mean that for a period of
ten years at least, she would be stateless. And even after having
acquired continuous residence in the Philippines for ten years, there
is no guarantee that her petition for naturalization will be granted, in
which case she would remain stateless for an indefinite period of
time.

2. Section 2 of the law likewise requires of the applicant for


naturalization that he "must own real estate in the Philippines worth
not less than five thousand pesos, Philippine currency, or must have
some known lucrative trade, profession, or lawful occupation."
Considering the constitutional prohibition against acquisition by an
alien of real estate except in cases of hereditary succession (Art. XIII,
Sec. 5, Constitution), an alien wife desiring to acquire the citizenship
of her husband must have to prove that she has a lucrative income
derived from a lawful trade, profession or occupation. The income
requirement has been interpreted to mean that the petitioner herself
must be the one to possess the said income. (Uy v. Republic, L-
19578, Oct. 27, 1964; Tanpa Ong vs. Republic, L-20605, June 30,
1965; Li Tong Pek v. Republic, L-20912, November 29, 1965). In
other words, the wife must prove that she has a lucrative income
derived from sources other than her husband's trade, profession or
calling. It is of common knowledge, and judicial notice may be taken
of the fact that most wives in the Philippines do not have gainful
occupations of their own. Indeed, Philippine law, recognizing the
dependence of the wife upon the husband, imposes upon the latter
the duty of supporting the former. (Art. 291, Civil Code). It should be
borne in mind that universally, it is an accepted concept that when a
woman marries, her primary duty is to be a wife, mother and
housekeeper. If an alien wife is not to be remiss in this duty, how can
she hope to acquire a lucrative income of her own to qualify her for
citizenship?

3. Under Section 2 of the law, the applicant for naturalization "must


have enrolled his minor children of school age, in any of the public
schools or private schools recognized by the Office of the Private
Education of the Philippines, where Philippine history, government
and civics are taught or prescribed as part of the school curriculum
during the entire period of residence in the Philippines required of him
prior to the hearing of his petition for naturalization as Philippine
citizen." If an alien woman has minor children by a previous marriage
to another alien before she marries a Filipino, and such minor
children had not been enrolled in Philippine schools during her period
of residence in the country, she cannot qualify for naturalization
under the interpretation of this Court. The reason behind the
requirement that children should be enrolled in recognized
educational institutions is that they follow the citizenship of their
father. (Chan Ho Lay v. Republic, L-5666, March 30, 1954; Tan Hi v.
Republic, 88 Phil. 117 [1951]; Hao Lian Chu v. Republic, 87 Phil. 668
[1950]; Yap Chin v. Republic, L-4177, May 29, 1953; Lim Lian Hong
v. Republic, L-3575, Dec. 26, 1950). Considering that said minor
children by her first husband generally follow the citizenship of their
alien father, the basis for such requirement as applied to her does not
exist. Cessante ratione legis cessat ipsa lex.

4. Under Section 3 of the law, the 10-year continuous residence


prescribed by Section 2 "shall be understood as reduced to five years
for any petitioner (who is) married to a Filipino woman." It is absurd
that an alien male married to a Filipino wife should be required to
reside only for five years in the Philippines to qualify for citizenship,
whereas an alien woman married to a Filipino husband must reside
for ten years.

Thus under the interpretation given by this Court, it is more difficult for an alien wife
related by marriage to a Filipino citizen to become such citizen, than for a foreigner
who is not so related. And yet, it seems more than clear that the general purpose of
the first paragraph of Section 15 was obviously to accord to an alien woman, by
reason of her marriage to a Filipino, a privilege not similarly granted to other aliens. It
will be recalled that prior to the enactment of Act No. 3448 in 1928, amending Act
No. 2927 (the old Naturalization Law), there was no law granting any special
privilege to alien wives of Filipinos. They were treated as any other foreigner. It was
precisely to remedy this situation that the Philippine legislature enacted Act No.
3448. On this point, the observation made by the Secretary of Justice in 1941 is
enlightening:

It is true that under, Article 22 of the (Spanish) Civil Code, the wife
follows the nationality of the husband; but the Department of State of
the United States on October 31, 1921, ruled that the alien wife of a
Filipino citizen is not a Filipino citizen, pointing out that our Supreme
Court in the leading case of Roa v. Collector of Customs (23 Phil.
315) held that Articles 17 to 27 of the Civil Code being political have
been abrogated upon the cession of the Philippine Islands to the
United States. Accordingly, the stated taken by the Attorney-General
prior to the envictment of Act No. 3448, was that marriage of alien
women to Philippine citizens did not make the former citizens of this
counting. (Op. Atty. Gen., March 16, 1928) .

To remedy this anomalous condition, Act No. 3448 was enacted in


1928 adding section 13(a) to Act No. 2927 which provides that "any
woman who is now or may hereafter be married to a citizen of the
Philippine Islands, and who might herself be lawfully naturalized,
shall be deemed a citizen of the Philippine Islands. (Op. No. 22, s.
1941; emphasis ours).

If Section 15 of the, Revised Naturalization Law were to be interpreted, as this Court


did, in such a way as to require that the alien wife must prove the qualifications
prescribed in Section 2, the privilege granted to alien wives would become illusory. It
is submitted that such a construction, being contrary to the manifested object of the
statute must be rejected.

A statute is to be construed with reference to its manifest object, and


if the language is susceptible of two constructions, one which will
carry out and the other defeat such manifest object, it should receive
the former construction. (In re National Guard, 71 Vt. 493, 45 A.
1051; Singer v. United States, 323 U.S. 338, 89 L. ed. 285. See also,
U.S. v. Navarro, 19 Phil. 134 [1911]; U. S. v. Toribio, 15 Phil. 85
[1910).

... A construction which will cause objectionable results should be


avoided and the court will, if possible, place on the statute a
construction which will not result in injustice, and in accordance with
the decisions construing statutes, a construction which will result in
oppression, hardship, or inconveniences will also be avoided, as will
a construction which will prejudice public interest, or construction
resulting in unreasonableness, as well as a construction which will
result in absurd consequences.

So a construction should, if possible, be avoided if the result would


be an apparent inconsistency in legislative intent, as has been
determined by the judicial decisions, or which would result in futility,
redundancy, or a conclusion not contemplated by the legislature; and
the court should adopt that construction which will be the least likely
to produce mischief. Unless plainly shown to have been the intention
of the legislature, an interpretation which would render the
requirements of the statute uncertain and vague is to be avoided, and
the court will not ascribe to the legislature an intent to confer an
illusory right. ... (82 C.J.S., Statutes, sec. 326, pp. 623-632).

7. In Choy King Tee and the second Ly Giok Ha, emphasis was laid on the need for aligning the
construction of Section 15 with "the national policy of selective admission to Philippine citizenship."
But the question may be asked, is it reasonable to suppose that in the pursuit of such policy, the
legislature contemplated to make it more difficult if not practically impossible in some instances, for
an alien woman marrying a Filipino to become a Filipina than any ordinary applicant for
naturalization, as has just been demonstrated above? It seems but natural and logical to assume
that Section 15 was intended to extend special treatment to alien women who by marrying a Filipino
irrevocably deliver themselves, their possessions, their fate and fortunes and all that marriage
implies to a citizen of this country, "for better or for worse." Perhaps there can and will be cases
wherein the personal conveniences and benefits arising from Philippine citizenship may motivate
such marriage, but must the minority, as such cases are bound to be, serve as the criterion for the
construction of law? Moreover, it is not farfetched to believe that in joining a Filipino family the alien
woman is somehow disposed to assimilate the customs, beliefs and ideals of Filipinos among whom,
after all, she has to live and associate, but surely, no one should expect her to do so even before
marriage. Besides, it may be considered that in reality the extension of citizenship to her is made by
the law not so much for her sake as for the husband. Indeed, We find the following observations
anent the national policy rationalization in Choy King Tee and Ly Giok Ha (the second) to be quite
persuasive:

We respectfully suggest that this articulation of the national policy begs the question.
The avowed policy of "selectives admission" more particularly refers to a case where
citizenship is sought to be acquired in a judicial proceeding for naturalization. In such
a case, the courts should no doubt apply the national policy of selecting only those
who are worthy to become citizens. There is here a choice between accepting or
rejecting the application for citizenship. But this policy finds no application in cases
where citizenship is conferred by operation of law. In such cases, the courts have no
choice to accept or reject. If the individual claiming citizenship by operation of law
proves in legal proceedings that he satisfies the statutory requirements, the courts
cannot do otherwise than to declare that he is a citizen of the Philippines. Thus, an
individual who is able to prove that his father is a Philippine citizen, is a citizen of the
Philippines, "irrespective of his moral character, ideological beliefs, and identification
with Filipino ideals, customs, and traditions." A minor child of a person naturalized
under the law, who is able to prove the fact of his birth in the Philippines, is likewise a
citizen, regardless of whether he has lucrative income, or he adheres to the
principles of the Constitution. So it is with an alien wife of a Philippine citizen. She is
required to prove only that she may herself be lawfully naturalized, i.e., that she is
not one of the disqualified persons enumerated in Section 4 of the law, in order to
establish her citizenship status as a fact.

A paramount policy consideration of graver import should not be overlooked in this


regard, for it explains and justifies the obviously deliberate choice of words. It is
universally accepted that a State, in extending the privilege of citizenship to an alien
wife of one of its citizens could have had no other objective than to maintain a unity
of allegiance among the members of the family. (Nelson v. Nelson, 113 Neb. 453,
203 N. W. 640 [1925]; see also "Convention on the Nationality of Married Women:
Historical Background and Commentary." UNITED NATIONS, Department of
Economic and Social Affairs E/CN, 6/399, pp. 8 et seq.). Such objective can only be
satisfactorily achieved by allowing the wife to acquire citizenship derivatively through
the husband. This is particularly true in the Philippines where tradition and law has
placed the husband as head of the family, whose personal status and decisions
govern the life of the family group. Corollary to this, our laws look with favor on the
unity and solidarity of the family (Art. 220, Civil Code), in whose preservation of State
as a vital and enduring interest. (See Art. 216, Civil Code). Thus, it has been said
that by tradition in our country, there is a theoretic identity of person and interest
between husband and wife, and from the nature of the relation, the home of one is
that of the other. (See De la Viña v. Villareal, 41 Phil. 13). It should likewise be said
that because of the theoretic identity of husband and wife, and the primacy of the
husband, the nationality of husband should be the nationality of the wife, and the
laws upon one should be the law upon the other. For as the court, in Hopkins v.
Fachant (9th Cir., 1904) 65 C.C.A., 1, 130 Fed. 839, held: "The status of the wife
follows that of the husband, ... and by virtue of her marriage her husband's domicile
became her domicile." And the presumption under Philippine law being that
the property relations of husband and wife are under the regime of conjugal
partnership (Art. 119, Civil Code), the income of one is also that of the other.

It is, therefore, not congruent with our cherished traditions of family unity and identity
that a husband should be a citizen and the wife an alien, and that the national
treatment of one should be different from that of the other. Thus, it cannot be that the
husband's interests in property and business activities reserved by law to citizens
should not form part of the conjugal partnership and be denied to the wife, nor that
she herself cannot, through her own efforts but for the benefit of the partnership,
acquire such interests. Only in rare instances should the identity of husband and wife
be refused recognition, and we submit that in respect of our citizenship laws, it
should only be in the instances where the wife suffers from the disqualifications
stated in Section 4 of the Revised Naturalization Law. (Motion for
Reconsideration, Burca vs. Republic, supra.)

With all these considerations in mind, We are persuaded that it is in the best interest of all concerned
that Section 15 of the Naturalization Law be given effect in the same way as it was understood and
construed when the phrase "who may be lawfully naturalized," found in the American statute from
which it was borrowed and copied verbatim, was applied by the American courts and administrative
authorities. There is merit, of course in the view that Philippine statutes should be construed in the
light of Philippine circumstances, and with particular reference to our naturalization laws. We should
realize the disparity in the circumstances between the United States, as the so-called "melting pot"
of peoples from all over the world, and the Philippines as a developing country whose Constitution is
nationalistic almost in the come. Certainly, the writer of this opinion cannot be the last in rather
passionately insisting that our jurisprudence should speak our own concepts and resort to American
authorities, to be sure, entitled to admiration, and respect, should not be regarded as source of pride
and indisputable authority. Still, We cannot close our eyes to the undeniable fact that the provision of
law now under scrutiny has no local origin and orientation; it is purely American, factually taken
bodily from American law when the Philippines was under the dominating influence of statutes of the
United States Congress. It is indeed a sad commentary on the work of our own legislature of the late
1920's and 1930's that given the opportunity to break away from the old American pattern, it took no
step in that direction. Indeed, even after America made it patently clear in the Act of Congress of
September 22, 1922 that alien women marrying Americans cannot be citizens of the United States
without undergoing naturalization proceedings, our legislators still chose to adopt the previous
American law of August 10, 1855 as embodied later in Section 1994 of the Revised Statutes of
1874, Which, it is worth reiterating, was consistently and uniformly understood as conferring
American citizenship to alien women marrying Americans ipso facto, without having to submit to any
naturalization proceeding and without having to prove that they possess the special qualifications of
residence, moral character, adherence to American ideals and American constitution, provided they
show they did not suffer from any of the disqualifications enumerated in the American Naturalization
Law. Accordingly, We now hold, all previous decisions of this Court indicating otherwise
notwithstanding, that under Section 15 of Commonwealth Act 473, an alien woman marrying a
Filipino, native born or naturalized, becomes ipso facto a Filipina provided she is not disqualified to
be a citizen of the Philippines under Section 4 of the same law. Likewise, an alien woman married to
an alien who is subsequently naturalized here follows the Philippine citizenship of her husband the
moment he takes his oath as Filipino citizen, provided that she does not suffer from any of the
disqualifications under said Section 4.

As under any other law rich in benefits for those coming under it, doubtless there will be instances
where unscrupulous persons will attempt to take advantage of this provision of law by entering into
fake and fictitious marriages or mala fide matrimonies. We cannot as a matter of law hold that just
because of these possibilities, the construction of the provision should be otherwise than as dictated
inexorably by more ponderous relevant considerations, legal, juridical and practical. There can
always be means of discovering such undesirable practice and every case can be dealt with
accordingly as it arises.

III.

The third aspect of this case requires necessarily a re-examination of the ruling of this Court in
Burca, supra, regarding the need of judicial naturalization proceedings before the alien wife of a
Filipino may herself be considered or deemed a Filipino. If this case which, as already noted, was
submitted for decision in 1964 yet, had only been decided earlier, before Go Im Ty, the foregoing
discussions would have been sufficient to dispose of it. The Court could have held that despite her
apparent lack of qualifications, her marriage to her co-petitioner made her a Filipina, without her
undergoing any naturalization proceedings, provided she could sustain, her claim that she is not
disqualified under Section 4 of the law. But as things stand now, with the Burca ruling, the question
We have still to decide is, may she be deemed a Filipina without submitting to a naturalization
proceeding?

Naturally, if Burca is to be followed, it is clear that the answer to this question must necessarily be in
the affirmative. As already stated, however, the decision in Burca has not yet become final because
there is still pending with Us a motion for its reconsideration which vigorously submits grounds
worthy of serious consideration by this Court. On this account, and for the reasons expounded
earlier in this opinion, this case is as good an occasion as any other to re-examine the issue.

In the said decision, Justice Sanchez held for the Court:

We accordingly rule that: (1) An alien woman married to a Filipino who desires to be
a citizen of this country must apply therefore by filing a petition for citizenship reciting
that she possesses all the qualifications set forth in Section 2 and none of the
disqualifications under Section 4, both of the Revised Naturalization Law; (2) Said
petition must be filed in the Court of First Instance where petitioner has resided at
least one year immediately preceding the filing of the petition; and (3) Any action by
any other office, agency, board or official, administrative or otherwise — other than
the judgment of a competent court of justice — certifying or declaring that an alien
wife of the Filipino citizen is also a Filipino citizen, is hereby declared null and void.
3. We treat the present petition as one for naturalization. Or, in the words of law, a
"petition for citizenship". This is as it should be. Because a reading of the petition will
reveal at once that efforts were made to set forth therein, and to prove afterwards,
compliance with Sections 2 and 4 of the Revised Naturalization law. The trial court
itself apparently considered the petition as one for naturalization, and, in fact,
declared petitioner "a citizen of the Philippines."

In other words, under this holding, in order for an alien woman marrying a Filipino to be vested with
Filipino citizenship, it is not enough that she possesses the qualifications prescribed by Section 2 of
the law and none of the disqualifications enumerated in its Section 4. Over and above all these, she
has to pass thru the whole process of judicial naturalization apparently from declaration of intention
to oathtaking, before she can become a Filipina. In plain words, her marriage to a Filipino is
absolutely of no consequence to her nationality vis-a-vis that of her Filipino husband; she remains to
be the national of the country to which she owed allegiance before her marriage, and if she desires
to be of one nationality with her husband, she has to wait for the same time that any other applicant
for naturalization needs to complete, the required period of ten year residence, gain the knowledge
of English or Spanish and one of the principle local languages, make her children study in Filipino
schools, acquire real property or engage in some lawful occupation of her own independently of her
husband, file her declaration of intention and after one year her application for naturalization, with
the affidavits of two credible witnesses of her good moral character and other qualifications, etc.,
etc., until a decision is ordered in her favor, after which, she has to undergo the two years of
probation, and only then, but not before she takes her oath as citizen, will she begin to be
considered and deemed to be a citizen of the Philippines. Briefly, she can become a Filipino citizen
only by judicial declaration.

Such being the import of the Court's ruling, and it being quite obvious, on the other hand, upon a
cursory reading of the provision, in question, that the law intends by it to spell out what is the "effect
of naturalization on (the) wife and children" of an alien, as plainly indicated by its title, and inasmuch
as the language of the provision itself clearly conveys the thought that some effect beneficial to the
wife is intended by it, rather than that she is not in any manner to be benefited thereby, it behooves
Us to take a second hard look at the ruling, if only to see whether or not the Court might have
overlooked any relevant consideration warranting a conclusion different from that complained
therein. It is undeniable that the issue before Us is of grave importance, considering its
consequences upon tens of thousands of persons affected by the ruling therein made by the Court,
and surely, it is for Us to avoid, whenever possible, that Our decision in any case should produce
any adverse effect upon them not contemplated either by the law or by the national policy it seeks to
endorse.

AMICI CURIAE in the Burca case, respectable and impressive by their number and standing in the
Bar and well known for their reputation for intellectual integrity, legal acumen and incisive and
comprehensive resourcefulness in research, truly evident in the quality of the memorandum they
have submitted in said case, invite Our attention to the impact of the decision therein thus:

The doctrine announced by this Honorable Court for the first time in the present case
-- that an alien woman who marries a Philippine citizen not only does not ipso
facto herself become a citizen but can acquire such citizenship only through ordinary
naturalization proceedings under the Revised Naturalization Law, and that all
administrative actions "certifying or declaring such woman to be a Philippine citizen
are null and void" — has consequences that reach far beyond the confines of the
present case. Considerably more people are affected, and affected deeply, than
simply Mrs. Zita N. Burca. The newspapers report that as many as 15 thousand
women married to Philippine citizens are affected by this decision of the Court.
These are women of many and diverse nationalities, including Chinese, Spanish,
British, American, Columbian, Finnish, Japanese, Chilean, and so on. These
members of the community, some of whom have been married to citizens for two or
three decades, have all exercised rights and privileges reserved by law to Philippine
citizens. They will have acquired, separately or in conjugal partnership with their
citizen husbands, real property, and they will have sold and transferred such
property. Many of these women may be in professions membership in which is
limited to citizens. Others are doubtless stockholders or officers or employees in
companies engaged in business activities for which a certain percentage of Filipino
equity content is prescribed by law. All these married women are now faced with
possible divestment of personal status and of rights acquired and privileges
exercised in reliance, in complete good faith, upon a reading of the law that has been
accepted as correct for more than two decades by the very agencies of government
charged with the administration of that law. We must respectfully suggest that judicial
doctrines which would visit such comprehensive and far-reaching injury upon the
wives and mothers of Philippine citizens deserve intensive scrutiny and
reexamination.

To be sure, this appeal can be no less than what this Court attended to in Gan Tsitung vs. Republic,
G.R. No. L-20819, Feb. 21, 1967, 19 SCRA 401 — when Chief Justice Concepcion observed:

The Court realizes, however, that the rulings in the Barretto and Delgado cases —
although referring to situations the equities of which are not identical to those
obtaining in the case at bar — may have contributed materially to the irregularities
committed therein and in other analogous cases, and induced the parties concerned
to believe, although erroneously, that the procedure followed was valid under the
law.

Accordingly, and in view of the implications of the issue under consideration, the
Solicitor General was required, not only, to comment thereon, but, also, to state "how
many cases there are, like the one at bar, in which certificates of naturalization have
been issued after notice of the filing of the petition for naturalization had been
published in the Official Gazette only once, within the periods (a) from January 28,
1950" (when the decision in Delgado v. Republic was promulgated) "to May 29,
1957" (when the Ong Son Cui was decided) "and (b) from May 29, 1957 to
November 29, 1965" (when the decision in the present case was rendered).

After mature deliberation, and in the light of the reasons adduced in appellant's
motion for reconsideration and in the reply thereto of the Government, as well as of
the data contained in the latter, the Court holds that the doctrine laid down in the Ong
Son Cui case shall apply and affect the validity of certificates of naturalization
issued after, not on or before May 29, 1957.

Here We are met again by the same problem. In Gan Tsitung, the Court had to expressly enjoin the
prospective application of its construction of the law made in a previous decision,   which had
24

already become final, to serve the ends of justice and equity. In the case at bar, We do not have to
go that far. As already observed, the decision in Burca still under reconsideration, while the ruling
in Lee Suan Ay, Lo San Tuang, Choy King Tee and others that followed them have at the most
become the law of the case only for the parties thereto. If there are good grounds therefor, all We
have to do now is to reexamine the said rulings and clarify or modify them.

For ready reference, We requote Section 15:


Sec. 15. Effect of the naturalization on wife and children. — Any woman who is now
or may hereafter be married to a citizen of the Philippines, and who might herself be
lawfully naturalized shall be deemed a citizen of the Philippines.

Minor children of persons naturalized under this law who have been born in the
Philippines shall be considered citizens thereof.

A foreign-born minor child, if dwelling in the Philippines at the time of naturalization of


the parents, shall automatically become a Philippine citizen, and a foreign-born minor
child, who is not in the Philippines at the time the parent is naturalized, shall be
deemed a Philippine citizen only during his minority, unless he begins to reside
permanently in the Philippines when still a minor, in which case, he will continue to
be a Philippine citizen even after becoming of age.

A child born outside of the Philippines after the naturalization of his parent, shall be
considered a Philippine citizen, unless within one year after reaching the age of
majority, he fails to register himself as a Philippine citizen at the American Consulate
of the country where he resides, and to take the necessary oath of allegiance.

It is obvious that the main subject-matter and purpose of the statute, the Revised Naturalization Law
or Commonwealth Act 473, as a whole, is to establish a complete procedure for the judicial
conferment of the status of citizenship upon qualified aliens. After laying out such a procedure,
remarkable for its elaborate and careful inclusion of all safeguards against the possibility of any
undesirable persons becoming a part of our citizenry, it carefully but categorically states the
consequence of the naturalization of an alien undergoing such procedure it prescribes upon the
members of his immediate family, his wife and children,   and, to that end, in no uncertain terms it
25

ordains that: (a) all his minor children who have been born in the Philippines shall be "considered
citizens" also; (b) all such minor children, if born outside the Philippines but dwelling here at the time
of such naturalization "shall automatically become" Filipinos also, but those not born in the
Philippines and not in the Philippines at the time of such naturalization, are also redeemed citizens
of this country provided that they shall lose said status if they transfer their permanent residence to a
foreign country before becoming of age; (c) all such minor children, if born outside of the Philippines
after such naturalization, shall also be "considered" Filipino citizens, unless they expatriate
themselves by failing to register as Filipinos at the Philippine (American) Consulate of the country
where they reside and take the necessary oath of allegiance; and (d) as to the wife, she "shall be
deemed a citizen of the Philippines" if she is one "who might herself be lawfully naturalized".  26

No doubt whatever is entertained, so Burca holds very correctly, as to the point that the minor
children, falling within the conditions of place and time of birth and residence prescribed in the
provision, are vested with Philippine citizenship directly by legislative fiat or by force of the law itself
and without the need for any judicial proceeding or declaration. (At p. 192, 19 SCRA). Indeed, the
language of the provision, is not susceptible of any other interpretation. But it is claimed that the
same expression "shall be deemed a citizen of the Philippines" in reference to the wife, does not
necessarily connote the vesting of citizenship status upon her by legislative fiat because the
antecedent phrase requiring that she must be one "who might herself be lawfully naturalized" implies
that such status is intended to attach only after she has undergone the whole process of judicial
naturalization required of any person desiring to become a Filipino. Stated otherwise, the ruling
in Burca is that while Section 15 envisages and intends legislative naturalization as to the minor
children, the same section deliberately treats the wife differently and leaves her out for the ordinary
judicial naturalization.
Of course, it goes without saying that it is perfectly within the constitutional authority of the Congress
of the Philippines to confer or vest citizenship status by legislative fiat. (U.S. v. Wong Kim Ark, 169
U.S. 649, 42 L ed. 890 [1898]; See, 1 Tañada & Carreon, Political Law of the Philippines 152 [1961
ed.]) In fact, it has done so for particular individuals, like two foreign religious prelates,   hence there
27

is no reason it cannot do it for classes or groups of persons under general conditions applicable to
all of the members of such class or group, like women who marry Filipinos, whether native-born or
naturalized. The issue before Us in this case is whether or not the legislature hag done so in the
disputed provisions of Section 15 of the Naturalization Law. And Dr. Vicente G. Sinco, one of the
most respect authorities on political law in the Philippines   observes in this connection thus: "A
28

special form of naturalization is often observed by some states with respect to women. Thus in the
Philippines a foreign woman married to a Filipino citizen becomes ipso facto naturalized, if she
belongs to any of the classes who may apply for naturalization under the Philippine Laws." (Sinco,
Phil. Political Law 498-499 [10th ed. 1954]; emphasis ours; this comment is substantially reiterated
in the 1962 edition, citing Ly Giok Ha and Ricardo Cua, supra.)

More importantly, it may be stated, at this juncture, that in construing the provision of the United
States statutes from which our law has been copied,  a the American courts have held that the alien
28

wife does not acquire American citizenship by choice but by operation of law. "In the Revised
Statutes the words "and taken" are omitted. The effect of this statute is that every alien woman who
marries a citizen of the United States becomes perforce a citizen herself, without the formality of
naturalization, and regardless of her wish in that respect." (USCA 8, p. 601 [1970 ed.], citing
Mackenzie v. Hare, 1913, 134 P. 713, 165 Cal. 766, affirmed 36 S. Ct. 106, 239 U.S. 299, 60 L ed.
297.) .

We need not recount here again how this provision in question was first enacted as paragraph (a) of
Section 13, by way of an insertion into Act 2927 by Act 3448 of November 30, 1928, and that, in
turn, and paragraph was copied verbatim from Section 1994 of the Revised Statutes of the United
States, which by that time already had a long accepted construction among the courts and
administrative authorities in that country holding that under such provision an alien woman who
married a citizen became, upon such marriage, likewise a citizen by force of law and as a
consequence of the marriage itself without having to undergo any naturalization proceedings,
provided that, it could be shown that at the time of such marriage, she was not disqualified to be
naturalized under the laws then in force. To repeat the discussion We already made of these
undeniable facts would unnecessarily make this decision doubly extensive. The only point which
might be reiterated for emphasis at this juncture is that whereas in the United States, the American
Congress, recognizing the construction, of Section 1994 of the Revised Statutes to be as stated
above, and finding it desirable to avoid the effects of such construction, approved the Act of
September 22, 1922 Explicitly requiring all such alien wives to submit to judicial naturalization albeit
under more liberal terms than those for other applicants for citizenship, on the other hand, the
Philippine Legislature, instead of following suit and adopting such a requirement, enacted Act 3448
on November 30, 1928 which copied verbatim the aforementioned Section 1994 of the Revised
Statutes, thereby indicating its preference to adopt the latter law and its settled construction rather
than the reform introduced by the Act of 1922.

Obviously, these considerations leave Us no choice. Much as this Court may feel that as the United
States herself has evidently found it to be an improvement of her national policy vis-a-vis the alien
wives of her citizens to discontinue their automatic incorporation into the body of her citizenry without
passing through the judicial scrutiny of a naturalization proceeding, as it used to be before 1922, it
seems but proper, without evidencing any bit of colonial mentality, that as a developing country, the
Philippines adopt a similar policy, unfortunately, the manner in which our own legislature has
enacted our laws on the subject, as recounted above, provides no basis for Us to construe said law
along the line of the 1922 modification of the American Law. For Us to do so would be to indulge in
judicial legislation which it is not institutionally permissible for this Court to do. Worse, this court
would be going precisely against the grain of the implicit Legislative intent.

There is at least one decision of this Court before Burca wherein it seems it is quite clearly implied
that this Court is of the view that under Section 16 of the Naturalization Law, the widow and children
of an applicant for naturalization who dies during the proceedings do not have to submit themselves
to another naturalization proceeding in order to avail of the benefits of the proceedings involving the
husband. Section 16 provides: .

SEC. 16. Right of widow and children of petitioners who have died. — In case a
petitioner should die before the final decision has been rendered, his widow and
minor children may continue the proceedings. The decision rendered in the case
shall, so far as the widow and minor children are concerned, produce the same legal
effect as if it had been rendered during the life of the petitioner.

In Tan Lin v. Republic, G.R. No. L-13706, May 31, 1961, 2 SCRA 383, this Court held:

Invoking the above provisions in their favor, petitioners-appellants argue (1) that
under said Sec. 16, the widow and minor children are allowed to continue the same
proceedings and are not substituted for the original petitioner; (2) that the
qualifications of the original petitioner remain to be in issue and not those of the
widow and minor children, and (3) that said Section 16 applies whether the petitioner
dies before or after final decision is rendered, but before the judgment becomes
executory.

There is force in the first and second arguments. Even the second sentence of said
Section 16 contemplate the fact that the qualifications of the original petitioner
remains the subject of inquiry, for the simple reason that it states that "The decision
rendered in the case shall, so far as the widow and minor children are concerned,
produce the same legal effect as if it had been rendered during the life of the
petitioner." This phraseology emphasizes the intent of the law to continue the
proceedings with the deceased as the theoretical petitioner, for if it were otherwise, it
would have been unnecessary to consider the decision rendered, as far as it affected
the widow and the minor children.

xxx xxx xxx

The Chua Chian case (supra), cited by the appellee, declared that a dead person
can not be bound to do things stipulated in the oath of allegiance, because an oath is
a personal matter. Therein, the widow prayed that she be allowed to take the oath of
allegiance for the deceased. In the case at bar, petitioner Tan Lin merely asked that
she be allowed to take the oath of allegiance and the proper certificate of
naturalization, once the naturalization proceedings of her deceased husband, shall
have been completed, not on behalf of the deceased but on her own behalf and of
her children, as recipients of the benefits of his naturalization. In other words, the
herein petitioner proposed to take the oath of allegiance, as a citizen of the
Philippines, by virtue of the legal provision that "any woman who is now or may
hereafter be married to a citizen of the Philippines and who might herself be lawfully
naturalized shall be deemed a citizen of the Philippines. Minor children of persons
naturalized under this law who have been born in the Philippines shall be considered
citizens thereof." (Section 15, Commonwealth Act No. 473). The decision granting
citizenship to Lee Pa and the record of the case at bar, do not show that the
petitioning widow could not have been lawfully naturalized, at the time Lee Pa filed
his petition, apart from the fact that his 9 minor children were all born in the
Philippines. (Decision, In the Matter of the Petition of Lee Pa to be admitted a citizen
of the Philippines, Civil Case No. 16287, CFI, Manila, Annex A; Record on Appeal,
pp. 8-11). The reference to Chua Chian case is, therefore, premature.

Section 16, as may be seen, is a parallel provision to Section 15. If the widow of an applicant for
naturalization as Filipino, who dies during the proceedings, is not required to go through a
naturalization preceeding, in order to be considered as a Filipino citizen hereof, it should follow that
the wife of a living Filipino cannot be denied the same privilege. This is plain common sense and
there is absolutely no evidence that the Legislature intended to treat them differently.

Additionally, We have carefully considered the arguments advanced in the motion for
reconsideration in Burca, and We see no reason to disagree with the following views of counsel: .

It is obvious that the provision itself is a legislative declaration of who may be


considered citizens of the Philippines. It is a proposition too plain to be disputed that
Congress has the power not only to prescribe the mode or manner under which
foreigners may acquire citizenship, but also the very power of conferring citizenship
by legislative fiat. (U. S. v. Wong Kim Ark, 169 U. S. 649, 42 L. Ed. 890 [1898] ; see 1
Tañada and Carreon, Political Law of the Philippines 152 [1961 ed.]) The
Constitution itself recognizes as Philippine citizens "Those who are naturalized in
accordance with law" (Section 1[5], Article IV, Philippine Constitution). Citizens by
naturalization, under this provision, include not only those who are naturalized in
accordance with legal proceedings for the acquisition of citizenship, but also those
who acquire citizenship by "derivative naturalization" or by operation of law, as, for
example, the "naturalization" of an alien wife through the naturalization of her
husband, or by marriage of an alien woman to a citizen. (See Tañada & Carreon, op.
cit. supra, at 152, 172; Velayo, Philippine Citizenship and Naturalization 2 [1965 ed.];
1 Paras, Civil Code 186 [1967 ed.]; see also 3 Hackworth, Digest of International
Law 3).

The phrase "shall be deemed a citizen of the Philippines" found in Section 14 of the
Revised Naturalization Law clearly manifests an intent to confer citizenship.
Construing a similar phrase found in the old U.S. naturalization law (Revised
Statutes, 1994), American courts have uniformly taken it to mean that upon her
marriage, the alien woman becomes by operation of law a citizen of the United
States as fully as if she had complied with all the provisions of the statutes upon the
subject of naturalization. (U.S. v. Keller, 13 F. 82; U.S. Opinions of the US Attorney
General dated June 4, 1874 [14 Op. 4021, July 20, 1909 [27 Op. 507], December 1,
1910 [28 Op. 508], Jan. 15, 1920 [32 Op. 2091 and Jan. 12, 1923 [23 398]).

The phrase "shall be deemed a citizen," in Section 1994 Revised


Statute (U.S. Comp. Stat. 1091, 1268) or as it was in the Act of 1855
(10 Stat. at L. 604, Chapt. 71, Sec. 2), "shall be deemed and taken to
be a citizens" while it may imply that the person to whom it relates
has not actually become a citizen by the ordinary means or in the
usual way, as by the judgment of a competent court, upon a proper
application and proof, yet it does not follow that such person is on
that account practically any the less a citizen. The word "deemed" is
the equivalent of "considered" or "judged," and therefore, whatever
an Act of Congress requires to be "deemed" or "taken" as true of any
person or thing must, in law, be considered as having been duly
adjudged or established concerning such person or thing, and have
force and effect accordingly. When, therefore, Congress declares
that an alien woman shall, under certain circumstances, be "deemed"
an American citizen, the effect when the contingency occurs, is
equivalent to her being naturalized directly by an Act of Congress or
in the usual mode thereby prescribed. (Van Dyne, Citizenship of the
United States 239, cited in Velayo, Philippine Citizenship and
Naturalization 146-147 [1965 ed.]; emphasis ours).

That this was likewise the intent of the Philippine legislature when it enacted the first
paragraph of Section 15 of the Revised Naturalization Law is shown by a textual
analysis of the entire statutory provision. In its entirety, Section 15 reads:

(See supra).

The phrases "shall be deemed" "shall be considered," and "shall automatically


become" as used in the above provision, are undoubtedly synonymous. The leading
idea or purpose of the provision was to confer Philippine citizenship by operation of
law upon certain classes of aliens as a legal consequence of their relationship, by
blood or by affinity, to persons who are already citizens of the Philippines. Whenever
the fact of relationship of the persons enumerated in the provision concurs with
the fact of citizenship of the person to whom they are related, the effect is for said
persons to become ipso factocitizens of the Philippines. "Ipso facto" as here used
does not mean that all alien wives and all minor children of Philippine citizens, from
the mere fact of relationship, necessarily become such citizens also. Those who do
not meet the statutory requirements do not ipso facto become citizens; they must
apply for naturalization in order to acquire such status. What it does mean, however,
is that in respect of those persons enumerated in Section 15, the relationship to a
citizen of the Philippines is the operative fact which establishes the acquisition of
Philippine citizenship by them. Necessarily, it also determines the point of time at
which such citizenship commences. Thus, under the second paragraph of Section
15, a minor child of a Filipino naturalized under the law, who was born in the
Philippines, becomes ipso facto a citizen of the Philippines from the time the fact of
relationship concurs with the fact of citizenship of his parent, and the time when the
child became a citizen does not depend upon the time that he is able to prove that
he was born in the Philippines. The child may prove some 25 years after the
naturalization of his father that he was born in the Philippines and should, therefore,
be "considered" a citizen thereof. It does not mean that he became a Philippine
citizen only at that later time. Similarly, an alien woman who married a Philippine
citizen may be able to prove only some 25 years after her marriage (perhaps,
because it was only 25 years after the marriage that her citizenship status became in
question), that she is one who might herself be lawfully naturalized." It is not
reasonable to conclude that she acquired Philippine citizenship only after she had
proven that she "might herself be lawfully naturalized." It is not reasonable to
conclude that she acquired Philippine citizenship only after she had proven that she
"might herself be lawfully naturalized."

The point that bears emphasis in this regard is that in adopting the very phraseology
of the law, the legislature could not have intended that an alien wife should not be
deemed a Philippine citizen unless and until she proves that she might herself be
lawfully naturalized. Far from it, the law states in plain terms that she shall be
deemed a citizen of the Philippines if she is one "who might herself be lawfully
naturalized." The proviso that she must be one "who might herself be lawfully
naturalized" is not a condition precedent to the vesting or acquisition of citizenship; it
is only a condition or a state of fact necessary to establish her citizenship as
a factum probandum, i.e., as a fact established and proved in evidence. The word
"might," as used in that phrase, precisely replies that at the time of her marriage to a
Philippine citizen, the alien woman "had (the) power" to become such a citizen
herself under the laws then in force. (Owen v. Kelly, 6 DC 191 [1867], aff'd Kelly v.
Owen, 76 US 496, 19 L ed 283 [1869). That she establishes such power long after
her marriage does not alter the fact that at her marriage, she became a citizen.

(This Court has held) that "an alien wife of a Filipino citizen may not acquire the
status of a citizen of the Philippines unless there is proof that she herself may be
lawfully naturalized" (Decision, pp. 3-4). Under this view, the "acquisition" of
citizenship by the alien wife depends on her having proven her qualifications for
citizenship, that is, she is not a citizen unless and until she proves that she may
herself be lawfully naturalized. It is clear from the words of the law that the proviso
does not mean that she must first prove that she "might herself be lawfully
naturalized" before she shall be deemed (by Congress, not by the courts) a citizen.
Even the "uniform" decisions cited by this Court (at fn. 2) to support its holding did
not rule that the alien wife becomes a citizen only after she has proven her
qualifications for citizenship. What those decisions ruled was that the alien wives in
those cases failed to prove their qualifications and therefore they failed to establish
their claim to citizenship. Thus in Ly Giok Ha v. Galang, 101 Phil. 459 [l957], the case
was remanded to the lower court for determination of whether petitioner, whose claim
to citizenship by marriage to a Filipino was disputed by the Government, "might
herself be lawfully naturalized," for the purpose of " proving her alleged change of
political status from alien to citizen" (at 464). In Cua v. Board, 101 Phil. 521 [1957],
the alien wife who was being deported, claimed she was a Philippine citizen by
marriage to a Filipino. This Court finding that there was no proof that she was not
disqualified under Section 4 of the Revised Naturalization Law, ruled that: "No such
evidence appearing on record, the claim of assumption of Philippine citizenship by
Tijoe Wu Suan, upon her marriage to petitioner, is untenable." (at 523) It will be
observed that in these decisions cited by this Court, the lack of proof that the alien
wives "might (themselves) be lawfully naturalized" did not necessarily imply that they
did not become, in truth and in fact, citizens upon their marriage to Filipinos. What
the decisions merely held was that these wives failed to establish their claim to that
status as a proven fact.

In all instances where citizenship is conferred by operation of law, the time when
citizenship is conferred should not be confused with the time when citizenship status
is established as a proven fact. Thus, even a natural-born citizen of the Philippines,
whose citizenship status is put in issue in any proceeding would be required to prove,
for instance, that his father is a citizen of the Philippines in order to factually establish
his claim to citizenship.* His citizenship status commences from the time of birth,
although his claim thereto is established as a fact only at a subsequent time.
Likewise, an alien woman who might herself be lawfully naturalized becomes a
Philippine citizen at the time of her marriage to a Filipino husband, not at the time
she is able to establish that status as a proven fact by showing that she might herself
be lawfully naturalized. Indeed, there is no difference between a statutory declaration
that a person is deemed a citizen of the Philippines provided his father is such
citizen from a declaration that an alien woman married to a Filipino citizen of the
Philippines provided she might herself be lawfully naturalized. Both become citizens
by operation of law; the former becomes a citizen ipso facto upon birth; the later ipso
facto upon marriage.

It is true that unless and until the alien wife proves that she might herself be lawfully
naturalized, it cannot be said that she has established her status as a proven fact.
But neither can it be said that on that account, she did not become a citizen of the
Philippines. If her citizenship status is not questioned in any legal proceeding, she
obviously has no obligation to establish her status as a fact. In such a case, the
presumption of law should be that she is what she claims to be. (U.S. v. Roxas, 5
Phil. 375 [1905]; Hilado v. Assad, 51 O.G. 4527 [1955]). There is a presumption that
a representation shown to have been made is true. (Aetna Indemnity Co. v. George
A. Fuller, Co., 73 A. 738, 74 A. 369, 111 ME. 321).

The question that keeps bouncing back as a consequence of the foregoing views is, what substitute
is them for naturalization proceedings to enable the alien wife of a Philippine citizen to have the
matter of her own citizenship settled and established so that she may not have to be called upon to
prove it everytime she has to perform an act or enter in to a transaction or business or exercise a
right reserved only to Filipinos? The ready answer to such question is that as the laws of our
country, both substantive and procedural, stand today, there is no such procedure, but such paucity
is no proof that the citizenship under discussion is not vested as of the date of marriage or the
husband's acquisition of citizenship, as the case may be, for the truth is that the same situation
objections even as to native-born Filipinos. Everytime the citizenship of a person is material or
indispensable in a judicial or administrative case, whatever the corresponding court or administrative
authority decides therein as to such citizenship is generally not considered as res adjudicata, hence
it has to be threshed out again and again as the occasion may demand. This, as We view it, is the
sense in which Justice Dizon referred to "appropriate proceeding" in Brito v. Commissioner, supra.
Indeed, only the good sense and judgment of those subsequently inquiring into the matter may make
the effort easier or simpler for the persons concerned by relying somehow on the antecedent official
findings, even if these are not really binding.

It may not be amiss to suggest, however, that in order to have a good starting point and so that the
most immediate relevant public records may be kept in order, the following observations in Opinion
No. 38, series of 1958, of then Acting Secretary of Justice Jesus G. Barrera, may be considered as
the most appropriate initial step by the interested parties:

Regarding the steps that should be taken by an alien woman married to a Filipino
citizen in order to acquire Philippine citizenship, the procedure followed in the Bureau
of Immigration is as follows: The alien woman must file a petition for the cancellation
of her alien certificate of registration alleging, among other things, that she is married
to a Filipino, citizen and that she is not disqualified from acquiring her husband's
citizenship pursuant to section 4 of Commonwealth Act No. 473, as amended. Upon
the filing of said petition, which should be accompanied or supported by the joint
affidavit of the petitioner and her Filipino husband to the effect that the petitioner
does not belong to any of the groups disqualified by the cited section from becoming
naturalized Filipino citizen (please see attached CEB Form 1), the Bureau of
Immigration conducts an investigation and thereafter promulgates its order or
decision granting or denying the petition.

Once the Commissioner of Immigration cancels the subject's registration as an alien, there will
probably be less difficulty in establishing her Filipino citizenship in any other proceeding, depending
naturally on the substance and vigor of the opposition.
Before closing, it is perhaps best to clarify that this third issue We have passed upon was not
touched by the trial court, but as the point is decisive in this case, the Court prefers that the matter
be settled once and for all now.

IN VIEW OF ALL THE FOREGOING, the judgment of the Court a quo dismissing appellants' petition
for injunction is hereby reversed and the Commissioner of Immigration and/or his authorized
representative is permanently enjoined from causing the arrest and deportation and the confiscation
of the bond of appellant Lau Yuen Yeung, who is hereby declared to have become a Filipino citizen
from and by virtue of her marriage to her co-appellant Moy Ya Lim Yao alias Edilberto Aguinaldo
Lim, a Filipino citizen on January 25, 1962. No costs.

G.R. No. 104654 June 6, 1994

REPUBLIC OF THE PHILIPPINES, petitioner, 


vs.
HON. ROSALIO G. DE LA ROSA, PRESIDING JUDGE OF THE REGIONAL TRIAL COURT,
BRANCH 28, MANILA and JUAN G. FRIVALDO, respondents.

G.R. No. 105715 June 6, 1994

RAUL R. LEE, petitioner, 
vs.
COMMISSION ON ELECTIONS and JUAN G. FRIVALDO, respondents.

G.R. No. 105735 June 6, 1994

RAUL R. LEE, petitioner, 
vs.
COMMISSION ON ELECTIONS and JUAN G. FRIVALDO, respondents.

The Solicitor General for petitioner in G.R. No. 104654.

Yolando F. Lim counsel for private respondent.

QUIASON, J.:

In Frivaldo v. Commission on Elections, 174 SCRA 245 (1989), this Court declared private
respondent, Juan G. Frivaldo, an alien and therefore disqualified from serving as Governor of the
Province of Sorsogon.

Once more, the citizenship of private respondent is put in issue in


these petitions docketed as G.R. No.104654 and G.R. No. 105715 and G.R. No. 105735. The
petitions were consolidated since they principally involve the same issues and parties.
I

G.R. No. 104654

This is a petition for certiorari under Rule 45 of the Revised Rules of Court in relation to R.A. No.
5440 and Section 25 of the Interim Rules, filed by the Republic of the Philippines: (1) to annul the
Decision dated February 27, 1992 of the Regional Trial Court, Branch 28, Manila, in SP Proc. No.
91-58645, which re-admitted private respondent as a Filipino citizen under the Revised
Naturalization Law (C.A. No. 63 as amended by C.A. No. 473); and (2) to nullify the oath of
allegiance taken by private respondent on February 27, 1992.

On September 20, 1991, petitioner filed a petition for naturalization captioned: "In the Matter of
Petition of Juan G. Frivaldo to be Re-admitted as a Citizen of the Philippines under Commonwealth
Act No. 63" (Rollo, pp. 17-23).

In an Order dated October 7, 1991 respondent Judge set the petition for hearing on March 16, 1992,
and directed the publication of the said order and petition in the Official Gazette and a newspaper of
general circulation, for three consecutive weeks, the last publication of which should be at least six
months before the said date of hearing. The order further required the posting of a copy thereof and
the petition in a conspicuous place in the Office of the Clerk of Court of the Regional Trial Court,
Manila (Rollo, pp. 24-26).

On January 14, 1992, private respondent filed a "Motion to Set Hearing Ahead of Schedule," where
he manifested his intention to run for public office in the May 1992 elections. He alleged that the
deadline for filing the certificate of candidacy was March 15, one day before the scheduled hearing.
He asked that the hearing set on March 16 be cancelled and be moved to January 24 (Rollo, pp. 27-
28).

The motion was granted in an Order dated January 24, 1992, wherein the hearing of the petition was
moved to February 21, 1992. The said order was not published nor a copy thereof posted.

On February 21, the hearing proceeded with private respondent as the sole witness. He submitted
the following documentary evidence: (1) Affidavit of Publication of the Order dated October 7, 1991
issued by the publisher of The Philippine Star (Exh. "A"); (2) Certificate of Publication of the order
issued
by the National Printing Office (Exh. "B"); (3) Notice of Hearing of Petition (Exh. "B-1"); (4)
Photocopy of a Citation issued by the National Press Club with private respondent’s picture (Exhs.
"C" and "C-2"); (5) Certificate of Appreciation issued by the Rotary Club of Davao (Exh. "D"); (6)
Photocopy
of a Plaque of Appreciation issued by the Republican College, Quezon City (Exh. "E"); (7)
Photocopy of a Plaque of Appreciation issued by the Davao-Bicol Association (Exh. "F"); (8)
Certification issued by the Records Management and Archives Office that the record of birth of
private respondent was not on file (Exh. "G"); and (8) Certificate of Naturalization issued by the
United States District Court (Exh. "H").

Six days later, on February 27, respondent Judge rendered the assailed Decision, disposing as
follows:

WHEREFORE, the petition is GRANTED. Petitioner JUAN G. FRIVALDO, is re-


admitted as a citizen of the Republic of the Philippines by naturalization, thereby
vesting upon him, all the rights and privileges of a natural born Filipino citizen (Rollo,
p. 33).
On the same day, private respondent was allowed to take his oath of allegiance before respondent
Judge (Rollo, p. 34).

On March 16, a "Motion for Leave of Court to Intervene and to Admit Motion for Reconsideration"
was filed by Quiterio H. Hermo. He alleged that the proceedings were tainted with jurisdictional
defects, and prayed for a new trial to conform with the requirements of the Naturalization Law.

After receiving a copy of the Decision on March 18, 1992, the Solicitor General interposed a timely
appeal directly with the Supreme Court.

G.R. No. 105715

This is a petition for certiorari, mandamus with injunction under Rule 65 of the Revised Rules of


Court in relation to Section 5(2) of Article VIII of the Constitution with prayer for temporary restraining
order filed by Raul R. Lee against the Commission on Elections (COMELEC) and private
respondent, to annul the en banc Resolution of the COMELEC, which dismissed his petition
docketed as SPC Case No. 92-273. The said petition sought to annul the proclamation of private
respondent as Governor-elect of the Province of Sorsogon.

Petitioner was the official candidate of the Laban ng Demokratikong Pilipino (LDP) for the position of
governor of the Province of Sorsogon in the May 1992 elections. Private respondent was the official
candidate of the Lakas-National Union of Christian Democrats (Lakas-NUCD) for the same position.

Private respondent was proclaimed winner on May 22, 1992.

On June 1, petitioner filed a petition with the COMELEC to annul the proclamation of private
respondent as Governor-elect of the Province of Sorsogon on the grounds: (1) that the proceedings
and composition of the Provincial Board of Canvassers were not in accordance with law; (2) that
private respondent is an alien, whose grant of Philippine citizenship is being questioned by the State
in G.R. No. 104654; and (3) that private respondent is not a duly registered voter. Petitioner further
prayed that the votes case in favor of private respondent be considered as stray votes, and that he,
on the basis of the remaining valid votes cast, be proclaimed winner.

On June 10, the COMELEC issued the questioned en banc resolution which dismissed the petition
for having been filed out of time, citing Section 19 of R.A. No. 7166. Said section provides that the
period to appeal a ruling of the board of canvassers on questions affecting its composition or
proceedings was three days.

In this petition, petitioner argues that the COMELEC acted with grave abuse of discretion when it
ignored the fundamental issue of private respondent’s disqualification in the guise of technicality.

Petitioner claims that the inclusion of private respondent’s name in the list of registered voters in Sta.
Magdalena, Sorsogon was invalid because at the time he registered as a voter in 1987, he was as
American citizen.

Petitioner further claims that the grant of Filipino citizenship to private respondent is not yet
conclusive because the case is still on appeal before us.

Petitioner prays for: (1) the annulment of private respondent’s proclamation as Governor of the
Province of Sorsogon; (2) the deletion of private respondent’s name from the list of candidates for
the position of governor; (3) the proclamation of the governor-elect based on the remaining votes,
after the exclusion of the votes for private respondent; (4) the issuance of a temporary restraining
order to enjoin private respondent from taking his oath and assuming office; and (5) the issuance of
a writ of mandamus to compel the COMELEC to resolve the pending disqualification case docketed
as SPA Case No. 92-016, against private respondent.

G.R. No. 105735

This is a petition for mandamus under Rule 65 of the Revised Rules of Court in relation to Section
5(2) of Article VIII of the Constitution, with prayer for temporary restraining order. The parties herein
are identical with the parties in G.R. No. 105715.

In substance, petitioner prays for the COMELEC’s immediate resolution of SPA Case No. 92-016,
which is a petition for the cancellation of private respondent’s certificate of candidacy filed on March
23, 1992 by Quiterio H. Hermo, the intervenor in G.R. No. 104654 (Rollo, p. 18).

The petition for cancellation alleged: (1) that private respondent is an American citizen, and therefore
ineligible to run as candidate for the position of governor of the Province of Sorsogon; (2) that the
trial court’s decision
re-admitting private respondent as a Filipino citizen was fraught with legal infirmities rendering it null
and void; (3) that assuming the decision to be valid, private respondent’s oath of allegiance, which
was taken on the same day the questioned decision was promulgated, violated Republic Act No.
530, which provides for a two-year waiting period before the oath of allegiance can be taken by the
applicant; and (4) that the hearing of the petition on February 27, 1992, was held less than four
months from the date of the last publication of the order and petition. The petition prayed for the
cancellation of private respondent’s certificate of candidacy and the deletion of his name from the list
of registered voters in Sta. Magdalena, Sorsogon.

In his answer to the petition for cancellation, private respondent denied the allegations therein and
averred: (1) that Quiterio H. Hermo, not being a candidate for the same office for which private
respondent was aspiring, had no standing to file the petition; (2) that the decision re-admitting him to
Philippine citizenship was presumed to be valid; and (3) that no case had been filed to exclude his
name as a registered voter.

Raul R. Lee intervened in the petition for cancellation of private respondent’s certificate of candidacy
(Rollo, p. 37.).

On May 13, 1992, said intervenor urged the COMELEC to decide the petition for cancellation, citing
Section 78 of the Omnibus Election Code, which provides that all petitions on matters involving the
cancellation of a certificate of candidacy must be decided "not later than fifteen days before
election," and the case of Alonto v. Commission on Election, 22 SCRA 878 (1968), which ruled that
all pre-proclamation controversies should be summarily decided (Rollo,
p. 50).

The COMELEC concedes that private respondent has not yet reacquired his Filipino citizenship
because the decision granting him the same is not yet final and executory (Rollo, p. 63). However, it
submits that the issue of disqualification of a candidate is not among the grounds allowed in a
pre-proclamation controversy, like SPC Case No. 92-273. Moreover, the said petition was filed out of
time.

The COMELEC contends that the preparation for the elections occupied much of its time, thus its
failure to immediately resolve SPA Case No. 92-016. It argues that under Section 5 of Rule 25 of the
COMELEC Rules of Procedure, it is excused from deciding a disqualification case within the period
provided by law for reasons beyond its control. It also assumed that the same action was
subsequently abandoned by petitioner when he filed before it a petition for quo warranto docketed as
EPC No. 92-35. The quo warranto proceedings sought private respondent’s disqualification because
of his American citizenship.

II

G.R. No. 104654

We shall first resolve the issue concerning private respondent’s citizenship.

In his comment to the State’s appeal of the decision granting him Philippine citizenship in G.R. No.
104654, private respondent alleges that the precarious political atmosphere in the country during
Martial Law compelled him to seek political asylum in the United States, and eventually to renounce
his Philippine citizenship.

He claims that his petition for naturalization was his only available remedy for his reacquisition of
Philippine citizenship. He tried to reacquire his Philippine citizenship through repatriation and direct
act of Congress. However, he was later informed that repatriation proceedings were limited to army
deserters or Filipino women who had lost their citizenship by reason of their marriage to foreigners
(Rollo, pp. 49-50). His request to Congress for sponsorship of a bill allowing him to reacquire his
Philippine citizenship failed to materialize, notwithstanding the endorsement of several members of
the House of Representatives in his favor (Rollo, p. 51). He attributed this to the maneuvers of his
political rivals.

He also claims that the re-scheduling of the hearing of the petition to an earlier date, without
publication, was made without objection from the Office of the Solicitor General. He makes mention
that on the date of the hearing, the court was jam-packed.

It is private respondent’s posture that there was substantial compliance with the law and that the
public was well-informed of his petition for naturalization due to the publicity given by the media.

Anent the issue of the mandatory two-year waiting period prior to the taking of the oath of allegiance,
private respondent theorizes that the rationale of the law imposing the waiting period is to grant the
public an opportunity to investigate the background of the applicant and to oppose the grant of
Philippine citizenship if there is basis to do so. In his case, private respondent alleges that such
requirement may be dispensed with, claiming that his life, both private and public, was well-known.
Private respondent cites his achievement as a freedom fighter and a former Governor of the
Province of Sorsogon for six terms.

The appeal of the Solicitor General in behalf of the Republic of the Philippines is meritorious. The
naturalization proceedings in SP Proc. No. 91-58645 was full of procedural flaws, rendering the
decision an anomaly.

Private respondent, having opted to reacquire Philippine citizenship thru naturalization under the
Revised Naturalization Law, is duty bound to follow the procedure prescribed by the said law. It is
not for an applicant to decide for himself and to select the requirements which he believes, even
sincerely, are applicable to his case and discard those which be believes are inconvenient or merely
of nuisance value. The law does not distinguish between an applicant who was formerly a Filipino
citizen and one who was never such a citizen. It does not provide a special procedure for the
reacquisition of Philippine citizenship by former Filipino citizens akin to the repatriation of a woman
who had lost her Philippine citizenship by reason of her marriage to an alien.

The trial court never acquired jurisdiction to hear the petition for naturalization of private respondent.
The proceedings conducted, the decision rendered and the oath of allegiance taken therein, are null
and void for failure to comply with the publication and posting requirements under the Revised
Naturalization Law.

Under Section 9 of the said law, both the petition for naturalization and the order setting it for hearing
must be published once a week for three consecutive weeks in the Official Gazette and a newspaper
of general circulation respondent cites his achievements as a freedom fighter and a former Governor
of the Province of Sorsogon for six terms.

The appeal of the Solicitor General in behalf of the Republic of


the Philippines is meritorious. The naturalization proceedings in SP Proc.
No. 91-58645 was full of procedural flaws, rendering the decision an anomaly.

Private respondent, having opted to reacquire Philippine citizenship thru naturalization under the
Revised Naturalization Law, is duty bound to follow the procedure prescribed by the said law. It is
not for an applicant to decide for himself and to select the requirements which he believes, even
sincerely, are applicable to his case and discard those which he believes are inconvenient or merely
of nuisance value. The law does not distinguish between an applicant who was formerly a Filipino
citizen and one who was never such a citizen. It does not provide a special procedure for the
reacquisition of Philippine citizenship by former Filipino citizens akin to the repatriation of a woman
who had lost her Philippine citizenship by reason of her marriage to an alien.

The trial court never acquired jurisdiction to hear the petition for naturalization of private respondent.
The proceedings conducted, the decision rendered and the oath of allegiance taken therein, are null
and void for failure to comply with the publication and posting requirements under the Revised
Naturalization Law.

Under Section 9 of the said law, both the petition for naturalization and the order setting it for hearing
must be published once a week for three consecutive weeks in the Official Gazette and a newspaper
of general circulation. Compliance therewith is jurisdictional (Po Yi Bo v. Republic, 205 SCRA 400
[1992]). Moreover, the publication and posting of the petition and the order must be in its full test for
the court to acquire jurisdiction (Sy v. Republic, 55 SCRA 724 [1974]).

The petition for naturalization lacks several allegations required by Sections 2 and 6 of the Revised
Naturalization Law, particularly: (1) that the petitioner is of good moral character; (2) that he resided
continuously in the Philippines for at least ten years; (3) that he is able to speak and write English
and any one of the principal dialects; (4) that he will reside continuously in the Philippines from the
date of the filing of the petition until his admission to Philippine citizenship; and (5) that he has filed a
declaration of intention or if he is excused from said filing, the justification therefor.

The absence of such allegations is fatal to the petition (Po Yi Bi v. Republic, 205 SCRA 400 [1992]).

Likewise, the petition is not supported by the affidavit of at least two credible persons who vouched
for the good moral character of private respondent as required by Section 7 of the Revised
Naturalization Law. Private respondent also failed to attach a copy of his certificate of arrival to the
petition as required by Section 7 of the said law.
The proceedings of the trial court was marred by the following irregularities: (1) the hearing of the
petition was set ahead of the scheduled date of hearing, without a publication of the order advancing
the date of hearing, and the petition itself; (2) the petition was heard within six months from the last
publication of the petition; (3) petitioner was allowed to take his oath of allegiance before the finality
of the judgment; and (4) petitioner took his oath of allegiance without observing the two-year waiting
period.

A decision in a petition for naturalization becomes final only after 30 days from its promulgation and,
insofar as the Solicitor General is concerned, that period is counted from the date of his receipt of
the copy of the decision (Republic v. Court of First Instance of Albay, 60 SCRA 195 [1974]).

Section 1 of R.A. No. 530 provides that no decision granting citizenship in naturalization proceedings
shall be executory until after two years from its promulgation in order to be able to observe if: (1) the
applicant has left the country; (2) the applicant has dedicated himself continuously to a lawful calling
or profession; (3) the applicant has not been convicted of any offense or violation of government
promulgated rules; and (4) the applicant has committed any act prejudicial to the interest of the
country or contrary to government announced policies.

Even discounting the provisions of R.A. No. 530, the courts cannot implement any decision granting
the petition for naturalization before its finality.

G.R. No. 105715

In view of the finding in G.R. No. 104654 that private respondent is not yet a Filipino citizen, we have
to grant the petition in G.R. No. 105715 after treating it as a petition for certiorari instead of a petition
for mandamus. Said petition assails the en banc resolution of the COMELEC, dismissing SPC Case
No. 92-273, which in turn is a petition to annul private respondent’s proclamation on three grounds:
1) that the proceedings and composition of the Provincial Board of Canvassers were not in
accordance with law; 2) that private respondent is an alien, whose grant of Filipino citizenship is
being questioned by the State in G.R. No. 104654; and 3) that private respondent is not a duly
registered voter. The COMELEC dismissed the petition on the grounds that it was filed outside the
three-day period for questioning the proceedings
and composition of the Provincial Board of Canvassers under Section 19 of R.A. No. 7166.

The COMELEC failed to resolve the more serious issue — the disqualification of private respondent
to be proclaimed Governor on grounds of lack of Filipino citizenship. In this aspect, the petition is
one for quo warranto. In Frivaldo v. Commission on Elections, 174 SCRA 245 (1989), we held that a
petition for quo warranto, questioning the respondent’s title and seeking to prevent him from holding
office as Governor for alienage, is not covered by the ten-day period for appeal prescribed in Section
253 of the Omnibus Election Code. Furthermore, we explained that "qualifications for public office
are continuing requirements and must be possessed not only at the time of appointment or election
or assumption of office but during the officer’s entire tenure; once any of the required qualification is
lost, his title may be seasonably challenged."

Petitioner’s argument, that to unseat him will frustrate the will of the electorate, is untenable. Both
the Local Government Code and the Constitution require that only Filipino citizens can run and be
elected to public office. We can only surmise that the electorate, at the time they voted for private
respondent, was of the mistaken belief that he had legally reacquired Filipino citizenship.

Petitioner in G.R. No. 105715, prays that the votes cast in favor of private respondent be considered
stray and that he, being the candidate obtaining the second highest number of votes, be declared
winner. In Labo, Jr. v. COMELEC, 176 SCRA 1 (1989), we ruled that where the candidate who
obtained the highest number of votes is later declared to be disqualified to hold the office to which he
was elected, the candidate who garnered the second highest number of votes is not entitled to be
declared winner (See also Geronimo v. Ramos, 136 SCRA 435 [1985]; Topacio v. Paredes, 23 Phil.
238 [1912]).

G.R. No. 105735

In view of the discussions of G.R. No. 104654 and G.R. No. 105715, we find the petition in G.R. No.
105735 moot and academic.

WHEREFORE, the petitions in G.R. No. 104654 and G.R. No. 105715 are both GRANTED while the
petition in G.R. No. 105735 is DISMISSED. Private respondent is declared NOT a citizen of the
Philippines and therefore DISQUALIFIED from continuing to serve as GOVERNOR of the Province
of Sorsogon. He is ordered to VACATE his office and to SURRENDER the same to the Vice-
Governor of the Province of Sorsogon once this decision becomes final and executory. No
pronouncement as to costs.

SO ORDERED.

G.R. No. L-35947 October 20, 1992

REPUBLIC OF THE PHILIPPINES, movant-appellee, 


vs.
WILLIAM LI YAO, petitioner-appellant.

ROMERO, J.:

This is an appeal from the order of the then Court of First Instance of Manila over twenty years ago, or on July 22, 1971, cancelling the
certificate of naturalization of William Li Yao as well as from the ordered dated December 29, 1971 denying Li Yao's motion for
reconsiderations.

William Li Yao, a Chinese national, filed a petition for naturalization on June 3, 1949 with the then Court of First Instance of Manila, which
petition was docketed as Case No. 8225. 1

After several hearings on the petition were held wherein the Office of the Solicitor General, in the representation of the Republic of the
Philippines appeared, the lower court rendered a decision dated October 25, 1950, the dispositive portion of which reads as follows:

IN VIEW OF ALL THE FOREGOING, the Court hereby declares William Li Yao, for all intents and purposes a
naturalized Filipino citizen, it appearing that he possesses all the qualifications to become a naturalized Filipino and
none of the disqualifications provided for by the law. However, in view of the provisions of Republic Act No. 530, this
decision shall not become final and executory until after two (2) years from its promulgation and after this Court, on
proper hearing, with the attendance of the Solicitor General or his representative, is satisfied, and so finds, that during
the intervening time the applicant herein has (1) not left the Philippines, (2) has dedicated himself continuously to a
lawful calling or profession, (3) has not been convicted of any offense and violation of the government promulgated
rules, (4) or committed any act prejudicial to the interest of the nation or contrary to any Government announce
policies. After the finding mentioned herein, this decision granting Philippine citizenship to the applicant herein shall be
registered and the oath provided by existing law shall be taken by said applicant, whereupon, and not before, he will be
entitled to all the privileges of the Filipino citizen and the certificate of naturalization shall forthwith issue in his favor by
the Clerk of this Court. 2

On November 20, 1952, acting on the petition of William Li Yao praying for the execution of the foregoing decision and that he be allowed to
take his oath of allegiance as a Filipino citizen, the lower court issued an order, the dispositive portion of which reads as follows:
WHEREFORE, it appearing that the petitioner has complied, within the two year probation period, with the provisions of
Republic Act No. 530, he is hereby allowed to take his oath of allegiance as a Filipino citizen, and Clerk of Court is
directed to issue in his favor to the corresponding certificate of naturalization. 3

About fifteen years later, or on January 5, 1968, the Republic of the Philippines, through the Solicitor General, filed a motion to cancel
William Li Yao's certificate of naturalization on the ground that it was fraudulently and illegally obtained for the following reasons:

1. At the time of the filing of the petition, the applicant was not qualified to acquire Filipino citizenship by naturalization
because:

a. He was not a person of good moral character, having had illicit amorous relationship (sic) with
several women other than his lawfully wedded wife, by whom he fathered illegitimate children (Li
Siu Liat vs. Republic, L-25356, November 25, 1967).

b. Nor had he conducted himself in an irreproachable manner in his dealings with the duly
constituted authorities:

(i) In contracting marriage, he used the name Fransisco Li Yao (Exh. "J," p.
31, rec.) without prior judicial authority to use the aforesaid first name
Fransisco, the same not appearing to be his baptismal name (Cosme Co
Tian An vs. Republic, L-1983, August 31, 1966).

(ii) He was also known and had used the name and/or alias LI CHAY TOO,
JR. before the last World War, and under which name, a trust fund was
created for him (see Decision, Court of Tax Appeals, CTA Case No. 30,
dated July 31, 1956; also Decision, Supreme Court, G.R. No. L-11861, Dec.
28, 1963).

(iii) He evaded the payment of lawful taxes due to the government by


underdeclaration of income as reflected in his income tax returns for the
years 1946-1951 (see Decision, Supreme Court, William Li Yao v. Collector
of Internal Revenue, L-11875, December 28, 1963).

(iv) He committed violations of the Constitution and Anti-Dummy laws


prohibiting aliens from acquiring real properties by employing dummies in
the formation of a private domestic corporation, which acquired the real
properties.

(v) He made it appear, falsely, in the baptismal certificate of an illegitimate


son he fathered, named William Jose Antonio, that the latter's mother is
Juanita Tan Ho Ti, his law-mother is another woman (sic). 4

William Li Yao opposed the forgoing motion on July 22, 1971. The lower court, however, without touching on all the grounds upon which the
said motion was based, relied solely on ground (iii) that William Li Yao evaded the payment of lawful taxes due the government by
underdeclaration of income as reflected in his income tax returns for the years 1946-1951. It issued an order, the dispositive portion of which
reads as follows:

WHEREFORE, the motion of the Republic of the Philippines to cancel Certificate of Naturalization No. 1139 dated
November 20, 1952 issued to the petitioner is hereby granted, and the said Certificate of Naturalization should be, as it
is hereby cancelled. Without pronouncement as to cost. 5

William Li Yao filed a motion for reconsideration on December 29, 1971, which the lower court denied. 6

On January 7, 1972, William LI Yao filed a notice of appeal to this Court, manifesting that he was appealing from the order of the lower court
dated July 22, 1971, and from the order dated December 29, 1971. 7

After the parties had filed their respective briefs, petitioner-appellant Li Yao died. 8 The case has not, however, become moot and academic
since its disposition, either way, will have grave implications for the late petitioner-appellant's wife and children.

The issue in this case is whether or not the cancellation of the certificate of naturalization of the deceased petitioner-appellant William Li Yao
made by the government through the Office of the Solicitor General is valid.

The appeal is without merit.


In his motion filed on January 5, 1968, the Solicitor General asked for the cancellation of the naturalization certificate of appellant on the
ground that it was "fraudelently and illegally obtained." This based on Section 18(a) of Com. Act No. 473, known as the Revised
Naturalization Act, which provides that a naturalization certificate may be cancelled "[i]f it is shown that said naturalization certificate was
obtained fraudelently and illegally."

It is indisputable that a certificate of naturalization may be cancelled if it is subsequently discovered that the applicant therefore obtained it by
misleading the court upon any material fact. 9 Law and jurisprudence even authorize the cancellation of a certificate of naturalization upon
grounds had conditions arising subsequent to the granting of the certificate. 10 Moreover, a naturalization proceeding is not a judicial
adversary proceeding, the decision rendered therein, not constituting res judicata as to any matter that would support a judgment cancelling
a certificate of naturalization on the ground of illegal or fraudulent procurement thereof. 11

In ordering the cancellation of the naturalization certificate previously issued to appellant, the lower court sustained the government's motion
for cancellation on the sole finding that Li Yao had committed underdeclaration of income and underpayment of income tax.

In the case entitled In the Matter of the Petition for Naturalization as Citizen of the Philippines, Lim Eng Yu vs . Republic, 12 It was held that
the concealment of applicant's income to evade payment of lawful taxes shows that his moral character is not irreproachable, thus
disqualifying him for naturalization.

Assuming arguendo, that appellant, as alleged, has fully paid or settled his tax liability under P.D. No. 68 which granted a tax amnesty, such
payment is not a sufficient ground for lifting the order of the lower court of July 22, 1971 cancelling his certificate of naturalization. The legal
effect of payment under the decree is merely the removal of any civil, criminal or administrative liability on the part of the taxpayer, only
insofar as his tax case is concerned. Thus, paragraph 4 of the decree provides;

4. That after full settlement of the accounts mentioned herein, the taxpayer shall be free of any civil, criminal or
administrative liability insofar as his tax case is involved (Emphasis supplied)

In other words, the tax amnesty does not have the effect of obliterating his lack of good moral character and irreproachable
conduct which are grounds for denaturalization.

The lower court based its order of cancellation of citizenship on the finding of evasion of payment of lawful taxes which is sufficient ground,
under Sec. 2 of the Revised Naturalization Law requiring, among others, that applicant conduct himself "in a proper and irreproachable
manner during the entire period of his residence in the Philippines in his relation with constituted government as well as with the community
in which he is living," 13 to strip him of his citizenship without going into the other grounds for cancellation presented by the Solicitor General.

Finally, taking into account the fact that naturalization laws should be rigidly enforced in favor of the Government and against the applicant,
this Court has repeatedly maintained the view that where the applicant failed to meet the qualifications required for naturalization, the latter is
not entitled to Filipino citizenship. 14 More specifically, the Court has had occasion to state: "Admission to citizenship is one of the highest
privileges that the Republic of the Philippines can confer upon an alien. It is a privilege that should not be conferred except upon persons
fully qualified for it, and upon strict compliance with the law." 15 Philippine citizenship is a pearl of great price which should be cherished and
not taken for granted. Once acquired, its sheen must be burnished and not stained by any wrongdoing which could constitute ample ground
for divesting one of said citizenship. Hence, compliance with all the requirements of the law must be proved to the satisfaction of the
Court. 16

WHEREFORE, the instant appeal is hereby DISMISSED and the assailed decision AFFIRMED.

SO ORDERED.

G.R. No. 86564 August 1, 1989

RAMON L. LABO, JR., petitioner, 


vs.
THE COMMISSION ON ELECTIONS (COMELEC) EN BANC AND LUIS L.
LARDIZABAL, respondents

Estelito P. Mendoza for petitioner.

Rillera and Quintana for private respondent.


CRUZ, J.:

The petitioner asks this Court to restrain the Commission on Elections from looking into the question
of his citizenship as a qualification for his office as Mayor of Baguio City. The allegation that he is a
foreigner, he says, is not the issue. The issue is whether or not the public respondent has jurisdiction
to conduct any inquiry into this matter, considering that the petition for quo warranto against him was
not filed on time.

It is noteworthy that this argument is based on the alleged tardiness not of the petition itself but of
the payment of the filing fee, which the petitioner contends was an indispensable requirement. The
fee is, curiously enough, all of P300.00 only. This brings to mind the popular verse that for want of a
horse the kingdom was lost. Still, if it is shown that the petition was indeed filed beyond the
reglementary period, there is no question that this petition must be granted and the challenge
abated.

The petitioner's position is simple. He was proclaimed mayor-elect of Baguio City, on January 20,
1988. The petition for quo warranto was filed by the private respondent on January 26, 1988, but no
filing fee was paid on that date. This fee was finally paid on February 10, 1988, or twenty-one days
after his proclamation. As the petition by itself alone was ineffectual without the filing fee, it should be
deemed filed only when the fee was paid. This was done beyond the reglementary period provided
for under Section 253 of the Omnibus Election Code reading as follows:

SEC. 253. Petition for quo warranto. — Any voter contesting the election of a
Member of the Batasang Pambansa, regional, provincial, or city officer on the ground
of ineligibility or of disloyalty to the Republic of the Philippines shall file a sworn
petition for quo warranto with the Commission within ten days after the proclamation
of the result of the election.

The petitioner adds that the payment of the filing fee is required under Rule 36, Section 5, of the
Procedural Rules of the COMELEC providing that —

Sec. 5. No petition for quo warranto shall be given due course without the payment
of a filing fee in the amount of Three Hundred Pesos (P300.00) and the legal
research fee as required by law.

and stresses that there is abundant jurisprudence holding that the payment of the filing fee is
essential to the timeliness of the filling of the petition itself. He cites many rulings of the Court to this
effect, specifically Manchester v. Court of Appeals.  1

For his part, the private respondent denies that the filing fee was paid out of time. In fact he says, it
was flied ahead of time. His point is that when he filed his "Petition for Quo Warranto with Prayer for
Immediate Annulment of Proclamation and Restraining Order or Injunction" on January 26, 1988, the
COMELEC treated it as a pre-proclamation controversy and docketed it as SPC Case No. 88-288.
No docket fee was collected although it was offered. It was only on February 8, 1988, that the
COMELEC decided to treat his petition as solely for quo warranto and re-docketed it as EPC Case
No. 88-19, serving him notice on February 10, 1988. He immediately paid the filing fee on that date.

The private respondent argues further that during the period when the COMELEC regarded his
petition as a pre-proclamation controversy, the time for filing an election protest or quo
warranto proceeding was deemed suspended under Section 248 of the Omnibus Election Code.   At 2

any rate, he says, Rule 36, Section 5, of the COMELEC Rules of Procedure cited by the petitioner,
became effective only on November 15, 1988, seven days after publication of the said Rules in the
Official Gazette pursuant to Section 4, Rule 44 thereof.   These rules could not retroact to January
3

26,1988, when he filed his petition with the COMELEC.

In his Reply, the petitioner argues that even if the Omnibus Election Code did not require it, the
payment of filing fees was still necessary under Res. No. 1996 and, before that, Res. No. 1450 of
the respondent COMELEC, promulgated on January 12, 1988, and February 26, 1980, respectively.
To this, the private respondent counters that the latter resolution was intended for the local elections
held on January 30, 1980, and did not apply to the 1988 local elections, which were supposed to be
governed by the first-mentioned resolution. However, Res. No. 1996 took effect only on March 3,
1988, following the lapse of seven days after its publication as required by RA No. 6646, otherwise
known as the Electoral Reform Law of 1987, which became effective on January 5, 1988. Its Section
30 provides in part:

Sec. 30. Effectivity of Regulations and Orders of the Commission. — The rules and
regulations promulgated by the Commission shall take effect on the seventh day
after their publication in the Official Gazette or in at least (2) daily newspapers of
general circulation in the Philippines.

The Court has considered the arguments of the parties and holds that the petition for quo
warranto was filed on time. We agree with the respondents that the fee was paid during the ten-day
period as extended by the pendency of the petition when it was treated by the COMELEC as a pre-
proclamation proceeding which did not require the payment of a filing fee. At that, we reach this
conclusion only on the assumption that the requirement for the payment of the fees in quo
warranto proceedings was already effective. There is no record that Res. No. 1450 was even
published; and as for Res. No. 1996, this took effect only on March 3, 1988, seven days after its
publication in the February 25, 1988 issues of the Manila Chronicle and the Philippine Daily Inquirer,
or after the petition was filed.

The petitioner forgets Tañ;ada v. Tuvera   when he argues that the resolutions became effective
4

"immediately upon approval" simply because it was so provided therein. We held in that case that
publication was still necessary under the due process clause despite such effectivity clause.

In any event, what is important is that the filing fee was paid, and whatever delay there may have
been is not imputable to the private respondent's fault or neglect. It is true that in
the Manchester Case, we required the timely payment of the filing fee as a precondition for the
timeliness of the filing of the case itself. In Sun Insurance Office, Ltd. v. Asuncion,   however this
5

Court, taking into account the special circumstances of that case, declared:

This Court reiterates the rule that the trial court acquires jurisdiction over a case only
upon the payment of the prescribed filing fee. However, the court may allow the
payment of the said fee within a reasonable time. In the event of non-compliance
therewith, the case shall be dismissed.

The same idea is expressed in Rule 42, Section 18, of the COMELEC Rules of Procedure adopted
on June 20, 1988, thus:

Sec. 18. Non-payment of prescribed fees. — If the fees above prescribed are not
paid, the Commission may refuse to take action thereon until they are paid and may
dismiss the action or the proceeding. (Emphasis supplied.)

The Court notes that while arguing the technical point that the petition for quo warranto should be
dismissed for failure to pay the filing fee on time, the petitioner would at the same time minimize his
alleged lack of citizenship as "a futile technicality," It is regrettable, to say the least, that the
requirement of citizenship as a qualification for public office can be so demeaned. What is worse is
that it is regarded as an even less important consideration than the reglementary period the
petitioner insists upon.

This matter should normally end here as the sole issue originally raised by the petitioner is the
timeliness of the quo warranto proceedings against him. However, as his citizenship is the subject of
that proceeding, and considering the necessity for an early resolution of that more important
question clearly and urgently affecting the public interest, we shall directly address it now in this
same action.

The Court has similarly acted in a notable number of cases, thus:

From the foregoing brief statement of the nature of the instant case, it would appear
that our sole function in this proceeding should be to resolve the single issue of
whether or not the Court of Appeals erred in ruling that the motion for new trial of the
GSIS in question should indeed be deemed pro forma.But going over the extended
pleadings of both parties, the Court is immediately impressed that substantial justice
may not be timely achieved, if we should decide this case upon such a technical
ground alone. We have carefully read all the allegations and arguments of the
parties, very ably and comprehensively expounded by evidently knowledgeable and
unusually competent counsel, and we feel we can better serve the interests of justice
by broadening the scope of our inquiry, for as the record before us stands, we see
that there is enough basis for us to end the basic controversy between the parties
here and now, dispensing, however, with procedural steps which would not anyway
affect substantially the merits of their respective claims. 
6

xxx

While it is the fault of the petitioner for appealing to the wrong court and thereby
allowing the period for appeal to lapse, the more correct procedure was for the
respondent court to forward the case to the proper court which was the Court of
Appeals for appropriate action. Considering, however, the length of time that this
case has been pending, we apply the rule in the case of Del Castillo v. Jaymalin,
(112 SCRA 629) and follow the principle enunciated in Alger Electric, Inc. v. Court of
Appeals, (135 SCRA 37) which states:

... it is a cherished rule of procedure for this Court to always strive to


settle the entire controversy in a single proceeding leaving no root or
branch to bear the seeds of future litigation. No useful purpose will be
served if this case is remanded to the trial court only to have its
decision raised again to the Intermediate Appellate Court and from
there to this Court. (p. 43)

Only recently in the case of Beautifont, Inc., et al. v. Court of Appeals, et al. (G.R.
No. 50141, January 29, 1988), we stated that:

... But all those relevant facts are now before this Court. And those facts dictate the
rendition of a verdict in the petitioner's favor. There is therefore no point in referring
the case back to the Court of Appeals. The facts and the legal propositions involved
will not change, nor should the ultimate judgment. Considerable time has already
elapsed and, to serve the ends of justice, it is time that the controversy is finally laid
to rest. (See Sotto v. Samson, 5 SCRA 733; Republic v. Paredes, 108 Phil. 57;
Lianga Lumber Co. v. Lianga Timber Co., Inc., 76 SCRA 197; Erico v. Heirs of
Chigas, 98 SCRA 575; Francisco v. City of Davao, 12 SCRA 628; Valencia v.
Mabilangan, 105 Phil. 162). Sound practice seeks to accommodate the theory which
lâwphî1.ñèt 

avoids waste of time, effort and expense, both to the parties and the government, not
to speak of delay in the disposal of the case (cf. Fernandez v. Garcia, 92 Phil. 592,
597). A marked characteristic of our judicial set-up is that where the dictates of
justice so demand ... the Supreme Court should act, and act with finality.' (Li Siu Liat
v. Republic, 21 SCRA 1039, 1046, citing Samal v. CA, 99 Phil. 230 and U.S. v.
Gimenez, 34 Phil. 74). In this case, the dictates of justice do demand that this Court
act, and act with finality. 
7

xxx

Remand of the case to the lower court for further reception of evidence is not
necessary where the court is in a position to resolve the dispute based on the
records before it. On many occasions, the Court, in the public interest and the
expeditious administration of justice, has resolved actions on the merits instead of
remanding them to the trial court for further proceedings, such as where the ends of
justice would not be subserved by the remand of the case or when public interest
demands an early disposition of the case or where the trial court had already
received all the evidence of the parties.  8

This course of action becomes all the more justified in the present case where, to repeat for stress, it
is claimed that a foreigner is holding a public office.

We also note in his Reply, the petitioner says:

In adopting private respondent's comment, respondent COMELEC implicitly adopted


as "its own" private respondent's repeated assertion that petitioner is no longer a
Filipino citizen. In so doing, has not respondent COMELEC effectively disqualified
itself, by reason of prejudgment, from resolving the petition for quo warranto filed by
private respondent still pending before it?  9

This is still another reason why the Court has seen fit to rule directly on the merits of this case.

Going over the record, we find that there are two administrative decisions on the question of the
petitioner's citizenship. The first was rendered by the Commission on Elections on May 12, 1982,
and found the petitioner to be a citizen of the Philippines.   The second was rendered by the
10

Commission on Immigration and Deportation on September 13, 1988, and held that the petitioner
was not a citizen of the Philippines. 11

The first decision was penned by then COMELEC Chigas, Vicente Santiago, Jr., with
Commissioners Pabalate Savellano and Opinion concurring in full and Commissioner Bacungan
concurring in the dismissal of the petition "without prejudice to the issue of the respondent's
citizenship being raised anew in a proper case." Commissioner Sagadraca reserved his vote, while
Commissioner Felipe was for deferring decision until representations shall have been made with the
Australian Embassy for official verification of the petitioner's alleged naturalization as an Australian.

The second decision was unanimously rendered by Chairman Miriam Defensor-Santiago and
Commissioners Alano and Geraldez of the Commission on Immigration and Deportation. It is
important to observe that in the proceeding before the COMELEC, there was no direct proof that the
herein petitioner had been formally naturalized as a citizen of Australia. This conjecture, which was
eventually rejected, was merely inferred from the fact that he had married an Australian citizen,
obtained an Australian passport, and registered as an alien with the CID upon his return to this
country in 1980.

On the other hand, the decision of the CID took into account the official statement of the Australian
Government dated August 12, 1984, through its Consul in the Philippines, that the petitioner was still
an Australian citizen as of that date by reason of his naturalization in 1976. That statement   is 12

reproduced in full as follows:

I, GRAHAM COLIN WEST, Consul of Australia in the Philippines, by virtue of a certificate of


appointment signed and sealed by the Australian Minister of State for Foreign Affairs on 19 October
1983, and recognized as such by Letter of Patent signed and sealed by the Philippines Acting
Minister of Foreign Affairs on 23 November 1983, do hereby provide the following statement in
response to the subpoena Testificandum dated 9 April 1984 in regard to the Petition for
disqualification against RAMON LABO, JR. Y LOZANO (SPC No. 84-73), and do hereby certify that
the statement is true and correct.

STATEMENT

A) RAMON LABO, JR. Y LOZANO, date of birth 23 December 1934, was married in
the Philippines to an Australian citizen. As the spouse of an Australian citizen, he
was not required to meet normal requirements for the grant of citizenship and was
granted Australian citizenship by Sydney on 28 July 1976.

B) Any person over the age of 16 years who is granted Australian citizenship must
take an oath of allegiance or make an affirmation of allegiance. The wording of the
oath of affirmation is: "I ..., renouncing all other allegiance ..." etc. This need not
necessarily have any effect on his former nationality as this would depend on the
citizenship laws of his former country.

C) The marriage was declared void in the Australian Federal Court in Sydney on 27
June 1980 on the ground that the marriage had been bigamous.

D) According to our records LABO is still an Australian citizen.

E) Should he return to Australia, LABO may face court action in respect of Section 50
of Australian Citizenship Act 1948 which relates to the giving of false or misleading
information of a material nature in respect of an application for Australian citizenship.
If such a prosecution was successful, he could be deprived of Australian citizenship
under Section 21 of the Act.

F) There are two further ways in which LABO could divest himself of Australian
citizenship:

(i) He could make a declaration of Renunciation of Australian citizenship under


Section 18 of the Australian Citizenship Act, or

(ii) If he acquired another nationality, (for example, Filipino) by a formal and voluntary
act other than marriage, then he would automatically lose as Australian citizenship
under Section 17 of the Act.
IN WITNESS WHEREOF, I HAVE HEREUNTO SET MAY HAND AND SEAL OF
THE AUSTRALIAN EMBASSY, MANILA, THIS 12th DAY OF APRIL 1984. DONE AT
MANILA IN THE PHILIPPINES.

(Signed) GRAHAM C. WEST Consul

This was affirmed later by the letter of February 1, 1988, addressed to the private
respondent by the Department of Foreign Affairs reading as follows:  13

Sir:

With reference to your letter dated 1 February 1988, I wish to inform you that inquiry
made with the Australian Government through the Embassy of the Philippines in
Canberra has elicited the following information:

1) That Mr. Ramon L. Labo, Jr. acquired Australian citizenship on 28 July 1976.

2) That prior to 17 July 1986, a candidate for Australian citizenship had to either
swear an oath of allegiance or make an affirmation of allegiance which carries a
renunciation of "all other allegiance.

Very truly yours, For the Secretary of Foreign Affairs: (SGD) RODOLFO SEVERINO,
JR. Assistant Secretary

The decision also noted the oath of allegiance taken by every naturalized Australian reading as
follows:

OATH OF ALLEGIANCE

I, A.B., renouncing all other allegiance, swear by Almighty God that I will be faithful
and bear true allegiance to Her Majesty Elizabeth the Second, Queen of Australia,
Her heirs and successors according to law, and that I will faithfully observe the laws
of Australia and fulfill my duties as an Australian citizen. 
14

and the Affirmation of Allegiance, which declares:

AFFIRMATION OF ALLEGIANCE

I, A.B., renouncing all other allegiance, solemnly and sincerely promise and declare
that I will be faithful and bear true allegiance to Her Majesty Elizabeth the Second,
Queen of Australia, Her heirs and successors according to law, and that I will
faithfully observe the Laws of Australia and fulfill my duties as an Australian citizen. 
15

The petitioner does not question the authenticity of the above evidence. Neither does he deny that
he obtained Australian Passport No. 754705, which he used in coming back to the Philippines in
1980, when he declared before the immigration authorities that he was an alien and registered as
such under Alien Certificate of Registration No. B-323985.   He later asked for the change of his
16

status from immigrant to a returning former Philippine citizen and was granted Immigrant Certificate
of Residence No. 223809.   He also categorically declared that he was a citizen of Australia in a
17

number of sworn statements voluntarily made by him and. even sought to avoid the jurisdiction of
the barangay court on the ground that he was a foreigner.  18
The decision of the COMELEC in 1982 quaintly dismisses all these acts as "mistakes" that did not
divest the petitioner of his citizenship, although, as earlier noted, not all the members joined in this
finding. We reject this ruling as totally baseless. The petitioner is not an unlettered person who was
not aware of the consequences of his acts, let alone the fact that he was assisted by counsel when
he performed these acts.

The private respondent questions the motives of the COMELEC at that time and stresses Labo's
political affiliation with the party in power then, but we need not go into that now.

There is also the claim that the decision can no longer be reversed because of the doctrine of res
judicata, but this too must be dismissed. This doctrine does not apply to questions of citizenship, as
the Court has ruled in several cases.   Moreover, it does not appear that it was properly and
19

seasonably pleaded, in a motion to dismiss or in the answer, having been invoked only when the
petitioner filed his reply   to the private respondent's comment. Besides, one of the requisites of res
20

judicata, to wit, identity of parties, is not present in this case.

The petitioner's contention that his marriage to an Australian national in 1976 did not automatically
divest him of Philippine citizenship is irrelevant. There is no claim or finding that he automatically
ceased to be a Filipino because of that marriage. He became a citizen of Australia because he was
naturalized as such through a formal and positive process, simplified in his case because he was
married to an Australian citizen. As a condition for such naturalization, he formally took the Oath of
Allegiance and/or made the Affirmation of Allegiance, both quoted above. Renouncing all other
allegiance, he swore "to be faithful and bear true allegiance to Her Majesty Elizabeth the Second,
Queen of Australia ..." and to fulfill his duties "as an Australian citizen."

The petitioner now claims that his naturalization in Australia made him at worst only a dual national
and did not divest him of his Philippine citizenship. Such a specious argument cannot stand against
the clear provisions of CA No. 63, which enumerates the modes by which Philippine citizenship may
be lost. Among these are: (1) naturalization in a foreign country; (2) express renunciation of
citizenship; and (3) subscribing to an oath of allegiance to support the Constitution or laws of a
foreign country, all of which are applicable to the petitioner. It is also worth mentioning in this
connection that under Article IV, Section 5, of the present Constitution, "Dual allegiance of citizens is
inimical to the national interest and shall be dealt with by law."

Even if it be assumed that, as the petitioner asserts, his naturalization in Australia was annulled after
it was found that his marriage to the Australian citizen was bigamous, that circumstance alone did
not automatically restore his Philippine citizenship. His divestiture of Australian citizenship does not
concern us here. That is a matter between him and his adopted country. What we must consider is
the fact that he voluntarily and freely rejected Philippine citizenship and willingly and knowingly
embraced the citizenship of a foreign country. The possibility that he may have been subsequently
rejected by Australia, as he claims, does not mean that he has been automatically reinstated as a
citizen of the Philippines.

Under CA No. 63 as amended by PD No. 725, Philippine citizenship may be reacquired by direct act
of Congress, by naturalization, or by repatriation. It does not appear in the record, nor does the
petitioner claim, that he has reacquired Philippine citizenship by any of these methods. He does not
point to any judicial decree of naturalization as to any statute directly conferring Philippine citizenship
upon him. Neither has he shown that he has complied with PD No. 725, providing that:

... (2) natural-born Filipinos who have lost their Philippine citizenship may reacquire
Philippine citizenship through repatriation by applying with the Special Committee on
Naturalization created by Letter of Instruction No. 270, and, if their applications are
approved, taking the necessary oath of allegiance to the Republic of the Philippines,
after which they shall be deemed to have reacquired Philippine citizenship. The
Commission on Immigration and Deportation shall thereupon cancel their certificate
of registration. (Emphasis supplied.)

That is why the Commission on Immigration and Deportation rejected his application for the
cancellation of his alien certificate of registration. And that is also the reason we must deny his
present claim for recognition as a citizen of the Philippines.

The petitioner is not now, nor was he on the day of the local elections on January 18, 1988, a citizen
of the Philippines. In fact, he was not even a qualified voter under the Constitution itself because of
his alienage.   He was therefore ineligible as a candidate for mayor of Baguio City, under Section 42
21

of the Local Government Code providing in material part as follows:

Sec. 42. Qualifications. — An elective local official must be a citizen of the


Philippines, at least twenty-three years of age on election day, a qualified voter
registered as such in the barangay, municipality, city or province where he proposes
to be elected, a resident therein for at least one year at the time of the filing of his
certificate of candidacy, and able to read and write English, Filipino, or any other
local language or dialect.

The petitioner argues that his alleged lack of citizenship is a "futile technicality" that should not
frustrate the will of the electorate of Baguio City, who elected him by a "resonant and thunderous
majority." To be accurate, it was not as loud as all that, for his lead over the second-placer was only
about 2,100 votes. In any event, the people of that locality could not have, even unanimously,
changed the requirements of the Local Government Code and the Constitution. The electorate had
no power to permit a foreigner owing his total allegiance to the Queen of Australia, or at least a
stateless individual owing no allegiance to the Republic of the Philippines, to preside over them as
mayor of their city. Only citizens of the Philippines have that privilege over their countrymen.

The probability that many of those who voted for the petitioner may have done so in the belief that
he was qualified only strengthens the conclusion that the results of the election cannot nullify the
qualifications for the office now held by him. These qualifications are continuing requirements; once
any of them is lost during incumbency, title to the office itself is deemed forfeited. In the case at bar,
the citizenship and voting requirements were not subsequently lost but were not possessed at all in
the first place on the day of the election. The petitioner was disqualified from running as mayor and,
although elected, is not now qualified to serve as such.

Finally, there is the question of whether or not the private respondent, who filed the quo
warranto petition, can replace the petitioner as mayor. He cannot. The simple reason is that as he
obtained only the second highest number of votes in the election, he was obviously not the choice of
the people of Baguio city.

The latest ruling of the Court on this issue is Santos v. Commission on Elections   decided in 1985.
22

In that case, the candidate who placed second was proclaimed elected after the votes for his
winning rival, who was disqualified as a turncoat and considered a non-candidate, were all
disregarded as stray. In effect, the second placer won by default. That decision was supported by
eight members of the Court then   with three dissenting   and another two reserving their vote.   One
23 24 25

was on official leave.  26

Re-examining that decision, the Court finds, and so holds, that it should be reversed in favor of the
earlier case of Geronimo v. Ramos,   Which represents the more logical and democratic rule. That
27
case, which reiterated the doctrine first announced in 1912 in Topacio vs. Paredes   was supported
28

by ten members of the Court   without any dissent, although one reserved his vote,   another took
29 30

no part   and two others were on leave.   There the Court held:
31 32

... it would be extremely repugnant to the basic concept of the constitutionally


guaranteed right to suffrage if a candidate who has not acquired the majority or
plurality of votes is proclaimed a winner and imposed as the representative of a
constituency, the majority of which have positively declared through their ballots that
they do not choose him.

Sound policy dictates that public elective offices are filled by those who have
received the highest number of votes cast in the election for that office, and it is a
fundamental Idea in all republican forms of government that no one can be declared
elected and no measure can be declared carried unless he or it receives a majority
or plurality of the legal votes cast in the election. (20 Corpus Juris 2nd, S 243, p.
676.)

The fact that the candidate who obtained the highest number of votes is later
declared to be disqualified or not eligible for the office to which he was elected does
not necessarily entitle the candidate who obtained the second highest number of
votes to be declared the winner of the elective office. The votes cast for a dead,
disqualified, or non-eligible person may not be valid to vote the winner into office or
maintain him there. However, in the absence of a statute which clearly asserts a
contrary political and legislative policy on the matter, if the votes were cast in the
sincere belief that the candidate was alive, qualified, or eligible, they should not be
treated as stray, void or meaningless.

It remains to stress that the citizen of the Philippines must take pride in his status as such and
cherish this priceless gift that, out of more than a hundred other nationalities, God has seen fit to
grant him. Having been so endowed, he must not lightly yield this precious advantage, rejecting it for
another land that may offer him material and other attractions that he may not find in his own
country. To be sure, he has the right to renounce the Philippines if he sees fit and transfer his
allegiance to a state with more allurements for him.   But having done so, he cannot expect to be
33

welcomed back with open arms once his taste for his adopted country turns sour or he is himself
disowned by it as an undesirable alien.

Philippine citizenship is not a cheap commodity that can be easily recovered after its renunciation. It
may be restored only after the returning renegade makes a formal act of re-dedication to the country
he has abjured and he solemnly affirms once again his total and exclusive loyalty to the Republic of
the Philippines. This may not be accomplished by election to public office.

WHEREFORE, petitioner Ramon J. Labo, Jr. is hereby declared NOT a citizen of the Philippines and
therefore DISQUALIFIED from continuing to serve as Mayor of Baguio City. He is ordered to
VACATE his office and surrender the same to the Vice-Mayor of Baguio City, once this decision
becomes final and executory. The temporary restraining order dated January 31, 1989, is LIFTED.

G.R. No. 151914            July 31, 2002


TEODULO M. COQUILLA, petitioner, 
vs.
THE HON. COMMISSION ON ELECTIONS and MR. NEIL M. ALVAREZ, respondents.

MENDOZA, J.:

This is a petition for certiorari to set aside the resolution, 1 dated July 19, 2001, of the Second
Division of the Commission on Elections (COMELEC), ordering the cancellation of the certificate of
candidacy of petitioner Teodulo M. Coquilla for the position of mayor of Oras, Eastern Samar in the
May 14, 2001 elections and the order, dated January 30, 2002, of the COMELEC en banc denying
petitioner’s motion for reconsideration.

The facts are as follows:

Petitioner Coquilla was born on February 17, 1938 of Filipino parents in Oras, Eastern Samar. He
grew up and resided there until 1965, when he joined the United States Navy. He was subsequently
naturalized as a U.S. citizen. 2 From 1970 to 1973, petitioner thrice visited the Philippines while on
leave from the U.S. Navy.3 Otherwise, even after his retirement from the U.S. Navy in 1985, he
remained in the United States.

On October 15, 1998, petitioner came to the Philippines and took out a residence certificate,
although he continued making several trips to the United States, the last of which took place on July
6, 2000 and lasted until August 5, 2000.4 Subsequently, petitioner applied for repatriation under R.A.
No. 81715 to the Special Committee on Naturalization. His application was approved on November 7,
2000, and, on November 10, 2000, he took his oath as a citizen of the Philippines. Petitioner was
issued Certificate of Repatriation No. 000737 on November 10, 2000 and Bureau of Immigration
Identification Certificate No. 115123 on November 13, 2000.

On November 21, 2000, petitioner applied for registration as a voter of Butnga, Oras, Eastern
Samar. His application was approved by the Election Registration Board on January 12, 2001. 6 On
February 27, 2001, he filed his certificate of candidacy stating therein that he had been a resident of
Oras, Eastern Samar for "two (2) years."7

On March 5, 2001, respondent Neil M. Alvarez, who was the incumbent mayor of Oras and who was
running for reelection, sought the cancellation of petitioner’s certificate of candidacy on the ground
that the latter had made a material misrepresentation in his certificate of candidacy by stating that he
had been a resident of Oras for two years when in truth he had resided therein for only about six
months since November 10, 2000, when he took his oath as a citizen of the Philippines.

The COMELEC was unable to render judgment on the case before the elections on May 14, 2001.
Meanwhile, petitioner was voted for and received the highest number of votes (6,131) against private
respondent’s 5,752 votes, or a margin of 379 votes. On May 17, 2001, petitioner was proclaimed
mayor of Oras by the Municipal Board of Canvassers.8 He subsequently took his oath of office.

On July 19, 2001, the Second Division of the COMELEC granted private respondent’s petition and
ordered the cancellation of petitioner’s certificate of candidacy on the basis of the following findings:

Respondent’s frequent or regular trips to the Philippines and stay in Oras, Eastern Samar
after his retirement from the U.S. Navy in 1985 cannot be considered as a waiver of his
status as a permanent resident or immigrant . . . of the U.S.A. prior to November 10, 2000 as
would qualify him to acquire the status of residency for purposes of compliance with the one-
year residency requirement of Section 39(a) of the Local Government Code of 1991 in
relation to Sections 65 and 68 of the Omnibus Election Code. The one (1) year residency
requirement contemplates of the actual residence of a Filipino citizen in the constituency
where he seeks to be elected.

All things considered, the number of years he claimed to have resided or stayed in Oras,
Eastern Samar since 1985 as an American citizen and permanent resident of the U.S.A.
before November 10, 2000 when he reacquired his Philippine citizenship by [repatriation]
cannot be added to his actual residence thereat after November 10, 2000 until May 14, 2001
to cure his deficiency in days, months, and year to allow or render him eligible to run for an
elective office in the Philippines. Under such circumstances, by whatever formula of
computation used, respondent is short of the one-year residence requirement before the
May 14, 2001 elections.9

Petitioner filed a motion for reconsideration, but his motion was denied by the COMELEC en
banc on January 30, 2002. Hence this petition.

I.

Two questions must first be resolved before considering the merits of this case: (a) whether the 30-
day period for appealing the resolution of the COMELEC was suspended by the filing of a motion for
reconsideration by petitioner and (b) whether the COMELEC retained jurisdiction to decide this case
notwithstanding the proclamation of petitioner.

A.         With respect to the first question, private respondent contends that the petition in this case
should be dismissed because it was filed late; that the COMELEC en banc had denied petitioner’s
motion for reconsideration for being pro forma; and that, pursuant to Rule 19, §4 of the COMELEC
Rules of Procedure, the said motion did not suspend the running of the 30-day period for filing this
petition. He points out that petitioner received a copy of the resolution, dated July 19, 2001, of the
COMELEC’s Second Division on July 28, 2001, so that he had only until August 27, 2001 within
which to file this petition. Since the petition in this case was filed on February 11, 2002, the same
should be considered as having been filed late and should be dismissed.

Private respondent’s contention has no merit.

Rule 19 of the COMELEC Rules of Procedure provides in pertinent parts:

Sec. 2. Period for Filing Motions for Reconsideration. – A motion to reconsider a decision,
resolution, order, or ruling of a Division shall be filed within five days from the promulgation
thereof. Such motion, if not pro-forma, suspends the execution for implementation of the
decision, resolution, order, or ruling.

Sec. 4. Effect of Motion for Reconsideration on Period to Appeal. – A motion to reconsider a


decision, resolution, order, or ruling, when not pro-forma, suspends the running of the period
to elevate the matter to the Supreme Court.

The five-day period for filing a motion for reconsideration under Rule 19, §2 should be counted from
the receipt of the decision, resolution, order, or ruling of the COMELEC Division. 10 In this case,
petitioner received a copy of the resolution of July 19, 2001 of the COMELEC’s Second Division on
July 28, 2001. Five days later, on August 2, 2001, he filed his motion for reconsideration. On
February 6, 2002, he received a copy of the order, dated January 30, 2002, of the COMELEC en
banc denying his motion for reconsideration. Five days later, on February 11, 2002, he filed this
petition for certiorari. There is no question, therefore, that petitioner’s motion for reconsideration of
the resolution of the COMELEC Second Division, as well as his petition for certiorari to set aside of
the order of the COMELEC en banc, was filed within the period provided for in Rule 19, §2 of the
COMELEC Rules of Procedure and in Art. IX(A), §7 of the Constitution.

It is contended, however, that petitioner’s motion for reconsideration before the COMELEC en
banc did not suspend the running of the period for filing this petition because the motion was pro
forma and, consequently, this petition should have been filed on or before August 27, 2001. It was
actually filed, however, only on February 11, 2002. Private respondent cites the finding of the
COMELEC en banc that —

An incisive examination of the allegations in the Motion for Reconsideration shows that the
same [are] a mere rehash of his averments contained in his Verified
Answer and Memorandum. Neither did respondent raise new matters that would sufficiently
warrant a reversal of the assailed resolution of the Second Division. This makes the said
Motion pro forma.11

We do not think this contention is correct. The motion for reconsideration was not pro forma and its
filing did suspend the period for filing the petition for certiorari in this case. The mere reiteration in a
motion for reconsideration of the issues raised by the parties and passed upon by the court does not
make a motion pro forma; otherwise, the movant’s remedy would not be a reconsideration of the
decision but a new trial or some other remedy.12 But, as we have held in another case:13

Among the ends to which a motion for reconsideration is addressed, one is precisely to
convince the court that its ruling is erroneous and improper, contrary to the law or the
evidence; and in doing so, the movant has to dwell of necessity upon the issues passed
upon by the court. If a motion for reconsideration may not discuss these issues, the
consequence would be that after a decision is rendered, the losing party would be confined
to filing only motions for reopening and new trial.

Indeed, in the cases where a motion for reconsideration was held to be pro forma, the motion was
so held because (1) it was a second motion for reconsideration,14 or (2) it did not comply with the rule
that the motion must specify the findings and conclusions alleged to be contrary to law or not
supported by the evidence,15 or (3) it failed to substantiate the alleged errors, 15 or (4) it merely
alleged that the decision in question was contrary to law,17 or (5) the adverse party was not given
notice thereof.18 The 16-page motion for reconsideration filed by petitioner in the COMELEC en
banc suffers from none of the foregoing defects, and it was error for the COMELEC en banc to rule
that petitioner’s motion for reconsideration was pro forma because the allegations raised therein are
a mere "rehash" of his earlier pleadings or did not raise "new matters." Hence, the filing of the motion
suspended the running of the 30-day period to file the petition in this case, which, as earlier shown,
was done within the reglementary period provided by law.

B. As stated before, the COMELEC failed to resolve private respondent’s petition for cancellation of
petitioner’s certificate of candidacy before the elections on May 14, 2001. In the meantime, the votes
were canvassed and petitioner was proclaimed elected with a margin of 379 votes over private
respondent. Did the COMELEC thereby lose authority to act on the petition filed by private
respondent?

R.A. No. 6646 provides:


SECTION 6. Effect of Disqualification Case. – Any candidate who has been declared by final
judgment to be disqualified shall not be voted for, and the votes cast for him shall not be
counted. If for any reason a candidate is not declared by final judgment before an election to
be disqualified and he is voted for and receives the winning number of votes in such election,
the Court or Commission shall continue with the trial and hearing of the action, inquiry, or
protest and, upon motion of the complainant or any intervenor, may during the pendency
thereof order the suspension of the proclamation of such candidate whenever the evidence
of his guilt is strong. (Emphasis added)

SECTION 7. Petition to Deny Due Course To or Cancel a Certificate of Candidacy. — The


procedure hereinabove provided shall apply to petitions to deny due course to or cancel a
certificate of candidacy as provided in Section 78 of Batas Pambansa Blg. 881.

The rule then is that candidates who are disqualified by final judgment before the election shall not
be voted for and the votes cast for them shall not be counted. But those against whom no final
judgment of disqualification had been rendered may be voted for and proclaimed, unless, on motion
of the complainant, the COMELEC suspends their proclamation because the grounds for their
disqualification or cancellation of their certificates of candidacy are strong. Meanwhile, the
proceedings for disqualification of candidates or for the cancellation or denial of certificates of
candidacy, which have been begun before the elections, should continue even after such elections
and proclamation of the winners. In Abella v. COMELEC19 and Salcedo II v. COMELEC,20 the
candidates whose certificates of candidacy were the subject of petitions for cancellation were voted
for and, having received the highest number of votes, were duly proclaimed winners. This Court, in
the first case, affirmed and, in the second, reversed the decisions of the COMELEC rendered after
the proclamation of candidates, not on the ground that the latter had been divested of jurisdiction
upon the candidates’ proclamation but on the merits.

II.

On the merits, the question is whether petitioner had been a resident of Oras, Eastern Samar at
least one (1) year before the elections held on May 14, 2001 as he represented in his certificate of
candidacy. We find that he had not.

First, §39(a) of the Local Government Code (R.A No. 7160) provides:

Qualifications. - (a) An elective local official must be a citizen of the Philippines; a registered
voter in the barangay, municipality, city, or province or, in the case of a member of the
sangguniang panlalawigan, sangguniang panlungsod, or sangguniang bayan, the district
where he intends to be elected; a resident therein for at least one (1) year immediately
preceding the day of the election; and able to read and write Filipino or any other local
language or dialect. (Emphasis added)

The term "residence" is to be understood not in its common acceptation as referring to "dwelling" or
"habitation,"21but rather to "domicile" or legal residence, 22 that is, "the place where a party actually or
constructively has his permanent home, where he, no matter where he may be found at any given
time, eventually intends to return and remain (animus manendi)."23 A domicile of origin is acquired by
every person at birth. It is usually the place where the child’s parents reside and continues until the
same is abandoned by acquisition of new domicile (domicile of choice).24

In the case at bar, petitioner lost his domicile of origin in Oras by becoming a U.S. citizen after
enlisting in the U.S. Navy in 1965. From then on and until November 10, 2000, when he reacquired
Philippine citizenship, petitioner was an alien without any right to reside in the Philippines save as
our immigration laws may have allowed him to stay as a visitor or as a resident alien.

Indeed, residence in the United States is a requirement for naturalization as a U.S. citizen. Title 8,
§1427(a) of the United States Code provides:

Requirements of naturalization. – Residence

(a) No person, except as otherwise provided in this subchapter, shall be naturalized unless


such applicant, (1) immediately preceding the date of filing his application for
naturalization has resided continuously, after being lawfully admitted for permanent
residence, within the United States for at least five years and during the five years
immediately preceding the date of filing his petition has been physically present therein for
periods totaling at least half of that time, and who has resided within the State or within the
district of the Service in the United States in which the applicant filed the application for at
least three months, (2) has resided continuously within the United States from the date of the
application up to the time of admission to citizenship, and (3) during all the period referred to
in this subsection has been and still is a person of good moral character, attached to the
principles of the Constitution of the United States, and well disposed to the good order and
happiness of the United States. (Emphasis added)

In Caasi v. Court of Appeals,25 this Court ruled that immigration to the United States by virtue of a
"greencard," which entitles one to reside permanently in that country, constitutes abandonment of
domicile in the Philippines. With more reason then does naturalization in a foreign country result in
an abandonment of domicile in the Philippines.

Nor can petitioner contend that he was "compelled to adopt American citizenship" only by reason of
his service in the U.S. armed forces.26 It is noteworthy that petitioner was repatriated not under R.A.
No. 2630, which applies to the repatriation of those who lost their Philippine citizenship by accepting
commission in the Armed Forces of the United States, but under R.A. No. 8171, which, as earlier
mentioned, provides for the repatriation of, among others, natural-born Filipinos who lost their
citizenship on account of political or economic necessity. In any event, the fact is that, by having
been naturalized abroad, he lost his Philippine citizenship and with it his residence in the Philippines.
Until his reacquisition of Philippine citizenship on November 10, 2000, petitioner did not reacquire his
legal residence in this country.

Second, it is not true, as petitioner contends, that he reestablished residence in this country in 1998
when he came back to prepare for the mayoralty elections of Oras by securing a Community Tax
Certificate in that year and by "constantly declaring" to his townmates of his intention to seek
repatriation and run for mayor in the May 14, 2001 elections. 27 The status of being an alien and a
non-resident can be waived either separately, when one acquires the status of a resident alien
before acquiring Philippine citizenship, or at the same time when one acquires Philippine citizenship.
As an alien, an individual may obtain an immigrant visa under §13 28 of the Philippine Immigration Act
of 1948 and an Immigrant Certificate of Residence (ICR)29 and thus waive his status as a non-
resident. On the other hand, he may acquire Philippine citizenship by naturalization under C.A. No.
473, as amended, or, if he is a former Philippine national, he may reacquire Philippine citizenship by
repatriation or by an act of Congress,30 in which case he waives not only his status as an alien but
also his status as a non-resident alien.

In the case at bar, the only evidence of petitioner’s status when he entered the country on October
15, 1998, December 20, 1998, October 16, 1999, and June 23, 2000 is the statement "Philippine
Immigration [–] Balikbayan" in his 1998-2008 U.S. passport. As for his entry on August 5, 2000, the
stamp bore the added inscription "good for one year stay."31 Under §2 of R.A. No. 6768 (An Act
Instituting a Balikbayan Program), the term balikbayan includes a former Filipino citizen who had
been naturalized in a foreign country and comes or returns to the Philippines and, if so, he is
entitled, among others, to a "visa-free entry to the Philippines for a period of one (1) year" (§3(c)). It
would appear then that when petitioner entered the country on the dates in question, he did so as a
visa-free balikbayan visitor whose stay as such was valid for one year only. Hence, petitioner can
only be held to have waived his status as an alien and as a non-resident only on November 10, 2000
upon taking his oath as a citizen of the Philippines under R.A. No. 8171.32 He lacked the requisite
residency to qualify him for the mayorship of Oras, Eastern, Samar.

Petitioner invokes the ruling in Frivaldo v. Commission on Elections33 in support of his contention that
the residency requirement in §39(a) of the Local Government Code includes the residency of one
who is not a citizen of the Philippines. Residency, however, was not an issue in that case and this
Court did not make any ruling on the issue now at bar. The question in Frivaldo was whether
petitioner, who took his oath of repatriation on the same day that his term as governor of Sorsogon
began on June 30, 1995, complied with the citizenship requirement under §39(a). It was held that he
had, because citizenship may be possessed even on the day the candidate assumes office. But in
the case of residency, as already noted, §39(a) of the Local Government Code requires that the
candidate must have been a resident of the municipality "for at least one (1) year immediately
preceding the day of the election."

Nor can petitioner invoke this Court’s ruling in Bengzon III v. House of Representatives Electoral
Tribunal.34 What the Court held in that case was that, upon repatriation, a former natural-born Filipino
is deemed to have recovered his original status as a natural-born citizen.

Third, petitioner nonetheless says that his registration as a voter of Butnga, Oras, Eastern Samar in
January 2001 is conclusive of his residency as a candidate because §117 of the Omnibus Election
Code requires that a voter must have resided in the Philippines for at least one year and in the city
or municipality wherein he proposes to vote for at least six months immediately preceding the
election. As held in Nuval v. Guray,35 however, registration as a voter does not bar the filing of a
subsequent case questioning a candidate’s lack of residency.

Petitioner’s invocation of the liberal interpretation of election laws cannot avail him any. As held
in Aquino v. Commission on Elections:36

A democratic government is necessarily a government of laws. In a republican government


those laws are themselves ordained by the people. Through their representatives, they
dictate the qualifications necessary for service in government positions. And as petitioner
clearly lacks one of the essential qualifications for running for membership in the House of
Representatives, not even the will of a majority or plurality of the voters of the Second District
of Makati City would substitute for a requirement mandated by the fundamental law itself.

Fourth, petitioner was not denied due process because the COMELEC failed to act on his motion to
be allowed to present evidence. Under §5(d), in relation to §7, of R.A. No. 6646 (Electoral Reforms
Law of 1987), proceedings for denial or cancellation of a certificate of candidacy are summary in
nature. The holding of a formal hearing is thus not de rigeur. In any event, petitioner cannot claim
denial of the right to be heard since he filed a Verified Answer, a Memorandum and a Manifestation,
all dated March 19, 2001, before the COMELEC in which he submitted documents relied by him in
this petition, which, contrary to petitioner’s claim, are complete and intact in the records.

III.
The statement in petitioner’s certificate of candidacy that he had been a resident of Oras, Eastern
Samar for "two years" at the time he filed such certificate is not true. The question is whether the
COMELEC was justified in ordering the cancellation of his certificate of candidacy for this reason.
We hold that it was. Petitioner made a false representation of a material fact in his certificate of
candidacy, thus rendering such certificate liable to cancellation. The Omnibus Election Code
provides:

SEC. 74. Contents of certificate of candidacy. – The certificate of candidacy shall state that
the person filing it is announcing his candidacy for the office stated therein and that he is
eligible for said office; if for Member of the Batasang Pambansa, the province, including its
component cities, highly urbanized city or district or sector which he seeks to represent; the
political party to which he belongs; civil status; his date of birth; residence; his post office
address for all election purposes; his profession or occupation; that he will support and
defend the Constitution of the Philippines and will maintain true faith and allegiance thereto;
that he will obey the laws, legal orders, and decrees promulgated by the duly constituted
authorities; that he is not a permanent resident or immigrant to a foreign country; that the
obligation imposed by his oath is assumed voluntarily, without mental reservation or purpose
of evasion; and that the facts stated in the certificate of candidacy are true to the best of his
knowledge.

SEC. 78. Petition to deny due course to or cancel a certificate of candidacy. – A verified
petition seeking to deny due course or to cancel a certificate of candidacy may be filed by
any person exclusively on the ground that any material representation contained therein as
required under Section 74 hereof is false. The petition may be filed at any time not later than
twenty-five days from the time of the filing of the certificate of candidacy and shall be
decided, after due notice and hearing, not later than fifteen days before the election.

Indeed, it has been held that a candidate’s statement in her certificate of candidacy for the position
of governor of Leyte that she was a resident of Kananga, Leyte when this was not so37 or that the
candidate was a "natural-born" Filipino when in fact he had become an Australian
citizen38 constitutes a ground for the cancellation of a certificate of candidacy. On the other hand, we
held in Salcedo II v. COMELEC 39 that a candidate who used her husband’s family name even though
their marriage was void was not guilty of misrepresentation concerning a material fact. In the case at
bar, what is involved is a false statement concerning a candidate’s qualification for an office for
which he filed the certificate of candidacy. This is a misrepresentation of a material fact justifying the
cancellation of petitioner’s certificate of candidacy. The cancellation of petitioner’s certificate of
candidacy in this case is thus fully justified.

WHEREFORE, the petition is DISMISSED and the resolution of the Second Division of the
Commission on Elections, dated July 19, 2001, and the order, dated January 30, 2002 of the
Commission on Elections en banc are AFFIRMED.

SO ORDERED.

G.R. No. 120295 June 28, 1996

JUAN G. FRIVALDO, petitioner, 
vs.
COMMISSION ON ELECTIONS, and RAUL R. LEE, respondents.
G.R. No. 123755 June 28, 1996

RAUL R. LEE, petitioner, 
vs.
COMMISSION ON ELECTIONS and JUAN G. FRIVALDO, respondents.

PANGANIBAN, J.:p

The ultimate question posed before this Court in these twin cases is: Who should be declared the rightful governor of Sorsogon -

(i) Juan G. Frivaldo, who unquestionably obtained the highest number of votes in three successive
elections but who was twice declared by this Court to be disqualified to hold such office due to his
alien citizenship, and who now claims to have re-assumed his lost Philippine citizenship thru
repatriation;

(ii) Raul R. Lee, who was the second placer in the canvass, but who claims that the votes cast in
favor of Frivaldo should be considered void; that the electorate should be deemed to have
intentionally thrown away their ballots; and that legally, he secured the most number of valid votes;
or

(iii) The incumbent Vice-Governor, Oscar G. Deri, who obviously was not voted directly to the
position of governor, but who according to prevailing jurisprudence should take over the said post
inasmuch as, by the ineligibility of Frivaldo, a "permanent vacancy in the contested office has
occurred"?

In ruling for Frivaldo, the Court lays down new doctrines on repatriation, clarifies/reiterates/amplifies
existing jurisprudence on citizenship and elections, and upholds the superiority of substantial justice
over pure legalisms.

G.R. No. 123755

This is a special civil action under Rules 65 and 58 of the Rules of Court for certiorari and
preliminary injunction to review and annul a Resolution of the respondent Commission on Elections
(Comelec), First Division,  promulgated on December 19, 1995  and another Resolution of the
1 2

Comelec en banc promulgated February 23, 1996  denying petitioner's motion for reconsideration.
3

The Facts

On March 20, 1995, private respondent Juan G. Frivaldo filed his Certificate of Candidacy for the
office of Governor of Sorsogon in the May 8, 1995 elections. On March 23, 1995, petitioner Raul R.
Lee, another candidate, filed a petition  with the Comelec docketed as SPA No. 95-028 praying that
4

Frivaldo "be disqualified from seeking or holding any public office or position by reason of not yet
being a citizen of the Philippines", and that his Certificate of Candidacy be canceled. On May 1,
1995, the Second Division of the Comelec promulgated a Resolution  granting the petition with the 5

following disposition : 6

WHEREFORE, this Division resolves to GRANT the petition and declares that
respondent is DISQUALIFIED to run for the Office of Governor of Sorsogon on the
ground that he is NOT a citizen of the Philippines. Accordingly, respondent's
certificate of candidacy is canceled.

The Motion for Reconsideration filed by Frivaldo remained unacted upon until after the May 8, 1995
elections. So, his candidacy continued and he was voted for during the elections held on said date.
On May 11, 1995, the Comelec en banc  affirmed the aforementioned Resolution of the Second
7

Division.

The Provincial Board of Canvassers completed the canvass of the election returns and a Certificate
of Votes  dated May 27, 1995 was issued showing the following votes obtained by the candidates for
8

the position of Governor of Sorsogon:

Antonio H. Escudero, Jr. 51,060

Juan G. Frivaldo 73,440

Raul R. Lee 53,304

Isagani P. Ocampo 1,925

On June 9, 1995, Lee filed in said SPA No. 95-028, a (supplemental) petition  praying for his
9

proclamation as the duly-elected Governor of Sorsogon.

In an order  dated June 21, 1995, but promulgated according to the petition "only on June 29, 1995,"
10

the Comelec en banc directed "the Provincial Board of Canvassers of Sorsogon to reconvene for the
purpose of proclaiming candidate Raul Lee as the winning gubernatorial candidate in the province of
Sorsogon on June 29, 1995 . . ." Accordingly, at 8:30 in the evening of June 30, 1995, Lee was
proclaimed governor of Sorsogon.

On July 6, 1995, Frivaldo filed with the Comelec a new petition,  docketed as SPC No. 95-317,
11

praying for the annulment of the June 30, 1995 proclamation of Lee and for his own proclamation.
He alleged that on June 30, 1995, at 2:00 in the afternoon, he took his oath of allegiance as a citizen
of the Philippines after "his petition for repatriation under P.D. 725 which he filed with the Special
Committee on Naturalization in September 1994 had been granted". As such, when "the said order
(dated June 21, 1995) (of the Comelec) . . . was released and received by Frivaldo on June 30, 1995
at 5:30 o'clock in the evening, there was no more legal impediment to the proclamation (of Frivaldo)
as governor . . ." In the alternative, he averred that pursuant to the two cases of Labo
vs. Comelec,  the Vice-Governor - not Lee - should occupy said position of governor.
12

On December 19, 1995, the Comelec First Division promulgated the herein assailed
Resolution  holding that Lee, "not having garnered the highest number of votes," was not legally
13

entitled to be proclaimed as duly-elected governor; and that Frivaldo, "having garnered the highest
number of votes,
and . . . having reacquired his Filipino citizenship by repatriation on June 30, 1995 under the
provisions of Presidential Decree No. 725 . . . (is) qualified to hold the office of governor of
Sorsogon"; thus:

PREMISES CONSIDERED, the Commission (First Division), therefore RESOLVES


to GRANT the Petition.
Consistent with the decisions of the Supreme Court, the proclamation of Raul R. Lee
as Governor of Sorsogon is hereby ordered annulled, being contrary to law, he not
having garnered the highest number of votes to warrant his proclamation.

Upon the finality of the annulment of the proclamation of Raul R. Lee, the Provincial
Board of Canvassers is directed to immediately reconvene and, on the basis of the
completed canvass, proclaim petitioner Juan G. Frivaldo as the duly elected
Governor of Sorsogon having garnered the highest number of votes, and he having
reacquired his Filipino citizenship by repatriation on June 30, 1995 under the
provisions of Presidential Decree No. 725 and, thus, qualified to hold the office of
Governor of Sorsogon.

Conformably with Section 260 of the Omnibus Election Code (B.P. Blg. 881), the
Clerk of the Commission is directed to notify His Excellency the President of the
Philippines, and the Secretary of the Sangguniang Panlalawigan of the Province of
Sorsogon of this resolution immediately upon the due implementation thereof.

On December 26, 1995, Lee filed a motion for reconsideration which was denied by the Comelec en
banc in its Resolution   promulgated on February 23, 1996. On February 26, 1996, the present
14

petition was filed. Acting on the prayer for a temporary restraining order, this Court issued on
February 27, 1996 a Resolution which inter alia directed the parties "to maintain the status
quo prevailing prior to the filing of this petition."

The Issues in G.R. No. 123755

Petitioner Lee's "position on the matter at hand may briefly be capsulized in the following
propositions" :
15

First -- The initiatory petition below was so far insufficient in form and substance to
warrant the exercise by the COMELEC of its jurisdiction with the result that, in effect,
the COMELEC acted without jurisdiction in taking cognizance of and deciding said
petition;

Second -- The judicially declared disqualification of respondent was a continuing


condition and rendered him ineligible to run for, to be elected to and to hold the
Office of Governor;

Third -- The alleged repatriation of respondent was neither valid nor is the effect
thereof retroactive as to cure his ineligibility and qualify him to hold the Office of
Governor; and

Fourth -- Correctly read and applied, the Labo Doctrine fully supports the validity of
petitioner's proclamation as duly elected Governor of Sorsogon.

G.R. No. 120295

This is a petition to annul three Resolutions of the respondent Comelec, the first two of which are
also at issue in G.R. No. 123755, as follows:
1. Resolution  of the Second Division, promulgated on May 1, 1995, disqualifying
16

Frivaldo from running for governor of Sorsogon in the May 8, 1995 elections "on the
ground that he is not a citizen of the Philippines";

2. Resolution 7 of the Comelec en banc, promulgated on May 11, 1995; and


1

3. Resolution  of the Comelec en banc, promulgated also on May 11, 1995
18

suspending the proclamation of, among others, Frivaldo.

The Facts and the Issue

The facts of this case are essentially the same as those in G.R. No. 123755. However, Frivaldo
assails the above-mentioned resolutions on a different ground: that under Section 78 of the Omnibus
Election Code, which is reproduced hereinunder:

Sec. 78. Petition to deny due course or to cancel a certificate of candidacy. -- A


verified petition seeking to deny due course or to cancel a certificate of candidacy
may be filed by any person exclusively on the ground that any material
representation contained therein as required under Section 74 hereof is false. The
petition may be filed at any time not later than twenty-five days from the time of the
filing of the certificate of candidacy and shall be decided, after notice and
hearing, not later than fifteen days before the election. (Emphasis supplied.)

the Comelec had no jurisdiction to issue said Resolutions because they were not rendered
"within the period allowed by law" i.e., "not later than fifteen days before the election."

Otherwise stated, Frivaldo contends that the failure of the Comelec to act on the petition for
disqualification within the period of fifteen days prior to the election as provided by law is a
jurisdictional defect which renders the said Resolutions null and void.

By Resolution on March 12, 1996, the Court consolidated G.R. Nos. 120295 and 123755 since they
are intimately related in their factual environment and are identical in the ultimate question
raised, viz., who should occupy the position of governor of the province of Sorsogon.

On March 19, 1995, the Court heard oral argument from the parties and required them thereafter to
file simultaneously their respective memoranda.

The Consolidated Issues

From the foregoing submissions, the consolidated issues may be restated as follows:

1. Was the repatriation of Frivaldo valid and legal? If so, did it seasonably cure his lack of citizenship
as to qualify him to be proclaimed and to hold the Office of Governor? If not, may it be given
retroactive effect? If so, from when?

2. Is Frivaldo's "judicially declared" disqualification for lack of Filipino citizenship a continuing bar to
his eligibility to run for, be elected to or hold the governorship of Sorsogon?

3. Did the respondent Comelec have jurisdiction over the initiatory petition in SPC No. 95-317
considering that said petition is not "a pre-proclamation case, an election protest or a quo
warranto case"?
4. Was the proclamation of Lee, a runner-up in the election, valid and legal in light of existing
jurisprudence?

5. Did the respondent Commission on Elections exceed its jurisdiction in promulgating the assailed
Resolutions, all of which prevented Frivaldo from assuming the governorship of Sorsogon,
considering that they were not rendered within the period referred to in Section 78 of the Omnibus
Election Code, viz., "not later than fifteen days before the elections"?

The First Issue: Frivaldo's Repatriation

The validity and effectivity of Frivaldo's repatriation is the lis mota, the threshold legal issue in this
case. All the other matters raised are secondary to this.

The Local Government Code of 1991  expressly requires Philippine citizenship as a qualification for
19

elective local officials, including that of provincial governor, thus:

Sec. 39. Qualifications. -- (a) An elective local official must be a citizen of the
Philippines; a registered voter in the barangay, municipality, city, or province or, in
the case of a member of the sangguniang panlalawigan, sangguniang panlungsod,
or sangguniang bayan, the district where he intends to be elected; a resident therein
for at least one (1) year immediately preceding the day of the election; and able to
read and write Filipino or any other local language or dialect.

(b) Candidates for the position of governor, vice governor or member


of the sangguniang panlalawigan, or mayor, vice mayor or member of
the sangguniang panlungsod of highly urbanized cities must be at
least twenty-three (23) years of age on election day.

x x x           x x x          x x x

Inasmuch as Frivaldo had been declared by this Court  as a non-citizen, it is therefore incumbent
20

upon him to show that he has reacquired citizenship; in fine, that he possesses the qualifications
prescribed under the said statute (R.A. 7160).

Under Philippine law,  citizenship may be reacquired by direct act of Congress, by naturalization or
21

by repatriation. Frivaldo told this Court in G.R. No. 104654  and during the oral argument in this
22

case that he tried to resume his citizenship by direct act of Congress, but that the bill allowing him to
do so "failed to materialize, notwithstanding the endorsement of several members of the House of
Representatives" due, according to him, to the "maneuvers of his political rivals." In the same case,
his attempt at naturalization was rejected by this Court because of jurisdictional, substantial and
procedural defects.

Despite his lack of Philippine citizenship, Frivaldo was overwhelmingly elected governor by the
electorate of Sorsogon, with a margin of 27,000 votes in the 1988 elections, 57,000 in 1992, and
20,000 in 1995 over the same opponent Raul Lee. Twice, he was judicially declared a non-Filipino
and thus twice disqualified from holding and discharging his popular mandate. Now, he comes to us
a third time, with a fresh vote from the people of Sorsogon and a favorable decision from the
Commission on Elections to boot. Moreover, he now boasts of having successfully passed through
the third and last mode of reacquiring citizenship: by repatriation under P.D. No. 725, with no less
than the Solicitor General himself, who was the prime opposing counsel in the previous cases he
lost, this time, as counsel for co-respondent Comelec, arguing the validity of his cause (in addition to
his able private counsel Sixto S. Brillantes, Jr.). That he took his oath of allegiance under the
provisions of said Decree at 2:00 p.m. on June 30, 1995 is not disputed. Hence, he insists that he --
not Lee -- should have been proclaimed as the duly-elected governor of Sorsogon when the
Provincial Board of Canvassers met at 8:30 p.m. on the said date since, clearly and unquestionably,
he garnered the highest number of votes in the elections and since at that time, he already
reacquired his citizenship.

En contrario, Lee argues that Frivaldo's repatriation is tainted with serious defects, which we shall
now discuss in seriatim.

First, Lee tells us that P.D. No. 725 had "been effectively repealed", asserting that "then President
Corazon Aquino exercising legislative powers under the Transitory Provisions of the 1987
Constitution, forbade the grant of citizenship by Presidential Decree or Executive Issuances as the
same poses a serious and contentious issue of policy which the present government, in the exercise
of prudence and sound discretion, should best leave to the judgment of the first Congress under the
1987 Constitution", adding that in her memorandum dated March 27, 1987 to the members of the
Special Committee on Naturalization constituted for purposes of Presidential Decree No. 725,
President Aquino directed them "to cease and desist from undertaking any and all proceedings
within your functional area of responsibility as defined under Letter of Instructions (LOI) No. 270
dated April 11, 1975, as amended." 23

This memorandum dated March 27, 1987  cannot by any stretch of legal hermeneutics be construed
24

as a law sanctioning or authorizing a repeal of P.D. No. 725. Laws are repealed only by subsequent
ones   and a repeal may be express or implied. It is obvious that no express repeal was made
25

because then President Aquino in her memorandum -- based on the copy furnished us by Lee -- did
not categorically and/or impliedly state that P.D. 725 was being repealed or was being rendered
without any legal effect. In fact, she did not even mention it specifically by its number or text. On the
other hand, it is a basic rule of statutory construction that repeals by implication are not favored. An
implied repeal will not be allowed "unless it is convincingly and unambiguously demonstrated that
the two laws are clearly repugnant and patently inconsistent that they cannot co-exist". 26

The memorandum of then President Aquino cannot even be regarded as a legislative enactment, for
not every pronouncement of the Chief Executive even under the Transitory Provisions of the 1987
Constitution can nor should be regarded as an exercise of her law-making powers. At best, it could
be treated as an executive policy addressed to the Special Committee to halt the acceptance and
processing of applications for repatriation pending whatever "judgment the first Congress under the
1987 Constitution" might make. In other words, the former President did not repeal P.D. 725 but left
it to the first Congress -- once created -- to deal with the matter. If she had intended to repeal such
law, she should have unequivocally said so instead of referring the matter to Congress. The fact is
she carefully couched her presidential issuance in terms that clearly indicated the intention of "the
present government, in the exercise of prudence and sound discretion" to leave the matter of repeal
to the new Congress. Any other interpretation of the said Presidential Memorandum, such as is now
being proffered to the Court by Lee, would visit unmitigated violence not only upon statutory
construction but on common sense as well.

Second, Lee also argues that "serious congenital irregularities flawed the repatriation proceedings,"
asserting that Frivaldo's application therefor was "filed on June 29, 1995 . . . (and) was approved in
just one day or on June 30, 1995 . . .", which "prevented a judicious review and evaluation of the
merits thereof." Frivaldo counters that he filed his application for repatriation with the Office of the
President in Malacañang Palace on August 17, 1994. This is confirmed by the Solicitor General.
However, the Special Committee was reactivated only on June 8, 1995, when presumably the said
Committee started processing his application. On June 29, 1995, he filled up and re-submitted the
FORM that the Committee required. Under these circumstances, it could not be said that there was
"indecent haste" in the processing of his application.

Anent Lee's charge that the "sudden reconstitution of the Special Committee on Naturalization was
intended solely for the personal interest of respondent," 7 the Solicitor General explained during the
2

oral argument on March 19, 1996 that such allegation is simply baseless as there were many others
who applied and were considered for repatriation, a list of whom was submitted by him to this Court,
through a Manifestation  filed on April 3, 1996.
28

On the basis of the parties' submissions, we are convinced that the presumption of regularity in the
performance of official duty and the presumption of legality in the repatriation of Frivaldo have not
been successfully rebutted by Lee. The mere fact that the proceedings were speeded up is by itself
not a ground to conclude that such proceedings were necessarily tainted. After all, the requirements
of repatriation under P.D. No. 725 are not difficult to comply with, nor are they tedious and
cumbersome. In fact, P.D.
725  itself requires very little of an applicant, and even the rules and regulations to implement the
29

said decree were left to the Special Committee to promulgate. This is not unusual since, unlike in
naturalization where an alien covets a first-time entry into Philippine political life, in repatriation the
applicant is a former natural-born Filipino who is merely seeking to reacquire his previous
citizenship. In the case of Frivaldo, he was undoubtedly a natural-born citizen who openly and
faithfully served his country and his province prior to his naturalization in the United States -- a
naturalization he insists was made necessary only to escape the iron clutches of a dictatorship he
abhorred and could not in conscience embrace -- and who, after the fall of the dictator and the re-
establishment of democratic space, wasted no time in returning to his country of birth to offer once
more his talent and services to his people.

So too, the fact that ten other persons, as certified to by the Solicitor General, were granted
repatriation argues convincingly and conclusively against the existence of favoritism vehemently
posited by Raul Lee. At any rate, any contest on the legality of Frivaldo's repatriation should have
been pursued before the Committee itself, and, failing there, in the Office of the President, pursuant
to the doctrine of exhaustion of administrative remedies.

Third, Lee further contends that assuming the assailed repatriation to be valid, nevertheless it could
only be effective as at 2:00 p.m. of June 30, 1995 whereas the citizenship qualification prescribed by
the Local Government Code "must exist on the date of his election, if not when the certificate of
candidacy is filed," citing our decision in G.R. 104654  which held that "both the Local Government
30

Code and the Constitution require that only Philippine citizens can run and be elected to public
office." Obviously, however, this was a mere obiter as the only issue in said case was whether
Frivaldo's naturalization was valid or not -- and NOT the effective date thereof. Since the Court held
his naturalization to be invalid, then the issue of when an aspirant for public office should be a citizen
was NOT resolved at all by the Court. Which question we shall now directly rule on.

Under Sec. 39 of the Local Government Code, "(a)n elective local official must be:

* a citizen of the Philippines;

* a registered voter in the barangay, municipality, city, or province . . . where he


intends to be elected;

* a resident therein for at least one (1) year immediately preceding the day of the
election;
* able to read and write Filipino or any other local language or dialect.

* In addition, "candidates for the position of governor . . . must be at least twenty-


three (23) years of age on election day.

From the above, it will be noted that the law does not specify any particular date or time when the
candidate must possess citizenship, unlike that for residence (which must consist of at least one
year's residency immediately preceding the day of election) and age (at least twenty three years of
age on election day).

Philippine citizenship is an indispensable requirement for holding an elective public office,  and the
31

purpose of the citizenship qualification is none other than to ensure that no alien, i.e., no person
owing allegiance to another nation, shall govern our people and our country or a unit of territory
thereof. Now, an official begins to govern or to discharge his functions only upon his
proclamation and on the day the law mandates his term of office to begin. Since Frivaldo re-
assumed his citizenship on June 30, 1995 -- the very day  the term of office of governor (and other
32

elective officials) began -- he was therefore already qualified to be proclaimed, to hold such office
and to discharge the functions and responsibilities thereof as of said date. In short, at that time, he
was already qualified to govern his native Sorsogon. This is the liberal interpretation that should give
spirit, life and meaning to our law on qualifications consistent with the purpose for which such law
was enacted. So too, even from a literal (as distinguished from liberal) construction, it should be
noted that Section 39 of the Local Government Code speaks of "Qualifications" of "ELECTIVE
OFFICIALS", not of candidates. Why then should such qualification be required at the time of
election or at the time of the filing of the certificates of candidacies, as Lee insists? Literally, such
qualifications -- unless otherwise expressly conditioned, as in the case of age and residence --
should thus be possessed when the "elective [or elected] official" begins to govern, i.e., at the time
he is proclaimed and at the start of his term -- in this case, on June 30, 1995. Paraphrasing this
Court's ruling in Vasquez vs. Giap and Li Seng Giap & Sons,   if the purpose of the citizenship
33

requirement is to ensure that our people and country do not end up being governed by aliens, i.e.,
persons owing allegiance to another nation, that aim or purpose would not be thwarted but instead
achieved by construing the citizenship qualification as applying to the time of proclamation of the
elected official and at the start of his term.

But perhaps the more difficult objection was the one raised during the oral argument  to the effect
34

that the citizenship qualification should be possessed at the time the candidate (or for that matter the
elected official) registered as a voter. After all, Section 39, apart from requiring the official to be a
citizen, also specifies as another item of qualification, that he be a "registered voter". And, under the
law  a "voter" must be a citizen of the Philippines. So therefore, Frivaldo could not have been a voter
35

-- much less a validly registered one -- if he was not a citizen at the time of such registration.

The answer to this problem again lies in discerning the purpose of the requirement. If the law
intended the citizenship qualification to be possessed prior to election consistent with the
requirement of being a registered voter, then it would not have made citizenship a SEPARATE
qualification. The law abhors a redundancy. It therefore stands to reason that the law intended
CITIZENSHIP to be a qualification distinct from being a VOTER, even if being a voter presumes
being a citizen first. It also stands to reason that the voter requirement was included as another
qualification (aside from "citizenship"), not to reiterate the need for nationality but to require that the
official be registered as a voter IN THE AREA OR TERRITORY he seeks to govern, i.e., the law
states: "a registered voter in the barangay, municipality, city, or province . . . where he intends to be
elected." It should be emphasized that the Local Government Code requires an elective official to be
a registered voter. It does not require him to vote actually. Hence, registration -- not the actual voting
-- is the core of this "qualification". In other words, the law's purpose in this second requirement is to
ensure that the prospective official is actually registered in the area he seeks to govern -- and not
anywhere else.

Before this Court, Frivaldo has repeatedly emphasized -- and Lee has not disputed -- that he "was
and is a registered voter of Sorsogon, and his registration as a voter has been sustained as valid by
judicial declaration . . . In fact, he cast his vote in his precinct on May 8, 1995."
36

So too, during the oral argument, his counsel steadfastly maintained that "Mr. Frivaldo has always
been a registered voter of Sorsogon. He has voted in 1987, 1988, 1992, then he voted again in
1995. In fact, his eligibility as a voter was questioned, but the court dismissed (sic) his eligibility as a
voter and he was allowed to vote as in fact, he voted in all the previous elections including on May 8,
1995."  7
3

It is thus clear that Frivaldo is a registered voter in the province where he intended to be elected.

There is yet another reason why the prime issue of citizenship should be reckoned from the date of
proclamation, not necessarily the date of election or date of filing of the certificate of candidacy.
Section 253 of the Omnibus Election Code   gives any voter, presumably including the defeated
38

candidate, the opportunity to question the ELIGIBILITY (or the disloyalty) of a candidate. This is the
only provision of the Code that authorizes a remedy on how to contest before the Comelec an
incumbent's ineligibility arising from failure to meet the qualifications enumerated under Sec. 39 of
the Local Government Code. Such remedy of Quo Warranto can be availed of "within ten days after
proclamation" of the winning candidate. Hence, it is only at such time that the issue of ineligibility
may be taken cognizance of by the Commission. And since, at the very moment of Lee's
proclamation (8:30 p.m., June 30, 1995), Juan G. Frivaldo was already and indubitably a citizen,
having taken his oath of allegiance earlier in the afternoon of the same day, then he should have
been the candidate proclaimed as he unquestionably garnered the highest number of votes in the
immediately preceding elections and such oath had already cured his previous "judicially-declared"
alienage. Hence, at such time, he was no longer ineligible.

But to remove all doubts on this important issue, we also hold that the repatriation of Frivaldo
RETROACTED to the date of the filing of his application on August 17, 1994.

It is true that under the Civil Code of the Philippines,   "(l)aws shall have no retroactive effect, unless
39

the contrary is provided." But there are settled exceptions  to this general rule, such as when the
40

statute is CURATIVE or REMEDIAL in nature or when it CREATES NEW RIGHTS.

According to Tolentino,  curative statutes are those which undertake to cure errors and irregularities,
41

thereby validating judicial or administrative proceedings, acts of public officers, or private deeds and
contracts which otherwise would not produce their intended consequences by reason of some
statutory disability or failure to comply with some technical requirement. They operate on conditions
already existing, and are necessarily retroactive in operation. Agpalo,  on the other hand, says that
42

curative statutes are


"healing acts . . . curing defects and adding to the means of enforcing existing obligations . . . (and)
are intended to supply defects, abridge superfluities in existing laws, and curb certain evils. . . . By
their very nature, curative statutes are retroactive . . . (and) reach back to past events to correct
errors or irregularities and to render valid and effective attempted acts which would be otherwise
ineffective for the purpose the parties intended."

On the other hand, remedial or procedural laws, i.e., those statutes relating to remedies or modes of
procedure, which do not create new or take away vested rights, but only operate in furtherance of
the remedy or confirmation of such rights, ordinarily do not come within the legal meaning of a
retrospective law, nor within the general rule against the retrospective operation of statutes.43

A reading of P.D. 725 immediately shows that it creates a new right, and also provides for a new
remedy, thereby filling certain voids in our laws. Thus, in its preamble, P.D. 725 expressly
recognizes the plight of "many Filipino women (who) had lost their Philippine citizenship by marriage
to aliens" and who could not, under the existing law (C.A. No. 63, as amended) avail of repatriation
until "after the death of their husbands or the termination of their marital status" and who could
neither be benefitted by the 1973 Constitution's new provision allowing "a Filipino woman who
marries an alien to retain her Philippine citizenship . . ." because "such provision of the new
Constitution does not apply to Filipino women who had married aliens before said constitution took
effect." Thus, P.D. 725 granted a new right to these women -- the right to re-acquire Filipino
citizenship even during their marital coverture, which right did not exist prior to P.D. 725. On the
other hand, said statute also provided a new remedy and a new right in favor of other "natural born
Filipinos who (had) lost their Philippine citizenship but now desire to re-acquire Philippine
citizenship", because prior to the promulgation of P.D. 725 such former Filipinos would have had to
undergo the tedious and cumbersome process of naturalization, but with the advent of P.D. 725 they
could now re-acquire their Philippine citizenship under the simplified procedure of repatriation.

The Solicitor General  argues:


44

By their very nature, curative statutes are retroactive, (DBP vs. CA, 96 SCRA 342),
since they are intended to supply defects, abridge superfluities in existing laws (Del
Castillo vs. Securities and Exchange Commission, 96 Phil. 119) and curb certain
evils (Santos vs. Duata, 14 SCRA 1041).

In this case, P.D. No. 725 was enacted to cure the defect in the existing
naturalization law, specifically C.A. No. 63 wherein married Filipino women are
allowed to repatriate only upon the death of their husbands, and natural-born
Filipinos who lost their citizenship by naturalization and other causes faced the
difficulty of undergoing the rigid procedures of C.A. 63 for reacquisition of Filipino
citizenship by naturalization.

Presidential Decree No. 725 provided a remedy for the aforementioned legal
aberrations and thus its provisions are considered essentially remedial and curative.

In light of the foregoing, and prescinding from the wording of the preamble, it is unarguable that the
legislative intent was precisely to give the statute retroactive operation. "(A) retrospective operation
is given to a statute or amendment where the intent that it should so operate clearly appears from a
consideration of the act as a whole, or from the terms thereof."  It is obvious to the Court that the
45

statute was meant to "reach back" to those persons, events and transactions not otherwise covered
by prevailing law and jurisprudence. And inasmuch as it has been held that citizenship is a political
and civil right equally as important as the freedom of speech, liberty of abode, the right against
unreasonable searches and seizures and other guarantees enshrined in the Bill of Rights, therefore
the legislative intent to give retrospective operation to P.D. 725 must be given the fullest effect
possible. "(I)t has been said that a remedial statute must be so construed as to make it effect the
evident purpose for which it was enacted, so that if the reason of the statute extends to past
transactions, as well as to those in the future, then it will be so applied although the statute does not
in terms so direct, unless to do so would impair some vested right or violate some constitutional
guaranty."  This is all the more true of P.D. 725, which did not specify any restrictions on or delimit
46

or qualify the right of repatriation granted therein.


At this point, a valid question may be raised: How can the retroactivity of P.D. 725 benefit Frivaldo
considering that said law was enacted on June 5, 1975, while Frivaldo lost his Filipino citizenship
much later, on January 20, 1983, and applied for repatriation even later, on August 17, 1994?

While it is true that the law was already in effect at the time that Frivaldo became an American
citizen, nevertheless, it is not only the law itself (P.D. 725) which is to be given retroactive effect, but
even the repatriation granted under said law to Frivaldo on June 30, 1995 is to be deemed to have
retroacted to the date of his application therefor, August 17, 1994. The reason for this is simply that
if, as in this case, it was the intent of the legislative authority that the law should apply to past events
-- i.e., situations and transactions existing even before the law came into being -- in order to benefit
the greatest number of former Filipinos possible thereby enabling them to enjoy and exercise the
constitutionally guaranteed right of citizenship, and such legislative intention is to be given the fullest
effect and expression, then there is all the more reason to have the law apply in a retroactive or
retrospective manner to situations, events and transactions subsequent to the passage of such law.
That is, the repatriation granted to Frivaldo on June 30, 1995 can and should be made to take effect
as of date of his application. As earlier mentioned, there is nothing in the law that would bar this or
would show a contrary intention on the part of the legislative authority; and there is no showing that
damage or prejudice to anyone, or anything unjust or injurious would result from giving retroactivity
to his repatriation. Neither has Lee shown that there will result the impairment of any contractual
obligation, disturbance of any vested right or breach of some constitutional guaranty.

Being a former Filipino who has served the people repeatedly, Frivaldo deserves a liberal
interpretation of Philippine laws and whatever defects there were in his nationality should now be
deemed mooted by his repatriation.

Another argument for retroactivity to the date of filing is that it would prevent prejudice to applicants.
If P.D. 725 were not to be given retroactive effect, and the Special Committee decides not to act, i.e.,
to delay the processing of applications for any substantial length of time, then the former Filipinos
who may be stateless, as Frivaldo -- having already renounced his American citizenship -- was, may
be prejudiced for causes outside their control. This should not be. In case of doubt in the
interpretation or application of laws, it is to be presumed that the law-making body intended right and
justice to prevail.  7
4

And as experience will show, the Special Committee was able to process, act upon and grant
applications for repatriation within relatively short spans of time after the same were filed.  The fact
48

that such interregna were relatively insignificant minimizes the likelihood of prejudice to the
government as a result of giving retroactivity to repatriation. Besides, to the mind of the Court, direct
prejudice to the government is possible only where a person's repatriation has the effect of wiping
out a liability of his to the government arising in connection with or as a result of his being an alien,
and accruing only during the interregnum between application and approval, a situation that is not
present in the instant case.

And it is but right and just that the mandate of the people, already twice frustrated, should now
prevail. Under the circumstances, there is nothing unjust or iniquitous in treating Frivaldo's
repatriation as having become effective as of the date of his application, i.e., on August 17, 1994.
This being so, all questions about his possession of the nationality qualification -- whether at the
date of proclamation (June 30, 1995) or the date of election (May 8, 1995) or date of filing his
certificate of candidacy (March 20, 1995) would become moot.

Based on the foregoing, any question regarding Frivaldo's status as a registered voter would also be
deemed settled. Inasmuch as he is considered as having been repatriated -- i.e., his Filipino
citizenship restored -- as of August 17, 1994, his previous registration as a voter is likewise deemed
validated as of said date.

It is not disputed that on January 20, 1983 Frivaldo became an American. Would the retroactivity of
his repatriation not effectively give him dual citizenship, which under Sec. 40 of the Local
Government Code would disqualify him "from running for any elective local position?"  We answer
49

this question in the negative, as there is cogent reason to hold that Frivaldo was really STATELESS
at the time he took said oath of allegiance and even before that, when he ran for governor in 1988.
In his Comment, Frivaldo wrote that he "had long renounced and had long abandoned his American
citizenship -- long before May 8, 1995. At best, Frivaldo was stateless in the interim -- when he
abandoned and renounced his US citizenship but before he was repatriated to his Filipino
citizenship."
50

On this point, we quote from the assailed Resolution dated December 19, 1995: 51

By the laws of the United States, petitioner Frivaldo lost his American citizenship
when he took his oath of allegiance to the Philippine Government when he ran for
Governor in 1988, in 1992, and in 1995. Every certificate of candidacy contains an
oath of allegiance to the Philippine Government."

These factual findings that Frivaldo has lost his foreign nationality long before the elections of 1995
have not been effectively rebutted by Lee. Furthermore, it is basic that such findings of the
Commission are conclusive upon this Court, absent any showing of capriciousness or arbitrariness
or
abuse.52

The Second Issue: Is Lack of Citizenship


a Continuing Disqualification?

Lee contends that the May 1, 1995 Resolution   of the Comelec Second Division in SPA No. 95-028
53

as affirmed in toto by Comelec En Banc in its Resolution of May 11, 1995 "became final and
executory after five (5) days or on May 17, 1995, no restraining order having been issued by this
Honorable Court.  Hence, before Lee "was proclaimed as the elected governor on June 30, 1995,
54

there was already a final and executory judgment disqualifying" Frivaldo. Lee adds that this Court's
two rulings (which Frivaldo now concedes were legally "correct") declaring Frivaldo an alien have
also become final and executory way before the 1995 elections, and these "judicial pronouncements
of his political status as an American citizen absolutely and for all time disqualified (him) from
running for, and holding any public office in the Philippines."

We do not agree.

It should be noted that our first ruling in G.R. No. 87193 disqualifying Frivaldo was rendered in
connection with the 1988 elections while that in G.R. No. 104654 was in connection with the 1992
elections. That he was disqualified for such elections is final and can no longer be changed. In the
words of the respondent Commission (Second Division) in its assailed Resolution: 55

The records show that the Honorable Supreme Court had decided that Frivaldo was
not a Filipino citizen and thus disqualified for the purpose of the 1988 and 1992
elections. However, there is no record of any "final judgment" of the disqualification
of Frivaldo as a candidate for the May 8, 1995 elections. What the Commission said
in its Order of June 21, 1995 (implemented on June 30, 1995), directing the
proclamation of Raul R. Lee, was that Frivaldo was not a Filipino citizen "having
been declared by the Supreme Court in its Order dated March 25, 1995, not a citizen
of the Philippines." This declaration of the Supreme Court, however, was in
connection with the 1992 elections.

Indeed, decisions declaring the acquisition or denial of citizenship cannot govern a person's future
status with finality. This is because a person may subsequently reacquire, or for that matter lose, his
citizenship under any of the modes recognized by law for the purpose. Hence, in Lee
vs. Commissioner of Immigration,  we held:
56

Everytime the citizenship of a person is material or indispensable in a judicial or


administrative case, whatever the corresponding court or administrative authority
decides therein as to such citizenship is generally not considered res judicata, hence
it has to be threshed out again and again, as the occasion demands.

The Third Issue: Comelec's Jurisdiction


Over The Petition in SPC No. 95-317

Lee also avers that respondent Comelec had no jurisdiction to entertain the petition in SPC No. 95-
317 because the only "possible types of proceedings that may be entertained by the Comelec are a
pre-proclamation case, an election protest or a quo warranto case". Again, Lee reminds us that he
was proclaimed on June 30, 1995 but that Frivaldo filed SPC No. 95-317 questioning his (Lee's)
proclamation only on July 6, 1995 -- "beyond the 5-day reglementary period." Hence, according to
him, Frivaldo's "recourse was to file either an election protest or a quo warranto action."

This argument is not meritorious. The Constitution 7 has given the Comelec ample power to
5

"exercise exclusive original jurisdiction over all contests relating to the elections, returns and
qualifications of all elective . . . provincial . . . officials." Instead of dwelling at length on the various
petitions that Comelec, in the exercise of its constitutional prerogatives, may entertain, suffice it to
say that this Court has invariably recognized the Commission's authority to hear and decide petitions
for annulment of proclamations -- of which SPC No. 95-317 obviously is one.  Thus, in Mentang
58

vs. COMELEC,  we ruled:


59

The petitioner argues that after proclamation and assumption of office, a pre-
proclamation controversy is no longer viable. Indeed, we are aware of cases holding
that pre-proclamation controversies may no longer be entertained by the COMELEC
after the winning candidate has been proclaimed. (citing Gallardo vs. Rimando, 187
SCRA 463; Salvacion vs. COMELEC, 170 SCRA 513; Casimiro vs. COMELEC, 171
SCRA 468.) This rule, however, is premised on an assumption that the proclamation
is no proclamation at all and the proclaimed candidate's assumption of office cannot
deprive the COMELEC of the power to make such declaration of nullity.
(citing Aguam vs. COMELEC, 23 SCRA 883; Agbayani vs. COMELEC, 186 SCRA
484.)

The Court however cautioned that such power to annul a proclamation must "be done within ten (10)
days following the proclamation." Inasmuch as Frivaldo's petition was filed only six (6) days after
Lee's proclamation, there is no question that the Comelec correctly acquired jurisdiction over the
same.

The Fourth Issue: Was Lee's Proclamation Valid?

Frivaldo assails the validity of the Lee proclamation. We uphold him for the following reasons:
First. To paraphrase this Court in Labo vs. COMELEC,  "the fact remains that he (Lee) was not the
60

choice of the sovereign will," and in Aquino vs. COMELEC,  Lee is "a second placer, . . . just that, a
61

second placer."

In spite of this, Lee anchors his claim to the governorship on the pronouncement of this Court in the
aforesaid Labo  case, as follows:
62

The rule would have been different if the electorate fully aware in fact and in law of a
candidate's disqualification so as to bring such awareness within the realm of
notoriety, would nonetheless cast their votes in favor of the ineligible candidate. In
such case, the electorate may be said to have waived the validity and efficacy of their
votes by notoriously misapplying their franchise or throwing away their votes, in
which case, the eligible candidate obtaining the next higher number of votes may be
deemed elected.

But such holding is qualified by the next paragraph, thus:

But this is not the situation obtaining in the instant dispute. It has not been shown,
and none was alleged, that petitioner Labo was notoriously known as an ineligible
candidate, much less the electorate as having known of such fact. On the contrary,
petitioner Labo was even allowed by no less than the Comelec itself in its resolution
dated May 10, 1992 to be voted for the office of the city Payor as its resolution dated
May 9, 1992 denying due course to petitioner Labo's certificate of candidacy had not
yet become final and subject to the final outcome of this case.

The last-quoted paragraph in Labo, unfortunately for Lee, is the ruling appropriate in this case
because Frivaldo was in 1995 in an identical situation as Labo was in 1992 when the Comelec's
cancellation of his certificate of candidacy was not yet final on election day as there was in both
cases a pending motion for reconsideration, for which reason Comelec issued an (omnibus)
resolution declaring that Frivaldo (like Labo in 1992) and several others can still be voted for in the
May 8, 1995 election, as in fact, he was.

Furthermore, there has been no sufficient evidence presented to show that the electorate of
Sorsogon was "fully aware in fact and in law" of Frivaldo's alleged disqualification as to "bring such
awareness within the realm of notoriety;" in other words, that the voters intentionally wasted their
ballots knowing that, in spite of their voting for him, he was ineligible. If Labo has any relevance at
all, it is that the vice-governor -- and not Lee -- should be pro- claimed, since in losing the election,
Lee was, to paraphrase Labo again, "obviously not the choice of the people" of Sorsogon. This is the
emphatic teaching of Labo:

The rule, therefore, is: the ineligibility of a candidate receiving majority votes does not
entitle the eligible candidate receiving the next highest number of votes to be
declared elected. A minority or defeated candidate cannot be deemed elected to the
office.

Second. As we have earlier declared Frivaldo to have seasonably reacquired his citizenship and
inasmuch as he obtained the highest number of votes in the 1995 elections, he -- not Lee -- should
be proclaimed. Hence, Lee's proclamation was patently erroneous and should now be corrected.

The Fifth Issue: Is Section 78 of the


Election Code Mandatory?
In G.R. No. 120295, Frivaldo claims that the assailed Resolution of the Comelec (Second Division)
dated May 1, 1995 and the confirmatory en banc Resolution of May 11, 1995 disqualifying him for
want of citizenship should be annulled because they were rendered beyond the fifteen (15) day
period prescribed by Section 78, of the Omnibus Election Code which reads as follows:

Sec. 78. Petition to deny due course or to cancel a certificate of candidacy. -- A


verified petition seeking to deny due course or to cancel a certificate of candidacy
may be filed by any person exclusively on the ground that any material
representation contained therein as required under Section 74 hereof is false. The
petition may be filed at any time not later than twenty-five days from the time of the
filing of the certificate of candidacy and shall be decided after notice and hearing, not
later than fifteen days before the election. (Emphasis supplied.)

This claim is now moot and academic inasmuch as these resolutions are deemed superseded by the
subsequent ones issued by the Commission (First Division) on December 19, 1995, affirmed en
banc  on February 23, 1996; which both upheld his election. At any rate, it is obvious that Section 78
63

is merely directory as Section 6 of R.A. No. 6646 authorizes the Commission to try and decide
petitions for disqualifications even after the elections, thus:

Sec. 6. Effect of Disqualification Case. -- Any candidate who has been declared by
final judgment to be disqualified shall not be voted for, and the votes cast for him
shall not be counted. If for any reason a candidate is not declared by final judgment
before an election to be disqualified and he is voted for and receives the winning
number of votes in such election, the Court or Commission shall continue with the
trial and hearing of the action, inquiry or protest and upon motion of the complainant
or any intervenor, may during the pendency thereof order the suspension of the
proclamation of such candidate whenever the evidence of his guilt is strong.
(emphasis supplied)

Refutation of
Mr. Justice Davide's Dissent

In his dissenting opinion, the esteemed Mr. Justice Hilario G. Davide, Jr. argues that President
Aquino's memorandum dated March 27, 1987 should be viewed as a suspension (not a repeal, as
urged by Lee) of P.D. 725. But whether it decrees a suspension or a repeal is a purely academic
distinction because the said issuance is not a statute that can amend or abrogate an existing law.
The existence and subsistence of P.D. 725 were recognized in the first Frivaldo case;  viz., "(u)nder
64

CA No. 63 as amended by CA No. 473 and P.D. No. 725, Philippine citizenship maybe reacquired


by . . . repatriation". He also contends that by allowing Frivaldo to register and to remain as a
registered voter, the Comelec and in effect this Court abetted a "mockery" of our two previous
judgments declaring him a non-citizen. We do not see such abetting or mockery. The retroactivity of
his repatriation, as discussed earlier, legally cured whatever defects there may have been in his
registration as a voter for the purpose of the 1995 elections. Such retroactivity did not change his
disqualifications in 1988 and 1992, which were the subjects of such previous rulings.

Mr. Justice Davide also believes that Quo Warranto is not the sole remedy to question the ineligibility
of a candidate, citing the Comelec's authority under Section 78 of the Omnibus Election Code
allowing the denial of a certificate of candidacy on the ground of a false material representation
therein as required by Section 74. Citing Loong, he then states his disagreement with our holding
that Section 78 is merely directory. We really have no quarrel. Our point is that Frivaldo was in error
in his claim in G.R. No. 120295 that the Comelec Resolutions promulgated on May 1, 1995 and May
11, 1995 were invalid because they were issued "not later than fifteen days before the election" as
prescribed by Section 78. In dismissing the petition in G.R. No. 120295, we hold that the Comelec
did not commit grave abuse of discretion because "Section 6 of R.A. 6646 authorizes the Comelec to
try and decide disqualifications even after the elections." In spite of his disagreement with us on this
point, i.e., that Section 78 "is merely directory", we note that just like us, Mr. Justice Davide
nonetheless votes to "DISMISS G.R. No. 120295". One other point. Loong, as quoted in the dissent,
teaches that a petition to deny due course under Section 78 must be filed within the 25-day period
prescribed therein. The present case however deals with the period during which the Comelec
may decide such petition. And we hold that it may be decided even after the fifteen day period
mentioned in Section 78. Here, we rule that a decision promulgated by the Comelec even after the
elections is valid but Loong held that a petition filed beyond the 25-day period is out of time. There is
no inconsistency nor conflict.

Mr. Justice Davide also disagrees with the Court's holding that, given the unique factual
circumstances of Frivaldo, repatriation may be given retroactive effect. He argues that such
retroactivity "dilutes" our holding in the first Frivaldo case. But the first (and even the second
Frivaldo) decision did not directly involve repatriation as a mode of acquiring citizenship. If we may
repeat, there is no question that Frivaldo was not a Filipino for purposes of determining his
qualifications in the 1988 and 1992 elections. That is settled. But his supervening repatriation has
changed his political status -- not in 1988 or 1992, but only in the 1995 elections.

Our learned colleague also disputes our holding that Frivaldo was stateless prior to his repatriation,
saying that "informal renunciation or abandonment is not a ground to lose American citizenship".
Since our courts are charged only with the duty of determining who are Philippine nationals, we
cannot rule on the legal question of who are or who are not Americans. It is basic in international law
that a State determines ONLY those who are its own citizens -- not who are the citizens of other
countries.  The issue here is: the Comelec made a finding of fact that Frivaldo was stateless and
65

such finding has not been shown by Lee to be arbitrary or whimsical. Thus, following settled case
law, such finding is binding and final.

The dissenting opinion also submits that Lee who lost by chasmic margins to Frivaldo in all three
previous elections, should be declared winner because "Frivaldo's ineligibility for being an American
was publicly known". First, there is absolutely no empirical evidence for such "public" knowledge.
Second, even if there is, such knowledge can be truepost facto only of the last two previous
elections. Third, even the Comelec and now this Court were/are still deliberating on his nationality
before, during and after the 1995 elections. How then can there be such "public" knowledge?

Mr. Justice Davide submits that Section 39 of the Local Government Code refers to the qualifications
of electivelocal officials, i.e., candidates, and not elected officials, and that the citizenship
qualification [under par. (a) of that section] must be possessed by candidates, not merely at the
commencement of the term, but by election day at the latest. We see it differently. Section 39, par.
(a) thereof speaks of "elective local official" while par. (b) to (f) refer to "candidates". If the
qualifications under par. (a) were intended to apply to "candidates" and not elected officials, the
legislature would have said so, instead of differentiating par. (a) from the rest of the paragraphs.
Secondly, if Congress had meant that the citizenship qualification should be possessed at election
day or prior thereto, it would have specifically stated such detail, the same way it did in pars. (b) to (f)
far other qualifications of candidates for governor, mayor, etc.

Mr. Justice Davide also questions the giving of retroactive effect to Frivaldo's repatriation on the
ground, among others, that the law specifically provides that it is only after taking the oath of
allegiance that applicants shall be deemed to have reacquired Philippine citizenship. We do not
question what the provision states. We hold however that the provision should be understood
thus: that after taking the oath of allegiance the applicant is deemed to have reacquired Philippine
citizenship, which reacquisition (or repatriation) is deemed for all purposes and intents to have
retroacted to the date of his application therefor.

In any event, our "so too" argument regarding the literal meaning of the word "elective" in reference
to Section 39 of the Local Authority Code, as well as regarding Mr. Justice Davide's thesis that the
very wordings of P.D. 725 suggest non-retroactivity, were already taken up rather extensively earlier
in this Decision.

Mr. Justice Davide caps his paper with a clarion call: "This Court must be the first to uphold the Rule
of Law." We agree -- we must all follow the rule of law. But that is NOT the issue here. The issue
is how should the law be interpreted and applied in this case so it can be followed, so it can rule!

At balance, the question really boils down to a choice of philosophy and perception of how to
interpret and apply laws relating to elections: literal or liberal; the letter or the spirit, the naked
provision or its ultimate purpose; legal syllogism or substantial justice; in isolation or in the context of
social conditions; harshly against or gently in favor of the voters' obvious choice. In applying election
laws, it would be far better to err in favor of popular sovereignty than to be right in complex but little
understood legalisms. Indeed, to inflict a thrice rejected candidate upon the electorate of Sorsogon
would constitute unmitigated judicial tyranny and an unacceptable assault upon this Court's
conscience.

EPILOGUE

In sum, we rule that the citizenship requirement in the Local Government Code is to be possessed
by an elective official at the latest as of the time he is proclaimed and at the start of the term of office
to which he has been elected. We further hold P.D. No. 725 to be in full force and effect up to the
present, not having been suspended or repealed expressly nor impliedly at any time, and Frivaldo's
repatriation by virtue thereof to have been properly granted and thus valid and effective. Moreover,
by reason of the remedial or curative nature of the law granting him a new right to resume his
political status and the legislative intent behind it, as well as his unique situation of having been
forced to give up his citizenship and political aspiration as his means of escaping a regime he
abhorred, his repatriation is to be given retroactive effect as of the date of his application therefor,
during the pendency of which he was stateless, he having given up his U.S. nationality. Thus, in
contemplation of law, he possessed the vital requirement of Filipino citizenship as of the start of the
term of office of governor, and should have been proclaimed instead of Lee. Furthermore, since his
reacquisition of citizenship retroacted to August 17, 1994, his registration as a voter of Sorsogon is
deemed to have been validated as of said date as well. The foregoing, of course, are precisely
consistent with our holding that lack of the citizenship requirement is not a continuing disability or
disqualification to run for and hold public office. And once again, we emphasize herein our previous
rulings recognizing the Comelec's authority and jurisdiction to hear and decide petitions for
annulment of proclamations.

This Court has time and again liberally and equitably construed the electoral laws of our country to
give fullest effect to the manifest will of our people,  for in case of doubt, political laws must be
66

interpreted to give life and spirit to the popular mandate freely expressed through the ballot.
Otherwise stated, legal niceties and technicalities cannot stand in the way of the sovereign will.
Consistently, we have held:

. . . (L)aws governing election contests must be liberally construed to the end that the
will of the people in the choice of public officials may not be defeated by mere
technical objections (citations omitted). 7
6
The law and the courts must accord Frivaldo every possible protection, defense and refuge, in
deference to the popular will. Indeed, this Court has repeatedly stressed the importance of giving
effect to the sovereign will in order to ensure the survival of our democracy. In any action involving
the possibility of a reversal of the popular electoral choice, this Court must exert utmost effort to
resolve the issues in a manner that would give effect to the will of the majority, for it is merely sound
public policy to cause elective offices to be filled by those who are the choice of the majority. To
successfully challenge a winning candidate's qualifications, the petitioner must clearly demonstrate
that the ineligibility is so patently antagonistic  to constitutional and legal principles that overriding
68

such ineligibility and thereby giving effect to the apparent will of the people, would ultimately create
greater prejudice to the very democratic institutions and juristic traditions that our Constitution and
laws so zealously protect and promote. In this undertaking, Lee has miserably failed.

In Frivaldo's case. it would have been technically easy to find fault with his cause. The Court could
have refused to grant retroactivity to the effects of his repatriation and hold him still ineligible due to
his failure to show his citizenship at the time he registered as a voter before the 1995 elections. Or, it
could have disputed the factual findings of the Comelec that he was stateless at the time of
repatriation and thus hold his consequent dual citizenship as a disqualification "from running for any
elective local position." But the real essence of justice does not emanate from quibblings over
patchwork legal technicality. It proceeds from the spirit's gut consciousness of the dynamic role of
law as a brick in the ultimate development of the social edifice. Thus, the Court struggled against
and eschewed the easy, legalistic, technical and sometimes harsh anachronisms of the law in order
to evoke substantial justice in the larger social context consistent with Frivaldo's unique situation
approximating venerability in Philippine political life. Concededly, he sought American citizenship
only to escape the clutches of the dictatorship. At this stage, we cannot seriously entertain any doubt
about his loyalty and dedication to this country. At the first opportunity, he returned to this land, and
sought to serve his people once more. The people of Sorsogon overwhelmingly voted for him three
times. He took an oath of allegiance to this Republic every time he filed his certificate of candidacy
and during his failed naturalization bid. And let it not be overlooked, his demonstrated tenacity and
sheer determination to re-assume his nationality of birth despite several legal set-backs speak more
loudly, in spirit, in fact and in truth than any legal technicality, of his consuming intention and burning
desire to re-embrace his native Philippines even now at the ripe old age of 81 years. Such loyalty to
and love of country as well as nobility of purpose cannot be lost on this Court of justice and equity.
Mortals of lesser mettle would have given up. After all, Frivaldo was assured of a life of ease and
plenty as a citizen of the most powerful country in the world. But he opted, nay, single-mindedly
insisted on returning to and serving once more his struggling but beloved land of birth. He therefore
deserves every liberal interpretation of the law which can be applied in his favor. And in the final
analysis, over and above Frivaldo himself, the indomitable people of Sorsogon most certainly
deserve to be governed by a leader of their overwhelming choice.

WHEREFORE, in consideration of the foregoing:

(1) The petition in G.R. No. 123755 is hereby DISMISSED. The assailed Resolutions of the
respondent Commission are AFFIRMED.

(2) The petition in G.R. No. 120295 is also DISMISSED for being moot and academic. In any event,
it has no merit.

No costs.

SO ORDERED.
G.R. No. 132244 September 14, 1999

GERARDO ANGAT, petitioner, 
vs.
REPUBLIC OF THE PHILIPPINES, respondent.

VITUG, J.:

The instant petition for review under Rule 45 assails the orders, dated 22 September 1997 and 29
December 1997, issued by the Regional Trial Court ("RTC") of Marikina City in Case No. N-96-03-
MK, entitled "in the Matter of the Petition of Gerardo Angat y Legaspi to be Re-admitted as a Citizen
of the Philippines under Commonwealth Act No. 63, as amended, and Republic Act ("R.A.") No. 965
and 263[0]."

Petitioner Gerardo Angat was a natural born citizen of the Philippines until he lost his citizenship by
naturalization in the United States of America. Now residing at No. 69 New York Street, Provident
Village, Marikina City, Angat filed on 11 March 1996 before the RTC of Marikina City, Branch 272, a
petition to regain his Status as a citizen of the Philippines under Commonwealth Act No. 63,
Republic Act No. 965 and Republic Act No. 2630 (docketed as N-96-03-MK). In his petition,
"applying for naturalization," he averred that —

FIRST. — His full name is GERARDO LEGASPI ANGAT. Copy of his latest picture is
hereto attached and made an integral part of this petition.

SECOND. — His present place of residence is #69 New York St., Provident Village,
Marikina, Metro Manila and his former residence was in Las Vegas, U.S. 1âwphi1.nêt

THIRD. — His trade or profession is in buy and sell and managing the properties of
his parents which he has been engaged since his arrival here in the Philippines.

FOURTH. — He was born on the 22nd day of June 1954 at Tondo, Manila. He was
formerly a citizen of the Philippines. He lost his Philippine citizenship by
naturalization in a foreign country. He is at present a citizen or subject of the United
States of America. Copy of his birth certificate is hereto attached as Annex "A."

FIFTH. — He is newly married to Zenaida Lim who was born in Tondo, Manila and
now resides at petitioner's residence at Marikina, Metro Manila. Copy of their
marriage contract is hereto attached as Annex "B."

SIXTH. — He returned to the Philippines from the United States of America in 1991.
Copy of his alien registration is hereto attached as Annex "C."

SEVENTH. — He has the qualifications required by Commonwealth Act No. 63 as


amended, and Republic Act Nos. 965 and 2639 to reacquire Philippine citizenship,
and possesses none of the disqualification prescribed in Commonwealth Act No.
473. He has resided in the Philippines at least six months immediately preceding the
date of this petition, to wit: since 1991. He has conducted himself in a proper and
irreproachable manner during the entire period of his residence in the Philippines, in
his relations with the constituted government as well as with the community in which
he is living.

EIGHT. — He is not opposed to an organized government or affiliated with any


association or group of persons who uphold and teach doctrines opposing all
organized government. He is not defending or teaching the necessity or propriety of
violence, personal assault or assassination for the success and predominance of
men's ideas. He is not a polygamist or believer in the practice of polygamy. He has
not been convicted of any crime involving moral turpitude. He is not suffering from
any mental alienation or incurable contagious disease. The nation of which he is a
citizen or subject is not at war with the Philippines.

NINTH. — It is his intention to reacquire Philippine citizenship and to renounce


absolutely and forever all allegiance and fidelity to any foreign prince, potentate,
state, or sovereignty, and particularly to the United State of America to which at this
time he is a citizen. 1

On 30 April 1996, the trial court, through the branch clerk of court, issued a notice setting the case for
initial hearing on 27 January 1997 2 which, along with the petition and its annexes, was received by the
Office of the Solicitor General ("OSG") on 10 May 1996.

On 13 June 1996, petitioner sought to be allowed to take his oath of allegiance to the Republic of the
Philippines pursuant to R.A. 8171. The motion was denied by the trial judge in his order of 12 July
1996. Another motion filed by petitioner on 13 August 1996 to have the denial reconsidered was
found to be meritorious by the court a quo in an order, dated 20 September 1996, which stated,
among other things, that —

A close scrutiny of R.A. 8171 shows that petitioner is entitled to the benefits of the
said law considering that herein petitioner is a natural born Filipino citizen who lost
his citizenship by naturalization in a foreign country. The petition and motion of the
petitioner to take his oath of allegiance to the Republic of the Philippines likewise
show that the petitioner possesses all the qualifications and none of the
disqualifications under R.A. 8171. 3

Concluding, the court ruled:

WHEREFORE, foregoing premises considered, the Order of the Court dated July 12,
1996 is hereby set aside. The petitioner is ordered to take his oath of allegiance to
the Republic of the Philippines pursuant to R.A. 8171 before the undersigned on
October 03, 1996 at 11:00 in the morning.

SO ORDERED. 4

After taking his Oath of Allegiance on 03 October 1996, another order was issued by the trial judge on 04
October 1996 to the following effect; viz:

After the oath of allegiance to the Republic of the Philippines had been taken by the
petitioner, Gerardo Angat y Legaspi before the undersigned, the petitioner is hereby
repatriated and declared as citizen of the Republic of the Philippines pursuant to
Republic Act No. 8171.
The Bureau of Immigration is ordered to cancel the pertinent alien certificate of
registration and issue the certificate of identification as Filipino citizen to the
petitioner upon the finality of this order.

Likewise, let a copy of this Order be registered in the Local Civil Registry of the
Municipality of Marikina, Metro Manila and the General Civil Registrar, Sta. Mesa,
Manila, after its finality.

SO ORDERED. 5

On 19 March 1997, a Manifestation and Motion (virtually a motion for reconsideration) filed by the OSG
asserted that the petition itself should have been dismissed by the court a quo for lack of jurisdiction
because the proper forum for it was the Special Committee on Naturalization consistently with
Administrative Order No. 285 ("AO 285"), dated 22 August 1996, issued by President Fidel V. Ramos. AO
285 had tasked the Special Committee on Naturalization to be the implementing agency of R.A 8171. The
motion was found to be well taken by the trial court; thus, in an order, dated 22 September 1997, it
adjudged:

This resolves the Manifestation and Motion filed by the Office of the Solicitor General
on March 19, 1997.

The motion alleges that pursuant to Administrative Order No. 285 dated August 22,
1996 issued by President Fidel V. Ramos, any person desirous of repatriating or
reacquiring Filipino citizenship pursuant to R.A. 8171 shall file a petition with the
Special Committee on Naturalization, which is composed of the Solicitor General as
Chairman, the Undersecretary of Foreign Affairs and the Director-General of the
National Intelligence Coordinating Agency, as members, which shall process the
application; that if their applications are approved they shall take the necessary oath
of allegiance to the Republic of the Philippines, affect which they shall be deemed to
have reacquired their Philippine citizenship and the Commission of Immigration and
Deportation shall thereupon cancel their certificate of registration.

The motion prays that the herein petition be dismissed on the ground that the same
should be filed with the Special Committee on Naturalization.

The records show that on September 20, 1996, the Court granted the herein petition
and as a consequence thereof, the petitioner Gerardo Angat y Legaspi took his oath
of allegiance to the Republic of the Philippines before the Presiding Judge of this
Court on October 03, 1996 and on October 04, 1996, the petitioner was ordered
repatriated and declared as citizen of the Philippines.

On February 21, 1997, the Office of the Solicitor General entered its appearance as
counsel of the State in the subject petition and on March 19, 1997 filed the herein
manifestation and motion.

The allegations in the manifestation and motion of the Office of the Solicitor General
clearly shows that this Court has no jurisdiction over the herein petition as the same
falls within the jurisdiction of the Special Committee on Naturalization. Considering
that this court has no jurisdiction over this case, the order granting the same is
therefore null and void.
WHEREFORE, foregoing premises considered, the motion to dismiss filed by the
Office of the Solicitor General is hereby granted. The orders of this Court dated
September 20, 1996 and October 04, 1996 are hereby set aside and the herein
petition is ordered DISMISSED on the ground of lack of jurisdiction without prejudice
to its re-filing before the Special Committee on Naturalization.

SO ORDERED. 6

A motion for reconsideration, filed by petitioner on 13 October 1997, questioned the aforequoted order
asservating that since his petition was filed on 14 March 1996, or months before the Special Committee
on Naturalization was constituted by the President under AO 285 on 22 August 1996, the court a quo had
the authority to take cognizance of the case.

In the Order, dated 29 December 1997, the trial judge denied the motion for reconsideration.

The instant appeal by certiorari under Rule 45 of the 1997 Rules of Procedure submits the lone
assignment of error that —

The Regional Trial Court (has) seriously erred in dismissing the petition by giving
retroactive effect to Administrative Order No. 285, absent a provision on Retroactive
Application.

Petitioner would insist that the trial court had jurisdiction over his petition for naturalization 7 filed on
11 March 1996, and that he had acquired a vested right as a repatriated citizen of the Philippines when
the court declared him repatriated following the order, dated 20 September 1996, allowing him to take an
oath of allegiance to the Republic of the Philippines which was, in fact, administered to him on 03 October
1996.

The contention is not meritorious.

R.A. No. 8171, which has lapsed into law on 23 October 1995, is an act providing for the repatriation
(a) of Filipino women who have lost their Philippine citizenship by marriage to aliens and (b) of
natural-born Filipinos who have lost their Philippine citizenship on account or political or economic
necessity. The pertinent provisions of the law read:

Sec. 1. Filipino women who have lost their Philippine citizenship by marriage to
aliens and natural-born Filipinos who have lost their Philippine citizenship, including
their minor children, on account of political or economic necessity, may reacquire
Philippine citizenship through repatriation in the manner provided in Section 4 of
Commonwealth Act No. 631, as amended: Provided, That the applicant is not a:

(1) Person opposed to organized government or affiliated with any association or


group of persons who uphold and teach doctrines opposing organized government;

(2) Person defending or teaching the necessity or propriety of violence, personal


assault, or association for the predominance of their ideas;

(3) Person convicted of crimes involving moral turpitude: or

(4) Person suffering from mental alienation or incurable contagious diseases.


Sec. 2. Repatriation shall be effected by taking the necessary oath of allegiance to
the Republic of the Philippines and registration in the proper civil registry and in the
Bureau of Immigration. The Bureau of Immigration shall thereupon cancel the
pertinent alien certificate of registration and issue the certificate of identification as
Filipino citizen to the repatriated citizen.

Under Section 1 of Presidential Decree ("P.D.") No. 725, 8 dated 05 June 1975, amending
Commonwealth Act No. 63, an application for repatriation could be filed by Filipino women who lost their
Philippine citizenship by marriage to aliens, as well as by natural born Filipinos who lost their Philippine
citizenship, with the Special Committee on Naturalization. The committee, chaired by the Solicitor General
with the Undersecretary of Foreign Affairs and the Director of the National Intelligence Coordinating
Agency as the other members, was created pursuant to Letter of Instruction ("LOI") No. 270, dated 11
April 1975, as amended by LOI No. 283 and LOI No. 491 issued, respectively, on 04 June 1975 and on
29 December 1976. Although the agency was deactivated  by virtue of President Corazon C. Aquino's
Memorandum of 27 March 1987, it was not however, abrogated. In Frivaldo vs. Commission on
Elections, 9 the Court observed that the aforedated memorandum of President Aquino had merely
directed the Special Committee on Naturalization "to cease and desist from undertaking any and all
proceedings . . . under Letter of Instruction ("LOI") 270."  The Court elaborated:
10

This memorandum dated March 27, 1987 cannot by any stretch of legal
hermeneutics be construed as a law sanctioning or authorizing a repeal of P.D. No.
725. Laws are repealed only by subsequent ones and a repeal may be express or
implied. It is obvious that no express repeal was made because then President
Aquino in her memorandum-based on the copy furnished us by Lee-did not
categorically and/or impliedly state that P.D. 725 was being repealed or was being
rendered without any legal effect. In fact, she did not even mention it specifically by
its number or text. On the other hand, it is a basic rule of statutory construction
that repeals by implication are not favored. An implied repeal will not be allowed
"unless it is convincingly and unambiguously demonstrated that the two laws are
clear repugnant and patently inconsistent that they cannot co-exist."

The memorandum of then President Aquino cannot even be regarded as a legislative


enactment, for not every pronouncement of the Chief Executive even under the
Transitory Provisions of the 1987 Constitution can nor should be regarded as an
exercise of her law-making powers. At best, it could be treated as an executive policy
addressed to the Special Committee to halt the acceptance and processing of
applications for repatriation pending whatever "judgment the first Congress under the
1987 Constitution" might make. In other words, the former President did not-repeal
P.D. 725 but left it to the first Congress — once created — to deal with the matter. If
she had intended to repeal such law, she should have unequivocally said so instead
of referring the matter to Congress. The fact is she carefully couched her presidential
issuance in terms that clearly indicated the intention of "the present government, in
the exercise of prudence and sound discretion" to leave the matter of repeal to the
new Congress. Any other interpretation of the said Presidential Memorandum, such
as is now being proffered to the Court by Lee, would visit unmitigated violence not
only upon statutory construction but on common sense as well. 11

Indeed, the Committee was reactivated on 08 June 1995; 12 hence, when petitioner filed his
petition on 11 March 1996, the Special Committee on Naturalization constituted pursuant to LOI
No. 270 under P.D. No. 725 was in place. Administrative Order 285, 13 promulgated on 22 August
1996 relative to R.A. No. 8171, in effect, was merely then a confirmatory issuance.
The Office of the Solicitor General was right in maintaining that Angat's petition should have been
filed with the Committee, aforesaid, and not with the RTC which had no jurisdiction thereover. The
court's order of 04 October 1996 was thereby null and void, and it did not acquire finality 14 nor could
be a source of right on the part of petitioner. 15 It should also be noteworthy that the petition in Case No.
N-96-03-MK was one for repatriation, and it was thus incorrect for petitioner to initially invoke Republic Act
No. 965 16 and R.A. No. 2630  since these laws could only apply to persons who had lost their citizenship
17

by rendering service to, or accepting commission in, the armed forces of an allied foreign country or the
armed forces of the United States of America, a factual matter not alleged in the petition, Parenthetically,
under these statutes, the person desiring to re-acquire Philippine citizenship would not  even be required
to file a petition in court, and all that he had to do was to take an oath of allegiance to the Republic of the
Philippines and to register that fact with the civil registry in the place of his residence or where he had last
resided in the Philippines.

WHEREFORE, the petition for review is DENIED, and the Order, dated 22 September 1996, issued
by the court a quo, dismissing the petition of petitioner in Civil Case No. N-96-03-MK for want of
jurisdiction, is AFFIRMED. No costs.

SO ORDERED.

G.R. No. 163256             November 10, 2004

CICERON P. ALTAREJOS, petitioner, 
vs.
COMMISSION ON ELECTIONS, JOSE ALMIÑE and VERNON VERSOZA, respondents.

DECISION

AZCUNA, J.:

This is a petition for certiorari, with prayer for the issuance of a temporary restraining order and/or a
writ of prohibitory and mandatory injunction, to set aside the Resolution promulgated by the
Commission on Elections (COMELEC), First Division, on March 22, 2004 disqualifying petitioner
Ciceron P. Altarejos from running as mayor of San Jacinto, Masbate, and another resolution of the
COMELEC en banc promulgated on May 7, 2004 denying petitioner's motion for reconsideration.

The factual antecedents are as follows:

Petitioner Altarejos was a candidate for mayor in the Municipality of San Jacinto, Masbate in the May
10, 2004 national and local elections.

On January 15, 2004, private respondents Jose Almiñe Altiche and Vernon Versoza, registered
voters of San Jacinto, Masbate, filed with the COMELEC, a petition to disqualify and to deny due
course or cancel the certificate of candidacy of petitioner on the ground that he is not a Filipino
citizen and that he made a false representation in his certificate of candidacy that "[he] was not a
permanent resident of or immigrant to a foreign country."

Private respondents alleged that based on a letter from the Bureau of Immigration dated June 25,

2001, petitioner was a holder of a permanent U.S. resident visa, an Alien Certificate of Registration
No. E139507 issued on November 3, 1997, and an Immigration Certificate of Residence No. 320846
issued on November 3, 1997 by the Bureau of Immigration. 2

On January 26, 2004, petitioner filed an Answer stating, among others, that he did not commit false

representation in his application for candidacy as mayor because as early as December 17, 1997,
he was already issued a Certificate of Repatriation by the Special Committee on Naturalization, after
he filed a petition for repatriation pursuant to Republic Act No. 8171. Thus, petitioner claimed that his
Filipino citizenship was already restored, and he was qualified to run as mayor in the May 10, 2004
elections. Petitioner sought the dismissal of the petition.

On the date of the hearing, the parties were required to submit their Memoranda within three days.
Private respondents filed their Memorandum, while petitioner did not file one within the required
period. Petitioner, however, filed a Reply Memorandum subsequently.
4  5 

Atty. Zacarias C. Zaragoza, Jr., regional election director for Region V and hearing officer of this
case, recommended that petitioner Altarejos be disqualified from being a candidate for the position
of mayor of San Jacinto, Masbate in the May 10, 2004 national and local elections. He found, thus:

xxx

The provisions of law governing the qualifications and disqualifications of elective local
officials are found in Sections 39 and 40 of Republic Act No. 7160 otherwise known as the
Local Government Code of 1991, which provide as follows:

SEC. 39. Qualifications. – (a) An elective local official must be a citizen of the Philippines; a
registered voter in the barangay, municipality, city or province or, in the case of member of
the sangguniang panlalawigan, sangguniang panlungsod, or sangguniang bayan, the district
where he intends to be elected; a resident therein for at least one (1) year immediately
preceding the day of the election; and able to read and write Filipino or any other local
language or dialect.

xxx.

(c) Candidates for the position of mayor or vice-mayor of independent component cities,
component cities or municipalities must be at least twenty-one (21) years of age on election
day.

[SEC. 40. Disqualifications. – The following persons are disqualified from running for any
elective position:]

xxx.

(d) Those with dual citizenship.

xxx.
(f) Permanent residents in a foreign country or those who have acquired the right to reside
abroad and continue to avail of the same right after the effectivity of this Code; xxx

Under the terms of the above quoted statutory provisions, it is required that an elective local
official must be a citizen of the Philippines, and he must not have a dual citizenship; must not
be a permanent resident in a foreign country or must not have acquired the right to reside
abroad.

In the present case, it has been established by clear and convincing evidence that
respondent is a citizen of the United States of America. Such fact is proven by his Alien
Certificate of Registration (ACR) No. E139507 issued on 3 November 1997 and Immigration
Certificate of Residence (ICR) with No. 320846 issued on 3 November 1997 by the Alien
Registration Division, Bureau of Immigration and Deportation. This was further confirmed in a
letter dated 25 June 2001 of then Commissioner ANDREA D. DOMINGO of the Bureau of
Immigration and Deportation.

Although respondent had petitioned for his repatriation as a Filipino citizen under Republic
Act No. 8171 on 17 December 1997, this did not restore to respondent his Filipino
citizenship, because Section 2 of the aforecited Republic Act No. 8171 specifically provides
that "repatriation shall be effected by taking the necessary oath of allegiance to the Republic
of the Philippines and registration in the proper civil registry and in the Bureau of
Immigration."

It appears from the records of this case that respondent failed to prove that he has fully
complied with requirements of the above-quoted Section 2 of Republic Act 8171 to perfect
his repatriation and reacquire his Filipino citizenship. Respondent has not submitted any
document to prove that he has taken his oath of allegiance to the Republic of the Philippines
and that he has registered his fact of repatriation in the proper civil registry and in the Bureau
of Immigration. In fact, in a letter date 25 June 2001, Commissioner ANDREA DOMINGO
stated that RESPONDENT is still a holder of visa under Section 13 (g) of the Philippine
Immigration Act of 1940 as amended, with an indefinite authorized stay in the Philippines,
implying that respondent did not register his supposed Certificate of Repatriation with the
Bureau of Immigration otherwise his Alien Visa would have already been cancelled. The rule
is that in case of doubt concerning the grant of citizenship, such doubt should be resolved in
favor of the State and against the applicant (Cheng vs. Republic, L-16999, 22 June 1965).

xxx

Not having been able to prove that he has fully reacquired his Filipino citizenship after being
naturalized as a citizen of the United States, it is clear that respondent is not qualified to be
candidate for the position of Mayor of San Jacinto, Masbate, in the 10 May 2004 National
and Local Elections, pursuant to the aforequoted Sections 39 and 40 of the Local
Government Code of 1991.

As a further consequence of his not being a Filipino citizen, respondent has also committed
false representation in his certificate of candidacy by stating therein that he is a natural-born
Filipino citizen, when in fact, he has not yet even perfected the reacquisition of Filipino
citizenship. Such false representation constitutes a material misrepresentation as it relates to
his qualification as a candidate for public office, which could be a valid ground for the
cancellation of his certificate of candidacy under Section 78 of the Omnibus Election Code x
x x. 
6
In its Resolution promulgated on March 22, 2004, the COMELEC, First Division, adopted the
findings and recommendation of Director Zaragoza. The dispositive portion of said Resolution
stated, thus:

WHEREFORE, premises considered, respondent CICERON PEREZ ALTAREJOS is hereby


disqualified to run as Mayor of San Jacinto, Masbate. Accordingly, his certificate of
candidacy for the position of Municipal Mayor of San Jacinto, Masbate is denied due course
and cancelled and his name deleted from the certified list of candidates for the May 10, 2004
elections.7

On March 25, 2004, petitioner filed a motion for reconsideration and attached the following
documents to prove that he had completed all the requirements for repatriation which thus entitled
him to run for an elective office, viz:

(1) Oath of Allegiance dated December 17, 1997;

(2) Identification Certificate No. 116543 issued by the Bureau of Immigration on March 1, 2004;

(3) Certification from the City Civil Registration Office, Makati City, that the Certificate of Repatriation
and Oath of Allegiance of petitioner was received by said office and registered, with the
corresponding fee paid, on February 18, 2004;

(4) A letter dated December 17, 1997 from the Special Committee on Naturalization to the Bureau
on Immigration and Deportation that it was furnishing said office with the Oath of Allegiance and
Certificate of Repatriation of petitioner for the cancellation of petitioner's registration in said office as
an alien, and the issuance to him of the corresponding Identification Card as Filipino citizen;

(5) A letter dated December 17, 1997 from the Special Committee on Naturalization to the Local
Registrar of San Jacinto, Masbate that it was sending petitioner's Oath of Allegiance and Certificate
of Repatriation for registration in their records and for petitioner's reacquisition of his former
Philippine citizenship.

On May 7, 2004, the COMELEC en banc promulgated a resolution denying the motion for
reconsideration, the dispositive portion of which reads:

WHEREFORE, premises considered, the Commission (En Banc) RESOLVED as it hereby


RESOLVES to DENY the Motion for Reconsideration for UTTER LACK OF MERIT and
AFFIRMS the Resolution of the First Division. 8

The Comelec en banc held, thus:

The Comelec Rules of Procedure provides that insufficiency of evidence to justify the
decision is a ground for a motion for reconsideration (Rule 19, Section 1). The evidence
referred to in the above provision and to be considered in the Motion for Reconsideration are
those which were submitted during the hearing and attached to the respective Memoranda of
the parties which are already part of the records of the case. In this regard, the evidence of
the respondent were not able to overcome the evidence of the petitioners.

When the entire records of the case was forwarded to the Commission (First Division) the
respondent's only evidence was his Certificate of Repatriation dated 17 December 1977 and
marked as Annex 1 of his answer. This piece of evidence was not enough to controvert the
evidence of the petitioners which consist of the letter of the then Bureau of Immigration
Commissioner Andrea Domingo dated 25 June 2001 which stated that as of the even date
respondent is a holder of permanent resident visa (page 15 of the records) and the
certification of Josephine C. Camata dated 28 January 2004 certifying, that the name of the
respondent could not be found in the records of repatriation. (page 42 of the records) The
questioned resolution, is therefore, in order as the evidence submitted by the respondent
were insufficient to rebut the evidence of the petitioner.

Now, the respondent, in his Motion for Reconsideration, attempted to introduce to the record
new pieces of evidence, which introduction is not anymore allowed in a Motion for
Reconsideration. These are the following a) Annex "2" – Oath of Allegiance; b) Annex "3" –
Bureau of Immigration Identification Certificate; c) Annex "4" – Certification of the City Civil
Registrar of Makati City; d) Annex "5" – Letter addressed to the Local Civil Registrar of San
Jacinto, Masbate by Aurora P. Cortes of Special Committee on Naturalization; and e) Annex
"6" – Letter addressed to the Bureau of Immigration and Deportation by Aurora P. Cortes of
Special Committee on Naturalization.

Assuming that the new evidence of the respondent are admitted, with more reason should
we cancel his certificate of candidacy for his act of [misrepresenting] himself as a Filipino
citizen when at the time he filed his certificate of candidacy, he has not yet perfected the
process of repatriation. He failed to comply with the requirements under Section 2 of
[Republic Act No.] 8171 which provides that repatriation shall be effected by taking the
necessary oath of allegiance to the Republic of the Philippines and registration in the proper
civil registry and in the Bureau of Immigration.

The certification was issued by the same Ms. Josephine C. Camata, City Civil Registrar,
dated February 18, 2004. This time, she certifies that Ciceron Perez Altarejos was registered
under Registry No. 1, Page 19, Book No. 1, Series of 2004 and paid under OR nos.
88325/8833256 dated February 18, 2004. (page 65 of the records). Obviously, he was able
to register in the proper civil registry only on February 18, 2004.

The respondent was able to register with the Bureau of Immigration only on March 1, 2004
as evidenced by the Bureau of Immigration Identification Certificate attached to the Motion
as Annex "3."

This fact confirms the finding of the Commission (First Division) that at the time respondent
filed his certificate of candidacy he is yet to complete the requirement under section two (2)
of RA 8171.

As a consequence of not being a Filipino citizen, he has committed false representation in


his certificate of candidacy. Such false representation constitutes a material
misrepresentation as it relates to his qualification as a candidate. As such the certificate of
candidacy may be cancelled on such ground. (Ycain vs. Caneja, 18 Phil. 778) 9

On May 10, 2004, the election day itself, petitioner filed this petition praying that: (1) The petition be
given due course and a temporary restraining order and/or writ of preliminary injunction be issued ex
parte restraining the respondents and all persons acting on their behalf, from fully implementing the
questioned COMELEC Resolutions promulgated on March 22, 2004 and May 7, 2004; (2) a writ of
preliminary mandatory injunction be issued ordering the COMELEC and all persons acting on its
behalf to allow petitioner to run as Mayor of San Jacinto, Masbate in the May 10, 2004 elections, and
to count and canvass the votes cast in his favor and to proclaim him as the winning mayor of San
Jacinto, Masbate; and (3) after proper proceedings, judgment be rendered declaring null and void
and setting aside the COMELEC Resolutions promulgated on March 22, 2004 and May 7, 2004 and
other related Orders of the COMELEC or its representatives which have the effect of illegally
preventing petitioner from running as Mayor of San Jacinto, Masbate.

In its Comment, the Office of the Solicitor General stated that, based on the information relayed to it
10 

by the COMELEC, petitioner's name, as a mayoralty candidate in San Jacinto, Masbate, was
retained in the list of candidates voted upon by the electorate in the said municipality. Hence, the
cancellation of petitioner's certificate of candidacy was never implemented. The COMELEC also
informed the Office of the Solicitor General that petitioner's opponent, Dr. Emilio Aris V. Espinosa,
was already proclaimed duly elected Mayor of San Jacinto, Masbate.

The Office of the Solicitor General contends that said supervening event has rendered the instant
petition moot and academic, and it prayed for the dismissal of the petition.

In his Reply, petitioner opposed the dismissal of his petition. He claims that the COMELEC
11 

resolutions disqualifying him from running as a mayoralty candidate adversely affected his
candidacy, since his supporters were made to believe that his votes would not be counted.
Moreover, he stated that said COMELEC resolutions cast a doubt on his Philippine citizenship.

Petitioner points out that he took his Oath of Allegiance to the Republic of the Philippines on
December 17, 1997. In view thereof, he ran and was even elected as Mayor of San Jacinto,
Masbate during the 1998 elections. He argues that if there was delay in the registration of his
Certificate of Repatriation with the Bureau of Immigration and with the proper civil registry, the same
was brought about by the inaction on the part of said offices since the records of the Special
Committee on Naturalization show that his Certificate of Repatriation and Oath of Allegiance have
long been transmitted to said offices.

Petitioner also asserts that the subsequent registration of his Certificate of Repatriation with the
Bureau of Immigration and with the Civil Registry of Makati City prior to the May 10, 2004 elections
has the effect of curing the defect, if any, in the reacquisition of his Filipino citizenship as his
repatriation retroacted to the date of his application for repatriation as held in Frivaldo v. Comelec.

The pertinent issues raised are the following: (1) Is the registration of petitioner's repatriation with the
proper civil registry and with the Bureau of Immigration a prerequisite in effecting repatriation; and
(2) whether or not the COMELEC en banc committed grave abuse of discretion amounting to excess
or lack of jurisdiction in affirming the Resolution of the COMELEC, First Division.

As stated by the Office of the Solicitor General, where the issues have become moot and academic,
there is no justiciable controversy, thereby rendering the resolution of the same of no practical use or
value. Nonetheless, courts will decide a question otherwise moot and academic if it is capable of
12 

repetition, yet evading review. 13

First Issue: Is the registration of petitioner's repatriation


with the proper civil registry and with the Bureau of
Immigration a prerequisite in effecting repatriation?

The provision of law applicable in this case is Section 2 of Republic Act No. 8171, thus: 14 

SEC. 2. Repatriation shall be effected by taking the necessary oath of allegiance to the
Republic of the Philippines and registration in the proper civil registry and in the Bureau of
Immigration. The Bureau of Immigration shall thereupon cancel the pertinent alien certificate
of registration and issue the certificate of identification as Filipino citizen to the repatriated
citizen.

The law is clear that repatriation is effected "by taking the oath of allegiance to the Republic of the
Philippines and registration in the proper civil registry and in the Bureau of Immigration." Hence, in
addition to taking the Oath of Allegiance to the Republic of the Philippines, the registration of the
Certificate of Repatriation in the proper civil registry and the Bureau of Immigration is a prerequisite
in effecting the repatriation of a citizen.

In this case, petitioner took his Oath of Allegiance on December 17, 1997, but his Certificate of
Repatriation was registered with the Civil Registry of Makati City only after six years or on February
18, 2004, and with the Bureau of Immigration on March 1, 2004. Petitioner, therefore, completed all
the requirements of repatriation only after he filed his certificate of candidacy for a mayoralty
position, but before the elections.

When does the citizenship qualification of a candidate for an elective office apply?

In Frivaldo v. Commission on Elections, the Court ruled that the citizenship qualification must be
15 

construed as "applying to the time of proclamation of the elected official and at the start of his term."
The Court, through Justice Artemio V. Panganiban, discussed, thus:

Under Sec. 39 of the Local Government Code, "(a)n elective local official must be:

* a citizen of the Philippines;

* a registered voter in the barangay, municipality, city, or province x x x where he intends to


be elected;

* a resident therein for at least one (1) year immediately preceding the day of the election;

* able to read and write Filipino or any other local language or dialect."

* In addition, "candidates for the position of governor x x x must be at least twenty-three (23)
years of age on election day."

From the above, it will be noted that the law does not specify any particular date or time
when the candidate must possess citizenship, unlike that for residence (which must consist
of at least one year's residency immediately preceding the day of election) and age (at least
twenty three years of age on election day).

Philippine citizenship is an indispensable requirement for holding an elective public office,


and the purpose of the citizenship qualification is none other than to ensure that no alien,
i.e., no person owing allegiance to another nation, shall govern our people and our country
or a unit of territory thereof. Now, an official begins to govern or to discharge his functions
only upon his proclamation and on the day the law mandates his term of office to begin.
Since Frivaldo re-assumed his citizenship on June 30, 1995—the very day the term of office
of governor (and other elective officials) began—he was therefore already qualified to be
proclaimed, to hold such office and to discharge the functions and responsibilities thereof as
of said date. In short, at that time, he was already qualified to govern his native Sorsogon.
This is the liberal interpretation that should give spirit, life and meaning to our law on
qualifications consistent with the purpose for which such law was enacted. x x x
Paraphrasing this Court's ruling in Vasquez v. Giap and Li Seng Giap & Sons, if the purpose
of the citizenship requirement is to ensure that our people and country do not end up being
governed by aliens, i.e., persons owing allegiance to another nation, that aim or purpose
would not be thwarted but instead achieved by construing the citizenship qualification as
applying to the time of proclamation of the elected official and at the start of his
term. (Emphasis supplied.)
16 

Moreover, in the case of Frivaldo v. Commission on Elections, the Court ruled that "the
repatriation of Frivaldo RETROACTED to the date of the filing of his application." In said
case, the repatriation of Frivaldo was by virtue of Presidential Decree No. 725, which took
effect on June 5, 1975. The Court therein declared that Presidential Decree No. 725 was a
curative statute, which is retroactive in nature. The retroactivity of Frivaldo's repatriation to
the date of filing of his application was justified by the Court, thus:

xxx

…The reason for this is simply that if, as in this case, it was the intent of the legislative
authority that the law should apply to past events—i.e., situations and transactions existing
even before the law came into being—in order to benefit the greatest number of former
Filipinos possible thereby enabling them to enjoy and exercise the constitutionally
guaranteed right of citizenship, and such legislative intention is to be given the fullest effect
and expression, then there is all the more reason to have the law apply in a retroactive or
retrospective manner to situations, events and transactions subsequent to the passage of
such law. That is, the repatriation granted to Frivaldo x x x can and should be made to take
effect as of date of his application. As earlier mentioned, there is nothing in the law that
would bar this or would show a contrary intention on the part of the legislative authority; and
there is no showing that damage or prejudice to anyone, or anything unjust or injurious
would result from giving retroactivity to his repatriation. Neither has Lee shown that there will
result the impairment of any contractual obligation, disturbance of any vested right or breach
of some constitutional guaranty.

xxx

Another argument for retroactivity to the date of filing is that it would prevent prejudice to
applicants. If P.D. 725 were not to be given retroactive effect, and the Special Committee
decides not to act, i.e., to delay the processing of applications for any substantial length of
time, then the former Filipinos who may be stateless, as Frivaldo—having already renounced
his American citizenship—was, may be prejudiced for causes outside their control. This
should not be. In case of doubt in the interpretation or application of laws, it is to be
presumed that the law-making body intended right and justice to prevail. 17

Republic Act No. 8171 has impliedly repealed Presidential `Decree No. 725. They cover the same
18 

subject matter: Providing for the repatriation of Filipino women who have lost their Philippine
citizenship by marriage to aliens and of natural-born Filipinos. The Court's ruling in Frivaldo v.
Commission on Elections that repatriation retroacts to the date of filing of one's application for
repatriation subsists for the same reasons quoted above.

Accordingly, petitioner's repatriation retroacted to the date he filed his application in 1997. Petitioner
was, therefore, qualified to run for a mayoralty position in the government in the May 10, 2004
elections. Apparently, the COMELEC was cognizant of this fact since it did not implement the
assailed Resolutions disqualifying petitioner to run as mayor of San Jacinto, Masbate.
Second Issue: Whether or not the COMELEC en banc
gravely abused its discretion in affirming the
Resolution of the COMELEC, First Division?

The Court cannot fault the COMELEC en banc for affirming the decision of the COMELEC, First
Division, considering that petitioner failed to prove before the COMELEC that he had complied with
the requirements of repatriation. Petitioner submitted the necessary documents proving compliance
with the requirements of repatriation only during his motion for reconsideration, when the COMELEC
en banc could no longer consider said evidence. As the COMELEC en banc correctly stated:

The Comelec Rules of Procedure provides that insufficiency of evidence to justify the
decision is a ground for a motion for reconsideration (Rule 19, Section 1). The evidence
referred to in the above provision and to be considered in the Motion for Reconsideration are
those which were submitted during the hearing and attached to the respective Memoranda of
the parties which are already part of the records of the case. In this regard, the evidence of
the respondent were not able to overcome the evidence of the petitioners. 19

It is, therefore, incumbent upon candidates for an elective office, who are repatriated citizens, to be
ready with sufficient evidence of their repatriation in case their Filipino citizenship is questioned to
prevent a repetition of this case.

WHEREFORE, the petition seeking the nullification of the Resolution of the COMELEC en banc of
May 7, 2004, affirming the Resolution of its First Division dated March 22, 2004, is hereby DENIED.
No costs.

SO ORDERED.

G.R. No. 120295 June 28, 1996

JUAN G. FRIVALDO, petitioner, 
vs.
COMMISSION ON ELECTIONS, and RAUL R. LEE, respondents.

G.R. No. 123755 June 28, 1996

RAUL R. LEE, petitioner, 
vs.
COMMISSION ON ELECTIONS and JUAN G. FRIVALDO, respondents.

PANGANIBAN, J.:p

The ultimate question posed before this Court in these twin cases is: Who should be declared the rightful governor of Sorsogon -

(i) Juan G. Frivaldo, who unquestionably obtained the highest number of votes in three successive
elections but who was twice declared by this Court to be disqualified to hold such office due to his
alien citizenship, and who now claims to have re-assumed his lost Philippine citizenship thru
repatriation;
(ii) Raul R. Lee, who was the second placer in the canvass, but who claims that the votes cast in
favor of Frivaldo should be considered void; that the electorate should be deemed to have
intentionally thrown away their ballots; and that legally, he secured the most number of valid votes;
or

(iii) The incumbent Vice-Governor, Oscar G. Deri, who obviously was not voted directly to the
position of governor, but who according to prevailing jurisprudence should take over the said post
inasmuch as, by the ineligibility of Frivaldo, a "permanent vacancy in the contested office has
occurred"?

In ruling for Frivaldo, the Court lays down new doctrines on repatriation, clarifies/reiterates/amplifies
existing jurisprudence on citizenship and elections, and upholds the superiority of substantial justice
over pure legalisms.

G.R. No. 123755

This is a special civil action under Rules 65 and 58 of the Rules of Court for certiorari and
preliminary injunction to review and annul a Resolution of the respondent Commission on Elections
(Comelec), First Division,  promulgated on December 19, 1995  and another Resolution of the
1 2

Comelec en banc promulgated February 23, 1996  denying petitioner's motion for reconsideration.
3

The Facts

On March 20, 1995, private respondent Juan G. Frivaldo filed his Certificate of Candidacy for the
office of Governor of Sorsogon in the May 8, 1995 elections. On March 23, 1995, petitioner Raul R.
Lee, another candidate, filed a petition  with the Comelec docketed as SPA No. 95-028 praying that
4

Frivaldo "be disqualified from seeking or holding any public office or position by reason of not yet
being a citizen of the Philippines", and that his Certificate of Candidacy be canceled. On May 1,
1995, the Second Division of the Comelec promulgated a Resolution  granting the petition with the
5

following disposition :
6

WHEREFORE, this Division resolves to GRANT the petition and declares that
respondent is DISQUALIFIED to run for the Office of Governor of Sorsogon on the
ground that he is NOT a citizen of the Philippines. Accordingly, respondent's
certificate of candidacy is canceled.

The Motion for Reconsideration filed by Frivaldo remained unacted upon until after the May 8, 1995
elections. So, his candidacy continued and he was voted for during the elections held on said date.
On May 11, 1995, the Comelec en banc  affirmed the aforementioned Resolution of the Second
7

Division.

The Provincial Board of Canvassers completed the canvass of the election returns and a Certificate
of Votes  dated May 27, 1995 was issued showing the following votes obtained by the candidates for
8

the position of Governor of Sorsogon:

Antonio H. Escudero, Jr. 51,060

Juan G. Frivaldo 73,440

Raul R. Lee 53,304


Isagani P. Ocampo 1,925

On June 9, 1995, Lee filed in said SPA No. 95-028, a (supplemental) petition  praying for his
9

proclamation as the duly-elected Governor of Sorsogon.

In an order  dated June 21, 1995, but promulgated according to the petition "only on June 29, 1995,"
10

the Comelec en banc directed "the Provincial Board of Canvassers of Sorsogon to reconvene for the
purpose of proclaiming candidate Raul Lee as the winning gubernatorial candidate in the province of
Sorsogon on June 29, 1995 . . ." Accordingly, at 8:30 in the evening of June 30, 1995, Lee was
proclaimed governor of Sorsogon.

On July 6, 1995, Frivaldo filed with the Comelec a new petition,  docketed as SPC No. 95-317,
11

praying for the annulment of the June 30, 1995 proclamation of Lee and for his own proclamation.
He alleged that on June 30, 1995, at 2:00 in the afternoon, he took his oath of allegiance as a citizen
of the Philippines after "his petition for repatriation under P.D. 725 which he filed with the Special
Committee on Naturalization in September 1994 had been granted". As such, when "the said order
(dated June 21, 1995) (of the Comelec) . . . was released and received by Frivaldo on June 30, 1995
at 5:30 o'clock in the evening, there was no more legal impediment to the proclamation (of Frivaldo)
as governor . . ." In the alternative, he averred that pursuant to the two cases of Labo
vs. Comelec,  the Vice-Governor - not Lee - should occupy said position of governor.
12

On December 19, 1995, the Comelec First Division promulgated the herein assailed
Resolution  holding that Lee, "not having garnered the highest number of votes," was not legally
13

entitled to be proclaimed as duly-elected governor; and that Frivaldo, "having garnered the highest
number of votes,
and . . . having reacquired his Filipino citizenship by repatriation on June 30, 1995 under the
provisions of Presidential Decree No. 725 . . . (is) qualified to hold the office of governor of
Sorsogon"; thus:

PREMISES CONSIDERED, the Commission (First Division), therefore RESOLVES


to GRANT the Petition.

Consistent with the decisions of the Supreme Court, the proclamation of Raul R. Lee
as Governor of Sorsogon is hereby ordered annulled, being contrary to law, he not
having garnered the highest number of votes to warrant his proclamation.

Upon the finality of the annulment of the proclamation of Raul R. Lee, the Provincial
Board of Canvassers is directed to immediately reconvene and, on the basis of the
completed canvass, proclaim petitioner Juan G. Frivaldo as the duly elected
Governor of Sorsogon having garnered the highest number of votes, and he having
reacquired his Filipino citizenship by repatriation on June 30, 1995 under the
provisions of Presidential Decree No. 725 and, thus, qualified to hold the office of
Governor of Sorsogon.

Conformably with Section 260 of the Omnibus Election Code (B.P. Blg. 881), the
Clerk of the Commission is directed to notify His Excellency the President of the
Philippines, and the Secretary of the Sangguniang Panlalawigan of the Province of
Sorsogon of this resolution immediately upon the due implementation thereof.

On December 26, 1995, Lee filed a motion for reconsideration which was denied by the Comelec en
banc in its Resolution   promulgated on February 23, 1996. On February 26, 1996, the present
14

petition was filed. Acting on the prayer for a temporary restraining order, this Court issued on
February 27, 1996 a Resolution which inter alia directed the parties "to maintain the status
quo prevailing prior to the filing of this petition."

The Issues in G.R. No. 123755

Petitioner Lee's "position on the matter at hand may briefly be capsulized in the following
propositions" :
15

First -- The initiatory petition below was so far insufficient in form and substance to
warrant the exercise by the COMELEC of its jurisdiction with the result that, in effect,
the COMELEC acted without jurisdiction in taking cognizance of and deciding said
petition;

Second -- The judicially declared disqualification of respondent was a continuing


condition and rendered him ineligible to run for, to be elected to and to hold the
Office of Governor;

Third -- The alleged repatriation of respondent was neither valid nor is the effect
thereof retroactive as to cure his ineligibility and qualify him to hold the Office of
Governor; and

Fourth -- Correctly read and applied, the Labo Doctrine fully supports the validity of
petitioner's proclamation as duly elected Governor of Sorsogon.

G.R. No. 120295

This is a petition to annul three Resolutions of the respondent Comelec, the first two of which are
also at issue in G.R. No. 123755, as follows:

1. Resolution  of the Second Division, promulgated on May 1, 1995, disqualifying


16

Frivaldo from running for governor of Sorsogon in the May 8, 1995 elections "on the
ground that he is not a citizen of the Philippines";

2. Resolution 7 of the Comelec en banc, promulgated on May 11, 1995; and


1

3. Resolution  of the Comelec en banc, promulgated also on May 11, 1995
18

suspending the proclamation of, among others, Frivaldo.

The Facts and the Issue

The facts of this case are essentially the same as those in G.R. No. 123755. However, Frivaldo
assails the above-mentioned resolutions on a different ground: that under Section 78 of the Omnibus
Election Code, which is reproduced hereinunder:

Sec. 78. Petition to deny due course or to cancel a certificate of candidacy. -- A


verified petition seeking to deny due course or to cancel a certificate of candidacy
may be filed by any person exclusively on the ground that any material
representation contained therein as required under Section 74 hereof is false. The
petition may be filed at any time not later than twenty-five days from the time of the
filing of the certificate of candidacy and shall be decided, after notice and
hearing, not later than fifteen days before the election. (Emphasis supplied.)
the Comelec had no jurisdiction to issue said Resolutions because they were not rendered
"within the period allowed by law" i.e., "not later than fifteen days before the election."

Otherwise stated, Frivaldo contends that the failure of the Comelec to act on the petition for
disqualification within the period of fifteen days prior to the election as provided by law is a
jurisdictional defect which renders the said Resolutions null and void.

By Resolution on March 12, 1996, the Court consolidated G.R. Nos. 120295 and 123755 since they
are intimately related in their factual environment and are identical in the ultimate question
raised, viz., who should occupy the position of governor of the province of Sorsogon.

On March 19, 1995, the Court heard oral argument from the parties and required them thereafter to
file simultaneously their respective memoranda.

The Consolidated Issues

From the foregoing submissions, the consolidated issues may be restated as follows:

1. Was the repatriation of Frivaldo valid and legal? If so, did it seasonably cure his lack of citizenship
as to qualify him to be proclaimed and to hold the Office of Governor? If not, may it be given
retroactive effect? If so, from when?

2. Is Frivaldo's "judicially declared" disqualification for lack of Filipino citizenship a continuing bar to
his eligibility to run for, be elected to or hold the governorship of Sorsogon?

3. Did the respondent Comelec have jurisdiction over the initiatory petition in SPC No. 95-317
considering that said petition is not "a pre-proclamation case, an election protest or a quo
warranto case"?

4. Was the proclamation of Lee, a runner-up in the election, valid and legal in light of existing
jurisprudence?

5. Did the respondent Commission on Elections exceed its jurisdiction in promulgating the assailed
Resolutions, all of which prevented Frivaldo from assuming the governorship of Sorsogon,
considering that they were not rendered within the period referred to in Section 78 of the Omnibus
Election Code, viz., "not later than fifteen days before the elections"?

The First Issue: Frivaldo's Repatriation

The validity and effectivity of Frivaldo's repatriation is the lis mota, the threshold legal issue in this
case. All the other matters raised are secondary to this.

The Local Government Code of 1991  expressly requires Philippine citizenship as a qualification for
19

elective local officials, including that of provincial governor, thus:

Sec. 39. Qualifications. -- (a) An elective local official must be a citizen of the
Philippines; a registered voter in the barangay, municipality, city, or province or, in
the case of a member of the sangguniang panlalawigan, sangguniang panlungsod,
or sangguniang bayan, the district where he intends to be elected; a resident therein
for at least one (1) year immediately preceding the day of the election; and able to
read and write Filipino or any other local language or dialect.
(b) Candidates for the position of governor, vice governor or member
of the sangguniang panlalawigan, or mayor, vice mayor or member of
the sangguniang panlungsod of highly urbanized cities must be at
least twenty-three (23) years of age on election day.

x x x           x x x          x x x

Inasmuch as Frivaldo had been declared by this Court  as a non-citizen, it is therefore incumbent
20

upon him to show that he has reacquired citizenship; in fine, that he possesses the qualifications
prescribed under the said statute (R.A. 7160).

Under Philippine law,  citizenship may be reacquired by direct act of Congress, by naturalization or
21

by repatriation. Frivaldo told this Court in G.R. No. 104654  and during the oral argument in this
22

case that he tried to resume his citizenship by direct act of Congress, but that the bill allowing him to
do so "failed to materialize, notwithstanding the endorsement of several members of the House of
Representatives" due, according to him, to the "maneuvers of his political rivals." In the same case,
his attempt at naturalization was rejected by this Court because of jurisdictional, substantial and
procedural defects.

Despite his lack of Philippine citizenship, Frivaldo was overwhelmingly elected governor by the
electorate of Sorsogon, with a margin of 27,000 votes in the 1988 elections, 57,000 in 1992, and
20,000 in 1995 over the same opponent Raul Lee. Twice, he was judicially declared a non-Filipino
and thus twice disqualified from holding and discharging his popular mandate. Now, he comes to us
a third time, with a fresh vote from the people of Sorsogon and a favorable decision from the
Commission on Elections to boot. Moreover, he now boasts of having successfully passed through
the third and last mode of reacquiring citizenship: by repatriation under P.D. No. 725, with no less
than the Solicitor General himself, who was the prime opposing counsel in the previous cases he
lost, this time, as counsel for co-respondent Comelec, arguing the validity of his cause (in addition to
his able private counsel Sixto S. Brillantes, Jr.). That he took his oath of allegiance under the
provisions of said Decree at 2:00 p.m. on June 30, 1995 is not disputed. Hence, he insists that he --
not Lee -- should have been proclaimed as the duly-elected governor of Sorsogon when the
Provincial Board of Canvassers met at 8:30 p.m. on the said date since, clearly and unquestionably,
he garnered the highest number of votes in the elections and since at that time, he already
reacquired his citizenship.

En contrario, Lee argues that Frivaldo's repatriation is tainted with serious defects, which we shall
now discuss in seriatim.

First, Lee tells us that P.D. No. 725 had "been effectively repealed", asserting that "then President
Corazon Aquino exercising legislative powers under the Transitory Provisions of the 1987
Constitution, forbade the grant of citizenship by Presidential Decree or Executive Issuances as the
same poses a serious and contentious issue of policy which the present government, in the exercise
of prudence and sound discretion, should best leave to the judgment of the first Congress under the
1987 Constitution", adding that in her memorandum dated March 27, 1987 to the members of the
Special Committee on Naturalization constituted for purposes of Presidential Decree No. 725,
President Aquino directed them "to cease and desist from undertaking any and all proceedings
within your functional area of responsibility as defined under Letter of Instructions (LOI) No. 270
dated April 11, 1975, as amended." 23

This memorandum dated March 27, 1987  cannot by any stretch of legal hermeneutics be construed
24

as a law sanctioning or authorizing a repeal of P.D. No. 725. Laws are repealed only by subsequent
ones   and a repeal may be express or implied. It is obvious that no express repeal was made
25
because then President Aquino in her memorandum -- based on the copy furnished us by Lee -- did
not categorically and/or impliedly state that P.D. 725 was being repealed or was being rendered
without any legal effect. In fact, she did not even mention it specifically by its number or text. On the
other hand, it is a basic rule of statutory construction that repeals by implication are not favored. An
implied repeal will not be allowed "unless it is convincingly and unambiguously demonstrated that
the two laws are clearly repugnant and patently inconsistent that they cannot co-exist". 26

The memorandum of then President Aquino cannot even be regarded as a legislative enactment, for
not every pronouncement of the Chief Executive even under the Transitory Provisions of the 1987
Constitution can nor should be regarded as an exercise of her law-making powers. At best, it could
be treated as an executive policy addressed to the Special Committee to halt the acceptance and
processing of applications for repatriation pending whatever "judgment the first Congress under the
1987 Constitution" might make. In other words, the former President did not repeal P.D. 725 but left
it to the first Congress -- once created -- to deal with the matter. If she had intended to repeal such
law, she should have unequivocally said so instead of referring the matter to Congress. The fact is
she carefully couched her presidential issuance in terms that clearly indicated the intention of "the
present government, in the exercise of prudence and sound discretion" to leave the matter of repeal
to the new Congress. Any other interpretation of the said Presidential Memorandum, such as is now
being proffered to the Court by Lee, would visit unmitigated violence not only upon statutory
construction but on common sense as well.

Second, Lee also argues that "serious congenital irregularities flawed the repatriation proceedings,"
asserting that Frivaldo's application therefor was "filed on June 29, 1995 . . . (and) was approved in
just one day or on June 30, 1995 . . .", which "prevented a judicious review and evaluation of the
merits thereof." Frivaldo counters that he filed his application for repatriation with the Office of the
President in Malacañang Palace on August 17, 1994. This is confirmed by the Solicitor General.
However, the Special Committee was reactivated only on June 8, 1995, when presumably the said
Committee started processing his application. On June 29, 1995, he filled up and re-submitted the
FORM that the Committee required. Under these circumstances, it could not be said that there was
"indecent haste" in the processing of his application.

Anent Lee's charge that the "sudden reconstitution of the Special Committee on Naturalization was
intended solely for the personal interest of respondent," 7 the Solicitor General explained during the
2

oral argument on March 19, 1996 that such allegation is simply baseless as there were many others
who applied and were considered for repatriation, a list of whom was submitted by him to this Court,
through a Manifestation  filed on April 3, 1996.
28

On the basis of the parties' submissions, we are convinced that the presumption of regularity in the
performance of official duty and the presumption of legality in the repatriation of Frivaldo have not
been successfully rebutted by Lee. The mere fact that the proceedings were speeded up is by itself
not a ground to conclude that such proceedings were necessarily tainted. After all, the requirements
of repatriation under P.D. No. 725 are not difficult to comply with, nor are they tedious and
cumbersome. In fact, P.D.
725  itself requires very little of an applicant, and even the rules and regulations to implement the
29

said decree were left to the Special Committee to promulgate. This is not unusual since, unlike in
naturalization where an alien covets a first-time entry into Philippine political life, in repatriation the
applicant is a former natural-born Filipino who is merely seeking to reacquire his previous
citizenship. In the case of Frivaldo, he was undoubtedly a natural-born citizen who openly and
faithfully served his country and his province prior to his naturalization in the United States -- a
naturalization he insists was made necessary only to escape the iron clutches of a dictatorship he
abhorred and could not in conscience embrace -- and who, after the fall of the dictator and the re-
establishment of democratic space, wasted no time in returning to his country of birth to offer once
more his talent and services to his people.

So too, the fact that ten other persons, as certified to by the Solicitor General, were granted
repatriation argues convincingly and conclusively against the existence of favoritism vehemently
posited by Raul Lee. At any rate, any contest on the legality of Frivaldo's repatriation should have
been pursued before the Committee itself, and, failing there, in the Office of the President, pursuant
to the doctrine of exhaustion of administrative remedies.

Third, Lee further contends that assuming the assailed repatriation to be valid, nevertheless it could
only be effective as at 2:00 p.m. of June 30, 1995 whereas the citizenship qualification prescribed by
the Local Government Code "must exist on the date of his election, if not when the certificate of
candidacy is filed," citing our decision in G.R. 104654  which held that "both the Local Government
30

Code and the Constitution require that only Philippine citizens can run and be elected to public
office." Obviously, however, this was a mere obiter as the only issue in said case was whether
Frivaldo's naturalization was valid or not -- and NOT the effective date thereof. Since the Court held
his naturalization to be invalid, then the issue of when an aspirant for public office should be a citizen
was NOT resolved at all by the Court. Which question we shall now directly rule on.

Under Sec. 39 of the Local Government Code, "(a)n elective local official must be:

* a citizen of the Philippines;

* a registered voter in the barangay, municipality, city, or province . . . where he


intends to be elected;

* a resident therein for at least one (1) year immediately preceding the day of the
election;

* able to read and write Filipino or any other local language or dialect.

* In addition, "candidates for the position of governor . . . must be at least twenty-


three (23) years of age on election day.

From the above, it will be noted that the law does not specify any particular date or time when the
candidate must possess citizenship, unlike that for residence (which must consist of at least one
year's residency immediately preceding the day of election) and age (at least twenty three years of
age on election day).

Philippine citizenship is an indispensable requirement for holding an elective public office,  and the
31

purpose of the citizenship qualification is none other than to ensure that no alien, i.e., no person
owing allegiance to another nation, shall govern our people and our country or a unit of territory
thereof. Now, an official begins to govern or to discharge his functions only upon his
proclamation and on the day the law mandates his term of office to begin. Since Frivaldo re-
assumed his citizenship on June 30, 1995 -- the very day  the term of office of governor (and other
32

elective officials) began -- he was therefore already qualified to be proclaimed, to hold such office
and to discharge the functions and responsibilities thereof as of said date. In short, at that time, he
was already qualified to govern his native Sorsogon. This is the liberal interpretation that should give
spirit, life and meaning to our law on qualifications consistent with the purpose for which such law
was enacted. So too, even from a literal (as distinguished from liberal) construction, it should be
noted that Section 39 of the Local Government Code speaks of "Qualifications" of "ELECTIVE
OFFICIALS", not of candidates. Why then should such qualification be required at the time of
election or at the time of the filing of the certificates of candidacies, as Lee insists? Literally, such
qualifications -- unless otherwise expressly conditioned, as in the case of age and residence --
should thus be possessed when the "elective [or elected] official" begins to govern, i.e., at the time
he is proclaimed and at the start of his term -- in this case, on June 30, 1995. Paraphrasing this
Court's ruling in Vasquez vs. Giap and Li Seng Giap & Sons,   if the purpose of the citizenship
33

requirement is to ensure that our people and country do not end up being governed by aliens, i.e.,
persons owing allegiance to another nation, that aim or purpose would not be thwarted but instead
achieved by construing the citizenship qualification as applying to the time of proclamation of the
elected official and at the start of his term.

But perhaps the more difficult objection was the one raised during the oral argument  to the effect
34

that the citizenship qualification should be possessed at the time the candidate (or for that matter the
elected official) registered as a voter. After all, Section 39, apart from requiring the official to be a
citizen, also specifies as another item of qualification, that he be a "registered voter". And, under the
law  a "voter" must be a citizen of the Philippines. So therefore, Frivaldo could not have been a voter
35

-- much less a validly registered one -- if he was not a citizen at the time of such registration.

The answer to this problem again lies in discerning the purpose of the requirement. If the law
intended the citizenship qualification to be possessed prior to election consistent with the
requirement of being a registered voter, then it would not have made citizenship a SEPARATE
qualification. The law abhors a redundancy. It therefore stands to reason that the law intended
CITIZENSHIP to be a qualification distinct from being a VOTER, even if being a voter presumes
being a citizen first. It also stands to reason that the voter requirement was included as another
qualification (aside from "citizenship"), not to reiterate the need for nationality but to require that the
official be registered as a voter IN THE AREA OR TERRITORY he seeks to govern, i.e., the law
states: "a registered voter in the barangay, municipality, city, or province . . . where he intends to be
elected." It should be emphasized that the Local Government Code requires an elective official to be
a registered voter. It does not require him to vote actually. Hence, registration -- not the actual voting
-- is the core of this "qualification". In other words, the law's purpose in this second requirement is to
ensure that the prospective official is actually registered in the area he seeks to govern -- and not
anywhere else.

Before this Court, Frivaldo has repeatedly emphasized -- and Lee has not disputed -- that he "was
and is a registered voter of Sorsogon, and his registration as a voter has been sustained as valid by
judicial declaration . . . In fact, he cast his vote in his precinct on May 8, 1995."
36

So too, during the oral argument, his counsel steadfastly maintained that "Mr. Frivaldo has always
been a registered voter of Sorsogon. He has voted in 1987, 1988, 1992, then he voted again in
1995. In fact, his eligibility as a voter was questioned, but the court dismissed (sic) his eligibility as a
voter and he was allowed to vote as in fact, he voted in all the previous elections including on May 8,
1995."  7
3

It is thus clear that Frivaldo is a registered voter in the province where he intended to be elected.

There is yet another reason why the prime issue of citizenship should be reckoned from the date of
proclamation, not necessarily the date of election or date of filing of the certificate of candidacy.
Section 253 of the Omnibus Election Code   gives any voter, presumably including the defeated
38

candidate, the opportunity to question the ELIGIBILITY (or the disloyalty) of a candidate. This is the
only provision of the Code that authorizes a remedy on how to contest before the Comelec an
incumbent's ineligibility arising from failure to meet the qualifications enumerated under Sec. 39 of
the Local Government Code. Such remedy of Quo Warranto can be availed of "within ten days after
proclamation" of the winning candidate. Hence, it is only at such time that the issue of ineligibility
may be taken cognizance of by the Commission. And since, at the very moment of Lee's
proclamation (8:30 p.m., June 30, 1995), Juan G. Frivaldo was already and indubitably a citizen,
having taken his oath of allegiance earlier in the afternoon of the same day, then he should have
been the candidate proclaimed as he unquestionably garnered the highest number of votes in the
immediately preceding elections and such oath had already cured his previous "judicially-declared"
alienage. Hence, at such time, he was no longer ineligible.

But to remove all doubts on this important issue, we also hold that the repatriation of Frivaldo
RETROACTED to the date of the filing of his application on August 17, 1994.

It is true that under the Civil Code of the Philippines,   "(l)aws shall have no retroactive effect, unless
39

the contrary is provided." But there are settled exceptions  to this general rule, such as when the
40

statute is CURATIVE or REMEDIAL in nature or when it CREATES NEW RIGHTS.

According to Tolentino,  curative statutes are those which undertake to cure errors and irregularities,
41

thereby validating judicial or administrative proceedings, acts of public officers, or private deeds and
contracts which otherwise would not produce their intended consequences by reason of some
statutory disability or failure to comply with some technical requirement. They operate on conditions
already existing, and are necessarily retroactive in operation. Agpalo,  on the other hand, says that
42

curative statutes are


"healing acts . . . curing defects and adding to the means of enforcing existing obligations . . . (and)
are intended to supply defects, abridge superfluities in existing laws, and curb certain evils. . . . By
their very nature, curative statutes are retroactive . . . (and) reach back to past events to correct
errors or irregularities and to render valid and effective attempted acts which would be otherwise
ineffective for the purpose the parties intended."

On the other hand, remedial or procedural laws, i.e., those statutes relating to remedies or modes of
procedure, which do not create new or take away vested rights, but only operate in furtherance of
the remedy or confirmation of such rights, ordinarily do not come within the legal meaning of a
retrospective law, nor within the general rule against the retrospective operation of statutes. 43

A reading of P.D. 725 immediately shows that it creates a new right, and also provides for a new
remedy, thereby filling certain voids in our laws. Thus, in its preamble, P.D. 725 expressly
recognizes the plight of "many Filipino women (who) had lost their Philippine citizenship by marriage
to aliens" and who could not, under the existing law (C.A. No. 63, as amended) avail of repatriation
until "after the death of their husbands or the termination of their marital status" and who could
neither be benefitted by the 1973 Constitution's new provision allowing "a Filipino woman who
marries an alien to retain her Philippine citizenship . . ." because "such provision of the new
Constitution does not apply to Filipino women who had married aliens before said constitution took
effect." Thus, P.D. 725 granted a new right to these women -- the right to re-acquire Filipino
citizenship even during their marital coverture, which right did not exist prior to P.D. 725. On the
other hand, said statute also provided a new remedy and a new right in favor of other "natural born
Filipinos who (had) lost their Philippine citizenship but now desire to re-acquire Philippine
citizenship", because prior to the promulgation of P.D. 725 such former Filipinos would have had to
undergo the tedious and cumbersome process of naturalization, but with the advent of P.D. 725 they
could now re-acquire their Philippine citizenship under the simplified procedure of repatriation.

The Solicitor General  argues:


44

By their very nature, curative statutes are retroactive, (DBP vs. CA, 96 SCRA 342),
since they are intended to supply defects, abridge superfluities in existing laws (Del
Castillo vs. Securities and Exchange Commission, 96 Phil. 119) and curb certain
evils (Santos vs. Duata, 14 SCRA 1041).

In this case, P.D. No. 725 was enacted to cure the defect in the existing
naturalization law, specifically C.A. No. 63 wherein married Filipino women are
allowed to repatriate only upon the death of their husbands, and natural-born
Filipinos who lost their citizenship by naturalization and other causes faced the
difficulty of undergoing the rigid procedures of C.A. 63 for reacquisition of Filipino
citizenship by naturalization.

Presidential Decree No. 725 provided a remedy for the aforementioned legal
aberrations and thus its provisions are considered essentially remedial and curative.

In light of the foregoing, and prescinding from the wording of the preamble, it is unarguable that the
legislative intent was precisely to give the statute retroactive operation. "(A) retrospective operation
is given to a statute or amendment where the intent that it should so operate clearly appears from a
consideration of the act as a whole, or from the terms thereof."  It is obvious to the Court that the
45

statute was meant to "reach back" to those persons, events and transactions not otherwise covered
by prevailing law and jurisprudence. And inasmuch as it has been held that citizenship is a political
and civil right equally as important as the freedom of speech, liberty of abode, the right against
unreasonable searches and seizures and other guarantees enshrined in the Bill of Rights, therefore
the legislative intent to give retrospective operation to P.D. 725 must be given the fullest effect
possible. "(I)t has been said that a remedial statute must be so construed as to make it effect the
evident purpose for which it was enacted, so that if the reason of the statute extends to past
transactions, as well as to those in the future, then it will be so applied although the statute does not
in terms so direct, unless to do so would impair some vested right or violate some constitutional
guaranty."  This is all the more true of P.D. 725, which did not specify any restrictions on or delimit
46

or qualify the right of repatriation granted therein.

At this point, a valid question may be raised: How can the retroactivity of P.D. 725 benefit Frivaldo
considering that said law was enacted on June 5, 1975, while Frivaldo lost his Filipino citizenship
much later, on January 20, 1983, and applied for repatriation even later, on August 17, 1994?

While it is true that the law was already in effect at the time that Frivaldo became an American
citizen, nevertheless, it is not only the law itself (P.D. 725) which is to be given retroactive effect, but
even the repatriation granted under said law to Frivaldo on June 30, 1995 is to be deemed to have
retroacted to the date of his application therefor, August 17, 1994. The reason for this is simply that
if, as in this case, it was the intent of the legislative authority that the law should apply to past events
-- i.e., situations and transactions existing even before the law came into being -- in order to benefit
the greatest number of former Filipinos possible thereby enabling them to enjoy and exercise the
constitutionally guaranteed right of citizenship, and such legislative intention is to be given the fullest
effect and expression, then there is all the more reason to have the law apply in a retroactive or
retrospective manner to situations, events and transactions subsequent to the passage of such law.
That is, the repatriation granted to Frivaldo on June 30, 1995 can and should be made to take effect
as of date of his application. As earlier mentioned, there is nothing in the law that would bar this or
would show a contrary intention on the part of the legislative authority; and there is no showing that
damage or prejudice to anyone, or anything unjust or injurious would result from giving retroactivity
to his repatriation. Neither has Lee shown that there will result the impairment of any contractual
obligation, disturbance of any vested right or breach of some constitutional guaranty.
Being a former Filipino who has served the people repeatedly, Frivaldo deserves a liberal
interpretation of Philippine laws and whatever defects there were in his nationality should now be
deemed mooted by his repatriation.

Another argument for retroactivity to the date of filing is that it would prevent prejudice to applicants.
If P.D. 725 were not to be given retroactive effect, and the Special Committee decides not to act, i.e.,
to delay the processing of applications for any substantial length of time, then the former Filipinos
who may be stateless, as Frivaldo -- having already renounced his American citizenship -- was, may
be prejudiced for causes outside their control. This should not be. In case of doubt in the
interpretation or application of laws, it is to be presumed that the law-making body intended right and
justice to prevail.  7
4

And as experience will show, the Special Committee was able to process, act upon and grant
applications for repatriation within relatively short spans of time after the same were filed.  The fact
48

that such interregna were relatively insignificant minimizes the likelihood of prejudice to the
government as a result of giving retroactivity to repatriation. Besides, to the mind of the Court, direct
prejudice to the government is possible only where a person's repatriation has the effect of wiping
out a liability of his to the government arising in connection with or as a result of his being an alien,
and accruing only during the interregnum between application and approval, a situation that is not
present in the instant case.

And it is but right and just that the mandate of the people, already twice frustrated, should now
prevail. Under the circumstances, there is nothing unjust or iniquitous in treating Frivaldo's
repatriation as having become effective as of the date of his application, i.e., on August 17, 1994.
This being so, all questions about his possession of the nationality qualification -- whether at the
date of proclamation (June 30, 1995) or the date of election (May 8, 1995) or date of filing his
certificate of candidacy (March 20, 1995) would become moot.

Based on the foregoing, any question regarding Frivaldo's status as a registered voter would also be
deemed settled. Inasmuch as he is considered as having been repatriated -- i.e., his Filipino
citizenship restored -- as of August 17, 1994, his previous registration as a voter is likewise deemed
validated as of said date.

It is not disputed that on January 20, 1983 Frivaldo became an American. Would the retroactivity of
his repatriation not effectively give him dual citizenship, which under Sec. 40 of the Local
Government Code would disqualify him "from running for any elective local position?"  We answer
49

this question in the negative, as there is cogent reason to hold that Frivaldo was really STATELESS
at the time he took said oath of allegiance and even before that, when he ran for governor in 1988.
In his Comment, Frivaldo wrote that he "had long renounced and had long abandoned his American
citizenship -- long before May 8, 1995. At best, Frivaldo was stateless in the interim -- when he
abandoned and renounced his US citizenship but before he was repatriated to his Filipino
citizenship."
50

On this point, we quote from the assailed Resolution dated December 19, 1995: 51

By the laws of the United States, petitioner Frivaldo lost his American citizenship
when he took his oath of allegiance to the Philippine Government when he ran for
Governor in 1988, in 1992, and in 1995. Every certificate of candidacy contains an
oath of allegiance to the Philippine Government."

These factual findings that Frivaldo has lost his foreign nationality long before the elections of 1995
have not been effectively rebutted by Lee. Furthermore, it is basic that such findings of the
Commission are conclusive upon this Court, absent any showing of capriciousness or arbitrariness
or
abuse.52

The Second Issue: Is Lack of Citizenship


a Continuing Disqualification?

Lee contends that the May 1, 1995 Resolution   of the Comelec Second Division in SPA No. 95-028
53

as affirmed in toto by Comelec En Banc in its Resolution of May 11, 1995 "became final and
executory after five (5) days or on May 17, 1995, no restraining order having been issued by this
Honorable Court.  Hence, before Lee "was proclaimed as the elected governor on June 30, 1995,
54

there was already a final and executory judgment disqualifying" Frivaldo. Lee adds that this Court's
two rulings (which Frivaldo now concedes were legally "correct") declaring Frivaldo an alien have
also become final and executory way before the 1995 elections, and these "judicial pronouncements
of his political status as an American citizen absolutely and for all time disqualified (him) from
running for, and holding any public office in the Philippines."

We do not agree.

It should be noted that our first ruling in G.R. No. 87193 disqualifying Frivaldo was rendered in
connection with the 1988 elections while that in G.R. No. 104654 was in connection with the 1992
elections. That he was disqualified for such elections is final and can no longer be changed. In the
words of the respondent Commission (Second Division) in its assailed Resolution: 55

The records show that the Honorable Supreme Court had decided that Frivaldo was
not a Filipino citizen and thus disqualified for the purpose of the 1988 and 1992
elections. However, there is no record of any "final judgment" of the disqualification
of Frivaldo as a candidate for the May 8, 1995 elections. What the Commission said
in its Order of June 21, 1995 (implemented on June 30, 1995), directing the
proclamation of Raul R. Lee, was that Frivaldo was not a Filipino citizen "having
been declared by the Supreme Court in its Order dated March 25, 1995, not a citizen
of the Philippines." This declaration of the Supreme Court, however, was in
connection with the 1992 elections.

Indeed, decisions declaring the acquisition or denial of citizenship cannot govern a person's future
status with finality. This is because a person may subsequently reacquire, or for that matter lose, his
citizenship under any of the modes recognized by law for the purpose. Hence, in Lee
vs. Commissioner of Immigration,  we held:
56

Everytime the citizenship of a person is material or indispensable in a judicial or


administrative case, whatever the corresponding court or administrative authority
decides therein as to such citizenship is generally not considered res judicata, hence
it has to be threshed out again and again, as the occasion demands.

The Third Issue: Comelec's Jurisdiction


Over The Petition in SPC No. 95-317

Lee also avers that respondent Comelec had no jurisdiction to entertain the petition in SPC No. 95-
317 because the only "possible types of proceedings that may be entertained by the Comelec are a
pre-proclamation case, an election protest or a quo warranto case". Again, Lee reminds us that he
was proclaimed on June 30, 1995 but that Frivaldo filed SPC No. 95-317 questioning his (Lee's)
proclamation only on July 6, 1995 -- "beyond the 5-day reglementary period." Hence, according to
him, Frivaldo's "recourse was to file either an election protest or a quo warranto action."

This argument is not meritorious. The Constitution 7 has given the Comelec ample power to
5

"exercise exclusive original jurisdiction over all contests relating to the elections, returns and
qualifications of all elective . . . provincial . . . officials." Instead of dwelling at length on the various
petitions that Comelec, in the exercise of its constitutional prerogatives, may entertain, suffice it to
say that this Court has invariably recognized the Commission's authority to hear and decide petitions
for annulment of proclamations -- of which SPC No. 95-317 obviously is one.  Thus, in Mentang
58

vs. COMELEC,  we ruled:


59

The petitioner argues that after proclamation and assumption of office, a pre-
proclamation controversy is no longer viable. Indeed, we are aware of cases holding
that pre-proclamation controversies may no longer be entertained by the COMELEC
after the winning candidate has been proclaimed. (citing Gallardo vs. Rimando, 187
SCRA 463; Salvacion vs. COMELEC, 170 SCRA 513; Casimiro vs. COMELEC, 171
SCRA 468.) This rule, however, is premised on an assumption that the proclamation
is no proclamation at all and the proclaimed candidate's assumption of office cannot
deprive the COMELEC of the power to make such declaration of nullity.
(citing Aguam vs. COMELEC, 23 SCRA 883; Agbayani vs. COMELEC, 186 SCRA
484.)

The Court however cautioned that such power to annul a proclamation must "be done within ten (10)
days following the proclamation." Inasmuch as Frivaldo's petition was filed only six (6) days after
Lee's proclamation, there is no question that the Comelec correctly acquired jurisdiction over the
same.

The Fourth Issue: Was Lee's Proclamation Valid?

Frivaldo assails the validity of the Lee proclamation. We uphold him for the following reasons:

First. To paraphrase this Court in Labo vs. COMELEC,  "the fact remains that he (Lee) was not the
60

choice of the sovereign will," and in Aquino vs. COMELEC,  Lee is "a second placer, . . . just that, a
61

second placer."

In spite of this, Lee anchors his claim to the governorship on the pronouncement of this Court in the
aforesaid Labo  case, as follows:
62

The rule would have been different if the electorate fully aware in fact and in law of a
candidate's disqualification so as to bring such awareness within the realm of
notoriety, would nonetheless cast their votes in favor of the ineligible candidate. In
such case, the electorate may be said to have waived the validity and efficacy of their
votes by notoriously misapplying their franchise or throwing away their votes, in
which case, the eligible candidate obtaining the next higher number of votes may be
deemed elected.

But such holding is qualified by the next paragraph, thus:

But this is not the situation obtaining in the instant dispute. It has not been shown,
and none was alleged, that petitioner Labo was notoriously known as an ineligible
candidate, much less the electorate as having known of such fact. On the contrary,
petitioner Labo was even allowed by no less than the Comelec itself in its resolution
dated May 10, 1992 to be voted for the office of the city Payor as its resolution dated
May 9, 1992 denying due course to petitioner Labo's certificate of candidacy had not
yet become final and subject to the final outcome of this case.

The last-quoted paragraph in Labo, unfortunately for Lee, is the ruling appropriate in this case
because Frivaldo was in 1995 in an identical situation as Labo was in 1992 when the Comelec's
cancellation of his certificate of candidacy was not yet final on election day as there was in both
cases a pending motion for reconsideration, for which reason Comelec issued an (omnibus)
resolution declaring that Frivaldo (like Labo in 1992) and several others can still be voted for in the
May 8, 1995 election, as in fact, he was.

Furthermore, there has been no sufficient evidence presented to show that the electorate of
Sorsogon was "fully aware in fact and in law" of Frivaldo's alleged disqualification as to "bring such
awareness within the realm of notoriety;" in other words, that the voters intentionally wasted their
ballots knowing that, in spite of their voting for him, he was ineligible. If Labo has any relevance at
all, it is that the vice-governor -- and not Lee -- should be pro- claimed, since in losing the election,
Lee was, to paraphrase Labo again, "obviously not the choice of the people" of Sorsogon. This is the
emphatic teaching of Labo:

The rule, therefore, is: the ineligibility of a candidate receiving majority votes does not
entitle the eligible candidate receiving the next highest number of votes to be
declared elected. A minority or defeated candidate cannot be deemed elected to the
office.

Second. As we have earlier declared Frivaldo to have seasonably reacquired his citizenship and
inasmuch as he obtained the highest number of votes in the 1995 elections, he -- not Lee -- should
be proclaimed. Hence, Lee's proclamation was patently erroneous and should now be corrected.

The Fifth Issue: Is Section 78 of the


Election Code Mandatory?

In G.R. No. 120295, Frivaldo claims that the assailed Resolution of the Comelec (Second Division)
dated May 1, 1995 and the confirmatory en banc Resolution of May 11, 1995 disqualifying him for
want of citizenship should be annulled because they were rendered beyond the fifteen (15) day
period prescribed by Section 78, of the Omnibus Election Code which reads as follows:

Sec. 78. Petition to deny due course or to cancel a certificate of candidacy. -- A


verified petition seeking to deny due course or to cancel a certificate of candidacy
may be filed by any person exclusively on the ground that any material
representation contained therein as required under Section 74 hereof is false. The
petition may be filed at any time not later than twenty-five days from the time of the
filing of the certificate of candidacy and shall be decided after notice and hearing, not
later than fifteen days before the election. (Emphasis supplied.)

This claim is now moot and academic inasmuch as these resolutions are deemed superseded by the
subsequent ones issued by the Commission (First Division) on December 19, 1995, affirmed en
banc  on February 23, 1996; which both upheld his election. At any rate, it is obvious that Section 78
63

is merely directory as Section 6 of R.A. No. 6646 authorizes the Commission to try and decide
petitions for disqualifications even after the elections, thus:
Sec. 6. Effect of Disqualification Case. -- Any candidate who has been declared by
final judgment to be disqualified shall not be voted for, and the votes cast for him
shall not be counted. If for any reason a candidate is not declared by final judgment
before an election to be disqualified and he is voted for and receives the winning
number of votes in such election, the Court or Commission shall continue with the
trial and hearing of the action, inquiry or protest and upon motion of the complainant
or any intervenor, may during the pendency thereof order the suspension of the
proclamation of such candidate whenever the evidence of his guilt is strong.
(emphasis supplied)

Refutation of
Mr. Justice Davide's Dissent

In his dissenting opinion, the esteemed Mr. Justice Hilario G. Davide, Jr. argues that President
Aquino's memorandum dated March 27, 1987 should be viewed as a suspension (not a repeal, as
urged by Lee) of P.D. 725. But whether it decrees a suspension or a repeal is a purely academic
distinction because the said issuance is not a statute that can amend or abrogate an existing law.
The existence and subsistence of P.D. 725 were recognized in the first Frivaldo case;  viz., "(u)nder
64

CA No. 63 as amended by CA No. 473 and P.D. No. 725, Philippine citizenship maybe reacquired


by . . . repatriation". He also contends that by allowing Frivaldo to register and to remain as a
registered voter, the Comelec and in effect this Court abetted a "mockery" of our two previous
judgments declaring him a non-citizen. We do not see such abetting or mockery. The retroactivity of
his repatriation, as discussed earlier, legally cured whatever defects there may have been in his
registration as a voter for the purpose of the 1995 elections. Such retroactivity did not change his
disqualifications in 1988 and 1992, which were the subjects of such previous rulings.

Mr. Justice Davide also believes that Quo Warranto is not the sole remedy to question the ineligibility
of a candidate, citing the Comelec's authority under Section 78 of the Omnibus Election Code
allowing the denial of a certificate of candidacy on the ground of a false material representation
therein as required by Section 74. Citing Loong, he then states his disagreement with our holding
that Section 78 is merely directory. We really have no quarrel. Our point is that Frivaldo was in error
in his claim in G.R. No. 120295 that the Comelec Resolutions promulgated on May 1, 1995 and May
11, 1995 were invalid because they were issued "not later than fifteen days before the election" as
prescribed by Section 78. In dismissing the petition in G.R. No. 120295, we hold that the Comelec
did not commit grave abuse of discretion because "Section 6 of R.A. 6646 authorizes the Comelec to
try and decide disqualifications even after the elections." In spite of his disagreement with us on this
point, i.e., that Section 78 "is merely directory", we note that just like us, Mr. Justice Davide
nonetheless votes to "DISMISS G.R. No. 120295". One other point. Loong, as quoted in the dissent,
teaches that a petition to deny due course under Section 78 must be filed within the 25-day period
prescribed therein. The present case however deals with the period during which the Comelec
may decide such petition. And we hold that it may be decided even after the fifteen day period
mentioned in Section 78. Here, we rule that a decision promulgated by the Comelec even after the
elections is valid but Loong held that a petition filed beyond the 25-day period is out of time. There is
no inconsistency nor conflict.

Mr. Justice Davide also disagrees with the Court's holding that, given the unique factual
circumstances of Frivaldo, repatriation may be given retroactive effect. He argues that such
retroactivity "dilutes" our holding in the first Frivaldo case. But the first (and even the second
Frivaldo) decision did not directly involve repatriation as a mode of acquiring citizenship. If we may
repeat, there is no question that Frivaldo was not a Filipino for purposes of determining his
qualifications in the 1988 and 1992 elections. That is settled. But his supervening repatriation has
changed his political status -- not in 1988 or 1992, but only in the 1995 elections.
Our learned colleague also disputes our holding that Frivaldo was stateless prior to his repatriation,
saying that "informal renunciation or abandonment is not a ground to lose American citizenship".
Since our courts are charged only with the duty of determining who are Philippine nationals, we
cannot rule on the legal question of who are or who are not Americans. It is basic in international law
that a State determines ONLY those who are its own citizens -- not who are the citizens of other
countries.  The issue here is: the Comelec made a finding of fact that Frivaldo was stateless and
65

such finding has not been shown by Lee to be arbitrary or whimsical. Thus, following settled case
law, such finding is binding and final.

The dissenting opinion also submits that Lee who lost by chasmic margins to Frivaldo in all three
previous elections, should be declared winner because "Frivaldo's ineligibility for being an American
was publicly known". First, there is absolutely no empirical evidence for such "public" knowledge.
Second, even if there is, such knowledge can be truepost facto only of the last two previous
elections. Third, even the Comelec and now this Court were/are still deliberating on his nationality
before, during and after the 1995 elections. How then can there be such "public" knowledge?

Mr. Justice Davide submits that Section 39 of the Local Government Code refers to the qualifications
of electivelocal officials, i.e., candidates, and not elected officials, and that the citizenship
qualification [under par. (a) of that section] must be possessed by candidates, not merely at the
commencement of the term, but by election day at the latest. We see it differently. Section 39, par.
(a) thereof speaks of "elective local official" while par. (b) to (f) refer to "candidates". If the
qualifications under par. (a) were intended to apply to "candidates" and not elected officials, the
legislature would have said so, instead of differentiating par. (a) from the rest of the paragraphs.
Secondly, if Congress had meant that the citizenship qualification should be possessed at election
day or prior thereto, it would have specifically stated such detail, the same way it did in pars. (b) to (f)
far other qualifications of candidates for governor, mayor, etc.

Mr. Justice Davide also questions the giving of retroactive effect to Frivaldo's repatriation on the
ground, among others, that the law specifically provides that it is only after taking the oath of
allegiance that applicants shall be deemed to have reacquired Philippine citizenship. We do not
question what the provision states. We hold however that the provision should be understood
thus: that after taking the oath of allegiance the applicant is deemed to have reacquired Philippine
citizenship, which reacquisition (or repatriation) is deemed for all purposes and intents to have
retroacted to the date of his application therefor.

In any event, our "so too" argument regarding the literal meaning of the word "elective" in reference
to Section 39 of the Local Authority Code, as well as regarding Mr. Justice Davide's thesis that the
very wordings of P.D. 725 suggest non-retroactivity, were already taken up rather extensively earlier
in this Decision.

Mr. Justice Davide caps his paper with a clarion call: "This Court must be the first to uphold the Rule
of Law." We agree -- we must all follow the rule of law. But that is NOT the issue here. The issue
is how should the law be interpreted and applied in this case so it can be followed, so it can rule!

At balance, the question really boils down to a choice of philosophy and perception of how to
interpret and apply laws relating to elections: literal or liberal; the letter or the spirit, the naked
provision or its ultimate purpose; legal syllogism or substantial justice; in isolation or in the context of
social conditions; harshly against or gently in favor of the voters' obvious choice. In applying election
laws, it would be far better to err in favor of popular sovereignty than to be right in complex but little
understood legalisms. Indeed, to inflict a thrice rejected candidate upon the electorate of Sorsogon
would constitute unmitigated judicial tyranny and an unacceptable assault upon this Court's
conscience.
EPILOGUE

In sum, we rule that the citizenship requirement in the Local Government Code is to be possessed
by an elective official at the latest as of the time he is proclaimed and at the start of the term of office
to which he has been elected. We further hold P.D. No. 725 to be in full force and effect up to the
present, not having been suspended or repealed expressly nor impliedly at any time, and Frivaldo's
repatriation by virtue thereof to have been properly granted and thus valid and effective. Moreover,
by reason of the remedial or curative nature of the law granting him a new right to resume his
political status and the legislative intent behind it, as well as his unique situation of having been
forced to give up his citizenship and political aspiration as his means of escaping a regime he
abhorred, his repatriation is to be given retroactive effect as of the date of his application therefor,
during the pendency of which he was stateless, he having given up his U.S. nationality. Thus, in
contemplation of law, he possessed the vital requirement of Filipino citizenship as of the start of the
term of office of governor, and should have been proclaimed instead of Lee. Furthermore, since his
reacquisition of citizenship retroacted to August 17, 1994, his registration as a voter of Sorsogon is
deemed to have been validated as of said date as well. The foregoing, of course, are precisely
consistent with our holding that lack of the citizenship requirement is not a continuing disability or
disqualification to run for and hold public office. And once again, we emphasize herein our previous
rulings recognizing the Comelec's authority and jurisdiction to hear and decide petitions for
annulment of proclamations.

This Court has time and again liberally and equitably construed the electoral laws of our country to
give fullest effect to the manifest will of our people,  for in case of doubt, political laws must be
66

interpreted to give life and spirit to the popular mandate freely expressed through the ballot.
Otherwise stated, legal niceties and technicalities cannot stand in the way of the sovereign will.
Consistently, we have held:

. . . (L)aws governing election contests must be liberally construed to the end that the
will of the people in the choice of public officials may not be defeated by mere
technical objections (citations omitted). 7
6

The law and the courts must accord Frivaldo every possible protection, defense and refuge, in
deference to the popular will. Indeed, this Court has repeatedly stressed the importance of giving
effect to the sovereign will in order to ensure the survival of our democracy. In any action involving
the possibility of a reversal of the popular electoral choice, this Court must exert utmost effort to
resolve the issues in a manner that would give effect to the will of the majority, for it is merely sound
public policy to cause elective offices to be filled by those who are the choice of the majority. To
successfully challenge a winning candidate's qualifications, the petitioner must clearly demonstrate
that the ineligibility is so patently antagonistic  to constitutional and legal principles that overriding
68

such ineligibility and thereby giving effect to the apparent will of the people, would ultimately create
greater prejudice to the very democratic institutions and juristic traditions that our Constitution and
laws so zealously protect and promote. In this undertaking, Lee has miserably failed.

In Frivaldo's case. it would have been technically easy to find fault with his cause. The Court could
have refused to grant retroactivity to the effects of his repatriation and hold him still ineligible due to
his failure to show his citizenship at the time he registered as a voter before the 1995 elections. Or, it
could have disputed the factual findings of the Comelec that he was stateless at the time of
repatriation and thus hold his consequent dual citizenship as a disqualification "from running for any
elective local position." But the real essence of justice does not emanate from quibblings over
patchwork legal technicality. It proceeds from the spirit's gut consciousness of the dynamic role of
law as a brick in the ultimate development of the social edifice. Thus, the Court struggled against
and eschewed the easy, legalistic, technical and sometimes harsh anachronisms of the law in order
to evoke substantial justice in the larger social context consistent with Frivaldo's unique situation
approximating venerability in Philippine political life. Concededly, he sought American citizenship
only to escape the clutches of the dictatorship. At this stage, we cannot seriously entertain any doubt
about his loyalty and dedication to this country. At the first opportunity, he returned to this land, and
sought to serve his people once more. The people of Sorsogon overwhelmingly voted for him three
times. He took an oath of allegiance to this Republic every time he filed his certificate of candidacy
and during his failed naturalization bid. And let it not be overlooked, his demonstrated tenacity and
sheer determination to re-assume his nationality of birth despite several legal set-backs speak more
loudly, in spirit, in fact and in truth than any legal technicality, of his consuming intention and burning
desire to re-embrace his native Philippines even now at the ripe old age of 81 years. Such loyalty to
and love of country as well as nobility of purpose cannot be lost on this Court of justice and equity.
Mortals of lesser mettle would have given up. After all, Frivaldo was assured of a life of ease and
plenty as a citizen of the most powerful country in the world. But he opted, nay, single-mindedly
insisted on returning to and serving once more his struggling but beloved land of birth. He therefore
deserves every liberal interpretation of the law which can be applied in his favor. And in the final
analysis, over and above Frivaldo himself, the indomitable people of Sorsogon most certainly
deserve to be governed by a leader of their overwhelming choice.

WHEREFORE, in consideration of the foregoing:

(1) The petition in G.R. No. 123755 is hereby DISMISSED. The assailed Resolutions of the
respondent Commission are AFFIRMED.

(2) The petition in G.R. No. 120295 is also DISMISSED for being moot and academic. In any event,
it has no merit.

No costs.

SO ORDERED.

G.R. No. L-29397 March 29, 1983

MODESTA DUGCOY JAO (SOMETIMES SPELLED HAO), petitioner, 


vs.
REPUBLIC OF THE PHILIPPINES, oppositor.

Julius A Artes for petitioner-appellee.

The Solicitor General for oppositor-appellant.

RESOLUTION

VASQUEZ, J.:

The petitioner filed a petition in the Court of First Instance of Davao for repatriation under
Commonwealth Act No. 63, as amended, alleging therein that although her father was Chinese, she
was a citizen of the Philippines because her mother was a Filipina who was not legally married to
her Chinese husband; that she lost her Philippine citizenship when she married Go Wan, a Chinese,
with whom she had three children; and that Go Wan died in September 6, 1962. Petitioner further
alleged that her illiterate mother erroneously registered her as an alien with the Bureau of
Immigration, by virtue of which she was issued Alien Certificate of Registration No. A-176678. The
petition contains no prayer for relief.

The petition was not published, but notice thereof was served on the Provincial Fiscal who appeared
at the hearing but presented no evidence. After receiving the evidence of the petitioner, the trial
court issued an Order declaring the petitioner as "judicially repatriated," and ordering the
cancellation of her alien certificate of registration. The Provincial Fiscal, in behalf of the Republic of
the Philippines, took this appeal.

The proceedings taken in the trial court are a complete nullity. There is no law requiring or
authorizing that repatriation should be effected by a judicial proceeding. All that is required for a
female citizen of the Philippines who lost her citizenship to an alien to reacquire her Philippine
citizen, upon the termination of her marital status, "is for her to take necessary oath of allegiance to
the Republic of the Philippines and to register the said oath in the proper civil registry" (Lim vs.
Republic, 37 SCRA 783). Moreover, the petitioner's claim of Philippine citizenship prior to her
marriage for being allegedly an illegitimate child of a Chinese father and a Filipino mother may not
be established in an action where the mother or her heirs are not patties (Tan Pong vs. Republic, 30
Phil. 380). It is the consistent rule in this jurisdiction that Philippine citizenship may not be declared in
a non- adversary suit where the persons whose rights are affected by such a declaration are not
parties, such as an action for declaratory relief (Tiu Navarro vs. Commissioner of Immigration, 107
Phil. 632) a petition for judicial repatriation (Lim vs. Republic, supra), or an action to cancel
registration as an alien (Tan vs. Republic, L-16108, Oct. 31, 1961). As was held in Lim vs. Republic,
supra, "there is no proceeding established by law or the rules by which any person claiming to be a
citizen may get a declaration in a court of justice to that effect or in regard to his citizenship."

ACCORDINGLY, the decision appealed from is hereby revoked and set aside.

SO ORDERED.

Das könnte Ihnen auch gefallen